You are on page 1of 1931

Pathoanatomy - Acute Inflammation, Vascular And Cellular Change

Total questions: 34; Correctly answered: 11; Percentage of


correct: 32.4% Skipped question: 0;
correct answer
incorrect answer
skipped question

 1. All of the following vascular changes are observed in acute


inflammation, except:

Vasodilation



Stasis of blood



Increased vascular permeability



Decreased hydrostatic pressure

 2. Free radicals are generated by all except



Superoxide dismutase



NADPH Oxidase



Myeloperoxidase



NO synthase

 3. A 5-year-old female Sukanya is hospitalized with fever and


hemorrhagic skin lesions on her lower extremities. Aout five
months ao she was successully treated with penicillin for bacterial
meningitis. She likely to be suffering from which of the following

Pure T-cell dysfunction


Ineffective intracellular killing



Insufficient IgA production



Inability to form the membrane-attack complex

 4. Diapedesis is

Immigration of leukocytes through the basement membrane



Immigration of the leukocytes through the vessel wall to the site of
inflammation



Aggregation of platelets at the site of bleeding



Auto digestion of the cells

 5. All of the following are signs of inflammation except



Pain



Swelling



Redness



Absence of functional loss


 6. Which among the following is the hallmark of acute
inflammation?

Vasoconstriction



Stasis



Vasodilation and increase in permeability



Leukocyte margination

 7. Which of the following statements in context of the enzyme



It is a major mode of defense mechanism in eosinophils



Its deficiency results in Chediak Higashi syndrome



It causes formation of a more important bactericidal agent than
defensins and lysozyme



It is required for attracting the white blood cells near a targeted
organism

 8. The complex process of leukocyte movements through the


blood vessels are all except

Rolling



Adhesion



Migration


Phagocytosis

 9. Characteristic of acute inflammation is:



Vasodilation and increased vascular permeability



Vasoconstriction



Platelet aggregation



Infiltration by neutrophils

 10. In acute inflammation due to the contraction of endothelial cell


cytoskeleton, which of the following results?

Delayed transient increase in permeability



Early transient increase



Delayed permanent increase



Early permanent increase

 11. Main feature of chemotaxis is



Increased random movement of neutrophils



Increase adhesiveness to intima



Increased phagocytosis



Unidirectional locomotion of the neutrophils

 12. Delayed prolonged bleeding is caused by:



Histamine



Endothelial retraction



IL-1



Direct injury to endothelial cells

 13. Earliest transient change following tissue injury will be:



Neutropenia



Neutrophilia



Monocytosis



Lymphocytosis

 14. Basement membrane degeneration is mediated by:



Metalloproteinases



Oxidases


Elastases



Hydroxylases

 15. A 14 month old boy Chunnu is being evaluated for recurrent,


indolent skin infections and gingivitis. On taking a detailed history
from the mother, she tells very valuable point that he had delayed
separation of the umbilical cord which occurred around 9-

Late complement components



Transcobalamin II



Integrins



a2-globulins

 16. A 3-year-old boy, Krish presents with recurrent bacterial and


fungal infections primarily involving his skin and respiratory tract.
Physical examination reveals the presence of oculocutan-eous
albinism. Examination of a peripheral blood smear reveals larg

Ataxia-telangiectasia



Chediak-Higashi syndrome



Chronic granulomatous disease



Ehlers-Danlos syndrome


 17. The function common to neutrophils, monocytes, and
macrophages is

Immune response is reduced



Phagocytosis



Liberation of histamine



Destruction of old erythrocytes

 18. Which of the following helps in generating reactive O2


intermediates in the neutrophils?

NADPH oxidase



SOD (superoxide dismutase)



Catalase



Glutathione peroxidase

 19. Endothelium leukocyte interaction during inflammation is


mediated by/due to

Selectins



Integrins



Defensins



Endothelin

 20. Which of the following is not true?



NADPH oxidase generate superoxide ion



MPO kills by OCl



Chediak-Higashi syndrome is due to defective phagolysosome
formation



In Bruton

 21. Which of the following statements in context of endothelial cell


contraction in inflammation is false

Endothelial cell contraction is the commonest mechanism of increased
permeability



Endothelial cell contraction is responsible for immediate transient
response



It affects venules, capillaries and arterioles commonly.



It is associated with the release of histamine, substance P and
bradykinin

 22. Most important for diapedesis?



PECAM



Selectin



Integrin



Mucin like glycoprotein

 23. Nitroblue tetrazolium test is used for?



Phagocytes



Complement



T cell



B cell

 24. All of the following are a family of selectin except



P selectin



L selectin



A selectin



E selectin

 25. Oxygen dependent killing is done through



NADPH oxidase


Superoxide dismutase



Catalase



Glutathione peroxidase

 26. In acute inflammation the tissue response consists of all


except

Vasodilatation



Exudation



Neutrophilic response



Granuloma formation

 27. In genetic deficiency of MPO the increased suscep tibility to


infection is due to:

Defective production of prostaglandins



Defective rolling of neutrophils



Inability to produce hydroxyl-halide radicals



Inability to produce hydrogen peroxide


 28. All are true about exudate except

More protein



Less protein



More specific gravity



All

 29. In acute inflammation endothelial retraction leads to



Delayed transient increase in permeability



Immediate transient increase in permeability



Delayed prolonged increase in permeability



Immediate transient decrease in permeability

 30. Endogenous chemoattractant is:



C5a



Bacterial products



Lipopolysaccharide A



C8


 31. The following host tissue responses can be seen in acute
infection, except:

Exudation



Vasodilation



Margination



Granuloma formation

 32. After binding of complement and antibody on the surface of


encapsulated bacteria, the process of phagocytosis by
polymorphonuclear leukocytes involves which of the following Fc
and C3b

Fc and C3b



Receptor-mediated endocytosis



Respiratory burst



Pseudopod extension

 33. After extravasation, leukocytes emigrate in the tissue towards


the site of injury. It is called as

Margination



Chemotaxis



Diapedesis


Pavementing

 34. A middle aged scientist Sudarshan is working in the laboratory


on the mechanisms involved in inflammation. He observes that the
leucocytes leave the blood vessels and move towards the site of
bacteria. Which of the following is likely to mediate this mov

Histamine



C3b



C3a



C5a

Pathoanatomy - Wound healing; stem cell biology

Total questions: 18; Correctly answered: 0; Percentage of


correct: 0.0% Skipped question: 0;
correct answer
incorrect answer
skipped question

 1. Which one of the following statements is not correct regarding



Developmental elasticity



Transdifferentiation



Can be harvested from embryo



Amebiasis

 2. Which of the following is the source of hepatic stem cells?



Limbus cells



Ito cell



Oval cell



Paneth cell

 3. An adult old man gets burn injury to his handsOver few weeks,
the burned skin heals without the need for skin graftingThe most
critical factor responsible for the rapid healing in this case is:

Remnant skin appendages



Underlying connective tissues



Minimal edema and erythema



Granulation tissue

 4. Oval cells : are seen in the stem cells of which of the following
tissues?

Skin



Cornea



Liver


Bone

 5. Chronic granulomatous disease is:



Associated with formation of multiple granulomas



A benign neoplastic process



A parasitic disease



Acquired leukocyte function defect

 6. Which of the following adhesion molecules is involved in


morphogenesis?

Osteopontin



Osteonectin SPARC



Tenascin



Thrombospondins

 7. Wound contraction is mediated by:



Epithelial cells



Myofibroblasts



Collagen



Elastin

 8. In regeneration

Granulation tissue



Repairing by same type of tissue



Repairing by different type of tissue



Cellular proliferation is largely regulated by biochemical factors

 9. Absolute lymphocytosis is seen in



SLE



T.B



CLL



Brucellosis

 10. When a cell transforms itself into different lineage the ability us
know as:

De-differentiation



Re-differentiation


Trans-differentiation



Sub-differentiation

 11. Maximum collagen in wound healing is seen at which stage of


healing

End of first week



End of second week



End of third week



End of 2 months

 12. Tensile strength of wound after laparoscopic cholecystectomy


in a 30 year old woman depends upon:

Replacement of type 3 collagen



Extensive cross-linking of tropocollagen



Macrophage activity



Granulation tissue

 13. Prion disease is caused by:



Misfolding of protein


Denaturation of proteins



Reduced formation of proteins



Exces formation of proteins

 14. Which of the following is absolutely essential for wound


healing?

Vitamin D



Carbohydrates



Vitamin C



Balanced diet

 15. One of the following statements about hematopoietic stem cell


is false?

Stem cells have self renewal property



Subset of stem cells normally circulate in peripheral blood



Marrow derived stem cells can seed other tissues and develop into non
hematopoietic cells as well



Stem cells resemble lymphoblasts morphologically


 16. First sign of wound injury is:

Epithelialization



Dilatation of capillaries



Leukocytic infiltration



Localized edema

 17. A 45-year-old man, Suveen presents with pain in the mid


portion of his chestThe pain is associated with eating and
swallowing foodEndoscopic examination reveals an ulcerated
area in the lower portion of his esophagusHistologic sections of
tissue taken fro

Caseating granulomatous inflammation



Dysplastic epithelium



Granulation tissue



Squamous cell carcinoma

 18. Vitamin used for post translational modification of glutamic


acid to gamma carboxy glutamate is

A



D



E


K

Pathoanatomy - Alcoholic liver disease, nodular hyperplasia

Total questions: 17; Correctly answered: 7; Percentage of


correct: 41.2% Skipped question: 0;
correct answer
incorrect answer
skipped question

 1. In Alcoholic liver disease, which of the following pigments is


deposited in the hepatocytes?

Hemosiderin



Hemoglobin



Lipofuschin



Melanin

 2. A chronic alcoholic has an elevated serum alpha fetoprotein


levels Which of the following neoplasms is most likely seen?

Prostatic adenocarcinoma



Multiple myeloma



Hepatocellular carcinoma



Glioblastoma multiforme


 3. Nodular regenerative changes in liver most commonly occur in:

Drugs induced hepatitis



Alcoholic hepatitis



Hepatitis B



Autoimmune hepatitis

 4. A 46-year-old man, Sushil who has a long history of excessive


drinking presents with signs of alcoholic hepatitis Microscopic
examination of a biopsy of this patient

Immunoglobulin



Excess plasma proteins



Prekeratin intermediate filaments



Basement membrane material

 5. Mallory hyaline body is seen in all except:



Indian childhood cirrhosis



Alcoholism



Secondary biliary cirrhosis



a-1 antitrypsin deficiency

 6. The following are true attributes of hepatitis B infection except:



Infants develop chronic infections



HBc Ag in serum is indicative of active infection



Can cause hepatocellular cancer



Interferons are used for treatment

 7. Mallory bodies are composed of:



Fat droplets



Mitochondria



Lysosomal enzymes



Intermediate filaments

 8. Alcoholic hyaline seen in alcoholic liver disease is composed of



Lipofuschin



Eosinophilic intracytoplasmic inclusions



Basophilic intracytoplasmic inclusions


Hemazoin

 9. Mallory's hyaline is seen in:



Hepatitis C infection



Amoebic liver abscess



Indian childhood cirrhosis



Autoimmune hepatitis

 10. In a chronic alcoholic all the following may be seen in the liver
except

Fatty degeneration



Chronic hepatitis



Granuloma formation



Cholestatic hepatitis

 11. Which of the following is not associated with Mallory hyaline


bodies?

Alcoholic liver disease



Primary biliary cirrhosis


Secondary biliary cirrhosis



Indian childhood cirrhosis

 12. Mallory hyaline is seen in:



Alcoholic liver disease



Hepatocellular carcinoma



Wilson's disease



All Of Above

 13. All of the following are true except?



LKM 1



LKM 2



LKM 1



LKM 2

 14. Mallory bodies are seen is



Viral hepatitis



Toxic hepatitis


Alcoholic hepatitis



All

 15. Mallory bodies are seen in all except



Alcoholic cirrhosis



Biliary cirrhosis



Cardiac cirrhosis



Wilson disease

 16. Mallory bodies contain:



Vimentin



Cytokeratin



Keratin



Collagen

 17. Which of the following may not cause microvesicular


steatosis?

Alcoholic fatty liver



Tetracycline toxicity



Acute fatty liver of pregnancy



Reye

Pathoanatomy - Amyloidosis

Total questions: 33; Correctly answered: 5; Percentage of


correct: 15.2% Skipped question: 0;
correct answer
incorrect answer
skipped question

 1. Which one of the following stains is specific for Amyloid?



Periodic Acid Schiff (PAS



Alizarin red



Congo red



Von-Kossa

 2. Cause of death in amyloidosis involving kidney:



Cardiac failure



Renal failure



Sepsis


Liver failure

 3. Which type of Amyloidosis is caused by mutation of the


transthyretin protein?

Familial Mediterranean fever



Familial amyloidotic polyneuropathy



Dialysis associated amyloidosis



Prion protein associated amyloidosis

 4. The best investigation for the diagnosis of amyloidosis is



Colonoscopy



Rectal biopsy



Upper GI endoscopy



CT scan

 5. Secondary amyloidosis complicates which of the following:



Pneumonia



Chronic glomerulonephritis



Irritable bowel syndrome



Chronic osteomyelitis

 6. Amyloid is

Mucopolysaccharide



Lipoprotein



Glycoprotein



Intermediate filament

 7. Serum amyloid associated protein is found in



Alzheimer



Chronic inflammatory states



Chronic renal failure



Malignant hypertension

 8. T-lymphocytes from a 6-year-old female Ramya with severe


recurrent respiratory infections are found to lack the IL-12 receptor.
Supplementation with which of the following substances would be
most helpful in treating this patient

Immunoglobulins



Interferon-g (IFN-g)



Interleukin-2 (IL-2



GM-CSF

 9. Correctly matched pairs in amyloidosis are:



Multiple myeloma - light chain



Chronic inflammation - AA



Cardiac - ATTR



Neural

 10. Amyloidosis is most commonly seen in:



Maturity onset DM



Type 1 DM



Type 2 DM



Equally seen with all forms of DM

 11. On electron microscopy amyloid characteristically exhibits:



Beta-pleated sheat


Hyaline globules



7.5-10 nm fibrils



20-25 nm fibrils

 12. Which type of amyloidosis is caused by mutations in


transthyretin gene?

Familial Mediterranean fever



Familial amyloidosis polyneuropathy



Dialysis associated amyloidosis



Prion protein associated amyloidosis

 13. A diabetic patient is undergoing dialysis. Aspiration done


around the knee joint would show:

A-p2 Microglobulin



AA



AL



Lactoferrin


 14. On Congo- red staining, amyloid is seen as:

Dark brown color



Blue color



Brilliant pink color



Brilliant pink color

 15. A diabetic patient is undergoing dialysis. Aspiration done


around the knee joint would show:

A beta 2 microglobulin



AA



AL



Lactoferin

 16. Familial amyloidotic polyneuropathy is due to amyloidosis of


nerves caused by deposition of:

Amyloid associated protein



Mutant calcitonin



Mutant transthyretin



Normal transthyretin

 17. Which of the following is the chemical nature of Hemodialysis


associated with amyloid?

AA



AL



Beta



ATTR

 18. A 50-year-old presented with signs and symptoms of restrictive


heart disease. A right ventricular endomyocardial biopsy revealed
deposition of extracellular eosinophilic hyaline material. On
transmission electron microscopy, this material is most likely t

Non branching filaments of indefinite length



Cross banded fibers with 67 m periodicity



Weibel Palade bodies



Concentric whorls of lamellar structures

 19. What is the best method for confirming amyloidosis?



Colonoscopy



Sigmoidoscopy


Rectal biopsy



Tongue biopsy

 20. Lardaceous spleen is due to deposition of amyloid in:



Sinusoids of red pulp



White pulp



Pencillary artery



Splenic trabeculae

 21. In amyloidosis Beta pleated sheet will be seen in:



X-ray crystallography



Electron microscope



Spiral electron microscope



Congo red stain

 22. Which of the following is the most serious organ involvement


in amyloidosis?

Cardiac tissue



Renal tissue



Splenic tissue



Hepatic tissue

 23. Neointimal hyperplasia causes vascular graft failure as a result


of hypertrophy of:

Endothelial cells



Collagen fibers



Smooth muscle cells



Elastic fibers

 24. In Hemodialysis associated amyloidosis, which of the


following is seen

Transthyretin



B2 Microglobulin



SAA



A2 Microglobulin

 25. Bone marrow in AL amyloidosis shows



Bone marrow plasmacytosis



Granulomatous reaction



Fibrosis



Giant cell formation

 26. Lardaceous spleen is due to deposition of amyloid in:



Sinusoids of red pulp



White pulp



Pencillary artery



Splenic trabeculae

 27. A 60 year old female is suffering from renal failure and is on


hemodialysis since last 8 years. She developed carpal tunnel
syndrome. Which of the following finding will be associated?

AL



AA



ATTR



P2 microglobulin

 28. Amyloid deposits stain positively with all of the following


EXCEPT:

Congo-red



Crystal violet



Methanamine silver



Thioflavin T

 29. Most common site of biopsy in amyloidosis



Liver



Spleen



Kidney



Lung

 30. Gingival biopsy is useful in the diagnosis of:



Sarcoidosis



Amyloidosis



Histoplasmosis



Scurvy

 31. A 43-year-old women Kanata Devi presents with a several year


history of progressive abdominal colic and constipation. Colonic
biopsy stained with Congo red reveals the acellular material
exhibiting green birefringence. The birefringence is thought to be

Ability to bind to oxygen



Beta-pleated sheet tertiary structure



Electrophoretic mobility



Hydroxyproline content

 32. What are the stains used for Amyloid?



Thioflavin



Congo red



Rhodamine



Auramine

 33. Secondary amyloidosis is associated with



Ab



AL



AA



APrP

Pathoanatomy - Bilirubin metabolism, hyperbilirubinemias

Total questions: 9; Correctly answered: 4; Percentage of


correct: 44.4% Skipped question: 0;
correct answer
incorrect answer
skipped question

 1. In post-hepatic jaundice, the concentration of conjugated


bilirubin in the blood is higher than that of unconjugated bilirubin
because:

There is an increased rate of destruction of red blood cells



The unconjugated bilirubin is trapped by the bile stone produced in the
bile duct.



The conjugation process of bilirubin in liver remains operative without
any interference



The UDP glucuronyl transferase activity is increased manifold in
obstructive jaundice

 2. Which of the following is not a function of liver:



Production of vitamin K



Production of albumin



Detoxification of ammonia


Metabolism of drugs

 3. Function of hepatic stellate cells is/are:



Formation of sinusoids



Vitamin A storage



Ncreases blood perfusion



Phagocytosis

 4. A 40 years old woman Hema Thapar presents with generalized


pruritus for last 4 months which is not relieved by various lotions
available in the market On physical examination is unremarkable
but her blood sample is sent to the laboratory Her reports are a

Anti-parietal cell antibody



Antimitochondrial antibody



Anti-centromere antibody



Anti ribonucleoprotein antibody

 5. Which of the following condition is associated with


unconjugated hyperbilirubinemia?

Dubin-Johnson syndrome



Rotor syndrome



Gilbert syndrome



Gall stones

 6. Which of the following diseases is not a cause of indirect


hyperbilirubinemia?

Rotor



Criggler Najjar syndrome



Gilbert syndrome



Hereditary spherocytosis

 7. Primary biliary cirrhosis is positive for:



P-ANCA



Anti nuclear antibody



Anti-microsomal antibody



Anti-mitochondrial antibody

 8. A patient with unconjugated bilirubinemia has increased


excretion of urobilinogen in his urine This can be seen in all of the
following conditions, except:

G6 PD deficiency



Hemolytic anemia



Hereditary spherocytosis



Biliary cirrhosis

 9. In unconjugated hyperbilirubinemia, the fraction of


unconjugated bilirubin to total bilirubin exceeds

0.65



0.5



0.35



0.8

Pathoanatomy - Billiary tract disorder

Total questions: 10; Correctly answered: 2; Percentage of


correct: 20.0% Skipped question: 0;
correct answer
incorrect answer
skipped question

 1. A 50-year-old male film actor Sallu Kahn looses weight rapidly


for one of his forthcoming films He experiences occasional
abdominal discomfort few days after that and guided by his
physician, he undergoes a radiological scan (HIDA scan) is shown
to have s

Black pigment stones


Brown pigment stones



Biliary sludge



Phospholipid stones

 2. Onion skin fibrosis of the common bile duct is:



Primary biliary cirrhosis



Primary sclerosing cholangitis



Extrahepatic biliary fibrosis



Congenital hepatic fibrosis

 3. Most common bile duct tumor is



Adenocarcinoma



Squamous cell cancer



Transitional cell carcinoma



All

 4. Most common site of Cholangiocarcinoma is?



Distal biliary tree


Hilum



Intrahepatic biliary duct



Multifocal

 5. Focal diffuse gall bladder wall thickening with comet tail


reverberation artifacts on USG are seen in:

Adenomyomatosis of gall bladder



Carcinoma gall bladder



Adenomatous Polyps



Xanthogranulomatous gall bladder

 6. Klatskin tumor is

Nodular type of cholangiocarcinoma



Fibrolamellar hepatocellular carcinoma



Gall bladder carcinoma



Hepatocellular carcinoma

 7. Which is risk factor for cholangiocarcinoma:



Obesity



Primary sclerosing cholangitis



Salmonella carrier state



HBV infection

 8. All of the following are risk factors for carcinoma gall bladder,
except:

Typhoid carriers



Adenomatous gall bladder polyps



Choledochal cysts



Oral contraceptives

 9. A middle aged woman comes to the emergency room


complaining of severe, right-sided abdominal pain, fever, and
chills for the past several hours She has a history of gallstones
and her family doctor recommended a cholecystectomy after a
similar episode

Common bile duct



Cystic duct



Fundus of gallbladder



Proximal duodenum

 10. Cholangiocarcinoma of liver is caused by



Hepatitis B infection



Cirrhosis of liver



Antitrypsin deficiency



Clonorchis sinensis infection

Pathoanatomy - Blood grouping, blood transfusion, anti coagulants

Total questions: 38; Correctly answered: 9; Percentage of


correct: 23.7% Skipped question: 0;
correct answer
incorrect answer
skipped question

 1. All cause pseudohyperkalemia, except



Thrombocytopenia



Leucocytosis



Clenching of fists



Hemolysis

 2. Stored plasma is deficient in



Factors 7 and 8



Factors 5 and 7



Factors 5 and 8



Factors 5, 7 and 8

 3. Hemophilia A is characterized by:



Prolonged PTT



Prolonged PT



Low platelet count



Abnormal BT

 4. Which of the following is seen in peripheral smear of a patient


who has underwent splenectomy:

Howell-Jolly bodies



Eosinophilia



Macrocytosis



Thrombocytopenia


 5. The anticoagulant of choice for performing coagulation studies
is:

EDTA



Heparin



Trisodium citrate



Double oxalate

 6. Hemophilia is associated with :



X chromosome



Y Chromosome



Chromosome 3



Chromosome 16

 7. Most common blood transfusion reaction is:



Febrile non-hemolytic transfusion reaction



Hemolysis



Transmission of infections



Electrolyte imbalance

 8. Hypersplenism is characterized by all except:



Leukemoid reaction



Thrombocytopenia



Splenomegaly



Responds to splenectomy

 9. Secondary hemochromatosis is associated with all except:



Thalassemia



Sideroblastic anemia



Multiple drug transfusions



Paroxysmal nocturnal hemoglobinuria

 10. The antigen lacking in Rh negative person is



C



D



d



E

 11. A 55 years old male accident victim in casualty urgently needs


blood. The blood bank is unable to determine his ABO group, as
his red cell group and plasma group do not match. Emergency
transfusion of the patient should be with:

RBC corresponding to his red cell group and colloids/crystalloid



Whole blood corresponding to his plasma group



O positive RBC and colloids/crystalloid



AB negative whole blood

 12. Patient with hemophilia A have bleeding disorder because of:



Lack of platelet aggregation



Lack of reaction accelerator during activation of factor X in coagulation
cascade



Neutralization of antithrombin III



Release of Thromboxane A2

 13. True about hemophilia is:



If the male is affected, it will transmit to male



Normal PT



Low PT



Low aPTT

 14. Rh antigen is a/an:



Antibody



Mucopolysaccharide



Protein



Fatty acid

 15. A newborn with ABO incompatibility, the characteristic feature


on peripheral smear is the presence of

Microspherocytes



Fragmented RBC



Polychromasia



Elliptocytosis

 16. Blood component products are all except



Whole blood



Platelets


Fresh frozen plasma



Packed red blood cells

 17. Which of the following regarding Bombay blood group is false?



Lack of H, A and B antigen on RBCs



Lack of H, A and B substance in saliva



Lack of antigens of several blood group systems



H, A and B antibody will always be present in serum

 18. The anticoagulant of choice for anticoagulation testing



Heparin



EDTA



Sodium oxalate



3.2% trisodium citrate

 19. Which of the following is the genotype of a person with blood


group A

BO



AO



AB



OO

 20. True about von Willebrand factor are all except



Component of factor VIII



Synthesized by hepatocytes



Facilitate the adhesion of platelets



None

 21. A 40 years old male had undergone splenectomy 20 years ago.


Peripheral blood smear examination would show the presence of:

Dohle bodies



Hypersegmented neutrophils



Spherocytes



Howell-Jolly bodies

 22. Hemophilia B is due to deficiency of:



Factor VIII


Factor VII



Factor IX



Factor X

 23. Bence Jones protein in urine are due to the presence of



Light chain of monoclonal immunoglobulins



Heavy chain of monoclonal immunoglobulins



Light chain of polyclonal immunoglobulins



Heavy chain of polyclonal immunoglobulins

 24. Carbohydrate present in blood group substance is



Fucose



Deoxyribose



Ribulose



Ribose


 25. You are working in a PHC and have to send a sample for blood
glucose estimation. Which of the following anticoagulant will you
use for sending your sample?

EDTA



Heparin



Potassium oxalate + sodium fluoride



Tri Sodium citrate

 26. Howell Jolly bodies are seen in:



Liver disease



Postsplenectomy



Hemolysis



DIC

 27. Platelets growth factor are synthesized by



Glial cells



Endothelium



Fibroblasts



All of the above

 28. All are true about polycythemia vera except:



Increased LAP score



Increased erythropoietin level



Splenomegaly



May cause Budd-Chiari syndrome

 29. Blood group antigens are



Carried by sex chromosomes



Attached to plasma proteins



Attached to hemoglobin molecule



Sometimes found in saliva

 30. A 7-year-old girl presents with bleeding in joints. She has


prolonged aPTT, normal PT and platelet counts. What could be the
deficiency?

Factor IX



Factor VIII



Factor VII



Von Willebrand Factor

 31. Spur cell anemia is caused by:



Chronic liver disease



Acute blood loss



Chronic blood loss



None

 32. “Starry Sky” pattern is seen in all of these except



Burkitt’s lymphoma



Large B cell lymphoma



Small cleaved cell lymphoma



Lymphoblast lymphoma

 33. Elevated ESR is seen in following conditions except:



Polymyositis rheumatica



Multiple myeloma


Temporal arteritis



Polycythemia rubra

 34. Which of the following complications is likely to result after


several units of blood have been transferred?

Metabolic alkalosis



Metabolic acidosis



Respiratory alkalosis



Respiratory acidosis

 35. Although more than 400 blood groups have been identified, the
ABO blood system re-mains the most impor tant in clinical
medicine because:

It was the first blood group system to be discovered



It has four different blood group A, B, AB, O (H)



ABO (H) antigens are present in most body tissues and fluids



ABO (H) antibodies are invariably present in plasma when persons
RBC lack the corres-ponding antigen

 36. ABO incompatibility not seen with:



Fresh frozen plasma



Platelet rich plasma



Single donor platelets



Cryoprecipitate

 37. The major hemoglobin present in an adult is



HbA2



HbA1



HbA1c



HbA1b

 38. Platelets growth factor are synthesized by



Glial cells



Endothelium



Fibroblasts



All of the above


Pathoanatomy - Breast tissue

Total questions: 17; Correctly answered: 2; Percentage of


correct: 11.8% Skipped question: 0;
correct answer
incorrect answer
skipped question

 1. A female patient presented with a firm mass of



P53



BRCA-2



Her 2/Neu gene



C-myc gene

 2. Histologic hallmark of Paget



Caseous necrosis



Infiltration of the epidermis by malignant cells



Atypical lobular hyperplasia



Desmoplasia

 3. The type of mammary ductal carcinoma in situ (DCIS) most


likely to result in a palpable abnormality in the breast is:

Apocrine DCIS



Neuroendocrine DCIS


Will-differentiated DCIS



Comedo DCIS

 4. BRCA 1 gene is located on:



Chromosome 13



Chromosome 11



Chromosome 17



Chromosome 22

 5. Bilateral breast carcinoma is:



Scirrhous carcinoma



Medullary carcinoma



Lobular carcinoma



Ductal carcinoma

 6. A 54-year-old female Shanti presents for an annual exam. Her


right breast is swollen, red, and tender. The physician palpates a
firm area in the breast and suspects inflammatory breast cancer.
Which of the following best describes the histological changes

Acute inflammation in breast carcinoma


Chronic inflammation in breast carcinoma



Dermal lymphatic invasion by cancer cells



Epidermal invasion by cancer cells

 7. Increased susceptibility to breast cancer is likely to be


associated with a mutation in the following gene:

P53



BRCA-1



Retinoblastoma (Rb)



H-Ras

 8. Indian file pattern is seen in histopathological examination of:



Infiltrating duct carcinoma



Fibroadenoma



Fibro carcinoma



Lobular carcinoma

 9. BRCA-1 gene lies on chromosome:



17



18



20



21

 10. Paget's disease of the nipple is:



Infection



Dermatisis



Neoplasia



Hypopigmentation

 11. Which of the following breast tumors is bilateral?



Colloid carcinoma



Invasive ductal carcinoma



Invasive lobular carcinoma



Medullary carcinoma


 12. Fleshy, soft lymphatic infiltration of skin in breast cancer
appears as:

Puckering



Peau



Cancer encurasse



All of the above

 13. Tumor marker useful in the diagnosis of the cancer of the


breast is:

CEA



AFP



CA-125



CA-15-3

 14. Rare histological variants of carcinoma breast with better


prognosis include all except:

Colloid carcinoma



Medullary carcinoma



Inflammatory carcinoma



Tubular carcinoma

 15. BRCA-1 gene lies on chromosome:



17



18



20



21

 16. Commonest carcinoma of the breast with multifocal origin is:



Scirrhous carcinoma



Adenocystic carcinoma



Lobular carcinoma



Ductal carcinoma

 17. Lesions affecting the teminal duct lobulat unit (TDLU) in breast
are all except

Nipple adenoma



Blunt duct adenosis



Intraductal papilloma



Fibroadenoma

Pathoanatomy - Cardiac tumour

Total questions: 4; Correctly answered: 1; Percentage of


correct: 25.0% Skipped question: 0;
correct answer
incorrect answer
skipped question

 1. Atrial myxoma commonly arises from:



Left ventricle



Left atrium



Right ventricle



Right atrium

 2. Which malignancy metastasizes to heart?



Bronchial carcinoma



Prostate carcinoma



Breast carcinoma



Wilms


 3. Most common benign heart tumor is:

Rhabdomyoma



Hemangioma



Lipoma



Myxoma

 4. Most common tumour of heart is:



Myxoma



Rhabdomyosarcoma



Fibroma



Leiomyosarcoma

Pathoanatomy - Cellinjury, necrosis, apoptosis

Total questions: 50; Correctly answered: 10; Percentage of


correct: 20.0% Skipped question: 0;
correct answer
incorrect answer
skipped question

 1. Annexin v on non-permeable cell is indicative of:



Apoptosis



Necrosis


Cell entering replication phase



Cell cycle arrest

 2. Following gene when mutated protects tumor cells from


Apoptosis

BCL



BRCA



RB



TGF

 3. In apoptosis, Apaf-I is activated by release of which of the


following substances from the mitochondria?

Bcl-2



Bax



Bcl-XL



Cytochrome C

 4. Which of the following if accumulated is suggestive of


reversible cell injury due to hypoperfusion of different

Carbon dioxide


Creatinine



Lactic acid



Troponin I

 5. Fibrinoid necrosis may be observed in all of the following,


except?

Malignant hypertension



Polyarteritis nodosa



Diabetic glomerulosclerosis



Aschoffs nodule

 6. Coagulative necrosis as a primary event is most often seen in all


except:

Kidneys



CNS



Spleen



Liver


 7. Which of the following best represents the biochemical change
in the myocardial cells of this patient during the transient
hypoxia?

Decreased hydrogen ion concentration



Increase in oxidative phosphorylation



Loss of intracellular Na+ and water



Stimulation of anaerobic glycolysis and glycogenolysis

 8. Apoptosis is alternatively called as



Ischemic cell death



Programmed cell death



Post traumtic cell death



All

 9. Caspases are associated with which of the following?



Hydopic degeneration



Collagen hyalinization



Embryogenesis



Fatty degeneration

 10. In cell death, myelin figures are derived from:



Nucleus



Cell membrane



Cytoplasm



Mitochondria

 11. If we presume that the patient has experienced several similar


episodes of pain over the last 10 hours, which of the following
ultra-structural changes would most likely indicate irreversible
myocardial cell injury in this patient?

Myofibril relaxation



Disaggregation of polysomes



Mitochondrial vacuolization



Disaggregation of nuclear granules

 12. Which of the following is the characteristic of irreversible


injury on electron microscopy?

Disruption of ribosomes



Amorphous densities in mitochondria


Swelling of endoplasmic reticulum



Cell swelling

 13. Cytosolic cytochrome C plays an important function in



Apoptosis



Cell necrosis



Electron transport chain



Cell division

 14. Cells most sensitive to hypoxia are:



Myocardial cells



Neurons



Hepatocytes



Renal tubular epithelial cells

 15. Apoptosis is

Cell degeneration



Type of cell injury


Cell regeneration



Cell activation

 16. All of the following statements are true regarding reversible


cell injury, except

Formation of amorphous densities in the mitochondrial matrix



Diminished generation of adenosine triphosphate



Formation of blebs in the plasma membrane.



Detachment of ribosomes from the granular endoplasmic reticulum.

 17. Irreversible cell injury is characterised by which of the


following?

Mitochondrial densities



Cellular swelling



Blebs



Myelin figures

 18. About apoptosis, true statement is:



Injury due to hypoxia


Inflammatory reaction is present



Councilman bodies is associated with apoptosis



Cell membrane is damaged

 19. A patient subbu is diagnosed with a cancer. It was observed


that he shows a poor response to a commonly used anti-cancer
drug which acts by increasing programmed cell death. Inactivation
of which of the following molecules/genes is responsible for the
res

Granzyme and perforin



Bcl-2



p53



Cytochrome P450

 20. Fibrinoid necrosis may be observed in all of the following,


except:

Malignant hypertension



Polyarteritis nodos



Diabetic glomerulosclerosis



Aschoff nodule

 21. Irreversible injury in cell is



Deposition of Ca++ in mitochondria



Swelling



Mitotic figure



Ribosomal detachment

 22. Organelle that plays a pivotal role in apoptosis:



Endoplasmic reticulum



Golgi complex



Mitochondria



Nucleus

 23. Which one of the listed statements best describes the


mechanism through which Fas(CD95) initiates apoptosis?

BCL2 product blocks channels



Cytochrome activates Apaf-1



FADD stimulates caspase 8



TNF inhibits Ikb

 24. Pyogenic infection and brain infarction are associated with



Coagulative necrosis



Liquefactive necrosis



Caseous necrosis



Fat necrosis

 25. All of the following are morphological features of apoptosis


except

Cell shrinkage



Chromatin condensation



Inflammation



Apoptotic bodies

 26. Coagulative necrosis is seen in:



Brain



Breast



Liver


All

 27. Which of the following is not seen is apoptosis?



Chromatin condensation



DNA fragmentation



Inflammation



Cell membrane shrinkage

 28. Coagulative necrosis is found in which infection?



TB



Sarcoidosis



Gangrene



Fungal infection

 29. liquefactive necrosis is seen in:



Heart



Brain



Lung


Spleen

 30. In apoptosis initiation:



The death receptors induce apoptosis when it engaged by fas ligand
system



Cytochrome C binds to a protein Apoptosis Activating (Apaf-1) Factor



Apoptosis may be initiated by caspase activation



Apoptosis mediated through DNA damage

 31. In apoptosis, cytochrome C acts through:



Apaf 1



Bcl-2



FADD



TNF

 32. liquefactive necrosis is seen in:



Heart



Brain



Lung



Spleen

 33. Intracellular calcification begins in which of the following


organelles?

Mitochondria



Golgi body



Lysososme



Endoplasmic reticulum

 34. Coagulative necrosis as a primary event is most often seen in


all except

Kidneys



CNS



Spleen



Liver

 35. Dr Maalu Gupta is carrying out an experiment in which a


genetic mutation decreased the cell survival of a cell culture line.
These cells have clumping of the nuclear chromatin and reduced
size as compared to normal cells. Which of the following is the
mos

Fas



Bax



Bcl-2



Myc

 36. Light microscopic characteristic feature of apoptosis is:



Intact cell membrane



Eosinophilic cytoplasm



Nuclear moulding



Condensation of the nucleus

 37. Most pathognomic sign of irreversible cell injury chromatin



Amorphous densities in mitochondria



Swelling of the cell membrane



Ribosomes detached from endoplasmic reticulum (



Clumping of nuclear chromatin


 38. Ultra-structural finding of irreversible injury

Ribosomal detachment from endoplasmic reticulum



Amorphous densities in mitochondria



Formation of phagolysosomes



Cell swelling

 39. Which of the following induces apoptosis in a cell?



Oleic acid



Isoprenoids



Myristic acid



Glucocorticoids

 40. First cellular change in hypoxia:



Decreased oxidative phosphorylation in mitochondria



Cellular swelling



Alteration in cellular membrane permeability



Clumping of nuclear chromatin


 41. Ladder pattern of DNA electrophoresis in apoptosis is caused
by the action of the following enzyme:

Endonuclease



Transglutaminase



DNAse



Caspase

 42. Internucleosomal cleavage of DNA is characteristic of



Reversible cell injury



Irreversible cell injury



Necrosis



Apoptosis

 43. Following is seen in both apoptosis and necrosis:



Both may be physiological



Both may be pathological



Inflammation



Intact cell membrane

 44. True about apoptosis is all, except:



Considerable apoptosis may occur in a tissue before it becomes
apparent in histology



Apoptotic cells appear round mass of the intensely eosinophilic
cytoplasm with dense nuclear chromatin fragments



Apoptosis of cells induce inflammatory reaction



Macrophages phagocytose the apoptotic cells and degrade them

 45. Fournier’s gangrene is seen in:



Nose



Scrotal skin



Oral cavity



All are true

 46. A 55-year-old man, vikas develops a thrombus in his left


anterior descending coronary artery. The area of myocardium
supplied by this vessel is irreversibly injured. The thrombus is
destroyed by the infusion of streptokinase, which is a
plasminogen activa

Apoptotic bodies



Flocculent densities


Myelin figures



Russell bodies

 47. A 23-year-old lady sweety was driving her car when she had to
apply brakes suddenly. she suffered from

Apocrine metaplasia



Dystrophic fat necrosis



Enzymatic fat necrosis



Granulomatous inflammation

 48. A patient Fahim presents to the hospital with jaundice, right


upper quadrant pain and fatigue. He tests positive for hepatitis b
surface antigen. The serum bilirubin levels is 4.8mg/dl (direct is
0.8mg/dl and indirect bilirubin is 4.0mg/dl), AsT levels is

Bleb formation



Cell membrane rupture



Clumping of nuclear chromatin



Swelling of endoplasmic reticulum

 49. Caspases are involved in



Necrosis



Apoptosis



Atherosclerosis



Inflammation

 50. CD 95 is a marker of

Intrinsic pathway of apoptosis



Extrinsic pathway of apoptosis



Necrosis of cell



Cellular adaption


Pathoanatomy - Cellular adaptation, intracellular accumulation

Total questions: 34; Correctly answered: 11; Percentage of


correct: 32.4% Skipped question: 0;
correct answer
incorrect answer
skipped question

 1. Example of hypertrophy is:



Breast in puberty



Uterus during pregnancy



Ovary after menopause


Liver after resection

 2. Gamma Gandy bodies contain hemosiderin and:



Na+



Ca++



Mg++



K+

 3. Psammoma bodies show which type of calcification



Metastatic



Dystrophic



Secondary



Any of the above

 4. Oncocytes are modified form of which of the following:



Lysososmes



Endoplasmic reticulum



Mitochondria


None of the above

 5. A 50-year-old male alcoholic, Rajesh presents with symptoms of


liver disease and is found to have mildly elevated liver enzymes. A
liver biopsy examined with a routine hematoxylin and eosin (H & e)
stain reveals abnormal clear spaces in the cytoplasm of m

Calcium



Cholesterol



Hemosiderin



Triglyceride

 6. An increase in the size of a cell in response to stress is called as


hypertrophy. Which of the following does not represent the
example of smooth muscle hypertrophy as an adaptive response
to the relevant situation?

Urinary bladder in urine outflow obstruction



Small intestine in intestinal obstruction



Triceps in body builders



None of the above

 7. The light brown perinuclear pigment seen on H & e staining of


the cardiac muscle fibres in the grossly normal appearing heart of
an 83 year old man at autopsy is due to deposition as:

Hemosiderin



Lipochrome



Cholesterol metabolite



Anthracotic pigment

 8. True about metastatic calcification is



Calcium level is normal



Occur in dead and dying tissue



Occur in damaged heart valve



Mitochondria involved earliest

 9. Mallory hyaline is seen in:



Alcoholic liver disease



Hepatocellular carcinoma



Wilson



I.C.C. (Indian childhood cirrhosis)


 10. Calcification of soft tissues without any disturbance of calcium
metabolism is called

Inotrophic calcification



Monotrophic calcification



Dystrophic calcification



Calcium induced calcification

 11. A 36-year-old woman, Geeta presents with intermittent pelvic


pain. Physical examination reveals a 3-cm mass in the area of her
right ovary. Histologic sections from this ovarian mass reveal a
papillary tumor with multiple, scattered small, round, laminate

Bacterial infection



Dystrophic calcification



Enzymatic necrosis



Metastatic calcification

 12. The Fenton reaction leads to free radical generation when:



Radiant energy is absorbed by water



Hydrogen peroxide is formed by Myeloperoxidase



Ferrous ions are converted to ferric ions


Nitric oxide is converted to peroxynitrite anion

 13. All are true about metaplasia except



Slow growth



Reverse back to normal with appropriate treatment



Irreversible



If persistent may induce cancer transformation

 14. True about psammoma bodies are all except:



Seen in meningioma



Concentric whorled appearance



Contains calcium deposits



Seen in teratoma

 15. Both hyperplasia and hypertrophy are seen in?



Breast enlargement during lactation



Uterus during pregnancy



Skeletal muscle enlargement during exercise


Left ventricular hypertrophy during heart failure

 16. An old man Muthoot has difficulty in urination associated with


increased urge and frequency. He has to get up several times in
night to relieve himself. There is no history of any burning
micturition and lower back pain. On rectal examination, he has
enla

Hyperplasia



Atrophy



Hypertrophy



Metaplasia

 17. Transformation of one epithelium to other epithelium is known


as

Dysplasia



Hyperplasia



Neoplasia



Metaplasia

 18. Metastatic calcification is most often seen in:



Lymph nodes



Lungs



Kidney



Liver

 19. Brown atrophy is due to



Fatty necrosis



Hemosiderin



Lipofuscin



Ceruloplasmin

 20. A patient Ramu Kaka presented with complaints of slow


progressive breathlessness, redness in the eyes and skin lesions.
His chest X ray had bilateral hilar lymphadenopathy. His serum
ACe levels were elevated. On doing Kveim test, it came out to be
positi

The calcification in sarcoidosis begins at a cellular level in mitochondria



There is presence of dystrophic calcification



The granulomatous lesions contain macrophages which cause
activation of vitamin D precursors



None of the above

 21. Heterotopic calcification occurs in:



Ankylosing spondylitis



Reiter



Forrestier



Rheumatoid arthritis

 22. About hyperplasia, which of the following statement is false?



No of cells



Size of the affected cell



Endometrial response to estrogen is an example



All

 23. Russell bodies are seen in



Lymphocytes



Neutrophils



Macrophages



Plasma cells

 24. Dystrophic calcification is seen in:



Rickets



Hyperparathyroidism



Atheromatous plaque



Vitamin A intoxication

 25. Which of the following is not a common site for metastatic


calcification?

Gastric mucosa



Kidney



Parathyroid



Lung

 26. Pigmentation in the liver is caused by all except:



Lipofuscin



Pseudomelanin



Wilson



None

 27. Dystrophic calcification is seen in:



Atheroma



Paget



Renal osteodystrophy



Milk-alkali syndrome

 28. Wear and tear pigment in the body refers to



Lipochrome



Melanin



Anthracotic pigment



Hemosiderin

 29. Mallory hyaline bodies are seen all except



Indian childhood cirrhosis



Wilson



Alcoholic hepatitis


Crigler-Najjar syndrome

 30. Psammoma bodies are seen in all except:



Follicular carcinoma of thyroid



Papillary carcinoma of thyroid



Serous cystadenoma of ovary



Meningioma

 31. Psammoma bodies are typically associated with all of the


following neoplasms except

Medulloblastoma



Meningioma



Papillary carcinoma of the thyroid



Papillary serous cystadenocarcinoma of the ovary

 32. Metastatic calcification occurs in all except:



Kidney



Atheroma



Fundus of stomach



Pulmonary veins

 33. Dystrophic calcification is:



Calcification in dead tissue



Calcification in living tissue



Calcification in dead man



None

 34. A 28-year-old male executive presents to the doctor with


complaints of “heartburn” non responsive to usual medicines
undergoes endoscopy with biopsy of the distal esophagus is
taken. What type of mucosa is normal for the distal esophagus?

Ciliated, columnar epithelium



Keratinized, stratified, squamous epithelium



Non-keratinized, simple, squamous epitheliu



Non-keratinized, stratified, squamous epithelium

Pathoanatomy - Chemical mediators of inflammation

Total questions: 50; Correctly answered: 7; Percentage of


correct: 14.0% Skipped question: 0;
correct answer
incorrect answer
skipped question

 1. Cytokines:

Includes interleukins



Produced only in sepsis



Are polypeptide (complex proteins)



Have highly specific action

 2. Procalcitonin is used as marker of



Cardiac dysfunction in acute coronary syndrome



Menstrual periodicity



Pituitary function



Procalcitonin

 3. Which of the following complement component can be activated


is both common as well as alternative pathways?

C1



C2



C3


C4

 4. Birbeck’s granules in the cytoplasm are seen in:



Langerhans cells



Mast cells



Myelocytes



Thrombocytes

 5. Cytokines are secreted in sepsis and Systemic Inflammatory


Response Syndrome (SIRS) by:

Neutrophils



Adrenal



Platelets



Collecting duct

 6. Interleukin secreted by macrophages, stimulating lymphocytes


is:

IFN alpha



TNF alpha


IL-1



IL-6

 7. Bleeding time assesses:



Extrinsic clotting pathway



Intrinsic clotting pathway



Fibrinogen level



Function of platelets

 8. An 8 year old girl Geetu presents to the physician with wheezing


and difficulty in breathing. The breathlessness increases when
she went to the fields to play with her friends. Her blood contains
higher than normal concentration of IgE. The physician beli

Neutrophil



Monocyte



Basophil



Eosinophil

 9. A 14-year-old girl Radha has high grade fever. She goes to a


physician Dr. Jeeva Roy who orders for some blood investigations.
A complete blood count with differential implies the presence of a
viral infection. Which of the following best describes the ce

They are basophilic with spherical dark-stained nuclei



They are precursors of osteoclasts and liver Kupffer cells



They have azurophilic granules and multilobed nuclei



They contain a peripheral hyalomere and central granulomere

 10. Lewis triple response is caused due to:



Histamine



Axon reflex



Injury to endothelium



Increased permeability

 11. The role of bradykinin in process of inflammation is:



Vasoconstriction



Bronchodilation



Pain



Increased vascular permeability

 12. Eosinophils are activated by:



IL1



IL5



IL4



IL6

 13. All are mediators of neutrophils except:



Elastase



Cathepsin



Nitric oxide



None

 14. The Eosinophils secrete all except



Major basic protein



Hydrolytic enzyme



Reactive form of O2



Eosinophilic chemotactic factor

 15. Which of the following is secondary mediator of the


anaphylaxis is:

Histamine



Proteases



Eosinophilic chemotactic factor



Leukotriene B4

 16. Nephrocalcinosis in a systemic granulomatous disease is due


to

Over production of 1,25 dihydroxy vitamin D



Dystrophic calcification



Mutation in calcium sensing receptors



Increased reabsorption of calcium

 17. An 18-year-old woman, Sheila is being evaluated for recurrent


facial edema, especially around her lips. She also has recurrent
bouts of intense abdo-minal pain and cramps, sometimes
associated with vomiting. Laboratory examination finds decreased
C4, whi

B2-integrins



C1 esterase inhibitor


Decay-accelerating factor



Complement components C3 and C5

 18. PAF causes all except



Bronchoconstriction



Vasoconstriction



Decreased vascular permeability



Vasodilation

 19. All of the following are mediators of acute inflammation except



Angiotensin



Prostaglandin E2



Kallikrein



C3a

 20. Conversion of prothrombin to thrombin requires:



V only



V and Ca++


XII



X and Ca++

 21. Histamine causes



Hypertension



Vasoconstriction



Vasodilation



Tachycardia

 22. Most important mediator of chemotaxis is:



C3b



C5a



C5-7



C2

 23. To which of the following family of chemical mediators of


inflammation, the Lipoxins belong?

Kinin system



Cytokines



Chemokines



Arachidonic acid metabolites

 24. Which complement fragments are called



C3a and C3b



C3b and C5b



C5a and C3b



C3a and C5a

 25. Bradykinin is for:



Pain



Vasodilatation



Vasoconstriction



Increase vascular permeability

 26. All of the following are true in respect of angioneurotic edema


except?

It is caused by deficiency of complement proteins


It is more common in females



It manifests as pitting edema



It is an autosomal dominant disorder

 27. A 72 year-old man Kishori Lal presented to surgery OPD with a


history of difficulty in micturition, increased frequency of urine
and lower backache for the past 8 months. Digital rectal
examination reveals an enlarged prostate with irregular surface.
The

Fibroblast growth factor



Interleukin-2



Tumor necrosis factor-a



Vascular endothelial growth factor

 28. Bradykinin causes:



Vasoconstriction



Pain at the site of inflammation



Bronchodilation



Decreased vascular permeability

 29. The most important source of histamine:



Mast cells



Eosinophil



Neutrophil



Macrophages

 30. Cryoprecipitate is rich in which of the following clotting


factors?

Factor II



Factor V



Factor VII



Factor VIII

 31. Cell-matrix adhesions are mediated by?



Cadherins



Integrins



Selectins



Calmodulin

 32. E cadherin gene deficiency is seen in:



Gastric cancer



Intestinal cancer



Thyroid cancer



Pancreatic cancer

 33. Ultra-structurally, endothelial cells contain



Weibel Palade bodies



Langerhan



Abundant glycogen



Kallikrein

 34. Febrile response in CNS is mediated by



Bacterial toxin



IL-l



IL-6


Interferon

 35. Most important bactericidal agent is:



Cationic basic protein



Lactoferrin



Lysozyme



Reactive O2 species

 36. All of the following are mediators of inflammation except:



Tumour necrosis factor-a (TNF-a)



Interleukin-1



Myeloperoxidase



Prostaglandins

 37. Which of the following is found in secondary granules of


neutrophils?

Catalase



Gangliosidase



Proteolytic enzyme



Lactoferrin

 38. Cryoprecipitate is rich in which of the following clotting


factors:

Factor II



Factor V



Factor VII



Factor VIII

 39. Which of the following is not a pyrogenic cytokine?



IL - 1



TNF



IFN - a



IL - 18

 40. The estimation of the prothrombin level is useful in the


following clotting factor deficiency, except:

II



V



VII



IX

 41. Both antibody dependent and independent complement


pathway converge on which complement component?

C3



C5



C1q



C8

 42. Pro inflammatory Cytokines include all of the following except:



Interleukin 1



Interleukin-10



Interleukin 6



TNF- Alpha

 43. Partial thromboplastin time correlates with:



Intrinsic and common pathway


Extrinsic and common pathway



Vessel wall integrity and intrinsic pathway



Platelet functions and common pathway

 44. A 28-year-old woman, Vimla is being evaluated to find the


cause o her urine turnin a dark rown color after a recent upper
respiratory tract infection. She has been otherwise symptomatic,
and her blood pressure has een within normal limits. Urinalysis
find

Decreased Normal Normal



Serum C2 normal, C3 decreased, C4 normal



Normal Normal Decreased



Decreased Normal Decreased

 45. Which of the following is not an inflammatory mediator?



Tumor Necrosis Factor



Myeloperoxidase



Interferons



Interleukin

 46. Factor present in the final common terminal complement


pathway is:

C4



C3



C5



Protein B

 47. Prostaglandins are synthesized from:



Linoleic acid



Linolenic acid



Arachidonic acid



Butyric acid

 48. Following injury to a blood vessel, immediate haemostasis is


achieved by which of the following?

Fibrin deposition



Vasoconstriction



Platelet adhesion


Thrombosis

 49. Fever occurs due to:



IL 1



Endorphin



Enkephalin



Histamine

 50. C-C beta chemokines includes



IL-8



Eotaxin



Lymphotactin



Fractalkine


Pathoanatomy - Chromosomal disorders and karyotyping

Total questions: 33; Correctly answered: 3; Percentage of


correct: 9.1% Skipped question: 0;
correct answer
incorrect answer
skipped question

 1. Trisomy 13 is identified as

Edward


Patau



Down



Klinefelter

 2. Osteogenesis imperfecta defect in:



Collagen type I



Elastin



Collagen type IV



Fibrillin 2

 3. A married middle aged female gives history of repeated


abortions for the past 5 years. The prenatal karyogram of the
conceptus is given below This karyogram suggests the following

Klinefelter



Turner



Down’s syndrome



Patau


 4. The genetics involved in Down syndrome is:

Maternal non-disjunction



Paternal non-disjunction



Mosacism



Monosomy

 5. Karyotype is:

Size, shape and number of chromosome



Gene packing



DNA assay



None

 6. Y-chromosome is

Telocentric



Metacentric



Submetacentric



Acrocentric


 7. In Down syndrome, there is non disjunction of chromosome

13



15



18



21

 8. In marfan syndrome, the defect is in:



Fibrillin I



Fibrillin II



Collagen



Elastin

 9. Patient present with skin bullae on sun exposure. There is a


defect in which of the following

Thymidine dimers



Trinucleotide repeats



Sugar changes



DNA methylation

 10. A nineteen year old female with short stature, wide spread
nipples and primary amenorrhea most likely has a karyotype of:

47, XX + 18



46, XXY



47, XXY



45 X

 11. A 8-year-old boy Gullu with the Down’s syndrome has an


intelligence quotient (IQ) in the mid-normal range. Which of the
following genetic mechanisms would most likely account for the
discrepancy between the child’s IQ and his appearance?

Balanced translocation



Chiasma



Mosaicism



Spermiogenesis

 12. Chromosomes are visualized through light microscope with


resolution of:

5 Kb



50 mb



5 mb



500 Kb

 13. Barr body is not seen in:



Klinefelter syndrome



Turner syndrome



Normal female



XXX syndrome

 14. Males who are sexually underdeveloped with rudimentary


testes and prostate glands, sparse pubic and facial hair, long arms
and legs and large hands and feet are likely to have the
chromosome complement of:

45, XYY



46, XY



46, XXY



46, X

 15. If a chromosome divides in an axis perpendicular to usual axis


of division it is going to form:

Ring chromosome


Isochromosome



Acrocentric chromosome



Subtelocentric chromosome

 16. Karyotyping is done for:



Chromosomal disorders



Autosomal recessive disorders



Autosomal dominant disorders



Linkage disorders

 17. Karyotyping is done in which phase of cell cycle?



Anaphase



Metaphase



Telophase



S phase

 18. Which of the following tests is used to differentiate the


chromosome of normal and cancer cells?

PCR



Comparative genomic hybridization



Western blotting



Karyotyping

 19. Chromosomal abnormality in Mongolism is



Trisomy 21



Trisomy 22



Trisomy 17



Trisomy 5

 20. The classic karyotype of Klinefelter



47 XXY



45 XO



48 XXXY



46 XY/47 XXY


 21. Chromosomes are visualized through light microscope with
resolution of:

Trisomy 21



Mosaic 21



Translocation t (14,21), t (21,21)



Deletion of 21

 22. A 36-year-old retarded man with a strong history of mental


retardation among male relatives undergoes genetic testing. His
lymphocytes on metaphase arrest show a breakpoint at q27.3 on
the X chromosome. This man is at increased risk for which of the
foll

Aortic stenosis



Atrial septal defect



Mitral valve prolapse



Tricuspid atresia

 23. The number of chromosomes in Klinefelter syndrome is:



47



46



45


44

 24. Which of the following is true of Klinefelter



Chromosome pattern in 47 XXY



Mental retardation is present



Hypogonadism occurs



Increased FSH level

 25. The number of chromosomes in Turner syndrome is:



47



46



45



44

 26. Which of the following techniques can be used to detect exact


localisation of a genetic locus?

Chromosome painting



FISH



Comparative genomic hybridization



Western blot

 27. No change of genetic material occurs in which of the following


cytogenetic abnormalities?

Deletion



Insertion



Translocation



Inversion

 28. Karyotyping most commonly done under light microscopy



G-banding



Q banding



C banding



R banding

 29. A tall man with gynecomastia and testicular atrophy has a


testicular biopsy that shows sparse, completely hyalinized
seminiferous tubules with a complete absence of germ cells and
only rare Sertoli cells. Leydig cells are present in large clumps
between

Testicular feminization syndrome


Trisomy 18



Trisomy 21



47, XXY

 30. Patau syndrome is due to which of the following?



Trisomy 21



Trisomy 18



Trisomy 21



Trisomy 13

 31. Which of the following procedures as routine technique for


karyotyping using light microscopy

C-banding



G-banding



Q-banding



Brd V-staining


 32. A 16-year-old female Bholi presents to Dr. Sindhu, a
gynecologist because she has never had menstrual bleeding. She
is 132 cm tall, weighs 44 kg, has swelling around the neck,
increased carrying angle at the elbow and poorly developed
secondary sexual ch

Mitotic error in early development



Trinucleotide repeat expansion



Uniparental disomy



Balanced reciprocal translocation

 33. Effective polymerase reaction was repeated for 3 cycles on a


DNA molecule. What will be the resulting formation of the copies?

Double number of copies



Three times the number of DNA molecule



Four times the number of DNA molecule



8 times

Pathoanatomy - Chronic inflammation; granulomatous inflammation

Total questions: 19; Correctly answered: 3; Percentage of


correct: 15.8% Skipped question: 0;
correct answer
incorrect answer
skipped question

 1. The epithelioid cell and multinucleated giant cells of


Granulomatous inflammation are derived from:

Basophils



Eosinophils



CD4-T lymphocytes



Monocytes-Macrophages

 2. A 36-year-old man, Avnish presents with a cough, fever, night


sweats, and weight loss. A chest X-ray reveals irregular densities
in the upper lobe of his right lung. Histologic sections from this
area reveal groups of epithelioid cells with rare acid-fast

Bronchial cells



Pneumocytes



Lymphocytes



Monocytes

 3. Epithelioid granulomatous lesions are found in all of the


following diseases, except:

Tuberculosis



Sarcoidosis



Berylliosis



Pneumocystis carinii

 4. Which of the following is the most characteristic of granuloma:



Epithelioid cell



Giant cell



Fibroblasts



Endothelial cell

 5. Epithelioid granuloma is caused by:



Neutrophil



Cytotoxic T-cells



Helper T-cells



NK cells

 6. Which of these is not a granulomatous disease



Leprosy



Tuberculosis



Sarcoidosis


Amebiasis

 7. Granulomatous inflammatory reaction is caused by all, except:



M. tuberculosis



M. leprae



Yersinia pestis



Mycoplasma

 8. In a granuloma, epithelioid cells and giant cells and derived from



T



Monocyte – macrophages



B



Mast cells

 9. Non-caseating granuloma is characteristically seen in



Syphilis



Sarcoidosis



Tuberculosis


Histoplasmosis

 10. Necrotizing epithelioid cell granulomas are seen in all, except:



Tuberculosis



Wegener



Cat Scratch disease



Leprosy

 11. A 45-year-old poor man Teja has a chronic cough, a cavitary


lesion of the lung, and is sputum positive for acid-fast bacilli.
Which of the following is the principle form of defense by which
the patient

Antibody-mediated phagocytosis



Cell-mediated immunity



IgA-mediated hypersensitivity



Neutrophil ingestion of bacteria

 12. Caseous necrosis in granuloma are not found in



Tuberculosis



Leprosy


Histoplasmosis



CMV

 13. Non-caseating granulomas are seen in all of the following


except

Byssinosis



Hodgkin



Metastatic carcinoma of lung



Tuberculosis

 14. Caseating granuloma are seen in:



Histoplasmosis



Sarcoidosis



Coccidiodomycosis



All

 15. The most important function of epithelioid cells in tuberculosis


is:

Phagocytosis


Secretory



Antigenic



Healing

 16. Caseous granuloma is seen in



Histoplasmosis



Silicosis



Sarcoidosis



Foreign body

 17. Granuloma is pathological feature of all, except



Giant cell arteritis



Microscopic polyangiitis



Wegener



Churg Strauss disease

 18. All are granulomatous diseases except



Syphilis


Sarcoidosis



Schistosomiasis



P. carinii

 19. In a lymph node showing non necrotizing and non–caseating


granuloma which of the following is suspected?

Toxoplaxmosis



Lymphogranuloma venereum



Cat scratch disease



Kikuchis lymphadenitis


Pathoanatomy - Cirrhosis, ncpf

Total questions: 11; Correctly answered: 1; Percentage of


correct: 9.1% Skipped question: 0;
correct answer
incorrect answer
skipped question

 1. Commonest site of varices in portal hypertension is



Esophagus



Anal canal



Periumbilical


Liver

 2. In cirrhosis of liver collagen is laid down by



Hepatocytes



Hepatic stellate cells



Biliary epithelial cells



Kupffer cells

 3. Micronodular cirrhosis is seen in all except:



Alcoholic cirrhosis



Viral hepatitis



Budd-Chiari syndrome



Indian childhood cirrhosis

 4. An old alcoholic Chivas R Signature presents to the medical


emergency of GS Hospital with confusion and lethargy The
medical specialist on duty, Dr Thamim carried out the physical
examination in which Mr Signature was visibly jaundiced with
ascites His

Capillary telangiectasias



Asterixis



Caput medusae



Gynecomastia

 5. Nutmeg liver is seen in:



Right sided heart failure



Left sided heart failure



Increased pulmonary pressure



Decreased pulmonary pressure

 6. Nutmeg liver is seen in which of the following conditions?



Right sided heart failure



Left sided heart failure



Increased pulmonary pressure



Decreased pulmonary pressure

 7. A 50-year-old chronic alcoholic with jaundice and ascites


secondary to known cirrhosis becomes disoriented and confused
Asterixis (flapping tremor) can be demonstrated Which of the
following is not associated with the development of ascites?

Hypoalbuminemia



Increased hepatic lymph formation



Increased portal venous pressure



Portal-systemic venous shunting

 8. Which one of the following is not a feature of liver histology in


Non cirrhotic portal fibrosis?

Fibrosis in and around the portal tracts



Thrombosis of the medium and small portal vein branches



Non specific inflammatory cell infiltrates in the portal tracts



Bridging fibrosis

 9. A 30 years old man Surajmal visits his physician because he


noticed the development of yellowish skin during last 5 days His
physical examination has absence of abdominal pain or
tenderness His blood reports are as follows: Haemoglobin 11 5
g/dL, MCV 94

Cholelithiasis



HAV infection



Micronodular cirrhosis


Hemolytic anemia

 10. Nutmeg liver is seen in



Portal cirrhosis



Biliary cirrhosis



Chronic venous congestion of liver



Fatty liver

 11. A 42-year-old woman Kiran with polycythemia vera develops


progressive severe ascites and tender hepatomegaly over a period
of several months Liver function tests are near normal Which of
the following tests would be most likely to establish the probable

Endoscopic retrograde cholangiopancreatography



Hepatic venography



Serum alpha fetoprotein



Serum iron

Pathoanatomy - Cns infections, demyelinating disease

Total questions: 21; Correctly answered: 6; Percentage of


correct: 28.6% Skipped question: 0;
correct answer
incorrect answer
skipped question
 1. Cerebral infarction is caused by:

Toxoplasma



Cryptococcus



Aspergillus



Mucor

 2. Complications of tubercular meningitis are:



Endarteritis



Hydrocephalus



Deafness



Venous sinus infarct

 3. Perivascular lymphocytes and microglial nodules are seen in:



Multiple sclerosis



CMV meningitis



Bacterial meningitis



HIV encephalitis


 4. The pathogenesis of cerebral malaria includes:

Cytoadhesion



Sequestration of cerebral vessels by RBCs



Reticulocytopenia



Also caused by P. vivax

 5. Locomotor ataxia, a late manifestation of syphilis due to


parenchymatous involvement of the spinal cord is called:

General paralysis of insane



Tabes dorsalis



Meningovascular syphilis



Syphilitic amyotrophy

 6. Most common type of pathological changes seen in Rabies are:



Meningitis



Cranial arteritis



Ventriculitis



Brain stem encephalitis

 7. All of the following diseases show abnormal folding of proteins


except:

Creutzfeldt-Jakob disease



Prion disease



Multiple sclerosis



Amyloidosis

 8. Albumino-cytologic dissociation occurs in cases of:



Guillain Barre syndrome



TB meningitis



Motor neuron disease



Demyelinating disorder

 9. A 17 year old female presents with a history of fever and


headache and now develops altered sensorium. CT scan shows
basal exudates with meningeal enhancement. The CSF is most
likely to show

Lymphocytic pleocytosis, low sugar, low protein



Polymorphonuclear pleocytosis, normal sugar, high protein



Lymphocytic pleocytosis, low sugar, high protein



Lymphocytic pleocytosis, normal sugar, high protein

 10. Pathologic features of brain in AIDS are all, except:



Perivascular giant cell invasion



Microglial nodules



Vasculitis



Temporal lobe infarction

 11. All of the following are seen in thymoma except



Hypogamma globulinemia



Hyperalbuminemia



Red cell aplasia



Myasthenia Gravis

 12. Spongiform degeneration of cerebral cortex occurs in which of


the following?

Subacute sclerosing panencephalitis



Fatal familial insomnia


Creutzfeldt-Jakob disease



Cerebral toxoplasmosis

 13. Febrile response in CNS is mediated by:



Bacterial toxin



IL-l



IL-6



Interferon

 14. Brain infarct is seen in:



TB



Cryptococcosis



Aspergillosis



Toxoplasmosis

 15. Commonest cause of cerebral infarction is:



Arterial thrombosis



Arteritis


Venous thrombosis



Embolism

 16. Prion includes:



DNA and RNA



Only RNA



Proteins



Only DNA

 17. Inclusion body in oligodendroglia is a feature of which of the


following?

Progressive Multifocal Leucoencephalopathy



Japanese Encephalitis



Polio



CJD

 18. Which of the following is not a Prion disease?



Creutzfeldt- Jakob disease



Fatal familial insomnia



Gerstmann-Straussler-Scheinker syndrome



Parkinson

 19. Albumino-cytologic dissociation occurs in cases of:



Guillain Barre syndrome



TB meningitis



Motor neuron disease



Demyelinating disorder

 20. Dissociated sensory loss is seen in:



Syringomyelia



Vitamin B12 deficiency



Transverse myelitis



Pellagra

 21. What is the histological appearance of brain in


Creutzfeldt-Jakob disease?

Neuronophagia


Spongiform change in brain



Microabscesses



Demyelination

Pathoanatomy - Developmental defects, cerebral hemorrhage,


aneurysm

Total questions: 6; Correctly answered: 0; Percentage of


correct: 0.0% Skipped question: 0;
correct answer
incorrect answer
skipped question

 1. Cervical syringomyelia all are seen except



Burning sensation in hands



Hypertrophy of abductor pollicis brevis



Extensor plantar response is present



Absent biceps reflex.

 2. Which of the following would distinguish hydrocephalus due to


aqueductal stenosis when compared to that due to Dandy walker
malformation?

Third ventricle size



Posterior fossa volume



Lateral ventricular size



Head circumference

 3. The defect in Berry aneurysm is



Degeneration of internal elastic lamina



Degeneration of media



Deposition of mucoid material in media



Low grade inflammation of vessel wall

 4. Middle meningeal vessel damage results in:



Subdural hemorrhage



Extradural hemorrhage



Subarachnoid hemorrhage



Intracerebral hemorrhage

 5. The best described etiology for Berry aneurysm is which of the


following?

Degeneration of internal elastic lamina



Degeneration of tunica media



Defect in muscular layer



Low grade inflammation in the vessel wall

 6. Most common site for berry aneurysm is:



Basilar artery



Anterior communicating artery



Posterior communicating artery



Posterior cerebral artery

Pathoanatomy - Esophagus

Total questions: 7; Correctly answered: 3; Percentage of


correct: 42.9% Skipped question: 0;
correct answer
incorrect answer
skipped question

 1. All are precancerous for carcinoma of esophagus except:



Achalasia



Peterson Kelly syndrome



Zenker diverticulum



Ectodermal dysplasia

 2. Which of the following locations is most likely for the


development of carcinoma in a 50-year-old mason who has
chewed tobacco for 25 years?

Floor of the mouth



Lower lip



Tongue



Buccal mucosa

 3. A 50 year old male Grisham who complains of nocturnal cough


undergoes an esophageal biopsy which shows elongation of the
lamina propriae, many eosinophils, and occasional neutrophils
within the squamous epithelium. He takes metformin and enalapril
for his

Pill-induced esophageal mucosa damage



Gastroesophageal junction incompetence



Mucosal disruption from fungal infection



Absent esophageal peristaltic movements

 4. Most common anatomical location of tongue cancer is:



Anterior third



Lateral margin



Dorsum



Posterior third

 5. A 80 year old man Baba complains of pain in the upper portion


of his neck on swallowing. He occasionally regurgitates
undigested food shortly after eating. Which of the following is the
most likely etiology of his problems?

Mallory-Weiss tears



Zenker’s diverticulum



Schatzki rings



Traction diverticula

 6. A 43-year-old female Divya with chronic dysphagia undergoes


an upper endoscopy that reveals massive dilation of the distal
esophagus. The esophagus is kinked and tortuous and partly filled
with undigested foods. What is the most likely diagnosis for this

Achalasia



Barrett



Hiatal hernia



Plummer-Vinson syndrome

 7. A middle aged man Nitesh complains of increasing difficulty in


swallowing over the past 3 years. He reports a feeling of pressure
in his chest occurring 2-3 seconds after swallowing a solid bolus.
He also experiences regurgitation of undigested food eaten

Achalasia



Diffuse esophageal spasm



Incompetent lower esophageal sphincter



Oropharyngeal dysphagia

Pathoanatomy - Etiological agents for cancer: chemical, radiation,


microbes, multistep carcinogenesis

Total questions: 24; Correctly answered: 6; Percentage of


correct: 25.0% Skipped question: 0;
correct answer
incorrect answer
skipped question

 1. One of the following leukemia almost never develops after


radiation?

Acute myeloblastic leukemia



Chronic myeloid leukemia



Acute lumphoblastic leukemia



Chronic lymphocytic leukemia


 2. Biopsy of an ulcerated gastric lesion of a 26-year-old smoker
Akki demonstrates glands containing cells with enlarged,
hyperchromatic nuclei below the muscularis mucosa. Two tripolar
mitotic figures are noted. With which of the following infectious
agents

Epstein-Barr virus



Helicobacter pylori



Human papilloma virus



Molluscum contagiosum virus

 3. Post transplant lymphoma is caused by which of the following?



CMV



EBV



Herpes simplex



HHV-6

 4. Which of the following is an oncogenic RNA virus?



Hepatitis B virus



Human papilloma virus



Epstein Barr virus


Hepatitis C virus

 5. Which among the following is not a neoplastic virus:



Cytomegalovirus



Hepatitis B virus



Human papilloma virus



All of these

 6. Kaposi’s sarcoma is seen with



HCV



HPV



HSV



HHV

 7. Radiation exposure during infancy has been linked to which one


of the following carcinoma?

Breast



Melanoma



Thyroid



Lung

 8. Skin cancers develop due to sunlight exposure induced by:



UVA rays



UVB rays



UVC rays



UVD rays

 9. Thorium induced tumor is which of the following?



Renal cell carcinoma



Lymphoma



Angiosarcoma of liver



Astrocytoma

 10. UV radiation has which of the following effects on the cells?



Prevents formation of pyrimidine dimers



Stimulates formation of pyrimidine dimers


Prevents formation of purine dimers



All of the above

 11. LMP-1 gene plays a role in oncogenesis induced by:



Human T cell leukemia virus type I



Hepatitis B virus



Epstein-Barr virus



Human papilloma virus

 12. The SI unit of radiation absorbed dose is



Rad



Becquerel



Gray



Sievert

 13. Smoking is a risk factor for all carcinomas, except:



Oral



Bronchial


Bladder



Thyroid

 14. H. pylori infection is associated with development of which


malignancy:

MALTomas



Atherosclerosis



Sarcoma



Gastrointestinal stromal tumor (GIST)

 15. The following parasitic infections predispose to malignancies?



Paragonimus westermani



Guinea worm infection



Clonorchiasis



Schistosomiasis

 16. A 37-year-old man, Gagan presents with increasing abdominal


pain and jaundice. He gives a history of intake of groundnuts
which did not taste appropriate. Physical examination reveals a
large mass involving the right side of his liver, and a biopsy
specim

Aflaxotin B1



Direct-acting alkylating agents



Vinyl chloride



Azo dyes

 17. Which of the following is essential for tumor metastasis?



Angiogenesis



Tumorogenesis



Apoptosis



Inhibition of tyrosine kinase activity

 18. The most radiosensitive cells are:



Neutrophils



Lymphocytes



Erythrocytes



Megakaryocytes


 19. Helicobacter pylori infection is associated with all of the
following conditions, except:

Peptic ulcer disease



Gastric adenocarcinoma



B cell lymphoma



Burkitt’s lymphoma

 20. Workers exposed to polyvinyl chloride may develop following


liver malignancy

Cholangiocarcinoma



Fibrolamellar carcinoma



Angiosarcoma



All of the above

 21. Which of the following statements about carcinogenesis is


false?

Asbestos exposure increases the incidence of lung cancer



Papilloma viruses produce tumors in animals but not in humans



Exposure to aniline dyes predisposes to cancer of the urinary bladder



Hepatitis B virus has been implicated in hepatocellular carcinoma

 22. Tumors associated with organisms are all except:



Hepatocellular cancer



Non-small Cell Carcinoma of Lung



Gastric cancer



Nasopharyngeal cancer

 23. Most radiosensitive tumor is:



Renal cell carcinoma



Carcinoma colon



Hepatocellular carcinoma



Testicular seminoma

 24. A man Alok Nath contracts HTLV-1 infection through sexual


contact. Twenty-one years later he develops generalized
lymphadenopathy with hepatosplenomegaly, a skin rash,
hypercalcemia, and an elevated white blood count. This man has
most likely developed wh

AIDS



Autoimmunity


Delayed hypersensitivity reaction



Leukemia


Pathoanatomy - Female genital tract

Total questions: 20; Correctly answered: 7; Percentage of


correct: 35.0% Skipped question: 0;
correct answer
incorrect answer
skipped question

 1. A 30-year-old woman Shagun visits her gynecologist for a


surgery. After laparotomy, a mass is removed which on
microscopic examination demonstrates a cystic cavity filled with
hair and keratin debris, and the wall contains skin, adnexal tissue,
thyroid ti

Immature teratoma



Leiomyoma



Leiomyosarcoma



Mature teratoma

 2. With regard to the malignant behavior of leiomyosarcoma, the


most important criterion is:

Blood vessel penetration by tumor cells



Tumor cells in lymphatic channels



Lymphocyte infiltration


The number of mitoses per high power field

 3. The incidence of bilaterality in a dermoid cyst is approximately:



10%



30%



50%



70%

 4. Carcino-sarcoma occurs in:



Uterus



Liver



Breast



Lungs

 5. Tennis Racquet cells are seen in :



Rhabdomyoma



Rhabdomyosarcoma



Histiocytoma


Eosinophilic granuloma

 6. All are true about polycystic ovarian disease except:



Persistently elevated LH



Increased LH/FSH ratio



Increased DHEAS



Increased prolactin

 7. Schiller - Duval bodies are seen in:



Teratoma



Seminoma



Yolk-Sac tumor



Choriocarcinoma

 8. Uterine leiomyoma is least likely to undergo:



Malignant change



Hyaline change



Calcification


Red degeneration

 9. A patient with chronic pelvic pain undergoes a hysterectomy.


The resected uterus is filled with nodules composed of benign
smooth muscle cells. Which of the following terms best describes
these nodules?

Angiosarcoma



Leiomyoma



Leiomyosarcoma



Rhabdomyoma

 10. Choriocarcinoma is characterized by all except:



Primarily trophoblastic tumor



It can occur following hydatidiform mole



Villi present



It can metastasize to lungs

 11. An adenofibroma of the ovary in which the epithelial


component consists of the nests of transitional cells is called:

Thecoma



Brenner tumour



Serous cystadenoma



Granulose cell tumor

 12. Bilateral ovarian masses are identified on pelvic examination of


a 40-year-old woman for which she undergoes total abdominal
hysterectomy. Pathologic examination demonstrates papillary
carcinoma producing serous fluid. Which of the following tumor
markers

Alpha-fetoprotein



Bombesin



CA-125



PSA

 13. Hormone produced by endodermal sinus tumor is



AFP



Alpha1 antitrypsin



Both



hcG


 14. The risk of sarcoma developing in a fibroid uterus is
approximately:

< 1%



10%



30%



50%

 15. Endodermal sinus tumor is characterized by:



Call Exner body



Psammoma body



Schiller duval body



Homer wright body

 16. Call-Exner bodies are seen in:



Mature teratoma



Endodermal sinus tumor



Granulosa cell tumor



Sertoli Leydig cell tumor

 17. The cytogenicity of solid tumors is not easily assessed


especially in carcinoma cervix because

Metaphase is distinct



Due to contamination with infectious agents



High mitotic rate



Deficient tissue sample

 18. Sections from a solid-cystic unilateral ovarian tumor in a


30-year old female show a tumor composed of diffuse sheets of
small cells with doubtful nuclear grooving and scanty cytoplasm.
No Call-Exner bodies are seen. The ideal immunohistochemistry
panel w

Vimentin, epithelial membrane antigen, inhibin, CD99



Desmin, S-100 protein, smooth muscle antigen, cytokeratin



Chromogranin, CD45, CD99, CD20



CD3, Chromogranin, CD 45, Synaptophysin

 19. An ovarian neoplasm in a 14-year old girl is most likely to be:



Germ cell tumor



Epithelial tumor



Sertoli-Leydig cell tumor



Granulosa cell tumor

 20. Call-Exner bodies are characteristic feature of



Granulosa theca cell tumor



Brenner tumor



Dysgerminoma



Endodermal sinus tumor


Pathoanatomy - Genetic mechanisms of carcinogenesis: protooncogene,
tumour suppressor gene, defective dna repair

Total questions: 46; Correctly answered: 12; Percentage of


correct: 26.1% Skipped question: 0;
correct answer
incorrect answer
skipped question

 1. The inheritance pattern of familial Retinoblastoma is:



Autosomal recessive



Autosomal dominant



X-linked dominant



X-linked recessive


 2. Rosettes are characteristically seen in

Retinoblastoma



Melanoma



Dysgerminoma



Lymphoma

 3. Which of the following is DNA repair defect?



Retinoblastoma



Neurofibromatosis



Xeroderma pigmentosum



MEN-I

 4. Rosettes are characteristically seen in



Retinoblastoma



Melanoma



Dysgerminoma



Lymphoma


 5. Proto-oncogene erb-B is not related to:

Breast carcinoma



Small cell lung carcinoma



Non-small cell lung carcinoma



Ovarian carcinoma

 6. Which is not a tumor suppressor gene?



WT-1



Rb



P53



Ras

 7. Angiogenesis is

Formation of the new blood vessels



Repair by connective tissues



Formation of the blood clot



All of the above


 8. The normal cellular counterparts of oncogenes are important for
the following functions, except:

Promotion of cell cycle progression



Inhibition of apoptosis



Promotion of DNA repair



Promotion of nuclear transcription

 9. An example of a tumour suppressor gene is:



Myc



Fos



Ras



RB

 10. Rosettes are characteristically seen in



Retinoblastoma



Melanoma



Dysgerminoma



Lymphoma

 11. The tumor suppressor gene p 53 induces cell cycle arrest at:

G2



S



G1 – S phase



G0 phase

 12. Which of the following is known as the “guardian of the


genome”?

P53



Mdm2



P14



ATM

 13. Which of the following is a true statement regarding the above


gene?

It is associated with the



It is called as



It is active in hypopohosphorylated form


It is active in hyperphosphorylated form

 14. True statements about p53 gene are all except:



Arrests cell cycle at G1 phase



Product is 53 kD protein



Located on chromosome 17



Wild/non-mutated form is associated with increased risk of childhood
tumors

 15. A patient Madhu undergoes total thyroidectomy for a mass


lesion of the thyroid. During the surgery it is found that the
parathyroid glands appeared enlarged. The thyroid lesion shows
neuroendocrine-type cells and amyloid deposition. This patient’s
thyroid

Bcl-2



C-myc



Ret



L-myc

 16. Following are required for normal growth



Proto-oncogenes



Tumor suppressor genes



Oncogenes



DNA repair genes

 17. True about oncogene is:



Present in normal cell



They are of viral origin



They are transduced from virus infected cells



P53 is most common oncogene mutation causing malignancy

 18. True about proto-oncogenes is:



Important for normal cell growth



May get converted into oncogenes



C-myc over-expression causes lymphoma



Their mutation causes retinoblastoma

 19. Loss of heterozygosity associated with:



Acute myeloid leukemia


ALL



Retinoblastoma



Promyelocytic leukemia

 20. Endometrial carcinoma is associated with which of the


following tumor suppression gene mutation?

P53



Rb



PTEN



APC

 21. The most common secondary malignancy in a patient having


retinoblastoma is:

Osteosarcoma



Renal cell carcinoma



Pineoblastoma



Osteoblastoma


 22. Which of the following gene defect is associated with
development of medullary carcinoma of thyroid:

RET Proto Oncogene



Fap gene



Rb gene



BRCA 1 gene

 23. Growth factor oncogene is:



Myc



Fos



Sis



Jun

 24. HER2/neu receptor plays a role in



Predicting therapeutic response



Diagnosis of breast cancer



Screening of breast cancer



Recurrence of tumor

 25. Knudson two hit hypothesis is seen with



Melanoma



Retinoblastoma



Ulcerative colitis



Crohn disease

 26. Xeroderma pigmentosum is caused due to a group of closely


related abnormalities in:

Mismatch repair



Base excision repair



Nucleotide excision repair



SOS repair

 27. Dr. Marwah, a pediatrician, performing an ophthalmoscopic


examination on a four-year-old boy, notices several small
pigmented nodules in his irises. He also notices six light brown
macules on the trunk of the child of variable sizes. This boy may
have a

Bladder



Colon



Peripheral nerve



Skin

 28. Most common genetic mutation in carcinogenesis involves:



p53



Rb



HPC



PTEN

 29. All of the following are tumor markers, except:



Beta-2 macroglobulin



HCG



Alpha-fetoprotein



CEA

 30. Lynch syndrome is associated with cancers of the:



Breast, colon, ovary



Breast, endometrium, ovary


Breast, colon, endometrium



Colon, endometrium, ovary

 31. Cancer cell survival is enhanced by:



Suppression of p53 protein



Over expression of p53 gene



bcl-2



Bax

 32. Which of the following mutations in a tumor suppressor agent


causes breast carcinoma?

p43



P53



p73



P83

 33. MYC gene is:



Protein kinase inhibitor



Growth factor inhibitor



GTPase



Transcription activator

 34. An example of a tumor suppressor gene is:



Myc



Fos



ras



Rb

 35. In the mitogen activated protein kinase pathway, the activation


of RAS is counteracted by:

Protein kinase C



GTPase activating protein



Phosphatidyl inositol



Inositol triphosphate

 36. Medullary carcinoma of thyroid is associated with mutation in:



RET


RAS



NF



Rb

 37. Regarding Fanconi anemia, the wrong statement is:



Autosomal dominant



Bone marrow show pancytopenia



Usually aplastic anemia



It is due to defective DNA repair

 38. Retinoblastomas arising in the context of germ-line mutations


not only may be bilateral, but also may be associated with (so
called “trilateral” retinoblastoma)

Medulloblastoma



Pinealoblastoma



Neuroblastoma



Hemangioblastoma


 39. Retinoblastoma is associated with which of the following
tumours?

Osteoclastoma



Hepatocellular cancer



Squamous cell cancer



Osteosarcoma

 40. Aisha, a 51 year old woman discovers a lump in her left breast
on a weekly self-examination. Mammography is performed which
confirms the presence of a suspicious “mass”, and needle core
biopsy is performed to determine whether the mass is malignant.
Dr. D

GTPase



GTPase-activating protein



Receptor tyrosine kinase



Retinoic acid receptor protein

 41. RET gene mutation is associated with which malignancy?



Pheochromocytoma



Medullary carcinoma thyroid



Lymphoma


Renal cell carcinoma

 42. All are true about Fanconi anemia, except:



Defect in DNA repair



Bone marrow hyper function



Congenital anomaly present



Increased chances of cancer

 43. Increased expression of which of the following causes


oncogenesis

IGF receptor



EGF receptor



GH receptor



Aldosterone receptor

 44. APC gene is located on which chromosome



Chromosome 5



Chromosome 6



Chromosome 9



Chromosome 11

 45. The following statements are true about Tumor Suppressor


Gene p53, except:

It regulates certain genes involved in cell cycle regulation



Its increased levels can induce apoptosis



Its activity in the cells decreases following UV irradiation and stimulates
cell cycle



Mutations of the p53 gene are most common genetic alteration seen in
human cancer

 46. Tumor suppressor genes are all, except



APC



p53



Rb



C-myc


Pathoanatomy - Glomerular disease: nephritic syndrome, nephrotic
syndrome, glomerulonephritis

Total questions: 50; Correctly answered: 14; Percentage of


correct: 28.0% Skipped question: 0;
correct answer
incorrect answer
skipped question

 1. Post streptococcal glomerulonephritis in children is diagnosed


by:

Heavy protienuria, high cholesterol, high ASO titre



Heavy protienuria, hematuria, low ASO titre



Mild proteinuria, hematuria, high ASO titre



Mild protienuria, high cholesterol, normal ASO titre

 2. All of the following decrease in Nephrotic syndrome except:



Thyroxin



Transferrin



Fibrinogen



Albumin

 3. In a specimen of kidney, fibrinoid necrosis is seen and onion


peel appearance is also present. Most probable pathology is:

Hyaline degeneration



Hyperplastic arteriosclerosis



Glomerulosclerosis


Fibrillary glomerulonephritis

 4. What is the cause of hypercoagulation in nephrotic syndrome:



Loss of antithrombin III (AT III)



Decreased fibrinogen



Decreased metabolism of vitamin K



Increase in Protein C

 5. All of the following are associated with low complement levels


except:

Lupus nephritis



Mesangiocapillary glomerulonephritis



Diarrhea-associated hemolytic uremic syndrome



Post-infections glomerulonephritis

 6. A 7 year old boy presented with generalized edema. Urine


examination revealed marked albuminuria. Serum biochemical
examinations showed hypoalbuminemia with hyperlipidemia.
Kidney biopsy was undertaken. On light microscopic examination,
the kidney appear

Fusion of foot processes of the glomerular epithelial cells



Rarefaction of glomerular basement membrane



Deposition of electron dense material in the basement membrane



Thin basement membrane

 7. An IV drug abuser Chulbul develops an aggressive form of


nephrotic syndrome that does not respond to steroids. A renal
biopsy is performed. Which of the following histological
diagnoses will most likely be made from the biopsy tissue?

Focal segmental glomerulosclerosis



IgA nephropathy



Membranous glomerulonephritis



Membranoproliferative glomerulonephritis

 8. Which of the following is a feature of Collapsing


glomerulopathy?

Tuft necrosis



Mesangiolysis



Parietal epithelial proliferation



Hypertrophy and necrosis of visceral epithelium


 9. Pauci-immune glomerulonephritis is seen in:

RPGN



IgA nephropathy



Microscopic polyangitis



FSGS

 10. Pauci-immune crescentic glomerulonephritis is associated


with:

Microscopic polyangiitis



SLE



H S Purpura



PAN

 11. A 60-year-old man develops oliguria and peripheral edema over


a period of weeks. Urinalysis reveals hematuria and proteinuria;
examination of the urinary sediment reveals red cell casts.
Radiologic and ultrasound studies fail to demonstrate an
obstructive

Anti-glomerular basement membrane disease



Diabetic nephropathy



Hypertensive nephropathy


Lupus nephritis

 12. A child had hematuria and nephrotic syndrome (minimal


change disease) was diagnosed. True about it is:

A type of focal segmental GN



IgA deposition on basement membrane



Foot process of glomerular membrane normal



Glomerular function is lost due to loss of polyanionic charge on both
sites of glomerular foot process

 13. Which of the following is the diagnosis for a condition having


mutation in COL4A5 chain?

Alports syndrome



Good pasture’s syndrome



Thin membrane disease



Nodular glomerulosclerosis

 14. Which of these does not cause crescentic glomerulonephritis?



Rapidly progressive glomerulonephritis



Alport syndrome


Goodpasture’s syndrome



Henoch-Schönlein purpura

 15. Which of the following is not true about Berger’s disease?



The pathological changes are proliferative and usually confined to
mesangial cells; usually focal and segmental



Hematuria may be gross or microscopic



On immunofluorescence deposits contain with IgA and IgG



Absence of associated proteinuria is pathognomic

 16. True about Heymann rat glomerulonephritis is:



Heymann antigen is called megalin



Electron dense deposits in subendothelial space



Electron dense deposits in mesangium



Subepithelial aspect of basement membrane have deposits

 17. Finnish type of nephrotic syndrome is associated with:



Nephrin



Podocin



Alpha actinin



CD2 activated protein

 18. Most common mutation seen in congenital nephrotic


syndrome is:

Nephrin



Podocin



A 4 actinin



Megalin

 19. The Electron Microscopy is virtually diagnostic in renal biopsy


study of:

Goodpasture’s syndrome



Churg-Strauss syndrome



Alport syndrome



Wegner’s granulomatosis

 20. Mutation in alpha 5 chain of collagen type IV is seen in:



Alport syndrome



Thin membrane disease



Nodular glomerulosclerosis



Good pasture syndrome

 21. IgA nephropathy is characterized by all of the following except:



Hypertension



Hematuria



Nephritic syndrome



Renal biopsy having thin basement membrane

 22. A 35-year-old woman Sumitra has had type 1 diabetes mellitus


for 20 years. She is now developing advanced disease with visual
complaints, foot ulcers, and renal disease. Which of the following
features that might be seen on renal biopsy is most specific f

Mesangial IgA deposits



Necrotic epithelial cells in tubules



Nests of cells with abundant clear cytoplasm



Ovoid, periodic acid-Schiff (PAS)-positive, hyaline masses

 23. Flea bitten appearance of the kidney is seen in:



Malignant hypertension



Benign hypertension



Chronic pyelonephritis



Diabetes mellitus

 24. Fibronectin nephropathy has all of the following features


except

Autosomal recessive inheritance



Associated with mesangial expansion



Glomeruli do not stain for immunoglobulin or complement



PAS- positive amyloid negative deposits

 25. Visceral leishmaniasis causes



Membranous glomerulonephritis



Mesangioproliferative glomerulonephritis



Focal segmental glomerulonephritis



Rapidly progressive glomerulonephritis

 26. The prognosis of rapidly proliferating glomerulonephritis


(Crescentic GN) depends upon

Number of crescents



Size of crescents



Shape of crescents



Cellularity of crescents

 27. In glomerular disease which of the following is mainly excreted


in urine:

Albumin



Globulin



Light chain



Heavy chain

 28. All are non-proliferative glomerulonephritis, except:



Membranous glomerulonephritis



Mesangiocapillary glomerulonephritis



Diabetic glomerulosclerosis



Amyloidosis

 29. Which type of FSGS has worst prognosis?



Tip variant



Collapsing



NOS



Perihilar

 30. Crescents are derived from which of the following?



Epithelial cells + fibrin + macrophage



Mesangial cell + fibrin + macrophage



Tubular cell + mesangial cell + fibrin



Neutrophil + tubular cell + fibrin

 31. Most common cause of nephrotic syndrome in adults is:



Membranous glomerulonephritis



Minimal change disease


Acute GN



Focal segmental glomerulosclerosis

 32. Type I membranoproliferative glomerulonephritis is commonly


associated with all except:

SLE



Persistent hepatitis C infections



Partial lipodystrophy



Neoplastic diseases

 33. A person with radiologically confirmed reflux nephropathy


develops nephritic range proteinuria. Which of the following would
be the most likely histological finding in this patient?

Focal segmental glomerulosclerosis



Nodular glomerulosclerosis



Membranous glomerulopathy



Proliferative glomerulonephritis with crescents

 34. Steroid resistant nephrotic syndrome is caused due to


mutation in the gene encoding for?

Nephrin



Alpha-actinin-4



Podocin



Transient Receptor Potential 6

 35. A 7 yrs old girl is brought with complaints of generalised


swelling of the body. Urinary examination reveals grade 3
proteinuria and the presence of hyaline and fatty casts. She has no
history of hematuria. Which of the following statements about her
cond

IgA nephropathy is likely diagnosis



Her C3 levels will be low



No IgG deposits or C3 deposition on renal biopsy



Alport syndrome is likely diagnosis

 36. Mesangial deposits of monoclonal kappa/Lambda light chains


in indicative of

Mesangioproliferative glomerulonephritis



Focal and segmental glomerulosclerosis



Kimmelstiel-Wilson lesions



Amyloidosis

 37. In which one of the primary glomerulonephritis the glomeruli


are normal by light microscopy but shows loss of foot processes
of the visceral epithelial cells and no deposits by electron
microscopy

Poststreptococcal glomerulonephritis



Membrano-proliferative glomerulonephritis type I



IgA nephropathy



Minimal change disease

 38. The most common gene defect in idiopathic steroid resistance


nephrotic syndrome

ACE



NPHS 2



HOX11



PAX

 39. Microalbuminuria is defined as protein levels of:



100-150 mg/d



151-200 mg/d


30-300 mg/d



301-600 mg/d

 40. The pathological feature in Wegener’s granulomatosis on renal


biopsy is

Nodular glomerulosclerosis



Focal necrotizing glomerulonephritis



Granulomas in the vascular wall



Granuloma of parenchyma of kidney

 41. In renal disease, Albumin is first to appear in urine because



Of its high concentration in plasma



Has molecular weight slightly greater than the molecules normally
getting filtered



High Albumin Globulin ratio



Tubular epithelial cells are sensitive to albumin

 42. Histology of Alport syndrome:



Foamy cells in interstitium


Foamy cells in tubular epithelial cells



Thickening of GBM > 100 nm



Thinning of GBM < 100 nm

 43. In Wegener’s granulomatosis, kidney has which of the


following lesions?

Glomerular granuloma



Interstitial granuloma



Crescentic glomerulonephritis



Glomerulosclerosis

 44. An old man from a village Ram Khilavan has progressively


increasing back pain for last 6 months. He also had repeated bouts
respiratory tract infections with Streptococcus pneumonia within
past 10 months. He ignored these thinking they are associated
with

Analgesic nephropathy



Goodpasture syndrome



Diabetes mellitus



Multiple myeloma

 45. Kidney biopsy from a child with hemolytic uremic syndrome


characteristically most likely presents features of:

Thrombotic microangiopathy



Proliferative glomerulonephritis



Focal segmental glomerulosclerosis



Minimal change disease

 46. Pathological changes of diabetic nephropathy are all except:



Fibrin caps and capsular drops



Kimmelstein-Wilson lesion



Basement membrane thickening



Focal glomerular sclerosis

 47. Serum C3 is persistently low in the following except:



Post streptococcal glomerulonephritis



Membranoproliferative glomerulonephritis



Lupus nephritis



Glomerulonephritis related to bacterial endocarditis

 48. Crescent formation is characteristic of which of the following


glomerular disease:

Minimal change disease



Rapidly progressive glomerulonephritis



Focal and segmental glomerulosclerosis



Rapidly non progressive glomerulonephritis

 49. A 6 year old child Devanand presented to the skin OPD with
some honey-colored crusts on his face. Dr Priyanka sends a
microbiological culture which comes out to be positive for group A
b hemolytic streptococci. Antibiotics are initiated in the child.
Howe

Development of rheumatic heart disease



Chronic renal failure



Complete recovery without treatment



Progression to crescentic glomerulonephritis

 50. Most common cause of nephritic syndrome in adults is:



Rapidly progressive glomerulonephritis



Focal segmental glomerulosclerosis


Membranous glomerulonephritis



Minimal change disease

Pathoanatomy - Heart failure, rheumatic heart disease, endocarditis

Total questions: 37; Correctly answered: 13; Percentage of


correct: 35.1% Skipped question: 0;
correct answer
incorrect answer
skipped question

 1. Chronic constrictive pericarditis is most commonly caused by



Staphylococcus



TB



Viral



Autoimmune

 2. Mc Callum s patch is diagnostic of



Infective endocarditis



Rheumatic endocarditis



Myocardial infarction



Tetralogy of Fallot (ToF)

 3. Calcification of aortic valve is seen in



Hurler



Marfan



Syphilis



None

 4. Most common heart valve involved in IV drug user is



Mitral valve



Aortic valve



Pulmonary valve



Tricuspid valve

 5. Pathognomic feature of acute rheumatic fever is



Pericarditis



Myocarditis



Mitral stenosis



Aschoff’s nodule

 6. Mitral valve vegetations do not embolise usually to



Brain



Liver



Spleen



Lung

 7. A 10-year-old boy, Pappu, died of acute rheumatic fever. All the


following can be expected at autopsy except

Aschoff



Rupture of Chordae tendinae



McCallum patch



Fibrinous pericarditis

 8. Tigered effect in myocardium is due to



Malignant change



Fat deposition



Seen in rheumatic fever



Associated with myocarditis

 9. Heart failure cells are seen in which of the following organs?



Kidney



Heart



Lungs



Brain

 10. Vegetations on under surface of cusps are found in



Infective endocarditis



Libman-Sacks endocarditis



SABE



Rheumatic fever

 11. Which of the following is not a complication of infective


endocarditis?

Myocardial ring abscess



Suppurative pericarditis



Myocardial infarction


Focal and diffuse glomerulonephritis

 12. Most common cause of mitral stenosis is



Rheumatic heart disease



Infective-endocarditis



Diabetes mellitus



Congenital

 13. Aschoff s nodules are seen in



Acute rheumatic fever



Bacterial endocarditis



Pneumoconiosis



Asbestosis

 14. Infective endocarditis is known to be caused by different


bacterial species. Which of the following scenarios is most
consistent with infective endocarditis caused by Staphylococcus
aureus?

A 34-year-old female with known mitral stenosis develops low-grade
fever and negative blood culture



A 28-year-old male with persistently high fever with tricuspid
vegetations and tricuspid regurgitation on trans-thoracic
echocardiogram



A 62-year-old female has persistent fever after being diagnosed with
colon cancer



A 64-year-old male with fever and malaise has repeatedly negative
blood culture and small mitral vegetation on trans-esophageal
echocardiogram

 15. The mechanism of the development of Acute Rheumatic Fever


is which of the following?

Cross reactivity with exogenous antigen



Innocent bystander effect



Due to toxin secretion by streptococci



Release of pyrogenic cytokines

 16. Which of the following cardiac valves is not commonly


involved in rheumatic fever?

Mitral



Aortic



Pulmonary



Tricuspid

 17. ASLO titers are used in the diagnosis of



Acute rheumatoid arthritis



Acute rheumatic fever



Ankylosing spondylitis



Osteoarthritis

 18. Libman-Sacks endocarditis is found in



Rheumatoid arthritis



SLE



Syphilis



Lymphoma

 19. Anitschkow cells are pathognomonic for



Acute rheumatic fever



Yellow fever



Malarial spleen



ITP

 20. NOT true about ASO titer



May be positive in normal people



Major Jones’ criteria



May be negative in post streptococcal glomerulonephritis



May not be elevated even in presence of Carditis

 21. Major criteria for rheumatic fever, consists of all except



Pancarditis



Arthritis



Subcutaneous nodule



Erythema nodosum

 22. Aschoff s nodules are seen in



Subacute bacterial endocarditis



Libman-Sacks endocarditis



Rheumatic carditis



Non-bacterial thrombotic endocarditis

 23. Rheumatic heart disease can be diagnosed on the basis of



Aschoff bodies



Vegetations along the lines of closure of valves



Endocardial involvement only



Follows skin and throat infection

 24. Most friable vegetation is seen in



Infective endocarditis



Libman Sacks endocarditis



Rheumatic heart disease



Rheumatoid heart disease

 25. Rheumatoid factor is



lgM directed against lgG



lgE directed against lgM



lgG directed against lgM



None

 26. Cardiac involvement in carcinoid syndrome is characterized by



Calcification tricuspid valve



Intimal fibrosis of right ventricle, tricuspid and pulmonary valve.



Involvement of the major blood vessels is commonly seen



Equal involvement of both the sides of the heart

 27. Which of the following is the feature of vegetations in Libmann


Sacks endocarditis?

Large and fragile



Small warty along the line of closure of valve



Small or medium sized on either or both sides of valve



Small bland vegetations

 28. An 8 year old girl, Guniya had sore throat following which she
developed severe joint pains. She has been diagnosed with acute
rheumatic fever. Instead of recovering as expected, her condition
worsened, and she died. Which of the following is the most lik

Central nervous system involvement



Endocarditis



Myocarditis



Streptococcal sepsis

 29. Disarrangement of myofibrils is found in



Dilated cardiomyopathy



Constrictive cardiomyopathy



Fibroelastic cardiomyopathy



Hypertrophic cardiomyopathy

 30. Aschoff bodies in Rheumatic heart disease show all of the


following features, except

Anitschkow cells



Epithelioid cells



Giant cells



Fibrinoid necrosis

 31. Most common cause of left sided cardiac failure is



Myocardial infarction



Systemic hypertension


Rheumatic heart disease



Infective endocarditis

 32. Which one of the following is not included as major criteria in


Jones criteria?

Pancarditis



Arthritis



Subcutaneous nodules



Elevated ESR

 33. Which type of endocarditis has vegetation on both sides of the


valves ?

Infective endocarditis



Libman Sack



Rheumatic fever



Non bacterial thrombotic enodcarditis

 34. Diagnostic feature of rheumatic fever is



Antischkow cells


Aschoff



MacCallum



Epithelioid cells

 35. All are the causes of myocarditis except



Trichinosis



Mycobacterium tuberculosis



Corynebacterium diphtheriae



Systemic lupus erythematosus

 36. Aschoff s bodies are seen in



Acute rheumatic fever



SLE



SABE



TB

 37. In mitral valve prolapse syndrome, histopathology of mitral


valve shows

Hyaline degeneration



Elastic degeneration



Myxomatous degeneration



Fibrinoid necrosis

Pathoanatomy - Hemodynamics and hemostasis

Total questions: 19; Correctly answered: 4; Percentage of


correct: 21.1% Skipped question: 0;
correct answer
incorrect answer
skipped question

 1. Coagulation defects associated with increased coagulation are


seen in:

Increased Protein C



Increased Protein S



Increased Anti-thrombin III



Protein C resistance

 2. Platelet adhesion to collagen is mediated by which of the


following?

Factor VIII



Factor IX


Von willebrand factor



Fibronectin

 3. Extrinsic pathway of clotting factors is measured by?



Prothromin time



Activated partial thromboplastin time



Bleeding time



Clotting time

 4. Tissue thromboplastin activates:



Factor VII



Factor IV



Factor VI



Factor XII

 5. Gandy gamma body is typically seen in chronic venous


congestion of which of the following?

Lung



Kidney



Spleen



Liver

 6. All are true about blood coagulation except?



Factor X is a part of both intrinsic and extrinsic pathway.



Extrinsic pathway is activated by contact of plasma with negatively
charged surfaces



Calcium is very important for coagulation



Intrinsic pathway can be activated in vitro.

 7. Edema in nephrotic syndrome occurs due to



Na+ and water restriction



Increased venous pressure



Decreased serum albumin



Decreased fibrinogen

 8. Thrombomodulin thrombin complex prevents clotting because



Thrombomodulin inhibits prothrombin activator


The complex activates antithrombin III



Thrombomodulin-thrombin complex activates heparin



The complex removes thrombin and also activates protein C which
inactivates the activated factors V and VIII

 9. Vitamin K is responsible for the carboxylation of which amino


acid in the clotting factors?

Aspartate



Glutamate



Proline



Lysine

 10. Vitamin K associated clotting factors are:



IX, X



I, V



VII, VIII



I, VIII


 11. Which of the following is a procoagulation protein?

Thrombomodulin



Protein C



Protein S



Thrombin

 12. Which is not involved in local hemostasis?



Fibrinogen



Calcium



Vitamin K



Collagen

 13. Cause of edema is



Decreased plasma protein concentration



Increased lymph flow



Increased ECF volume



Increased plasma protein concentration


 14. All endothelial cells produce thrombomodulin except those
found in:

Hepatic circulation



Cutaneous circulation



Cerebral microcirculation



Renal circulation

 15. A 54 year old chronic alcoholic Adhiya Kumar is brought by his


son as he has developed progressively increasing abdominal
distension from past 3 months. The physician aspirates the
abdominal fluid which is straw-colored and clear and is found to
have prot

Blockage of lymphatics



Decreased oncotic pressure



Decreased capillary permeability



Inflammatory exudation

 16. Which is the following not synthesized in the liver?



Factor II



Factor VII



Factor IX


Factor VIII

 17. Endothelium derived relaxing factor (EDRF) is associated with:



Ras



C-myc



Bcl



N NOS

 18. All of the following are correct about Thromboxane A2 except



Low dose aspirin inhibits its synthesis



Causes vasoconstriction in blood vessels



Causes bronchoconstriction



Secreted by WBC

 19. All of the following are anticoagulant substances except



Antithrombin III



Protein S



Von Willebrand factor


Nitric oxide


Pathoanatomy - Hemoglobinopathies: sickle cell anemia, thalassemia

Total questions: 31; Correctly answered: 5; Percentage of


correct: 16.1% Skipped question: 0;
correct answer
incorrect answer
skipped question

 1. In sickle cell anemia defect is in which chain:



Alpha chain



Beta chain



Both the chains



None of these

 2. A 23-year-old male Subhas Raj presents to the medicine OPD


with complaint of hematuria. There is no burning sensation during
urination. He also has no history suggestive of respiratory tract
infection. He gives a history of two of his relatives suffering

Irreversible sickle cells are present on the peripheral smear



Reticulocyte count is elevate(d)



He is protected from Plasmodium falciparum



MCHC is decrease(d)


 3. All of the following aggravate sickling phenomenon in sickle cell
disease EXCEPT:

Higher concentration of HbS



Higher concentration of HbF



Lower concentration of HbC



A fall in blood pH

 4. Mutation causing sickle cell anemia is protective for which of


the following?

Malaria



Filaria



Leishmania



None of the above

 5. A 18 year old Afro American boy presenting with a non healing


ulcer of the foot, with recurrent pneumonia and chronic hemolytic
anemia. The peripheral blood erythrocytes showed some peculiar
appearance. Most likely cause is:

Trinucleotide repeat



Genomic imprinting



Single amino acid base substitution


Antibody to red cell membrane

 6. Molecular pathogenesis of a thalassemia involves



Mutation in transcription sequence



Gene deletion



Codon termination mutation



MRNA splicing defect

 7. The primary defect which leads to sickle cell anemia is:



An abnormality in porphyrin part of hemoglobin



Replacement of glutamate by valine in b-chain of HbA



A nonsense mutation in the b-chain of HbA



Substitution of valine by glutamate in the a-chain of HbA

 8. Which of the following manifestations is common to sickle cell


anemia and thalassemia major?

Autosplenectomy



Bone marrow expansion in the calvarium



Ineffective erythropoiesis



Predisposition to Hemophilus influenzae infections

 9. Sickle cell anemia is due to:



Presence of a structurally abnormal Hb



Red cell enzyme deficiency



Unknown multiple mechanisms



Disturbance of proliferation and differentiation of stem cells

 10. Which of the following is the cause of alpha thalassemia?



Deletion of alpha genes



Deletion of beta genes



Excess of alpha genes



Single amino acid substitution in alpha chain

 11. The primary defect which lead to stickle cell anemia is ?



An abnormality in porphyrin part of haemoglobin



Substitution of valine by glutamate in the -chain of HbA


Replacement of glutamate by valine in -chain of HbA



A nonsense mutation in the -chain of HbA

 12. Bone infarcts are seen in:



Iron deficiency anemia



Thalassemia



Sickle cell anemia



Hereditary spherocytosis

 13. NESTROF test is a screening test for which of the following


conditions?

P-Thalassemia



Hereditary spherocytosis



Autoimmune haemolytic anaemia



Megaloblastic anaemia

 14. Sickle cell trait patient do not have manifest-tations of sickle


cell disease, because:

50% HbS is required for occurrence of sickling


HbA prevents sickling



50% sickles



HbA prevents polymerization of HbS

 15. All are seen in Thalasemia major except-



Transfusion dependency



Splenoheptatomegaly



Ineffective erythropoiesis



Macrocytic anaemia

 16. True about Sickle cell anemia are all except:



Commonly seen in blacks



R.(b)(c) size is altered



Valine is substituted for glutamic acid in beta chain of globin



Deletion of gene

 17. What is affected in HbS (Hemoglobin S)?



Stability


Function



Affinity



Solubility

 18. HbH is formed due to which of the following?



Deletion of 4 alpha chains



Deletion of 3 alpha chains



Deletion of 2 alpha chains



Deletion of 1 alpha chain

 19. In a-thalassemia, which of the following is a finding?



No -chain



Excess a-chain



No a-chain



Relative excess of , y, and 6 chains

 20. Person having heterozygous sickle cell trait are protected from
infection of?

P. falciparum



P. vivax



Pneumococcous



Salmonella

 21. A couple, with a family history of beta thalassemia major in a


distant relative, has come for counseling. The husband has HbA2
of 4.8% and the wife has HbA2 of 2.3%. The risk of having a child
with beta thalassemia major is:

50%



25%



5%



0%

 22. Which one of the following statements about hemoglobin S


(HbS) is not true:

Hemoglobin HbS differs from hemoglobin HbA by the substitution of Val
for Glu in position 6 of the beta chain



One altered peptide of HbS migrates faster towards the cathode (–)
than the corresponding peptide of HbA



Binding of HbS to the deoxygenated HbA can extend the polymer and
cause sickling of the red blood cells


Lowering the concentration of deoxygenated HbS can prevent sickling

 23. Hemoglobin H disease is caused by deletion of:



A single alpha globin gene



Two alpha globin genes



Three alpha globin genes



All four alpha globin genes

 24. A 14-year-old male Kaalu is brought to the emergency room


with high grade fever, chest pain, and dyspnea. His past medical
history is significant for two prior hospitalizations for abdominal
pain, which resolved with analgesics and hydration. Evaluation t

Work hypertrophy



Follicular hyperplasia



Vascular occlusion



Pressure atrophy

 25. Sickle cell anemia is the clinical manifestation of homozygous


genes for an abnormal haemoglobin molecule. The event
responsible for the mutation in the 1 chain is

Insertion



Deletion



Non- disjunction



Point mutation

 26. True about beta-thalassemia trait is:



Increased HbF



Increased HbA2



Microcytosis



Severe anemia

 27. A 6yrs old child belonging to Punjabi family with past history of
blood transfusions presented with hemoglobin 3.5 g/dl, MCV - 30 fl,
MCHC - 20. Peripheral smear findings of microcytic hypochromic
anemia with target cell and reduced osmotic fragility. Wha

Alpha thalassemia



Beta thalassemia



Sickle cell anemia



G6PD deficiency


 28. Ideally children with thalassemia should be transfused with

Packed RBC



Platelet rich plasma



Saline washed packed RBC



Whole blood

 29. Sickle cell red blood cells have:



Altered stability



Altered functions



Decreased oxygen carrying capacity



Protective action against adult malaria

 30. In sickle cell disease, the defect is in:



Alpha-chain



1-chain



Gamma-chain



Hb formation


 31. One of the common variants of sickle cell anemia frequently
marked by lesser degree of haemolytic anemia and greater
propensity for the development of retinopathy and aseptic
necrosis of bones is

Sickle cell trait



Haemoglobin SC disease



Sickle thalassaemia



Sickle –Hb E disease

Pathoanatomy - Hemolytic anemia: pnh, hs, g6pd, immune hemolytic


anemia

Total questions: 40; Correctly answered: 7; Percentage of


correct: 17.5% Skipped question: 0;
correct answer
incorrect answer
skipped question

 1. G6PD help in maintaining the integrity of RBC by?



Controlling reduction stress on RBC



Controlling oxidative stress on RBC



Maintaining flexibility of cell membrane



Component of electron transport chain

 2. Cause of fragmented RBC in peripheral blood:



Microangiopathic hemolytic anemia


DIC



Hemophilia-A



Malignant hypertension

 3. The peripheral smear of hereditary spherocytosis will show


spherocytes:

Usually of same size



Reticulocytosis seen



Smaller size



Anemia is negligible

 4. Microangiopathic hemolytic anemia seen in:



Thrombotic thrombocytopenic purpura



Hemolytic uremic syndrome



Henoch-Schonlein purpura



DIC


 5. A 30-year-old woman A. Jolie with SLE and chronic renal failure
manifests rapidly progressive weakness. She appears pale and
has slightly yellow sclerae and an enlarged spleen. Blood tests
reveal severe anemia and mild, mostly unconjugated,
hyperbilirubin

Bone marrow aplasia



IgG directed against red blood cells



IgM directed against red blood cells



Spleen sequestration

 6. A 5 year old male child presents with episodic anemia and


jaundice since birth. He is least likely to have which of the
following?

Hereditary spherocytosis



Sickle cell disease



G6PD deficiency



Paroxysmal nocturnal hemoglobinuria

 7. An infant presents with mild anemia, jaundice, and


splenomegaly. A complete blood count with differential reveals
spherocytosis; with elevated reticulocyte count. The parents state
that several relatives have also suffered from a similar illness. The
infa

Clathrin



Connexon



Ankyrin



Spectrin

 8. Microangiopathic hemolytic anemia is seen in all of the


following diseases except

Antiphospholipid antibody syndrome



Macroangiopathic



Thrombotic thrombocytopenic purpura



Metallic cardiac valves

 9. All are the features of hemolytic anemia except:



Hemoglobinuria



Jaundice



Increased haptoglobin



Hemosiderinuria

 10. A young male photographer Alok Nath from Bangalore is


evaluated for recurrent episodes of jaundice. He has no history of
recent travel. Past medical history is insignificant and the person
is not on any medication. He does admit to unprotected sex with
mu

Cirrhosis and hepatocellular carcinoma



Pigmented gallstones



Avascular necrosis of the femur



Episodic venous thrombosis

 11. The following protein defects can cause hereditary


spherocytosis except:

Ankyrin



Palladin



Glycophorin C



Anion transport protein

 12. Thrombotic event is seen in all of following except:



Paroxysmal nocturnal hemogloninuria



Disseminated intravascular coagulation



Idiopathic thrombocytopenic purpura



Heparin induced thrombocytopenia

 13. Spherocytosis in blood smear is seen in:



Hemoglobin C



Mechanical trauma



Hereditary spherocytosis



Hereditary elliptosis

 14. Autoimmune hemolytic anemia is seen in:



ALL



AML



CLL



CML

 15. Schistocyte is/are found in:



TTP



March hemoglobinuria



Severe iron deficiency


All of the above

 16. Intravascular hemolysis occurs in:



Hereditary spherocytosis



Autoimmune haemolytic anemia



Paroxysmal nocturnal hemoglobinuria



Thalassemia

 17. Bite cells are seen in:



G6PD deficiency



Sickle cell anemia



Hereditary spherocytosis



Trauma

 18. In DIC, following are seen except



Fibrinogen decreased



Thrombocytopenia



Normal APTT


PT elevation

 19. A 20-year-old man John Abraham is transported to the


emergency department within 20 minutes of sustaining a road
traffic accident on his bike. The patient is poorly responsive. If it is
presumed that he may have lost about 1.5 liters of blood at the
scen

Increased mean corpuscular hemoglobin concentration



Decreased hematocrit



Decreased blood pressure



Decreased red blood cell count

 20. Donath-Landsteiner antibodies are seen in:



Warm agglutination



Cold agglutination



Paroxysmal nocturnal hemoglobinuria



ITP

 21. Rahul Singh migrates from UP to Bihar to work under the


NREGA scheme as a daily wage labourer. The district has high
endemicity of malaria. So, his physician Dr. Dubey gives him
primaquine chemoprophylaxis for Plasmodium vivax malaria.
Several days after

Duffy antigen



Glucose-6-phospate dehydrogenase



Intrinsic factor



PIG-A

 22. An Rh -ve woman became pregnant with Rh +ve fetus. Within


few days after birth, the infant developed jaundice, ascites,
hepatomegaly and edema. The likely substance(s) deposited in
skin and sclera in jaundice is/ are given below. Which is the best
possibl

Biliverdin



Conjugated and unconjugated bilirubin



Unconjugated bilirubin



Conjugated bilirubin

 23. A 23-year-old female presented with jaundice and pallor for 2


months. Her peripheral blood smear shows the presence of
spherocytes. The most relevant investigation to arrive at a
diagnosis is which of the following?

Tests for PNH



Osmotic fragility test



Coombs test


Reticulocyte count

 24. Helmet cells are characteristic of anemia of:



Hemolytic uremic syndrome



Polysplenia



Spherocytosis



Acanthocytosis

 25. A 38-year-old male Kritesh presents to his physician with the


complaints of sudden onset of fever, chills and dysuria. He has no
significant past medical history. A per-rectal examination reveals
tenderness. The prostatic secretions are collected and on m

Anti body-mediated erythrocyte destruction



Hereditary erythrocyte membrane defect



Hereditary erythrocyte enzyme deficiency



Microangiopathic hemolytic anemia

 26. Intravascular hemolysis is seen in



Glucose-6 phosphate dehydrogenase deficiency



Thalassemia


Sickle cell anemia



Hemophilia

 27. D.I.C is seen in:



Acute promyelocytic leukemia



Acute myelomonocytic leukemia



CML



Autoimmune haemolytic anemia

 28. Hereditary spherocytosis is due to:



Acquired membrane defect



Ankyrin deficiency



Defective hemoglobin synthesis



Mechanical trauma to red cells

 29. A 16-year-old female Gitika notices that her urine becomes red
after she is given sulfonamides for treatment of a urinary tract
infection. Both urine and serum test positive for free hemoglobin,
and the urine red cell count is 1.2 million/mm3. A periphera

Alpha-chain of hemoglobin


Beta-chain of hemoglobin



Glucose-6-phosphate dehydrogenase



Glycoprotein IIb/IIIa

 30. Hot agglutinin is found in all except:



Mycoplasma infection



SLE



Methyl dopa



Rheumatoid arthritis

 31. PNH due to defect in:



CD 59



CD 15



CD 100



CD 20

 32. Not seen in paroxysmal nocturnal hemoglobinuria is:



LDH levels are raised


LDH levels are raised



Decreased leukocyte alkaline phosphatase



Increased platelets

 33. Cold agglutinin is



IgG



IgM



IgA



IgD

 34. In hereditary spherocytosis an inherited abnormality is seen in


which of the following red blood cell component:

A-globin chain



B-globin chain



Phosphatidyl inositol glycan A



Spectrin

 35. Features seen in hemolytic anemia are all except:



Teardrop and Burr cells



? Haptoglobin



Reticulocytosis



Hemoglobinuria

 36. PNH associated with somatic mutation affecting:



Decay accelerating factor (DAF)



Membrane inhibitor of reactive lysis (MIRL



Glycosylphosphatidylinositol (GPI)



C8 binding protein

 37. An abnormal Ham test is most likely associated with which of


the following?

Spectrin



Defect in complement activating proteins



Defective GPI anchor



Mannose-binding residue effect


 38. Intravascular hemolysis occurs in:

Hereditary spherocytosis



Acute G6PD deficiency



Sickle cell disease



Thalassemia

 39. Microangiopathic hemolytic anemia is seen in:



HUS



ITP



Malignant hypertension



Prosthetic valves

 40. Cold hemagglutinin is associated with:



Anti IgM



Anti IgG



Anti IgA



Donath-Landsteiner antibody


Pathoanatomy - Hepatic tumours

Total questions: 8; Correctly answered: 3; Percentage of


correct: 37.5% Skipped question: 0;
correct answer
incorrect answer
skipped question

 1. Thorium induced tumor:



Angiosarcoma of liver



Renal cell carcinoma



Lymphoma



Astrocytoma

 2. Most common primary malignant tumour of liver in adult is



Squamous cell carcinoma



Hepatoblastoma



Hepatocellular carcinoma



Hepatoma

 3. Commonest benign tumor of liver is:



Hamartoma



Hemangioma


Adenoma



Nodular focal hyperplasia

 4. True about Fibrolamellar carcinoma of Liver is all, except:



Females do not increased incidence than males



Has good prognosis



Not associated with liver cirrhosis



Serum AFP levels are usually > 1000 mg/litre

 5. A young woman Ms Shaano who is otherwise normal goes for


an annual examination in a nursing home Her blood investigations
reveal hemoglobin is 15 gm/ dl, TLC is 7,000/mm3, ESR is 12mm/hr
Her kidney and liver function tests are also normal She undergoes

Polycythemia vera



Hepatitis B



Oral contraceptives



Polyvinyl chloride

 6. Which of the following is not correct about fibrolamellar variant


of hepatocellular carcinoma?

Occurs in young males and females



Hepatitis B virus is an important risk factor



Often has a better prognosis



Is a hard scirrhous tumor

 7. Which of the following most significantly increases the risk of


hepatocellular cancer?

Hep A



Hep B



CMV



EBV

 8. Which is not correct about hepatocellular carcinoma?



More in females



Rise of AFP noted



Has stronger propensity to invade vascular channels



CPathoanatomy - Hepatitis
Total questions: 22; Correctly answered: 5; Percentage of
correct: 22.7% Skipped question: 0;
correct answer
incorrect answer
skipped question

 1. A 30 year old woman Aishwarya goes to her gynecologist Dr


Harmeet for a pre-pregnancy examination Routine prenatal
laboratory testing demonstrates normal hematological profile with
controlled sugar as well negative TORCH infections She normal
liver fun

Hepatitis B carrier



Recently infected with hepatitis B



Immunized against hepatitis B



Infected with hepatitis B and highly transmissible

 2. Piece meal necrosis is pathognomic of



Alcoholic Liver disease



Chronic active hepatitis



Toxic hepatitis



Wilson disease

 3. In Chronic Viral Hepatitis



Hepatitis A virus infection is a common cause in children



Morphological classification into Chronic Active Hepatitis and Chronic
Persistent Hepatitis are important



Fatty change is pathognomic of Hepatitis C virus infection



Grading refers to the extent of necrosis and inflammation

 4. An eminent hepatobiliary expert Dr Sarin conducts a study in


hepatitis B patients for which the patients are followed for almost a
decade Their detailed history regarding the mode of transmission
of HBV is taken A battery of tests including periodic

Blood transfusion



Heterosexual transmission



Vertical transmission during childbirth



Needle stick injury

 5. Councilman bodies are seen in:



Wilson disease



Alcoholic hepatitis



Acute viral hepatitis



Auto immune hepatitis


 6. Piece meal necrosis is seen in

Alcoholic hepatitis



Toxic hepatitis



Chronic active hepatitis



Malignancy

 7. In pregnancy, which viral infection has maximum mortality?



Hepatitis A Virus



Hepatitis B Virus



Hepatitis C Virus



Hepatitis E Virus

 8. Most common Pathological change seen in acute viral hepatitis


is

Ballooning degeneration



Neutrophilic infiltration



Piece meal necrosis



Periportal fatty change

 9. Hepatitis B virus is not associated with



Fulminant hepatitis



Chronic active hepatitis



Hepatocellular carcinoma



Cholangiocarcinoma

 10. The liver biopsy in acute hepatitis due to hepatitis B virus is


likely to show all of the following, except

Ballooning change of hepatocytes



Ground glass hepatocytes



Focal or spotty necrosis



Acidophil bodies

 11. Steatosis is NOT seen in



Hepatitis-B infection



Hepatitis-C infection



Alcoholic person



Protein malnutrition

 12. A 20 yr old man with HBs Ag +ve, HbeAg



Wild type HBV



Surface mutant HBV



PreCore mutant HBV



Inactive HBV carrier

 13. Incubation period of hepatitis B is



6 weeks to 6 months



6 days to 6 weeks



6 months to 6 year



More than 6 years

 14. In a pioneering clinical study at Spartans Institute, patients


having infection with infectious hepatitis, (as in hepatitis A, B, C, D,
E, F, and G) are being followed for 40 months During that time the
subjects are required to undergo regular investigat

Presence of inflammatory cell in the sinusoids on a liver biopsy
specimen



Degree to which hepatic transaminase enzymes are elevated


Length of time that hepatic enzymes remain elevated



Specific form of hepatitis virus responsible of the infection

 15. After passing his physical exam, a young army recruit gives
urine and blood samples for further testing Serum analysis yields
elevated ALT, HBsAg, Anti-HBc, HBeAg, and bilirubin All other
values are normal Which of the following is the hepatitis B st

Asymptomatic carrier



Chronic active carrier



Fulminant hepatitis B



Recovered from acute self-limited HBV

 16. Indicator of active multiplication of hepatitis B virus is:



HBs Ag



HBc Ag



Hbe Ag



Anti-HBs Ab

 17. All are correctly matched except:



Hepatitis B - Ground glass hepatocytes


Reye's syndrome-ground glass appearance of hepatocytes



Alcohol - Mallory bodies



Wilson disease - Mallory bodies

 18. A 34-year-old man Bholu presents to his physician with loss of


appetite, nausea and vomiting, and fatigue Laboratory
examination confirms the diagnosis of hepatitis B, and the man
becomes icteric 2 weeks later This patient may also be particularly
vulne

Berry aneurysm



Coronary artery aneurysm



Polyarteritis nodosa



Giant cell arteritis

 19. Hepatitis E is transmitted by



Blood



Feco-oral



Venereal



All of the above

 20. Which one of the following diseases characteristically causes


fatty change in liver?

Hepatitis B virus infection



Wilson



Hepatitis C infection



Chronic alcoholism

 21. Chronic carrier stage is not found in



Hepatitis B Virus



Hepatitis C Virus



Both a and b



Hepatitis A Virus

 22. Centrilobular necrosis of liver occurs in



Phosphorus



Phenol



Arsenic



Mercury

hronic HBV has high rate of hepatocellular carcinoma Pathoanatomy -


Hodgkin lymphoma

Total questions: 16; Correctly answered: 5; Percentage of


correct: 31.3% Skipped question: 0;
correct answer
incorrect answer
skipped question

 1. Lacunar cells are seen in which type of Hodgkin's disease



Nodular sclerosis



Mixed cellularity



Lymphocyte depletion



Lymphocyte predominant

 2. Popcorn cells' are seen in which type of Hodgkin's disease?



Lymphocyte dominant



Lymphocyte depleted



Nodular sclerosis



Mixed type

 3. The subtype of Hodgkin’s disease



Lymphocyte predominant



Nodular sclerosis



Mixed cellularity



Lymphocyte depleted

 4. Lacunar cells are present in which Hodgkin's disease



Nodular sclerosis



Lymphocyte predominant



Mixed cellularity



All

 5. A 35-year-old man is diagnosed with non-Hodgkin lymphoma.


Even without knowing the specific diagnosis, it can safely be said
that this patient

B-lymphocytic origin



Histiocytic origin



Lymph node localization



Monoclonal origin

 6. A 26-year-old man Ronit has progressive, painless enlargement


of neck lymph nodes. Routine chest film and CT scan show
marked enlargement of mediastinal nodes. No nodules are seen in
the liver or lungs. When evaluating the biopsy of one of the
involved

Abnormal plasma cells



Giant platelets



Immature neutrophil precursors



Reed-Sternberg cells

 7. The lymphocytic and histiocytic variant of ReedSternberg cell is


seen in

Follicular center lymphoma



Lymphocyte depleted Hodgkin



Nodular sclerosis Hodgkin



Lymphocyte predominant Hodgkin

 8. Lacunar cells are seen in which type of Hodgkin's disease



Lymphocyte predominance



Lyphocyte depletion



Nodular sclerosis



Mixed cellularity

 9. All of the following are the good prognostic features for


Hodgkin's disease

Hemoglobin > 10 gm/dl



WBC count < 15000/mm3



Absolute lymphocyte count < 600/µl



Age < 45 years

 10. Which cell is not seen in Hodgkins lymphoma



Reed Sternberg cell



Lacunar cell



L and H cell



Langerhans' cell

 11. An elderly patient presented with hypercellular bone marrow,


peripheral blood smear shows pancytopenia, and 15% myeloblast
cells. Most likely diagnosis is

Myelodysplastic syndrome


Blast crisis in CML



AML



Polycythemia vera

 12. Lacunar type of Reed Sternberg cells are seen in:



Mixed cellularity Hodgkin



Nodular sclerosis Hodgkin



Lymphocyte depleted Hodgkin



Lymphocyte predominant Hodgkin

 13. Which Hodgkin's disease



Lymphocyte depletion



Mixed cellularity



Lymphocytic predominance



Nodular sclerosis

 14. In a 45 year old female presenting with painless


supraclavicular lymphadenopathy, biopsy was taken. It revealed
the presence of binucleated acidophlic owl eye appearance with
clear vacuolated space. The cell was CD15 and CD30 positive.
Which is the most l

Lymphocyte predominant Hodgkin lymphoma



Nodular sclerosis Hodgkin lymphoma



Mixed cellularity Hodgkin lymphoma



Lymphocyte depleted Hodgkin lymphoma

 15. Classical markers for Hodgkin's disease



CD 15 and CD 30



CD 15 and CD 22



CD 15 and CD 20



CD 20 and CD 30

 16. The sub-type of Hodgkin's disease



Nodular sclerosis



Mixed cellularity



Lymphocyte depletion



Lymphocyte predominant


Pathoanatomy - Htn, atherosclerosis, aneurysm

Total questions: 33; Correctly answered: 6; Percentage of


correct: 18.2% Skipped question: 0;
correct answer
incorrect answer
skipped question

 1. Autopsy of a 14 year-old male who died in a motor vehicle


accident shows many yellow spots on the inner surface of the
aorta. Which of the following best describes the lesions?

They are a very uncommon finding for a patient of this age.



Their distribution closely mimics that of future atherosclerosis.



Once formed, they inevitably progress to atheromas



They show predominantly intracellular lipid accumulation

 2. Which of the following is the commonest histological finding in


benign hypertension?

Proliferative end arteritits



Necrotizing arteriolitis



Hyaline arteriosclerosis



Cystic medial necrosis

 3. Cystic medical necrosis is seen in:



Marfan syndrome



Friedrichs ataxia



Down syndrome



Kawasaki disease

 4. Most common site of atherosclerotic aneurysm is:



Coronary artery



Renal artery



Arch of aorta



Abdominal aorta

 5. Commonest histological finding in benign hypertension is:



Proliferating endarteritis



Necrotising arteriolitis



Hyaline arteriosclerosis



Cystic medial necrosis


 6. Monckeberg’s calcific sclerosis affects the medium sized
muscular arteries by involving the structure of:

Intima



Media



Adventitia



All of the above

 7. Recurrent ischemic events following thrombolysis has been


pathophysiologically linked to which of the following factors:

Antibodies to thrombolytic agents



Fibrinopeptide A



Lipoprotein (A)



Triglycerides

 8. A 52 year old male, Puneet, presents to the emergency of a


tertiary care hospital with chest pain and syncope. The physician
suspects a coronary pathology. Coronary arteriography
demonstrates significant atherosclerotic involvement of the left
anterior de

Neutrophils



Eosinophils



Platelets



Erythrocytes

 9. In atherosclerosis, increased LDL in monocyte macrophage is


due to:

LDL receptors on macrophage



LDL receptors on endothelium



Lipids in LDL get auto-oxidized



All of the above

 10. All are seen in malignant hypertension, except:



Fibrinoid necrosis



Hyaline arteriosclerosis



Necrotizing glomerulonephritis



Hyperplastic arteriosclerosis

 11. In 2 patients with atherosclerosis, one is diabetic and the other


is non diabetic. In relation to the non diabetic, the diabetic patient
has 100 times risk of which of the following?

MI



Stroke



Lower limb ischemia



Vertebrobasilar insufficiency

 12. In malignant hypertension hyperplastic arteriosclerosis is seen


in all except:

Heart



Kidney



Pericardial fat



Peripancreatic fat

 13. Malignant hypertension causes which of the following changes


in the kidney?

No change in kidney



Flea bitten kidney



Irregular granular contracted kidney



Large white kidney

 14. Most common cause of dissecting hematoma is because of:



Hypertension



Marfan syndrome



Iatrogenic



Kawasaki

 15. The presence of stroke, peripheral vascular disease and


atherosclerosis is associated with which hormone?

Insulin deficiency



Hyperestrogenemia



Hypothyroidism



Progesterone

 16. Medial calcification is seen in:



Atherosclerosis



Arteriolosclerosis



Monckeberg’s sclerosis



Dissecting aneurysm


 17. Features of essential hypertension:

Concentric hypertrophy of LV



Increased heart size



Increased size of the heart muscles



Myohypertrophy

 18. Most common cause of abdominal aortic aneurysm is:



Atherosclerosis



Syphilis



Trauma



Congenital

 19. Which of the following is the least common site of


atherosclerotic lesions?

Aortic bifurcation



Pulmonary arterial trunk



Common carotid artery



Middle cerebral artery

 20. Atheromatous changes of blood vessels affects early in ?



Kidney



Heart



Liver



Spleen

 21. Most common cause of aortic aneurysm is:



Syphilis



Marfan



Atherosclerosis



Congenital

 22. Onion skin thickening of arteriolar wall is seen in:



Atherosclerosis



Median calcific sclerosis



Hyaline arteriosclerosis



Hyperplastic arteriosclerosis

 23. ALL of the following statements regarding atherosclerosis are


true except:

Omega-3 fatty acid (abundant in fish oil) decrease LDL



Atherosclerosis in less important in age more than 45 years age



Cigarette smoking is independent risk factor for M.I



C reactive protein is independent risk factor for M.I

 24. 70-year-old man has abdominal pain with mass in abdomen.


Angiography reveals aneurysm of aorta. Most likely cause is:

Trauma



Atherosclerosis



Syphilis



Congenital

 25. Atherosclerosis is seen with which bacteria



Staph aureus



Streptococcus pneumonia



Chlamydia pneumoniae


Chlamydia trachomatis

 26. Ascending aorta involvement is the commonest site of which


aneurysm:

Syphilitic



Atherosclerotic



Berry aneurysm



Traumatic

 27. Visceral aneurysm is most commonly seen in:



Splenic



Renal



Hepatic



Coronary

 28. Accelerated phase of hypertension is characterized


microscopically by:

Fibrinoid necrosis of arteriolar wall



Hyaline arteriosclerosis


Elastosis of the intima



Marked calcification of the media

 29. CAD predisposing factors:



Homocysteinemia



Increased lipoprotein B



Increased fibrinogen



Increased HDL

 30. Hallmark feature of benign HTN is:



Hyaline arteriosclerosis



Cystic medial necrosis



Fibrinoid necrosis



Hyperplastic arteriosclerosis

 31. Vascular pathology of benign hypertension includes:



Segmental fibrinoid necrosis



Hyaline arteriosclerosis


Periarteritis



Loss of internal elastic lamina

 32. Most common site of artery of atherosclerosis:



Right coronary artery



Left anterior descending coronary artery



Left circumflex artery



Diagonal branch of LAD

 33. Renal biopsy in an old man Hitesh demonstrates concentric,


laminated thickening of arteriolar walls due to proliferation of
smooth muscle cells. This process is best described by which of
the following terms?

Atherosclerosis



Hyaline arteriolosclerosis



Hyperplastic arteriolosclerosis



MI

Pathoanatomy - Hypersensitivity reactions


Total questions: 21; Correctly answered: 5; Percentage of
correct: 23.8% Skipped question: 0;
correct answer
incorrect answer
skipped question

 1. Cell mediated immunity is:



Type I



Type II



Type III



Type IV

 2. Myasthenia gravis is most commonly associated with which of


the following?

Thymoma



Thymic carcinoma



Thymic hyperplasia



Lymphoma

 3. Raji cell assays are used to quantitate:



Complement levels



Immune complexes



T cells



Interferon levels

 4. Example of Type IV Hypersensitivity is/are:



Farmer



Contact hypersensitivity



Immediate hypersensitivity



Myasthenia gravis

 5. Granuloma in Sarcoidosis is called



Hard sore



Soft sore



Hard tubercle



Caseating granuloma

 6. Example of Type II Hypersensitivity is/are:



Blood transfusion reaction



Arthus reaction


Hay Fever



Post-streptococcal glomerulonephritis

 7. Hemolytic disease of newborn is an example of:



Type 3 hypersensitivity reaction



Type 2 hypersensitivity reaction



Arthus reaction



Type 4 hypersensitivity reaction

 8. Myasthenia gravis may be associated with



Thymoma



Systemic lupus erythematosus



Hyperthyroidism



All of the above

 9. A man after consuming sea food develops rashes. It is due to:



IgE mediated response



Complement activation


Cell mediated response



None of the above

 10. Which of following statements is not true about Mycobacterium


tuberculosis infection

M. tuberculosis leads to development of delayed hypersensitivity



Lymphocytes are the primary cells infected by M. tuberculosis



Positive tuberculin test signifies cell mediated hypersensitivity



Tuberculin test does not differentiate between infection and disease.

 11. What type of hypersensitivity reaction is seen in myasthenia


gravis?

Type 1 hypersensitivity reaction



Type 2 hypersensitivity reaction



Type 3 hypersensitivity reaction



Type 4 hypersensitivity reaction

 12. Antibody found in patients with myasthenia gravis is directed


against

Acetylcholine


Acetylcholine receptors



Acetylcholine vesicles in nerve terminal



Actin-myosin complex of the muscle

 13. The immunoglobulin involved in type I hypersensitivity


reaction is:

IgE



IgM



IgA



IgG

 14. A 40 year old man has chronic cough with fever for several
months. The chest radiograph reveals a diffuse reticulondular
pattern. Microscopically on transbronchial biopsy there are focal
areas of inflammation containing epitheloid cell granuloma,
Langhan

Type I



Type II



Type III



Type IV

 15. Arthus reaction is what type of hypersensitivity reaction:



Localized immune complex



Ag-Ab reaction



Complement mediated



Ab mediated

 16. Ram Devi presented with generalized edema sweating and


flushing tachycardia and fever after bee sting. This is:

T cell mediated cytotoxicity



IgE mediated reaction



IgG mediated reaction



IgA mediated hypersensitivity reaction

 17. Tuberculin test positivity indicates



Good humoral immunity



Infection with mycobacterium



Good cell mediated immunity



None

 18. Which of the following type of hypersensitivity reaction is


found in blood transfusion reaction?

Anaphylactic type



Cytotoxic type



Type III hypersensitivity



Cell mediated hypersensitivity

 19. Which of the following type of hypersensitivity reactions


occurs in Farmer's lung?

Type I



Type II



Type III



Type IV

 20. Which of the following diseases is/are mediated through


complement activation

Atopic dermatitis



Graft versus Host disease



Photoallergy



Necrotizing vasculitis

 21. Hypersensitivity pneumonitis is classically a/an:



Allergic reaction



Type II hypersensitivity



Immune complex mediated hypersensitivity



Cell mediated hypersensitivity

Pathoanatomy - Immune cell: general aspects

Total questions: 37; Correctly answered: 11; Percentage of


correct: 29.7% Skipped question: 0;
correct answer
incorrect answer
skipped question

 1. Plasma cells

Contain nucleus



Helps in the formation of antibody



Are deficient in cytoplasm



Are derived from T-cells


 2. CD4 is not important for which of the following?

Antibody production



Cytotoxicity of T cells



None



Opsonisation

 3. Antigen presenting cells are which of the following:



Astrocytes



Endothelial cells



Epithelial cells



Langerhan’s cells

 4. Memory T cells can be identified by using the following marker:



CD45RA



CD45RB



CD45RC



CD45RO


 5. IL-1 causes

Increased leukocyte adherence



Fibroblast proliferation



Increased collagen synthesis



All of the above

 6. Which of the following immune cells have the expression of CD8


on their surface?

T-cells



B-cells



Null cells



Macrophages

 7. Antigen presenting cells are:



Langerhan’s cell



Macrophage



Cytotoxic T cells



Helper T cells

 8. CD3 is marker for:



Monocyte



T cell



B cell



None

 9. The following interleukin is characteristically produced in a TH1


response?

IL-2



IL-4



IL-5



IL-10

 10. All of the following statements about NK cells are true except:

They are derived from large granular cells



They comprise about 5% of human peripheral lymphoid cells



They are MHC restricted cytotoxic cells



They express IgG Fc receptors

 11. Natural killer cells attacks which of the following cells



Cells which express MHC1



Cells which are not able to express MHC1



MHC cells which express MHC2



Cells which are not able to express MHC

 12. NK cells express:



CD 15, CD 55



CD 16, CD 56



CD 16, CD 57



CD 21, CD 66

 13. Marker for B-Lymphocyte



CD34



CD33



CD19


CD20

 14. Most potent stimulator of naive T cell is?



Mature dendritic cell



Follicular dndritic cell



Macrophages



B cell

 15. The following feature is common to both cytotoxic T-cells and


NK cells:

Synthesize antibody



Require antibodies to be present for action



Effective against virus infected cells



Recognize antigen in association with HLA class II markers

 16. The complement is fixed best by which of the following


immunoglobulins:

IgG



IgM


IgA



IgD

 17. Immunity against cancer cells:



Basophills



Eosinophils



NK cells



Neutrophils

 18. Type 1 MHC presents peptide antigen to T cell , so that peptide


binding site is formed by:

Alfa and Beta chain



Distal domain alfa 1 and 2



Alfa and beta microglobulin



Proximal domain alfa 1 and 2

 19. The normal ratio of CD4 to CD8 is



1: 1



2: 1



8: 1



10: 1

 20. Function of CD4 is all except:



Memory



Immunoglobin production



Activation of macrophages



Cytotoxicity

 21. Which of the following features is not shared between ‘T cells’


and B cells’?

Antigen Specific Receptors



Class I MHC Expression



Positive selection during development



All of the above

 22. Macroglobulin is derived from:



B cells


T cells



Both



Natural killer cells

 23. Cell surface molecules involved in peripheral tolerance


induction are

B7 and CD28



CD40 and CD40L



CD34 and CD51



B7 and CD3

 24. MHC restriction to antigen presentation is not done for:



Killing of viruses by cytotoxic cells



Killing of bacteria by helper cells



T cell activation in autoimmunity



Graft rejection

 25. NK cell CD marker is:



16



60



32



25

 26. Kupffer cells are found in:



Heart



Lungs



Liver



Spleen

 27. Most potent stimulator of Naïve T-cells:



Mature dentritic cells



Follicular dendritic cells



Macrophages



B-cell


 28. Which one of the listed receptors is the type of receptor on
leukocytes that binds to pathogen-associated molecular patterns
(PAMPs) and mediates immune response to bacterial
lipopolysaccharide?

Cytokine receptor



G-protein-coupled receptor



Mannose receptor



Toll-like receptor

 29. Toll like receptors, recognize bacterial products and stimulates


immune response by:

Perforin and granzyme mediated apoptosis



FADD ligand apoptosis



Transcription of nuclear factor mediated by NFκB which recruits
cytokines



Cyclin

 30. CD-95 has a major role in:



Apoptosis



Cell necrosis



Interferon activation


Proteolysis

 31. The following interleukin is characteristically produced in a


TH1 response:

IL-2



IL-4



IL-5



IL-10

 32. CD4 cells is used to identify which of the following



MHC I



MHC II



T cells



B cells

 33. Which of the following immunoglobulin does not fix


complement?

IgA



IgG


IgM



IgE

 34. Antigen presenting cells present in skin are called



Langerhan’s cells



Kupffer’s cells



Microglia



Melanocytes

 35. Which of the following is not true about innate immunity?



It is present prior to antigenic exposure



It is relatively non-specific



Memory is seen



It is the first line of defense

 36. Birbeck granules are present in



Merkel cell



Langerhans cell


Langhans cell



Melanocyte

 37. A super-antigen is a bacterial product that



Binds to B7 and CD28 co-stimulatory molecules



Binds to the beta chain of TCR and MHC class II molecules of APC
stimulating T cell activation



Binds to the CD4 + molecule causing T cell activation



Is presented by macrophages to a larger-than-normal number of T
helper CD4 + lymphocytes


Pathoanatomy - Immunopathology

Total questions: 50; Correctly answered: 7; Percentage of


correct: 14.0% Skipped question: 0;
correct answer
incorrect answer
skipped question

 1. Which disease is associated with immediate type of


hypersensitivity?

Syphilis



Myocardial infarction



Liver cirrhosis



Purulent meningitis

 2. Changes in peripheric lymphoid tissue in congenital


immunodeficiency syndromes are characterized by:

Decrease in size of follicles of spleen



Increase in size of follicles of spleen



Hyperplasia of lymph nodes



Myeloid metaplasia of bone marrow

 3. Delayed type of hypersensitivity morphologically occurs as:



Acute immune inflammation



Purulent inflammation



Chronic immune inflammation



Hemorragic inflammation

 4. Morphologically delayed type of hypersensitivity occurs as:



Mucoid and Fibrinoid changes



Plasmatic saturation



Fibrinoid necrosis


Macrophageal infiltration

 5. Morphologically immediate type of hypersensitivity occurs as:



Fibrinoid necrosis



Lympho-hystiocytic infiltration



Macrophageal infiltration



Granulomatosis

 6. Which disease is associated with action of allergic


reagin-antibodies?

Chronic bronchitis



Pulmonary carcinoma



Atopic bronchial asthma



Bronchiectasis

 7. Collection of B-lymphocytes and plasma cells with formation of


lymphoid follicles in the intralobular perivascular spaces of
thymus parenchyma is called:

Aplasia



Hypoplasia


Thymomegaly



Hyperplasia with lymphoid follicles

 8. Changes in spleen under antigene stimulation include are all the


following processes, EXCEPT:

Hyperplasia of red pulp



Hypoplasia of follicles



Plasmatization of red pulp



Accumulaton of macrophages

 9. Morphologically delayed type of hypersensitivity occurs as:



Mucoid and Fibrinoid changes



Plasmatic saturation



Fibrinoid necrosis



Cytolysis

 10. Changes in peripheric lymphoid tissue under antigene


stimulation include are all the following processes, EXCEPT:

Macrophageal reaction


Hyperplasia of lymphocytes



Dysplasia of lymphocytes



Plasma cell transformation

 11. Morphological signs of delayed type of hypersensitivity are all


the following, EXCEPT:

Lympho-hystiocytic infiltration



Macrophageal infiltration



Granulomatosis



Plasmatic saturation

 12. Changes in bone marrow under antigene stimulation include


are all the following processes, EXCEPT:

Hyperplasia



Macrophageal-plasma cell transformation



Myeloid metaplasia



Hypoplasia


 13. Morphologically delayed type of hypersensitivity occurs as:

Lympho-hystiocytic infiltration



Mucoid and Fibrinoid changes



Plasmatic saturation



Fibrinoid necrosis

 14. Morphological signs of immediate type of hypersensitivity are


all the following, EXCEPT:

Mucoid and Fibrinoid changes



Plasmatic saturation



Granulomatosis



Fibrinoid necrosis

 15. Which disease is associated with immediate type of


hypersensitivity?

Myocardial infarction



Liver cirrhosis



Purulent meningitis



Rheumatic fever

 16. Immediate type of hypersensitivity morphologically occurs


like:

Acute immune inflammation



Purulent inflammation



Hemorragic inflammation



Chronic immune inflammation

 17. Morphologically delayed type of hypersensitivity occurs as:



Mucoid and Fibrinoid changes



Plasmatic saturation



Granulomatosis



Fibrinoid necrosis

 18. Which disease is associated with immediate type of


hypersensitivity?

Myocardial infarction



Nodular periarteritis



Purulent meningitis



Liver cirrhosis

 19. Changes in spleen under antigene stimulation include are all


the following processes, EXCEPT:

Hypoplasia of red pulp



Hyperplasia of red pulp



Plasmatization of red pulp



Accumulaton of macrophages

 20. Morphological signs of delayed type of hypersensitivity are all


the following, EXCEPT:

Fibrinoid necrosis



Lympho-hystiocytic infiltration



Macrophageal infiltration



Granulomatosis

 21. Which disease is associated with immediate type of


hypersensitivity?

Myocardial infarction



Liver cirrhosis



Аrthus reaction



Purulent meningitis

 22. Aquired decrease in weight of thymus is called:



Aplasia



Hypoplasia



Dysplasia



Atrophy

 23. Changes in lymph nodes under antigene stimulation include


are all the following processes, EXCEPT:

Hyperemia



Ischemia



Edema



Accumulation of plasma cells

 24. Changes in peripheric lymphoid tissue under antigene


stimulation include are all the following processes, EXCEPT:

Macrophageal reaction



Anaplastic transformation of lymphocytes



Hyperplasia of lymphocytes



Plasma cell transformation

 25. Morphological signs of immediate type of hypersensitivity are


all the following, EXCEPT:

Mucoid and Fibrinoid changes



Plasmatic saturation



Fibrinoid necrosis



Macrophageal infiltration

 26. Which disease is associated with immediate type of


hypersensitivity?

Myocardial infarction



Glomerulonephritis



Liver cirrhosis



Purulent meningitis

 27. Changes in spleen under antigene stimulation include are all


the following processes, EXCEPT:

Hyperplasia of red pulp



Plasmatization of red pulp



Accumulaton of giant cells



Accumulaton of macrophages

 28. Changes in bone marrow under antigene stimulation include


are all the following processes, EXCEPT:

Hyperplasia



Macrophageal-plasma cell transformation



Myeloid metaplasia



Myeloid dysplasia

 29. Changes in peripheric lymphoid tissue in congenital


immunodeficiency syndromes are characterized by:

Increase in size of follicles of spleen



Absence of germinal centers in follicles of spleen



Hyperplasia of lymph nodes


Myeloid metaplasia of bone marrow

 30. Which disease is not associated with immediate type of


hypersensitivity?

Myocardial infarction



Nodular periarteritis



Glomerulonephritis



Syphilis

 31. Signs of acute immune inflammation are all the following


processes, EXCEPT:

Fast development



Prevalence of alterative changes



Slow development



Prevalence of exudative changes

 32. Morphologically immediate type of hypersensitivity occurs as:



Lympho-hystiocytic infiltration



Fibrinous-hemorrhagic exudate


Macrophageal infiltration



Granulomatosis

 33. Morphologically immediate type of hypersensitivity occurs as:



Lympho-hystiocytic infiltration



Macrophageal infiltration.



Granulomatosis



Plasmatic saturation

 34. Morphologically delayed type of hypersensitivity occurs as:



Mucoid and Fibrinoid changes



Plasmatic saturation



Fibrinoid necrosis



Cytoplasmic bridges between lymphocytes and macrophages

 35. Changes in peripheric lymphoid tissue in congenital


immunodeficiency syndromes are characterized by:

Increase in size of follicles of spleen



Hyperplasia of lymph nodes



Absence of B-zones in cortex of lymph nodes



Myeloid metaplasia of bone marrow

 36. Reagin reactions are associated with action of which type of


allergic reagin-antibodies?

IgA



IgB



IgC



IgE

 37. Which disease is associated with immediate type of


hypersensitivity?

Myocardial infarction



Liver cirrhosis.



Purulent meningitis



Systemic lupus erytematosus


 38. Decrease in size and weight of thymus under different stress
situations including infectious diseases, intoxications, traumas is
called:

Aplasia



Hypoplasia



Atrophy



Accidental involution

 39. Which disease is not associated with immediate type of


hypersensitivity?

Glomerulonephritis



Nodular periarteritis



Arterial hypertension



Syphilis

 40. Reagin reactions are associated with formation of which type


of exudate?

Basophylic



Eosinophylic



Neutrophylic


Hemorrhagic

 41. Сongenital absence of thymus is called:



Aplasia



Hypoplasia



Dysplasia



Atrophy

 42. Morphologically immediate type of hypersensitivity occurs as:



Lympho-hystiocytic infiltration



Mucoid changes



Macrophageal infiltration



Granulomatosis

 43. Abnormal development of thymus is called:



Aplasia



Hypoplasia



Dysplasia


Atrophy

 44. Increase in size and weight of thymus more than age level with
normal histological structure is called:

Aplasia



Hypoplasia



Thymomegaly



Atrophy

 45. Morphological signs of immediate type of hypersensitivity are


all the following, EXCEPT:

Lympho-hystiocytic infiltration



Mucoid and Fibrinoid changes



Plasmatic saturation



Fibrinoid necrosis

 46. Which disease is not associated with immediate type of


hypersensitivity?

Liver cirrhosis



Nodular periarteritis


Glomerulonephritis



Syphilis

 47. Congenital incomplete development of thymus is called:



Aplasia



Hypoplasia



Dysplasia



Atrophy

 48. Changes in peripheric lymphoid tissue under antigene


stimulation include are all the following processes, EXCEPT:

Macrophageal reaction



Hyperplasia of lymphocytes



Plasma cell transformation



Infarction

 49. Changes in lymph nodes under antigene stimulation include


are all the following processes, EXCEPT:

Hyperemia


Edema



Accumulation of plasma cells



Accumulation of neutrophils

 50. Morphologically immediate type of hypersensitivity occurs as:



Lympho-hystiocytic infiltration



Fibrinoid changes



Macrophageal infiltration



Granulomatosis

Pathoanatomy - Infective lung disease: pneumonia, tb, lung abscess

Total questions: 23; Correctly answered: 10; Percentage of


correct: 43.5% Skipped question: 0;
correct answer
incorrect answer
skipped question

 1. Autopsy of a person reveals a small cluster of caseating


granulomas in the right lung just above the interlobar fissure and
similar granulomas in the hilar lymph nodes Acid-fast staining
demonstrates acid-fast bacilli within these lesions No other lesio

Cavitary tuberculosis



Ghon complex


Remote healed tuberculosis



Miliary tuberculosis

 2. Characteristic histopathological feature of pneumocystis carinii


pneumonia -

Interstitial pneumonitis



Mononuclear cell in bronchoalveolar lavage



Foamy vacuolated exudates



All Of above

 3. In the stage of Grey hepatisation, which of the following is a


finding?

WBC



RBC



Organisms fill the alveoli



Accumulation of fibrin

 4. Infraclavicular lesion of tuberculosis is known as:



Gohn’s focus


Puhl’s focus



Assman’s focus



Simmon’s focus

 5. Collapse of lung is called



Emphysema



Bronchiactasis



Atelectasis



Bronchitis

 6. In primary tuberculosis, all of the following may be seen except:



Cavitation



Caseation



Calcification



Langhans giant cell

 7. Predisposing factors of lung abscess are:



Altered sensorium


Dental sepsis



Aggressive treatment of pneumonia



Subpulmonic effusion

 8. The alveoli are filled with exudates the air is displaced


converting the lung into a solid organ this description suggests

Chronic bronchitis



Bronchial asthma



Bronchiectasis



Lobar pneumonia

 9. The earliest feature of tuberculosis is:



Caseation



Recruitment of lymphocytes



Formation of giant cells (Langhans)



Granuloma formation


 10. ESR is a very critical investigation is the diagnosis of TB Which
of the following is true about ESR in TB?

No change is ESR



Confirms recovery from TB



ESR is raised because of increased RBC aggregate



ESR is raised due to decreased RBC size

 11. Which of these is seen in primary tuberculosis:



Ghon’s focus



Pleural effusion



Miliary mottling



All Of above

 12. Atypical pneumonia can be caused by the following microbial


agents except?

Mycoplasma pneumoniae



Legionella pneumophila



Human corona virus



Klebsiella pneumoniae

 13. Maximum smooth muscle relative to wall thickness is seen in



Terminal bronchiole



Trachea



Bronchi



Respiratory bronchioles

 14. True about Ghon



Left apical parenchymal lesion



Right apical parenchymal lesion



Subpleural caseous lesion in right upper lobe



Subpleural caseous lesion just above or below interlobar fissure

 15. Gray hepatization of lungs is seen on day:



1



2_3



3_4


5_7

 16. All of the following features are seen in the viral pneumonia
except

Presence of interstitial inflammation



Predominance of alveolar exudate



Bronchiolitis



Multinucleate giant cells in the bronchiolar wall

 17. Pulmonary tuberculosis is more common in following


associated diseases, except:

Acquired immune deficiency syndrome



Diabetes



Chronic renal failure



Mitral stenosis

 18. A 60 year old smoker BD Thapa with inguinal hernia undergoes


surgery under general anaesthesia On the third post operative day,
he complains of increasing difficulty in respiration The finger
probe reveals presence of pO2 of 60 mm Hg but the patient is

Compression atelectasis



Microatelectasis



Resorption atelectasis



Contraction atelectasis

 19. Lung granuloma with necrosis is seen in



PAN



TB



Histoplasmosis



Cryptococcosis

 20. Reactivated TB is most commonly located near:



Apex



Near bronchus



Subpleurally



Base

 21. Lung granuloma found in A/E:



Berylliosis


Asbestosis



SLE



Sarcoidosis

 22. A 9 year old girl Bandhini developed a 10 mm area of induration


on the left forearm 72 hours after intradermal injection of 0 1 ml of
purified protein derivative (PPD) Though she appears healthy, a
chest X ray was performed Which of the following is mos

Marked hilar adenopathy



Upper lobe calcifications



No abnormal findings



Reticulo-nodular densities

 23. Pulmonary, renal syndrome is seen in:



Good pasture syndrome



Leptospirosis



Legionella



Wegener


Pathoanatomy - Intestine: ibd, polyp, tumours

Total questions: 18; Correctly answered: 2; Percentage of


correct: 11.1% Skipped question: 0;
correct answer
incorrect answer
skipped question

 1. Pseudopolyps are seen in:



Crohn



Ulcerative colitis



Juvenile polyposis



Tuberculosis

 2. Toxic megacolon is seen in:



Chronic nonspecific ulcerative colitis



Crohn



Colonic diverticulosis



Hamartomatous polyp

 3. Most common site of carcinoma pancreas is



Head



Body


Tail



None

 4. The minimum number of polyps necessary for a diagnosis of


Familial Adenomatous Polyposis (FAP) is:

5



10



50



100

 5. Carcinoid tumour develops from:



Hematopioetic cells



Kulschitsky cells



Neuroglial cells



Chromaffin cell

 6. Zollinger Ellison syndrome is not caused by tumors from:



Pancreas



Ovary



Colon



Duodenum

 7. A highly sensitive and specific marker for detecting intestinal


inflammation in ulcerative colitis is:

CRP



Fecal lactoferrin



Fecal calprotectin



Leukocytosis

 8. Which of the following type of anemia would be associated with


carcinoma of the colon?

Megaloblastic anemia



Iron deficiency anemia



Aplastic anemia



Hemolytic anemia

 9. Which of the following is a special stain for


rhabdomyosarcoma?

Cytokeratin



Juvenile polyp



Desmin



Myeloperoxidase

 10. The highest malignant potential is seen in:



Crohn



Ulcerative colitis



Familial polyposis



Infantile polyp

 11. Osteomas, adenomatous polyps of intestine and periampullary


carcinomas are seen in which oif the following conditions?

Cowden syndrome



Peutz Jegers syndrome



FAP



Gardener syndrome


 12. Following statements regarding ulcerative colitis is:

Smoking does not have a protective effect



Smoking has a protective effect



No relation with smoking



Smoking causing relapses

 13. Which of the following is the most common location of


carcinoid tumour?

Pancreas



Lung



Gastrointestinal tract



Gonads

 14. Which of the following is inheritance of Gardner syndrome?



Autosomal recessive



Autosomal dominant



X linked dominant



X linked recessive

 15. Crohn disease is associated with:



NOD2/CARD15 gene



P53 gene



Philadelphia chromosome



BRCA1 gene

 16. Antibody suggestive of diagnosis of ulcerative colitis?



P-ANCA



C-ANCA



A.M.A



A.N.A

 17. True about ulcerative colitis, all except:



Rectum involved



Pseudopolyps



Pancolitis



Noncaseating granuloma

 18. Commonest malignant small intestinal tumor



Adenocarcinoma



Lymphosarcoma



Leiomyosarcoma



Carcinoid tumor

Pathoanatomy - Intestine: infections, malabsorption diseases

Total questions: 12; Correctly answered: 2; Percentage of


correct: 16.7% Skipped question: 0;
correct answer
incorrect answer
skipped question

 1. Which of the following is not considered a premalignant lesion:



Leukoplakia



Erythroplakia



Chronic hyperplastic candidiasis



Oral lichen planus

 2. Perforation of typhoid ulcer usually occurs during which week?



1st



2nd



3rd



4th

 3. Paneth cells contain:



Zinc



Copper



Molybdenum



Selenium

 4. Which is incorrect of typhoid ulcers:



Hemorrhage is common



Occurs on lymphoid aggregation



Horizontal ulcers



Longitudinal ulcers

 5. Serum amylase level are raised in all of the following EXCEPT:



Duodeneal ulcer perforation


Pancreatitis



Appendicitis



Small Bowel Strangulation

 6. Purtscher’s retinopathy is seen in:



Meningitis



Pancreatitis



Uncontrolled hypertension



Unilateral carotid artery occlusion

 7. Which of the followingis the commonest site of intestinal


tuberculosis?

Stomach



Jejunum



Ileum



Colon

 8. Anti-gliadin antibodies are detectable in:



Tropical sprue



Whipple



Celiac disease



Intestinal lymphoma

 9. Which of the following is not associated with celiac sprue?



Turner syndrome



Down syndrome



Klinefleter syndrome



Type 1 diabetes

 10. The most common site for amoebiasis:



Sigmoid colon



Transverse colon



Cecum



Liver


 11. Usually, gall stones consists of these types, except:

Oxalate



Bile salts



Bile pigments



Cholesterol

 12. Diagnosis of typhoid in first week is by



Widal test



Stool culture



Urine culture



Blood culture

Pathoanatomy - Intracellular accumulations

Total questions: 50; Correctly answered: 7; Percentage of


correct: 14.0% Skipped question: 0;
correct answer
incorrect answer
skipped question

 1. Mucinous degeneration is occurs in:



Inflammation of heart



Inflammation of brain


Inflammation of bronchi



Inflammation of spinal cord

 2. Substance accumulating in tissues as a result is fatty change:



Water



Cholesterol



Protein



Triglycerides

 3. Glycogen storage disease is caused by



Genetic disorders



Hypoxia



Infection



Immune disorders

 4. One of cause of intracellular accumulation of metabolic


substances is:

Genetic defects



Inflammation



Embolism



Necrosis

 5. Ballooning degeneration of hepatocytes is caused by:



Accumulation of water and cellular swelling



Retaining of biliary materia



Accumulation of iron or copper substances



Accumulation of fat droplets

 6. Accumulation of cholesterol and cholesterol esters with


formation of tumorous masses is called:

Atheroma



Xanthoma



Adenoma



Papilloma

 7. Mallory’s bodies may be found in



Neurons


Cardiomyocytes



Hepatocytes



Epithelial cells of stomach

 8. Which type of lipids accumulate in atherosclerosis?



Triglycerides



Cholesterol



Complex lipids



Miscellaneous lipids

 9. Foam cells are characterized by accumulation of:



Neutral fat



Triglycerides



Cholesterol



Phospholipids

 10. Which type of lipids accumulates at fatty change in liver cells?



Triglycerides


Cholesterol



Complex lipids



Miscellaneous lipids

 11. Cells, which can accumulate cholesterol and cholesterol esters


in atherosclerotic plaque, are:

Macrophages and smooth muscle cells



Leucocytes and fibroblasts



Lymphocytes and erythrocytes



Fibroblasts and leucocytes

 12. A 48-year-old male who has a long history of excessive


drinking presents with sings of alcoholic hepatitis. Microscopic
examination of a biopsy of this patient’s liver reveals irregular
eosinophilic hyaline inclusions within the cytoplasm of the
hepatocyt

Immunoglobulin



Excess plasma proteins



Prekeratin intermediate filaments



Basement membrane material

 13. One of manifestations of metabolic derangements in cells is:



Apoptosis



Intracellular accumulation of abnormal amounts of various substances



Hypertrophy



Hyperplasia

 14. Hyaline droplets in renal tubular epithelial cells are seen in:

Proteinuria



Lipiduria



Hematuria



Cylindruria

 15. Lipids that can accumulate intracellularly are



Cholesterol



Derived lipids



Miscellaneous lipids



Lecithin

 16. In fatty liver to chronic alcoholism, the following mechanisms


are involved except:

Increased free fatty acid synthesis



Decreased triglyceride utilization



Increased α-glycerophosphate



Block in lipoprotein excretion

 17. What substances are accumulating within parenchymal cells in


steatosis?

Cholesterol



Apoproteins



Triglycerides



Vitamins

 18. Russel’s bodies may be found in



Epithelioid cells



Plasma cells



Lymphocytes


Histiocytes

 19. Give some example of intracellular hyaline degeneration:



Mallory body in alcoholic liver cells



Keloid



Renal glomeritis in chronic nephritis



Diabetic mellitus kidney

 20. Ballooning degeneration of hepatocytes results from:



Viral hepatitis



Alcoholic liver disease



Biliary material accumulation



Obesity

 21. The causes of steatosis include all of the following pathologic


states, except:

Obesity



Anoxia



Inflammation



Protein malnutrition

 22. One of manifestations of metabolic derangements in cells is:



Apoptosis



Atrophy



Hypertrophy



Intracellular accumulation of abnormal amounts of various substances

 23. Colloid degeneration is occurs in:



Salivary gland



Prostate gland



Thyroid gland



Pituitary gland

 24. Dystrophies are based on following mechanism, except one:



Decomposition



Transformation


Regeneration



Infiltration

 25. Give some example of intracellular hyaline degeneration:



Tubular epithelium of kidney



Keloid



Renal glomeritis in chronic nephritis



Diabetic mellitus kidney

 26. The cells accumulating fat within the intima layer of the aorta
and large arteries in atherosclerotic plaques are called:

Fibroblasts



Epithelial cells



Foam cells



Lymphocytes

 27. All of the following features characterize the ballooning


degeneration of hepatocytes, except:

Swollen cells


Edematous appearance



Clumped cytoplasm



Large droplets of fat

 28. Fatty change in the heart is characterized by



Tiger heart



Small size heart



Red heart



Solid heart

 29. Fatty change is seen by light microscopy as:



Intracellular granule



Basophilic granules



Basophilic granules



Vacuoles in the cytoplasm around the nucleus

 30. The stain used to identify fat is:



Hematoxylin and eosin stain


Red oil O



Congo red stain



PAS reaction

 31. One of the possible causes of intracellular accumulation of


metabolic substances is:

Genetic defects



Inflammation



Embolism



Necrosis

 32. Alzheimer disease is associated with intracellular


accumulations of proteins in

Liver



Heart



Spinal cord



Brain


 33. Tumor arising from epithelial cells with accumulation of mucin
is called:

Mucinous teratoma



Mucinous sarcoma



Mucinous myeloma



Mucinous carcinoma

 34. Following are the features of reversible injury, except:



Swelling of mitochondria and endoplasmic reticulum



Detachment of ribosomes from endoplasmic reticulum



Clumping of nuclear chromatin



Pyknosis and karyolysis of nucleus

 35. Fatty change in the heart characterized by:



Red heart



Small size heart



Tiger heart



Solid hear

 36. Xanthoma is a pathological process with accumulation of:



Neutral fat



Lipoids



Cholesterol



Phospholipids

 37. The stain used to identify fat is:



Hematoxylin and eosin stain



Sudan III stain



Congo red stain



PAS reaction

 38. The most common cause of fatty change in the heart is:

Inflammation



Neoplasia



Hypoxia



Amyloidosis

 39. Mallory’s bodies may be found in:



Hepatocytes



Neurons



Cardiomyocytes



Epithelial cells of stomach

 40. Xantoma is a pathological process with accumulation of:



Cholesterol



Lipoproteins



Phospholipids



Neutral fat

 41. The fatty liver has all pathologic features, except:



Enlarged



Yellow



Red



Soft

 42. The stain used to identify glycogen is:



Hematoxylin and eosin stain



Sudan III stain



Congo red stain



PAS reaction

 43. Fatty change is often seen in all of the following organs,


except:

Heart



Kidney



Muscles



Lung

 44. Dystrophy is defined as the process



Degeneration of tissue



Inflammation



Edema



Tumor

 45. All of the following mechanisms cause intracellular


accumulation, except

Abnormal metabolism



Protein mutation



Inflammation



Enzyme deficiency

 46. Increased lipolysis of fat stores, which can results from


starvations, diabetes mellitus, or corticosteroid use, it most likely
to cause steatosis (fatty liver) through which one of the listed
mechanisms?

Decreased free fatty acid excretion from the liver leads of free fatty acid
accumulation in hepatocytes



Excess NADH (high NADH/NNAD ratio) causes excess production of
lactate from pyruvate, which accumulates in hepatocytes



Increased free fatty acid delivery to the liver leads to triglyceride
accumulation in hepatocytes



Inhibition of apoprotein synthesis by the liver leads to phospholipids
accumulation in hepatocytes

 47. Derangement of glycogen metabolism is seen in:



Atherosclerosis


Diabetes mellitus



Viral hepatitis



Arterial hypertension

 48. Diabetes mellitus is characterized by the accumulation of


glycogen in all cells, except:

Epithelial cells of the proximal tubules



Liver cells



B-cells of the islets of Langerhans



Smooth muscle cells

 49. Lipids that can accumulate intracellularly are:



Triglycerides



Derived lipids



Miscellaneous lipids



Lecithin

 50. The most common cause of fatty change is



Inflammation



Neoplasia



Hypoxia



Amyloidosis


Pathoanatomy - Ischemic heart disease

Total questions: 27; Correctly answered: 7; Percentage of


correct: 25.9% Skipped question: 0;
correct answer
incorrect answer
skipped question

 1. In myocardial infarction scarring completes by:



1 day



1 week



1 month



3 months

 2. Autopsy finding after 12 hrs in a case of death due to M.I. is



Caseous pecrosis



Coagulative necrosis



Fat necrosis


Liquefactive necrosis

 3. Myocarditis is most commonly caused by



Influenza



Measles virus



Coxsackie virus



Epstein barr virus

 4. 60 year-old male smoker develops severe chest pain. He is


diagnosed with MI based on his electrocardiogram and serial
CK-MB levels. One week later he again complains of precordial
pain and develops a fever of 101.5?F. Physical examination is
remarkable fo

Caseous pericarditis



Fibrinous pericarditis



Hemorrhagic pericarditis



Purulent pericarditis

 5. Most common artery involved in myocardial infarction is



Right coronary artery



Left coronary artery



Left anterior descending coronary artery



Left circumflex coronary artery

 6. Earliest light microscopic change in myocardial infarction is:



Waviness of the fibers



Neutrophilic infiltration



Phagocytic infiltration



Coagulative necrosis

 7. Post MI day 10 which enzyme is raised:



CPK



Troponin



LDH



Myoglobin

 8. A 70 year old male Kulsheen presents to your OPD with


epigastric pain that typically starts about 30 minutes after meals
and does not respond to antacids. He lost almost 3-4 kilograms
over the last few weeks because he eats less for fear of pain. His
past

Peptic ulcer disease



Esophageal spasm



Pulmonary embolism



Stable angina

 9. A 70-year-old male Rohan with advanced visceral cancer dies of


extensive myocardial infarction. Autopsy also reveals sterile
non-destructive vegetations along the mitral leaflet edges. The
pathogenesis of this patient

Hypercalcemia of malignancy



Distant metastases



Trousseau syndrome



Raynaud

 10. In myocardial infarctions, microscopes picture of coagulation


necrosis with neutrophilic infiltration is seen after:

4-12 hr



12-24 hr



1-3 days


3-7 days

 11. 7 day old MI the most sensitive biochemical marker



Troponin T



CPK MB



LDH



Myoglobin

 12. Heart muscle contains the isoenzymes:



MM



MB



MM and MB



BB

 13. A 56-years-old male presented with sudden substernal pain,


impending doom and died 4 days after. On autopsy, there was a
large transmural anterior wall infarction. It would be associated
with:

Presence of collagen and fibroblasts



Presence of neutrophils


Granulomatous inflammation



Granulation tissue

 14. In myocardial reperfusion injury, the maximum effect is caused


due to which of the following?

Neutrophil



Monocytes



Eosinophils



Free radicals

 15. Troponin-T is a marker of:



Renal disease



Muscular dystrophy



Cirrhosis of liver



Myocardial infarction

 16. Enzyme elevated in first 2 hours of MI is:



CPK MB



LDH



SGPT



Acid phosphatase

 17. In MI with hypothyroidism, what is the marker of choice?



LDH



CPK-MB



Aldolase



Troponin-I

 18. What is the investigation for second MI after 1 week of previous


MI?

Troponin I



Troponin T



CPK-MB



LDH

 19. Which of the following increases the susceptibility to coronary


artery disease?

Type V hyperlipoproteinemia



Von Willebrand



Nephrotic syndrome



Systemic lupus erythematosus

 20. A 60-year-old male presented with acute chest pain of 4 hours


duration. Electrocardiographic examination revealed new Q wave
with ST segment depression. He succumbed to his illness within
24 hours of admission. The heart revealed presence of a
transmural

Edema in between normal myofibers



Necrotic myofibers with presence of neutrophils



Coagulative necrosis of the myocytes with presence of granulation
tissue



Infiltration by histiocytes with hemosiderin laden macrophages

 21. A 60 year old man Bhadru Kalraj is discharged after being


observed in the hospital for 4 days following a myocardial
infarction. He returns to his normal activities, which include
sedentary work only. This point in time following a myocardial
infarct is n

Arrhythmia



Ventricular aneurysm



Myocardial (pump) failure



Myocardial rupture

 22. A myocardial infarct showing early granulation tissue has most


likely occurred:

Less than 1 hour



Within 24 hours



Within 1 week



Within 1 month

 23. Dressler’s syndrome is:



Viral



Bacterial



Fungal



Autoimmune

 24. All of the following statements regarding subendocardial


infarction are true, except:

These are multifocal in nature



These often result from hypotension or shock



Epicarditis is not seen



These may result in aneurysm

 25. The cells seen after 72 hours in the infarcted area in MI are:

Neutrophils



Lymphocytes



Macrophages



Monocytes

 26. A 62 year-old male Sathish is admitted following an acute


ST-segment elevation myocardial infarction experiences chest
pain on day four of his hospitalization. He describes the pain as
sharp in quality, and adds that it increases with coughing and
swallow

Recurrent thrombosis of the affected coronary vessel



Thrombosis of a new coronary vessel



Pericardial inflammation overlying the necrotic segment of myocardium



Pericardial inflammation due to autoimmune reaction to necrotic tissue


 27. An old man is found dead in his home. Autopsy reveals
hemopericardium secondary to ventricular wall rupture. Roughly
how long before his death did the man probably have a myocardial
infarction?

2 days



7 days



12 days



20 days


Pathoanatomy - Kidney: general aspects, polycystic kidney disease

Total questions: 14; Correctly answered: 6; Percentage of


correct: 42.9% Skipped question: 0;
correct answer
incorrect answer
skipped question

 1. Major cause of death in End Stage Renal Disease patients on


display is which one of the following?

Cardiovascular disease



Infections



Uremia



Respiratory Failure

 2. Polycystic kidney may be associated with cysts in all the sites,


except:

Lung


Liver



Pancreas



Brain

 3. What is oliguria

Excretion of less than 300 ml in 24 hrs



Excretion of less than 500 ml in 24 hrs



Excretion of less than 300 ml in 12 hrs



Excretion of less than 100 ml in 24 hrs

 4. Which one of the following is not associated with adult


polycystic kidney disease?

Autosomal dominant inheritance



Mutations involving gene affecting cell-cell matrix interactions



Intracranial berry aneurysm may be present



Tricuspid valve prolapse


 5. What is the minimum number of red blood cells per microliter of
urine required for diagnosis of hematuria?

3



5



8



10

 6. True about adult polycystic kidney disease is all, except:



Autosomal dominant inheritance



Hypertension is rare



Can be associated with cysts in liver, lungs and pancreas



Pyelonephritis is common

 7. Which of the following is associated with adult polycystic


kidney disease?

Berry aneurysms of Circle of Willis



Saccular aneurysms of aorta



Fusiform aneurysms of aorta



Leutic aneurysms

 8. Chromosomes involved in adult polycystic kidney disease


(APKD) are:

6 and 11



4 and 16



7 and 17



4 and 12

 9. A 28 year old man has lenticonus and end stage renal disease
now. His maternal uncle also died of the same illness. What is the
most likely diagnosis?

Autosomal dominant polycystic kidney disease



Autosomal recessive polycystic kidney disease



Oxalosis



Alport syndrome

 10. Podocytes are seen in:



Proximal convoluted tubule



Distal convoluted tubule



Collecting tubule of the kidney



Bowman’s capsule

 11. A 42-year-old man Kumaran is having flank pain and


hypertension. He has increased blood urea nitrogen and creatinine
with hematuria. Ultrasound studies demonstrate markedly
enlarged kidneys with irregular margins and many fluid-filled
spaces of different

ACTH



Cortisol



Parathormone



Renin

 12. Adult polycystic kidney disease is inherited by:



Autosomal dominant



Autosomal recessive



X-linked



Mitochondrial

 13. True about autosomal dominant type of APKD:



Small kidney


Bilateral medullary cysts



Mutation of polycystin 1and 2 gene



Renal failure seen in middle life

 14. Acquired cystic disease of kidney is associated with:



Xanthogranulomatous pyelonephritis



Dialysis



Renal stones



Renal dysplasia

Pathoanatomy - Leukemia: all, aml, cll, cml

Total questions: 50; Correctly answered: 6; Percentage of


correct: 12.0% Skipped question: 0;
correct answer
incorrect answer
skipped question

 1. Causes of eosinophilia are:



Hodgkin's disease



Filariasis



MI


HIV infection

 2. Which of the following is having poor prognosis in ALL:



TLC 4000-10000



Age < 2 yrs



Presence of testicular involvement at presentation



Presence of blasts in peripheral smear

 3. Acid phosphatase is specific to which of the following cells



Monocyte



T-lymphocyte



B-lymphocyte



Myelocytes

 4. Which of the following is the least likely to be a preleukaemic


condition?

Paroxysmal nocturnal haemoglobinuria



Aplastic anaemia



Paroxysmal cold haemoglobinuria



Myelodysplastic syndrome

 5. Aplastic anemia can progress to:



AML



Myelodysplastic syndrome



CLL



PNH

 6. Absolute monocytosis is seen in:



Infectious mononucleosis



Kala-azar



TB



Brucellosis

 7. Pancytopenia with a cellular marrow is seen in all except



PNH



Megaloblastic anemia


Myelodysplastic syndrome



Congenital dyserythropoietic anemia

 8. An experimental study performed by Ayush Medical


International reveals that children with an unknown disease have
blood cells with absence of surface molecules like CD1a, CD2,
CD3, CD4, CD5 and CD8. Which of the following may be the
karyotype anomaly seen

21



XXY



22q11.2



T (15:17)

 9. Examination of a peripheral blood smear demonstrates


leukemia composed of small mature lymphocytes without blast
forms. Which of the following is the most likely age of this patient?

1 year



20 years



45 years



65 years


 10. Flow cytometry is done in:

Polycythemia



Thrombocytosis



Neutrophilia



Lymphocytosis

 11. In a patient with acute leukemia, immunophenotype pattern is


CD 19+ve, CD 10+ve, CD33+ve, CD 13+ve. He may probably have

Biphenotypic leukemia



ALL



AML-M



AML-M0

 12. In an ablated animal, myeloid series cells are injected. Which of


following is seen after incubation period?

Fibroblast



T lymphocytes



RBC



Hematopoetic stem cell

 13. All the following are poor prognostic indicators in AML except

Inv 16



Complex karyotype



AML M7



Deletion 7q

 14. Non-specific esterase is positive in all the categories of AML


except

M3



M4



M5



M6

 15. Highest LAP score is seen in



CML



Polycythemia vera



PNH


Pregnancy

 16. What is the chromosomal translocation in AML M3:



T (18, 21)



T (15, 17)



T (8, 21)



T (9, 11)

 17. True about aplastic anemia:



Splenomegaly



Nucleated RBC in peripheral blood



Reticulocytopenia



Thrombocytopenia

 18. Marker of myeloid cancers:



S100



HMB45



Common leukocyte antigen


Cyto-keratin

 19. T (2,8) is associated with:



T cell ALL



B cell ALL



CML



CLL

 20. Most specific marker for myeloid series is



CD 34



CD 45



CD 99



CD 117

 21. AML with gum infiltration, hepato- splenomegaly is most likely


to be:

ALL



M3



M2



M4

 22. AML with worst prognosis



8/21 translocation



Inversion 16



Normal cytogenetics



Monosomy 7

 23. A 17-year-old boy presented with TLC of 138



No surface antigens (null phenotype)



An immature T-cell phenotype (Tdt/CD34/CD7 positive)



Myeloid markers, such as CD13, CD33 and CD15



B cell markers, such as CD19, CD20 and CD22

 24. Which of the following statements in context of leukemias is


true?

Chronic myeloid leukemia occurs beyond 50 years of age



Hairy cell leukemic in less than 50 years has a good prognosis


Acute lymphoid leukemic in less than 1 year has a poor prognosis



Chronic lymphocytic leukemia occurs in less than 50 years of age

 25. Reed Sternberg cells are found in:



Hodkin's disease



Sickle cell anaemia



Thalassemia



CML

 26. Marker for granulocytic sarcoma:



CD33



CD38



CD117



CD137

 27. Which is the most common cytogenetic abnormality in adult


myelodysplastic syndrome (MDS)?

Trisomy 8



20q



5q



Monosomy 7

 28. Sideroblasts are seen in:



Thalassemia



Myelofibrosis



Alcoholism



Iron overload

 29. Which of the following is a pan-T lymphocyte marker?



CD2



CD3



CD19



CD25

 30. A 42-year old man was referred with a 2 week history of fever
weakness and bleeding gum. Peripheral smear showed
pancytopenia. The bone marrow examination revealed 26% blasts
frequency exhibiting Auer rods and mature myeloid cells. An
occasional neutrophi

Acid phosphatase



Non-specific esterase



Myeloperoxidase



Toluidine blue

 31. Myelofibrosis leading to a dry tap on bone marrow aspiration is


seen with which of the following conditions?

Burkitt



Acute erythroblastic leukemia



Acute megakaryocytic leukemia



Acute undifferentiated leukemia

 32. B cell marker are all except:



CD 19



CD 20



CD 10



CD135

 33. In CML, serum vitamin B12 level is



Slightly decreased



Normal



Markedly decreased



Increased

 34. All are B-cell marker except:



CD-15



CD-19



CD-21



CD-24

 35. The presence of the Philadelphia chromosome is associated


with a worse prognosis in patients with which of the following
diseases?

Acute lymphoblastic leukemia



Acute myelogenous leukemia



Chronic lymphocytic leukemia



Chronic myelogenous leukemia

 36. A peripheral smear with increased neutrophils, basophils,


eosinophils, and platelets is highly suggestive of:

Acute myeloid leukemia



Acute lymphoblastic leukemia



Chronic myelogenous leukemia



Myelodysplastic syndrome

 37. Adult patient presents with generalized lymphadenopathy and


blood film shows 70% immature looking lymphocytes. What
should be the next best investigation?

Genotyping/karyotyping



Immunophenotyping



Bone marrow



Peripheral smear study

 38. The blast cells of acute lymphocytic leukemia in childhood


contain:

Surface antigen


CALLA Ag



Antibodies to WBC



Thrombocytosis

 39. Specific stain for myeloblasts is:



Sudan black



PAS



Myeloperoxidase (MPO)



LAP

 40. A 15-year-old boy presented with one day history of bleeding


gums, subconjunctival bleed and purpuric rash. Investigations
revealed the following results: Hb-6.4 gm/dL; TLC-26,500/mm3
Platelet 35,000 mm3 ; prothrombin time-20 sec with a control of 13
sec;

Myeloblastic leukemia without maturation



Myeloblastic leukemia with maturation



Promyelocytic leukemia



Myelomonocytic leukemia

 41. Dohle bodies are seen in which of the following?



Multiple myeloma



May-Heggline anomaly



Waldenstorm Macroglobulinemia



Lymphoma

 42. Akshay, an 8 years old boy presents with severe headache for
10 days. His examination revealed petechial hemorrhages, bone
tenderness, hepatosplenomegaly and generalized
lymphadenopathy. His CBC shows: Hemoglobin 8.6 g/dl Platelet
count 38,000/mm3 WBC cou

Early pre-B (CD19+, TdT +) ; Hyperdiploidy



Early pre



Early pre



Early pre

 43. Which of the following is not compatible with a diagnosis of


chronic myelomonocytic leukemia?

Peripheral blood monocytosis more than 1



Absence of Philadelphia chromosome



More than 20% blasts in blood or bone marrow



Absent or minimal dysplasia in myeloid lineages

 44. A 42 year woman Sunaya presents with complaints of bleeding


gums for the past 20 days. The oral examination shows thickened
and friable gums. Also, she has hepatosplenomegaly with
generalized non tender lymphadenopathy. The blood count
reveals: Hemoglobin

Acute lymphoblastic leukemia



Acute megakaryocytic leukemia



Acute promyelocytic leukemia



Acute monocytic leukemia

 45. A 6 year old child presents with pallor that required two blood
transfusions previously. He has now developed fever and
petechial haemorrhages. His hemoglobin is is 9 g/dL, platelet
count is 20,000/mm3 and TLC is 60,000/ mm3 . Flow cytometry
reveals the c

ALL



AML



Mixed phenotypic leukaemia



Undifferentiated leukaemia


 46. A 60 year old man presented with fatigue, weight loss and
heaviness in left hypochondrium for 6 months. The hemogram
showed Hb, 10gm/dL, TLC 5 lakhs/ mm3 , platelet count 4
lakhs/mm3 , DLC, neutrophil 55%, lymphocytes 4%, monocytes 2%,
basophils 6%, metam

T (1:21)



T (9:22)



T (15, 17)



Trisomy 21

 47. Poor prognostic factor for ALL is



Hyperdiploidy



T(9;22) t(4;11)



Age at presentation is 2-8 yrs



Total Leucocyte count <50000

 48. Which of the following statements pertaining to leukemia is


correct?

Blasts of acute myeloid leukemia are typically Sudan black negative



Blasts of acute lymphoblastic leukemia are typically myeloperoxidase
positive



Low leucocyte alkaline phosphatase score is characteristically seen in
blastic phase of chronic myeloid leukemia



Tartarate resistant acid phosphatase positivity is typically seen in hairy
cell leukemia

 49. ALL L3 morphology is a malignancy arising from which cell


lineage?

Mature B-cell



Precursor B-cell



Immature T-cell



Mixed B cell and T-cell

 50. A 38 year old female Raman presented with the complaints of


fever with chills and rigors for last 10 days. Her blood reports
show Hemoglobin 14.1 g/dl Hematocrit 42.3% MCV 90 fL Platelet
count 2.4 lac/mm3 WBC count 14,000/mm3 Differential count
Segmented

Steroid (glucocorticoid) therapy



Pulmonary abscess Read explanation below



Vigorous exercise



Acute myelogenous leukemia


Pathoanatomy - Lung malignancies
Total questions: 24; Correctly answered: 4; Percentage of
correct: 16.7% Skipped question: 0;
correct answer
incorrect answer
skipped question

 1. Lung cancer most commonly associated with?



Asbestosis



Silicosis



Berylliosis



Coal worker pneumoconiosis

 2. Least common cause of clubbing is:



Adenocarcinoma



Squamous cell cancer



Small cell cancer



Mesothelioma

 3. Neuroendocrine lesions of lung are:



Carcinoid hamartoma



Alveolar carcinoma



Hamartoma



Asthma

 4. Which of the following is a finding in biopsy of mesothelioma of


pleura -

Myelin figures



Desmosomes



Weibel-Palade bodies



Microvilli invasion

 5. All are true regarding mesothelioma except:



Bilaterally symmetrical



Associated with asbestos exposure



Histopathalogy shows biphasic pattern



Occurs in late middle age

 6. True about oat cell carcinoma of lung is:



Secrete ectopic hormone



Variant of small cell carcinoma


Cause SIADH



All

 7. Most common site of metastasis in lung carcinoma is:



Brain



Kidney



Adrenal



Testes

 8. APUD cells are seen in:



Bronchial adenoma



Bronchial carcinoid



Hepatic adenoma



Villous adenoma

 9. A patient with small-cell carcinoma of the lung complains of


muscle weakness, fatigue, confusion, and weight gain Physical
examination is unremarkable Serum sodium is found to be 120
mEq/L Which of the following abnormal laboratory results would
also b

Decreased plasma atrial natriuretic peptide (ANP) concentration



Decreased plasma vasopressin concentration



Decreased serum osmolarity



Decreased urinary sodium concentration

 10. Pleural mesothelioma is associated with:



Asbestosis



Berylliosis



Silicosis



Berylliosis

 11. Which of the following is having the minimal chances of


causing a mesothelioma?

Amphibole



Crysolite



Amesolite



Tremolite


 12. In a 70 year old man who was working in asbestos factory for
10-15 years On routine X ray, a mass was seen in the right apical
region of the lung Biopsy was taken from the mass Which of the
following is seen on electron microscopic examination?

Numerous long slender microvilli



Melanosomes



Desmososmes with secretory endoplasmic reticulum



Neurosecretory granules in the cytoplasm

 13. Cavity formation is observed in one of the following


bronchogenic carcinoma:

Squamous cell



Oat cell



Adenocarcinoma



Bronchoalveolar

 14. All give rise to malignancy except



Cholelithiasis



Bronchiectasis



Ulcerative colitis


Paget

 15. A medical examination of a student reveals absence of cardiac


sounds on left side of the chest but surprisingly the normal heart
beat on the right side of the chest The liver edge can be palpated
on the left but not the right side of the abdomen He also

Down syndrome



Kartagener syndrome



Kawasaki disease



Marfan syndrome

 16. A patient Rasool is brought to the emergency room following a


seizure Serum electrolyte studies demonstrate serum sodium of
128 mEq/L The urine osmolarity is higher than the serum
osmolarity Chest x-ray demonstrates a lung mass Which of the
following

Adenocarcinoma



Bronchioloalveolar carcinoma



Large cell carcinoma



Small cell carcinoma

 17. A 42 years old woman Sugahi Ramamurty has a 3 month


history of mild persistent left sided chest pain She is a non-smoker
There is no physical finding but chest X ray shows a left side
pleural mass without pleural effusion CT scan of the chest shows
al

Hamartoma



Malignant mesothelioma



Metastatic breast carcinoma



Solitary fibrous tumor

 18. A 67 yr male with history of chronic smoking hemoptysis with


cough Bronchoscopic biopsy from centrally located mass shows
undifferentiated tumor histopathologically Most useful I H C
(immunohistochemical) marker to make a proper diagnosis would
be:

Cytokeratin



Parvalbumin



HMB-45



Hep-par1

 19. The most common lesions in the anterior mediastinum are all
except:

Thymomas



Lymphomas



Lymph node enlargement from metastasis



Teratomatous neoplasms

 20. Scar in lung tissue may get transformed into:



Adenocarcinoma



Oat cell carcinoma



Squamous cell carcinoma



Columnar cell carcinoma

 21. Indoor air pollution does not lead to:



Chronic lung disease



Impaired neurological development



Pneumonia in child



Adverse pregnancy outcome

 22. Hypersecretory granules are seen in which carcinoma of


lung?:

Adenocarcinoma



Small cell carcinoma


Large cell carcinoma



Bronchoalveolar carcinoma

 23. Which of the following can develop into lung cancer?



Asbestosis



Silicosis



Byssinosis



Anthracosis

 24. A 75-year-old man Sukhdev Singh with a significant smoking


history presents to the emergency room with complaints of
dyspnea and truncal, arm, and facial swelling for one week
Physical examination is remarkable for facial erythema and facial,
truncal, an

Adenocarcinoma



Hodgkin



Large cell carcinoma



Small cell carcinoma


 Dashboard

Test selection
Select semester
Select subject
Select
chapter

Pathoanatomy - Male genital tract

Total questions: 17; Correctly answered: 7; Percentage of


correct: 41.2% Skipped question: 0;
correct answer
incorrect answer
skipped question

 1. Which of the following is not a malignant tumor of germ cell


origin?

Mature teratoma



Choriocarcinoma



Dysgerminoma



Embryonal carcinoma


 2. In which of the following respects do a seminoma involving the
testis and a dysgerminoma involving the ovary differ most
significantly?

Most common age of presentation



Number of mitoses



Potential to contain foci of more aggressive tumors



Ultrastructural appearance

 3. Which one of the following is not used as a tumor marker in


testicular tumors?

AFP



LDH



HCG



CEA

 4. Condyloma are mostly caused by HPV types:



11 and 13



6 and 11



6 and 13



30 and 33

 5. Infertility is a common feature in



Too many Sertoli cells inhibit spermatogenesis via inhibin



Proper blood testis barrier is not established



There is no germ cell in this condition



Sufficient numbers of spermatozoa are not produced

 6. A 25-year-old man, Ramesh presents with a testicular mass and


is found to have high serum levels of ?-fetoprotein (AFP).
Microscopic examination of a biopsy from this mass reveals
sheets of undifferentiated cells along with focal primitive glandular
diffe

Benign Low Low



Benign Low High



Malignant Low Low



Malignant High Low

 7. Which one of the following is not used as a tumor marker in


testicular tumors?

AFP



LDH



HCG



CEA

 8. Metastasis is least common with:



Embryonal cell carcinoma



Endodermal sinus tumor



Teratocarcinoma



Spermatocytic seminoma

 9. A 10-year-old child develops a testicular mass and undergoes


orchiectomy. On cut section, the mass shows a variety of
appearances and colors. Histologically, many different tissues are
seen, including cartilage, thyroid, and neural tissue. A small focus
o

Dermoid cyst



Teratoma with malignant transformation



Immature teratoma



Solid mature teratoma


 10. Gleason's classification is used for:

Carcinoma breast



Carcinoma prostate



Carcinoma pancreas



Carcinoma rectum

 11. Benign hyperplasia of prostate first develops in:



Central zone



Peripharal zone



Periurethral transition zone



Any of the above

 12. Predisposing factors for germ cell tumor are:



Cryptorchidism



Testicular feminizing syndrome



Klinefelter



Smoking


 13. Alpha fetoprotein is Not raised in which testicular tumors?

Choriocarcinoma



Teratocarcinoma



Yolk sac tumor



Embryonal cell carcinoma

 14. In the testis intratubular germ cell neoplasia is seen in all,


except:

Seminomas



Spermatocytic seminoma



Yolk sac tumor of testis



Embryonal carcinoma

 15. All of the following statements about Gleason grading system


are true except:

Score range from 1 to 10



High score is associated with bad prognosis



Helps in grading of tumor



Helps decide treatment modality

 16. Which one of the following is not used as a tumor marker in


testicular tumors?

AFP



LDH



HCG



CEA

 17. The commonest site for extragonadal germ cell tumour is:

Pineal gland



Mediastinum



Retroperitoneum



Sacrococyygeal regionPathoanatomy - Megaloblastic anemia, aplastic
anemia

Total questions: 29; Correctly answered: 6; Percentage of


correct: 20.7% Skipped question: 0;
correct answer
incorrect answer
skipped question

 1. Howell-Jolly bodies are seen in:



Alcoholics



Cirrhosis



Nephrotic syndrome



Postsplenectomy

 2. All of the following can cause reticulocytosis except



Aplastic anemia



Thalassemia



Sickle cell anemia



Chronic blood loss

 3. Vitamin B12 malabsorption is caused by:



Ankylostoma duodenale



Diphyllobothrium latum



Giardiasis



Taenia solium

 4. TRUE about Schilling test is all except:



B12 deficiency


Folic acid deficiency



Ileal disease



Bacterial overgrowth

 5. Schilling test is used for identification of which of the following?



Fat absorption



Vit K absorption



Vitamin B12 absorption



Vitamin D absorption

 6. Macrocytic anemia is caused by:



Hookworm infestation



Iron deficiency



Diphyllobothrium latum infestation



All of the above

 7. Macrocytic anemia may be seen in all of these except:



Liver disease


Copper deficiency



Thiamine deficiency



Vitamin B12 deficiency

 8. Hb A2 is raised in which of the following conditions?



Beta thalassemia trait



Sickle cell anemia



Hereditory spherocytosis



G6 PD deficiency

 9. Pure red cell aplasia is associated with:



Thymoma



Renal cell carcinoma



Hepatocellular carcinoma



Prostate carcinoma

 10. Normocytic normochromic anemia is seen in all except:



Aplastic anemia


Chronic renal disease



Pure red cell aplasia



Thalassemia

 11. Which is the true statement regarding megaloblastic anemia



Megaloblastic precursors are present in bone marrow



Mean corpuscular volume is increased



Serum LDH is increased



Thrombocytosis occurs

 12. Which of the does not indicate megaloblastic anemia?



Raised bilirubin



Mild splenomegaly



Increased reticulocyte count



Nucleated red cells

 13. A 50-year-old female Ramya with megaloblastic anemia and


ataxia is given radiolabeled cobalamin by mouth followed by an
intramuscular injection of unlabeled cobalamin. The urine
radioactivity level measured afterwards is determined to be normal.
Which of

Dietary cobalamin deficiency



Atrophic gastritis



Nontropical sprue



Fish tapeworm infestation

 14. Aplastic anemia can progress to all except:



AML



Myelodysplastic anemia



Pure red cell aplasia



Paroxysmal nocturnal hemoglobinuria

 15. Macrocytosis in complete blood count can be diagnosed by:



? MCV



? MCHC



? Hematocrit



? Red cell distribution width

 16. A 50-year-old male Pandey undergoing evaluation for fatigue


and exertional dyspnea is diagnosed with macrocytic anemia.
Upper gastrointestinal endoscopy is consistent with atrophic
gastritis. Some blood parameter changes after the first dose of
cyanocoba

RBC count



Reticulocyte count



Haptoglobin



Gastrin

 17. A 76 years old male presented with anemia with splenomegaly.


PBS shows tear drop shaped cells and bone marrow examination
was normal. The diagnosis is:

Myelofibrosis



Iron deficiency anemia



Folic acid deficiency



CML

 18. Hypersegmented neutrophils are present in which of the


following anemia?

Hemolytic



Iron deficiency



Megaloblastic



Aplastic

 19. Hypersegmented neutrophils are seen in:



Thalassemia



Iron deficiency



Megaloblastic



All

 20. Reticulocytosis is not seen in which of the following


conditions?

Thalassemia



Hereditary spherocytosis



Chronic renal failure



Sickle cell anemia

 21. Maturation failure in poor absorption of the vitamin B12 is


associated with

Microcytic hypochromic anemia



Sickle cell anemia



Anemia occurs after 3-4 months of poor absorption



Causes polycythemia

 22. A patient with Hb-6 gm%, TLC 1200, platelet-60,000, MCV 12fl,
what is the diagnosis?

Aplastic anemia



Megaloblastic anemia



PNH



Myelofibrosis

 23. Which of these does not indicate megaloblastic anemia?



Increased reticulocyte count



Raised Bilirubin



Mild splenomegaly



Nucleated RBC


 24. Serum vitamin B12 level is increased in all except:

Hepatitis



Cirrhosis of liver



Hepatocellular carcinoma



Cholestatic jaundice

 25. Causes of vitamin B12 deficiency megaloblastic anemia are:



Fish tap worm infestation



Dilantin therapy



Gastrectomy



Ileal resection

 26. Which of following viruses causes hemolysis of red blood


cells?

Rubella



Human parvo virus B19



Measles



Dengue virus

 27. FIGLU test is done for:



Cyanocobalamin deficiency



Folic acid deficiency



Thiamine deficiency



Riboflavin deficiency

 28. Cause of macrocytic anemia is:



Sideroblastic anemia



Iron deficiency



Thalassemia



Hypothyroidism

 29. An adult who develops pure red cell aplasia should be


explicitly evaluated for which of the following?

Gastric adenocarcinoma



Pancreatic adenocarcinoma



Papillary thyroid cancer



Thymoma

Pathoanatomy - Mhc

Total questions: 9; Correctly answered: 1; Percentage of


correct: 11.1% Skipped question: 0;
correct answer
incorrect answer
skipped question

 1. MHC-II positive cells are all except:



B cells



T cells



Macrophages



Platelets

 2. Major histocompatibility complex class I is seen on which of the


following cell?

Macrophages only



All body cells



B cell only



All blood cells except erythrocytes

 3. Antigen presented along with HLA class II stimulate



CD8 cell



CD4 cell



CD2 cell



CD19 cell

 4. MHC class I are present on all except



Platelets



All nucleated cells



RBCs



WBCs

 5. HLA B27 is not seen in which of the following?



Ankylosing spondylitis



Reiter



Rheumatoid arthritis



Psoriatic arthritis


 6. The role played by Major Histocompatibility Complex 1 and 2:

Transduce the signal to T cells following antigen recognition



Mediate immunogenic class switching



Present antigens for recognition by T cell antigen receptors



Enhance the secretion of cytokines

 7. Epitope binding floor of the MHC molecule conists of



Alpha helices



Beta pleated structure



Alpha and beta-1 chain



Beta-2 microglobin

 8. True about MHC:



Transplantation reaction



Autoimmune disease



Immunosuppression



Involved in T-cell function


 9. True about MHC-class II:

Not involved in innate immunity



Cytotoxic T-cell involved



Present in nucleated cells



Present in B-cells

Pathoanatomy - Microcytic anemia: ida, aocd, sideroblastic anemia

Total questions: 39; Correctly answered: 9; Percentage of


correct: 23.1% Skipped question: 0;
correct answer
incorrect answer
skipped question

 1. A patient of anemia due to chronic inflammation, the positive


finding is

Serum iron is increased



S. ferritin is decreased



TIBC is decreased



Presence of normal iron in blasts

 2. Anemia in humans can be caused by which of the following


worm?

Roundworm



Hookworm



Strongyloides



Tapeworm

 3. Ringed sideroblasts are seen in:



Iron deficiency anemia



Myelodysplastic syndrome



Thalassemia



Anemia of chronic disease

 4. Microcytosis is seen in:



Thalassemia



Hb Lepore



Hb Barts



Gastrectomy

 5. Earliest feature of correction of iron deficiency anemia is:



Reticulocytosis


Increase in serum ferritin



Increase in RBC count



Increase in serum iron level immediately

 6. Microcytic hypochromic anemia is seen in:



Hereditary spherocytosis



Thalassemia major



Iron deficiency anemia



Pernicious anemia

 7. A patient presents with increased serum ferritin, decreased


TIBC, increased serum iron, % saturation increased. Most
probable diagnosis is:

Anemia of chronic disease



Sideroblastic anemia



Iron deficiency anemia



Thalassemia minor


 8. All of the following if present provide protection against malaria
except:

Duffy blood group



Sickle cell anemia



Thalassemia



G6PD deficiency

 9. A 13 yr girl with fatigue and weakness was found to be having


reduced hemoglobin. Her MCV 70fl, MCH 22pg and RDW was 28.
What is her most likely diagnosis?

Iron deficiency anemia



Thalassemia minor



Sideroblastic anemia



Thalassemia major

 10. Sideroblastic anemia is seen in chronic poisoning with:



Lead



Arsenic



Copper



Mercury

 11. A 60-year old male patient with history of rheumatoid arthritis


presents with the following: Hb:4.5g/dL. platelet count is 2
lakh/mm3. TLC: 6000/mL, serum ferritin is 200

Anaemia of chronic disease



Thalassemia minor



Iron deficiency anemia



Autoimmune haemolytic anemia

 12. A 30 years old female, RBC count 4.5 million, MCV 55fl, TLC
8000/mm3. There is no history of blood transfusion. What is the
likely diagnosis?

Iron deficiency anemia



Thalassemia major



Thalassemia minor



Megaloblastic anemia

 13. The pathogenesis of hypochromic anemia in lead poisoning is


due to:

Inhibition of enzymes involved in heme biosynthesis



Binding of lead to transferrin, inhibiting the transport of iron



Binding of lead to cell membrane of erythroid precursors.



Binding of lead to ferritin inhibiting their breakdown into hemosiderin

 14. The condition which does not cause microcytic hypochromic


anemia is:

Iron deficiency



Hookworm infestation



Absence of intrinsic factor



Prolonged bleeding episodes

 15. A 68 year-old man Babu Rao Apte presents with skin


pigmentation, cirrhosis and diabetes mellitus. Which pattern for
serum iron and total iron-binding capacity (TIBC) is most
consistent with the familial illness suggested by these findings?

Increased Serum iron, Increased TIBC



Increased Serum iron, Decreased TIBC



Decreased Serum iron, Decreased TIBC



Decreased Serum iron, Increased TIBC


 16. Most common cause of anemia is:

Iron deficiency



Folic acid deficiency



Sideroblastic anemia



Pernicious anemia

 17. Hemochromatosis affects all of the following organs except



Liver



Pancreas



Heart



Salivary gland

 18. Anemia of chronic disease is characterized by of all except:



? Serum iron level



? TIBC



? Serum ferritin level



Increased macrophages iron in marrow


 19. Hypochromic microcytic blood picture is seen in all of the
following conditions except:

Iron deficiency anemia



Lead poisoning



Rheumatoid arthritis



Sideroblastic anemia

 20. Iron deficiency anemia is seen in:



Chronic renal failure



Billroth II operation



Hookworm infection



All

 21. Rate of iron uptake is regulated by which one of the following:



Mucosal cell iron stores



Route of administration



Preparation administered



Age of the patient

 22. Skin pigmentation in hemochromatosis occurs due to:



Melanin



Ferritin



Hemosiderin



All

 23. Echinocytes are types of:



RBCs



Lymphocytes



Monocytes



Platelets

 24. Low iron and low TIBC is seen in



Anaemia of chronic disease



Sideroblasticanaemia



Iron deficiency anaemia



Aplastic anemia

 25. All are laboratory finding in iron deficiency anemia except:



Decreased serum iron



Increased total iron binding capacity



Decreased serum ferritin



Increased mean corpuscular volume

 26. In Anemia of chronic disease, what is seen?



TIBC



S. Iron ?



BM iron ?



S. ferritin

 27. Storage form of iron in body is:



Ferritin



Transferrin



Ceruloplasmin



Ferriportin

 28. Sideroblastic anemia is caused by all except:



Collagen vascular disease



Erythropoetic porphyria



Lead poisoning



Cutaneous porphyria

 29. A 30 years old female asymptomatic not requiring blood


transfusion has Hb-13 gm%, HbF-95%, HbA2 1.5%. Which of the
following is the most likely diagnosis?

Beta-Heterozygous thalassemia



Beta-Homozygous thalassemia



Intermediate thalassemia



Persistently raised HbF

 30. True about iron deficiency anemia is:



Microcytic hypochromic anemia



Decreased TIBC



Increased ferritin


Bone marrow iron decreased earlier than serum iron

 31. Lead poisoning is associated with:



Microcytic hypochromic anemia



Macrocytic anemia



Decreased levels of zinc protoporphyrin



Howell-Jolly bodies

 32. Which of the following glycoproteins is transported in plasma


in iron metabolism:

Spectrin



Transferrin



Ferritin



Hemosiderin

 33. Bone marrow iron is increased in:



Thalassemi(a)



Iron deficiency anemi(a)



Anemia in chronic disease



PNH

 34. A 20 year old female presents with the following laboratory


values: hemoglobin 9gm%, MCV is 55%, RBC is 4.5 million/mm3.
There is no history of blood transfusion. What is the most likely
diagnosis out of the following?

Thalassemia major



Thalassemia minor



Iron deficiency anemia



Anemia of chronic disease

 35. Anemia in CRF is due to:



? erythropoietin



?RBC survival



? folate



Bone marrow hypoplasia

 36. Microspherocytes in peripheral blood smear are seen in:



Congenital spherocytosis



Autoimmune acquired hemolytic anemia



Thalassemia



All of the above

 37. Response of iron therapy in a patient with iron deficiency


anemia is denoted by:

Restoration of enzymes



Reticulocytosis



Increase in iron binding capacity



Increase in hemoglobin

 38. Lead causes following except



Uroporphyrinuria



Sideroblastic anemia



Basophilic stippling



Macrocytic anemia

 39. Best parameter for assessment of body iron stores is



Serum iron



Serum TIBC



Serum ferritin



Serum transferin

Pathoanatomy - Miscellaneous

Total questions: 16; Correctly answered: 4; Percentage of


correct: 25.0% Skipped question: 0;
correct answer
incorrect answer
skipped question

 1. Finding on histopathological examination of liver in case of


malaria is

Microabscess formation



Kupffer s cell hyperplasia with macrophage infiltration around periportal
area laden with pigments



Non caseating granuloma



Non specific finding of neutrophilic infiltration

 2. Centrilobular necrosis is seen in



CCl4



White phosphorus


Yellow fever



Eclampsia

 3. A retired man Pradyuman R complains of vague abdominal pain


since last 6 months. One fine day, he experienced acute chest pain
with dyspnea. He was rushed to the emergency of Gangaram
Hospital where his chest and abdomen CT scans demonstrate a
pulmonary e

BRCA -2



K-RAS



PRSS1



SPINK1

 4. Histological finding in Reye s syndrome is



Budding and branching of mitochondria



Swelling of endoplasmic reticulum



Para-nuclear micro-dense deposits



Glycogen depletion

 5. Gene of Wilsons disease is:



ATP 7A



ATP 7B



ADP 7A



ADP 7B

 6. A-1 antitrypsin deficiency causes



Congenital cystic fibrosis



Neonatal hepatitis



Pulmonary fibrosis



All of the above

 7. Liver granulomas may be associated with all of the following


except

Candida



Halothane



Sarcoidosis



Hepatic metastasis


 8. All are true about Wilson s disease except

JLiver Cu



JUrine Cu



JCeruloplasmin



JSerum Cu

 9. Liver in hemochromatosis is stained by which of the following


stain?

Perls iron stain



Alcian blue



Congo Red



Masson trichome

 10. Kayser-Fleischer ring is seen in



Wilson s disease



A- 1 antitrypsin deficiency



Hemochromatosis



Primary biliary cirrhosis

 11. Copper is mainly transported by:



Albumin



Haptoglobin



Ceruloplasmin



Globulin

 12. True statements about a-l antitrypsin deficiency is



Autosomal dominant disease



Emphysema



Fibrosis of portal tract



Diastase resistant positive hepatocytes

 13. All are seen in hemochromatosis except:



Hypogonadism



Arthropathy



Bronze diabetes



Desferrioxamine is the treatment of choice

 14. Bronze diabetes is seen in:



Wilson s disease



Sarcoidosis



Lead intoxication



Hemochromatosis

 15. True about hemochromatosis is:



Complete penetrance



Autosomal recessive



Phlebotomy leads to cure



More common in females

 16. Pigmentation in the liver is caused by all except



Lipofuscin



Pseudomelanin



Wilson s disease



None

Pathoanatomy - Miscellaneous :5

Total questions: 50; Correctly answered: 8; Percentage of


correct: 16.0% Skipped question: 0;
correct answer
incorrect answer
skipped question

 1. A granuloma is a focus of chronic inflammation consisting of a


microscopic aggregation of macrophages. Which of the following
is an immune granuloma and NOT a foreign body granuloma?

Talc



Sutures



Microbes



Soil

 2. Which of the following growth factors appears to be the most


important in scar formation during the fibroblast migration and
proliferation stage?

EFG



PDGF



FGF



TGF-α


 3. Which of the following is marked by the outpouring of a thin
fluid that, depending on the size of injury, is derived from either
the plasma or the secretions of mesothelial cells lining the
peritoneal, pleural, and pericardial cavities?

Serous inflammation



Fibrinous inflammation



Suppurative or purulent inflammation



Ulcers

 4. Which of the following growth factors comes from macrophages


and fibroblasts and stimulates synthesis of proteoglycans?

Tumor Nectosis Factor (TNF)



Transforming Growth Factor-β (TGF-β)



Keratinocyte Growth Factor (KGF)



Insulin-like Growth Factor (IGF-1)

 5. Nitric oxide (NO) is synthesized from what amino acid by the


enzyme nitric oxide synthase (NOS)?

Alanine



Arginine



Asparagine


Lysine

 6. Which of the following growth factors comes from macrophages,


mast cells, and lymphocytes and functions in chemotaxis and
regulation of cytokines?

Tumor Nectosis Factor (TNF)



Transforming Growth Factor-β (TGF-β)



Keratinocyte Growth Factor (KGF)



Interleukin-1 (IL-1)

 7. Which of the following is NOT capable of regeneration?



Epithelial tissue



Cardiac tissue



Skin



Liver

 8. The ECM contains cell adhesion molecules (CAMs). Which of


the following families of CAMs are generally involved in
calcium-dependent homotypic interactions?

Cadherins



Immunoglobulin



Integrins



Selectins

 9. Which of the following growth factors comes from platelets, T


cells, macrophages, fibroblast, and smooth muscle cells and is a
growth inhibitor for most epithelial cells?

Tumor Nectosis Factor (TNF)



Transforming Growth Factor-β (TGF-β)



Keratinocyte Growth Factor (KGF)



Insulin-like Growth Factor (IGF-1)

 10. Which of the following families of CAMs function in adhesion


of leukocytes to endothelial cells?

Immunoglobulin



Cadherins



Integrins



Selectins


 11. Vascular changes associated with acute inflammation include
____ (from histamine and NO) and ____ vascular permeability.

Vasoconstriction; Decreased



Vasoconstriction; Increased



Vasodilation; Decreased



Vasodilation; Increased

 12. Which of the following is the correct order of the cell cycle?

G1 => S => G2 => M



G2 => S => G1 => M



G1 => M => G2 => S



G2 => M => G1 => S

 13. Which of the following growth factors comes from


mesenchymal cells and enhances proliferation of epithelial and
endothelial cells?

Epidermal Growth Factor (EGF)



Transforming Growth Factor-α (TGF-α)



Hepatocyte Growth Factor (HGF)



Vascular Endothelial Growth Factor (VEGF)

 14. Which of the following growth factors comes from platelets,


macrophages, endothelial cells, and smooth muscle and functions
as a chemotactic for PMNs, macrophages, fibronblast, and smooth
muscle? It also functions to stimulate production of MMPs,
stimulat

Epidermal Growth Factor (EGF)



Hepatocyte Growth Factor (HGF)



Vascular Endothelial Growth Factor (VEGF)



Platelet-Derived Growth Factor (PDGF)

 15. NO causes vasodilation by what mechanism?



Constriction of smooth muscle



Relaxation of smooth muscle



Constriction of striated muscle



Relaxation of striated muscle

 16. Which of the following is NOT a general principle of the


chemical mediators of inflammation?

Mediators originate either from plasma or from cells



The production of active mediators is triggered by microbial products or
by host proteins



Most mediators perform their biologic activity by initially binding to
specific receptors on target cells



Once activated and released from the cell, most of these mediators last
a long time (long-lived)

 17. Which of the following growth factors comes from platelets,


macrophages, saliva,urine, milk, and plasma and functions to
stimulate keratinocyte migration and granule tissue formation?

Epidermal Growth Factor (EGF)



Transforming Growth Factor-α (TGF-α)



Hepatocyte Growth Factor (HGF)



Vascular Endothelial Growth Factor (VEGF)

 18. Hyaluronic acid (HA) is a component of the ECM and is bound


by what surface glycoprotein on leukocytes, allowing T cells to
remain bound to endothelium at sites of inflammation?

CD3



CD4



CD34



CD44

 19. Which of the following is associated with the IP3 signal


transduction pathway?

Receptors with intrinsic tyrosine kinase activity



Receptors with intrinsic tyrosine kinase activity



Seven transmembrane G-protein-coupled receptors (GPCRs)



Steroid hormone receptors

 20. Which of the following growth factors comes from


macrophages and T cells and activates macrophages as well as
regulates cytokines?

Tumor Nectosis Factor (TNF)



Transforming Growth Factor-β (TGF-β)



Keratinocyte Growth Factor (KGF)



Insulin-like Growth Factor (IGF-1)

 21. Vitamin D utilizes which of the following signal transduction


pathways?

Receptors with intrinsic tyrosine kinase activity



Receptors lacking intrinsic tyrosine kinase activity that recruit kinases



Seven transmembrane G-protein-coupled receptors (GPCRs)



Steroid hormone receptors

 22. One possible outcome of acute inflammation is resolution, with


the other outcomes being chronic inflammation and fibrosis (loss
of function). Which of the following is NOT associated with
resolution?

Clearance of injurious stimuli



Clearance of mediators and acute inflammatory cells



Replacement of injured cells



Angiogenesis

 23. Which of the following is true regarding target cell signal


transduction by growth factors?

It stimulates the transcription of genes that were silent in the resting
cells



It blocks the transcription of genes that were active in the resting cells



The genes regulate the entry of the cells into the cell cycle



A&C

 24. Which of the following cell types is pluripotent?



Quiescent


Labile



Permanent



Embryonic stem cell

 25. Unlike acute inflammation, chronic inflammation is


characterized by tissue destruction, induced by the persistent
offending agent or by the inflammatory cells. It is also
characterized by the cells involved, which include all of the
following EXCEPT:

Lymphocytes



Plasma cells



Macrophages



Neutrophils

 26. Mediators such as histamine, thrombin, and platelet activating


factor (PAF) stimulate the redistribution of which of the following
from its normal intracellular stores in granules (Weibel-Palade
bodies) to the cell surface?

P-selectin



E-selectin



ICAM-1



VCAM-1

 27. Which of the following extracellular matrix (ECM) fibrous


structural proteins is the most common protein in the animal world
and is composed of a triple helix of three polypeptide chains?

Collagen



Elastin



Fibrillin



Elastic fibers

 28. Which of the following families of CAMs participate in both


homotypic and heterotypic cell-to-cell interactions due to the
types of ligands they bind?

Immunoglobulin



Cadherins



Integrins



Selectins

 29. Which of the following is NOT a systemic effect of


inflammation?

Fever



Increased acute-phase proteins



Leukocytosis



Decreased pulse and blood pressure

 30. Which of the following would be associated with defective


inflammation?

Infections



Allergies



Asthma



Psoriasis

 31. Which of the following is NOT true regarding contribution to


inflammation?

Lysosomal constituents increase vascular permeability and tissue
damage



Oxygen free radicals amplify the cascade that elicits the inflammatory
response



Neuropeptides help initiate and propagate the inflammatory response



The response to hypoxia decreases vascular permeability


 32. Which of the following growth factors comes from fibroblasts
and stimulates keratinocyte migration, proliferation, and
differentiation?

Tumor Nectosis Factor (TNF)



Transforming Growth Factor-β (TGF-β)



Keratinocyte Growth Factor (KGF)



Insulin-like Growth Factor (IGF-1)

 33. Which of the following appears, histologically, as an


eosinophilic meshwork of threads or sometimes as an amorphous
coagulum?

Serous inflammation



Fibrinous inflammation



Suppurative or purulent inflammation



Ulcers

 34. Which of the following is NOT true regarding NO?



NO acts on target cells through induction of GMP



NO reduces platelet adhesion



NO reduces leukocyte adhesion


NO promotes leukocyte rolling

 35. Proteoglycans, such as heparan sulfate, chondroitin sulfate,


and dermatan sulfate, are a component of the ECM and made from
glycosaminoglycans (GAGs). Which of the following is NOT a
function of proteoglycans?

They can be integral member proteins



They bind with fibroblast growth factor



They bind leukocyte surface markers



They modulate cell growth

 36. Which of the following growth factors comes from


lymphocytes and fibroblasts, activates macrophages, and inhibits
fibroblast proliferation?

Tumor Nectosis Factor (TNF)



Transforming Growth Factor-β (TGF-β)



Keratinocyte Growth Factor (KGF)



Interferons

 37. Which of the following is NOT a cell cycle checkpoint?



Restriction checkpoint



G0 checkpoint



G2 checkpoint



Anaphase checkpoint

 38. Which of the following is characterized by the production of


large amounts of pus consisting of neutrophils, necrotic cells, and
edema fluid?

Serous inflammation



Fibrinous inflammation



Suppurative or purulent inflammation



Ulcers

 39. Which of the following signal transduction pathways includes


cytokines such as interleukin-2 (IL-2)?

Receptors with intrinsic tyrosine kinase activity



Receptors lacking intrinsic tyrosine kinase activity that recruit kinases



Seven transmembrane G-protein-coupled receptors (GPCRs)



Steroid hormone receptors


 40. Which of the following is associated with the cAMP signal
transduction pathway?

Receptors with intrinsic tyrosine kinase activity



Receptors lacking intrinsic tyrosine kinase activity that recruit kinases



Seven transmembrane G-protein-coupled receptors (GPCRs)



Steroid hormone receptors

 41. Which of the following growth factors comes from


macrophages, mast cells, and T cells and functions in
angiogenesis, wound repair, and hematopoiesis?

Epidermal Growth Factor (EGF)



Transforming Growth Factor-α (TGF-α)



Vascular Endothelial Growth Factor (VEGF)



Fibroblast Growth Factor (FGF)

 42. Which part of the cell cycle has the most redundancies, is
tightly regulated by proteins called cyclins, and associated
enzymes called cyclin-dependent kinases (CDKs)?

Between G0 and G1



Between G1 and S



Between S and G2


Between G2 and M

 43. Which of the following families of CAMs have broader ligand


specificity and are responsible for many events involving cell
adhesion?

Immunoglobulin



Cadherins



Integrins



Selectins

 44. Which of the following is NOT true regarding regeneration of


mammalian tissue?

Quiescent cells such as cardiac myotubes reenter the cell cycle



Stem cells in the area of injury differentiate efficiently



There is a rapid fibroproliferative response and scar formation after
wounding



Regeneration is complete (not compensatory growth)

 45. Which of the following growth factors comes from


mesenchymal cells and functions to replicate hepatocytes?

Epidermal Growth Factor (EGF)



Transforming Growth Factor-α (TGF-α)



Hepatocyte Growth Factor (HGF)



Vascular Endothelial Growth Factor (VEGF)

 46. Which of the following describes the JAK/STAT signal


transduction pathway?

Receptors with intrinsic tyrosine kinase activity



Receptors lacking intrinsic tyrosine kinase activity that recruit kinases



Seven transmembrane G-protein-coupled receptors (GPCRs)



Steroid hormone receptors

 47. Which of the following growth factors comes from


mesenchymal cells and increases vascular permeability and
vasculogenesis?

Epidermal Growth Factor (EGF)



Transforming Growth Factor-α (TGF-α)



Hepatocyte Growth Factor (HGF)



Vascular Endothelial Growth Factor (VEGF)


 48. Which of the following signaling modes uses blood vessels
and is associated with several cytokines?

Autocrine signaling



Paracrine signaling



Endocrine signaling



None of the above

 49. Cytokine activation of endothelium (with neutrophils) involves


which of the following?

IL-1 and IL-2



IL-3



IL-4 and IL-5



IL-1 and TNF

 50. A patient presents with an infection of the hand and red streaks
along the arm and into the axilla, with painful nodules in the axilla.
This is due to a secondary inflammation of the , which in flow due
to the infection.

Blood; Increased



Blood; Decreased



Lymph; Increased


Lymph; Decreased

Pathoanatomy - Miscellaneous: free radical injury: stains

Total questions: 23; Correctly answered: 6; Percentage of


correct: 26.1% Skipped question: 0;
correct answer
incorrect answer
skipped question

 1. All are components of basement membrane except



Nidogen



Laminin



Entactin



Rhodopsin

 2. In an evaluation of a 7-year-old boy, Ram who has had recurrent


infections since the first year of life, findings include enlargement
of the liver and spleen, lymph node inflammation, and a superficial
dermatitis resembling eczema. Microscopic examinatio

Defect in the enzyme NADPH oxidase



Defect in the enzyme adenosine deaminase (ADA)



Defect in the IL-2 receptor



Developmental defect at the pre-B stage


 3. PAs stains the following except

Glycogen



Lipids



Fungal cell wall



Basement membrane of bacteria

 4. Stain not used for lipid



Oil red O



Congo red



Sudan III



Sudan black

 5. Neutrophil secretes

Superoxide dismutase



Myeloperoxidase



Lysosomal enzyme



Catalase


 6. Which of the following is a negative stain?

Fontana



ZN stain



Nigrosin



Albert stain

 7. Which of the following is the most common fixative used in


electron microscopy?

Glutaraldehyde



Formalin



Picric acid



Absolute Alcohol

 8. Which of the following pigments are involved in free radical


injury?

Lipofuscin



Melanin



Bilirubin



Hematin

 9. An autopsy is performed on a 65-year-old man, suresh who died


of congestive heart failure. sections of the liver reveal
yellow-brown granules in the cytoplasm of most of the
hepatocytes. Which of the following stains would be most useful
to demonstrate w

Oil red O stain



Oil red O stain



Periodic acid- Schiff stain



H&E Stain

 10. The fixative used in histopathology



10% buffered neutral formalin



Bouins fixative



Glutaraldehyde



Ethyl alcohol

 11. lipid in the tissue is detected by:



PAS



Myeloperoxidase


Oil Red O



Mucicarmine

 12. Which is the most commonly used fixative in histopathological


specimens?

Glutaraldehyde



Formaldehyde



Alcohol



Picric acid

 13. Increased incidence of cancer in old age is due to



Telomerase reactivation



Telomerase deactivation



Inactivation of protooncogene



Increase in apoptosis

 14. Which process makes the bacteria



Margination



Diapedesis



Opsonisation



Chemotaxis

 15. True about cell ageing:



Free radicals injury



Mitochondria are increased



Lipofuscin accumulation in the cell



Size of cell increased

 16. Which of the following statements about Telomerase is true?



Has RNA polymerase activity



Causes carcinogenesis



Present in somatic cells



Absent in germ cells

 17. Stain used for melanin is



Oil red


Gomori methamine silver stain



Masson fontana stain



PAS stain

 18. Which of the following is a peroxisomal free radical


scavenger?

Superoxide dismutase



Glutathione peroxidase



Catalase



All of the above

 19. An AIDs patient Khalil develops symptoms of pneumonia, and


Pneumocystis carinii is suspected as the causative organism.
bronchial lavage is performed. Which of the following stains would
be most helpful in demonstrating the organism

Alcian blue



Hematoxylin and eosin



Methenamine silver



Trichrome stain

 20. The most abundant glycoprotein present in basement


membrane is:

Laminin



Fibronectin



Collagen type 4



Heparan sulphate

 21. Acridine orange is a fluorescent dye used to bind



DNA and RNA



Protein



Lipid



Carbohydrates

 22. Crooke’s hyaline body is present in:



Yellow fever



Basophil cells of the pituitary gland in Cushing



Parkinsonism



Huntington

 23. Enzyme that protects the brain from free radical injury is:

Myeloperoxidase



Superoxide dismutase



MAO



Hydroxylase

Pathoanatomy - Musculoskeletal system

Total questions: 38; Correctly answered: 7; Percentage of


correct: 18.4% Skipped question: 0;
correct answer
incorrect answer
skipped question

 1. Most common malignant bone tumor



Osteogenic sarcoma



Secondaries



Osteoma



Enchondroma

 2. A 9-year-old girl has difficulty in combing hairs and climbing


upstairs since 6 months She has Gower

ESR



RA factor



Creatine kinase



Electromyography

 3. Ewings sarcoma arises from:



G cells



Totipotent cells



Neuroectodermal cells



Neurons

 4. The rate of newly synthesized osteoid mineralization is best


estimated by

Tetracycline labeling



Alizarine red staining



Calceine stain



Von Kossa stain


 5. A 10 year old girl presents with a tibial mass Histopathological
examination reveals a small round cell tumor Which of the
following molecular findings is most likely to be present?

22q translocation



11q deletion



7p translocation



N-myc amplification

 6. Polyarticular rheumatoid arthritis is diagnosed when more than


_ joints are involved?

Two



Three



Four



Five

 7. Osteoclast are stimulated by:



Thyroxine



PTH



Calcitonin



Estrogen

 8. The commonest malignant bone tumor is:



Multiple myeloma



Osteosarcoma



Ewing



Giant cell tumor

 9. CD-99 is for:

Ewing



SLL



Dermatofibroma protruberans



Malignant histiocytic fibroma

 10. MIC-2 mutation associated with:



Osteosarcoma



Ewing sarcoma



Alveolar soft tissue sarcoma


Dermatofibrosarcoma protuberance

 11. Tophi in gout are found in all regions, except:



Bone



Skin



Muscle



Synovial membrane

 12. Bone resorption markers are all except:



Tartarate resistant alkaline phosphatase



Osteocalcin



Crosslinked-N-telopeptides



Urine total free deoxypyridinoline

 13. Bone tumor arising from epiphysis is:



Osteogenic sarcoma



Ewing



Chondromyxoid fibroma


Giant cell tumor

 14. Antibody found in myositis is?



Anti-Jo 1



Anti scl 70



Anti Sm



Anti Ku

 15. Which of the following is true about psoriatic arthritis?



Involves distal joints of hand and foot



Pencil in cup deformity



Sacroiliitis



All of the above

 16. Large intracytoplasmic glycogen storage is seen in which


malignancy?

osteosarcoma



Mesenchymal chondrosarcoma



Ewing



Leiomyosarcoma

 17. Dystrophin is lacking in:



Polio



Duchenne



Peroneal muscular atrophy



Spinal muscular atrophy

 18. Ground glass appearance is found in:



Inverted papilloma



Fibro calcification



Fibrous dysplasia of bones



Chronic osteomyelitis

 19. Which of the following is false in relation to Osteosarcoma?



Paget



Rb gene mutation is associated with hereditary variant


C-myc gene implicated in the genesis



Codman

 20. A 42 year-old woman Paro presents with slowly progressive


syndrome comprising of features like pain and tenderness in
multiple joints, with joint stuffiness on rising in the morning Joint
involvement is symmetric, with the proximal interphalangeal and
me

Antibodies to double-stranded DNA



IgM anti-IgG antibodies



HLA-B27 antigen



Urate crystals and neutrophils in synovial fluid

 21. Giant cells are seen in:



Osteoclastoma



Chondroblastoma



Chordoma



Osteitis fibrosa cystica

 22. Osteoblastoma resembles histologically:



Osteosarcoma



Osteoid osteoma



Chondroblastoma



Chondrosarcoma

 23. Syncytial osteoclastic giant cells are seen in All Except:



Osteosarcoma



Ewing



Chondroblastoma



Aneurysmal bone cyst

 24. A 50-year-old lady presented with a 3-month history of pain in


the lower third of the right thigh There was no local swelling;
tenderness was present on deep pressure Plain X-rays showed an
ill-defined intramedullary lesion with blotchy calcification at

Focal necrosis and lobulation



Tumor permeation between bone trabeculae at periphery



Extensive myxoid change



High cellularity

 25. Hyaline cartilage contains which type of collagen:



Type I



Type II



Type III



Type IV

 26. Which of the following is the most specific test for rheumatoid
arthritis?

Anti Ig M antibody



Anti CCP antibody



Anti Ig A antibody



Anti IgG antibody

 27. Paget disease of the bone is also called:



Osteitis fibrosa



Brittle bone disease



Fibrous dysplasia



Osteomalacia

 28. Which of these is characteristic of Gout?



Podagra



Anasarca



Cheiroarthropathy



Calcinosis cutis

 29. Dystrophic gene mutation leads to:



Myasthenia gravis



Motor neuron disease



Poliomyelitis



Duchenne

 30. Which one of the following inflammatory markers of muscle


biopsy is diagnostic of polymyositis?

CD8/MHC-I complex



Vascular cell adhesion molecules



Intracellular adhesion molecules


Membrane attack complex

 31. Onion bulb appearance of nerve ending on biopsy is seen in



Diabetic neuropathy



Amyloid neuropathy



Leprous neuritis



Chronic inflammatory demyelinating polyneuropathy (CIDP)

 32. Mosaic pattern of lamellar bone histology is found in:



Osteopetrosis



Osteoid osteoma



Osteitis deformans



Osteomalacia

 33. Characteristics microscopic features of osteogenic sarcoma


is:

Osteoid formation



Osteoid formation by mesenchymal cells with pleomorphism



Codman



Predominant osteoclast

 34. An epiphyseal bone lesion is:



Osteogenic sarcoma



Chondroblastoma



Ewing



Chondromyxoid fibroma

 35. Cytogenetics for synovial cell sarcoma is:



t (X: 18)



t (17, 9)



t (9, 22)



t (11, 14)

 36. Paget’s disease increases the risk of:



Osteoma



Osteosarcoma


Fibrosarcoma



All

 37. In the giant cell tumor of the bone, the cell of origin is:

Fibroblast cells



Osteoclast and precursors



Osteoblast and precursors



Sinusoidal cells

 38. Biphasic pattern on histology is seen in which tumor?



Rhabdomyosarcoma



Synovial cell sarcoma



Osteosarcoma



Neurofibroma

Pathoanatomy - Myeloproliferative disordrs

Total questions: 13; Correctly answered: 3; Percentage of


correct: 23.1% Skipped question: 0;
correct answer
incorrect answer
skipped question

 1. Splenomegaly is associated with all except:



CML



Polycythemia vera



Essential thrombocythemia



Primary myelofibrosis

 2. An old man Durga Prasad presents to you with complaints of


fatigue, weight loss, night sweats, and abdominal

Hodgkin lymphoma



Extramedullary hematopoiesis



Portal hypertension



Metastatic adenocarcinoma

 3. One of the following is not a myelo-proliferative disorder?



Essential thrombocytosis



Myelofibrosis with myeloid metaplasia



Acute myeloblastic leukemia



Chronic myeloid leukemia

 4. Increase in alkaline phosphatase is seen in:



Chronic myeloid leukemia; CML



Leukemoid reaction



Eosinophilia



Malaria

 5. Polycythemia is absolute venous haematocrit of more than:



45%



55%



65%



70%

 6. Which of the following is not a chronic myeloproliferative


disorder?

Polycythemia vera



Myeloid metaplasia



CML



Essential thrombocytopenia

 7. CD marker of histiocytosis is:



CD 1a



CD 1b



CD 1c



CD 1d

 8. Leucocyte alkaline phosphatase (LAP) is raised in all conditions


except:

Myelofibrosis



Essential thrombocythemia



Chronic myeloid leukemia



Polycythemia

 9. Leukoerythroblastic reaction is seen in the following except:



Secondaries in bone



Multiple myeloma


Hemolytic anemia



Lymphoma

 10. Essential criteria for polycythemia vera according to WHO is:



Low EPO



JAK 2 mutation



Bone marrow showing panmyelosis



MPL point mutation

 11. Isolated deletion of which chromosome is associated with


myelodysplastic syndrome?

2q



5q



8q



11q

 12. Shape of Birbeck granules is which of the following?



Hockey stick



Bat



Ball



Tennis racket

 13. A child presents with seborrheic dermatitis, sinusitis and


chronically draining ears. On examination child has failure to
thrive with hepato-splenomegaly and exophthalmos. Probable
diagnosis is

Histiocytosis-X



Wegener



Chronic grnaulomatous disease



Chediak higashi syndrome

Pathoanatomy - Non hodgkin lymphoma

Total questions: 33; Correctly answered: 12; Percentage of


correct: 36.4% Skipped question: 0;
correct answer
incorrect answer
skipped question

 1. Over-expression of BCL-2 proteins occurs in



Burkitt



Follicular lymphoma



Diffuse large B-cell lymphoma



Small lymphocytic lymphoma

 2. Which of the following statements on lymphoma is not true?



A single classification system for Hodgkin



HD more often tends to remain localized to a single group of lymph
nodes and spreads by contiguity



Several types of non Hodgkin



In general follicular (nodular) NHL has worse prognosis compared to
diffuse NHL

 3. Which of the following is false?



Bcl-6 is associated with Burkitts lymphoma



Bcl-2 is associated with follicular lymphoma



CD-10 is associated with mantle cell lymphoma



CD 34 is associated with Diffuse large B Cell Lymphoma

 4. A 48 year old woman was admitted with a history of weakness


for two months. On examination, cervical lymph nodes were found
enlarged and spleen was palpable 2 cm below the costal margin.
Her hemoglobin was 10.5 g/dl, platelet count 2.7

A pseudofollicular pattern with proliferation centers


A monomorphic lymphoid proliferation with a nodular pattern



A predominantly follicular pattern



A diffuse proliferation of medium to large lymphoid cells with high mitotic
rate

 5. Which one of the following Non-Hodgkin Lymphomas is


aggressive?

Follicular Lymphoma



Burkitt Lymphoma



Small lymphocytic lymphoma



Lymphoplasmacytic lymphoma

 6. Which of the following is the most common non Hodgkin


lymphoma?

Follicular lymphoma



Anaplastic large cell lymphoma



Diffuse large B cell lymphoma



Marginal zone lymphoma


 7. True about Burkitt

CD 34 and surface Ig both +ve



CD 34 negative but surface Ig+



CD 34 positive but surface Ig -



CD 34 and surface Ig both

 8. Cell of origin of hairy cell leukemia is



T cell



B cell



NK cell



Dendritic cell

 9. A 50 years old male presents with massive splenomegaly. His


differential diagnosis will include all, except:

Chronic myeloid leukemia



Polycythemia rubra vera



Hairy cell leukemia



Aplastic anemia

 10. All are B cell lymphomas except



Burkitt



Mycosis fungoides



Mantle cell lymphoma



Follicular cell lymphoma

 11. An old man, Amarnath presents with increasing abdominal


discomfort, fatigue and easy bruising of his skin. He also has
tender splenomegaly. Laboratory investigations show:
Hemoglobin 7.9 g/dl Platelet count 35,000/mm3 WBC count
2500/mm3 Serum AST 77 U/L S

Acute myelogenous leukemia



Cirrhosis



Systemic lupus erythematosus



Infectious mononucleosis

 12. Mycosis fungoides is:



Fungal infections of skin



Leukemia



Exfoliative erythroderma



Cutaneous lymphoma

 13. True statement regarding non Hodgkin



Increased incidence in adolescents



Predominantly in males



Prognosis is better than in diffuse type



Affects T cells only

 14. Most common Non-Hodgkin



B cell



T cell



NK cell



Plasma cell

 15. Which of the following is the most common site for extranodal
lymphoma?

Esophagus



Stomach


Intestine



Skin

 16. Eosinophilic Abscess in lymph node is characteristically seen


in:

Kimura's disease



Hodgkin disease



Tuberculosis



Sarcoidosis

 17. The classification proposed by the International Lymphoma


Study Group for non-Hodgkin lymphoma is

Kiel classification



REAL classification



WHO classification



Rappaport classification

 18. Which of the following is the marker of mantle cell cancer?



CD5 +, CD25


CD 5 +, CD 10 +



CD 5 +, CD 23 +



CD 5 +, CD 23 -

 19. All of the following immunohistochemical markers are positive


in the neoplastic cells of granulocytic sarcoma, except:

CD 45 RO



CD 43



Myeloperoxidase



Lysozyme

 20. Progressive transformation of germinal centres (PTGC) is a


precursor lesion of:

Hodgkin's Lymphoma, nodular sclerosis



Kimura's disease



Anaplastic large cell Lymphoma



Peripheral T cell Lymphoma


 21. Marginal lymphoma is type of:

B cell lymphoma



T cell lymphoma



NK cell lymphoma



Hodgkin lymphoma

 22. Histological presence of



Anaplastic large cell lymphoma (ALK positive)



Familial Medullary Carcinoma



Familial Neuroblastoma



Lymphocyte predominance type Hodgkin

 23. The low grade non- Hodgkin



Follicular small cleaved lymphoma



Follicular large cell lymphoma



Diffuse large cell lymphoma



Lymphoblastic lymphoma


 24. Burkitt's lymphoma is associated with:

T (8:14)



T (9:22)



T (11; 14)



T (8:21)

 25. A four year old boy was admitted with a history of abdominal
pain and fever for two months, maculopapular rash for ten days,
and dry cough, dyspnea and wheezing for three days. On
examination, liver and spleen were enlarged 4 cm and 3 cm
respectively belo

Eosinophils are not part of the neoplastic clone



T (5:14) rearrangement may be detected in blasts



Peripheral blood eosinophilia may normalize with chemotherapy



Inv (16) is often detected in the blasts and the eosinophil

 26. Molecular studies on an abdominal lymph node containing


lymphoma demonstrate (2;8)(p12;q24) translocation. This is most
compatible with which of the following diseases?

Burkitt's lymphoma



Mantle cell lymphoma



Multiple myeloma



Small cell lymphoma

 27. A 21-year-old male Imraan Hashmi with fatigue, recurrent fever,


and enlarged cervical lymph nodes has numerous atypical
lymphocytes in his peripheral blood smear. Lymph node biopsy
shows expansion of lymphoid follicles with preservation of the
underlying

AIDS



Burkitt



Hodgkin



Infectious mononucleosis

 28. Post transplant lymphoma occurs due to proliferation of which


of the following cells

T-cell



B-cell



NK cell



Monocyte

 29. Prevalence of burkitt lymphoma is highest in?



Australia


Africa



Asia



America

 30. Mantle cell lymphomas are positive for all of the following
except:

CD23



CD20



CD5



Cyclin D1

 31. MALToma is:



B-cell lymphoma



APUDoma



NK cell tumor



T cell lymphoma


 32. Mantle cell lymphomas are positive for all of the following,
except:

CD 23



CD 20



CD 5



CD 43

 33. All of the following statements about hairy cell leukemia are
true except:

Splenomegaly is conspicuous



Results from an expansion of neoplastic T lymphocytes



Cells are positive for Tartarate Resistant Acid phosphatase



The cells express CD25 consistently


Pathoanatomy - Obstructive lung disease: bronchitis, asthma,
bronchiectasis, emphysema

Total questions: 15; Correctly answered: 3; Percentage of


correct: 20.0% Skipped question: 0;
correct answer
incorrect answer
skipped question

 1. Creola bodies are seen in:



Bronchial asthma



Chronic bronchitis



Emphysema



Bronchiectatsis

 2. Increased Reid s index is increased in which of the following?



Bronchiectasis



Bronchial asthma



Chronic bronchitis



Emphysema

 3. A 30 year old woman Chinamma has had increasing dyspnea


with cough for the past week Over the past 2 days she is having
productive cough with copious sputum On examination, she is
afebrile but has extensive dullness to percussion over all the lung
field

CFTR



Granulocyte



DNA topoisomerase 1



Glomerular basement membrane


 4. Charcot-Leyden crystals and Curschmann s spirals are seen in:

Bronchial asthma



Chronic bronchitis



Bronchiectasis



Emphysema

 5. A 65-year-old smoker Sutta Ram with hemoptysis and weight


loss undergoes a left upper lobectomy for squamous cell
carcinoma The uninvolved lung tissue shows destruction of the
alveolar septae around the respiratory bronchioles, with marked
enlargement of

Asthma



Chronic bronchitis



Emphysema



Pulmonary hypertension

 6. Late response in bronchial asthma is due to:



Mast cells



Eosinophils



Neutrophils


Macrophages

 7. In a heavy smoker with chronic bronchiolitis, which of the


following is likely to be seen:

Centrilobular emphysema



Panacinar emphysema



Paraseptal emphysema



None of the above

 8. A 50 year old man Shahid K John has had increasing dyspnea


for the past 3 years with associated occasional cough but little
sputum production Auscultation reveals that his lungs are
hyper-resonant and is associated with expiratory wheeze
Pulmonary func

Sarcoidosis



Centriacinar emphysema



Diffuse alveolar damage



Chronic pulmonary embolism

 9. In emphysema, the destruction of many alveolar walls changes


the compliance of the respiratory system Which of the following
clinical observations is directly related to this change in
compliance?

Barrel chest



Chronic cough



Pink face



Long, slow, deep breathing pattern

 10. Thickening of pulmonary membrane is seen in:



Asthma



Emphysema



Bronchitis



Bronchiectasis

 11. Most common type of emphysema clinically is:



Panacinar



Centriacinar



Paraseptal



Segmental


 12. A 37 year old male Ranjir Kapoor presents to the hospital with
progressive exertional dyspnea His symptoms began insidiously
but have progressed gradually to the extent that now he has a
problem even in his daily activities Dr Gulaeria, a respiratory m

Panacinar emphysema



Centriacinar emphysema



Compensatory hyperinflation



Apical subpleural blebs

 13. A 52-year-old male smoker Naresh presents with fever and a


cough productive of greenish-yellow sputum The patient states
that he has had a morning cough with excessive mucus
production for the past 5 years Which of the following
abnormalities would most

Apical cavitary lesions on x-ray



Curschmann spirals in his sputum



Increased Reid index



Enlarged hilar lymph nodes on x-ray

 14. Alpha-1-antitrypsin deficiency occurs in:



Emphysem



Bronchiectasis



Empyema



Bronchogenic carcinoma

 15. True about alpha-1 antitrypsin deficiency, is/are -



Autosomal dominant



Pulmonary emphysema



Diastase resistant hepatic cell



Hepatic cells are orcein stain positive


Pathoanatomy - Pancreas

Total questions: 12; Correctly answered: 6; Percentage of


correct: 50.0% Skipped question: 0;
correct answer
incorrect answer
skipped question

 1. A 62 year-old woman Omvati with advanced, metastatic lung


cancer develops profound fatigue and weakness and alternating
diarrhea and constipation. Physical examination demonstrates
hyperpigmentation of skin, even in areas protected from the sun.
Tumor inv

Adrenal gland



Endocrine pancreas



Ovaries



Pituitary gland

 2. Which of the following is used to measure control of blood


sugar in diabetes mellitus?

HbA



HbS



HbA2



HbA 1C

 3. Two diabetic patients are seen by an endocrinologist, Dr. Saket.


The first patient is a 16-year-old boy Raju who 2 years previously
had presented with polyuria and polydipsia. The second patient is
a 65-year-old woman Antara whose diabetes was identified

Not have the HLA-DR3 or HLA-DR4 allele



Become euglycemic with oral hypoglycemic agents



Develop ketoacidosis



Have relatively high endogenous insulin levels

 4. All statements are true about Nesidioblastosis except?



Hypoglycemic episodes are seen



Occurs more commonly in adults than in children


Histopathology shows hyperplasia of Islet cells



Diazoxide is used in treatment

 5. Diabetes is diagnosed by which of the following criteria?



The level of fasting glucose is = 100 mg/dL and that of postprandial
glucose is = 140 mg/dL



The level of fasting glucose is > 125 mg/dL and that of post prandial
glucose is > 199 mg/dL



The level of plasma insulin is = 6 IU/dL



The HbA1c level is = 5.5%

 6. Insulin increases glucose entry into skeletal muscle, adipose


tissues and liver cells by:

Increasing the number of glucose transporter GLUT2 in all these tissues



Increasing the number of GLUT4 in muscle and adipose tissue and
glucokinase in liver cells



Increasing the number of GLUT3 in skeletal muscle and adipose tissues
and GLUT4 in liver cells



Increasing the number of GLUT1 in muscle, GLUT3 in adipose tissues
and GLUT4 in liver cells


 7. The term fetal adenoma is used for:

Hepatoma liver



Fibroadenoma breast



Follicular adenoma of thyroid



Craniopharyngioma

 8. Necrobiosis lipoidica is seen in



Diabetes insipidus



Lyme disease



Diabetes mellitus



Symmonds disease

 9. According to ADA guidelines, the diagnosis of diabetes is made


when the fasting blood glucose is more than

126 mg/dl



100 mg/dl



140 mg/dl



200 mg/dl

 10. Insulin resistance in liver disease is due to:



Decreased insulin release



Steatosis



Hepatocyte dysfunction



Decreased

 11. Mauriac's syndrome is characterized by all except



Diabetes



Obesity



Dwarfism



Cardiomegaly

 12. Amylin is secreted by which group of cells of pancreas?



Alpha cells



Beta cells



D cells



PP cells (Pancreatic polypeptide)

Pathoanatomy - Paraneoplastic syndromes, tumour markers, tumour


lysis syndrome

Total questions: 50; Correctly answered: 10; Percentage of


correct: 20.0% Skipped question: 0;
correct answer
incorrect answer
skipped question

 1. Secondaries are common in all, except:



Skull



Hand and feet bones



Proximal limb bones



Pelvic

 2. Popcorn calcification is seen in:



Chondrosarcoma



Fibrous dysplasia



Osteoblastoma



Wilms’ tumor

 3. Which of the following tumors have an increased elevation of


placental alkaline phosphatase in the serum as well as a positive
immunohistochemical staining for placental alkaline
phosphatase?

Seminoma



Hepatoblastoma



Hepatocellular carcinoma



Peripheral neuroectodermal tumor

 4. Which of the following is not true about Neuroblastoma?



Most common extracranial solid tumor in childhood



>50% patients present with metastasis at time of diagnosis



Lung metastases are common



Involve aorta and its branches early

 5. An old man Velu presents with complaints of abdominal and


back pain, malaise, nausea, 8 kg weight loss and weakness, which
have been present for 3 or 4 months. His history also reveals
several episodes of unilateral leg swelling, which have involved
both

Pancreatic cancer



Primary sclerosing cholangitis



Splenic infarction


Reflux esophagitis

 6. Alpha-fetoproteins are a marker of:



Secondaries in liver



Cholangiocarcinoma



Hepatoma



None of the above

 7. Hybridoma refers to

Collision tumor



A tumor of brown fat



A hamartoma



A technique for raising monoclonal antibodies

 8. Marker of small cell cancer of lung is:



Chromogranin



Cytokeratin



Desmin


Vimentin

 9. A 60-year-old man, Shibu is found to have a 3.5-cm mass in the


right upper lobe of his lung. A biopsy of this mass is diagnosed as
a moderately differentiated squamous cell carcinoma. Workup
reveals that no bone metastases are present, but laboratory exam

Parathyroid hormone



Parathyroid hormone-related peptide



Calcitonin



Calcitonin-related peptide

 10. Which of the following is tumor marker of seminoma?



AFP



LDH



PLAP



HCG

 11. CA·125 is associated with:



Colon ca



Breast ca


Ovarian ca



Bronchogenic ca

 12. AFP is a marker of:



Hepatoblastoma



Seminoma



Sertoli-Leydig cell tumor



Choriocarcinoma

 13. A 65 years old male diagnosed by biopsy a case of lung


carcinoma, with paraneoplastic syndrome and increased calcium.
Probable cause is

Parathyroid hormone



Parathyroid hormone related peptide



Calcitonin



Calcitonin related peptide

 14. A 20 year old female was diagnosed with granulose cell tumor
of the ovary. Which of the following bio markers would be most
useful for follow-up of patient?

CA 19-9



CA50



Inhibin



Neuron – specific enolase

 15. Spontaneous regression of tumor is seen in:



Wilm’s tumor



Neuroblastoma



Acute monocytic leukemia



Hepatoblastoma

 16. Alpha fetoprotein is a marker of:



Hepatoblastoma



Seminoma



Renal cell carcinoma



Choriocarcinoma


 17. All of the following are examples of tumor markers, except:

Alpha-HCG (a-HCG)



Alpha-Feto protein



Thyroglobulin



Beta 2-microglobulin

 18. True about Carcinoembryonic antigen (CEA):



Useful for screening of carcinoma colon



Gives confirmative evidence of Ca. colon



Helpful for follow-up after resection



Levels decrease immediately after resection of tumor

 19. Secondaries of all the following cause osteolytic lesions


except:

Prostate



Kidney



Bronchus



Thyroid

 20. For which one of the following tumors Gastrin is a biochemical


marker?

Medullary carcinoma of thyroid



Pancreatic neuroendocrine tumor



Pheochromocytoma



Gastrointestinal stromal tumor

 21. A 65-year-old woman Ramkali presents to the emergency room


with a pathologic fracture of the shaft of her humerus. X-ray
studies demonstrate multiple lytic and blastic bone lesions.
Biopsy of one of these lesions shows adenocarcinoma. Which of
the followi

Breast



Colon



Kidney



Lung

 22. Desmoid tumor arises from



Wall of the intestine



Anterior abdominal wall



Submucosa



Appendix

 23. Which of the following mutation is seen in malignant


melanoma

N-myc



CDKN2A



RET



Rb

 24. A 62 year-old woman Omvati with advanced, metastatic lung


cancer develops profound fatigue and weakness and alternating
diarrhea and constipation. Physical examination demonstrates
hyperpigmentation of skin, even in areas protected from the sun.
Tumor inv

Adrenal gland



Endocrine pancreas



Ovaries



Pituitary gland

 25. A-fetoprotein is seen in all except:



Hepatocellular carcinoma



Carcinoma colon



Pancreatic carcinoma



Germ cells of testes

 26. Migratory thrombophlebitis is seen in:



Disseminated cancer



Rheumatic heart disease



Libman-Sachs endocarditis



All of the above

 27. Which is associated with polycythemia:



Gastric carcinoma



Fibrosarcoma



Cerebellar hemangioblastoma



All


 28. Which of the following is incorrect about neuro- blastoma ?

Most common abdominal tumor in infants



X-ray abdomen shows calcification



Can show spontaneous regression



Urine contains 5H.I.A.A

 29. Which of the following tumors have an increased elevation of


placental alkaline phosphatase in the serum as well as a positive
immunohistochemical staining for placental alkaline
phosphatase?

Seminoma



Hepatoblastoma



Hepatocellular carcinoma



Peripheral neuroectodermal tumor

 30. Serum AFP is increased in all, except:



Acute hepatitis



Hepatocellular carcinoma



Hepatoma



Bladder carcinoma

 31. Which of the following is Not associated with thymoma?



SIADH



Myasthenia gravis



Polymyositis



Hypogammaglobinemia

 32. Carcinoembryonic antigen is elevated in all, except:



Alcoholic cirrhosis



Ca colon



Ulcerative colitis



Emphysema

 33. Alpha-fetoprotein is a tumor marker of:



Sertoli-Leydig cell tumor



Liver malignancies



Renal cell carcinoma


Choriocarcinoma

 34. Tumor that follows rule of 10 is:



Pheochromocytoma



Oncocytoma



Lymphoma



Renal cell carcinoma

 35. Which of the followingis a squamous cell carcinoma marker?



Vimentin



Desmin



Cytokeratin



Glial fibrillary acid protein

 36. The diagnostic tumor marker of liver carcinoma is:



CEA



AFP



CA - 125


All of the above

 37. All of the following about tumor markers are properly matched,
except:

Prostate cancer - PSA



Colon cancer - CEA



Ovarian cancer – CA 125



Cholangiocarcinoma - AFP

 38. Which of the following is a special stain for


rhabdomyosarcoma?

Cytokeratin



Synaptophysin



Desmin



Myeloperoxidase

 39. An undifferentiated malignant tumor on immunohistochemical


stain shows cytoplasmic positivity of most of the tumor cells for
cytokeratin. The most probable diagnosis of the tumor is:

Sarcoma



Lymphoma



Carcinoma



Malignant melanoma

 40. During a routine physical examination, a 45-year-old woman


Nusheen is noted to have a ruddy complexion. Her hematocrit is
52%. Her lungs are clear and she does not smoke. Serum
erythropoietin levels are elevated. Cancer of which of the following
organs is

Breast



Colon



Kidney



Stomach

 41. HMB 45 is a tumor marker for:



Neuroblastoma



Neurofibroma



Malignant melanoma



Angiosarcoma


 42. Uses of tumor marker are:

Screening of a cancer



Follow up of a cancer patient, esp. for knowing about recurrence



Confirmation of a diagnosed cancer



For monitoring the treatment of a cancer

 43. In tumor lysis syndrome, all of the following are seen, except:

Hypernatremia



Hypercalcemia



Hyperkalemia



Hyperphosphatemia

 44. BCL2 is a marker for:



Follicular lymphoma



Mycosis fungoides



B-cell lymphoma



Mantle cell lymphoma


 45. Marker for ovarian carcinoma in serum is:

CA-125



Fibronectin



Acid Phosphatase



PSA

 46. Sacrococcygeal teratoma, marker is:



CEA



β- HCG



S100



CA-125

 47. Increased level of alpha fetoprotein is found in



Yolk sac tumor



Seminoma



Teratoma



Choriocarcinoma


 48. Migratory thrombophlebitis is associated with all of the
following malignancies, except:

Prostate



Lung



GIT



Pancreas

 49. Hyperglycemia associated with:



Multiple myeloma



Ewing sarcoma



Osteosarcoma



Chondroblastoma

 50. The most common cause of malignant adrenal mass is



Adrenocortical carcinoma



Malignant Phaeochromocytoma



Lymphoma



Metastasis from another solid tissue tumor

Pathoanatomy - Paraneoplastic syndromes, tumour markers, tumour


lysis syndrome

Total questions: 50; Correctly answered: 16; Percentage of


correct: 32.0% Skipped question: 0;
correct answer
incorrect answer
skipped question

 1. Spontaneous regression of tumor is seen in:



Wilm’s tumor



Neuroblastoma



Acute monocytic leukemia



Hepatoblastoma

 2. Secondaries are common in all, except:



Skull



Hand and feet bones



Proximal limb bones



Pelvic

 3. Desmoid tumor arises from



Wall of the intestine


Anterior abdominal wall



Submucosa



Appendix

 4. Carcinoembryonic antigen is elevated in all, except:



Alcoholic cirrhosis



Ca colon



Ulcerative colitis



Emphysema

 5. Tumor that follows rule of 10 is:



Pheochromocytoma



Oncocytoma



Lymphoma



Renal cell carcinoma

 6. Which of the following is a special stain for


rhabdomyosarcoma?

Cytokeratin



Synaptophysin



Desmin



Myeloperoxidase

 7. A 60-year-old man, Shibu is found to have a 3.5-cm mass in the


right upper lobe of his lung. A biopsy of this mass is diagnosed as
a moderately differentiated squamous cell carcinoma. Workup
reveals that no bone metastases are present, but laboratory exam

Parathyroid hormone



Parathyroid hormone-related peptide



Calcitonin



Calcitonin-related peptide

 8. A 65 years old male diagnosed by biopsy a case of lung


carcinoma, with paraneoplastic syndrome and increased calcium.
Probable cause is

Parathyroid hormone



Parathyroid hormone related peptide



Calcitonin



Calcitonin related peptide

 9. Marker for ovarian carcinoma in serum is:



CA-125



Fibronectin



Acid Phosphatase



PSA

 10. CA·125 is associated with:



Colon ca



Breast ca



Ovarian ca



Bronchogenic ca

 11. Which of the followingis a squamous cell carcinoma marker?



Vimentin



Desmin



Cytokeratin


Glial fibrillary acid protein

 12. Alpha fetoprotein is a marker of:



Hepatoblastoma



Seminoma



Renal cell carcinoma



Choriocarcinoma

 13. Which of the following is not true about Neuroblastoma?



Most common extracranial solid tumor in childhood



>50% patients present with metastasis at time of diagnosis



Lung metastases are common



Involve aorta and its branches early

 14. An old man Velu presents with complaints of abdominal and


back pain, malaise, nausea, 8 kg weight loss and weakness, which
have been present for 3 or 4 months. His history also reveals
several episodes of unilateral leg swelling, which have involved
both

Pancreatic cancer



Primary sclerosing cholangitis



Splenic infarction



Reflux esophagitis

 15. A 62 year-old woman Omvati with advanced, metastatic lung


cancer develops profound fatigue and weakness and alternating
diarrhea and constipation. Physical examination demonstrates
hyperpigmentation of skin, even in areas protected from the sun.
Tumor inv

Adrenal gland



Endocrine pancreas



Ovaries



Pituitary gland

 16. Alpha-fetoprotein is a tumor marker of:



Sertoli-Leydig cell tumor



Liver malignancies



Renal cell carcinoma



Choriocarcinoma


 17. Which of the following mutation is seen in malignant
melanoma

N-myc



CDKN2A



RET



Rb

 18. A 65-year-old woman Ramkali presents to the emergency room


with a pathologic fracture of the shaft of her humerus. X-ray
studies demonstrate multiple lytic and blastic bone lesions.
Biopsy of one of these lesions shows adenocarcinoma. Which of
the followi

Breast



Colon



Kidney



Lung

 19. True about Carcinoembryonic antigen (CEA):



Useful for screening of carcinoma colon



Gives confirmative evidence of Ca. colon



Helpful for follow-up after resection


Levels decrease immediately after resection of tumor

 20. BCL2 is a marker for:



Follicular lymphoma



Mycosis fungoides



B-cell lymphoma



Mantle cell lymphoma

 21. HMB 45 is a tumor marker for:



Neuroblastoma



Neurofibroma



Malignant melanoma



Angiosarcoma

 22. AFP is a marker of:



Hepatoblastoma



Seminoma



Sertoli-Leydig cell tumor


Choriocarcinoma

 23. Which of the following is Not associated with thymoma?



SIADH



Myasthenia gravis



Polymyositis



Hypogammaglobinemia

 24. The most common cause of malignant adrenal mass is



Adrenocortical carcinoma



Malignant Phaeochromocytoma



Lymphoma



Metastasis from another solid tissue tumor

 25. Secondaries of all the following cause osteolytic lesions


except:

Prostate



Kidney



Bronchus



Thyroid

 26. Sacrococcygeal teratoma, marker is:



CEA



β- HCG



S100



CA-125

 27. A 20 year old female was diagnosed with granulose cell tumor
of the ovary. Which of the following bio markers would be most
useful for follow-up of patient?

CA 19-9



CA50



Inhibin



Neuron – specific enolase

 28. Popcorn calcification is seen in:



Chondrosarcoma



Fibrous dysplasia



Osteoblastoma



Wilms’ tumor

 29. Which of the following is incorrect about neuro- blastoma ?



Most common abdominal tumor in infants



X-ray abdomen shows calcification



Can show spontaneous regression



Urine contains 5H.I.A.A

 30. Hyperglycemia associated with:



Multiple myeloma



Ewing sarcoma



Osteosarcoma



Chondroblastoma

 31. For which one of the following tumors Gastrin is a biochemical


marker?

Medullary carcinoma of thyroid


Pancreatic neuroendocrine tumor



Pheochromocytoma



Gastrointestinal stromal tumor

 32. Which of the following is tumor marker of seminoma?



AFP



LDH



PLAP



HCG

 33. Serum AFP is increased in all, except:



Acute hepatitis



Hepatocellular carcinoma



Hepatoma



Bladder carcinoma

 34. Uses of tumor marker are:



Screening of a cancer


Follow up of a cancer patient, esp. for knowing about recurrence



Confirmation of a diagnosed cancer



For monitoring the treatment of a cancer

 35. All of the following are examples of tumor markers, except:



Alpha-HCG (a-HCG)



Alpha-Feto protein



Thyroglobulin



Beta 2-microglobulin

 36. Migratory thrombophlebitis is seen in:



Disseminated cancer



Rheumatic heart disease



Libman-Sachs endocarditis



All of the above

 37. Marker of small cell cancer of lung is:



Chromogranin


Cytokeratin



Desmin



Vimentin

 38. Migratory thrombophlebitis is associated with all of the


following malignancies, except:

Prostate



Lung



GIT



Pancreas

 39. Which of the following tumors have an increased elevation of


placental alkaline phosphatase in the serum as well as a positive
immunohistochemical staining for placental alkaline
phosphatase?

Seminoma



Hepatoblastoma



Hepatocellular carcinoma



Peripheral neuroectodermal tumor

 40. Hybridoma refers to



Collision tumor



A tumor of brown fat



A hamartoma



A technique for raising monoclonal antibodies

 41. Which is associated with polycythemia:



Gastric carcinoma



Fibrosarcoma



Cerebellar hemangioblastoma



All

 42. Increased level of alpha fetoprotein is found in



Yolk sac tumor



Seminoma



Teratoma



Choriocarcinoma

 43. All of the following about tumor markers are properly matched,
except:

Prostate cancer - PSA



Colon cancer - CEA



Ovarian cancer – CA 125



Cholangiocarcinoma - AFP

 44. The diagnostic tumor marker of liver carcinoma is:



CEA



AFP



CA - 125



All of the above

 45. A-fetoprotein is seen in all except:



Hepatocellular carcinoma



Carcinoma colon



Pancreatic carcinoma



Germ cells of testes

 46. During a routine physical examination, a 45-year-old woman


Nusheen is noted to have a ruddy complexion. Her hematocrit is
52%. Her lungs are clear and she does not smoke. Serum
erythropoietin levels are elevated. Cancer of which of the following
organs is

Breast



Colon



Kidney



Stomach

 47. An undifferentiated malignant tumor on immunohistochemical


stain shows cytoplasmic positivity of most of the tumor cells for
cytokeratin. The most probable diagnosis of the tumor is:

Sarcoma



Lymphoma



Carcinoma



Malignant melanoma

 48. Which of the following tumors have an increased elevation of


placental alkaline phosphatase in the serum as well as a positive
immunohistochemical staining for placental alkaline
phosphatase?

Seminoma


Hepatoblastoma



Hepatocellular carcinoma



Peripheral neuroectodermal tumor

 49. In tumor lysis syndrome, all of the following are seen, except:

Hypernatremia



Hypercalcemia



Hyperkalemia



Hyperphosphatemia

 50. Alpha-fetoproteins are a marker of:



Secondaries in liver



Cholangiocarcinoma



Hepatoma



None of the above

Pathoanatomy - Pedigree analysis, gene location, lyon hypothesis


Total questions: 35; Correctly answered: 14; Percentage of
correct: 40.0% Skipped question: 0;
correct answer
incorrect answer
skipped question

 1. Thalassemia occurs due to which mutation



Missense



Splicing



Transition



Frame-shift

 2. True statement about inheritance of an X linked recessive trait


is:

50% of boys of carrier mother are affected



50% of girls of diseased father are carrier



Father transmits disease to the son



Mother transmits the disease to the daughter

 3. Individual II-2 in the associated pedigree was diagnosed with


hemophilia A. Which one of the following individuals would be
most at risk for developing also hemophilia A?

II-3



II-6


III-3



III – 8

 4. Which one of the following is not a germ cell tumor?



Dermoid



Granulosa cell tumor



Choriocarcinoma



Gynandroblastoma

 5. An albino girl gets married to a normal boy, what are the


chances of their having an affected child and what are the chances
of their children being carriers?

None affected, all carriers



All normal



50% carriers



50% affected, 50% carriers

 6. Gene for major histocompatibility complex is located on which


chromosome?

Chromosome 10


Chromosome 6



X chromosome



Chromosome 13

 7. Cystic fibrosis transmembrane conductance regulator gene is


located on chromosome

5



6



7



8

 8. The gene that regulates normal morphogenesis during


development is:

FMR-1 gene



Homeobox gene



P-16



PTEN


 9. Study the following carefully: Read the pedigree. Inheritance
pattern of the disease in the family is:

Autosomal recessive type



Autosomal dominant type



X-linked dominant type



X-linked recessive type

 10. Gene for Wilm’s tumor is located on:



Chromosome 1



Chromosome 10



Chromosome 11



Chromosome 12

 11. Gene for folate carrier protein is located on chromosome?



Chromosome 10



Chromosome 5



Chromosome 21



Chromosome 9

 12. Gene therapy is used for:



Cystic fibrosis



Sickle cell anemia



Thalassemia



All of the above

 13. Males are more commonly affected than females in which of


the following genetic disorders?

Autosomal Recessive Disorder



Autosomal Dominant Disorder



X-linked Recessive Disorder



X-linked Dominant Disorder

 14. The technique used for separation and detection of RNA is


which one of the following

Northern blot



Southern blot



Eastern blot


Western blot

 15. A one year old boy presented with hepatosplenomegaly and


delayed milestones. The liver biopsy and bone marrow biopsy
revealed presence of histiocytes with PAS-positive
Diastase-resistant material in the cytoplasm. Electron-microscopic
examination of thes

Birbeck



Myelin figures in the cytoplasm



Parallel rays of tubular structures in lysosomes



Electron dense deposit in the mitochondria

 16. BRCA 1 gene is located on?



Chromosome 13



Chromosome 11



Chromosome 17



Chromosome 22

 17. A relatively recent advancement is the use of a technique


called DNA fingerprinting. It is based in part on recombinant DNA
technology and in part on those techniques originally used in
medical genetics to detect slight variations in the genomes of diffe

The banding pattern


The triplet code



The haplotypes



The chromosomes

 18. The chances of having an unaffected baby, when both parents


have achondroplasia, are:

0%



25%



50%



100%

 19. Male to male transmission is not seen in:



Autosomal dominant diseases



Autosomal recessive disease



X-linked dominant disease



Genomic imprinting


 20. A 22-year-old woman, Sheena presents with progressive
bilateral loss of central vision. You obtain a detailed family history
from this patient and produce the associated pedigree (dark
circles or squares indicate affected individuals). Which of the follo

Autosomal recessive



Autosomal dominant



X-linked recessive



Mitochondrial

 21. Congenital syndrome associated with lymphoproliferative


malignancy is:

Bloom syndrome



Fanconi



Turner syndrome



Chediak Higashi syndrome.

 22. Ability of stem cells to cross barrier of differentiation to


transform into a cell of another lineage expressing the molecular
characteristics of different cell type with the ability to perform the
function of the new cell type is referred as:

De differentiation



Re differentiation



Trans-differentiation



Sub differentiation

 23. Loss of heterozygosity means:



Loss of single arm of chromosome



Loss of mutant allele in mutant gene



Loss of normal allele in mutant gene



Loss of normal allele in normal gene

 24. A pathologist Dr. Gobind is examining four placentas from four


different b irths. He notes the following characteristics in the four
placentas: Patient A: fused dichorionic diamnionic Patient B:
dichorionic diamnionic Patient C: circumvallate placenta

Patient A



Patient B



Patient C



Patient D

 25. Microarray is best characterised by:



Study of multiple genes


Study of disease



Study of organisms



Study of blood group

 26. The mother has sickle cell disease; Father is normal; Chances
of children having sickle cell disease and sickle cell trait
respectively are:

0 and 100%



25 and 25%



50 and 50%



10 and 50%

 27. Long and short arm of chromosome are called respectively



P and q



Q and p



m and n



r and s


 28. Father has a blood group B; Mother has AB; Children are not
likely to have the following blood group:

O



A



B



AB

 29. The CFTR gene associated what cystic fibrosis is located on


chromosome

5



12



4



7

 30. In cystic fibrosis the most frequent pulmonary pathogen is



Pseudomonas



Enterococci



Staphylococci



Klebsiella

 31. In-situ DNA nick end labeling can quantitate:



Fraction of cells in apoptotic pathways



Fraction of cells in S phase



P53 gene product



Bcr/abl gene

 32. Kinky hair disease is a disorder where an affected child has


peculiar white stubby hair, does not grow, brain degeneration is
seen and dies by age of two years. Mrs. A is hesitant about having
children because her two sisters had sons who had died from ki

X-linked recessive



X-linked dominant



Autosomal recessive



Autosomal dominant

 33. A baby’s blood group was determined as O Rh negative. Select


the blood group the baby’s mother or father will not have

A, Rh positive



B, Rh positive


AB, Rh negative



O, Rh positive

 34. If both husband and wife are suffering with achondroplasia,


what are their chances of having a normal child

0%



25%



50%



100%

 35. Which of the following potentially represents the most


dangerous situation

Rh+ve mother with 2nd Rh-ve child



Rh-ve mother with 2nd Rh+ve child



Rh+ve mother with 1st Rh-ve child



Rh-ve mother with 1st Rh+ve child


Pathoanatomy - Pitutary

Total questions: 1; Correctly answered: 1; Percentage of


correct: 100.0% Skipped question: 0;
correct answer
incorrect answer
skipped question

 1. Which of the following is true about pituitary tumor?



It present in 10% of brain tumors.



Erodes the sella and extends into surrounding area



Prolactinoma is least common



It is differentiated by reticulin stain

Pathoanatomy - Plasma cell dyscrasias

Total questions: 20; Correctly answered: 3; Percentage of


correct: 15.0% Skipped question: 0;
correct answer
incorrect answer
skipped question

 1. M-spike in multiple myeloma is due to?



IgM



IgA



IgG



None of these

 2. Russell bodies are found in which of the following conditions?



Multiple Myeloma


Gonadal tumor



Parkinsonism



Intracranial neoplasms

 3. Proliferation and survival of myeloma cells are dependent on


which of the following cytokines?

IL-1



IL-6



IL-2



IL-5

 4. Hyperviscosity is seen in

Megaloblastic anemia



Multiple myeloma



MGUS



Lymphoma

 5. Plasma cell dyscrasias include all of the following except



Waldenstorm



Heavy chain disease



Monoclonal gammopathy of uncertain significance



Systemic lupus erythematosus

 6. An old woman is seen in the emergency after fracturing her hip.


She is found to have a malignant tumor. She also has a history of
recurrent pneumonia over the last 6 months. Lab investigations
reveal a normal white blood cell count, but decreased platelet

IgG heavy chains



Kappa and lambda light chains of a 60:40 ratio



Kappa light chains



Lambda light chains

 7. Birbeck's granule is found in



Langerhans cell



Langhans giant cell



Lepra cell



Clue cell

 8. Finding of multiple myeloma in kidney are all except:



Tubular casts



Amyloidosis



Wire loop lesions



Renal tubular necrosis

 9. Histiocytosis is NOT associated with



Spontaneous fractures



Cutaneous eruptions



Bone marrow suppression



No lymphadenopathy

 10. A 3 year old female child presented with skin papules. Which of
the following is a marker of Langerhan

CD 1a



CD 3



CD 68



CD 57

 11. True about Langerhan



CD 68+



CD 1+



Birbeck



Proliferation of antigen presenting cells

 12. Which histiocytosis involves the bones:



Malignant



Langherhans



Sinus histiocytosis



Option not recalled

 13. Which of the following metabolic abnormality is seen in


multiple myeloma?

Hypernatremia



Hypokalemia



Hypercalcemia


Hyperphosphatemia

 14. Lymphoplasmacytoid lymphoma is associated with:



IgG



IgA



IgD



IgM

 15. Which of the following statement is not true?



Patients with IgD myeloma may present with no evident M-spike on
serum electrophoresis



A diagnosis of plasma cell leukemia can be made if circulating
peripheral blood plasma blasts comprise 14% of peripheral blood white
cells in a patient with white blood cell count of 1



In smoldering myeloma plasma cells constitute 10- 30% of total bone
marrow cellularity



In a patient with multiple myeloma, a monoclonal light chain may be
detected in both serum and urine

 16. Beta-2 microglobulin is a tumor marker for



Multiple myeloma



Lung cancer



Colonic neoplasm



Choriocarcinoma

 17. Which of the following is the least common presentation of


multiple myeloma?

Anemia



Hyperviscosity



Bone pains



Infection

 18. A 70-year-old male has a pathologic fracture of femur. The


lesion appears a lytic on X-rays film with a circumscribed punched
out appearance. The curetting from fracture site is most likely to
show which of the following?

Diminished and thinned trabecular bone



Sheets of atypical plasma cells



Metastatic prostatic adenocarcinoma



Malignant cells forming osteoid bone


 19. Which of the following is not a minor diagnostic criterion for
multiple myeloma?

Lytic bone lesions



Plasmacytosis greater than 20%



Plasmacytoma on biopsy



Monoclonal globulin spike on serum electrophoresis of < 2.5 g/dl for IgG,
< 1.5 g/dl for IgA)

 20. A 62 year old man Kuljeet developed weakness, fatigue and


weight loss over the past 4 months. He also complains of
decreasing vision, headache and dizziness. His hands have
become sensitive to cold. On the general physical examination of
this individual,

Monoclonal IgM spike in serum



WBC count of 2,40,000/mm3



Hypercalcemia



Bence Jones proteinuria

Pathoanatomy - Platelets and bleeding disorders

Total questions: 36; Correctly answered: 11; Percentage of


correct: 30.6% Skipped question: 0;
correct answer
incorrect answer
skipped question
 1. A 28-year-old woman Salma presents complaining of
nosebleeds. She also has easy bruising and excessively heavy
bleeding during her periods. There is no history of drug intake.
Physical examination shows scattered petechiae with normal
sized spleen. Labora

Acetylcholine receptor



Erythrocyte membrane protein



Glycoprotein IIb/IIIa



Intrinsic factor

 2. A young female Vibhuti presents with a several month history of


easy bruising and increased menstrual flow is evaluated for a
bleeding disorder. She has idiopathic thrombocytopenic purpura
(ITP). In this disorder, the low platelet count is due to which of

Antiplatelet antibodies



Defective platelet aggregation



Hypersplenism



Ineffective megakaryopoiesis

 3. Agranulocytosis means:

Decrease in neutrophil count



Decrease in platelet count



Increase in RBC count



Decrease in RBC count

 4. Cryoprecipitate contain all except



Fibrinogen



Factor VIII



von Willebrand factor



Antithrombin

 5. In DIC, which is/are seen:



Normal aPTT



Increased PT



Increased factor VIII



Decreased fibrinogen

 6. All of the following are true about DIC except:



Platelet aggregation



Fibrin deposition in microcirculation


Decreased fibrin degradation products



Release of tissue factor

 7. Conditions associated with incoagulable state:



Abruption placentae



Acute promyelocytic leukemia



Severe falciparum malaria



Snake envenomation

 8. D.I.C. is seen in:



Acute promyelocytic leukemia



Acute myelomonocytic leukemia



CMC



Autoimmune hemolytic anemia

 9. Thrombocytopenia syndrome is caused by decrease in platelet


counts below:

50,000/cmm



1,00,000/cmm



1.2 lac/cmm



20, 000/cmm

 10. A 25 years old asymptomatic female underwent a preop


coagulation test. Her bleeding time is 3minutes, PT is 15/14sec, a
PTT is 45/35 sec, platelet count is 2.5 lac/ mm3 and factor VIII
levels were 60IU/dL. What is her most likely diagnosis?

Factor IX deficiency



Lupus anticoagulant



Factor VIII inhibitors



VWD

 11. Pancreatic insufficiency and cyclic neutropenia is a part of


which syndrome

Young syndrome



Colts syndrome



Shwachman syndrome



Roots syndrome


 12. A 9-year-old boy presents with elevation in both PT and aPTT.
What is the diagnosis?

Defect in extrinsic pathway



Defect in intrinsic pathway



Platelet function defect



Defect in common pathway

 13. All are true about thrombotic thrombocytopenic purpura


except?

Microangiopathic hemolytic anemia



Thrombocytopenia



Normal complement level



Grossly abnormal coagulation tests

 14. Patient with bleeding due to platelet function defects has which
of the following features?

Normal platelet count and normal bleeding time



Normal platelet count and increased bleeding time



Decreased platelet count and increased bleeding time



Normal platelet count and decreased bleeding time

 15. A 22 year old female having a family history of autoimmune


disease presents with the complaints of recurrent joint pains. She
has now developed petechial hemorrhages. She is most likely to
have which of the following disorders?

Megakaryocytic thrombocytopenia



Amegakaryocytic thrombocytopenia



Platelet function defects/Functional platelet defect



Acquired Factor VIII inhibitors

 16. A newborn baby presented with profuse bleeding from the


umbilical stump after birth. Rest of the examination and PT, APTT
are within normal limits. Most likely diagnosis is which of the
following?

Factor X deficiency



Glanzmann's thrombasthenia



Von Willebrand disease



Bernard Soulier disease

 17. All of the following clotting factors are completely synthesized


from liver except

II


V



VII



VIII

 18. A patient with cirrhosis of liver has the following coagulation


parameters, Platelet count 2,00,000, Prothrombin time 25s/12s,
Activated partial thromboplastin time 60s/35s, thrombin time
15s/15s. In this patient

D-dimer will be normal



Fibrinogen will be < 100 mg



ATIII will be high



Protein C will be elevated

 19. All the following statements are correct about treatment in


chronic immune thrombocytopenic purpura except

Most of the patients respond to immunosuppressive doses of
glucocorticoids



Relapse is rare



Splenectomy is the treatment of choice for relapse



Minority have refractory forms of ITP and difficult to treat

 20. All of the following can cause megakaryocytic


thrombocytopenia, except

Idiopathic thrombocytopenia purpura



Systemic lupus erythematosus



Aplastic anemia



Disseminated intravascular coagulation (DIC)

 21. The chromosomal translocation involving bcl-2 in B-cell


lymphoma is

t (8: 14)



t (8: 12)



t (14: 18)



t (14: 22)

 22. Which is must for prothrombins time (PT)?



Thromboplastin



Prothrombin



Fibrin


Fibrinogen

 23. All are true regarding thrombotic thrombocytopenic purpura


except:

Normal complement levels



Microangiopathic hemolytic anemia



Thrombocytopenia



Thrombosis

 24. Bleeding time is abnormal in:



Hemophilia



Christmas disease



von Willebrand disease



Vitamin K-deficiency

 25. Platelet function defect is seen in:



Glanzmann syndrome.



Bernard Soulier syndrome



Wiskott Aldrich syndrome



Von-Willebrand disease

 26. True about prothrombin time to



Immediate refrigeration to preserve factor viability



Platelet-rich plasma is essential



Done within 2 hours



Activated with kaolin

 27. The presence of small sized platelets on the peripheral smear


is characteristic of:

Idiopathic thrombocytopenia purpura (ITP)



Bernard Soulier syndrome



Disseminated intravascular coagulation



Wiskott Aldrich syndrome

 28. Thrombocytosis is seen in:



Myelofibrosis



SLE


Azidothymidine therapy



Myelodysplastic syndrome

 29. Causes for DIC are:



Anaerobic sepsis



Malignancy



Lymphoma



Leukemia

 30. VWF factor deficiency causes:



Platelet aggregation



Factor VIII in plasma



Defective platelet adhesion



All of the above

 31. Platelet aggregation in vivo is mediated by



Serotonin



Ig mediators.


Interaction among the leukocytes



Interaction among the platelets

 32. All of the following are true about Willebrand factor except:

Synthesized by hepatocytes



Its deficiency can cause factor 8 defect also



Its deficiency may cause problem with platelet adhesion



It serves as carrier for the factor eight

 33. Splenectomy is useful in which of the following?



Chronic ITP



Sickle cell anemia



Tuberculosis



Good pasture syndrome

 34. Thrombospondin is

Coagulation protein



Coagulation promoting protein


Contractile protein



Angiogenesis inhibitory protein

 35. Glycoprotein IIb-IIa complex is deficient in



Bernard Soulier syndrome



Glanzmann disease



Von willebrand disease



Gray platelet syndrome

 36. Glanzmann disease is characterised by which of the following?



Congenital defect of RBCs



Defect of neutrophils



Congenital defect of platelets



Clotting factor deficiency

Pathoanatomy - Rbc: general aspects

Total questions: 33; Correctly answered: 7; Percentage of


correct: 21.2% Skipped question: 0;
correct answer
incorrect answer
skipped question

 1. Normal platelet count is found in:



Wiskott Aldrich syndrome



Henoch Schonlein purpura



Immune thrombocytopenia



Dengue fever

 2. Burr cell is seen in



Uremia



Hepatocellular carcinoma



Gastric carcinoma



Ovarian carcinoma

 3. Anemia which is associated with pancytopenia is



Hemolytic



Iron deficiency



Megaloblastic



All

 4. The best method for estimation of hemoglobin concentration in


blood is

Acid hematin method



Alkali hematin method



Cyanmethemoglobin method



Any of the above

 5. Which of the following surface glycoproteins is most often


expressed in human hematopoietic stem cell?

CD22



CD40



CD15



CD34

 6. Which of the following is associated with an intrinsic defect in


the RBC membrane?

Autoimmune hemolytic anemia



Hereditary spherocytosis



Microangiopathic haemolytic anemia



Thermal injury causing anemia

 7. Reticulocytes are stained with which of the following stains?



Brilliant cresyl blue



Sudan black



Warthin starry



Hemotoxylin-eosin stain

 8. A 74 year-old female Lajo Devi with renal failure presents for


hemodialysis. She complains of weakness and palpitations for
about a month. On further investigations, she is found to be
anemic and is given a dose of erythropoietin. Erythropoietin
stimulat

Basophilic erythroblasts



Colony forming units-erythroid



Proerythroblasts



Reticulocytes

 9. Reticulocytes are stained with



Methyl violet


Brilliant Cresyl blue



Sudan black



Indigo carmine

 10. Which of the following surface glycoproteins is most often


expressed in human hematopoietic stem cell?

CD 22



CD 40



CD 15



CD 34

 11. Which of the following is not a stem cell of the bone marrow?

Lymphoblast



Myeloblast



Myoblast



Normoblast

 12. Reticulocytosis is seen in all except:



P.N.H.



Hemolysis



Nutritional anemia



Dyserythropoietic syndrome

 13. Inappropriate erythropoietin level is found in all except



Renal cell carcinoma



Lung disease



High altitude



Benign liver tumor

 14. The type of anemia seen in chronic renal failure is:



Microcytic



Normocytic



Macrocytic



All of the above


 15. The longest living WBC is which one of the following

Lymphocyte



Eosinophil



Neutrophil



Monocyte

 16. Linzenmeyer is used to measure



Bleeding time



Clotting time



Prothrombin time



ESR

 17. Hb is a good buffer because of:



Histidine residues



Protein nature



Acidic nature



Iron molecule


 18. Hematuria with dysmorphic RBCs are seen in:

Acute glomerulonephritis



Renal TB



Renal calculi



Chronic renal failure

 19. The anaemia associated with leukaemia is ?



Iron deficiency



Megaloblastic type



Myelophthisic type



None of the above

 20. The number of Fe2+ atoms in one Hb molecule



1



2



4



8


 21. Haematocrit is the ratio of

WBC to whole blood



Platelets to whole blood



RBCs to whole blood



Total blood cells to plasma

 22. Storage form of iron



Ferritin



Transferrin



Hepcidin



Ferroportin

 23. In an adult man, there is about how much grams of hemoglobin


in the circulating blood?

350



500



900



1000

 24. The normal albumin: globulin (A/G) ratio blood is



5:1



2:1



1:2



1:1

 25. Serum contains all the clotting factors except



Plasma thromboplastin



Labile factor



Hageman factor



Christmas factor

 26. Thrombosthenin is:



Coagulation protein



Contractile protein



Thrombus inhibiting protein



Protein for platelet production

 27. Acanthocytes are seen in:



Abetalipoproteinemia



Hartnup disease



Whipple disease



None

 28. MCHC is increased in:



Iron deficiency anemia



Spherocytosis



Thalassemia



All

 29. The size of the red blood cells is measured by:



MCV



MCHC



ESR



MCH

 30. Which of these are seen on Romanowsky stain:



Reticulocytes



Basophilic stippling



Heinz bodies



Howell-Jolly bodies

 31. In polycythemia vera, all are raised except:



Hematocrit



Platelet count



RBCs



Erythropoietin

 32. Progenitor hematopoetic stem cells originate in which of the


following?

Bone marrow



Thymus



Lymph node



Spleen

 33. Freezing point of normal human plasma is:



4



0



–0.54° C



8

Pathoanatomy - Renal tumours: rcc, wilms tumour

Total questions: 15; Correctly answered: 7; Percentage of


correct: 46.7% Skipped question: 0;
correct answer
incorrect answer
skipped question

 1. True statement regarding Wilm s tumour is:



Common in adult



Associated with deletion of chromosome 11p13



Associated with MIC-2 genes



Commonest presentation is hematuria

 2. In Wilm s tumor the following leads to emergence of resistance


to chemotherapy:

Nephrogenic rests



Monophasic morphology



Anaplasia



Capsular infiltration

 3. An anxious couple brings in a 18 month boy with gonadal


dysgenesis to a pediatric clinic for a followup visit The physician
Dr Mayank Dhamija notices a large abdominal mass during his
physical examination Which of the following disorders does the
patient

Renal cell carcinoma



Renal hamartoma



Wilms tumor



Transitional cell carcinoma of the bladder

 4. A pediatrician Dr Jyoti Jain discovers a large mass in the


abdomen of a 3-year-old child Ultrasound examination
demonstrates that the mass appears to arise from the right kidney
Which of the following tumors is most likely present?

Cortical adenoma



Hemangioma



Nephroblastoma


Oncocytoma

 5. The cytogenetics of chromophilic renal cell carcinoma is


characterized by:

Mutant VHL gene



Loss of 3p



Trisomy 7/17



Loss of 5q 3

 6. Most common histological type of renal cell carcinoma is:



Clear cell



Medullary



Papillary



Mixed type

 7. In which of the following conditions, Aniridia and


Hemi-hypertrophy are most likely present

Neuroblastoma



Wilm s tumor


Non-Hodgkin s lymphoma



Germ cell tumor

 8. Most important prognostic factor of wilms tumour:



Histopathology and ploidy of cells



Tumour stage



Age of patient



Mutation of chromosome 1p

 9. Oncocytic carcinoma arises from:



Perivascular tissue



Glomerulus



Loop of henle



Collecting duct

 10. Which of the following is not associated with renal cell


carcinoma?

Polycythemia



Amyloidosis



Cushing s syndrome



Hypertension

 11. The most common histological variant of renal cell carcinoma


is

Clear cell type



Chromophobe type



Papillary type



Tubular type

 12. Wilms tumor is associated with all of the following except



Hemihypertrophy



Aniridia



Hypertension



Bilateral polycystic kidney

 13. Gene for Wilm s tumour is located on which of the following?



Chromosome 1


Chromosome 10



Chromosome 11



Chromosome 12

 14. Deletion of short arm of chromosome 11 is seen in:



Osteosarcoma



Meningioma



Wilm s tumor



Colon Carcinoma

 15. A 60 year old security guard Dharam Chaudhary presents to


the his physician because he is having malaise, fever and
weakness On taking history, a positive history of multiple
episodes of passage of altered colored urine is present However,
this is not ass

Straining of urine for calculi



Abdominal CT scan for renal mass



Collection of a 24 hour urine specimen for protein



Percutaneous renal biopsy

Pathoanatomy - Restrictive lung disease: ild, ards, pneumoconiosis

Total questions: 33; Correctly answered: 11; Percentage of


correct: 33.3% Skipped question: 0;
correct answer
incorrect answer
skipped question

 1. Which of the following is associated with hypersensitive


pneumonitis?

Silicosis



Asbestosis



Byssinosis



Berylliosis

 2. Which of the following is characteristically not associated with


the development of interstitial lung disease?

Organic dusts



Inorganic dusts



Toxic gases, e.g. chlorine, sulphur dioxide



None

 3. Pneumoconiosis is seen with which particle size?



0.5-3 microns



3.5-6 microns



6.5-8 microns



10-20 microns

 4. Pleural calcification is found in all of the following except:



Asbestosis



Hemothorax



Tuberculous pleural effusion



Coal worker pneumoconiosis

 5. Caplan

COPD



Pneumoconiosis



Pulmonary edema



Bronchial asthma

 6. Lower lung involvement is common in:



TB


Asbestosis



Silicosis



All

 7. Which of the following is the characteristic feature of adult


respiratory distress syndrome?

Diffuse Alveolar Damage



Interstitial tissue inflammation



Alveolar exudates



Interstitial fibrosis

 8. Features seen in bronchiolitis obliterans with organizing


pneumonia include:

Polypoid plugs in bronchioles



Ulceration and exudation of epithelium into the lumen



Exudation of proteinaceous material in terminal airways



Bronchoconstriction


 9. All are true about phagocytosis except

Size of the particle ingested is less than 0.5 micrometer



Size of the particle ingested is more than 0.5 micrometer



Combines with lysosome forming phagolysosome



Amoeba and other unicellular organisms make their living out of it

 10. All of the following are seen in asbestosis except:



Diffuse alveolar damage



Calcify pleural plaques



Diffuse pulmonary interstitial fibrosis



Mesothelioma

 11. All of the following features are seen in asbestosis except:



Diffuse pulmonary interstitial fibrosis



Fibrous pleural thickening



Emphysema



Calcific pleural plaques


 12. All are recognized causes off Adult Respiratory Distress
Syndrome (ARDS), except:

Smoke inhalation



Malignant hypertension



Gastric aspiration



Viral pneumonias

 13. Ferruginous bodies are seen in:



Silicosis



Byssinosis



Asbestosis



Bagassosis

 14. The following does not occur with asbestosis:



Methaemoglobinemia



Pneumoconiosis



Pleural mesothelioma



Pleural calcification

 15. Just 24 hours after moving in to a new apartment in


Benagaluru, a 27 years old photographer Akki develops acute
onset of fever, cough, dyspnea, headache, and malaise.
Surprisingly his symptoms subside over next 6 days when he
visits a friend in South Gaut

Antigen – antibody complex formation



Release of histamine



Release of leukotrienes



Toxic injury to type I pneumocytes

 16. Which of the following is characteristically not associated with


the development of interstitial lung disease?

Organic dusts



Inorganic dusts



Toxic gases, e.g. chlorine, sulphur dioxide



None

 17. Which of the following inhaled occupational pollutant


produces extensive nodular pulmonary fibrosis?

Silica



Asbestos


Wood dust



Carbon

 18. Asbestos exposure can cause all except:



Arthralgia



Mesothelioma



Carcinoma larynx



Bronchogenic carcinoma

 19. A 44-year-old woman presents with insidious onset of


shortness of breath, chest pain, and fatigue. Chest x-ray films
reveal bilateral pulmonary infiltrates and enlarged hilar lymph
nodes. There is no history of occupational exposure to mineral
dusts or or

Asbestosis



Berylliosis



TB



Sarcoidosis

 20. Baggasosis is caused by:



Cotton dust



Sugarcane



Asbestosis



None

 21. Earliest lesion seen in asbestosis is:



Pleural plaques



Hilar lymphadenopathy



Adenoma lung



Mesothelioma

 22. The commonest cause of death in ARDS is



Hypoxemia



Hypotension



Non pulmonary organ failure



Respiratory failure


 23. Hyaline membrane disease is associated with

Respiratory distress syndrome



Bronchopulmonary dysplasia



Sudden infant death syndrome



Bronchiolitis obliterans

 24. Which interstitial lung disease is caused by organic dust:



Silicosis



Asbestosis



Byssinosis



Anthracosis

 25. Which of the following increases tuberculosis?



Asbestosis



Sarcoidosis



Silicosis



Berylliosis


 26. Asbestosis of the lung is associated with all of the following
except:

Mesothelioma



Progression of lesion even after stopping exposure to asbestos



Nodular lesions involving upper lobe



Asbestos bodies in sputum

 27. Predominant constituent of Hyaline membrane is:



Albumi



Anthracotic pigment



Fibrin rich exudates



None of the above

 28. A 44-year-old woman presents with insidious onset of


shortness of breath, chest pain, and fatigue. Chest x-ray films
reveal ilateral pulmonary infiltrates and enlared hilar lymph nodes.
There is no history of occupational exposure to mineral dusts or
orga

Scleroderma



Goodpasture



Silicosis


Idiopathic pulmonary fibrosis

 29. Which of the following would most likely be observed in the


lung during an autopsy of a 2-week-old infant who died of neonatal
respiratory distress syndrome?

Alveoli filled with neutrophils



Dense fibrosis of the alveolar walls



Enlarged air space



Hyaline membranes and collapsed alveoli

 30. Most dangerous particles causing pneumoconiosis are of size:



1-5 micron



< 1 micron



5-10 micron



10-20 micron

 31. End stage lung disease is seen in:



Sarcoidosis



Interstitial lung disease


Langerhan



Aspergillosis

 32. Acute pulmonary sarcoidosis is least likely to be associated


with:

Uveitis



Pleural effusion



Erythema nodosum



Lymphadenopathy

 33. Egg-shell calcifications are seen in:



Silicosis



Berylliosis



Asbestosis



Bronchial asthma


Pathoanatomy - Shock, dic and miscellaneous

Total questions: 12; Correctly answered: 1; Percentage of


correct: 8.3% Skipped question: 0;
correct answer
incorrect answer
skipped question

 1. All of the following are true about DIC except?



Increased fibrinogen



Increased activated partial thromboplastin time



Decreased prothrombin time



Increased fibrin degradation products

 2. The initiating mechanism in endotoxic shock is



Peripheral vasodilatation



Endothelial injury



Increased vascular permeability



Reduced cardiac output

 3. Which of the following is a feature of Disseminated Intravascular


Coagulation (DIC)?

Normal prothrombin time



Reduced plasma fibrinogen



Normal platelet count



Normal clotting time

 4. Shock lung is characterized by



Alveolar proteinosis



Bronchiolitis obliterans



Diffuse pulmonary hemorrhage



Diffuse alveolar damage

 5. A 24-year-old pregnant Heena sustains a placental abruption,


and is admitted to the ICU where she begins bleeding from
multiple sites, including oral mucous membranes. Which of the
following studies would be most valuable in assessing this patient

Partial thromboplastin time, kininogen, and factor VIII levels



Platelet count, fibrinogen levels, and fibrin degradation products



Platelet count, thrombin time, and prekallikrein levels



Prothrombin time and factor VIII levels

 6. Conditions associated with incoagulable state are:



Abruption placentae



Acute promyelocytic leukemia


Severe falciparum malaria



Snake envenomation

 7. The histological features of shock includes:



ATN



Pulmonary congestion



Depletion of lipids in adrenal cortex



Hepatic necrosis

 8. A 29 year old woman Ruma is in labour but unfortunately during


parturition, the placental membranes tear and amniotic fluid
expressed into a lacerated cervical vein. Which of the following is
the woman most likely to experience immediately following this

Placental abruption



Renal failure



Respiratory distress



Splinter hemorrhages

 9. D-Dimer is the most sensitive diagnostic test for:



Pulmonary embolism


Acute pulmonary oedema



Cardiac tamponade



Acute myocardial infarction

 10. A primiparous woman Ritu at term experiences placental


abruption and is rushed to the operating room for emergency
Cesarean section. She develops shortness of breath, cyanosis,
and copious bleeding from her surgical wounds. Levels of which
of the followi

Factor V



Fibrin degradation products



Fibrinogen



Plasminogen

 11. The initiating mechanism in endotoxic shock is



Peripheral vasodilatation



Endothelial injury



Increased vascular permeability



Cytokine release

 12. Irshaan has been diagnosed with acute promyelocytic


leukemia recently. He presents suddenly in the medical
emergency because of a dangerous complication of his
malignancy called disseminated intravascular coagulation (DIC).
In DIC, micro emboli form lead

Immunologic failure



Renal failure



Right ventricular failure



Hepatic failure

Pathoanatomy - Single gene disorders with classical inheritance

Total questions: 25; Correctly answered: 9; Percentage of


correct: 36.0% Skipped question: 0;
correct answer
incorrect answer
skipped question

 1. Catastrophic variant of Ehler Danlos syndrome is



I



II



III



IV

 2. Neurofibroma is having which of the following inheritance?



Autosomal dominant



Autosomal recessive



X-linked recessive



X-linked dominat

 3. Autosomal recessive diseases are



Hereditary spherocytosis



Thalassemia



Sickle cell anemia



Cystic fibrosis

 4. The approximate number of genes contained in the human


genome is:

40,000



30,000 genes



80,000



1,00,000


 5. Which of the following is not X linked condition:

Duchenne muscular dystrophy



Emery-Dreifuss muscular dystrophy



Facioscapulohumeral muscular dystrophy



Becker muscular dystrophy

 6. Blue black pigmentation in alkaptonuria is due to



Homogentisic acid



Oxalic acid



Glucouronic acid



All

 7. True statements about a-l anti-trypsin deficiency is



Autosomal dominant disease



Emphysema



Fibrosis of portal tract



Diastase resistant positive hepatocytes


 8. All are autosomal dominant disorders except

Albinism



Marfan



Familial adenomatous polyposis



Von-Hippel Lindau syndrome

 9. Inheritance of Gardner syndrome is:



Autosomal recessive



Autosomal dominant



X linkeddominant



X linked recessive

 10. Duchenne dystrophy is a:



Autosomal dominant disorder



X-linked dominant disease



Autosomal recessive disease



X-linked recessive disease


 11. Neurofibromatosis is

Autosomal dominant



AR



X-linked recessive



All

 12. Which of the following is an autosomal dominant metabolic


disorder?

Cystic fibrosis



Phenylketonuria



a-1 antitrypsin deficiency



Familial hypercholesterolemia

 13. Which of the following disorders has been shown to be


genetically transmitted by single autosomal dominant genes?

Catatonic schizophrenia



Phenylketonuria



Creutzfeldt-Jakob



Huntington

 14. Adult polycystic kidney disease is inherited by:



Autosomal dominant



Autosomal recessive



X-linked



Mitochondrial

 15. Which of the following is the inheritance of Huntington



Autosomal dominant



Autosomal recessive



X-linked



Mitochondrial

 16. Autosomal dominant disorders are all except



Hereditary spherocytosis



Thalassemia



Sickle cell anemia


Cystic fibrosis

 17. Hemophilia is associated with:



X chromosome



Y Chromosome



Chromosome 3



Chromosome 16

 18. Which one is not a feature of cystic fibrosis?



Autososmal recessive disease



Abonormal chloride transport



Affects intestine only



Increased risk of pulmonary infections

 19. Which of the following is an X-linked dominant disorder



Vitamin D resistant rickets



Familial hypercholesterolemia



Red green color blindness


Achondroplasia

 20. Which one of the following is an autosomal dominant disorder:



Duchenn



Fragile X syndrome



Fanconi



Huntingtons chorea

 21. Duchenne muscular dystrophy is inherited as:



X linked



Autosomal dominant



Autosomal recessive



Codominant

 22. In marfan syndrome, the defect is in:



Collagen



Elastin



Fibrillin


All

 23. Sickle cell disease is due to



Point mutation



Frame shift mutation



Nucleotide receptor blockage



Non sequence mutation

 24. Which of the following is an autosomal recessive condition?



Ataxia telangectasia



Peutz Jeghers syndrome



Neurofibromatosis



Tuberous sclerosis

 25. A 26-year-old woman presents because of trouble with her


vision. Physical examination reveals a very tall, thin woman with
long, thin fingers. Examining her eyes reveals the lens of her left
eye to be in the anterior chamber. Her blood levels of methioni

Abnormal copper metabolism



Decreased levels of vitamin D


Decreased lysyl hydroxylation of collagen



Defective synthesis of fibrillin

Pathoanatomy - Single gene disorders with non classical inheritance

Total questions: 26; Correctly answered: 3; Percentage of


correct: 11.5% Skipped question: 0;
correct answer
incorrect answer
skipped question

 1. In Prader Willi syndrome, which of the following is increased?



LH



FSH



TSH



Ghrelin

 2. Preferential expression of the gene depending upon the parent


of origin is called:

Anticipation



Germ line mosaicism



Genomic imprinting



Aneuploidy

 3. True statements regarding the mitochondrial genes are:



Paternal transmission



Maternal transmission



Mendelian inheritance



Mitochondrial myopathy

 4. Dominant negative inheritance is seen in:



Ehler Danlos syndrome



Marfan



Hunter syndrome



Osteogenesis imperfecta

 5. Mitochondrial DNA is

Paternally inherited



Maternally inherited



Horizontal inheritance


Vertical inheritance

 6. Increasing severity of mental retardation in male members over


generations is a result of:

Mitochondrial DNA mutation



Frameshift mutation



Y linked disorder



Trinucleotide repeat mutation

 7. All of the following are chromosomal breakage syndromes


except

Fanconi



Ehler-Danlos syndrome



Bloom



Ataxia telangiectasia

 8. A 45-year-old male Arvind who was previously diagnosed with


depression is now experiencing involuntary grimacing and
strange movements of his arms and legs. Neurological
examination shows normal strength and normal deep tendon
reflexes. No sensory defici

Increased penetrance


Pleiotropy



Anticipation



Mosaicism

 9. Majority of the human characteristics are determined by multiple


pairs of genes, many with alternate codes, accounting for some
dissimilar forms that occur with certain genetic disorders. What
type of inheritance involves multiple genes at different loci,

Polygenic inheritance



Multifactorial inheritance



Monofactorial inheritance



Collaborative inheritance

 10. One of the following disorders is due to maternal disomy



Prader Willi syndrome



Angelman syndrome



Hydatidiform mole



Klinefelter’s syndrome


 11. A young boy, Rinku is being evaluated for developmental delay,
mild autism, and mental retardation. Physical examination reveals
the boy to have large, reverted ears and a long face with a large
mandible. He is also found to have macroorchidism (large te

Fragile X syndrome



Huntington



Myotonic dystrophy



Spinal-bulbar muscular atrophy

 12. All of the following are characterized by



Friedrich



Fragile X syndrome



Huntington’s disease



Huntington

 13. Mitochondrial chromosomal abnormality leads to:



Leber’s hereditary optic neuropathy



Angelman syndrome



Prader villi syndrome



Myotonic dystrophy

 14. A young man Ramkishore presents for an employment


physical. He is very tall,has long fingers, and hyperflexible joints.
He states that he has always been called

Dissecting aortic aneurysm



Hepatosplenomegaly



Polycystic kidneys



Progressive neurologic dysfunction

 15. In Huntington chorea the causative mutation in the protein


huntingtin is a

Point mutation



Gene deletion



Frameshift mutation



Trinucleotide repeat expansion

 16. Differential expression of same gene depending on parent of


origin is referred to as:

Genomic imprinting



Mosaicism


Anticipation



Non-penetrance

 17. An 18 month old child Nonu is brought to a pediatrician. His


mother noticed passage of altered color urine by him for past 2-3
days. Laboratory examination revealed it to be hematuria.
Examination also reveals hypertension and an abdominal mass. A
tumor

5



11



13



17

 18. Genomic imprinting is seen in



Klinefelter



Down



Angelman syndrome



Hydatidi form mole

 19. Mitochondrial DNA (mt-DNA) is known for all except:



Maternal inheritance



Heteroplasmy



Leber hereditary optic neuropathy is the prototype



Nemaline myopathy results due to mutations in mtDNA

 20. Genomic imprinting is associated with:



Silencing of paternal chromosome



Silencing of maternal chromosome



Angelman syndrome



Prader Willi syndrome

 21. Preferential expression of the gene depending upon the parent


of origin is called:

Genomic imprinting



Mosaicism



Alleles



Chimerism


 22. Maternal disomy of chromosome 15 is seen in:

Prader-Willi syndrome



Klinefelter



Angelman syndrome



Turner

 23. NARP syndrome is seen in:



Mitochondrial diseases



Glycogen storage diseases



Lysosomal storage diseases



Lipid storage diseases

 24. A couple has two children affected with tuberous sclerosis. On


detailed clinical and laboratory evaluation (including molecular
studies) both parents are normal. Which one of the following
explains the two affected children in this family? Non penetrance

Non penetrance



Uniparental disomy



Genomic imprinting



Germline mosaicism

 25. Two siblings with osteogenesis imperfecta have normal


parents. The mode of inheritance is explained by which of the
following?

Anticipation



Genomic imprinting



Germline mosaicism



New mutation

 26. Which of the following is/are an example/examples of


non-Mendelian inheritance?

Genomic imprinting



Uniparental disomy



Mitochondrial inheritance



All of the above

Pathoanatomy - Stomach: gastritis, pud, gist, gastric cancer

Total questions: 8; Correctly answered: 3; Percentage of


correct: 37.5% Skipped question: 0;
correct answer
incorrect answer
skipped question
 1. Which of the following sites contains striated muscle that is not
under voluntary control?

Bladder



Colon



Esophagus



Gallbladder

 2. Most common site of GIST is:



Ileum



Esophagus



Colon



Stomach

 3. One of the following can have malignant transformation:



Gastric ulcer



Duodenal ulcer



Stomal ulcer



Stress ulcer

 4. Gastrointestinal stromal tumor originates in which of the


following?

Parietal cells



Chief cells



Neuroendocrine cells



Interstitial cells of Cajal

 5. Which of the following artery is responsible for duodenal ulcer


hemorrhage?

Superior pancreaticoduodenal artery



Inferior pancreaticodudenal artery



Gastroduodenal artery



Left gastric artery

 6. An old man being evaluated for abdominal pain and weight loss
undergoes endoscopy showing a broad region of the gastric wall
in which the rugae are flattened. Biopsy of this area shows
infiltration by numerous polygonal tumor cells with small, dark,
round

Keratohyalin granules observed by electron microscopy



Melanosomes and premelanosomes by electron microscopy


Positive staining for gastrin by light microscopy



Positive staining for mucin by light microscopy

 7. Which of the following is not true about GIST?



Stomach is the most common site



High propensity of malignant change



Associated with c-KIT mutation



Histology shows spindle shaped cells

 8. The best prognosis is gastric carcinoma is in type:



Linitis plastic



Polypoidal growth



Ulcerative



Superficial spreading

athoanatomy - Test 4

Total questions: 50; Correctly answered: 11; Percentage of


correct: 22.0% Skipped question: 0;
correct answer
incorrect answer
skipped question

 1. Classification of necrosis by the mechanism of action of the


etiologic factor:

Direct



Gangrene



Sequestration



Infarction

 2. Frequent localization of Liquefactive necrosis:



Liver



Brain



Myocardium



Kidneys

 3. What etiological kind of necrosis develops under the


phenomenon of Arthus:

Vascular



Toxic



Traumatic


Allergic

 4. What is necrosis:

Death of an organism



Death of organs and tissues in the living body



Calcification of tissues



Standstill of breathing

 5. Characteristic of vascular necrosis:



Direct



Traumatic



Indirect



Trophoneurotic

 6. What necrosis develops from exposure to low and high


temperatures:

Direct



Indirect



Trophoneurotic



Vascular

 7. The process that constitute the essence of necrosis:



Chronic venous congestion



Tissue anemia



Protein coagulation



Sclerosis

 8. Frequent localization of dry gangrene:



Extremities



Intestine



Heart



Liver

 9. Characteristic of tissues with dry gangrene:



Edema



Mummification


Anemia



Fullness

 10. Cell ultrastructure that determine autolytic enzymatic


processes in a cell:

Golgi apparatus



Mitochondria



Lysosomes



Endoplasmic reticulum

 11. Characteristic of the infarction:



Traumatic necrosis



Direct necrosis



Vascular necrosis



Caseous necrosis

 12. Characteristics of bedsore:



Traumatic necrosis



A kind of gangrene



Toxic necrosis



Myocardial infarction

 13. Definition of "necrosis":



Anemia of the organ



Dystrophy



Death of an organism



Necrosis of tissues in a living body

 14. Cause of Vascular Necrosis:



Arterial hyperemia



Anemia



Thrombosis of the vessel



Hemorrhage

 15. Frequent localization of wet gangrene:



Intestine


Spinal cord



Umbilical cord



Spleen

 16. Petrification is the outcome of necrosis, characterized by:



Replacement with connective tissue



Formation of a capsule



Deposition of calcium



Formation of a cyst

 17. Synonym of vascular necrosis:



Infarction



Dystonic



Allergic



Dystrophic

 18. The outcome of the Liquefactive necrosis of the brain can be:

Petrification


Ossification



Cyst formation



Scarring

 19. Choose the favorable outcome of necrosis:



Sepsis



Septic melting



Encapsulation



Mummification

 20. A variety of coagulation necrosis:



Sequestration



Caseous



Colliquative



Bedsore

 21. The substance that determines the color of gangrenous


tissues:

Hemomelanin



Lipofuscin



Melanin



Sulphurous iron

 22. Select the stages of the necrotic process, EXCEPT:



Plasmolysis



Plasmorexis



Necrobiosis



Karyorexis

 23. Encapsulation is the outcome of necrosis, characterized by:



Replacement with connective tissue



Formation of a capsule



Deposition of calcium



Formation of a cyst


 24. The disease, accompanied by the development of caseous
necrosis:

Intestinal infections



Atherosclerosis



Tuberculosis



Rheumatism

 25. The organization is the outcome of necrosis, characterized by:



Replacement with connective tissue



Formation of a capsule



Deposition of calcium



Suppuration

 26. What is infarction:



Toxic necrosis



Allergic necrosis



Direct necrosis



Vascular necrosis

 27. In which organs the development of gangrene is possible:



The brain



The spleen



Intestine



Heart

 28. Changes in the stroma of the organ with necrosis:



Ischemia



Hyalinosis



Amyloidosis



Fibrinoid necrosis

 29. Microscopic signs of cell necrosis:



Plasmorrhagia



Plasmatization



Karyorexis



Karyokinesis

 30. Definition of the concept of "necrobiosis":



Spasm of blood vessels



Anemia of organs



Irreversible dystrophic processes



Autolysis

 31. What necrosis developed under the action of chemical and


physical factors:

Mechanical



Vascular



Trophoneurotic



Traumatic

 32. Enzyme that take part in the autolysis of cell nuclei:



Acidic phosphotase



DNAase



Alkaline phosphatase



Cytochrome oxidase

 33. A synonym of focal colliquative necrosis of the cell:



Fatty degeneration



Granular degeneration



Balloon dystrophy



Hyaline-drop dystrophy

 34. Adverse outcome of necrosis:



Suppuration



Ossification



Organization



Petification

 35. Characteristic of the demarcation zone:



Zone of ischemia



Accumulation of leukocytes and hyperemia



Accumulation of erythrocytes


Vascular spasm

 36. What is formed as a result of necrosis of cells and extracellular


substance:

Pigments



Polysaccharides



Hemoglobin



Tissue detritus

 37. What necrosis develops with the predominance of colliquation


processes:

Coagulation



Dry



Wet



Caseous

 38. Characteristic outcome of wet necrosis:



Encapsulation



Petification


Mummification



Cyst

 39. Type of gangrene:



Dense



Dry



Partial



Toxic

 40. What develops around the focus of necrosis in living tissues:



Autolysis



Tissue anemia



Demarcation inflammation



Necrobiosis

 41. Outcome of necrosis can be:



Organization



Embolism


Infarction



Thrombosis

 42. The organization is a favorable outcome of:



Embolism



Necrosis



Pathological accumulation (dystrophy)



Induration

 43. Causes of direct necrosis:



Nerve damage



Standstill of blood flow



The effect of acids



Rupture of blood vessels

 44. Favorable outcome of dry necrosis:



Organization



Septic autolysis


Regeneration



Cyst formation

 45. Microscopic signs of cell necrosis:



Fatty degeneration



Karyopicnosis



Karyokinesis



Degranulation

 46. Diseases in which fibrinoid necrosis often develops:



Intestinal infections



Rheumatic diseases



Influenza



Leukemia

 47. What necrosis develops in a sensitized organism:



Trophoneurotic



Toxic


Allergic



Traumatic

 48. Typical localization of wet necrosis:



Myocardium



Brain



Smooth muscles



Striated musculature

 49. Etiological type of necrosis:



Infarction



Dry



Sequestration



Trophoneurotic

 50. Clinico - morphological forms of necrosis:



Direct



Indirect


Infarction



Vascular


Pathoanatomy - Test 5

Total questions: 3; Correctly answered: 0; Percentage of


correct: 0.0% Skipped question: 0;
correct answer
incorrect answer
skipped question

 1. A 46-year-old woman with prominent splenomegaly presents


with a 3-month history of malaise, easy fatigability, weakness,
weight loss, and anorexia. A complete blood count and differential
demonstrates a white blood cell count of 250,000/mm3 (normal
3,000

T(8;14)



T(9;22)



T(11;22)



T(14;18)

 2. An 8-year-old child is evaluated by the pediatrician, who notes


what appear to be 10 small café-au-lait spots on the child’s torso.
In addition, on close inspection of the eyes, the presence of Lisch
nodules is noted. The patient is diagnosed with von Rec

Activates the GTPase activity of Ras.



Cleaves cellular proteins during apoptosis.



Functions as a regulator of the cell cycle.



Promotes angiogenesis in the growing tumor mass

 3. A 27-year-old woman has recently been diagnosed with a glioma


(a malignant brain tumor). Further family history reveals that her
4-year-old son has been diagnosed with leukemia and has been
undergoing chemotherapy. In addition, the patient’s mother died a

Activates the GTPase activity of the gene product of the Ras oncogene



Excises ultraviolet light–induced thymidine dimers



Functions as a cytoplasmic tyrosine kinase



Halts the cell cycle if DNA damage is detected

Pathoanatomy - Thrombosis: embolism: infarct

Total questions: 27; Correctly answered: 6; Percentage of


correct: 22.2% Skipped question: 0;
correct answer
incorrect answer
skipped question

 1. Which one of the following inherited disorders produces arterial


thrombosis?

Factor V Leiden mutation



Antithrombin deficiency



Homocysteinemia


Protein S deficiency

 2. Antiphospholipid syndrome is associated with all except:



Recurrent abortion



Venous thrombosis



Pancytopenia



Antibody to lupus

 3. A 62 year old man Ram Srinath is brought by his wife Shanti


Devi after sustaining a fall in the washroom while taking bath. He
has severe pain in his right leg. Dr. Amit Shersia, the orthopedic
surgeon diagnoses it as fracture femur. He is discharged afte

Hematoma of the right thigh



Fat embolism



Gangrene in the right foot



Pulmonary thromboembolism

 4. Arterial thrombosis is seen in



Homocysteinemia



Anti-phospholipid syndrome Protein C deficiency


Protein c deficiency



Protein S deficiency

 5. Pale infarct is seen in all except:



Lungs



Spleen



Kidney



Heart

 6. At autopsy, the spleen of a patient is noted to have a thickened


capsule and many small, scarred areas. Microscopic examination
of the scarred areas reveals fibrosis with hemosiderin and calcium
deposition. This type of spleen is usually seen in conjunct

Hepatic cirrhosis



Hodgkin



Rheumatoid arthritis



Sickle cell anemia

 7. Predisposing factor for venous thrombosis:



AT III deficiency


Protein S deficiency



Protein C deficiency



All

 8. Lines of Zahn occur in which of the following?



Postmortem clot



Infarct



Embolus



Coralline thrombus

 9. Inherited coagulation disorders are:



Protein S deficiency



Protein C deficiency



Leiden factor mutation



Lupus anticoagulant

 10. The five stages of hemostasis are given below in random order.
Put them into their correct order. (a) Clot dissolution (b) Blood
coagulation (c) Vessel spasm (d) Clot retraction (e) Formation of
platelet plug

cabed



Acbde



cebda



Ecdba

 11. White infarcts are seen in the following except:



Liver



Kidney



Spleen



Heart

 12. Necrosis with putrefaction is called as:



Desiccation



Gangrene



Liquefaction



Coagulative necrosis

 13. A 64 year old man Ojas Alok Nath resides in a city. He is a


known case of hypertension and is a smoker too. One day, while
watching tv, he developed severe pain in the chest. He is rushed to
the medical emergency of the city hospital where this episode is

Dilate coronary arteries



Inhibit platelet adhesion



Inhibit platelet aggregation.



Lyses thrombi

 14. Histologic sections of lung tissue from 66-year-old woman,


Sheena with congestive heart failure and progressive breathing
problems reveal numerous hemosiderin-laden cells within the
alveoli. Which of the following is the cell of origin of these

Endothelial cells



Pneumocytes



Lymphocytes



Macrophages

 15. Virchow’s triad includes all except



Injury to vein



Venous thrombosis


Venous stasis



Hypercoagulability of blood

 16. Lines of Zahn are found in:



Thrombus



Infarct tissue



Postmortem clot



All

 17. White infarct is seen in:



Lung



Intestine



Heart



Ovary

 18. Hypercoagulability due to defective factor V gene is called:



Lisbon mutation



Leiden mutation


Antiphospholipid syndrome



Inducible thrombocytopenia syndrome

 19. Hemorrhagic infarction is seen in:



Venous thrombosis



Thrombosis



Septicemia



Embolism

 20. A 27-year-old woman, Shama presents with a history of losing


pregnancies in the past 5 years. She also has a history of recurrent
pains in her legs secondary to recurrent thrombosis. Her
symptoms are most likely due to a deficiency of which one of the
fol

PA inhibitors



Protein C



Plasmin



Thrombin

 21. Fat embolism is commonly seen in:



Head injuries



Long bone fractures



Drowning



Hanging

 22. A pregnant woman develops deep, boring pain of her left thigh
muscles associated with swelling and enhanced warmth of the
same leg. The pain is worsened by extending the foot. The
superficial veins of the leg are engorged. Her condition puts her at
risk

Acute renal failure



Cerebral hemorrhage



Hepatic infarction



Pulmonary embolus

 23. Chicken fat clot is:



Postmortem clot



Thrombus



Infarct



All

 24. Heart failure cells are seen in



Chronic venous congestion of liver



Chronic venous congestion of lung



Acute venous congestion of lung



Acute venous congestion of liver

 25. Congenital hypercoagulability states are all of the followings


except

Protein C deficiency



Protein S deficiency



Anti-phospholipid antibody syndrome



MTHFR gene mutation

 26. A 20-year-old male, Akash fractured his right femur. He was


admitted to the hospital and over the next several days developed
progressive respiratory problems. Despite extensive medical
intervention, he died 3 days later. At the time of autopsy oil red O-

Air emboli



Amniotic fluid emboli


Fat emboli



Paradoxical emboli

 27. Which of the following statements about pulmonary emboli is


not correct?

60-80% pulmonary emboli are clinically silent



In more than 95% cases venous emboli originate from deep leg veins



Embolic obstruction of pulmonary vessels almost always cause
pulmonary infarction



Embolic obstruction of medium sized arteries may result in pulmonary
infarction

Pathoanatomy - Transplant rejection, gvhd

Total questions: 9; Correctly answered: 1; Percentage of


correct: 11.1% Skipped question: 0;
correct answer
incorrect answer
skipped question

 1. Hyperacute rejection is due to



Preformed antibodies



Cytotoxic T-lymphocyte mediated injury



Circulating macrophage mediated injury



Endothelitis caused by donor antibodies

 2. Transfer of the graft of different species are called as



Isograft



Allograft



Homograft



Xenograft

 3. All are affected in Graft-Versus host reaction:



Skin



GIT



Liver



Lung

 4. Acute graft versus host disease reaction occurs in all except



Liver



Adrenal



Gut


Skin

 5. Principal cause of death in renal transplant patient is



Uraemia



Malignancy



Rejection



Infection

 6. Acute humoral renal transplant rejection is characterized by the


following, except:

Presence of anti-donor antibodies



Interstitial and tubular mononuclear cell infiltrate



Necrotizing vasculitis



Acute cortical necrosis

 7. Preformed antibodies cause:



Hyperacute rejection



Acute rejection



Chronic rejection



Acute humoral rejection

 8. Preformed antibodies cause:



Hyperacute rejection



Acute rejection



Chronic rejection



Acute humoral rejection

 9. True about graft versus host disease is:



Associated with solid organ transplantation



Graft must contains immunocompetent T cell



It is seen in immunosuppressed persons



Also called as Runt disease in animals

Pathoanatomy - Urinary bladder disease

Total questions: 6; Correctly answered: 0; Percentage of


correct: 0.0% Skipped question: 0;
correct answer
incorrect answer
skipped question
 1. Chronic urethral obstruction due to benign prostatic
hyperplasia can lead to the following change in kidney
parenchyma

Hyperplasia



Hypertrophy



Atrophy



Dysplasia

 2. Bence Jones proteins are:



Light chain



Heavy chain



Medium chain



All

 3. Michaelis Guttmann bodies are present in:



Analgesic nephropathy



Homman



Malacoplakia



Erythroplasia

 4. Transitional cell carcinoma bladder is associated with which of


the following?

Schistosomiasis



Ascariasis



Malaria



Any of the above

 5. Michaelis Gutmann bodies are seen in



Xanthogranulomatous pyelonephritis



Malacoplakia



Nail patella syndrome



Tubercular cystitis

 6. Metabolic complication in CRF include all of the following


except

Hyperkalemia



Hypophosphatemia



Hypocalcemia



Hypokalemia

athoanatomy - Vascular lung disease: infarct, pulmonary edema,


pulmonary htn

Total questions: 8; Correctly answered: 3; Percentage of


correct: 37.5% Skipped question: 0;
correct answer
incorrect answer
skipped question

 1. The percentage of pulmonary emboli, that proceed to infarction,


is approximately:

0-5%



5-15%



20-30%



30-40%

 2. Bronchogenic sequestration is seen in which lobe:



Left lower lobe



Right upper lobe



Left middle lobe



Left upper lobe

 3. Dr. Sushant Verma conducted a study in MAMC which included


admitted patients hospitalized for more than 10 days and are
bedridden for more than 7 days. He carries a battery of
investigations on these patients including Doppler venous
ultrasound of the low

Cor pulmonale



Hemoptysis



Dyspnea



No symptoms

 4. On sectioning of an organ at the time of autopsy, a focal,


wedge-shaped firm area is seen accompanied by extensive
hemorrhage, with a red appearance. The lesion has a base on the
surface of the organ. This findings is typically of

Lung with pulmonary thromboembolism



Heart with coronary thrombosis



Liver with hypovolemic shock



Kidney with septic embolus

 5. Bilateral exudative pleural effusion is seen in:



SLE



Lymphoma


CCF



Nephrotic syndrome

 6. Which of the folloowing is not true about pulmonary embolus?



Saddle embolus may cause sudden death



Most lesions affect are in the lower lobes



Small arterioles are blocked



Most of the emboli cause infarction

 7. All are the histological features of pulmonary hypertension:



Capillaritis of alveolar septa



Saddle thrombi in pulmonary trunk



Thrombi in pulmonary vasculature



Veno-occlusive disease

 8. Sequestrated lobe of lung is commonly supplied by which of the


following vessels?

Pulmonary artery



Intercostal artery



Descending aorta



Bronchial artery


Pathoanatomy - Vasculitis, raynaud disease

Total questions: 33; Correctly answered: 6; Percentage of


correct: 18.2% Skipped question: 0;
correct answer
incorrect answer
skipped question

 1. Wegener’s granulomatosis:

Involve lung



Involves nose



Involve kidney



Treated with cytotoxic agent and/or steroids

 2. The term infantile polyarteritis nodosa was formerly used for:



Goodpasture



Henoch-Sch



Kawasaki disease



Takayasu

 3. A 30 year old male Munish with exertional calf pain and painful
foot ulcers demonstrates hypersensitivity to intradermally injected
tobacco extract. Which of the following pathologic processes is
most likely responsible for this patient

Lipid-filled intimal plaque



Onion-like concentric thickening of arteriolar walls



Transmural inflammation of the arterial wall with fibrinoid necrosis



Segmental vasculitis extending into contiguous veins and nerves

 4. A patient presents with respiratory symptoms, i.e. cough,


hemoptysis and glomerulonephritis. His c-ANCA levels in serum
were found to be raised. The most likely diagnosis is:

Goodpasture



Classic polyarteritis nodosa



Wegener’s granulomatosis



Kawasaki

 5. All of the following are true about pyogenic granuloma except



Bacterial infection



Bleeding



Benign tumour



Capillary hemangioma

 6. Which is associated with vasculitis of medium size vessels:



Temporal arteritis



Wegners granulomatosis



Polyarteritis nodosa



Henoch Schonlein purpura

 7. Which of the following is a feature of temporal arteritis?



Giant cell arteritis



Granulomatous vasculitis



Necrotizing vasculitis



Leucocytoclastic vasculitis

 8. Which of the following is not a common cause of Vasculitis in


adults?

Giant Cell Arteritis



Polyarteritis nodosa



Kawasaki disease



Henoch-Schonlein Purpura

 9. In PAN, the lesions are seen in all except:



Lung



Pancreas



Liver



Heart

 10. C-ANCA antibodies are characteristic of:



Sj



Giant cell arteritis



Wegener



Kawasaki

 11. ANCA is seen in all except:



Wegener


Henoch-Sch



Microscopic polyangiitis



Churg-Strauss disease

 12. Small vessels vasculitis seen in



Giant cell arteritis



Takayasu arteritis



PAN



Microscopic Polyangitis

 13. All of the following are small vessel vasculitis except:



Kawasaki disease



Churg-Strauss syndrome



Wegener granulomatosis



None of the above

 14. All are true about ANCA associated crescentic


glomerulonephritis, except:

Seen in Wegener



Seen in microscopic polyangitis



Seen in Henoch-Sch



Is pauci immune in nature

 15. ANCA is associated with:



Henoch-Schonlein Purpura



Goodpasture syndrome



Rheumatoid arthritis



Wegener’s granulomatosis

 16. Most common organs involved in Wegener



Skin and nose



Lung and kidney



Heart and kidney



Kidney and nervous system


 17. Polyarteritis nodosa does not involve:

Pulmonary artery



Bronchial artery



Renal artery



Cerebral artery

 18. Hypersensitivity vasculitis most commonly involves:



Arterioles



Post-capillary venules



Capillaries



Medium sized arteries

 19. A 50 year-old man presents to his physician with hematuria.


Renal biopsy demonstrates a focal necrotizing glomerulitis with
crescent formation. The patient has a history of intermittent
hemoptysis and intermittent chest pain of moderate intensity. A
previ

Aspergillosis



Wegener’s granulomatosis



Renal carcinoma metastatic to the lung


Tuberculosis

 20. All is true about Giant cell arteritis except:



Involves large to small sized arteries



Granulomatous inflammation



Most commonly involved artery is abdominal aorta



Segmental nature of the involvement

 21. Pathogenesis of all of the following is granulomatous, except:



Wegener



Buerger



Takayasu



Microscopic polyangiitis

 22. Which of the following is abdominal angiitis?



Giant cell arteritis



Takayasu arteritis



Kawasaki disease


Polyarteritis nodosa

 23. Polyarteritis nodosa can occur in association with which of the


following:

Hypertension



Trauma



Drugs



Bronchial asthma

 24. Raynaud’s phenomenon what change is seen in vessels initial


stage:

20-40%



>80%



40-60%



10%

 25. Raynaud’s phenomenon what change is seen in vessels initial


stage:

No change



Thrombosis


Fibrinoid necrosis



Hyaline sclerosis

 26. Wegener’s granulomatosis involve:



Lung



Liver



Kidney



Upper respiratory tract

 27. A 5 year old child presents with perivascular IgA deposition


and neutrophilic collection. There is erythematous rash on the
lower limbs and nonblanching purpura. The likely diagnosis in the
child is:

Henoch-Schonlein Purpura



Wegner



Giant cell Vasculitis



Kawasaki

 28. Which of the following is not a characteristic of Wegener



Granuloma is vessel wall


Focal necrotising glomerulonephritis



Positive for cANCA



Involves large vessels

 29. A 57 year-old woman Beenu Dash presents with frequent


headaches, which occur on one side and are throbbing. She has
fever and tenderness over both temples. Investigations reveal a
slightly decreased hematocrit and an elevated erythrocyte
sedimentation rat

Blindness



Deafness



Loss of tactile sensation



Loss of the ability to speak

 30. Sturge weber syndrome is associated with:



Port wine stain



Cavernous hemangioma



Lymphangioma



Hemangiosarcoma

 31. Glomus tumor is seen in:



Retroperitoneum



Soft tissue



Distal portion of digits



Proximal portion of digits

 32. Glomus cells are found in which of the following conditions?



Carotid body tumour



Thyroid carcinoma



Liver carcinoma



Glomus tumor

 33. Vasculitis is seen in:



Buerger’s disease



HSP



Gout



Reiter’s disease

Pathoanatomy - Wound healing; stem cell biology

Total questions: 18; Correctly answered: 5; Percentage of


correct: 27.8% Skipped question: 0;
correct answer
incorrect answer
skipped question

 1. Vitamin used for post translational modification of glutamic acid


to gamma carboxy glutamate is

A



D



E



K

 2. Maximum collagen in wound healing is seen at which stage of


healing

End of first week



End of second week



End of third week



End of 2 months

 3. Wound contraction is mediated by:



Epithelial cells



Myofibroblasts



Collagen



Elastin

 4. Which of the following is the source of hepatic stem cells?



Limbus cells



Ito cell



Oval cell



Paneth cell

 5. In regeneration

Granulation tissue



Repairing by same type of tissue



Repairing by different type of tissue



Cellular proliferation is largely regulated by biochemical factors

 6. Tensile strength of wound after laparoscopic cholecystectomy


in a 30 year old woman depends upon:

Replacement of type 3 collagen



Extensive cross-linking of tropocollagen



Macrophage activity



Granulation tissue

 7. First sign of wound injury is:



Epithelialization



Dilatation of capillaries



Leukocytic infiltration



Localized edema

 8. An adult old man gets burn injury to his handsOver few weeks,
the burned skin heals without the need for skin graftingThe most
critical factor responsible for the rapid healing in this case is:

Remnant skin appendages



Underlying connective tissues



Minimal edema and erythema



Granulation tissue

 9. Which one of the following statements is not correct regarding



Developmental elasticity



Transdifferentiation



Can be harvested from embryo



Amebiasis

 10. Chronic granulomatous disease is:



Associated with formation of multiple granulomas



A benign neoplastic process



A parasitic disease



Acquired leukocyte function defect

 11. A 45-year-old man, Suveen presents with pain in the mid


portion of his chestThe pain is associated with eating and
swallowing foodEndoscopic examination reveals an ulcerated
area in the lower portion of his esophagusHistologic sections of
tissue taken fro

Caseating granulomatous inflammation



Dysplastic epithelium



Granulation tissue



Squamous cell carcinoma

 12. When a cell transforms itself into different lineage the ability us
know as:

De-differentiation



Re-differentiation



Trans-differentiation



Sub-differentiation

 13. Prion disease is caused by:



Misfolding of protein



Denaturation of proteins



Reduced formation of proteins



Exces formation of proteins

 14. Oval cells : are seen in the stem cells of which of the following
tissues?

Skin



Cornea



Liver



Bone

 15. One of the following statements about hematopoietic stem cell


is false?

Stem cells have self renewal property



Subset of stem cells normally circulate in peripheral blood



Marrow derived stem cells can seed other tissues and develop into non
hematopoietic cells as well



Stem cells resemble lymphoblasts morphologically

 16. Which of the following adhesion molecules is involved in


morphogenesis?

Osteopontin



Osteonectin SPARC



Tenascin



Thrombospondins

 17. Absolute lymphocytosis is seen in



SLE



T.B



CLL



Brucellosis

 18. Which of the following is absolutely essential for wound


healing?

Vitamin D



Carbohydrates



Vitamin C



Balanced diet


1. Fournier’s gangrene is seen in: Scrotal skin
2. About apoptosis, true statement is: Councilman bodies is associated with apoptosis
3. A patient Fahim presents to the hospital with jaundice, right upper quadrant pain and fatigue. He tests
positive for hepatitis b surface antigen. The serum bilirubin levels is 4.8mg/dl (direct is 0.8mg/dl and indirect
bilirubin is 4.0mg/dl), AsT levels is 300 u/l, AlT is 325 u/l and alkaline phosphatase is within normal limits. The
elevation in AsT and AlT can be explained by which of the following?--- Cell membrane rupture
4. A 23-year-old lady sweety was driving her car when she had to apply brakes suddenly. she suffered from----
Dystrophic fat necrosis
5. A patient subbu is diagnosed with a cancer. It was observed that he shows a poor response to a commonly
used anti-cancer drug which acts by increasing programmed cell death. Inactivation of which of the following
molecules/genes is responsible for the resistance shown in the tumor cells?---- p53
6. Dr Maalu Gupta is carrying out an experiment in which a genetic mutation decreased the cell survival of a
cell culture line. These cells have clumping of the nuclear chromatin and reduced size as compared to normal
cells. Which of the following is the most likely involved gene in the above described situation?--- Bcl-2
7. If we presume that the patient has experienced several similar episodes of pain over the last 10 hours, which
of the following ultra-structural changes would most likely indicate irreversible myocardial cell injury in this
patient?--- Mitochondrial vacuolization
8. A 55-year-old man, vikas develops a thrombus in his left anterior descending coronary artery. The area of
myocardium supplied by this vessel is irreversibly injured. The thrombus is destroyed by the infusion of
streptokinase, which is a plasminogen activator, and the injured area is reperfused. The patient, however,
develops an arrhythmia and dies. An electron microscopic (eM) picture taken of the irreversibly injured
myocardium reveals the presence of large, dark, irregular amorphic densities within mitochondria. What are
these abnormal structures?--- Flocculent densities
9. Cells most sensitive to hypoxia are:--- Neurons
10. Which of the following induces apoptosis in a cell?--- Glucocorticoids
11. CD 95 is a marker of---Extrinsic pathway of apoptosis
12. Fibrinoid necrosis may be observed in all of the following, except:--- Diabetic glomerulosclerosis
13. Caspases are associated with which of the following?--- Embryogenesis
14. Light microscopic characteristic feature of apoptosis is:--- Condensation of the nucleus
15. Ultra-structural finding of irreversible injury---Amorphous densities in mitochondria
16. Internucleosomal cleavage of DNA is characteristic of—Apoptosis
17. Ladder pattern of DNA electrophoresis in apoptosis is caused by the action of the following enzyme:
Endonuclease
18. True about apoptosis is all, ----End products are phagocytosed by macrophage
19. Psammoma bodies are seen in all except:--- Follicular carcinoma of thyroid
20. True about metastatic calcification is---Mitochondria involved earliest
21. Which of the following is not a common site for metastatic calcification?--- Parathyroid
22. Calcification of soft tissues without any disturbance of calcium metabolism is called---Dystrophic calcification
23. The light brown perinuclear pigment seen on H & e staining of the cardiac muscle fibres in the grossly normal
appearing heart of an 83 year old man at autopsy is due to deposition as:--- Lipochrome
24. Dystrophic calcification is seen in:--- Atheromatous plaque
25. The Fenton reaction leads to free radical generation when:--- Hydrogen peroxide is formed by
Myeloperoxidase
26. Mallory hyaline is seen in:--- Alcoholic liver disease
27. Heterotopic calcification occurs in:---- Ankylosing spondylitis
28. Pigmentation in the liver is caused by all except:--- None
29. Wear and tear pigment in the body refers to---Lipochrome
30. Mallory hyaline bodies are seen all except---Crigler-Najjar syndrome
31. Dystrophic calcification is seen in:--- Atheroma
32. Brown atrophy is due to---Lipofuscin
33. Psammoma bodies are typically associated with all of the following neoplasms except—Medulloblastoma
34. All are true about metaplasia except---Irreversible
35. About hyperplasia, which of the following statement is false?--- size of the affected cell
36. Metastatic calcification occurs in all except:--- Atheroma
37. Dystrophic calcification is:--- Calcification in dead tissue
38. An old man Muthoot has difficulty in urination associated with increased urge and frequency. He has to get
up several times in night to relieve himself. There is no history of any burning micturition and lower back
pain. On rectal examination, he has enlarged prostate. Which of the following represents the most likely
change in the bladder of this patient?--- Hypertrophy
39. An increase in the size of a cell in response to stress is called as hypertrophy. Which of the following does not
represent the example of smooth muscle hypertrophy as an adaptive response to the relevant situation?---
Triceps in body builders
40. A patient Ramu Kaka presented with complaints of slow progressive breathlessness, redness in the eyes and
skin lesions. His chest X ray had bilateral hilar lymphadenopathy. His serum ACe levels were elevated. On
doing Kveim test, it came out to be positive. Final confirmation was done with a biopsy which demonstrated
presence of noncaseous granuloma. A diagnosis of sarcoidosis was established. Which of the following
statements regarding calcification and sarcoidosis is not true?--- There is presence of dystrophic calcification
41. A 50-year-old male alcoholic, Rajesh presents with symptoms of liver disease and is found to have mildly
elevated liver enzymes. A liver biopsy examined with a routine hematoxylin and eosin (H & e) stain reveals
abnormal clear spaces in the cytoplasm of most of the hepatocytes. Which of the following materials is most
likely forming cytoplasm spaces?--- Triglyceride
42. A 36-year-old woman, Geeta presents with intermittent pelvic pain. Physical examination reveals a 3-cm
mass in the area of her right ovary. Histologic sections from this ovarian mass reveal a papillary tumor with
multiple, scattered small, round, laminated calcifications. Which of the following is the basic defect
producing these abnormal structures?--- Dystrophic calcification
43. A 28-year-old male executive presents to the doctor with complaints of “heartburn” non responsive to usual
medicines undergoes endoscopy with biopsy of the distal esophagus is taken. What type of mucosa is
normal for the distal esophagus?--- Non-keratinized, stratified, squamous epithelium
44. True about psammoma bodies are all except:-- Seen in teratoma
45. Metastatic calcification is most often seen in---Lungs
46. Russell bodies are seen in---Plasma cells
47. Psammoma bodies show which type of calcification---Dystrophic
48. Gamma Gandy bodies contain hemosiderin and:--- Ca++
49. Oncocytes are modified form of which of the following:--- Mitochondria
50. Which of the following is the most common fixative used in electron microscopy?--- Glutaraldehyde
51. The fixative used in histopathology---10% buffered neutral formalin
52. Which is the most commonly used fixative in histopathological specimens?--- Formaldehyde
53. lipid in the tissue is detected by:--- Oil Red O
54. The most abundant glycoprotein present in basement membrane is:--- Laminin
55. Enzyme that protects the brain from free radical injury is:-- Superoxide dismutase
56. Increased incidence of cancer in old age is due to---Telomerase reactivation
57. Stain not used for lipid---Congo red
58. Acridine orange is a fluorescent dye used to bind---DNA and RNA
59. PAs stains the following except---Lipids
60. All are components of basement membrane except---Rhodopsin
61. True about cell ageing:--- Lipofuscin accumulation in the cell
62. Which of the following pigments are involved in free radical injury?--- Lipofuscin
63. Neutrophil secretes---Myeloperoxidase
64. Which of the following is a peroxisomal free radical scavenger?--- All of the above
65. Crooke---Basophil cells of the pituitary gland in Cushing
66. An autopsy is performed on a 65-year-old man, suresh who died of congestive heart failure. sections of the
liver reveal yellow-brown granules in the cytoplasm of most of the hepatocytes. Which of the following
stains would be most useful to demonstrate with positive staining that these yellow-brown cytoplasmic
granules are in fact composed of hemosiderin (iron)?--- Prussian blue stain
67. An AIDs patient Khalil develops symptoms of pneumonia, and Pneumocystis carinii is suspected as the
causative organism. bronchial lavage is performed. Which of the following stains would be most helpful in
demonstrating the organism---Methenamine silver
68. Which process makes the bacteria---Opsonisation
69. In an evaluation of a 7-year-old boy, Ram who has had recurrent infections since the first year of life,
findings include enlargement of the liver and spleen, lymph node inflammation, and a superficial dermatitis
resembling eczema. Microscopic examination of a series of peripheral blood smears taken during the course
of a staphylococcal infection indicates that the bactericidal capacity of the boy--Defect in the enzyme NADPH
oxidase
70. Which of the following is a negative stain?--- Nigrosin
71. Stain used for melanin is---Masson fontana stain
72. Which of the following statements about Telomerase is true?--- Causes carcinogenesis
73. In acute inflammation endothelial retraction leads to---Delayed prolonged increase in permeability
74. After binding of complement and antibody on the surface of encapsulated bacteria, the process of
phagocytosis by polymorphonuclear leukocytes involves which of the following Fc and C3b----Pseudopod
extension
75. Free radicals are generated by all except---Superoxide dismutase
76. Which among the following is the hallmark of acute inflammation?--- Vasodilation and increase in
permeability
77. Characteristic of acute inflammation is:--- Vasodilation and increased vascular permeability
78. Main feature of chemotaxis is---Unidirectional locomotion of the neutrophils
79. Which of the following helps in generating reactive O2 intermediates in the neutrophils?---- NADPH oxidase
80. Basement membrane degeneration is mediated by:--- Metalloproteinases
81. Delayed prolonged bleeding is caused by:--- Direct injury to endothelial cells
82. Earliest transient change following tissue injury will be:--- Neutrophilia
83. All of the following vascular changes are observed in acute inflammation, except:--- Decreased hydrostatic
pressure
84. The following host tissue responses can be seen in acute infection, except:--- Granuloma formation
85. Oxygen dependent killing is done through---NADPH oxidase
86. Which of the following is not true?--- In Bruton
87. Nitroblue tetrazolium test is used for?--- Phagocytes
88. In acute inflammation due to the contraction of endothelial cell cytoskeleton, which of the following
results?--- Early transient increase
89. Diapedesis is---Immigration of the leukocytes through the vessel wall to the site of inflammation
90. Endothelium leukocyte interaction during inflammation is mediated by/due to---Selectins
91. In genetic deficiency of MPO the increased suscep tibility to infection is due to:--- Inability to produce
hydroxyl-halide radicals
92. After extravasation, leukocytes emigrate in the tissue towards the site of injury. It is called as---Chemotaxis
93. The complex process of leukocyte movements through the blood vessels are all except----Phagocytosis
94. All are true about exudate excep----Less protein
95. All of the following are signs of inflammation except---Absence of functional loss
96. Endogenous chemoattractant is:--- C5a
97. Which of the following statements in context of the enzyme---It causes formation of a more important
bactericidal agent than defensins and lysozyme
98. A middle aged scientist Sudarshan is working in the laboratory on the mechanisms involved in inflammation.
He observes that the leucocytes leave the blood vessels and move towards the site of bacteria. Which of the
following is likely to mediate this movement of the bacteria?--- C5a
99. Which of the following statements in context of endothelial cell contraction in inflammation is false---It
affects venules, capillaries and arterioles commonly
100. A 14 month old boy Chunnu is being evaluated for recurrent, indolent skin infections and gingivitis.
On taking a detailed history from the mother, she tells very valuable point that he had delayed separation of
the umbilical cord which occurred around 9-10 weeks after his birth. Which of the following proteins is most
likely under-expressed in this boy?--- Integrins
101. A 5-year-old female Sukanya is hospitalized with fever and hemorrhagic skin lesions on her lower
extremities. Aout five months ao she was successully treated with penicillin for bacterial meningitis. She
likely to be suffering from which of the following immune system disorders?---- Inability to form the
membrane-attack complex
102. A 3-year-old boy, Krish presents with recurrent bacterial and fungal infections primarily involving his
skin and respiratory tract. Physical examination reveals the presence of oculocutan-eous albinism.
Examination of a peripheral blood smear reveals large granules within neutrophils, lymphocytes, and
monocytes. The total neutrophil count is found to be decreased. Further workup reveals ineffective
bactericidal capabilities of neutrophils due to defective fusion of phagosomes with lysosomes. Which of the
following is the most likely diagnosis?---- Chediak-Higashi syndrome
103. All of the following are a family of selectin except---A selectin
104. Most important for diapedesis?--- PECAM
105. In acute inflammation the tissue response consists of all except---Granuloma formation
106. The function common to neutrophils, monocytes, and macrophages is----Phagocytosis
107. The role of bradykinin in process of inflammation is:---i ncreased vascular permeability
108. Which of the following is not a pyrogenic cytokine?--- IL – 18
109. All of the following are true in respect of angioneurotic edema except? manifests as pitting edema
110. Which of the following complement component can be activated is both common as well as
alternative pathways?--- C3
111. Which of the following is not an inflammatory mediator?--- Myeloperoxidase
112. Nephrocalcinosis in a systemic granulomatous disease is due to---ns Over production of 1,25
dihydroxy vitamin D
113. Most important bactericidal agent is---Reactive O2 species
114. Bradykinin causes:---- Pain at the site of inflammation
115. Lewis triple response is caused due to:--- Histamine
116. Factor present in the final common terminal complement pathway is:--- C5
117. To which of the following family of chemical mediators of inflammation, the Lipoxins belong?---
Arachidonic acid metabolites
118. Both antibody dependent and independent complement pathway converge on which complement
component?--- C3
119. C-C beta chemokines includes---Eotaxin
120. All of the following are mediators of acute inflammation except----Angiotensin
121. All of the following are mediators of inflammation except:--- Myeloperoxidase
122. Interleukin secreted by macrophages, stimulating lymphocytes is:--- IL-1
123. Cytokines are secreted in sepsis and Systemic Inflammatory Response Syndrome (SIRS) by---
Neutrophils
124. Febrile response in CNS is mediated by---Bacterial toxin
125. Cytokines:--- Includes interleukins
126. Conversion of prothrombin to thrombin requires:--- X and Ca++
127. Which complement fragments are called---C3a and C5a
128. Cryoprecipitate is rich in which of the following clotting factors:--- Factor VIII
129. Most important mediator of chemotaxis is---C5a
130. Histamine causes---Vasodilation
131. Which of the following is found in secondary granules of neutrophils?--- Lactoferrin
132. All are mediators of neutrophils except:--- None
133. Ultra-structurally, endothelial cells contain---Weibel Palade bodies
134. Partial thromboplastin time correlates with:--- Intrinsic clotting pathway
135. Bleeding time assesses:--- Function of platelets
136. The estimation of the prothrombin level is useful in the following clotting factor deficiency, except---
IX
137. Which of the following is secondary mediator of the anaphylaxis is---Leukotriene B4
138. Birbeck’s granules in the cytoplasm are seen in:--- Langerhans cells
139. The Eosinophils secrete all except---Hydrolytic enzyme
140. In Lipooxygenase pathway of the arachidonic acid metabolism, which of the following products helps
to promote the platelet aggregation and vasoconstriction?--- Thromboxane A2
141. Chemotactic complement components are---C5a
142. In inflammatory process, the prostaglandin E1and E2 cause---Vasodilatation
143. Opsonins are---Carbohydrate binding proteins
144. Inflammatory mediator of generalized systemic inflammation is:--- IL-1
145. All are cytokines except:--- Monoclonal antibody
146. An 8 year old girl Geetu presents to the physician with wheezing and difficulty in breathing. The
breathlessness increases when she went to the fields to play with her friends. Her blood contains higher
than normal concentration of IgE. The physician believes that the cell shown in the photograph below is
implicated in the pathogenesis of her condition. This cell is most similar to which of the following white
blood cells?---- Basophil
147. A 72 year-old man Kishori Lal presented to surgery OPD with a history of difficulty in micturition,
increased frequency of urine and lower backache for the past 8 months. Digital rectal examination reveals an
enlarged prostate with irregular surface. The surgeon orders for the serum PSA levels which are found to be
increased and X ray spine shows osteoblastic lesions. A diagnosis of metastatic prostate cancer is made. Mr
Lal also complaints of significant weight loss, loss of appetite and loss of energy over the past 45 days. His
current complaints can be attributed to which of the following---Tumor necrosis factor-a
148. A 14-year-old girl Radha has high grade fever. She goes to a physician Dr. Jeeva Roy who orders for
some blood investigations. A complete blood count with differential implies the presence of a viral infection.
Which of the following best describes the cells that indicate a viral etiology to her illness?--- They are
basophilic with spherical dark-stained nuclei
149. A 28-year-old woman, Vimla is being evaluated to find the cause o her urine turnin a dark rown color
after a recent upper respiratory tract infection. She has been otherwise symptomatic, and her blood
pressure has een within normal limits. Urinalysis finds moderate blood present with red cells and red cell
casts. Immunofluorescence eamination o a renal iopsy reveals deposits of IgA within the mesangium. These
clinical findins suest that her disorder is associated with activation of the alternate complement system.
Which o the ollowin serum laoratory findins is the most suggestive of activation of the alternate
complement system rather than the classic complement system?--- Serum C2 normal, C3 decreased, C4
normal
150. An 18-year-old woman, Sheila is being evaluated for recurrent facial edema, especially around her
lips. She also has recurrent bouts of intense abdo-minal pain and cramps, sometimes associated with
vomiting. Laboratory examination finds decreased C4, while levels of C3, decay-accelerating factor, and IgE
are within normal limits. A deficiency of which one of the following substances is most likely to be associated
with these clinical findings?--- C1 esterase inhibitor
151. Fever occurs due to:--- IL 1
152. E cadherin gene deficiency is seen in:--- Gastric cancer
153. Cell-matrix adhesions are mediated by?—Integrins
154. Pro inflammatory Cytokines include all of the following except:--- Interleukin 6
155. The most important source of histamine---Mast cells
156. Following injury to a blood vessel, immediate haemostasis is achieved by which of the following?----
Vasoconstriction
157. PAF causes all except---Decreased vascular permeability
158. Eosinophils are activated by---IL5
159. Both antibody dependent and independent complement pathway converge on which complement
component?--- C3
160. Cryoprecipitate is rich in which of the following clotting factors?--- Factor VIII
161. Prostaglandins are synthesized from:--- Arachidonic acid
162. Which chemical mediator is an arachidonic acid metabolite produced by cyclo-oxygenase pathway?--
- PGH2
163. Procalcitonin is used as marker of---Procalcitonin
164. The epithelioid cell and multinucleated giant cells of Granulomatous inflammation are derived from:-
-- Monocytes-Macrophages
165. Granuloma is pathological feature of all, except---Microscopic polyangiitis
166. Granulomatous inflammatory reaction is caused by all, except:--- Mycoplasma
167. Non-caseating granulomas are seen in all of the following except---Metastatic carcinoma of lung
168. Epithelioid granuloma is caused by:--- Helper T-cells
169. Caseous necrosis in granuloma are not found in---Leprosy
170. The most important function of epithelioid cells in tuberculosis is:--- Phagocytosis
171. Necrotizing epithelioid cell granulomas are seen in all, except:--- Leprosy
172. Epithelioid granulomatous lesions are found in all of the following diseases, except:--- Pneumocystis
carinii
173. Caseous granuloma is seen in---Histoplasmosis
174. Non-caseating granuloma is characteristically seen in---Sarcoidosis
175. All are granulomatous diseases except---P. carinii
176. Which of the following is the most characteristic of granuloma:--- Epithelioid cell
177. Caseating granuloma are seen in:--- Histoplasmosis
178. A 45-year-old poor man Teja has a chronic cough, a cavitary lesion of the lung, and is sputum
positive for acid-fast bacilli. Which of the following is the principle form of defense by which the patient---
Cell-mediated immunity
179. A 36-year-old man, Avnish presents with a cough, fever, night sweats, and weight loss. A chest X-ray
reveals irregular densities in the upper lobe of his right lung. Histologic sections from this area reveal groups
of epithelioid cells with rare acid-fast bacilli and a few scattered giant cells. At the centre of these groups of
epithelioid cells are granular areas of necrosis. What is the source of these epithelioid cells?--- Monocytes
180. In a granuloma, epithelioid cells and giant cells and derived from---Monocyte – macrophages
181. In a lymph node showing non necrotizing and non–caseating granuloma which of the following is
suspected?--- Toxoplaxmosis
182. Which of these is not a granulomatous disease---Amebiasis
183. Which one of the following statements is not correct regarding---Developmental elasticity
184. An adult old man gets burn injury to his handsOver few weeks, the burned skin heals without the
need for skin graftingThe most critical factor responsible for the rapid healing in this case is:--- Remnant skin
appendages
185. Absolute lymphocytosis is seen in---T.B
186. Which of the following is absolutely essential for wound healing?--- Vitamin C
187. Chronic granulomatous disease is:--- Associated with formation of multiple granulomas
188. In regeneration----Repairing by same type of tissue
189. Wound contraction is mediated by:--- Myofibroblasts
190. A 45-year-old man, Suveen presents with pain in the mid portion of his chestThe pain is associated
with eating and swallowing foodEndoscopic examination reveals an ulcerated area in the lower portion of his
esophagusHistologic sections of tissue taken from this area reveal an ulceration of the esophageal mucosa
that is filled with blood, fibrin, proliferating blood vessels, and proliferating fibroblastsMitosis is easily found,
and most of the cells have prominent nucleoliWhich of the following statements best describes this
ulcerated area?---- Granulation tissue
191. Which of the following adhesion molecules is involved in morphogenesis?--- Tenascin
192. When a cell transforms itself into different lineage the ability us know as:--- Trans-differentiation
193. Prion disease is caused by:--- Misfolding of protein
194. Maximum collagen in wound healing is seen at which stage of healing---End of third week
195. Oval cells : are seen in the stem cells of which of the following tissues?--- Liver
196. First sign of wound injury is:--- Dilatation of capillaries
197. Which of the following is the source of hepatic stem cells?--- Oval cell
198. Vitamin used for post translational modification of glutamic acid to gamma carboxy glutamate is---K
199. Tensile strength of wound after laparoscopic cholecystectomy in a 30 year old woman depends
upon:--- Extensive cross-linking of tropocollagen
200. One of the following statements about hematopoietic stem cell is false?--- Subset of stem cells
normally circulate in peripheral blood
201. All are true about blood coagulation except?---- Extrinsic pathway is activated by contact of plasma
with negatively charged surfaces
202. Vitamin K is responsible for the carboxylation of which amino acid in the clotting factors?---
Glutamate
203. Edema in nephrotic syndrome occurs due to---Decreased serum albumin
204. Thrombomodulin thrombin complex prevents clotting because---The complex removes thrombin
and also activates protein C which inactivates the activated factors V and VIII
205. Vitamin K associated clotting factors are:--- IX, X
206. All endothelial cells produce thrombomodulin except those found in:--- Cerebral microcirculation
207. Which of the following is a procoagulation protein?--- Thrombin
208. All of the following are correct about Thromboxane A2 except---Secreted by WBC
209. Coagulation defects associated with increased coagulation are seen in:--- Protein C resistance
210. All of the following are anticoagulant substances except---Von Willebrand factor
211. Cause of edema is----Decreased plasma protein concentration
212. Endothelium derived relaxing factor (EDRF) is associated with:--- N NOS
213. Which is not involved in local hemostasis?--- Vitamin K
214. Which is the following not synthesized in the liver?--- Factor VIII
215. A 54 year old chronic alcoholic Adhiya Kumar is brought by his son as he has developed progressively
increasing abdominal distension from past 3 months. The physician aspirates the abdominal fluid which is
straw-colored and clear and is found to have protein content (mainly albumin) of 2.3 g/dl. Which of the
following is a major contributor to the fluid accumulation in this patient?--- Decreased oncotic pressure
216. Tissue thromboplastin activates:--- Factor VII
217. Platelet adhesion to collagen is mediated by which of the following---Von willebrand factor
218. Gandy gamma body is typically seen in chronic venous congestion of which of the following?---
Spleen
219. Extrinsic pathway of clotting factors is measured by?--- Prothromin time
220. Histologic sections of lung tissue from 66-year-old woman, Sheena with congestive heart failure and
progressive breathing problems reveal numerous hemosiderin-laden cells within the alveoli. Which of the
following is the cell of origin of these----Macrophages
221. At autopsy, the spleen of a patient is noted to have a thickened capsule and many small, scarred
areas. Microscopic examination of the scarred areas reveals fibrosis with hemosiderin and calcium
deposition. This type of spleen is usually seen in conjunction with which of the following disorders?---
Hepatic cirrhosis
222. Antiphospholipid syndrome is associated with all except:--- Pancytopenia
223. Pale infarct is seen in all except---lungs
224. Congenital hypercoagulability states are all of the followings except----Anti-phospholipid antibody
syndrome
225. Fat embolism is commonly seen in:--- Long bone fractures
226. Virchow’s triad includes all except---Venous thrombosis
227. Hypercoagulability due to defective factor V gene is called:--- Leiden mutation
228. Arterial thrombosis is seen in---Homocysteinemia
229. Hemorrhagic infarction is seen in:--- Venous thrombosis
230. Hyperviscosity is seen in---Cryoglobulinemia
231. Predisposing factor for venous thrombosis:---all
232. Inherited coagulation disorders are:--- Protein S deficiency
233. Which of the following statements about pulmonary emboli is not correct?--- Embolic obstruction of
pulmonary vessels almost always cause pulmonary infarction
234. Which one of the following inherited disorders produces arterial thrombosis?--- Homocysteinemia
235. Heart failure cells are seen in---Chronic venous congestion of lung
236. Necrosis with putrefaction is called as:--- Gangrene
237. Lines of Zahn are found in:--- Thrombus
238. Chicken fat clot is:--- Postmortem clot
239. The five stages of hemostasis are given below in random order. Put them into their correct order. (a)
Clot dissolution (b) Blood coagulation (c) Vessel spasm (d) Clot retraction (e) Formation of platelet plug---
cebda
240. A 64 year old man Ojas Alok Nath resides in a city. He is a known case of hypertension and is a
smoker too. One day, while watching tv, he developed severe pain in the chest. He is rushed to the medical
emergency of the city hospital where this episode is diagnosed as unstable angina. The emergency medical
officer Dr. Smiley Gupta immediately administers him an intravenous preparation of a glycoprotein IIb/IIIa
inhibitor. The mechanism of action of this agent is the ability to----Inhibit platelet aggregation.
241. A 62 year old man Ram Srinath is brought by his wife Shanti Devi after sustaining a fall in the
washroom while taking bath. He has severe pain in his right leg. Dr. Amit Shersia, the orthopedic surgeon
diagnoses it as fracture femur. He is discharged after administration of proper treatment. After about 12
days, Shanti observes that her husband has developed a swollen right leg below the knee. Ram is unable to
move his limb properly and there is presence of tenderness too. Which of the following is the most likely
complication in him?--- Pulmonary thromboembolism
242. A 27-year-old woman, Shama presents with a history of losing pregnancies in the past 5 years. She
also has a history of recurrent pains in her legs secondary to recurrent thrombosis. Her symptoms are most
likely due to a deficiency of which one of the following substances?--- Protein C
243. A 20-year-old male, Akash fractured his right femur. He was admitted to the hospital and over the
next several days developed progressive respiratory problems. Despite extensive medical intervention, he
died 3 days later. At the time of autopsy oil red O-positive material was seen in the small blood vessels of the
lungs and brain. Which of the following was the most likely diagnosis?--- Fat emboli
244. A pregnant woman develops deep, boring pain of her left thigh muscles associated with swelling and
enhanced warmth of the same leg. The pain is worsened by extending the foot. The superficial veins of the
leg are engorged. Her condition puts her at risk for which of the following?--- Pulmonary embolus
245. White infarct is seen in:-- Heart
246. Lines of Zahn occur in which of the following?--- Coralline thrombus
247. White infarcts are seen in the following except:--- Liver
248. All of the following are true about DIC except?--- Decreased prothrombin time
249. The initiating mechanism in endotoxic shock is---Endothelial injury
250. The initiating mechanism in endotoxic shock is---Cytokine release
251. D-Dimer is the most sensitive diagnostic test for:--- Pulmonary embolism
252. Shock lung is characterized by---Diffuse alveolar damage
253. The histological features of shock includes:--- ATN
254. Conditions associated with incoagulable state are:--- Abruption placentae
255. Which of the following is a feature of Disseminated Intravascular Coagulation (DIC)?--- Reduced
plasma fibrinogen
256. A 29 year old woman Ruma is in labour but unfortunately during parturition, the placental
membranes tear and amniotic fluid expressed into a lacerated cervical vein. Which of the following is the
woman most likely to experience immediately following this event?--- Respiratory distress
257. Irshaan has been diagnosed with acute promyelocytic leukemia recently. He presents suddenly in
the medical emergency because of a dangerous complication of his malignancy called disseminated
intravascular coagulation (DIC). In DIC, micro emboli form leading to obstruction of blood vessels and tissue
hypoxia. The common clinical signs observed in patient may be due to which of the following?--- Renal
failure
258. A primiparous woman Ritu at term experiences placental abruption and is rushed to the operating
room for emergency Cesarean section. She develops shortness of breath, cyanosis, and copious bleeding
from her surgical wounds. Levels of which of the following blood components is expected to rise in this
setting?--- Fibrin degradation products
259. A 24-year-old pregnant Heena sustains a placental abruption, and is admitted to the ICU where she
begins bleeding from multiple sites, including oral mucous membranes. Which of the following studies would
be most valuable in assessing this patient---Platelet count, fibrinogen levels, and fibrin degradation products
260. Which of the following features is not shared between ‘T cells’ and B cells’?--- Positive selection
during development
261. CD4 is not important for which of the following?--- None
262. Type 1 MHC presents peptide antigen to T cell , so that peptide binding site is formed by:--- Distal
domain alfa 1 and 2
263. Function of CD4 is all except:--- Immunoglobin production
264. A super-antigen is a bacterial product that----Binds to the beta chain of TCR and MHC class II
molecules of APC stimulating T cell activation
265. Memory T cells can be identified by using the following marker:--- CD45RO
266. All of the following statements about NK cells are true except:--- They are MHC restricted cytotoxic
cells
267. The following feature is common to both cytotoxic T-cells and NK cells:--- Effective against virus
infected cells
268. MHC restriction to antigen presentation is not done for:--- Graft rejection
269. Most potent stimulator of naive T cell is?--- Mature dendritic cell
270. Natural killer cells attacks which of the following cells----Cells which are not able to express MHC1
271. Toll like receptors, recognize bacterial products and stimulates immune response by:----
Transcription of nuclear factor mediated by NFκB which recruits cytokines
272. The following interleukin is characteristically produced in a TH1 response:--- IL-2
273. CD-95 has a major role in:--- Apoptosis
274. The complement is fixed best by which of the following immunoglobulins:--- IgM
275. Antigen presenting cells are which of the following:--- Endothelial cells
276. Antigen presenting cells are:--- Langerhan’s cell
277. Cell surface molecules involved in peripheral tolerance induction are---B7 and CD28
278. Marker for B-Lymphocyte---CD19
279. IL-1 causes---All of the above
280. Antigen presenting cells present in skin are called---Langerhan’s cells
281. Plasma cells---Helps in the formation of antibody
282. The normal ratio of CD4 to CD8 is---2: 1
283. CD4 cells is used to identify which of the following---T cells
284. CD3 is marker for:--- T cell
285. Which of the following is not true about innate immunity?--- Memory is seen
286. Which one of the listed receptors is the type of receptor on leukocytes that binds to pathogen-
associated molecular patterns (PAMPs) and mediates immune response to bacterial lipopolysaccharide?----
Toll-like receptor
287. NK cell CD marker is:---16
288. Immunity against cancer cells:--- NK cells
289. NK cells express:--- CD 16, CD 56
290. Which of the following immune cells have the expression of CD8 on their surface?--- T-cells
291. The following interleukin is characteristically produced in a TH1 response?--- IL-2
292. Most potent stimulator of Naïve T-cells:--- Mature dentritic cells
293. Macroglobulin is derived from:--- B cells
294. Kupffer cells are found in:---liver
295. Birbeck granules are present in---Langerhans cell
296. Which of the following immunoglobulin does not fix complement?--- IgE
297. Autoimmunity in EBV infection is the result of---Polyclonal B cell activation
298. A 14yrs. old girl on exposure to cold has pallor of extremities followed by pain and cyanosis. In later
ages of life she is prone to develop?--- Scleroderma
299. Which is not autoimmune disease?--- Sickle Cell Disease
300. Which among the following is seen in antiphospholipid antibody syndrome?--- Anti glycoprotein
antibody
301. Necrotizing lymphadenitis is seen in---Kikuchi Fujimoto disease
302. Wire loop lesions are seen in:---- SLE
303. Tissue from rat used for detection of antinuclear antibodies?---- Liver
304. Which is not found in CNS in a case of AIDS?--- Inclusion bodies
305. A person present with recurrent swelling on face and lips due to emotional stress. Likely cause is:---
C1 esterase inhibitor deficiency
306. All of the following statements are true about Wiskott Aldrich syndrome except?--- It is an
autosomal recessive disorder
307. Hematoxylin bodies seen in:--- SLE
308. Wire loop lesions are often characteristic for the following class of lupus nephritis:--- Diffuse
proliferative glomerulonephritis (WHO class IV)
309. A renal biopsy from a 56 year old woman with progressive renal failure for the past 3 years shows
glomerular and vascular deposition of pink amorphous material. It shows apple-green birefringence under
polarized light after Congo red staining. These deposits are positive for lambda light chains. The person is
most likely to suffer from:--- Multiple myeloma
310. A young lady presented with bilateral nodular lesions on shins. She was also found to have bi-lateral
hilar lymphadenopathy on chest X-ray. Mantoux test reveals indurations of 5 mms. Skin biopsy would
reveal:--- Non caseating Granuloma
311. Anti ds-DNA antibodies are commonly seen in:--- SLE
312. Low complement levels seen in:--- PSGN
313. Which is seen in Chediak-Higashi syndrome:--- Neutropenia
314. Adenosine deaminase deficiency is seen in:--- Severe combined immunodeficiency
315. True about alpha-1 antitrypsin deficiency, is/are:---- Pulmonary emphysema
316. All are true regarding Hyper IgE syndrome exept:--- High serum IgE with low IgG, IgA and IgM
317. All are true about Wiskott- Aldrich Syndrome except:--- Large size platelets
318. Diagnosis of X linked Agammaglobulinemia should be suspected if---Absent tonsils and no palpable
lymph nodes on physical examination
319. Which of the following cell types is not a target for initiation and maintenance of HIV infection?---
Neutrophil
320. All of the following are found in SLE except:--- Leucocytosis
321. Which of the following immunoglobulin is absent in Ataxia telangiectasia:--- IgA
322. Scl-70 antibody is characteristic of---Scleroderma
323. LE cell phenomenon is seen in----Neutrophil
324. Most sensitive test for screening of----Anti nuclear factor (ANF
325. According to WHO, the feature of class II lupus is----Hematuria
326. ANCA antibody with peripheral rim distribution is indicative of---Anti double stranded DNA antibody
327. Basic pathology in cystic fibrosis is---Defect in the transport of chloride across epithelia
328. Besbuer Boeck Schaumann disease is also called as---Sarcoidosis
329. Most common viral antigen used for diagnosis of HIV in blood before transfusion is---p24
330. Most common vascular tumor in AIDS patients is---Kaposi
331. Which in not an autoimmune disease---Syphilis
332. Bilateral parotid gland enlargement is seen in all except.---sle
333. Sarcoidosis does not involve---Brain
334. Characteristic of SLE of kidney is----Wire loop lesions
335. Libman-Sacks endocarditis is seen in-----sle
336. Chediak-Higashi syndrome is due to defect in:----LAD
337. Anti-double stranded DNA is highly specific for:---SLE
338. Anti-topoisomerase I is marker of:--- Systemic sclerosis
339. An 8-year-old boy presents with sarcoidosis. Which of the following is correct?--- Hilar
lymphadenopathy with perihilar calcification
340. Most common site for lymphoma in AIDS patients is----CNS
341. All are true about histological features of Kaposi---Nodule is the initial lesion of Kaposi
342. HIV affects which of the following most commonly?--- Helper cells
343. Which of the following lesions/conditions shows most specific anatomic changes in HIV infection?---
CNS lesions
344. A 21-year-old female Pallavi Kumari comes to the physician because of migratory arthralgia and a
skin rash that is exacerbated by sun exposure. Her urinalysis shows moderate proteinuria and red blood cell
casts. Serum autoantibodies with high specificity for this patient’s condition react with---Double stranded
DNA
345. A 50-year-old woman, Seeta presents with dry eyes, a dry mouth, and difficulty swallowing solid
food. Physical examination finds enlargement of her parotid glands along with marked dryness of her buccal
mucosa. Laboratory examination finds the presence of both SS-A and SS-B anti bodies. A biopsy of her lip is
likely to show infiltration of minor salivary glands by what type of inflammatory cell?--- Lymphocyte
346. A 16-year-old boy, Raju is being evaluated for failure to have a growth spurt and the recent
development of signs of premature aging. Physical examination finds the boy to be short with thin skin and
muscle autopsy. The skin of his face is wrinkled and his lips appear atrophic. In the last year, he also has
developed bilateral cataracts and early signs of osteoporosis. None of these signs were present in his first
decade of life. Which of the following is the most likely diagnosis?--- Werner syndrome
347. A 7-month-old baby boy Guddu is evaluated because of repeated episodes of pneumococcal
pneumonia. Serum studies demonstrate very low levels of IgM, IgG, and IgA. This patient’s condition is
thought to be related to a deficiency of which of the following proteins?--- Tyrosine kinase
348. A patient with systemic lupus erythematosus very much wants to become pregnant. What should
her physician tell her regarding pregnancy in lupus patients?--- There may be an increase in spontaneous
abortions and prematurity
349. A woman Kamlesh with swelling of the oral mucosa and dry mouth is found to have intense
destructive inflammation of the salivary glands and antibodies against the ribonucleoprotein La. Which of
the following clinical findings would most likely be associated with this syndrome?--- Conjunctivitis
350. A female being diagnosed with SLE has undergone a biopsy of ‘butterfly rash’. Which is most likely to
demonstrate which of the following?--- Granular complement and IgG at the dermalepidermal junction
351. A terminally ill HIV infected patient develops focal neurologic signs, dementia, and coma. Amoebic
parasites are demonstrated in CSF. Which of the following organisms is most likely to be the causative
agent?--- Acanthamoeba sp
352. A 21-year-old male Rohan presents with complaints of dull lower back pain and morning stiffness.
There is tenderness over the costosternal junctions, spinous processes of the vertebrae, and the iliac crests.
Which of the following tests be helpful in establishing a diagnosis of ankylosing spondylitis?--- HLA typing
353. A 27-year-old man Alok Kumar with AIDS develops a reddish, slightly raised rash on his face, neck,
and mouth, consistent in appearance with Kaposi’s sarcoma. Microscopically, the proliferating cells in this
malignancy most closely resemble which of the following?---- Angiosarcoma
354. Which of the following autoantibody is specific for SLE?--- Ds DNA
355. Which of the following autoantibody is least likely associated with SLE---Anti topoisomerase
356. Most common CNS neoplasm in HIV patient is:--- Primary CNS Iymphoma
357. Regarding severe combined immunodeficiency disease, which of the following statement is true?---
Adenosine deaminase deficiency
358. Which of the following is a finding in lymphoid tissues in individuals with common variable
hypogammaglobulinemia?--- Normal B cell count
359. Thymic hypoplasia is seen in which of the following?--- Digeorge syndrome
360. Onion peel appearance of splenic capsule is seen in---SLE
361. Following is not a feature of AIDS related lymphadenopathy:--- Haematoxylin bodies
362. A false negative tuberculin reaction may be obtained in all of the following situations EXCEPT:----
Children previously tested with tuberculin test
363. Risk of HIV transmission is not seen with---Plasma derived Hepatitis B vaccine
364. All of the following methods are used for the diagnosis of HIV infection in a 2months old child,
except---HIV ELISA
365. Mantoux test reading of less than 5 mm indicates:--- Disseminated TB
366. The poly-arthritic condition that is NOT common in males---Systemic lupus erythematosus
367. Epitope spreading refers to---A mechanism for the persistence and evolution of autoimmune disease
368. Heerfordt’s syndrome consists of fever, parotid enlargement, facial palsy and---Anterior uveitis
369. HIV affects CD4 cells by which protein---GP 120
370. Treatment for Asymptomatic HIV is done when CD4 count is belowTreatment for Asymptomatic HIV
is done when CD4 count is below----350
371. Hodgkins lymphoma caused for by---EBV
372. True about MHC-class II:––– Not involved in innate immunity
373. MHC-II positive cells are all except:––– T cells
374. True about MHC:––– Transplantation reaction
375. Epitope binding floor of the MHC molecule conists of---Alpha helices
376. MHC class I are present on all except---RBCs
377. HLA B27 is not seen in which of the following?--- Rheumatoid arthritis
378. The role played by Major Histocompatibility Complex 1 and 2:--- Present antigens for recognition by
T cell antigen receptors
379. Antigen presented along with HLA class II stimulate---CD4 cell
380. Major histocompatibility complex class I is seen on which of the following cell?--- All blood cells
except erythrocytes
381. What type of hypersensitivity reaction is seen in myasthenia gravis?---- Type 2 hypersensitivity
reaction
382. Hemolytic disease of newborn is an example of:--- Type 2 hypersensitivity reaction
383. Raji cell assays are used to quantitate:--- Immune complexes
384. Hypersensitivity pneumonitis is classically a/an:--- Immune complex mediated hypersensitivity
385. Immunoglobulin involved in type I hypersensitivity reaction is:--- IgE
386. Arthus reaction is what type of hypersensitivity reaction:---- Localized immune complex
387. A 40 year old man has chronic cough with fever for several months. The chest radiograph reveals a
diffuse reticulondular pattern. Microscopically on transbronchial biopsy there are focal areas of
inflammation containing epitheloid cell granuloma, Langhans giant cells, and lymphocytes. These findings
are typical for which of the following type of hypersensitivity immunologic responses:--- Type IV
388. Ram Devi presented with generalized edema sweating and flushing tachycardia and fever after bee
sting. This is:--- IgE mediated reaction
389. Example of Type IV Hypersensitivity is/are:--- Contact hypersensitivity
390. Example of Type II Hypersensitivity is/are:--- Blood transfusion reaction
391. Which of the following diseases is/are mediated through complement activation---Necrotizing
vasculitis
392. Which of following statements is not true about Mycobacterium tuberculosis infection---
Lymphocytes are the primary cells infected by M. tuberculosis
393. A man after consuming sea food develops rashes. It is due to:--- IgE mediated response
394. Granuloma in Sarcoidosis is called---Hard sore
395. Myasthenia gravis may be associated with----All of the above
396. Which of the following type of hypersensitivity reaction is found in blood transfusion reaction?---
Cytotoxic type
397. Which of the following type of hypersensitivity reactions occurs in Farmer's lung?--- Type III
398. Tuberculin test positivity indicates---Good cell mediated immunity
399. Myasthenia gravis is most commonly associated with which of the following?--- Thymic hyperplasia
400. Cell mediated immunity is:--- Type IV
401. Antibody found in patients with myasthenia gravis is directed against---Acetylcholine receptors
402. Hyperacute rejection is due to---Preformed antibodies
403. All are affected in Graft-Versus host reaction:--- Lung
404. Preformed antibodies cause:--- Hyperacute rejection
405. True about graft versus host disease is:---- Associated with solid organ transplantation
406. Acute humoral renal transplant rejection is characterized by the following, except:--- Interstitial and
tubular mononuclear cell infiltrate
407. Transfer of the graft of different species are called as---Xenograft
408. Acute graft versus host disease reaction occurs in all except---Adrenal
409. Preformed antibodies cause:--- Chronic rejection
410. Principal cause of death in renal transplant patient is---Infection
411. Secondary amyloidosis is associated with----AA
412. A 60 year old female is suffering from renal failure and is on hemodialysis since last 8 years. She
developed carpal tunnel syndrome. Which of the following finding will be associated?--- P2 microglobulin
413. The best investigation for the diagnosis of amyloidosis is---Rectal biopsy
414. Which type of amyloidosis is caused by mutations in transthyretin gene?--- Familial amyloidosis
polyneuropathy
415. In Hemodialysis associated amyloidosis, which of the following is seen---B2 Microglobulin
416. Bone marrow in AL amyloidosis shows---Bone marrow plasmacytosis
417. A diabetic patient is undergoing dialysis. Aspiration done around the knee joint would show:--- A
beta 2 microglobulin
418. What is the best method for confirming amyloidosis?--- Rectal biopsy
419. Neointimal hyperplasia causes vascular graft failure as a result of hypertrophy of:--- Smooth muscle
cells
420. Which one of the following stains is specific for Amyloid?---- Congo red
421. In amyloidosis Beta pleated sheet will be seen in:-- X-ray crystallography
422. A 50-year-old presented with signs and symptoms of restrictive heart disease. A right ventricular
endomyocardial biopsy revealed deposition of extracellular eosinophilic hyaline material. On transmission
electron microscopy, this material is most likely to reveal the presence of:--- Non branching filaments of
indefinite length
423. Amyloid deposits stain positively with all of the following EXCEPT:--- Methanamine silver
424. On electron microscopy amyloid characteristically exhibits:--- 7.5-10 nm fibrils
425. Familial amyloidotic polyneuropathy is due to amyloidosis of nerves caused by deposition of:--
Mutant transthyretin
426. Lardaceous spleen is due to deposition of amyloid in:--- Sinusoids of red pulp
427. What are the stains used for Amyloid?--- Thioflavin
428. Gingival biopsy is useful in the diagnosis of:--- Amyloidosis
429. Amyloid is---Glycoprotein
430. Serum amyloid associated protein is found in---Chronic inflammatory states
431. Most common site of biopsy in amyloidosis---Kidney
432. T-lymphocytes from a 6-year-old female Ramya with severe recurrent respiratory infections are
found to lack the IL12 receptor. Supplementation with which of the following substances would be most
helpful in treating this patient---Interferon-g (IFN-g)
433. A 43-year-old women Kanata Devi presents with a several year history of progressive abdominal
colic and constipation. Colonic biopsy stained with Congo red reveals the acellular material exhibiting green
birefringence. The birefringence is thought to be most closely related to which of the following protein
properties?--- Beta-pleated sheet tertiary structure
434. Correctly matched pairs in amyloidosis are:-- Multiple myeloma - light chain
435. Which of the following is the chemical nature of Hemodialysis associated with amyloid?----BETA
436. A diabetic patient is undergoing dialysis. Aspiration done around the knee joint would show:--- A-p2
Microglobulin
437. Amyloidosis is most commonly seen in:--- Type 2 DM
438. Which of the following is the most serious organ involvement in amyloidosis?--- Renal tissue
439. Which type of Amyloidosis is caused by mutation of the transthyretin protein?--- Familial
amyloidotic polyneuropathy
440. Cause of death in amyloidosis involving kidney:--- Sepsis
441. Secondary amyloidosis complicates which of the following:---- Chronic osteomyelitis
442. On Congo- red staining, amyloid is seen as:-- Brilliant pink color
443. Lardaceous spleen is due to deposition of amyloid in:--- Sinusoids of red pulp
1. All are true about Fanconi anemia, except:
• Defect in DNA repair
• Bone marrow hyper function
• Congenital anomaly present
• Increased chances of cancer

2. HER2/neu receptor plays a role in


• Predicting therapeutic response
• Diagnosis of breast cancer
• Screening of breast cancer
• Recurrence of tumor

3. The most common secondary malignancy in a patient having retinoblastoma is:


• Osteosarcoma
• Renal cell carcinoma
• Pineoblastoma
• Osteoblastoma

4. Regarding Fanconi anemia, the wrong statement is:


• Autosomal dominant
• Bone marrow show pancytopenia
• Usually aplastic anemia
• It is due to defective DNA repair

5. True statements about p53 gene are all except:


• Arrests cell cycle at G1 phase
• Product is 53 kD protein
• Located on chromosome 17
• Wild/non-mutated form is associated with increased risk of childhood tumors

6. Growth factor oncogene is:


• Myc
• Fos
• Sis
• Jun

7. Rosettes are characteristically seen in


• Retinoblastoma
• Melanoma
• Dysgerminoma
• Lymphoma

8. The normal cellular counterparts of oncogenes are important for the following functions, except:
• Promotion of cell cycle progression
• Inhibition of apoptosis
• Promotion of DNA repair
• Promotion of nuclear transcription

9. An example of a tumor suppressor gene is:


• Myc
• Fos
• ras
• Rb

10. Lynch syndrome is associated with cancers of the:


• Breast, colon, ovary
• Breast, endometrium, ovary
• Breast, colon, endometrium
• Colon, endometrium, ovary

11. Loss of heterozygosity associated with:


• Acute myeloid leukemia
• ALL
• Retinoblastoma
• Promyelocytic leukemia

12. Which is not a tumor suppressor gene?


• WT-1
• Rb
• P53
• Ras

13. The inheritance pattern of familial Retinoblastoma is:


• Autosomal recessive
• Autosomal dominant
• X-linked dominant
• X-linked recessive
14. Which of the following is known as the “guardian of the genome”?
• P53
• Mdm2
• P14
• ATM

15. The following statements are true about Tumor Suppressor Gene p53, except:
• It regulates certain genes involved in cell cycle regulation
• Its increased levels can induce apoptosis
• Its activity in the cells decreases following UV irradiation and stimulates cell cycle
• Mutations of the p53 gene are most common genetic alteration seen in human cancer

16. In the mitogen activated protein kinase pathway, the activation of RAS is counteracted by:
• Protein kinase C
• GTPase activating protein
• Phosphatidyl inositol
• Inositol triphosphate

17. Which of the following mutations in a tumor suppressor agent causes breast carcinoma?
• p43
• P53
• p73
• P83

18. True about proto-oncogenes is:


• Important for normal cell growth
• May get converted into oncogenes
• C-myc over-expression causes lymphoma
• Their mutation causes retinoblastoma

19. True about oncogene is:


• Present in normal cell
• They are of viral origin
• They are transduced from virus infected cells
• P53 is most common oncogene mutation causing malignancy

20. Cancer cell survival is enhanced by:


• Suppression of p53 protein
• Over expression of p53 gene
• bcl-2
• Bax

21. Following are required for normal growth


• Proto-oncogenes
• Tumor suppressor genes
• Oncogenes
• DNA repair genes

22. Xeroderma pigmentosum is caused due to a group of closely related abnormalities in:
• Mismatch repair
• Base excision repair
• Nucleotide excision repair
• SOS repair

23. Increased expression of which of the following causes oncogenesis


• IGF receptor
• EGF receptor
• GH receptor
• Aldosterone receptor

24. Tumor suppressor genes are all, except


• APC
• p53
• Rb
• C-myc

25. Angiogenesis is
• Formation of the new blood vessels
• Repair by connective tissues
• Formation of the blood clot
• All of the above

26. Medullary carcinoma of thyroid is associated with mutation in:


• RET
• RAS
• NF
• Rb
27. APC gene is located on which chromosome
• Chromosome 5
• Chromosome 6
• Chromosome 9
• Chromosome 11

28. Proto-oncogene erb-B is not related to:


• Breast carcinoma
• Small cell lung carcinoma
• Non-small cell lung carcinoma
• Ovarian carcinoma

29. Most common genetic mutation in carcinogenesis involves:


• p53
• Rb
• HPC
• PTEN

30. Which of the following is a true statement regarding the above gene?
• It is associated with the
• It is called as
• It is active in hypopohosphorylated form
• It is active in hyperphosphorylated form

Aisha, a 51 year old woman discovers a lump in her left breast on a weekly self-examination. Mammography is
performed which confirms the presence of a suspicious “mass”, and needle core biopsy is performed to determine
whether the mass is malignant. Dr. Devesh, the pathologist confirmed the mass to be malignant and said that the
tissue demonstrates amplification of her-2/ neu oncogene. What kind of protein is the gene product of Her-2/neu

• GTPase
• GTPase-activating protein
• Receptor tyrosine kinase
• Retinoic acid receptor protein

31. A patient Madhu undergoes total thyroidectomy for a mass lesion of the thyroid. During the surgery it is
found that the parathyroid glands appeared enlarged. The thyroid lesion shows neuroendocrine-type cells
and amyloid deposition. This patient’s thyroid and parathyroid lesions may be related to which of the
following oncogenes?
• Bcl-2
• C-myc
• Ret
• L-myc
32. Dr. Marwah, a pediatrician, performing an ophthalmoscopic examination on a four-year-old boy, notices
several small pigmented nodules in his irises. He also notices six light brown macules on the trunk of the
child of variable sizes. This boy may have a propensity to develop tumors in which of the following
structures?
• Bladder
• Colon
• Peripheral nerve
• Skin

33. RET gene mutation is associated with which malignancy?


• Pheochromocytoma
• Medullary carcinoma thyroid
• Lymphoma
• Renal cell carcinoma

34. Endometrial carcinoma is associated with which of the following tumor suppression gene mutation?
• P53
• Rb
• PTEN
• APC

35. The tumor suppressor gene p 53 induces cell cycle arrest at:
• G2
• S
• G1 – S phase
• G0 phase

36. MYC gene is:


• Protein kinase inhibitor
• Growth factor inhibitor
• GTPase
• Transcription activator

37. Retinoblastoma is associated with which of the following tumours?


• Osteoclastoma
• Hepatocellular cancer
• Squamous cell cancer
• Osteosarcoma
38. An example of a tumour suppressor gene is:
• Myc
• Fos
• Ras
• RB

39. Which of the following is DNA repair defect?


• Retinoblastoma
• Neurofibromatosis
• Xeroderma pigmentosum
• MEN-I

40. Which of the following gene defect is associated with development of medullary carcinoma of thyroid:
• RET Proto Oncogene
• Fap gene
• Rb gene
• BRCA 1 gene

41. All of the following are tumor markers, except:


• Beta-2 macroglobulin
• HCG
• Alpha-fetoprotein
• CEA

42. Knudson two hit hypothesis is seen with


• Melanoma
• Retinoblastoma
• Ulcerative colitis
• Crohn disease
43. Retinoblastomas arising in the context of germ-line mutations not only may be bilateral, but also may be
associated with (so called “trilateral” retinoblastoma)
• Medulloblastoma
• Pinealoblastoma
• Neuroblastoma
• Hemangioblastoma

44. Post transplant lymphoma is caused by which of the following?


• CMV
• EBV
• Herpes simplex
• HHV-6

45. H. pylori infection is associated with development of which malignancy:


• MALTomas
• Atherosclerosis
• Sarcoma
• Gastrointestinal stromal tumor (GIST)

46. Helicobacter pylori infection is associated with all of the following conditions, except:
• Peptic ulcer disease
• Gastric adenocarcinoma
• B cell lymphoma
• Burkitt’s lymphoma

47. Tumors associated with organisms are all except:


• Hepatocellular cancer
• Non-small Cell Carcinoma of Lung
• Gastric cancer
• Nasopharyngeal cancer

48. Which of the following is essential for tumor metastasis?

• Angiogenesis
• Tumorogenesis
• Apoptosis
• Inhibition of tyrosine kinase activity

49. Which of the following statements about carcinogenesis is false?


• Asbestos exposure increases the incidence of lung cancer
• Papilloma viruses produce tumors in animals but not in humans
• Exposure to aniline dyes predisposes to cancer of the urinary bladder
• Hepatitis B virus has been implicated in hepatocellular carcinoma

50. Smoking is a risk factor for all carcinomas , except;


• Oral
• Bronchial
• Bladder
• Thyroid
51. Workers exposed to polyvinyl chloride may develop following liver malignancy
• Cholangiocarcinoma
• Fibrolamellar carcinoma
• Angiosarcoma
• All of the above
52. Which among the following is not a neoplastic virus:
• Cytomegalovirus
• Hepatitis B virus
• Human papilloma virus
• All of these

53. A 37-year-old man, Gagan presents with increasing abdominal pain and jaundice. He gives a history of
intake of groundnuts which did not taste appropriate. Physical examination reveals a large mass involving
the right side of his liver, and a biopsy specimen from this mass confirms the diagnosis of liver cancer
(hepatocellular carcinoma). Which of the following substances is most closely associated with the
pathogenesis of this tumor?
• Aflaxotin B1
• Direct-acting alkylating agents
• Vinyl chloride
• Azo dyes

54. Biopsy of an ulcerated gastric lesion of a 26-year-old smoker Akki demonstrates glands containing cells with
enlarged, hyperchromatic nuclei below the muscularis mucosa. Two tripolar mitotic figures are noted. With
which of the following infectious agents has this type of lesion been most strongly associated?
• Epstein-Barr virus
• Helicobacter pylori
• Human papilloma virus
• Molluscum contagiosum virus

55. A man Alok Nath contracts HTLV-1 infection through sexual contact. Twenty-one years later he develops
generalized lymphadenopathy with hepatosplenomegaly, a skin rash, hypercalcemia, and an elevated white
blood count. This man has most likely developed which of the following?
• AIDS
• Autoimmunity
• Delayed hypersensitivity reaction
• Leukemia

56. Thorium induced tumor is which of the following?


• Renal cell carcinoma
• Lymphoma
• Angiosarcoma of liver
• Astrocytoma

57. Radiation exposure during infancy has been linked to which one of the following carcinoma?
• Breast
• Melanoma
• Thyroid
• Lung

58. The following parasitic infections predispose to malignancies?


• Paragonimus westermani
• Guinea worm infection
• Clonorchiasis
• Schistosomiasis

59. Kaposi’s sarcoma is seen with


• HCV
• HPV
• HSV
• HHV

60. UV radiation has which of the following effects on the cells?


• Prevents formation of pyrimidine dimers
• Stimulates formation of pyrimidine dimers
• Prevents formation of purine dimers
• All of the above

61. The most radiosensitive cells are:


• Neutrophils
• Lymphocytes
• Erythrocytes
• Megakaryocytes

62. The SI unit of radiation absorbed dose is


• Rad
• Becquerel
• Gray
• Sievert

63. One of the following leukemia almost never develops after radiation?
• Acute myeloblastic leukemia
• Chronic myeloid leukemia
• Acute lumphoblastic leukemia
• Chronic lymphocytic leukemia

64. A 20 year old female was diagnosed with granulose cell tumor of the ovary. Which of the following bio
markers would be most useful for follow-up of patient?
• CA 19-9
• CA50
• Inhibin
• Neuron – specific enolase

65. Alpha fetoprotein is a marker of:


• Hepatoblastoma
• Seminoma
• Renal cell carcinoma
• Choriocarcinoma
66. Hyperglycemia associated with:
• Multiple myeloma
• Ewing sarcoma
• Osteosarcoma
• Chondroblastoma

67. Which of the following is Not associated with thymoma?


• SIADH
• Myasthenia gravis
• Polymyositis
• Hypogammaglobinemia

68. Which of the following is not true about Neuroblastoma?


• Most common extracranial solid tumor in childhood
• >50% patients present with metastasis at time of diagnosis
• Lung metastases are common
• Involve aorta and its branches early

69. Migratory thrombophlebitis is associated with all of t


• Medullary carcinoma of thyroid
• Pancreatic neuroendocrine tumor
• Pheochromocytoma
• Gastrointestinal stromal tumor

70. All of the following are examples of tumor markers, except:


• Alpha-HCG (a-HCG)
• Alpha-Feto protein
• Thyroglobulin
• Beta 2-microglobulin

71. Which of the following tumors have an increased elevation of placental alkaline phosphatase in the serum
as well as a positive immunohistochemical staining for placental alkaline phosphatase?
• Seminoma
• Hepatoblastoma
• Hepatocellular carcinoma
• Peripheral neuroectodermal tumor

72. In tumor lysis syndrome, all of the following are seen, except:
• Hypernatremia
• Hypercalcemia
• Hyperkalemia
• Hyperphosphatemia

73. Uses of tumor marker are:


• Screening of a cancer
• Follow up of a cancer patient, esp. for knowing about recurrence
• Confirmation of a diagnosed cancer
• For monitoring the treatment of a cancer

74. True about Carcinoembryonic antigen (CEA):


• Useful for screening of carcinoma colon
• Gives confirmative evidence of Ca. colon
• Helpful for follow-up after resection
• Levels decrease immediately after resection of tumor

75. CA·125 is associated with:


• Colon ca
• Breast ca
• Ovarian ca
• Bronchogenic ca

76. Secondaries are common in all, except:


• Skull
• Hand and feet bones
• Proximal limb bones
• Pelvic
77. Hybridoma refers to
• Collision tumor
• A tumor of brown fat
• A hamartoma
• A technique for raising monoclonal antibodies

78. BCL2 is a marker for:


• Follicular lymphoma
• Mycosis fungoides
• B-cell lymphoma
• Mantle cell lymphoma

79. alfa-fetoproteins are a marker of:


• Secondaries in liver
• Cholangiocarcinoma
• Hepatoma
• None of the above

80. Increased level of alpha fetoprotein is found in


• Yolk sac tumor
• Seminoma
• Teratoma
• Choriocarcinoma

81. Migratory thrombophlebitis is seen in:


• Disseminated cancer
• Rheumatic heart disease
• Libman-Sachs endocarditis
• All of the above

A 65 years old male diagnosed by biopsy a case of lung carcinoma, with paraneoplastic syndrome and
increased calcium. Probable cause is

• Parathyroid hormone
• Parathyroid hormone related peptide
• Calcitonin
• Calcitonin related peptide

82. Which is associated with polycythemia:


• Gastric carcinoma
• Fibrosarcoma
• Cerebellar hemangioblastoma
• All
83. Serum AFP is increased in all, except:
• Acute hepatitis
• Hepatocellular carcinoma
• Hepatoma
• Bladder carcinoma

84. Carcinoembryonic antigen is elevated in all, except:


• Alcoholic cirrhosis
• Ca colon
• Ulcerative colitis
• Emphysema
85. Desmoid tumor arises from
• Wall of the intestine
• Anterior abdominal wall
• Submucosa
• Appendix

86. Alpha-fetoprotein is a tumor marker of:


• Carcinoma ovary
• Liver malignancies
• Endodermal sinus tumor of testis
• Both (b) and (c)

87. A-fetoprotein is seen in all except:


• Hepatocellular carcinoma
• Carcinoma colon
• Pancreatic carcinoma
• Germ cells of testes

88. The diagnostic tumor marker of liver carcinoma is:


• CEA
• AFP
• CA - 125
• All of the above

89. Spontaneous regression of tumor is seen in:


• Wilm’s tumor
• Neuroblastoma
• Acute monocytic leukemia
• Hepatoblastoma

90. All of the following about tumor markers are properly matched, except:
• Prostate cancer - PSA
• Colon cancer - CEA
• Ovarian cancer – CA 125
• Cholangiocarcinoma - AFP

91. Popcorn calcification is seen in:


• Chondrosarcoma
• Fibrous dysplasia
• Osteoblastoma
• Wilms’ tumor

92. Which one of the following is a frequent cause of serum alpha- fetoprotein level greater than 10 times the
normal upper limit?
• Seminoma
• Hepatocellular carcinoma of liver
• Cirrhosis of liver
• Oat cell tumor of lung

93. Rise of AFP is noted in all except:


• Hepatocellular carcinoma
• Cirrhosis
• Germ cell tumor
• Kidney tumor

94. Catecholamines are increased in:


• Neuroblastoma
• Retinoblastoma
• Medulloblastoma
• Nephroblastoma

95. A 60-year-old man, Shibu is found to have a 3.5-cm mass in the right upper lobe of his lung. A biopsy of this
mass is diagnosed as a moderately differentiated squamous cell carcinoma. Workup reveals that no bone
metastases are present, but laboratory examination reveals that the man’s serum calcium levels are 11.5
mg/dL. This patient’s paraneoplastic syndrome is most likely the result of the ectopic production of which of
the following substances?
• Parathyroid hormone
• Parathyroid hormone-related peptide
• Calcitonin
• Calcitonin-related peptide

96. During a routine physical examination, a 45-year-old woman Nusheen is noted to have a ruddy complexion.
Her hematocrit is 52%. Her lungs are clear and she does not smoke. Serum erythropoietin levels are
elevated. Cancer of which of the following organs is the most likely cause of her increased hematocrit?
• Breast
• Colon
• Kidney
• Stomach
97. A 62 year-old woman Omvati with advanced, metastatic lung cancer develops profound fatigue and
weakness and alternating diarrhea and constipation. Physical examination demonstrates
hyperpigmentation of skin, even in areas protected from the sun. Tumor involvement of which endocrine
organ is most strongly suggested by this patient’s presentation?
• Adrenal gland
• Endocrine pancreas
• Ovaries
• Pituitary gland

98. An old man Velu presents with complaints of abdominal and back pain, malaise, nausea, 8 kg weight loss
and weakness, which have been present for 3 or 4 months. His history also reveals several episodes of
unilateral leg swelling, which have involved both legs at different times. These findings are most consistent
with which of the following diagnoses?
• Pancreatic cancer
• Primary sclerosing cholangitis
• Splenic infarction
• Reflux esophagitis

99. A 65-year-old woman Ramkali presents to the emergency room with a pathologic fracture of the shaft of
her humerus. X-ray studies demonstrate multiple lytic and blastic bone lesions. Biopsy of one of these
lesions shows adenocarcinoma. Which of the following is the most likely source of the primary tumor?
• Breast
• Colon
• Kidney
• Lung

100. Tumor that follows rule of 10 is:


• Pheochromocytoma
• Oncocytoma
• Lymphoma
• Renal cell carcinoma

101. Which of the followingis a squamous cell carcinoma marker?


• Vimentin
• Desmin
• Cytokeratin
• Glial fibrillary acid protein
102. Marker for ovarian carcinoma in serum is:
• CA-125
• Fibronectin
• Acid Phosphatase
• PSA

103. Which of the following is a marker for carcinoma of lung and breast?
• CEA
• AEP
• HCG
• CA-15-3

104. Secondaries of all the following cause osteolytic lesions except:


• Prostate
• Kidney
• Bronchus
• Thyroid

105. Sacrococcygeal teratoma, marker is:


• CEA
• β- HCG
• S100
• CA-125

106. Which of the following mutation is seen in malignant melanoma


• N-myc
• CDKN2A
• RET
• Rb

107. Marker of small cell cancer of lung is:


• Chromogranin
• Cytokeratin
• Desmin
• Vimentin

108. Which of the following is tumor marker of seminoma?


• AFP
• LDH
• PLAP
• HCG
109. Which of the following is a special stain for rhabdomyosarcoma?
• Cytokeratin
• Synaptophysin
• Desmin
• Myeloperoxidase

110. The most common cause of malignant adrenal mass


• Adrenocortical carcinoma
• Malignant Phaeochromocytoma
• Lymphoma
• Metastasis from another solid tissue tumor

111. Commonest cancer in which metastasis is seen in brain in


• Breast
• Lung
• Kidney
• Intestines

112. Which of the following is incorrect about neuro- blastoma ?


• Most common abdominal tumor in infants
• X-ray abdomen shows calcification
• Can show spontaneous regression
• Urine contains 5H.I.A.A

113. Colloid degeneration is occurs in:


• Salivary gland
• Prostate gland.
• Thyroid gland
• Pituitary gland

114. An 87-year-old male develops worsening heart failure. Workup reveals decreased left ventricular
filling due to decreased compliance of the left ventricle. Two months later the patient dies, and postmortem
sections reveal deposits of eosinophilic, Congo red positive material in the interstitial of his heart. When
viewed under polarized light. This material displays an apple-green birefringence. What is the correct
diagnosis?
• Amyloidosis
• Glycogenosis
• Hemochromatosis
• Sarcoidosis
115. In cases of renal failure on long-term hemodialysis, there is development of following type of
amyloid:
• Amyloid light chain (AL)
• Amyloid-associated protein (AA)
• Amyloid β2 microglobulin (Aβ2m)
• β amyloid protein (Aβ)

116. The most common form of amyloid in third world countries is:
• Primary
• Secondary
• Hereditary
• Localized

117. Cardiac amyloidosis often produces:


• Dilated cardiomyopathy
• Constrictive cardiomyopathy
• Restrictive cardiomyopathy
• Ischemic cardiomyopathy

118. In senile cardiac amyloidosis, the biochemical form of amyloid is:


• AL
• AA
• ATTR
• Aβ2m

119. Health risk in obesity is due to weight in excess of the following for age and sex:
• 0.1
• 0.2
• 0.3
• 0.4

120. Obesity is due to:


• Hyperplasia of adipocytes only
• Hypertrophy of adipocytes only
• Hyperplasia as well as hypertrophy of adipocytes
• Fatty change in liver only

121. Most often secondary amyloidosis occurs when the following pathologies:
• Chronic suppuration
• Acute inflammation
• Cellular necrosis
• Hyaline degeneration

122. The substance with fibrillar structure, which forms under the pathological conditions, is:
• Reabsorption droplets
• Russell bodies
• Lipids
• Amyloid

123. The substance giving red color with the Congo red stain is:
• Lipid
• Hyaline
• Water
• Amyloid

124. In long-standing hypertension and diabetes mellitus, the walls of arterioles, especially in the kidney,
become:
• Serous
• Thined
• Hyalinized
• Ulcered

125. The pathologic proteinaceous substance, accumulating only between cells in various tissues and
organs of the body is:
• Glycogen
• Hyaline
• Water
• Amyloid

126. The pathologic proteinaceous substance, accumulating both within cells and in the extracellular
matrix in various tissues and organs of the body is:
• Glycogen
• Hyaline
• Water
• Amyloid
127. The characteristics of amyloid fibrils include all of the following, except:
• Fibril composed of paired filaments
• Nonbranching fibrils
• Fibrils with an indefinite diameter
• Fibrils with definite length

128. The organ affected in both primary and secondary amyloidosis is:
• Kidney
• Stomach
• Uterus
• Brain

129. The deposits of amyloid sees in all tissues, except:


• Mesangium and capillary loops
• Basement membranes of blood vessels
• Tubular basement membranes
• Epithelium in proximal renal tubules

130. All of the following pathological processes are reversible, except:


• Mucoid changes
• Cellular swelling
• Amyloidosis
• Edema

131. The organ most commonly and seriously damaged in amyloidosis is:
• Stomach
• Kidney
• Lung
• Liver

132. Reactive systemic amyloidosis knows to be all of the following, except:


• Secondary amyloidosis
• Complication of tuberculosis
• Hereditary amyloidosis
• Complication of osteomyelitis
133. Amyloid in primary amyloidosis is usually systemic and belongs to the following biochemical type:
• AL type
• AA type
• Aβ type
• ATTR type

134. Reactive systemic amyloidosis occurs in association with all of the following diseases, except:
• Tuberculosis
• Bronchiectasis
• Chronic osteomyelitis
• Hepatitis B

135. Reactive systemic amyloidosis occurs in association with all of the following diseases, except
• Rheumatoid arthritis
• Ankylosing spondylitis
• Chronic appendicitis
• Myeloma

136. Secondary amyloidosis damages all of the following organs, except:


• Kidneys
• Liver
• Spleen
• Brain

137. Macroscopically the organs affected by amyloidosis are characterized by all of the following, except:
• Enlarged
• Firm
• Waxy in appearance
• Soft

138. The common cause of death in patients with secondary amyloidosis is insufficiency of:
• Kidneys
• Heart
• Liver
• Lung

139. The organs that should be histological examined in patients with amyloidosis are all of the following,
except:
• Kidney
• Eye
• Rectum
• Gingiva

140. The reversible process caused by accumulation of glycosaminoglycans in extracellular matrix due to
the increase of vascular permeability is:
• Amyloidosis
• Glycogenoses
• Hyalinosis
• Mucoid changes
141. The irreversible process caused by accumulation of proteins with high molecular weight associated
with the destruction of connective tissue is:
• Amyloidosis
• Glycogenoses
• Hyalinosis
• Fibrinoid changes

142. All the pathological processes are irreversible, except


• Mucoid changes
• Fibrinoid changes
• Amyloidosis
• Apoptosis

143. In amyloidosis macroscopically kidney changed in the following way:


• Cut surface is pale and translucent
• Dense and sharply reduced
• Enlarged and dense
• True a + c

144. All examples of extracellular hyaline are change, except:


• Old scar
• Hyaline arteriolosclerosis
• Chronic glomerulonephritis
• Mallory’s hyaline

145. In amyloidosis macroscopically liver changed in the following way:


• Enlarged, pale, waxy and firm
• Dense and sharply reduced
• Enlarged and yellow color
• Soft and enlarged

146. In amyloidosis macroscopically spleen changed in the following way:


• Cut surface is translucent pale and waxy
• Dense and sharply reduced
• Cut surface – map-like areas of amyloid
• True a + c

147. Causes of hyalinosis are all of the following, EXCEPT:


• Fibrinoid changes
• Inflammation
• Apoptosis
• Sclerosis

148. Most specific histological sign in mucoid changes is:


• Metachromasia.
• Metaplasia.
• Metastasis.
• Metakinesis.

149. Most common localization of mucoid changes is


• Nervous system.
• Cardiovascular system
• Urinary system
• Blood system

150. Simple hyaline occurs in:


• Diabetes mellitus.
• Arterial hypertension
• Rheumatic fever
• Rheumatoid arthritis

151. Simple hyaline occurs in


• Diabetes mellitus
• systemic lupus erythematosus
• Atherosclerosis.
• Rheumatic fever.

152. Lipohyalin occurs in:


• Diabetes mellitus.
• Arterial hypertension
• Atherosclerosis
• Rheumatic fever

153. Compound hyalin occurs in:


• Diabetes mellitus.
• Arterial hypertension
• Atherosclerosis.
• Rheumatic fever.
154. Compound hyalin occurs in
• Diabetes mellitus
• Arterial hypertension
• Atherosclerosis
• Rheumatoid arthritis

155. The stain used to identify amyloid is


• Hematoxylin and eosin stain.
• Metachromatic stain
• Congo-red stain
• Sudan III stain.

156. The pathologic proteinaceous substance, accumulating only between cells in various tissues and
organs of the body is
• Glycogen.
• Hyaline.
• Water
• Amyloid.

157. Specific method for diagnosis of amyloid in fresh tissue is:


• Virchow test.
• Rokitansky test.
• Ewing test
• Masson test

158. Variant of systemic amyloidosis is:


• Senile cardiac.
• Secondary
• Senile cerebral.
• Endocrine.

159. Variant of systemic amyloidosis is


• Senile cardiac.
• Senile cerebral
• Heredofamilial
• Endocrine

160. Variant of systemic amyloidosis is


• Senile cardiac
• Senile cerebral
• Endocrine.
• Hemodialysis-associated
161. Variant of localized amyloidosis is:
• Senile cardiac.
• Primary
• Secondary
• Heredofamilial

162. Variant of localized amyloidosis is


• Primary
• Secondary.
• Senile cerebral
• Heredofamilial

163. Variant of localized amyloidosis is


• Primary
• Secondary.
• Heredofamilial.
• Endocrine

164. Variant of localized amyloidosis is?


• Primary.
• Secondary
• Heredofamilial
• Tumor-forming

165. Localization of pericollagenous amyloidosis is:


• Liver
• Spleen
• Heart.
• Kidneys

166. Localization of pericollagenous amyloidosis is


• Liver
• Spleen
• Bowel.
• Kidneys

167. Localization of pericollagenous amyloidosis is:


• Liver
• Spleen
• Nerves
• Kidneys

168. Localization of perireticulin amyloidosis is:


• Heart
• Tongue.
• Nerves
• Kidneys

169. Localization of perireticulin amyloidosis is:


• Heart
• Tongue
• Nerves
• Liver

170. Localization of perireticulin amyloidosis is :


• Heart
• Tongue
• Nerves
• Spleen

171. Cause of obesity which associated with excessive nutrition is called:


• Primary.
• Alimentary
• Cerebral
• Endocrine

172. Type of obesity with unknown cause is called:


• Primary
• Alimentary
• Cerebral
• Endocrine

173. 1st degree of obesity is associated with increasing of body weight over than normal by:
• 20-29%.
• 10-20%.
• 15-25%.
• 20-35%
174. 2nd degree of obesity is associated with increasing of body weight over than normal by:
• 30-40%
• 30-49%
• 25-45%.
• 30-55%.

175. 3rd degree of obesity is associated with increasing of body weight over than normal by:
• 50-99%.
• 45-85%
• 50-75%
• 60-90%.

176. 4th degree of obesity is associated with increasing of body weight over than normal by
• 90% and more
• 85% and more.
• 120% and more.
• 100% and more

177. Deposition of fat in abdomen area in obesity is called:


• Upper type.
• Middle type.
• Lower type.
• Simmetric type

178. Deposition of fat in area of face and neck in obesity is called:


• Upper type.
• Middle type
• Lower type
• Simmetric type

179. A 38-year-old female presents with intermittent pelvic pain. Physical examination reveals a 3-cm
mass in the area of her right ovary. Histological sections from this ovarian mass reveal a papillary tumor
with multiple, scattered small, round, and laminated calcifications. These structures are most likely the
result of
• Apoptosis
• Dystrophic calcification
• Enzymatic necrosis
• Hyperparathyroidism
180. The following pigments are stainable by Prussian blue reaction except:
• Hemosiderin
• Ferritin
• Hematin
• Hemochromatosis

181. Idiopathic calcinosis cutis is an example of:


• Necrotizing inflammation
• Dystrophic calcification
• Metastatic calcification
• Calcified thrombi in veins

182. Risk factors implicated in the etiology of cholesterol gallstones include the following except:
• Family history
• Obesity
• Hemolytic anemia
• Oral contraceptives

183. The following type of gallstones is generally unassociated with changes in the gallbladder wall:
• Cholesterol
• Mixed
• Combined
• Pigment

184. The following type of renal calculi is radiolucent:


• Calcium oxalate
• Struvite
• Uric acid
• Calcium phosphate

185. The following type of renal calculi is infection-induced:


• Calcium oxalate
• Struvite
• Uric acid
• Cystine

186. The color of organs hemosiderin is:


• Black
• Sky-blue
• Yellow
• Brown
187. The pigment hematoidin is most identical with:
• Porphyrin
• Bilirubin
• Hemosiderin
• Hemozoin

188. Lipofuscin, the golden yellow pigment sees in heart muscle


• Hypertrophy
• Atrophy
• Hyperplasia
• Metaplasia

189. What stain is specific for iron:


• Hematoxylin and eosin
• Sudan III
• Prussian blue reaction
• PAS-reaction

190. Hemosiderin has all of the following features, except:


• Hemoglobin-derived
• Golden yellow-to-brown
• Granular or crystalline
• Synthesized by enzyme tyrosinase

191. Hemosiderin in the lung accumulates in:


• Leukocytes
• Lymphocytes
• Macrophages
• Fibroblasts

192. Idiopathic pulmonary hemosiderosis characterizes by all of the following pathologic symptoms,
except:
• Productive cough
• Hemophtysis
• Anemia
• Heavy proteinuria

193. The lungs in pulmonary hemosiderosis are:


• Enlarged with turbid fluid exuding from cut surface
• Increased in weight with areas of red-brown consolidation
• Diminished with areas of red-brown consolidation
• Diminished with turbid fluid exuding from cut surface

194. The color of hemosiderin granules stained with Prussian blue reaction is:
• Yellow
• Brown
• Orange-red
• Blue-black

195. Hemosiderosis sees in all pathologic processes, except:


• Inflammation
• Hereditary increased absorption of dietary iron
• Impaired use of iron
• Hemolytic anemia

196. Morphologic changes in genetic hemochromatosis characterizes by all of the following, except:
• Metastatic calcification of many organs
• Deposition of hemosiderin in many organs
• Deposition of hemosiderin in the skin
• Liver cirrhosis

197. Melanin has all of the following features, except:


• Localized endogenous
• Formed of ferritin
• Non-hemoglobin-derived
• Black-brown

198. An increased amount of melanin in melanocytes and within basal keratinocytes is also known as:
• Vacuolization
• Vitiligo
• Hyperpigmentation (melanosis)
• Albinism

199. Lipofuscin has all of the following features, except:


• Aging pigment
• Most often seen in kidney
• Yellow-brown
• Noninjurious to the cells or their function
200. Lipofuscin pigment granules in cells result from:
• Hemosiderosis
• Accumulation of protein in cytoplasm
• Accumulation of lipids in cytoplasm
• Intracellular lipid peroxidation

201. Lipofuscin granules in cells see in:


• Necrosis
• Denervation atrophy
• Brown atrophy
• Atrophy from pressure

202. Heart and liver of a patient with cancer cachexia macroscopically sees as:
• Diminished and brown
• Diminished and brown
• Enlarged and brown
• Enlarged and yellow

203. Lipofuscin in the liver may founds in:


• Unchanged cells
• Cells with ballooning degeneration
• Cells with hyaline droplets
• Cells with regressive changes

204. Bilirubin has all of the following features, except:


• The end product of hem degradation
• Derived from breakdown erythrocytes
• Brilliant-yellow
• Stained in blue-black color with Prussian-blue reaction

205. Jaundice occurs in all of the following pathologic processes, except:


• Increased hepatocellular excretion
• Excessive production of bilirubin
• Reduced hepatocyte uptake
• Impaired conjugation of bilirubin

206. Biliary ducts obstruction by gallstones may lead to:


• Liver hemosiderosis
• Liver steatosis
• Ballooning degeneration of hepatocytes
• Cholestasis and jaundice
207. Complications or well-established associations of gallstones include all of the following, except:
• Biliary obstruction
• Brown atrophy of the liver
• Pancreatitis
• Intestinal obstruction

208. Which of the following sites is an example of metastatic calcification?


• The kidney in nephrocalcinosis
• The mitral valve in mitral stenosis of rheumatic organ
• The left anterior ascending coronary artery affected by atheromatous plaques
• The lung involved by metastatic carcinoma

209. Point out the posttuberculosis lung lesion:


• Granuloma
• Cavity
• Fibrocalcific scar
• Caseation in lymph node

210. Dystrophic calcification encounters in all of the following areas, except:


• Coagulative necrosis
• Intracellular fat accumulation
• Caseous necrosis
• Liquefactive necrosis

211. Initiation of intracellular calcification occurs in:


• Cytoplasm
• Lysosome
• Nucleus
• Mitochondria

212. The causes of metastatic calcification are all of the following, except:
• Diabetes mellitus
• Increased secretion of parathyroid hormone
• Destruction of bone tissue
• Vitamin D-related disorders

213. Metastatic calcification may occur in all of the following organs, except:
• Stomach (gastric mucosa)
• Kidneys
• Lungs
• Liver

214. Deficiency of vitamin D tends to cause:


• Hypercalcemia
• Hypocalcemia
• Hyperpigmentation
• Hypopigmentation

215. Deficiency of vitamin D in the adults leads to:


• Osteosclerosis
• Osteomyelitis
• Osteomalati
• Osteonecrosis

216. Hemoglobin derivative pigment which occurs in normal conditions:


• Hemosiderin
• Hematoidin
• Hemin
• Hemomelanin

217. Hemoglobin derivative pigment which occurs in normal conditions


• Ferritin
• Hematoidin.
• Hemin
• Hemomelanin.

218. Hemoglobin derivative pigment which occurs in normal conditions :


• Bilirubin
• Hematoidin
• Hemin.
• Hemomelanin

219. Hemoglobin derivative pigment which occurs only in pathologic conditions:


• Hemosiderin
• Hematoidin
• Ferritin
• Bilirubin
220. Hemoglobin derivative pigment which occurs only in pathologic conditions:
• Hemosiderin
• Porphyrin
• Ferritin.
• Bilirubin.

221. Hemoglobin derivative pigment which occurs only in pathologic conditions


• Hemosiderin
• Hematin
• Ferritin
• Bilirubin

222. Iron-containing pigment is:


• Bilirubin
• Hematoidin
• Hemosiderin
• Porphyrin

223. Iron-containing pigment is


• Bilirubin
• Hematoidin
• Ferritin
• Porphyrin

224. Iron-containing pigment is:


• Bilirubin.
• Hematoidin.
• Hematin
• Porphyrin

225. The stain used to identify iron-containing pigments is:


• Prussian-blue reaction.
• PAS reaction
• Congo-red stain
• Sudan III stain

226. In hemosiderosis organs become:


• Enlarged, brown, hard.
• Small, red, soft
• Enlarged, yellow, hard.
• Enlarged, brown, soft.
227. Cause of generalized hemosiderosis is:
• Intracerebral hemorrhage.
• Intoxications.
• Brown induration of lungs
• Pulmonary hemorrhagic infarct

228. Cause of generalized hemosiderosis is


• Intracerebral hemorrhage
• Heterohemotransfusions
• Brown induration of lungs
• Pulmonary hemorrhagic infarct

229. Cause of generalized hemosiderosis is ?


• Typhoid fever
• Yellow fever
• Malaria
• Diphteria

230. Cause of localized hemosiderosis is:


• Intoxications
• Heterohemotransfusions.
• Malaria
• Brown induration of lungs

231. Cause of localized hemosiderosis is :


• Intoxications
• Heterohemotransfusions
• Malaria
• Intracerebral hemorrhage

232. Cause of localized hemosiderosis is:


• Intoxications
• Heterohemotransfusions
• Malaria
• Pulmonary hemorrhagic infarct.

233. Accumulation of which pigment may leads to decrease of blood pressure?


• Ferritin
• Hemosiderin
• Bilirubin
• Hemomelanin

234. Accumulation of which pigment may leads to jaundice?


• Ferritin.
• Hemosiderin
• Bilirubin.
• Hemomelanin

235. Cause of prehepatic jaundice is:


• Hepatitis.
• Liver cirrhosis
• Obstruction of bile ducts by stones
• Hemolytic disease of newborns

236. Cause of prehepatic jaundice is :


• Hepatitis
• Liver cirrhosis
• Obstruction of bile ducts by stones.
• Heterohemotransfusions

237. Cause of hepatocellular jaundice is:


• Obstruction of bile ducts by stones
• Compression of bile ducts by tumors
• Heterohemotransfusions
• Hepatitis

238. Cause of hepatocellular jaundice is :


• Obstruction of bile ducts by stones
• Compression of bile ducts by tumors
• Heterohemotransfusions.
• Liver cirrhosis

239. Cause of posthepatic jaundice is:


• Obstruction of bile ducts by stones
• Hepatitis
• Heterohemotransfusions.
• Liver cirrhosis
240. Cause of posthepatic jaundice is :
• Compression of bile ducts by tumors
• Hepatitis.
• Heterohemotransfusions.
• Liver cirrhosis

241. Which pigment is “tear and wear”:


• Bilirubin
• Hematoidin
• Hematin.
• Lipofuscin

242. Which pigment is tyrosin-derived:


• Bilirubin
• Hematoidin
• Hematin.
• Melanin

243. Generalized hyperpigmentation of melanin occurs in:


• Addison’s disease
• Melanosis coli
• Lentigo
• Nevus.

244. Focal hyperpigmentation of melanin occurs in:


• Addison’s disease
• Cachexia.
• Avitaminosis
• Nevus.

245. General hypopigmentation of melanin occurs in


• Leukoderma.
• Albinism
• Vitiligo
• Cachexia

246. Focal hypopigmentation of melanin occurs in:


• Albinism
• Nevus
• Vitiligo
• Cachexia
247. Distrophic calcification occurs in:
• Hyperparathyroidism
• Hypervitaminosis D
• Hyperthyroidism
• Necrosis.

248. Metastatic calcification occurs in:


• Hyperparathyroidism.
• Damaged heart valves
• Atheromas.
• Necrosis.

249. Gross appearance of calcificate is:


• Hard, greywish-white.
• Soft, yellow
• Hard, greywish-red
• Soft, greywish-white.

250. Area of calcification in histological examination with hematoxylin and eosin staining is:
• Red
• Black.
• Brown
• Blue.

251. Сongenital absence of thymus is called:


• Aplasia
• Hypoplasia
• Dysplasia
• Atrophy

252. Congenital incomplete development of thymus is called:


• Aplasia
• Hypoplasia
• Dysplasia
• Atrophy
253. Aquired decrease in weight of thymus is called:
• Aplasia
• Hypoplasia
• Dysplasia
• Atrophy

254. Abnormal development of thymus is called:


• Aplasia
• Hypoplasia
• Dysplasia
• Atrophy

255. Morphologically immediate type of hypersensitivity occurs as:


• Fibrinoid necrosis
• Lympho-hystiocytic infiltration
• Macrophageal infiltration
• Granulomatosis

256. Morphologically immediate type of hypersensitivity occurs as


• Lympho-hystiocytic infiltration
• Macrophageal infiltration
• Granulomatosis
• Plasmatic saturation

257. Morphologically immediate type of hypersensitivity occurs as


• Lympho-hystiocytic infiltration
• Mucoid changes
• Macrophageal infiltration
• Granulomatosis

258. Morphologically immediate type of hypersensitivity occurs as:


• Lympho-hystiocytic infiltration
• Fibrinoid changes
• Macrophageal infiltration
• Granulomatosis

259. Morphologically immediate type of hypersensitivity occurs as:


• Lympho-hystiocytic infiltration
• Fibrinous-hemorrhagic exudate
• Macrophageal infiltration
• Granulomatosis
260. Morphological signs of immediate type of hypersensitivity are all the following, except:
• Lympho-hystiocytic infiltration
• Mucoid and Fibrinoid changes
• Plasmatic saturation
• Fibrinoid necrosis

261. Morphological signs of immediate type of hypersensitivity are all the following, except:
• Mucoid and Fibrinoid changes
• Plasmatic saturation
• Granulomatosis
• Fibrinoid necrosis

262. Morphological signs of immediate type of hypersensitivity are all the following, except:
• Mucoid and Fibrinoid changes
• Plasmatic saturation
• Fibrinoid necrosis
• Macrophage infiltration

263. Morphologically delayed type of hypersensitivity occurs as:


• Lympho-hystiocytic infiltration
• Mucoid and Fibrinoid changes
• Plasmatic saturation
• Fibrinoid necrosis

264. Morphologically delayed type of hypersensitivity occurs as:


• Mucoid and Fibrinoid changes
• Plasmatic saturation
• Granulomatosis
• Fibrinoid necrosis

265. Morphologically delayed type of hypersensitivity occurs as:


• Mucoid and Fibrinoid changes
• Plasmatic saturation
• Fibrinoid necrosis
• Macrophageal infiltration

266. Morphologically delayed type of hypersensitivity occurs as:


• Mucoid and Fibrinoid changes
• Plasmatic saturation
• Fibrinoid necrosis
• Cytoplasmic bridges between lymphocytes and macrophages

267. Morphological signs of delayed type of hypersensitivity are all the following, except:
• Fibrinoid necrosis
• Lympho-hystiocytic infiltration
• Macrophageal infiltration
• Granulomatosis

268. Morphological signs of delayed type of hypersensitivity are all the following, except:
• Lympho-hystiocytic infiltration
• Macrophageal infiltration
• Granulomatosis
• Plasmatic saturation

269. Morphological signs of delayed type of hypersensitivity are all the following, except:
• Lympho-hystiocytic infiltration
• Mucoid changes
• Macrophageal infiltration
• 4.Granulomatosis

270. Morphological signs of delayed type of hypersensitivity are all the following, except:
• Lympho-hystiocytic infiltration
• Fibrinous-hemorrhagic exudate
• Macrophageal infiltration
• Granulomatosis

271. Morphological signs of transplant rejection are all the following, except:
• Lympho-hystiocytic infiltration
• Edema of transplant
• Macrophageal infiltration
• 4.Granulomatosis

272. Morphological signs of transplant rejection are all the following, except ?
• Lympho-hystiocytic infiltration
• Edema of transplant
• Macrophageal infiltration
• Cytoplasmic bridges between lymphocytes and macrophages
273. Organ specific immune disease is:
• Hashimoto thyroiditis
• Rheumatoid arthritis
• Systemic Lupus Erythematosus
• Scleroderma

274. Organ specific immune disease is :


• Rheumatoid arthritis
• Systemic Lupus Erythematosus
• Scleroderma
• Encephalomyelitis

275. Non-organ specific immune disease is:


• Hashimoto thyroiditis
• Rheumatoid arthritis
• Encephalomyelitis
• Polyneuritis

276. Non-organ specific immune disease is :


• Hashimoto thyroiditis
• Encephalomyelitis
• Systemic Lupus Erythematosus.
• Polyneuritis.

277. Variant of primary immunodeficiency syndrome occurs:


• In leukemia
• Under radial therapy
• In aplasia of thymus
• In infections

278. Variant of primary immunodeficiency syndrome occurs :


• In leukemia
• Under radial therapy.
• In sarcoidosis
• In hypoplasia of thymus.
279. Variant of secondary immunodeficiency syndrome is:
• Lui-Bar syndrome
• Neseloff syndrome
• DiGeorge syndrome
• AIDS
280. Complication of immunodeficiency syndromes is:
• Arterial hypertension
• Myocardial infarction
• Typhoid fever
• Sepsis

281. Complication of immunodeficiency syndromes is :


• Recurrence of bronchial asthma
• Recurrence of chronic hepatitis
• Recurrence of chronic cholecystitis
• Recurrence of tuberculosis

282. Complication of immunodeficiency syndromes is:


• Arterial hypertension
• Myocardial infarction
• Typhoid fever
• Purulent pneumonia

283. A 22-year-old woman nursing her newborn develops a tender erythematous area around the nipple
of her left breast. A thick, yellow fluid is observed to drain from an open fissure. Examination of this breast
fluid under the light microscope will most likely reveal an abundance of which of the following inflammatory
cells?
• B lymphocytes
• Eosinophils
• Mast cells
• Neutrophils

284. A 63-year-old man becomes febrile and begins expectorating large amounts of mucopurulent
sputum. Sputum cultures are positive for Gram-positive diplococci. Which of the following mediators of
inflammation provides potent chemotactic factors for the directed migration of inflammatory cells into the
alveolar air spaces of this patient?
• Bradykinin
• Histamine
• Myeloperoxidase
• N-formylated peptides

285. A 59-year-old man suffers a massive heart attack and expires 24 hours later due to ventricular
arrhythmia. Histologic examination of the affected heart muscle at autopsy would show an abundance of
which of the following inflammatory cells?
• Fibroblasts
• Lymphocytes
• Macrophages
• Neutrophils

286. A 5-year-old boy punctures his thumb with a rusty nail. Four hours later, the thumb appears red and
swollen. Initial swelling of the boy’s thumb is primarily due to which of the following mechanisms?
• Decreased intravascular hydrostatic pressure
• Decreased intravascular oncotic pressure
• Increased capillary permeability
• Increased intravascular oncotic pressure

287. An 80-year-old woman presents with a 4-hour history of fever, shaking chills, and disorientation. Her
blood pressure is 80/40 mm Hg. Physical examination shows diffuse purpura on her upper arms and chest.
Blood cultures are positive for Gram negative organisms. Which of the following cytokines is primarily
involved in the pathogenesis of direct vascular injury in this patient with septic shock?
• Interferon-g
• Interleukin-1
• Platelet-derived growth factor
• Tumor necrosis factor-a

288. A 24-year-old intravenous drug abuser develops a 2-day history of severe headache and fever. His
temperature is 38.7°C (103°F). Blood cultures are positive for Gram-positive cocci The patient is given
intravenous antibiotics, but he deteriorates rapidly and dies. A cross section of the brain at autopsy (shown
in the image) reveals two encapsulated cavities. Which of the following terms best characterizes this
pathologic finding?
• Chronic inflammation
• Fibrinoid necrosis
• Granulomatous inflammation
• Suppurative inflammation

289. A 36-year-old woman with pneumococcal pneumonia develops a right pleural effusion. The pleural
fluid displays a high specific gravity and contains large numbers of polymorphonuclear (PMN) leukocytes.
Which of the following best characterizes this pleural effusion?
• Fibrinous exudate
• Lymphedema
• Purulent exudate
• Serosanguineous exudate

290. A 33-year-old man presents with a 5-week history of calf pain and swelling and low-grade fever.
Serum levels of creatine kinase are elevated. A muscle biopsy reveals numerous eosinophils. What is the
most likely etiology of this patient’s myalgia?
• Autoimmune disease
• Bacterial infection
• Muscular dystrophy
• Parasitic infection

291. A 10-year-old boy with a history of recurrent bacterial infections presents with fever and a
productive cough. Biochemical analysis of his neutrophils demonstrates that he has an impaired ability to
generate reactive oxygen species. This patient most likely has inherited mutations in the gene that encodes
which of the following proteins?
• Catalase
• Cytochrome P450
• Myeloperoxidase
• NADPH oxidase

292. A 25-year-old woman presents with a history of recurrent shortness of breath and severe wheezing.
Laboratory studies demonstrate that she has a deficiency of C1 inhibitor, an esterase inhibitor that
regulates the activation of the classical complement pathway. What is the diagnosis?
• Chronic granulomatous disease
• Hereditary angioedema
• Myeloperoxidase deficiency
• Selective IgA deficiency

293. A 40-year-old man complains of a 2-week history of increasing abdominal pain and yellow
discoloration of his sclera. Physical examination reveals right upper quadrant pain. Laboratory studies show
elevated serum levels of alkaline phosphatase (520 U/dL) and bilirubin (3.0 mg/dL). A liver biopsy shows
portal fibrosis, with scattered foreign bodies consistent with schistosome eggs. Which of the following
inflammatory cells is most likely to predominate in the portal tracts in the liver of this patient?
• Basophils
• Eosinophils
• Macrophages
• Monocytes

294. A 41-year-old woman complains of excessive menstrual bleeding and pelvic pain of 4 months. She
uses an intrauterine device for contraception. Endometrial biopsy (shown in the image) reveals an excess of
plasma cells (arrows) and macrophages within the stroma. The presence of these cells and scattered
lymphoid follicles within the endometrial stroma is evidence of which of the following conditions?
• Acute inflammation
• Chronic inflammation
• Granulation tissue
• Granulomatous inflammation

295. A 62-year-old woman undergoing chemotherapy for breast cancer presents with a 3-day history of
fever and chest pain. Cardiac catheterization reveals a markedly reduced ejection fraction with normal
coronary blood flow. A myocardial biopsy is obtained, and a PCR test for coxsackievirus is positive.
Histologic examination of this patient’s myocardium will most likely reveal an abundance of which of the
following inflammatory cells?
• Eosinophils
• Lymphocytes
• Macrophages
• Mast cells

296. A 58-year-old woman with long-standing diabetes and hypertension develops end-stage renal
disease and dies in uremia. A shaggy fi brin-rich exudate is noted on the visceral pericardium at autopsy
(shown in the image). Which of the following best explains the pathogenesis of this fibrinous exudate?
• Antibody binding and complement activation
• Chronic passive congestion
• Injury and increased vascular permeability
• Margination of segmented neutrophils

297. A 68-year-old man presents with fever, shaking chills, and shortness of breath. Physical examination
shows rales and decreased breath sounds over both lung fields. The patient exhibits grunting respirations,
30 to 35 breaths per minute, with flaring of the nares. The sputum is rusty yellow and displays numerous
polymorphonuclear leukocytes. Which of the following mediators of inflammation is chiefly responsible for
the development of fever in this patient?
• Arachidonic acid
• Interleukin-1
• Leukotriene B4
• Prostacyclin (PGI2)

298. A 35-year-old woman presents with a 5-day history of a painful sore on her back. Physical
examination reveals a 1- cm abscess over her left shoulder. Biopsy of the lesion shows vasodilation and
leukocyte margination (shown in the image). What glycoprotein mediates initial tethering of segmented
neutrophils to endothelial cells in this skin lesion?
• Cadherin
• Entactin
• Integrin
• Selectin

299. A 14-year-old boy receives a laceration on his forehead during an ice hockey game. When he is first
attended to by the medic, there is blanching of the skin around the wound. Which of the following
mechanisms accounts for this transient reaction to neurogenic and chemical stimuli at the site of injury?
• Constriction of postcapillary venules
• Constriction of precapillary arterioles
• Dilation of postcapillary venules
• Dilation of precapillary arterioles
300. An 8-year-old girl with asthma presents with respiratory distress. She has a history of allergies and
upper respiratory tract infections. She also has history of wheezes associated with exercise. Which of the
following mediators of inflammation is the most powerful stimulator of bronchoconstriction and
vasoconstriction in this patient?
• Bradykinin
• Complement proteins
• Interleukin-1
• Leukotrienes

301. A 75-year-old woman complains of recent onset of chest pain, fever, and productive cough with
rust-colored sputum. A chest X-ray reveals an infiltrate in the right middle lobe. Sputum cultures are
positive for Streptococcus pneumoniae. Phagocytic cells in this patient’s affected lung tissue generate
bacteriocidal hypochlorous acid using

which of the following enzymes?

• Catalase
• Cyclooxygenase
• Myeloperoxidase
• NADPH oxidase

302. A 28-year-old woman cuts her hand while dicing vegetables in the kitchen. The wound is cleaned and
sutured. Five days later, the site of injury contains an abundance of chronic inflammatory cells that actively
secrete interleukin-1, tumor necrosis factor-, interferon-, numerous arachidonic acid derivatives, and
various enzymes. Name these cells
• B lymphocytes
• Macrophages
• Plasma cells
• Smooth muscle cells

303. A 68-year-old man with prostate cancer and bone metastases presents with shaking chills and fever.
The peripheral WBC count is 1,000/L (normal = 4,000 to 11,000/L). Which of the following terms best
describes this hematologic finding?

• Leukocytosis
• Leukopenia
• Neutrophilia
• Pancytopenia

304. A 25-year-old machinist is injured by a metal sliver in his left hand. Over the next few days, the
wounded area becomes reddened, tender, swollen, and feels warm to the touch. Redness at the site of
injury in this patient is caused primarily by which of the following mechanisms?
• Hemorrhage
• Hemostasis
• Neutrophil margination
• Vasodilation

305. A 37-year-old man with AIDS is admitted to the hospital with a 3-week history of chest pain and
shortness of breath. An X-ray film of the chest shows bilateral nodularities of the lungs. A CT-guided lung
biopsy is shown in the image. The multinucleated cell in the center of this field is most likely derived from
which of the following inflammatory cells?
• Basophils
• Capillary endothelial cells
• Macrophages
• Myofi broblasts

306. A 10-year-old girl presents with a 2-week history of puffiness around her eyes and swelling of the
legs and ankles. Laboratory studies show hypoalbuminemia and proteinuria. The urinary sediment contains
no inflammatory cells or red blood cells. Which of the following terms describes this patient’s peripheral
edema?
• Effusion
• Exudate
• Hydropic change
• Transudate

307. A 25-year-old woman develops a sore, red, hot, swollen left knee. She has no history of trauma and
no familial history of joint disease. Fluid aspirated from the joint space shows an abundance of segmented
neutrophils. Transendothelial migration of acute inflammatory cells into this patient’s joint space was
mediated primarily by which of the following families of proteins?
• Entactins
• Fibrillins
• Fibronectins
• Integrins

308. A 50-year-old woman is discovered to have metastatic breast cancer. One week after receiving her
first dose of chemotherapy, she develops bacterial pneumonia. Which of the following best explains this
patient’s susceptibility to bacterial infection?
• Depletion of serum complement
• Impaired neutrophil respiratory burst
• Inhibition of clotting factor activation
• Neutropenia

309. A 53-year-old man develops weakness, malaise, cough with bloody sputum, and night sweats. A
chest X-ray reveals numerous apical densities bilaterally. Exposure to Mycobacterium tuberculosis was
documented 20 years ago, and M. tuberculosis I identified in the sputum. The patient subsequently dies of
respiratory insufficiency. The lungs are examined at autopsy (shown in the image). Which of the following
best characterizes the histopathologic features of this pulmonary lesion?
• Acute suppurative inflammation
• Chronic inflammation
• Fat necrosis
• Granulomatous inflammation

310. A 59-year-old man experiences acute chest pain and is rushed to the emergency room. Laboratory
studies and ECG demonstrate an acute myocardial infarction; however, coronary artery angiography
performed 2 hours later does not show evidence of thrombosis. Intravascular thrombolysis that occurred in
this patient was mediated by plasminogen activators that were released by which of the following cells?
• Cardiac myocytes
• Endothelial cells
• Macrophages
• Segmented neutrophils

311. A 68-year-old coal miner with a history of smoking and emphysema develops severe air-flow
obstruction and expires. Autopsy reveals a “black lung,” with coal-dust nodules scattered throughout the
parenchyma and a centra area of dense fibrosis. The coal dust entrapped within this miner’s lung was
sequestered primarily by which of the following cells?
• Endothelial cells
• Fibroblasts
• Lymphocytes
• Macrophages

312. A 40-year-old man presents with 5 days of productive cough and fever. Pseudomonas aeruginosa is
isolated from a pulmonary abscess. The CBC shows an acute effect characterized by marked leukocytosis
(50,000 WBC/L), and the differential count reveals numerous immature cells (band forms). Which of the
following terms best describes these hematologic findings?
• Leukemoid reaction
• Leukopenia
• Myeloid metaplasia
• Myeloproliferative disease

313. A 19-year-old woman presents with 5 days of fever (38°C/101°F) and sore throat. She reports that
she has felt fatigued for the past week and has difficulty swallowing. A physical examination reveals
generalized lymphadenopathy. If this patient has a viral infection, a CBC will most likely show which of the
following hematologic findings?
• Eosinophilia
• Leukopenia
• Lymphocytosis
• Neutrophilia
314. A 40-year-old woman presents with an 8-month history of progressive generalized itching, weight
loss, fatigue, and yellow sclerae. Physical examination reveals mild jaundice. The antimitochondrial antibody
test is positive. A liver biopsy discloses periductal inflammation and bile duct injury (shown in the image).
Which of the following inflammatory cells is the principal mediator of destructive cholangitis in this patient?
• Eosinophils
• B lymphocytes
• T lymphocytes
• Mast cells

315. A 25-year-old woman presents with a 2-week history of febrile illness and chest pain. She has an
erythematous, macular facial rash and tender joints, particularly in her left wrist and elbow. A CBC shows
mild anemia and thrombocytopenia. Corticosteroids are prescribed for the patient. This medication induces
the synthesis of an inhibitor of which of the following enzymes in inflammatory cells?
• Lipoxygenase
• Myeloperoxidase
• Phospholipase A2
• Phospholipase C

316. A 22-year-old man develops marked right lower quadrant abdominal pain over the past day. On
physical examination there is rebound tenderness on palpation over the right lower quadrant. Laparoscopic
surgery is performed, and the appendix is swollen, erythematous, and partly covered by a yellowish
exudate. It is removed, anda microscopic section shows infiltration with numerous neutrophils. The pain
experienced by this patient is predominantly the result of which of the following two chemical mediators?
• Complement C3b and IgG
• Interleukin-1 and tumor necrosis factor
• Histamine and serotonin
• Prostaglandin and bradykinin
317. A 40-year-old woman had laparoscopic surgery 3 months ago. Now she has a small 0.5 cm nodule
beneath the skin at the incision site that was sutured. Which of the following cell types is most likely to be
most characteristic of the inflammatory response in this situation?
• Mast cell
• Eosinophil
• Giant cell
• Neutrophil

318. A 39-year-old man incurs a burn injury to his hands and arms while working on a propane furnace.
Over the next 3 weeks, the burned skin heals without the need for skin grafting. Which of the following is
the most critical factor in determining whether the skin in the region of the burn will regenerate?
• Good cardiac output with tissue perfusion
• Persistence of skin appendages
• Maintenance of underlying connective tissue
• Diminished edema and erythema
319. A 58-year-old woman has had a cough with fever for 3 days. A chest radiograph reveals infiltrates in
the right lower lobe. A sputum culture grows Streptococcus pneumoniae. The clearance of these organisms
from the lung parenchyma would be most effectively accomplished through generation of which of the
following substances by the major inflammatory cell type responding to this infection?
• Platelet activating factor
• Prostaglandin E2
• Kallikrein
• Hydrogen peroxide

320. A clinical study is performed of patients with pharyngeal infections. The most typical clinical course
averages 3 days from the time of onset until the patient sees the physician. Most of these patients
experience fever and chills. On physical examination, the most common findings include swelling, erythema,
and pharyngeal purulent exudate. Which of the following types of inflammation did these patients most
likely have?
• Granulomatous
• Acute
• Gangrenous
• Resolving

321. A 56-year-old man has had increasing dyspnea for 6 years. He has no cough or fever. He had chronic
exposure to inhalation of silica dust for many years in his job. A chest x-ray now shows increased interstitial
markings an parenchymal 1 to 3 cm solid nodules. His pulmonary problems are most likely to be mediated
through which of the following inflammatory processes?
• Neutrophilic infiltrates producing leukotrienes
• Foreign body giant cell formation
• Plasma cell synthesis of immunoglobulins
• Macrophage elaboration of cytokines

322. A 22-year-old woman has premature labor with premature rupture of fetal membranes at 20 weeks
gestation. Prior to that time, the pregnancy had been proceeding normally. A stillbirth occurs two days
later. Microscopic examination of the normal-sized placenta reveals numerous neutrophils in the amnion
and chorion, but no villitis. The premature labor was most likely mediated by the effects from release of
which of the following substances?
• Immunoglobulin
• Prostaglandin
• Complement
• Fibrinogen

323. After two weeks in the hospital following a fall in which she incurred a fracture of her left femoral
trochanter, a 76- year-old woman now has a left leg that is swollen, particularly her lower leg below the
knee. She experiences pain on movement of this leg, and there is tenderness to palpation. Which of the
following complications is most likely to occur next after these events?
• Gangrenous necrosis of the foot
• Hematoma of the thigh
• Disseminated intravascular coagulation
• Pulmonary thromboembolism

324. A 43-year-old woman has had a chronic cough with fever and weight loss for the past month. A chest
radiograph reveals multiple nodules from 1 to 4 cm in size, some of which demonstrate cavitation in the
upper lobes. A sputum sample reveals the presence of acid fast bacilli. Which of the following cells is the
most important in the development her lung lesions?
• Macrophage
• Fibroblast
• Neutrophil
• Mast cell

325. A 20-year-old man has experienced painful urination for 4 days. A urethritis is suspected, and
Neisseria gonorrheae is cultured. Numerous neutrophils are present in a smear of the exudate from the
penile urethra. These neutrophils undergo diapedesis to reach the organisms. Release of which of the
following chemical mediators is most likely to drive neutrophil exudation?
• Histamine
• Prostaglandin
• Hageman factor
• Complement

326. An episode of marked chest pain lasting 4 hours brings a 51-year-old man to the emergency room.
He is found to have an elevated serum creatine kinase. An angiogram reveals a complete blockage of the
left circumflex artery 2 cm from its origin. Which of the following substances would you most expect to be
elaborated around the region of tissue damage in the next 3 days as an initial response to promote healing?
• Histamine
• Immunogloblulin G
• Complement component C3b
• Vascular endothelial growth factor

327. A 94-year-old woman has developed a fever and cough over the past 2 days. Staphylococcus aureus
is cultured from her sputum. She receives a course of antibiotic therapy. Two weeks later she no longer has
a productive cough, but she still has a fever. A chest radiograph reveals a 3 cm rounded density in the right
lower lobe whose liquefied contents form a central air-fluid level. There are no surrounding infiltrates.
Which of the following is the best description for this outcome of her pneumonia?
• Hypertrophic scar
• Abscess formation
• Regeneration
• Bronchogenic carcinoma
328. A 36-year-old woman has been taking acetylsalicylic acid (aspirin) for arthritis for the past 4 years.
Her joint pain is temporarily reduced via this therapy. However, she now has occult blood identified in her
stool. Which of the following substances is most likely inhibited by aspirin to cause this complication?
• Leukotriene B4
• Interleukin-1
• Thromboxane
• Bradykinin

329. A small sliver of wood becomes embedded in the finger of a 25-year-old man. He does not remove
it, and over then next 3 days the area around the sliver becomes red, swollen, and tender. Neutrophils
migrate into the injured tissue. Expression of which of the following substances on endothelial cells is most
instrumental in promoting this inflammatory reaction?
• Interferon gamma
• Hageman factor
• Lysozyme
• E-selectin

330. An inflammatory process that has continued for 3 months includes the transformation of tissue
macrophages to epithelioid cells. There are also lymphocytes present. Over time, fibroblasts lay down
collagen as the focus of inflammation heals. These events are most likely to occur as an inflammatory
response to which of the following infectious agents?
• Mycobacterium tuberculosis
• Pseudomonas aeruginosa
• Cytomegalovirus
• Giardia lamblia

331. A 37-year-old man has had nausea and vomiting for 5 weeks. He experienced an episode of
hematemesis yesterday. On physical examination he has no abnormal findings. Upper GI endoscopy is
performed, and there is a 1.5 c diameter lesion in the gastric antrum with loss of the epithelial surface.
These findings are most typical for which of the following pathologic processes?
• Abscess
• Serositis
• Granuloma
• Ulcer

332. A 17-year-old truck driver is involved in a collision. He incurs blunt force abdominal trauma. In
response to this injury, cells in tissues of the abdomen are stimulated to enter the G1 phase of the cell cycle
from the G0 phase. Which of the following cell types is most likely to remain in G0 following this injury?
• Smooth muscle
• Endothelium
• Skeletal muscle
• Fibroblast
333. A 19-year-old woman who works indoors spends a day outside gardening. She does not wear a hat
or sunscreen. That evening her partner remarks that her face appears red. Which of the following dermal
changes most likely accounts for her red appearance?
• Neutrophil aggregation
• Hemorrhage
• Edema
• Vasodilation

334. A 45-year-old woman has had a chronic, non-productive cough for 3 months, along with intermittent
fever. She has a chest radiograph that reveals multiple small parenchymal nodules along with hilar and
cervical lymphadenopathy. A cervical lymph node biopsy is performed. Microscopic examination of the
biopsy shows noncaseating granulomatous inflammation. Cultures for bacterial, fungal, and mycobacterial
organisms are negative. Which of the following chemical mediators is most important in the development
of her inflammatory response?
• Interferon gamma
• Bradykinin
• Complement C5a
• Histamine

335. A 55-year-old man has a history of hypercholesterolemia with coronary artery disease and suffered a
myocardial infarction 2 years ago. He now presents with crushing substernal chest pain. Which of the
following laboratory tests is most useful in diagnosing the cause of his chest pain?
• Increased white blood cell count
• Elevated sedimentation rate
• Decreased serum complement
• Increased serum troponin

336. A 15-year-old girl has had episodes of sneezing with watery eyes and runny nose for the past 2
weeks. On physical examination she has red, swollen nasal mucosal surfaces. She has had similar episodes
each Spring and Summer when the amount of pollen in the air is high. Her symptoms are most likely to be
mediated by the release of which of the following chemical mediators?
• Complement C3b
• Platelet activating factor (PAF)
• Tumor necrosis factor (TNF)
• Histamine

337. A 45-year-old man has been working hard all day long carrying loads of bricks to build a wall. He
takes a nonsteroidal anti-inflammatory drug (ibuprofen). Which of the following processes is this drug most
likely to diminish in his arms?
• Thrombosis
• Pain
• Necrosis
• Fibrinolysis
338. Within minutes following a bee sting, a 37-year-old man develops marked respiratory stridor with
dyspnea and wheezing. He also develops swelling and erythema seen in his arms and legs. An injection of
epinephrine helps to reverse these events and he recovers within minutes. Which of the following chemical
mediators is most important in the pathogenesis of this man's condition?
• Bradykinin
• Complement C5a
• Nitric oxide
• Histamine

339. A 72-year-old woman did not get a 'flu' shot in the fall as recommended for older persons. In the
wintertime, she became ill, as many people in her community did, with a respiratory illness that lasted for 3
weeks. During this illness, she had a fever with a non-productive cough, mild chest pain, myalgias, and
headache. What was her chest radiograph most likely to have shown during this illness?
• Hilar mass
• Interstitial infiltrates
• Hilar lymphadenopathy
• Lobar consolidation

340. In an experiment, Enterobacter cloacae organisms are added to a solution containing leukocytes and
blood plasma. Engulfment and phagocytosis of the microbes is observed to occur. Next a substance is added
which enhances engulfment, and more bacteria are destroyed. Which of the following substances in the
plasma is most likely to produce this effect?
• Complement C3b
• Glutathione peroxidase
• Immunoglobulin M
• P-selectin

341. A 43-year-old woman has had nausea with vomiting persisting for the past 5 weeks. On physical
examination there are no abnormal findings. She undergoes an upper GI endoscopy and gastric biopsies are
obtained. The microscopic appearance of these biopsies shows mucosal infiltration by lymphocytes,
macrophages, and plasma cells. Which of the following most likely caused her findings?
• Staphylococcus aureus septicemia
• Ingestion of chili peppers
• Diabetes mellitus
• Infection with Helicobacter pylori

342. In an experiment, lymphatic channels are observed in normal soft tissue preparations.
Staphylococcus aureus organisms are innoculated into the tissues and the immunologic response observed
over the next 24 hours. Which of the following functions is most likely to be served by these lymphatics to
produce a specific immune response to these organisms?
• Carry lymphocytes to peripheral tissue sites
• Remove extravascular tissue fluid
• Transport antigen presenting cells
• Serve as a route for dissemination of infection

343. In an experiment, surgical wound sites are observed following suturing. An ingrowth of new
capillaries is observed to occur within the first week. A substance elaborated by macrophages is found at
the wound site to stimulate this capillary proliferation. Which of the following substances is most likely to
have this function?
• Platelet-derived growth factor
• Phospholipase C-gamma
• Fibronectin
• Fibroblast growth factor

344. A 55-year-old man with a history of ischemic heart disease has worsening congestive heart failure.
He has noted increasing dyspnea and orthopnea for the past 2 months. On physical examination there is
dullness to percussion at lung bases. A chest x-ray shows bilateral pleural effusions. A left thoracentesis is
performed, and 500 mL of fluid is obtained. Which of the following characteristics of this fluid would most
likely indicate that it is a transudate?
• Cloudy appearance
• High protein content
• <3 lymphocytes/microliter
• Presence of fibrin

345. In a clinical study, patients undergoing laparoscopic cholecystectomy are followed to document the
post-surgical wound healing process. The small incisions are closed with sutures. Over the 4 weeks following
surgery, the wounds are observed to regain tensile strength and there is re-epithelialization. Of the
following substances, which is most likely found to function intracellularly in cells involved in this wound
healing process?
• Fibronectin
• Laminin
• Tyrosine kinase
• Hyaluronic acid

346. A 31-year-old woman has a laparotomy performed for removal of an ovarian cyst. She recovers
uneventfully, with no complications. At the time of surgery, a 12 cm long midline abdominal incision was
made. The tensile strength in the surgical scar will increase so her normal activities can be resumed. Most of
the tensile strength will likely be achieved in which of the following time periods?
• One week
• One month
• Three months
• Six months
347. A 9-year-old girl sustains a small 0.5 cm long laceration to her right index finger while playing 'Queen
of Swords' with a letter opener. Which of the following substances, on contact with injured vascular
basement membrane, activates both the coagulation sequence and the kinin system as an initial response
to this injury?
• Thromboxane
• Plasmin
• Platelet activating factor
• Hageman factor

348. A 65-year-old woman has had a fever for the past day. On physical examination her temperature is
39°C and blood pressure 90/50 mm Hg with heart rate of 106/minute. Laboratory studies show a WBC
count of 12,510/microliter and WBC differential count of 78 segs, 8 bands, 11 lymphs, and 3 monos. A blood
culture is positive for Escherichia coli. Her central venous pressure falls markedly. She goes into
hypovolemic shock as a result of the widespread inappropriate release of a chemical mediator derived from
macrophages. She develops multiple organ failure. Which of the following mediators is most likely to
produce these findings?
• Nitric oxide
• Bradykinin
• Histamine
• Prostacyclin

349. A 20-year-old woman sustains an injury to her right calf in a mountain biking accident. On physical
examination she has a 5 cm long laceration on the right lateral aspect of her lower leg. This wound is closed
with sutures. Wound healing proceeds over the next week. Which of the following factors will be most likely
to aid and not inhibit woun healing in this patient?
• Commensal bacteria
• Decreased tissue perfusion
• Presence of sutures
• Corticosteroid therapy

350. A 24-year-old primigravida is late in the second trimester of pregnancy. She experiences the sudden
onset of some cramping lower abdominal pain. This is immediately followed by passage of some fluid per
vagina along with a foul-smelling discharge. The fetus is stillborn two days later. Examination of the
placenta demonstrates extensive neutrophilic infiltrates in the chorion and amnion. Which of the following
organisms is most likely to be responsible for these findings?
• Mycobacterium tuberculosis
• Herpes simplex virus
• Escherichia coli
• Treponema pallidum

351. A 19-year-old man incurs a stab wound to the chest. The wound is treated in the emergency room.
Two months later there is a firm, 3 x 2 cm nodular mass with intact overlying epithelium in the region of the
wound. On examination the scar is firm, but not tender, with no erythema. This mass is excised and
microscopically shows fibroblasts with abundant collagen. Which of the following mechanisms has most
likely produced this series of events?
• Keloid formation
• Development of a fibrosarcoma
• Poor wound healing from diabetes mellitus
• Foreign body response from suturing

352. A 45-year-old man has had a fever and dry cough for 3 days, and now has difficulty breathing and a
cough productive of sputum. On physical examination his temperature is 38.5°C. Diffuse rales are
auscultated over lower lung fields. A chest radiograph shows a right pleural effusion. A right thoracentesis is
performed. The fluid obtained has a cloud appearance with a cell count showing 15,500 leukocytes per
microliter, 98% of which are neutrophils. Which of the following terms best describes his pleural process?
• Serous inflammation
• Purulent inflammation
• Fibrinous inflammation
• Chronic inflammation

353. A 52-year-old woman with no major medical problems takes a long airplane flight across the Pacific
Ocean. Upon arrival at Sydney's Kingsford Smith airport following the flight from Los Angeles, she cannot
put her shoes back on There is no pain or tenderness. Which of the following is the most likely explanation
for this phenomenon?
• Activation of Hageman factor has led to bradykinin production.
• A lot of drinks were served in the first class section.
• Femoral vein thrombosis developed
• Venous hydrostatic pressure became increased.

354. In an experiment, a lung tissue preparation is exposed to Mycobacterium tuberculosis organisms.


Over the next week, it is observed that granulomas form in the lung. Within the granuloma are found
inflammatory cells expressing class II MHC antigens. These cells elaborate cytokines that promote
fibroblastic production of collagen within the granulomas. From which of the following peripheral blood
leukocytes are these cells bearing class II antigen most likely to be derived?
• Neutrophils
• B cells
• Monocytes
• NK cells

355. A 56-year-old man has had increasing difficulty breathing for the past week. On physical examination
he is afebrile. Auscultation of his chest reveals diminished breath sounds and dullness to percussion
bilaterally. There is 2+ pitting edema present to the level of his thighs. A chest radiograph reveals bilateral
pleural effusions. Which of the following laboratory test findings is he most likely to have?
• Hypoalbuminemia
• Glucosuria
• Neutrophilia
• Anemia
356. A 72-year-old man presents with a 3-day history of progressively worsening productive cough, fever,
chills, and signs of toxicity. Prominent physical findings include signs of consolidation and rales over the
right lung base. Sputum culture is positive for Streptococcus pneumoniae. An intra-alveolar exudate filling
the alveoli of the involved portion of the lung is present. Which of the following types of inflammatory cells
is most likely a prominent feature of this exudate?
• Basophils
• Eosinophils
• Lymphocytes
• Neutrophils

357. A routine complete blood count performed on a 22-year-old medical student reveals an abnormality
in the differential leukocyte count. She has been complaining of frequent sneezing and “watery” eyes
during the past several weeks and reports that she frequently had such episodes in the spring and summer.
Which of the following cell types is most likely to be increased?
• Basophils
• Eosinophils
• Lymphocytes
• Monocytes

358. A 16-year-old boy presents with a 24-hour history of severe abdominal pain, nausea, vomiting, and
low-grade fever. The pain is initially periumbilical in location but has migrated to the right lower quadrant of
the abdomen, with maximal tenderness elicited at a site one-third of the way between the crest of the
ileum and the umbilicus (McBurney point). The leukocyte count is 14,000/mm3, with 74% segmented
neutrophils and 12% bands. Surgery is performed. Which of the following describes the expected findings at
the affected site?
• Fistula (abnormal duct or passage) connecting to the abdominal wall
• Granulation tissue (new vessels and young fibroblasts) with a prominent infiltrate of eosinophils
• Granulomatous inflammation with prominent aggregates of epithelioid cells and multinucleated
giant cells
• Prominent areas of edema, congestion, and a purulent reaction with localized areas of abscess
formation

359. A 2-year-old boy presents with recurrent infections involving multiple organ systems. Extensive
investigation results in a diagnosis of chronic granulomatous disease of childhood. Which of the following
most closely characterizes the abnormality in this patient’s phagocytic cells?
• Decreased killing of microorganisms because of enhanced production of hydrogen peroxide
• Deficiency of NADPH oxidase activity
• Impaired chemotaxis and migration caused by abnormal microtubule formation
• Inability to kill streptococci
360. A laboratory experiment is performed to evaluate the chemotactic potential of a group of potential
mediators. Which of the following substances most likely has the greatest affinity for neutrophils?
• C5a
• Fucosyl transferase
• β2-Integrin
• P-selectin

A 26-year-old African-American woman has bilateral hilar adenopathy, and radiography reveals multiple
reticular densities in both lung fields. A bronchoscopic biopsy reveals granulomatous inflammation with multiple
giant cells of the Langhans type and no evidence of caseous necrosis. Which of the following is the most likely
diagnosis?

• Aspergillosis
• Coccidioidomycosis
• Histoplasmosis
• Sarcoidosis

361. In a laboratory exercise for medical students, an unknown compound is studied. The students are
informed that the compound has been isolated from endothelial cells and that its synthesis can be inhibited
by aspirin. In the laboratory, the students demonstrate that the compound is a potent vasodilator and
platelet antiaggregant. Given these findings, the substance is most likely which of the following mediators?
• 5-HPETE
• LTC4
• LXA4
• PGI2

362. A 70-year-old man presents with the sudden onset of left-sided weakness, spasticity, and
hyperactive and pathologic reflexes. The most serious consequences of this disorder are the result of
damage to which of the following cell types?
• Labile cells
• Multipotent adult progenitor cells
• Permanent cells
• Stable cells

363. Inflammation of stomach is called:


• Gastritis.
• Enteritis.
• Colitis.
• Proctitis.

364. Inflammation of liver is called:


• Gastritis.
• Hepatitis
• Colitis.
• Proctitis.

365. Inflammation of kidney is called:


• Gastritis.
• Hepatitis.
• Nephritis.
• Proctitis.

366. Inflammation of small intestine is called:


• Gastritis.
• Enteritis.
• Colitis.
• Proctitis.

367. Inflammation of large intestine is called:


• Gastritis.
• Enteritis.
• Colitis.
• Proctitis.

368. Inflammation of rectum is called:


• Gastritis.
• Enteritis.
• Colitis.
• Proctitis.

369. Inflammation of skin is called:


• Gastritis.
• Hepatitis.
• Nephritis.
• Dermatitis.

370. Inflammation of blood vessels is called:


• Gastritis.
• Hepatitis.
• Nephritis.
• Vasculitis.
371. Inflammation of lung tissue is called:
• Pneumonia.
• Angina.
• Furuncle.
• Carbuncle.

372. Inflammation of throat is called:


• Pneumonia.
• Angina.
• Furuncle.
• Carbuncle.

373. Inflammation of hair follicle is called:


• Pneumonia.
• Angina.
• Furuncle.
• Carbuncle.

374. Inflammation of brain is called:


• Gastritis.
• Hepatitis.
• Enteritis.
• Encephalitis.

375. Local sign of inflammation “rubor” means:


• Heat.
• Pain.
• Redness.
• Swelling.

376. Local sign of inflammation “calor” means:


• Heat
• Pain.
• Redness.
• Swelling.

377. Local sign of inflammation “dolor” means:


• Heat.
• Pain.
• Redness.
• Swelling.
378. Local sign of inflammation “tumor” means:
• Heat.
• Pain.
• Redness.
• Swelling.

379. Local sign of inflammation which associated with pressure on nerve endings by exudate is:
• Rubor.
• Calor.
• Dolor.
• Tumor.

380. Local sign of inflammation which associated with accumulation of exudate is:
• Rubor.
• Calor.
• Dolor.
• Tumor.

381. Calor as a local sign of inflammation is associated with:


• Arterial hyperemia.
• Pressure of nerve endings.
• Exudation
• Interference with enzyme function

382. Tumor as a local sign of inflammation is associated with:


• Arterial hyperemia.
• Pressure of nerve endings.
• Exudation.
• Interference with enzyme function.

383. Dolor as a local sign of inflammation is associated with:


• Arterial hyperemia.
• Pressure of nerve endings.
• Exudation.
• Local pain.

384. Rubor as a local sign of inflammation is associated with:


• Arterial hyperemia.
• Pressure of nerve endings.
• Exudation.
• Interference with enzyme function.

385. Component of alteratio stage of inflammation is:


• Necrosis.
• Arterial hyperemia.
• Phagocytosis.
• Regeneration of epithelial cells.

386. Component of alteratio stage of inflammation is :


• Degeneration of cells.
• Arterial hyperemia.
• Phagocytosis.
• Regeneration of epithelial cells

387. Component of exudation stage of inflammation is:


• Degeneration of cells.
• Arterial hyperemia.
• Necrosis.
• Regeneration of epithelial cells.

388. Component of exudation stage of inflammation is:


• Degeneration of cells
• Necrosis.
• Formation of cell infiltrate
• Regeneration of epithelial cells.

389. Component of exudation stage of inflammation is:


• Degeneration of cells.
• Necrosis.
• Regeneration of epithelial cells.
• Phagocytosis.

390. Component of exudation stage of inflammation is:


• Degeneration of cells.
• Necrosis.
• Pinocytosis
• Regeneration of epithelial cells.
391. Component of exudation stage of inflammation is:
• Degeneration of cells.
• Necrosis.
• Regeneration of epithelial cells
• Leakage of blood plasma from vessels.

392. Component of exudation stage of inflammation is:


• Degeneration of cells.
• Necrosis.
• Regeneration of epithelial cells.
• Emigration of blood cells from vessels.

393. Component of proliferation stage of inflammation is:


• Degeneration of cells.
• Necrosis.
• Regeneration of epithelial cells.
• Emigration of blood cells from vessels

394. Component of proliferation stage of inflammation is :


• Degeneration of cells.
• Necrosis.
• Regeneration of fibroblasts.
• Emigration of blood cells from vessels.

395. Type of acute inflammation which associated with formation of protein-poor fluid is called:
• Serous.
• Fibrinous.
• Purulent
• Hemorrhagic.

396. Type of acute inflammation which associated with formation of hair-like deposits is called:
• Serous.
• Fibrinous.
• Purulent.
• Hemorrhagic.

397. Type of acute inflammation which associated with formation of pseudomembrane on mucosal
surfaces is called:
• Serous.
• Fibrinous.
• Purulent.
• Hemorrhagic.

398. Type of acute inflammation which associated with formation of thick turbid yellow-green fluid is
called:
• Serous.
• Fibrinous.
• Purulent.
• Catarrhal.

399. Which type of pneumonia acute is associated with formation of fibrinous exudate:
• Lobar pneumonia.
• Bronchpneumonia.
• Interstitial pneumonia.
• Septic pneumonia.

400. Localized type of purulent inflammation is called:


• Phlegmon.
• Empyema.
• Abscess.
• Furuncle.

401. Diffuse type of purulent inflammation is called:


• Phlegmon.
• Empyema.
• Abscess.
• Furuncle.

402. Type of purulent inflammation with accumulation of pus in serosal cavities and hollow organs is
called:
• Phlegmon.
• Empyema.
• Abscess.
• Furuncle.

403. Type of purulent inflammation with accumulation of pus in serosal cavities and hollow organs is
called :
• Phlegmon.
• Empyema.
• Abscess.
• Furuncle.

404. Purulent inflammation of hair follicle is called:


• Phlegmon.
• Empyema.
• Abscess.
• Furuncle.

405. Which of the following are thought to mediate, many of the systemic effects of inflammation are
chemotactic and stimulate adhesion molecules:
• Interleukin-1 (IL-1) and tumor necrosis factor
• C5 a and leukotriene B-4
• C3 b.
• Leukotriene C4, D4 and E4.

406. After initiation of an acute inflammatory process third in a sequence of changes in vascular flow is:
• Vasoconstriction.
• Redness.
• Leukocytic migration.
• Vasodilation

407. The term “croupous” inflammation is synonymous with


• Catarrhal inflammation
• Fibrinous inflammation
• Sero-fibrinous inflammation
• Suppurative inflammation

408. Vascular phenomenon in acute inflammation helps in:


• Bringing the defensive cells like neutrophils from interior of the vessels to the site of irritation
• Minimizing the effect of irritant by diluting it considerably
• Limiting the inflammatory reaction to the smallest possible area
• All of these

409. Which of the complement components act as chemokines?


• C3b
• C4b
• C5a
• C4a
410. All are types of tissue macrophages, except:
• Littoral cells
• Hoffbauer cells
• Osteoclasts
• Osteoblasts

411. Main cytokines acting as mediators of inflammation are as under, except:


• Interleukin-1 (IL-1)
• Tumor necrosis factor α (TNF-α)
• Nitric oxide (NO)
• Interferon –y (IF-y)
412.
All are true about Fanconi anemia, except:
• Defect in DNA repair
• Bone marrow hyper function
• Congenital anomaly present
• Increased chances of cancer

413. HER2/neu receptor plays a role in


• Predicting therapeutic response
• Diagnosis of breast cancer
• Screening of breast cancer
• Recurrence of tumor

414. The most common secondary malignancy in a patient having retinoblastoma is:
• Osteosarcoma
• Renal cell carcinoma
• Pineoblastoma
• Osteoblastoma

415. Regarding Fanconi anemia, the wrong statement is:


• Autosomal dominant
• Bone marrow show pancytopenia
• Usually aplastic anemia
• It is due to defective DNA repair

416. True statements about p53 gene are all except:


• Arrests cell cycle at G1 phase
• Product is 53 kD protein
• Located on chromosome 17
• Wild/non-mutated form is associated with increased risk of childhood tumors

417. Growth factor oncogene is:


• Myc
• Fos
• Sis
• Jun

418. Rosettes are characteristically seen in


• Retinoblastoma
• Melanoma
• Dysgerminoma
• Lymphoma

419. The normal cellular counterparts of oncogenes are important for the following functions, except:
• Promotion of cell cycle progression
• Inhibition of apoptosis
• Promotion of DNA repair
• Promotion of nuclear transcription

420. An example of a tumor suppressor gene is:


• Myc
• Fos
• ras
• Rb

421. Lynch syndrome is associated with cancers of the:


• Breast, colon, ovary
• Breast, endometrium, ovary
• Breast, colon, endometrium
• Colon, endometrium, ovary

422. Loss of heterozygosity associated with:


• Acute myeloid leukemia
• ALL
• Retinoblastoma
• Promyelocytic leukemia

423. Which is not a tumor suppressor gene?


• WT-1
• Rb
• P53
• Ras

424. The inheritance pattern of familial Retinoblastoma is:


• Autosomal recessive
• Autosomal dominant
• X-linked dominant
• X-linked recessive

425. Which of the following is known as the “guardian of the genome”?


• P53
• Mdm2
• P14
• ATM

426. The following statements are true about Tumor Suppressor Gene p53, except:
• It regulates certain genes involved in cell cycle regulation
• Its increased levels can induce apoptosis
• Its activity in the cells decreases following UV irradiation and stimulates cell cycle
• Mutations of the p53 gene are most common genetic alteration seen in human cancer

427. In the mitogen activated protein kinase pathway, the activation of RAS is counteracted by:
• Protein kinase C
• GTPase activating protein
• Phosphatidyl inositol
• Inositol triphosphate

428. Which of the following mutations in a tumor suppressor agent causes breast carcinoma?
• p43
• P53
• p73
• P83

429. True about proto-oncogenes is:


• Important for normal cell growth
• May get converted into oncogenes
• C-myc over-expression causes lymphoma
• Their mutation causes retinoblastoma

430. True about oncogene is:


• Present in normal cell
• They are of viral origin
• They are transduced from virus infected cells
• P53 is most common oncogene mutation causing malignancy

431. Cancer cell survival is enhanced by:


• Suppression of p53 protein
• Over expression of p53 gene
• bcl-2
• Bax
432. Following are required for normal growth
• Proto-oncogenes
• Tumor suppressor genes
• Oncogenes
• DNA repair genes

433. Xeroderma pigmentosum is caused due to a group of closely related abnormalities in:
• Mismatch repair
• Base excision repair
• Nucleotide excision repair
• SOS repair

434. Increased expression of which of the following causes oncogenesis


• IGF receptor
• EGF receptor
• GH receptor
• Aldosterone receptor

435. Tumor suppressor genes are all, except


• APC
• p53
• Rb
• C-myc

436. Angiogenesis is
• Formation of the new blood vessels
• Repair by connective tissues
• Formation of the blood clot
• All of the above

437. Medullary carcinoma of thyroid is associated with mutation in:


• RET
• RAS
• NF
• Rb

438. APC gene is located on which chromosome


• Chromosome 5
• Chromosome 6
• Chromosome 9
• Chromosome 11

439. Proto-oncogene erb-B is not related to:


• Breast carcinoma
• Small cell lung carcinoma
• Non-small cell lung carcinoma
• Ovarian carcinoma

440. Most common genetic mutation in carcinogenesis involves:


• p53
• Rb
• HPC
• PTEN

441. Which of the following is a true statement regarding the above gene?
• It is associated with the
• It is called as
• It is active in hypopohosphorylated form
• It is active in hyperphosphorylated form

Aisha, a 51 year old woman discovers a lump in her left breast on a weekly self-examination. Mammography is
performed which confirms the presence of a suspicious “mass”, and needle core biopsy is performed to determine
whether the mass is malignant. Dr. Devesh, the pathologist confirmed the mass to be malignant and said that the
tissue demonstrates amplification of her-2/ neu oncogene. What kind of protein is the gene product of Her-2/neu

• GTPase
• GTPase-activating protein
• Receptor tyrosine kinase
• Retinoic acid receptor protein

442. A patient Madhu undergoes total thyroidectomy for a mass lesion of the thyroid. During the surgery it is
found that the parathyroid glands appeared enlarged. The thyroid lesion shows neuroendocrine-type cells
and amyloid deposition. This patient’s thyroid and parathyroid lesions may be related to which of the
following oncogenes?
• Bcl-2
• C-myc
• Ret
• L-myc
443. Dr. Marwah, a pediatrician, performing an ophthalmoscopic examination on a four-year-old boy, notices
several small pigmented nodules in his irises. He also notices six light brown macules on the trunk of the child
of variable sizes. This boy may have a propensity to develop tumors in which of the following structures?
• Bladder
• Colon
• Peripheral nerve
• Skin

444. RET gene mutation is associated with which malignancy?


• Pheochromocytoma
• Medullary carcinoma thyroid
• Lymphoma
• Renal cell carcinoma

445. Endometrial carcinoma is associated with which of the following tumor suppression gene mutation?
• P53
• Rb
• PTEN
• APC

446. The tumor suppressor gene p 53 induces cell cycle arrest at:
• G2
• S
• G1 – S phase
• G0 phase

447. MYC gene is:


• Protein kinase inhibitor
• Growth factor inhibitor
• GTPase
• Transcription activator

448. Retinoblastoma is associated with which of the following tumours?


• Osteoclastoma
• Hepatocellular cancer
• Squamous cell cancer
• Osteosarcoma

449. An example of a tumour suppressor gene is:


• Myc
• Fos
• Ras
• RB

450. Which of the following is DNA repair defect?


• Retinoblastoma
• Neurofibromatosis
• Xeroderma pigmentosum
• MEN-I

451. Which of the following gene defect is associated with development of medullary carcinoma of thyroid:
• RET Proto Oncogene
• Fap gene
• Rb gene
• BRCA 1 gene

452. All of the following are tumor markers, except:


• Beta-2 macroglobulin
• HCG
• Alpha-fetoprotein
• CEA

453. Knudson two hit hypothesis is seen with


• Melanoma
• Retinoblastoma
• Ulcerative colitis
• Crohn disease
454. Retinoblastomas arising in the context of germ-line mutations not only may be bilateral, but also may be
associated with (so called “trilateral” retinoblastoma)
• Medulloblastoma
• Pinealoblastoma
• Neuroblastoma
• Hemangioblastoma

455. Post transplant lymphoma is caused by which of the following?


• CMV
• EBV
• Herpes simplex
• HHV-6
456. H. pylori infection is associated with development of which malignancy:
• MALTomas
• Atherosclerosis
• Sarcoma
• Gastrointestinal stromal tumor (GIST)

457. Helicobacter pylori infection is associated with all of the following conditions, except:
• Peptic ulcer disease
• Gastric adenocarcinoma
• B cell lymphoma
• Burkitt’s lymphoma

458. Tumors associated with organisms are all except:


• Hepatocellular cancer
• Non-small Cell Carcinoma of Lung
• Gastric cancer
• Nasopharyngeal cancer

459. Which of the following is essential for tumor metastasis?

• Angiogenesis
• Tumorogenesis
• Apoptosis
• Inhibition of tyrosine kinase activity

460. Which of the following statements about carcinogenesis is false?


• Asbestos exposure increases the incidence of lung cancer
• Papilloma viruses produce tumors in animals but not in humans
• Exposure to aniline dyes predisposes to cancer of the urinary bladder
• Hepatitis B virus has been implicated in hepatocellular carcinoma

461. Smoking is a risk factor for all carcinomas , except;


• Oral
• Bronchial
• Bladder
• Thyroid
462. Workers exposed to polyvinyl chloride may develop following liver malignancy
• Cholangiocarcinoma
• Fibrolamellar carcinoma
• Angiosarcoma
• All of the above

463. Which among the following is not a neoplastic virus:


• Cytomegalovirus
• Hepatitis B virus
• Human papilloma virus
• All of these

464. A 37-year-old man, Gagan presents with increasing abdominal pain and jaundice. He gives a history of intake
of groundnuts which did not taste appropriate. Physical examination reveals a large mass involving the right
side of his liver, and a biopsy specimen from this mass confirms the diagnosis of liver cancer (hepatocellular
carcinoma). Which of the following substances is most closely associated with the pathogenesis of this
tumor?
• Aflaxotin B1
• Direct-acting alkylating agents
• Vinyl chloride
• Azo dyes

465. Biopsy of an ulcerated gastric lesion of a 26-year-old smoker Akki demonstrates glands containing cells with
enlarged, hyperchromatic nuclei below the muscularis mucosa. Two tripolar mitotic figures are noted. With
which of the following infectious agents has this type of lesion been most strongly associated?
• Epstein-Barr virus
• Helicobacter pylori
• Human papilloma virus
• Molluscum contagiosum virus

466. A man Alok Nath contracts HTLV-1 infection through sexual contact. Twenty-one years later he develops
generalized lymphadenopathy with hepatosplenomegaly, a skin rash, hypercalcemia, and an elevated white
blood count. This man has most likely developed which of the following?
• AIDS
• Autoimmunity
• Delayed hypersensitivity reaction
• Leukemia

467. Thorium induced tumor is which of the following?


• Renal cell carcinoma
• Lymphoma
• Angiosarcoma of liver
• Astrocytoma

468. Radiation exposure during infancy has been linked to which one of the following carcinoma?
• Breast
• Melanoma
• Thyroid
• Lung

469. The following parasitic infections predispose to malignancies?


• Paragonimus westermani
• Guinea worm infection
• Clonorchiasis
• Schistosomiasis

470. Kaposi’s sarcoma is seen with


• HCV
• HPV
• HSV
• HHV

471. UV radiation has which of the following effects on the cells?


• Prevents formation of pyrimidine dimers
• Stimulates formation of pyrimidine dimers
• Prevents formation of purine dimers
• All of the above

472. The most radiosensitive cells are:


• Neutrophils
• Lymphocytes
• Erythrocytes
• Megakaryocytes

473. The SI unit of radiation absorbed dose is


• Rad
• Becquerel
• Gray
• Sievert

474. One of the following leukemia almost never develops after radiation?
• Acute myeloblastic leukemia
• Chronic myeloid leukemia
• Acute lumphoblastic leukemia
• Chronic lymphocytic leukemia

475. A 20 year old female was diagnosed with granulose cell tumor of the ovary. Which of the following bio
markers would be most useful for follow-up of patient?
• CA 19-9
• CA50
• Inhibin
• Neuron – specific enolase

476. Alpha fetoprotein is a marker of:


• Hepatoblastoma
• Seminoma
• Renal cell carcinoma
• Choriocarcinoma
477. Hyperglycemia associated with:
• Multiple myeloma
• Ewing sarcoma
• Osteosarcoma
• Chondroblastoma

478. Which of the following is Not associated with thymoma?


• SIADH
• Myasthenia gravis
• Polymyositis
• Hypogammaglobinemia

479. Which of the following is not true about Neuroblastoma?


• Most common extracranial solid tumor in childhood
• >50% patients present with metastasis at time of diagnosis
• Lung metastases are common
• Involve aorta and its branches early

480. Migratory thrombophlebitis is associated with all of t


• Medullary carcinoma of thyroid
• Pancreatic neuroendocrine tumor
• Pheochromocytoma
• Gastrointestinal stromal tumor

481. All of the following are examples of tumor markers, except:


• Alpha-HCG (a-HCG)
• Alpha-Feto protein
• Thyroglobulin
• Beta 2-microglobulin

482. Which of the following tumors have an increased elevation of placental alkaline phosphatase in the serum as
well as a positive immunohistochemical staining for placental alkaline phosphatase?
• Seminoma
• Hepatoblastoma
• Hepatocellular carcinoma
• Peripheral neuroectodermal tumor

483. In tumor lysis syndrome, all of the following are seen, except:
• Hypernatremia
• Hypercalcemia
• Hyperkalemia
• Hyperphosphatemia
484. Uses of tumor marker are:
• Screening of a cancer
• Follow up of a cancer patient, esp. for knowing about recurrence
• Confirmation of a diagnosed cancer
• For monitoring the treatment of a cancer

485. True about Carcinoembryonic antigen (CEA):


• Useful for screening of carcinoma colon
• Gives confirmative evidence of Ca. colon
• Helpful for follow-up after resection
• Levels decrease immediately after resection of tumor

486. CA·125 is associated with:


• Colon ca
• Breast ca
• Ovarian ca
• Bronchogenic ca

487. Secondaries are common in all, except:


• Skull
• Hand and feet bones
• Proximal limb bones
• Pelvic
488. Hybridoma refers to
• Collision tumor
• A tumor of brown fat
• A hamartoma
• A technique for raising monoclonal antibodies

489. BCL2 is a marker for:


• Follicular lymphoma
• Mycosis fungoides
• B-cell lymphoma
• Mantle cell lymphoma

490. alfa-fetoproteins are a marker of:


• Secondaries in liver
• Cholangiocarcinoma
• Hepatoma
• None of the above
491. Increased level of alpha fetoprotein is found in
• Yolk sac tumor
• Seminoma
• Teratoma
• Choriocarcinoma

492. Migratory thrombophlebitis is seen in:


• Disseminated cancer
• Rheumatic heart disease
• Libman-Sachs endocarditis
• All of the above

A 65 years old male diagnosed by biopsy a case of lung carcinoma, with paraneoplastic syndrome and
increased calcium. Probable cause is

• Parathyroid hormone
• Parathyroid hormone related peptide
• Calcitonin
• Calcitonin related peptide

493. Which is associated with polycythemia:


• Gastric carcinoma
• Fibrosarcoma
• Cerebellar hemangioblastoma
• All
494. Serum AFP is increased in all, except:
• Acute hepatitis
• Hepatocellular carcinoma
• Hepatoma
• Bladder carcinoma

495. Carcinoembryonic antigen is elevated in all, except:


• Alcoholic cirrhosis
• Ca colon
• Ulcerative colitis
• Emphysema

496. Desmoid tumor arises from


• Wall of the intestine
• Anterior abdominal wall
• Submucosa
• Appendix

497. Alpha-fetoprotein is a tumor marker of:


• Carcinoma ovary
• Liver malignancies
• Endodermal sinus tumor of testis
• Both (b) and (c)

498. A-fetoprotein is seen in all except:


• Hepatocellular carcinoma
• Carcinoma colon
• Pancreatic carcinoma
• Germ cells of testes

499. The diagnostic tumor marker of liver carcinoma is:


• CEA
• AFP
• CA - 125
• All of the above

500. Spontaneous regression of tumor is seen in:


• Wilm’s tumor
• Neuroblastoma
• Acute monocytic leukemia
• Hepatoblastoma

501. All of the following about tumor markers are properly matched, except:
• Prostate cancer - PSA
• Colon cancer - CEA
• Ovarian cancer – CA 125
• Cholangiocarcinoma - AFP

502. Popcorn calcification is seen in:


• Chondrosarcoma
• Fibrous dysplasia
• Osteoblastoma
• Wilms’ tumor
503. Which one of the following is a frequent cause of serum alpha- fetoprotein level greater than 10 times the
normal upper limit?
• Seminoma
• Hepatocellular carcinoma of liver
• Cirrhosis of liver
• Oat cell tumor of lung

504. Rise of AFP is noted in all except:


• Hepatocellular carcinoma
• Cirrhosis
• Germ cell tumor
• Kidney tumor

505. Catecholamines are increased in:


• Neuroblastoma
• Retinoblastoma
• Medulloblastoma
• Nephroblastoma

506. A 60-year-old man, Shibu is found to have a 3.5-cm mass in the right upper lobe of his lung. A biopsy of this
mass is diagnosed as a moderately differentiated squamous cell carcinoma. Workup reveals that no bone
metastases are present, but laboratory examination reveals that the man’s serum calcium levels are 11.5
mg/dL. This patient’s paraneoplastic syndrome is most likely the result of the ectopic production of which of
the following substances?
• Parathyroid hormone
• Parathyroid hormone-related peptide
• Calcitonin
• Calcitonin-related peptide

507. During a routine physical examination, a 45-year-old woman Nusheen is noted to have a ruddy complexion.
Her hematocrit is 52%. Her lungs are clear and she does not smoke. Serum erythropoietin levels are elevated.
Cancer of which of the following organs is the most likely cause of her increased hematocrit?
• Breast
• Colon
• Kidney
• Stomach

508. A 62 year-old woman Omvati with advanced, metastatic lung cancer develops profound fatigue and
weakness and alternating diarrhea and constipation. Physical examination demonstrates hyperpigmentation
of skin, even in areas protected from the sun. Tumor involvement of which endocrine organ is most strongly
suggested by this patient’s presentation?
• Adrenal gland
• Endocrine pancreas
• Ovaries
• Pituitary gland

509. An old man Velu presents with complaints of abdominal and back pain, malaise, nausea, 8 kg weight loss and
weakness, which have been present for 3 or 4 months. His history also reveals several episodes of unilateral
leg swelling, which have involved both legs at different times. These findings are most consistent with which
of the following diagnoses?
• Pancreatic cancer
• Primary sclerosing cholangitis
• Splenic infarction
• Reflux esophagitis

510. A 65-year-old woman Ramkali presents to the emergency room with a pathologic fracture of the shaft of her
humerus. X-ray studies demonstrate multiple lytic and blastic bone lesions. Biopsy of one of these lesions
shows adenocarcinoma. Which of the following is the most likely source of the primary tumor?
• Breast
• Colon
• Kidney
• Lung

511. Tumor that follows rule of 10 is:


• Pheochromocytoma
• Oncocytoma
• Lymphoma
• Renal cell carcinoma

512. Which of the followingis a squamous cell carcinoma marker?


• Vimentin
• Desmin
• Cytokeratin
• Glial fibrillary acid protein
513. Marker for ovarian carcinoma in serum is:
• CA-125
• Fibronectin
• Acid Phosphatase
• PSA

514. Which of the following is a marker for carcinoma of lung and breast?
• CEA
• AEP
• HCG
• CA-15-3

515. Secondaries of all the following cause osteolytic lesions except:


• Prostate
• Kidney
• Bronchus
• Thyroid

516. Sacrococcygeal teratoma, marker is:


• CEA
• β- HCG
• S100
• CA-125

517. Which of the following mutation is seen in malignant melanoma


• N-myc
• CDKN2A
• RET
• Rb

518. Marker of small cell cancer of lung is:


• Chromogranin
• Cytokeratin
• Desmin
• Vimentin

519. Which of the following is tumor marker of seminoma?


• AFP
• LDH
• PLAP
• HCG

520. Which of the following is a special stain for rhabdomyosarcoma?


• Cytokeratin
• Synaptophysin
• Desmin
• Myeloperoxidase
521. The most common cause of malignant adrenal mass
• Adrenocortical carcinoma
• Malignant Phaeochromocytoma
• Lymphoma
• Metastasis from another solid tissue tumor

522. Commonest cancer in which metastasis is seen in brain in


• Breast
• Lung
• Kidney
• Intestines

523. Which of the following is incorrect about neuro- blastoma ?


• Most common abdominal tumor in infants
• X-ray abdomen shows calcification
• Can show spontaneous regression
• Urine contains 5H.I.A.A

524. Colloid degeneration is occurs in:


• Salivary gland
• Prostate gland.
• Thyroid gland
• Pituitary gland

525. An 87-year-old male develops worsening heart failure. Workup reveals decreased left ventricular filling due
to decreased compliance of the left ventricle. Two months later the patient dies, and postmortem sections
reveal deposits of eosinophilic, Congo red positive material in the interstitial of his heart. When viewed under
polarized light. This material displays an apple-green birefringence. What is the correct diagnosis?
• Amyloidosis
• Glycogenosis
• Hemochromatosis
• Sarcoidosis
526. In cases of renal failure on long-term hemodialysis, there is development of following type of amyloid:
• Amyloid light chain (AL)
• Amyloid-associated protein (AA)
• Amyloid β2 microglobulin (Aβ2m)
• β amyloid protein (Aβ)

527. The most common form of amyloid in third world countries is:
• Primary
• Secondary
• Hereditary
• Localized

528. Cardiac amyloidosis often produces:


• Dilated cardiomyopathy
• Constrictive cardiomyopathy
• Restrictive cardiomyopathy
• Ischemic cardiomyopathy

529. In senile cardiac amyloidosis, the biochemical form of amyloid is:


• AL
• AA
• ATTR
• Aβ2m

530. Health risk in obesity is due to weight in excess of the following for age and sex:
• 0.1
• 0.2
• 0.3
• 0.4

531. Obesity is due to:


• Hyperplasia of adipocytes only
• Hypertrophy of adipocytes only
• Hyperplasia as well as hypertrophy of adipocytes
• Fatty change in liver only

532. Most often secondary amyloidosis occurs when the following pathologies:
• Chronic suppuration
• Acute inflammation
• Cellular necrosis
• Hyaline degeneration

533. The substance with fibrillar structure, which forms under the pathological conditions, is:
• Reabsorption droplets
• Russell bodies
• Lipids
• Amyloid
534. The substance giving red color with the Congo red stain is:
• Lipid
• Hyaline
• Water
• Amyloid

535. In long-standing hypertension and diabetes mellitus, the walls of arterioles, especially in the kidney, become:
• Serous
• Thined
• Hyalinized
• Ulcered

536. The pathologic proteinaceous substance, accumulating only between cells in various tissues and organs of
the body is:
• Glycogen
• Hyaline
• Water
• Amyloid

537. The pathologic proteinaceous substance, accumulating both within cells and in the extracellular matrix in
various tissues and organs of the body is:
• Glycogen
• Hyaline
• Water
• Amyloid
538. The characteristics of amyloid fibrils include all of the following, except:
• Fibril composed of paired filaments
• Nonbranching fibrils
• Fibrils with an indefinite diameter
• Fibrils with definite length

539. The organ affected in both primary and secondary amyloidosis is:
• Kidney
• Stomach
• Uterus
• Brain

540. The deposits of amyloid sees in all tissues, except:


• Mesangium and capillary loops
• Basement membranes of blood vessels
• Tubular basement membranes
• Epithelium in proximal renal tubules

541. All of the following pathological processes are reversible, except:


• Mucoid changes
• Cellular swelling
• Amyloidosis
• Edema

542. The organ most commonly and seriously damaged in amyloidosis is:
• Stomach
• Kidney
• Lung
• Liver

543. Reactive systemic amyloidosis knows to be all of the following, except:


• Secondary amyloidosis
• Complication of tuberculosis
• Hereditary amyloidosis
• Complication of osteomyelitis
544. Amyloid in primary amyloidosis is usually systemic and belongs to the following biochemical type:
• AL type
• AA type
• Aβ type
• ATTR type

545. Reactive systemic amyloidosis occurs in association with all of the following diseases, except:
• Tuberculosis
• Bronchiectasis
• Chronic osteomyelitis
• Hepatitis B

546. Reactive systemic amyloidosis occurs in association with all of the following diseases, except
• Rheumatoid arthritis
• Ankylosing spondylitis
• Chronic appendicitis
• Myeloma

547. Secondary amyloidosis damages all of the following organs, except:


• Kidneys
• Liver
• Spleen
• Brain

548. Macroscopically the organs affected by amyloidosis are characterized by all of the following, except:
• Enlarged
• Firm
• Waxy in appearance
• Soft

549. The common cause of death in patients with secondary amyloidosis is insufficiency of:
• Kidneys
• Heart
• Liver
• Lung

550. The organs that should be histological examined in patients with amyloidosis are all of the following, except:
• Kidney
• Eye
• Rectum
• Gingiva

551. The reversible process caused by accumulation of glycosaminoglycans in extracellular matrix due to the
increase of vascular permeability is:
• Amyloidosis
• Glycogenoses
• Hyalinosis
• Mucoid changes

552. The irreversible process caused by accumulation of proteins with high molecular weight associated with the
destruction of connective tissue is:
• Amyloidosis
• Glycogenoses
• Hyalinosis
• Fibrinoid changes

553. All the pathological processes are irreversible, except


• Mucoid changes
• Fibrinoid changes
• Amyloidosis
• Apoptosis

554. In amyloidosis macroscopically kidney changed in the following way:


• Cut surface is pale and translucent
• Dense and sharply reduced
• Enlarged and dense
• True a + c

555. All examples of extracellular hyaline are change, except:


• Old scar
• Hyaline arteriolosclerosis
• Chronic glomerulonephritis
• Mallory’s hyaline

556. In amyloidosis macroscopically liver changed in the following way:


• Enlarged, pale, waxy and firm
• Dense and sharply reduced
• Enlarged and yellow color
• Soft and enlarged

557. In amyloidosis macroscopically spleen changed in the following way:


• Cut surface is translucent pale and waxy
• Dense and sharply reduced
• Cut surface – map-like areas of amyloid
• True a + c

558. Causes of hyalinosis are all of the following, EXCEPT:


• Fibrinoid changes
• Inflammation
• Apoptosis
• Sclerosis

559. Most specific histological sign in mucoid changes is:


• Metachromasia.
• Metaplasia.
• Metastasis.
• Metakinesis.

560. Most common localization of mucoid changes is


• Nervous system.
• Cardiovascular system
• Urinary system
• Blood system

561. Simple hyaline occurs in:


• Diabetes mellitus.
• Arterial hypertension
• Rheumatic fever
• Rheumatoid arthritis

562. Simple hyaline occurs in


• Diabetes mellitus
• systemic lupus erythematosus
• Atherosclerosis.
• Rheumatic fever.

563. Lipohyalin occurs in:


• Diabetes mellitus.
• Arterial hypertension
• Atherosclerosis
• Rheumatic fever

564. Compound hyalin occurs in:


• Diabetes mellitus.
• Arterial hypertension
• Atherosclerosis.
• Rheumatic fever.

565. Compound hyalin occurs in


• Diabetes mellitus
• Arterial hypertension
• Atherosclerosis
• Rheumatoid arthritis
566. The stain used to identify amyloid is
• Hematoxylin and eosin stain.
• Metachromatic stain
• Congo-red stain
• Sudan III stain.

567. The pathologic proteinaceous substance, accumulating only between cells in various tissues and organs of
the body is
• Glycogen.
• Hyaline.
• Water
• Amyloid.

568. Specific method for diagnosis of amyloid in fresh tissue is:


• Virchow test.
• Rokitansky test.
• Ewing test
• Masson test

569. Variant of systemic amyloidosis is:


• Senile cardiac.
• Secondary
• Senile cerebral.
• Endocrine.

570. Variant of systemic amyloidosis is


• Senile cardiac.
• Senile cerebral
• Heredofamilial
• Endocrine

571. Variant of systemic amyloidosis is


• Senile cardiac
• Senile cerebral
• Endocrine.
• Hemodialysis-associated
572. Variant of localized amyloidosis is:
• Senile cardiac.
• Primary
• Secondary
• Heredofamilial
573. Variant of localized amyloidosis is
• Primary
• Secondary.
• Senile cerebral
• Heredofamilial

574. Variant of localized amyloidosis is


• Primary
• Secondary.
• Heredofamilial.
• Endocrine

575. Variant of localized amyloidosis is?


• Primary.
• Secondary
• Heredofamilial
• Tumor-forming

576. Localization of pericollagenous amyloidosis is:


• Liver
• Spleen
• Heart.
• Kidneys

577. Localization of pericollagenous amyloidosis is


• Liver
• Spleen
• Bowel.
• Kidneys

578. Localization of pericollagenous amyloidosis is:


• Liver
• Spleen
• Nerves
• Kidneys

579. Localization of perireticulin amyloidosis is:


• Heart
• Tongue.
• Nerves
• Kidneys

580. Localization of perireticulin amyloidosis is:


• Heart
• Tongue
• Nerves
• Liver

581. Localization of perireticulin amyloidosis is :


• Heart
• Tongue
• Nerves
• Spleen

582. Cause of obesity which associated with excessive nutrition is called:


• Primary.
• Alimentary
• Cerebral
• Endocrine

583. Type of obesity with unknown cause is called:


• Primary
• Alimentary
• Cerebral
• Endocrine

584. 1st degree of obesity is associated with increasing of body weight over than normal by:
• 20-29%.
• 10-20%.
• 15-25%.
• 20-35%

585. 2nd degree of obesity is associated with increasing of body weight over than normal by:
• 30-40%
• 30-49%
• 25-45%.
• 30-55%.
586. 3rd degree of obesity is associated with increasing of body weight over than normal by:
• 50-99%.
• 45-85%
• 50-75%
• 60-90%.

587. 4th degree of obesity is associated with increasing of body weight over than normal by
• 90% and more
• 85% and more.
• 120% and more.
• 100% and more

588. Deposition of fat in abdomen area in obesity is called:


• Upper type.
• Middle type.
• Lower type.
• Simmetric type

589. Deposition of fat in area of face and neck in obesity is called:


• Upper type.
• Middle type
• Lower type
• Simmetric type

590. A 38-year-old female presents with intermittent pelvic pain. Physical examination reveals a 3-cm mass in the
area of her right ovary. Histological sections from this ovarian mass reveal a papillary tumor with multiple,
scattered small, round, and laminated calcifications. These structures are most likely the result of
• Apoptosis
• Dystrophic calcification
• Enzymatic necrosis
• Hyperparathyroidism

591. The following pigments are stainable by Prussian blue reaction except:
• Hemosiderin
• Ferritin
• Hematin
• Hemochromatosis

592. Idiopathic calcinosis cutis is an example of:


• Necrotizing inflammation
• Dystrophic calcification
• Metastatic calcification
• Calcified thrombi in veins

593. Risk factors implicated in the etiology of cholesterol gallstones include the following except:
• Family history
• Obesity
• Hemolytic anemia
• Oral contraceptives

594. The following type of gallstones is generally unassociated with changes in the gallbladder wall:
• Cholesterol
• Mixed
• Combined
• Pigment

595. The following type of renal calculi is radiolucent:


• Calcium oxalate
• Struvite
• Uric acid
• Calcium phosphate

596. The following type of renal calculi is infection-induced:


• Calcium oxalate
• Struvite
• Uric acid
• Cystine

597. The color of organs hemosiderin is:


• Black
• Sky-blue
• Yellow
• Brown

598. The pigment hematoidin is most identical with:


• Porphyrin
• Bilirubin
• Hemosiderin
• Hemozoin
599. Lipofuscin, the golden yellow pigment sees in heart muscle
• Hypertrophy
• Atrophy
• Hyperplasia
• Metaplasia

600. What stain is specific for iron:


• Hematoxylin and eosin
• Sudan III
• Prussian blue reaction
• PAS-reaction

601. Hemosiderin has all of the following features, except:


• Hemoglobin-derived
• Golden yellow-to-brown
• Granular or crystalline
• Synthesized by enzyme tyrosinase

602. Hemosiderin in the lung accumulates in:


• Leukocytes
• Lymphocytes
• Macrophages
• Fibroblasts

603. Idiopathic pulmonary hemosiderosis characterizes by all of the following pathologic symptoms, except:
• Productive cough
• Hemophtysis
• Anemia
• Heavy proteinuria

604. The lungs in pulmonary hemosiderosis are:


• Enlarged with turbid fluid exuding from cut surface
• Increased in weight with areas of red-brown consolidation
• Diminished with areas of red-brown consolidation
• Diminished with turbid fluid exuding from cut surface

605. The color of hemosiderin granules stained with Prussian blue reaction is:
• Yellow
• Brown
• Orange-red
• Blue-black
606. Hemosiderosis sees in all pathologic processes, except:
• Inflammation
• Hereditary increased absorption of dietary iron
• Impaired use of iron
• Hemolytic anemia

607. Morphologic changes in genetic hemochromatosis characterizes by all of the following, except:
• Metastatic calcification of many organs
• Deposition of hemosiderin in many organs
• Deposition of hemosiderin in the skin
• Liver cirrhosis

608. Melanin has all of the following features, except:


• Localized endogenous
• Formed of ferritin
• Non-hemoglobin-derived
• Black-brown

609. An increased amount of melanin in melanocytes and within basal keratinocytes is also known as:
• Vacuolization
• Vitiligo
• Hyperpigmentation (melanosis)
• Albinism

610. Lipofuscin has all of the following features, except:


• Aging pigment
• Most often seen in kidney
• Yellow-brown
• Noninjurious to the cells or their function

611. Lipofuscin pigment granules in cells result from:


• Hemosiderosis
• Accumulation of protein in cytoplasm
• Accumulation of lipids in cytoplasm
• Intracellular lipid peroxidation

612. Lipofuscin granules in cells see in:


• Necrosis
• Denervation atrophy
• Brown atrophy
• Atrophy from pressure

613. Heart and liver of a patient with cancer cachexia macroscopically sees as:
• Diminished and brown
• Diminished and brown
• Enlarged and brown
• Enlarged and yellow

614. Lipofuscin in the liver may founds in:


• Unchanged cells
• Cells with ballooning degeneration
• Cells with hyaline droplets
• Cells with regressive changes

615. Bilirubin has all of the following features, except:


• The end product of hem degradation
• Derived from breakdown erythrocytes
• Brilliant-yellow
• Stained in blue-black color with Prussian-blue reaction

616. Jaundice occurs in all of the following pathologic processes, except:


• Increased hepatocellular excretion
• Excessive production of bilirubin
• Reduced hepatocyte uptake
• Impaired conjugation of bilirubin

617. Biliary ducts obstruction by gallstones may lead to:


• Liver hemosiderosis
• Liver steatosis
• Ballooning degeneration of hepatocytes
• Cholestasis and jaundice

618. Complications or well-established associations of gallstones include all of the following, except:
• Biliary obstruction
• Brown atrophy of the liver
• Pancreatitis
• Intestinal obstruction
619. Which of the following sites is an example of metastatic calcification?
• The kidney in nephrocalcinosis
• The mitral valve in mitral stenosis of rheumatic organ
• The left anterior ascending coronary artery affected by atheromatous plaques
• The lung involved by metastatic carcinoma

620. Point out the posttuberculosis lung lesion:


• Granuloma
• Cavity
• Fibrocalcific scar
• Caseation in lymph node

621. Dystrophic calcification encounters in all of the following areas, except:


• Coagulative necrosis
• Intracellular fat accumulation
• Caseous necrosis
• Liquefactive necrosis

622. Initiation of intracellular calcification occurs in:


• Cytoplasm
• Lysosome
• Nucleus
• Mitochondria

623. The causes of metastatic calcification are all of the following, except:
• Diabetes mellitus
• Increased secretion of parathyroid hormone
• Destruction of bone tissue
• Vitamin D-related disorders

624. Metastatic calcification may occur in all of the following organs, except:
• Stomach (gastric mucosa)
• Kidneys
• Lungs
• Liver

625. Deficiency of vitamin D tends to cause:


• Hypercalcemia
• Hypocalcemia
• Hyperpigmentation
• Hypopigmentation
626. Deficiency of vitamin D in the adults leads to:
• Osteosclerosis
• Osteomyelitis
• Osteomalati
• Osteonecrosis

627. Hemoglobin derivative pigment which occurs in normal conditions:


• Hemosiderin
• Hematoidin
• Hemin
• Hemomelanin

628. Hemoglobin derivative pigment which occurs in normal conditions


• Ferritin
• Hematoidin.
• Hemin
• Hemomelanin.

629. Hemoglobin derivative pigment which occurs in normal conditions :


• Bilirubin
• Hematoidin
• Hemin.
• Hemomelanin

630. Hemoglobin derivative pigment which occurs only in pathologic conditions:


• Hemosiderin
• Hematoidin
• Ferritin
• Bilirubin

631. Hemoglobin derivative pigment which occurs only in pathologic conditions:


• Hemosiderin
• Porphyrin
• Ferritin.
• Bilirubin.

632. Hemoglobin derivative pigment which occurs only in pathologic conditions


• Hemosiderin
• Hematin
• Ferritin
• Bilirubin

633. Iron-containing pigment is:


• Bilirubin
• Hematoidin
• Hemosiderin
• Porphyrin

634. Iron-containing pigment is


• Bilirubin
• Hematoidin
• Ferritin
• Porphyrin

635. Iron-containing pigment is:


• Bilirubin.
• Hematoidin.
• Hematin
• Porphyrin

636. The stain used to identify iron-containing pigments is:


• Prussian-blue reaction.
• PAS reaction
• Congo-red stain
• Sudan III stain

637. In hemosiderosis organs become:


• Enlarged, brown, hard.
• Small, red, soft
• Enlarged, yellow, hard.
• Enlarged, brown, soft.

638. Cause of generalized hemosiderosis is:


• Intracerebral hemorrhage.
• Intoxications.
• Brown induration of lungs
• Pulmonary hemorrhagic infarct
639. Cause of generalized hemosiderosis is
• Intracerebral hemorrhage
• Heterohemotransfusions
• Brown induration of lungs
• Pulmonary hemorrhagic infarct

640. Cause of generalized hemosiderosis is ?


• Typhoid fever
• Yellow fever
• Malaria
• Diphteria

641. Cause of localized hemosiderosis is:


• Intoxications
• Heterohemotransfusions.
• Malaria
• Brown induration of lungs

642. Cause of localized hemosiderosis is :


• Intoxications
• Heterohemotransfusions
• Malaria
• Intracerebral hemorrhage

643. Cause of localized hemosiderosis is:


• Intoxications
• Heterohemotransfusions
• Malaria
• Pulmonary hemorrhagic infarct.

644. Accumulation of which pigment may leads to decrease of blood pressure?


• Ferritin
• Hemosiderin
• Bilirubin
• Hemomelanin

645. Accumulation of which pigment may leads to jaundice?


• Ferritin.
• Hemosiderin
• Bilirubin.
• Hemomelanin
646. Cause of prehepatic jaundice is:
• Hepatitis.
• Liver cirrhosis
• Obstruction of bile ducts by stones
• Hemolytic disease of newborns

647. Cause of prehepatic jaundice is :


• Hepatitis
• Liver cirrhosis
• Obstruction of bile ducts by stones.
• Heterohemotransfusions

648. Cause of hepatocellular jaundice is:


• Obstruction of bile ducts by stones
• Compression of bile ducts by tumors
• Heterohemotransfusions
• Hepatitis

649. Cause of hepatocellular jaundice is :


• Obstruction of bile ducts by stones
• Compression of bile ducts by tumors
• Heterohemotransfusions.
• Liver cirrhosis

650. Cause of posthepatic jaundice is:


• Obstruction of bile ducts by stones
• Hepatitis
• Heterohemotransfusions.
• Liver cirrhosis

651. Cause of posthepatic jaundice is :


• Compression of bile ducts by tumors
• Hepatitis.
• Heterohemotransfusions.
• Liver cirrhosis

652. Which pigment is “tear and wear”:


• Bilirubin
• Hematoidin
• Hematin.
• Lipofuscin

653. Which pigment is tyrosin-derived:


• Bilirubin
• Hematoidin
• Hematin.
• Melanin

654. Generalized hyperpigmentation of melanin occurs in:


• Addison’s disease
• Melanosis coli
• Lentigo
• Nevus.

655. Focal hyperpigmentation of melanin occurs in:


• Addison’s disease
• Cachexia.
• Avitaminosis
• Nevus.

656. General hypopigmentation of melanin occurs in


• Leukoderma.
• Albinism
• Vitiligo
• Cachexia

657. Focal hypopigmentation of melanin occurs in:


• Albinism
• Nevus
• Vitiligo
• Cachexia

658. Distrophic calcification occurs in:


• Hyperparathyroidism
• Hypervitaminosis D
• Hyperthyroidism
• Necrosis.
659. Metastatic calcification occurs in:
• Hyperparathyroidism.
• Damaged heart valves
• Atheromas.
• Necrosis.

660. Gross appearance of calcificate is:


• Hard, greywish-white.
• Soft, yellow
• Hard, greywish-red
• Soft, greywish-white.

661. Area of calcification in histological examination with hematoxylin and eosin staining is:
• Red
• Black.
• Brown
• Blue.

662. Сongenital absence of thymus is called:


• Aplasia
• Hypoplasia
• Dysplasia
• Atrophy

663. Congenital incomplete development of thymus is called:


• Aplasia
• Hypoplasia
• Dysplasia
• Atrophy

664. Aquired decrease in weight of thymus is called:


• Aplasia
• Hypoplasia
• Dysplasia
• Atrophy

665. Abnormal development of thymus is called:


• Aplasia
• Hypoplasia
• Dysplasia
• Atrophy

666. Morphologically immediate type of hypersensitivity occurs as:


• Fibrinoid necrosis
• Lympho-hystiocytic infiltration
• Macrophageal infiltration
• Granulomatosis

667. Morphologically immediate type of hypersensitivity occurs as


• Lympho-hystiocytic infiltration
• Macrophageal infiltration
• Granulomatosis
• Plasmatic saturation

668. Morphologically immediate type of hypersensitivity occurs as


• Lympho-hystiocytic infiltration
• Mucoid changes
• Macrophageal infiltration
• Granulomatosis

669. Morphologically immediate type of hypersensitivity occurs as:


• Lympho-hystiocytic infiltration
• Fibrinoid changes
• Macrophageal infiltration
• Granulomatosis

670. Morphologically immediate type of hypersensitivity occurs as:


• Lympho-hystiocytic infiltration
• Fibrinous-hemorrhagic exudate
• Macrophageal infiltration
• Granulomatosis

671. Morphological signs of immediate type of hypersensitivity are all the following, except:
• Lympho-hystiocytic infiltration
• Mucoid and Fibrinoid changes
• Plasmatic saturation
• Fibrinoid necrosis
672. Morphological signs of immediate type of hypersensitivity are all the following, except:
• Mucoid and Fibrinoid changes
• Plasmatic saturation
• Granulomatosis
• Fibrinoid necrosis

673. Morphological signs of immediate type of hypersensitivity are all the following, except:
• Mucoid and Fibrinoid changes
• Plasmatic saturation
• Fibrinoid necrosis
• Macrophage infiltration

674. Morphologically delayed type of hypersensitivity occurs as:


• Lympho-hystiocytic infiltration
• Mucoid and Fibrinoid changes
• Plasmatic saturation
• Fibrinoid necrosis

675. Morphologically delayed type of hypersensitivity occurs as:


• Mucoid and Fibrinoid changes
• Plasmatic saturation
• Granulomatosis
• Fibrinoid necrosis

676. Morphologically delayed type of hypersensitivity occurs as:


• Mucoid and Fibrinoid changes
• Plasmatic saturation
• Fibrinoid necrosis
• Macrophageal infiltration

677. Morphologically delayed type of hypersensitivity occurs as:


• Mucoid and Fibrinoid changes
• Plasmatic saturation
• Fibrinoid necrosis
• Cytoplasmic bridges between lymphocytes and macrophages

678. Morphological signs of delayed type of hypersensitivity are all the following, except:
• Fibrinoid necrosis
• Lympho-hystiocytic infiltration
• Macrophageal infiltration
• Granulomatosis

679. Morphological signs of delayed type of hypersensitivity are all the following, except:
• Lympho-hystiocytic infiltration
• Macrophageal infiltration
• Granulomatosis
• Plasmatic saturation

680. Morphological signs of delayed type of hypersensitivity are all the following, except:
• Lympho-hystiocytic infiltration
• Mucoid changes
• Macrophageal infiltration
• 4.Granulomatosis

681. Morphological signs of delayed type of hypersensitivity are all the following, except:
• Lympho-hystiocytic infiltration
• Fibrinous-hemorrhagic exudate
• Macrophageal infiltration
• Granulomatosis

682. Morphological signs of transplant rejection are all the following, except:
• Lympho-hystiocytic infiltration
• Edema of transplant
• Macrophageal infiltration
• 4.Granulomatosis

683. Morphological signs of transplant rejection are all the following, except ?
• Lympho-hystiocytic infiltration
• Edema of transplant
• Macrophageal infiltration
• Cytoplasmic bridges between lymphocytes and macrophages

684. Organ specific immune disease is:


• Hashimoto thyroiditis
• Rheumatoid arthritis
• Systemic Lupus Erythematosus
• Scleroderma
685. Organ specific immune disease is :
• Rheumatoid arthritis
• Systemic Lupus Erythematosus
• Scleroderma
• Encephalomyelitis

686. Non-organ specific immune disease is:


• Hashimoto thyroiditis
• Rheumatoid arthritis
• Encephalomyelitis
• Polyneuritis

687. Non-organ specific immune disease is :


• Hashimoto thyroiditis
• Encephalomyelitis
• Systemic Lupus Erythematosus.
• Polyneuritis.

688. Variant of primary immunodeficiency syndrome occurs:


• In leukemia
• Under radial therapy
• In aplasia of thymus
• In infections

689. Variant of primary immunodeficiency syndrome occurs :


• In leukemia
• Under radial therapy.
• In sarcoidosis
• In hypoplasia of thymus.
690. Variant of secondary immunodeficiency syndrome is:
• Lui-Bar syndrome
• Neseloff syndrome
• DiGeorge syndrome
• AIDS

691. Complication of immunodeficiency syndromes is:


• Arterial hypertension
• Myocardial infarction
• Typhoid fever
• Sepsis
692. Complication of immunodeficiency syndromes is :
• Recurrence of bronchial asthma
• Recurrence of chronic hepatitis
• Recurrence of chronic cholecystitis
• Recurrence of tuberculosis

693. Complication of immunodeficiency syndromes is:


• Arterial hypertension
• Myocardial infarction
• Typhoid fever
• Purulent pneumonia

694. A 22-year-old woman nursing her newborn develops a tender erythematous area around the nipple of her
left breast. A thick, yellow fluid is observed to drain from an open fissure. Examination of this breast fluid
under the light microscope will most likely reveal an abundance of which of the following inflammatory cells?
• B lymphocytes
• Eosinophils
• Mast cells
• Neutrophils

695. A 63-year-old man becomes febrile and begins expectorating large amounts of mucopurulent sputum.
Sputum cultures are positive for Gram-positive diplococci. Which of the following mediators of inflammation
provides potent chemotactic factors for the directed migration of inflammatory cells into the alveolar air
spaces of this patient?
• Bradykinin
• Histamine
• Myeloperoxidase
• N-formylated peptides

696. A 59-year-old man suffers a massive heart attack and expires 24 hours later due to ventricular arrhythmia.
Histologic examination of the affected heart muscle at autopsy would show an abundance of which of the
following inflammatory cells?
• Fibroblasts
• Lymphocytes
• Macrophages
• Neutrophils

697. A 5-year-old boy punctures his thumb with a rusty nail. Four hours later, the thumb appears red and swollen.
Initial swelling of the boy’s thumb is primarily due to which of the following mechanisms?
• Decreased intravascular hydrostatic pressure
• Decreased intravascular oncotic pressure
• Increased capillary permeability
• Increased intravascular oncotic pressure

698. An 80-year-old woman presents with a 4-hour history of fever, shaking chills, and disorientation. Her blood
pressure is 80/40 mm Hg. Physical examination shows diffuse purpura on her upper arms and chest. Blood
cultures are positive for Gram negative organisms. Which of the following cytokines is primarily involved in
the pathogenesis of direct vascular injury in this patient with septic shock?
• Interferon-g
• Interleukin-1
• Platelet-derived growth factor
• Tumor necrosis factor-a

699. A 24-year-old intravenous drug abuser develops a 2-day history of severe headache and fever. His
temperature is 38.7°C (103°F). Blood cultures are positive for Gram-positive cocci The patient is given
intravenous antibiotics, but he deteriorates rapidly and dies. A cross section of the brain at autopsy (shown in
the image) reveals two encapsulated cavities. Which of the following terms best characterizes this pathologic
finding?
• Chronic inflammation
• Fibrinoid necrosis
• Granulomatous inflammation
• Suppurative inflammation

700. A 36-year-old woman with pneumococcal pneumonia develops a right pleural effusion. The pleural fluid
displays a high specific gravity and contains large numbers of polymorphonuclear (PMN) leukocytes. Which of
the following best characterizes this pleural effusion?
• Fibrinous exudate
• Lymphedema
• Purulent exudate
• Serosanguineous exudate

701. A 33-year-old man presents with a 5-week history of calf pain and swelling and low-grade fever. Serum levels
of creatine kinase are elevated. A muscle biopsy reveals numerous eosinophils. What is the most likely
etiology of this patient’s myalgia?
• Autoimmune disease
• Bacterial infection
• Muscular dystrophy
• Parasitic infection

702. A 10-year-old boy with a history of recurrent bacterial infections presents with fever and a productive cough.
Biochemical analysis of his neutrophils demonstrates that he has an impaired ability to generate reactive
oxygen species. This patient most likely has inherited mutations in the gene that encodes which of the
following proteins?
• Catalase
• Cytochrome P450
• Myeloperoxidase
• NADPH oxidase

703. A 25-year-old woman presents with a history of recurrent shortness of breath and severe wheezing.
Laboratory studies demonstrate that she has a deficiency of C1 inhibitor, an esterase inhibitor that regulates
the activation of the classical complement pathway. What is the diagnosis?
• Chronic granulomatous disease
• Hereditary angioedema
• Myeloperoxidase deficiency
• Selective IgA deficiency

704. A 40-year-old man complains of a 2-week history of increasing abdominal pain and yellow discoloration of
his sclera. Physical examination reveals right upper quadrant pain. Laboratory studies show elevated serum
levels of alkaline phosphatase (520 U/dL) and bilirubin (3.0 mg/dL). A liver biopsy shows portal fibrosis, with
scattered foreign bodies consistent with schistosome eggs. Which of the following inflammatory cells is most
likely to predominate in the portal tracts in the liver of this patient?
• Basophils
• Eosinophils
• Macrophages
• Monocytes

705. A 41-year-old woman complains of excessive menstrual bleeding and pelvic pain of 4 months. She uses an
intrauterine device for contraception. Endometrial biopsy (shown in the image) reveals an excess of plasma
cells (arrows) and macrophages within the stroma. The presence of these cells and scattered lymphoid
follicles within the endometrial stroma is evidence of which of the following conditions?
• Acute inflammation
• Chronic inflammation
• Granulation tissue
• Granulomatous inflammation

706. A 62-year-old woman undergoing chemotherapy for breast cancer presents with a 3-day history of fever and
chest pain. Cardiac catheterization reveals a markedly reduced ejection fraction with normal coronary blood
flow. A myocardial biopsy is obtained, and a PCR test for coxsackievirus is positive. Histologic examination of
this patient’s myocardium will most likely reveal an abundance of which of the following inflammatory cells?
• Eosinophils
• Lymphocytes
• Macrophages
• Mast cells
707. A 58-year-old woman with long-standing diabetes and hypertension develops end-stage renal disease and
dies in uremia. A shaggy fi brin-rich exudate is noted on the visceral pericardium at autopsy (shown in the
image). Which of the following best explains the pathogenesis of this fibrinous exudate?
• Antibody binding and complement activation
• Chronic passive congestion
• Injury and increased vascular permeability
• Margination of segmented neutrophils

708. A 68-year-old man presents with fever, shaking chills, and shortness of breath. Physical examination shows
rales and decreased breath sounds over both lung fields. The patient exhibits grunting respirations, 30 to 35
breaths per minute, with flaring of the nares. The sputum is rusty yellow and displays numerous
polymorphonuclear leukocytes. Which of the following mediators of inflammation is chiefly responsible for
the development of fever in this patient?
• Arachidonic acid
• Interleukin-1
• Leukotriene B4
• Prostacyclin (PGI2)

709. A 35-year-old woman presents with a 5-day history of a painful sore on her back. Physical examination
reveals a 1- cm abscess over her left shoulder. Biopsy of the lesion shows vasodilation and leukocyte
margination (shown in the image). What glycoprotein mediates initial tethering of segmented neutrophils to
endothelial cells in this skin lesion?
• Cadherin
• Entactin
• Integrin
• Selectin

710. A 14-year-old boy receives a laceration on his forehead during an ice hockey game. When he is first attended
to by the medic, there is blanching of the skin around the wound. Which of the following mechanisms
accounts for this transient reaction to neurogenic and chemical stimuli at the site of injury?
• Constriction of postcapillary venules
• Constriction of precapillary arterioles
• Dilation of postcapillary venules
• Dilation of precapillary arterioles

711. An 8-year-old girl with asthma presents with respiratory distress. She has a history of allergies and upper
respiratory tract infections. She also has history of wheezes associated with exercise. Which of the following
mediators of inflammation is the most powerful stimulator of bronchoconstriction and vasoconstriction in
this patient?
• Bradykinin
• Complement proteins
• Interleukin-1
• Leukotrienes
712. A 75-year-old woman complains of recent onset of chest pain, fever, and productive cough with rust-colored
sputum. A chest X-ray reveals an infiltrate in the right middle lobe. Sputum cultures are positive for
Streptococcus pneumoniae. Phagocytic cells in this patient’s affected lung tissue generate bacteriocidal
hypochlorous acid using

which of the following enzymes?

• Catalase
• Cyclooxygenase
• Myeloperoxidase
• NADPH oxidase

713. A 28-year-old woman cuts her hand while dicing vegetables in the kitchen. The wound is cleaned and
sutured. Five days later, the site of injury contains an abundance of chronic inflammatory cells that actively
secrete interleukin-1, tumor necrosis factor-, interferon-, numerous arachidonic acid derivatives, and various
enzymes. Name these cells
• B lymphocytes
• Macrophages
• Plasma cells
• Smooth muscle cells

714. A 68-year-old man with prostate cancer and bone metastases presents with shaking chills and fever. The
peripheral WBC count is 1,000/L (normal = 4,000 to 11,000/L). Which of the following terms best describes
this hematologic finding?

• Leukocytosis
• Leukopenia
• Neutrophilia
• Pancytopenia

715. A 25-year-old machinist is injured by a metal sliver in his left hand. Over the next few days, the wounded
area becomes reddened, tender, swollen, and feels warm to the touch. Redness at the site of injury in this
patient is caused primarily by which of the following mechanisms?
• Hemorrhage
• Hemostasis
• Neutrophil margination
• Vasodilation

716. A 37-year-old man with AIDS is admitted to the hospital with a 3-week history of chest pain and shortness of
breath. An X-ray film of the chest shows bilateral nodularities of the lungs. A CT-guided lung biopsy is shown
in the image. The multinucleated cell in the center of this field is most likely derived from which of the
following inflammatory cells?
• Basophils
• Capillary endothelial cells
• Macrophages
• Myofi broblasts

717. A 10-year-old girl presents with a 2-week history of puffiness around her eyes and swelling of the legs and
ankles. Laboratory studies show hypoalbuminemia and proteinuria. The urinary sediment contains no
inflammatory cells or red blood cells. Which of the following terms describes this patient’s peripheral edema?
• Effusion
• Exudate
• Hydropic change
• Transudate

718. A 25-year-old woman develops a sore, red, hot, swollen left knee. She has no history of trauma and no
familial history of joint disease. Fluid aspirated from the joint space shows an abundance of segmented
neutrophils. Transendothelial migration of acute inflammatory cells into this patient’s joint space was
mediated primarily by which of the following families of proteins?
• Entactins
• Fibrillins
• Fibronectins
• Integrins

719. A 50-year-old woman is discovered to have metastatic breast cancer. One week after receiving her first dose
of chemotherapy, she develops bacterial pneumonia. Which of the following best explains this patient’s
susceptibility to bacterial infection?
• Depletion of serum complement
• Impaired neutrophil respiratory burst
• Inhibition of clotting factor activation
• Neutropenia

720. A 53-year-old man develops weakness, malaise, cough with bloody sputum, and night sweats. A chest X-ray
reveals numerous apical densities bilaterally. Exposure to Mycobacterium tuberculosis was documented 20
years ago, and M. tuberculosis I identified in the sputum. The patient subsequently dies of respiratory
insufficiency. The lungs are examined at autopsy (shown in the image). Which of the following best
characterizes the histopathologic features of this pulmonary lesion?
• Acute suppurative inflammation
• Chronic inflammation
• Fat necrosis
• Granulomatous inflammation
721. A 59-year-old man experiences acute chest pain and is rushed to the emergency room. Laboratory studies
and ECG demonstrate an acute myocardial infarction; however, coronary artery angiography performed 2
hours later does not show evidence of thrombosis. Intravascular thrombolysis that occurred in this patient
was mediated by plasminogen activators that were released by which of the following cells?
• Cardiac myocytes
• Endothelial cells
• Macrophages
• Segmented neutrophils

722. A 68-year-old coal miner with a history of smoking and emphysema develops severe air-flow obstruction and
expires. Autopsy reveals a “black lung,” with coal-dust nodules scattered throughout the parenchyma and a
centra area of dense fibrosis. The coal dust entrapped within this miner’s lung was sequestered primarily by
which of the following cells?
• Endothelial cells
• Fibroblasts
• Lymphocytes
• Macrophages

723. A 40-year-old man presents with 5 days of productive cough and fever. Pseudomonas aeruginosa is isolated
from a pulmonary abscess. The CBC shows an acute effect characterized by marked leukocytosis (50,000
WBC/L), and the differential count reveals numerous immature cells (band forms). Which of the following
terms best describes these hematologic findings?
• Leukemoid reaction
• Leukopenia
• Myeloid metaplasia
• Myeloproliferative disease

724. A 19-year-old woman presents with 5 days of fever (38°C/101°F) and sore throat. She reports that she has
felt fatigued for the past week and has difficulty swallowing. A physical examination reveals generalized
lymphadenopathy. If this patient has a viral infection, a CBC will most likely show which of the following
hematologic findings?
• Eosinophilia
• Leukopenia
• Lymphocytosis
• Neutrophilia

725. A 40-year-old woman presents with an 8-month history of progressive generalized itching, weight loss,
fatigue, and yellow sclerae. Physical examination reveals mild jaundice. The antimitochondrial antibody test is
positive. A liver biopsy discloses periductal inflammation and bile duct injury (shown in the image). Which of
the following inflammatory cells is the principal mediator of destructive cholangitis in this patient?
• Eosinophils
• B lymphocytes
• T lymphocytes
• Mast cells
726. A 25-year-old woman presents with a 2-week history of febrile illness and chest pain. She has an
erythematous, macular facial rash and tender joints, particularly in her left wrist and elbow. A CBC shows
mild anemia and thrombocytopenia. Corticosteroids are prescribed for the patient. This medication induces
the synthesis of an inhibitor of which of the following enzymes in inflammatory cells?
• Lipoxygenase
• Myeloperoxidase
• Phospholipase A2
• Phospholipase C

727. A 22-year-old man develops marked right lower quadrant abdominal pain over the past day. On physical
examination there is rebound tenderness on palpation over the right lower quadrant. Laparoscopic surgery is
performed, and the appendix is swollen, erythematous, and partly covered by a yellowish exudate. It is
removed, anda microscopic section shows infiltration with numerous neutrophils. The pain experienced by
this patient is predominantly the result of which of the following two chemical mediators?
• Complement C3b and IgG
• Interleukin-1 and tumor necrosis factor
• Histamine and serotonin
• Prostaglandin and bradykinin
728. A 40-year-old woman had laparoscopic surgery 3 months ago. Now she has a small 0.5 cm nodule beneath
the skin at the incision site that was sutured. Which of the following cell types is most likely to be most
characteristic of the inflammatory response in this situation?
• Mast cell
• Eosinophil
• Giant cell
• Neutrophil

729. A 39-year-old man incurs a burn injury to his hands and arms while working on a propane furnace. Over the
next 3 weeks, the burned skin heals without the need for skin grafting. Which of the following is the most
critical factor in determining whether the skin in the region of the burn will regenerate?
• Good cardiac output with tissue perfusion
• Persistence of skin appendages
• Maintenance of underlying connective tissue
• Diminished edema and erythema

730. A 58-year-old woman has had a cough with fever for 3 days. A chest radiograph reveals infiltrates in the right
lower lobe. A sputum culture grows Streptococcus pneumoniae. The clearance of these organisms from the
lung parenchyma would be most effectively accomplished through generation of which of the following
substances by the major inflammatory cell type responding to this infection?
• Platelet activating factor
• Prostaglandin E2
• Kallikrein
• Hydrogen peroxide
731. A clinical study is performed of patients with pharyngeal infections. The most typical clinical course averages
3 days from the time of onset until the patient sees the physician. Most of these patients experience fever
and chills. On physical examination, the most common findings include swelling, erythema, and pharyngeal
purulent exudate. Which of the following types of inflammation did these patients most likely have?
• Granulomatous
• Acute
• Gangrenous
• Resolving

732. A 56-year-old man has had increasing dyspnea for 6 years. He has no cough or fever. He had chronic
exposure to inhalation of silica dust for many years in his job. A chest x-ray now shows increased interstitial
markings an parenchymal 1 to 3 cm solid nodules. His pulmonary problems are most likely to be mediated
through which of the following inflammatory processes?
• Neutrophilic infiltrates producing leukotrienes
• Foreign body giant cell formation
• Plasma cell synthesis of immunoglobulins
• Macrophage elaboration of cytokines

733. A 22-year-old woman has premature labor with premature rupture of fetal membranes at 20 weeks
gestation. Prior to that time, the pregnancy had been proceeding normally. A stillbirth occurs two days later.
Microscopic examination of the normal-sized placenta reveals numerous neutrophils in the amnion and
chorion, but no villitis. The premature labor was most likely mediated by the effects from release of which of
the following substances?
• Immunoglobulin
• Prostaglandin
• Complement
• Fibrinogen

734. After two weeks in the hospital following a fall in which she incurred a fracture of her left femoral
trochanter, a 76- year-old woman now has a left leg that is swollen, particularly her lower leg below the knee.
She experiences pain on movement of this leg, and there is tenderness to palpation. Which of the following
complications is most likely to occur next after these events?
• Gangrenous necrosis of the foot
• Hematoma of the thigh
• Disseminated intravascular coagulation
• Pulmonary thromboembolism

735. A 43-year-old woman has had a chronic cough with fever and weight loss for the past month. A chest
radiograph reveals multiple nodules from 1 to 4 cm in size, some of which demonstrate cavitation in the
upper lobes. A sputum sample reveals the presence of acid fast bacilli. Which of the following cells is the most
important in the development her lung lesions?
• Macrophage
• Fibroblast
• Neutrophil
• Mast cell

736. A 20-year-old man has experienced painful urination for 4 days. A urethritis is suspected, and Neisseria
gonorrheae is cultured. Numerous neutrophils are present in a smear of the exudate from the penile urethra.
These neutrophils undergo diapedesis to reach the organisms. Release of which of the following chemical
mediators is most likely to drive neutrophil exudation?
• Histamine
• Prostaglandin
• Hageman factor
• Complement

737. An episode of marked chest pain lasting 4 hours brings a 51-year-old man to the emergency room. He is
found to have an elevated serum creatine kinase. An angiogram reveals a complete blockage of the left
circumflex artery 2 cm from its origin. Which of the following substances would you most expect to be
elaborated around the region of tissue damage in the next 3 days as an initial response to promote healing?
• Histamine
• Immunogloblulin G
• Complement component C3b
• Vascular endothelial growth factor

738. A 94-year-old woman has developed a fever and cough over the past 2 days. Staphylococcus aureus is
cultured from her sputum. She receives a course of antibiotic therapy. Two weeks later she no longer has a
productive cough, but she still has a fever. A chest radiograph reveals a 3 cm rounded density in the right
lower lobe whose liquefied contents form a central air-fluid level. There are no surrounding infiltrates. Which
of the following is the best description for this outcome of her pneumonia?
• Hypertrophic scar
• Abscess formation
• Regeneration
• Bronchogenic carcinoma

739. A 36-year-old woman has been taking acetylsalicylic acid (aspirin) for arthritis for the past 4 years. Her joint
pain is temporarily reduced via this therapy. However, she now has occult blood identified in her stool. Which
of the following substances is most likely inhibited by aspirin to cause this complication?
• Leukotriene B4
• Interleukin-1
• Thromboxane
• Bradykinin

740. A small sliver of wood becomes embedded in the finger of a 25-year-old man. He does not remove it, and
over then next 3 days the area around the sliver becomes red, swollen, and tender. Neutrophils migrate into
the injured tissue. Expression of which of the following substances on endothelial cells is most instrumental in
promoting this inflammatory reaction?
• Interferon gamma
• Hageman factor
• Lysozyme
• E-selectin

741. An inflammatory process that has continued for 3 months includes the transformation of tissue
macrophages to epithelioid cells. There are also lymphocytes present. Over time, fibroblasts lay down
collagen as the focus of inflammation heals. These events are most likely to occur as an inflammatory
response to which of the following infectious agents?
• Mycobacterium tuberculosis
• Pseudomonas aeruginosa
• Cytomegalovirus
• Giardia lamblia

742. A 37-year-old man has had nausea and vomiting for 5 weeks. He experienced an episode of hematemesis
yesterday. On physical examination he has no abnormal findings. Upper GI endoscopy is performed, and
there is a 1.5 c diameter lesion in the gastric antrum with loss of the epithelial surface. These findings are
most typical for which of the following pathologic processes?
• Abscess
• Serositis
• Granuloma
• Ulcer

743. A 17-year-old truck driver is involved in a collision. He incurs blunt force abdominal trauma. In response to
this injury, cells in tissues of the abdomen are stimulated to enter the G1 phase of the cell cycle from the G0
phase. Which of the following cell types is most likely to remain in G0 following this injury?
• Smooth muscle
• Endothelium
• Skeletal muscle
• Fibroblast

744. A 19-year-old woman who works indoors spends a day outside gardening. She does not wear a hat or
sunscreen. That evening her partner remarks that her face appears red. Which of the following dermal
changes most likely accounts for her red appearance?
• Neutrophil aggregation
• Hemorrhage
• Edema
• Vasodilation

745. A 45-year-old woman has had a chronic, non-productive cough for 3 months, along with intermittent fever.
She has a chest radiograph that reveals multiple small parenchymal nodules along with hilar and cervical
lymphadenopathy. A cervical lymph node biopsy is performed. Microscopic examination of the biopsy shows
noncaseating granulomatous inflammation. Cultures for bacterial, fungal, and mycobacterial organisms are
negative. Which of the following chemical mediators is most important in the development of her
inflammatory response?
• Interferon gamma
• Bradykinin
• Complement C5a
• Histamine

746. A 55-year-old man has a history of hypercholesterolemia with coronary artery disease and suffered a
myocardial infarction 2 years ago. He now presents with crushing substernal chest pain. Which of the
following laboratory tests is most useful in diagnosing the cause of his chest pain?
• Increased white blood cell count
• Elevated sedimentation rate
• Decreased serum complement
• Increased serum troponin

747. A 15-year-old girl has had episodes of sneezing with watery eyes and runny nose for the past 2 weeks. On
physical examination she has red, swollen nasal mucosal surfaces. She has had similar episodes each Spring
and Summer when the amount of pollen in the air is high. Her symptoms are most likely to be mediated by
the release of which of the following chemical mediators?
• Complement C3b
• Platelet activating factor (PAF)
• Tumor necrosis factor (TNF)
• Histamine

748. A 45-year-old man has been working hard all day long carrying loads of bricks to build a wall. He takes a
nonsteroidal anti-inflammatory drug (ibuprofen). Which of the following processes is this drug most likely to
diminish in his arms?
• Thrombosis
• Pain
• Necrosis
• Fibrinolysis

749. Within minutes following a bee sting, a 37-year-old man develops marked respiratory stridor with dyspnea
and wheezing. He also develops swelling and erythema seen in his arms and legs. An injection of epinephrine
helps to reverse these events and he recovers within minutes. Which of the following chemical mediators is
most important in the pathogenesis of this man's condition?
• Bradykinin
• Complement C5a
• Nitric oxide
• Histamine
750. A 72-year-old woman did not get a 'flu' shot in the fall as recommended for older persons. In the wintertime,
she became ill, as many people in her community did, with a respiratory illness that lasted for 3 weeks.
During this illness, she had a fever with a non-productive cough, mild chest pain, myalgias, and headache.
What was her chest radiograph most likely to have shown during this illness?
• Hilar mass
• Interstitial infiltrates
• Hilar lymphadenopathy
• Lobar consolidation

751. In an experiment, Enterobacter cloacae organisms are added to a solution containing leukocytes and blood
plasma. Engulfment and phagocytosis of the microbes is observed to occur. Next a substance is added which
enhances engulfment, and more bacteria are destroyed. Which of the following substances in the plasma is
most likely to produce this effect?
• Complement C3b
• Glutathione peroxidase
• Immunoglobulin M
• P-selectin

752. A 43-year-old woman has had nausea with vomiting persisting for the past 5 weeks. On physical examination
there are no abnormal findings. She undergoes an upper GI endoscopy and gastric biopsies are obtained. The
microscopic appearance of these biopsies shows mucosal infiltration by lymphocytes, macrophages, and
plasma cells. Which of the following most likely caused her findings?
• Staphylococcus aureus septicemia
• Ingestion of chili peppers
• Diabetes mellitus
• Infection with Helicobacter pylori

753. In an experiment, lymphatic channels are observed in normal soft tissue preparations. Staphylococcus
aureus organisms are innoculated into the tissues and the immunologic response observed over the next 24
hours. Which of the following functions is most likely to be served by these lymphatics to produce a specific
immune response to these organisms?
• Carry lymphocytes to peripheral tissue sites
• Remove extravascular tissue fluid
• Transport antigen presenting cells
• Serve as a route for dissemination of infection

754. In an experiment, surgical wound sites are observed following suturing. An ingrowth of new capillaries is
observed to occur within the first week. A substance elaborated by macrophages is found at the wound site
to stimulate this capillary proliferation. Which of the following substances is most likely to have this function?
• Platelet-derived growth factor
• Phospholipase C-gamma
• Fibronectin
• Fibroblast growth factor
755. A 55-year-old man with a history of ischemic heart disease has worsening congestive heart failure. He has
noted increasing dyspnea and orthopnea for the past 2 months. On physical examination there is dullness to
percussion at lung bases. A chest x-ray shows bilateral pleural effusions. A left thoracentesis is performed,
and 500 mL of fluid is obtained. Which of the following characteristics of this fluid would most likely indicate
that it is a transudate?
• Cloudy appearance
• High protein content
• <3 lymphocytes/microliter
• Presence of fibrin

756. In a clinical study, patients undergoing laparoscopic cholecystectomy are followed to document the post-
surgical wound healing process. The small incisions are closed with sutures. Over the 4 weeks following
surgery, the wounds are observed to regain tensile strength and there is re-epithelialization. Of the following
substances, which is most likely found to function intracellularly in cells involved in this wound healing
process?
• Fibronectin
• Laminin
• Tyrosine kinase
• Hyaluronic acid

757. A 31-year-old woman has a laparotomy performed for removal of an ovarian cyst. She recovers uneventfully,
with no complications. At the time of surgery, a 12 cm long midline abdominal incision was made. The tensile
strength in the surgical scar will increase so her normal activities can be resumed. Most of the tensile
strength will likely be achieved in which of the following time periods?
• One week
• One month
• Three months
• Six months

758. A 9-year-old girl sustains a small 0.5 cm long laceration to her right index finger while playing 'Queen of
Swords' with a letter opener. Which of the following substances, on contact with injured vascular basement
membrane, activates both the coagulation sequence and the kinin system as an initial response to this injury?
• Thromboxane
• Plasmin
• Platelet activating factor
• Hageman factor

759. A 65-year-old woman has had a fever for the past day. On physical examination her temperature is 39°C and
blood pressure 90/50 mm Hg with heart rate of 106/minute. Laboratory studies show a WBC count of
12,510/microliter and WBC differential count of 78 segs, 8 bands, 11 lymphs, and 3 monos. A blood culture is
positive for Escherichia coli. Her central venous pressure falls markedly. She goes into hypovolemic shock as a
result of the widespread inappropriate release of a chemical mediator derived from macrophages. She
develops multiple organ failure. Which of the following mediators is most likely to produce these findings?
• Nitric oxide
• Bradykinin
• Histamine
• Prostacyclin

760. A 20-year-old woman sustains an injury to her right calf in a mountain biking accident. On physical
examination she has a 5 cm long laceration on the right lateral aspect of her lower leg. This wound is closed
with sutures. Wound healing proceeds over the next week. Which of the following factors will be most likely
to aid and not inhibit woun healing in this patient?
• Commensal bacteria
• Decreased tissue perfusion
• Presence of sutures
• Corticosteroid therapy

761. A 24-year-old primigravida is late in the second trimester of pregnancy. She experiences the sudden onset of
some cramping lower abdominal pain. This is immediately followed by passage of some fluid per vagina along
with a foul-smelling discharge. The fetus is stillborn two days later. Examination of the placenta demonstrates
extensive neutrophilic infiltrates in the chorion and amnion. Which of the following organisms is most likely
to be responsible for these findings?
• Mycobacterium tuberculosis
• Herpes simplex virus
• Escherichia coli
• Treponema pallidum

762. A 19-year-old man incurs a stab wound to the chest. The wound is treated in the emergency room. Two
months later there is a firm, 3 x 2 cm nodular mass with intact overlying epithelium in the region of the
wound. On examination the scar is firm, but not tender, with no erythema. This mass is excised and
microscopically shows fibroblasts with abundant collagen. Which of the following mechanisms has most likely
produced this series of events?
• Keloid formation
• Development of a fibrosarcoma
• Poor wound healing from diabetes mellitus
• Foreign body response from suturing

763. A 45-year-old man has had a fever and dry cough for 3 days, and now has difficulty breathing and a cough
productive of sputum. On physical examination his temperature is 38.5°C. Diffuse rales are auscultated over
lower lung fields. A chest radiograph shows a right pleural effusion. A right thoracentesis is performed. The
fluid obtained has a cloud appearance with a cell count showing 15,500 leukocytes per microliter, 98% of
which are neutrophils. Which of the following terms best describes his pleural process?
• Serous inflammation
• Purulent inflammation
• Fibrinous inflammation
• Chronic inflammation

764. A 52-year-old woman with no major medical problems takes a long airplane flight across the Pacific Ocean.
Upon arrival at Sydney's Kingsford Smith airport following the flight from Los Angeles, she cannot put her
shoes back on There is no pain or tenderness. Which of the following is the most likely explanation for this
phenomenon?
• Activation of Hageman factor has led to bradykinin production.
• A lot of drinks were served in the first class section.
• Femoral vein thrombosis developed
• Venous hydrostatic pressure became increased.

765. In an experiment, a lung tissue preparation is exposed to Mycobacterium tuberculosis organisms. Over the
next week, it is observed that granulomas form in the lung. Within the granuloma are found inflammatory
cells expressing class II MHC antigens. These cells elaborate cytokines that promote fibroblastic production of
collagen within the granulomas. From which of the following peripheral blood leukocytes are these cells
bearing class II antigen most likely to be derived?
• Neutrophils
• B cells
• Monocytes
• NK cells

766. A 56-year-old man has had increasing difficulty breathing for the past week. On physical examination he is
afebrile. Auscultation of his chest reveals diminished breath sounds and dullness to percussion bilaterally.
There is 2+ pitting edema present to the level of his thighs. A chest radiograph reveals bilateral pleural
effusions. Which of the following laboratory test findings is he most likely to have?
• Hypoalbuminemia
• Glucosuria
• Neutrophilia
• Anemia

767. A 72-year-old man presents with a 3-day history of progressively worsening productive cough, fever, chills,
and signs of toxicity. Prominent physical findings include signs of consolidation and rales over the right lung
base. Sputum culture is positive for Streptococcus pneumoniae. An intra-alveolar exudate filling the alveoli of
the involved portion of the lung is present. Which of the following types of inflammatory cells is most likely a
prominent feature of this exudate?
• Basophils
• Eosinophils
• Lymphocytes
• Neutrophils
768. A routine complete blood count performed on a 22-year-old medical student reveals an abnormality in the
differential leukocyte count. She has been complaining of frequent sneezing and “watery” eyes during the
past several weeks and reports that she frequently had such episodes in the spring and summer. Which of the
following cell types is most likely to be increased?
• Basophils
• Eosinophils
• Lymphocytes
• Monocytes

769. A 16-year-old boy presents with a 24-hour history of severe abdominal pain, nausea, vomiting, and low-
grade fever. The pain is initially periumbilical in location but has migrated to the right lower quadrant of the
abdomen, with maximal tenderness elicited at a site one-third of the way between the crest of the ileum and
the umbilicus (McBurney point). The leukocyte count is 14,000/mm3, with 74% segmented neutrophils and
12% bands. Surgery is performed. Which of the following describes the expected findings at the affected site?
• Fistula (abnormal duct or passage) connecting to the abdominal wall
• Granulation tissue (new vessels and young fibroblasts) with a prominent infiltrate of eosinophils
• Granulomatous inflammation with prominent aggregates of epithelioid cells and multinucleated
giant cells
• Prominent areas of edema, congestion, and a purulent reaction with localized areas of abscess
formation

770. A 2-year-old boy presents with recurrent infections involving multiple organ systems. Extensive investigation
results in a diagnosis of chronic granulomatous disease of childhood. Which of the following most closely
characterizes the abnormality in this patient’s phagocytic cells?
• Decreased killing of microorganisms because of enhanced production of hydrogen peroxide
• Deficiency of NADPH oxidase activity
• Impaired chemotaxis and migration caused by abnormal microtubule formation
• Inability to kill streptococci

771. A laboratory experiment is performed to evaluate the chemotactic potential of a group of potential
mediators. Which of the following substances most likely has the greatest affinity for neutrophils?
• C5a
• Fucosyl transferase
• β2-Integrin
• P-selectin

A 26-year-old African-American woman has bilateral hilar adenopathy, and radiography reveals multiple
reticular densities in both lung fields. A bronchoscopic biopsy reveals granulomatous inflammation with multiple
giant cells of the Langhans type and no evidence of caseous necrosis. Which of the following is the most likely
diagnosis?

• Aspergillosis
• Coccidioidomycosis
• Histoplasmosis
• Sarcoidosis

772. In a laboratory exercise for medical students, an unknown compound is studied. The students are informed
that the compound has been isolated from endothelial cells and that its synthesis can be inhibited by aspirin.
In the laboratory, the students demonstrate that the compound is a potent vasodilator and platelet
antiaggregant. Given these findings, the substance is most likely which of the following mediators?
• 5-HPETE
• LTC4
• LXA4
• PGI2

773. A 70-year-old man presents with the sudden onset of left-sided weakness, spasticity, and hyperactive and
pathologic reflexes. The most serious consequences of this disorder are the result of damage to which of the
following cell types?
• Labile cells
• Multipotent adult progenitor cells
• Permanent cells
• Stable cells

774. Inflammation of stomach is called:


• Gastritis.
• Enteritis.
• Colitis.
• Proctitis.

775. Inflammation of liver is called:


• Gastritis.
• Hepatitis
• Colitis.
• Proctitis.

776. Inflammation of kidney is called:


• Gastritis.
• Hepatitis.
• Nephritis.
• Proctitis.

777. Inflammation of small intestine is called:


• Gastritis.
• Enteritis.
• Colitis.
• Proctitis.

778. Inflammation of large intestine is called:


• Gastritis.
• Enteritis.
• Colitis.
• Proctitis.

779. Inflammation of rectum is called:


• Gastritis.
• Enteritis.
• Colitis.
• Proctitis.

780. Inflammation of skin is called:


• Gastritis.
• Hepatitis.
• Nephritis.
• Dermatitis.

781. Inflammation of blood vessels is called:


• Gastritis.
• Hepatitis.
• Nephritis.
• Vasculitis.

782. Inflammation of lung tissue is called:


• Pneumonia.
• Angina.
• Furuncle.
• Carbuncle.

783. Inflammation of throat is called:


• Pneumonia.
• Angina.
• Furuncle.
• Carbuncle.

784. Inflammation of hair follicle is called:


• Pneumonia.
• Angina.
• Furuncle.
• Carbuncle.

785. Inflammation of brain is called:


• Gastritis.
• Hepatitis.
• Enteritis.
• Encephalitis.

786. Local sign of inflammation “rubor” means:


• Heat.
• Pain.
• Redness.
• Swelling.

787. Local sign of inflammation “calor” means:


• Heat
• Pain.
• Redness.
• Swelling.

788. Local sign of inflammation “dolor” means:


• Heat.
• Pain.
• Redness.
• Swelling.

789. Local sign of inflammation “tumor” means:


• Heat.
• Pain.
• Redness.
• Swelling.

790. Local sign of inflammation which associated with pressure on nerve endings by exudate is:
• Rubor.
• Calor.
• Dolor.
• Tumor.
791. Local sign of inflammation which associated with accumulation of exudate is:
• Rubor.
• Calor.
• Dolor.
• Tumor.

792. Calor as a local sign of inflammation is associated with:


• Arterial hyperemia.
• Pressure of nerve endings.
• Exudation
• Interference with enzyme function

793. Tumor as a local sign of inflammation is associated with:


• Arterial hyperemia.
• Pressure of nerve endings.
• Exudation.
• Interference with enzyme function.

794. Dolor as a local sign of inflammation is associated with:


• Arterial hyperemia.
• Pressure of nerve endings.
• Exudation.
• Local pain.

795. Rubor as a local sign of inflammation is associated with:


• Arterial hyperemia.
• Pressure of nerve endings.
• Exudation.
• Interference with enzyme function.

796. Component of alteratio stage of inflammation is:


• Necrosis.
• Arterial hyperemia.
• Phagocytosis.
• Regeneration of epithelial cells.

797. Component of alteratio stage of inflammation is :


• Degeneration of cells.
• Arterial hyperemia.
• Phagocytosis.
• Regeneration of epithelial cells

798. Component of exudation stage of inflammation is:


• Degeneration of cells.
• Arterial hyperemia.
• Necrosis.
• Regeneration of epithelial cells.

799. Component of exudation stage of inflammation is:


• Degeneration of cells
• Necrosis.
• Formation of cell infiltrate
• Regeneration of epithelial cells.

800. Component of exudation stage of inflammation is:


• Degeneration of cells.
• Necrosis.
• Regeneration of epithelial cells.
• Phagocytosis.

801. Component of exudation stage of inflammation is:


• Degeneration of cells.
• Necrosis.
• Pinocytosis
• Regeneration of epithelial cells.

802. Component of exudation stage of inflammation is:


• Degeneration of cells.
• Necrosis.
• Regeneration of epithelial cells
• Leakage of blood plasma from vessels.

803. Component of exudation stage of inflammation is:


• Degeneration of cells.
• Necrosis.
• Regeneration of epithelial cells.
• Emigration of blood cells from vessels.
804. Component of proliferation stage of inflammation is:
• Degeneration of cells.
• Necrosis.
• Regeneration of epithelial cells.
• Emigration of blood cells from vessels

805. Component of proliferation stage of inflammation is :


• Degeneration of cells.
• Necrosis.
• Regeneration of fibroblasts.
• Emigration of blood cells from vessels.

806. Type of acute inflammation which associated with formation of protein-poor fluid is called:
• Serous.
• Fibrinous.
• Purulent
• Hemorrhagic.

807. Type of acute inflammation which associated with formation of hair-like deposits is called:
• Serous.
• Fibrinous.
• Purulent.
• Hemorrhagic.

808. Type of acute inflammation which associated with formation of pseudomembrane on mucosal surfaces is
called:
• Serous.
• Fibrinous.
• Purulent.
• Hemorrhagic.

809. Type of acute inflammation which associated with formation of thick turbid yellow-green fluid is called:
• Serous.
• Fibrinous.
• Purulent.
• Catarrhal.

810. Which type of pneumonia acute is associated with formation of fibrinous exudate:
• Lobar pneumonia.
• Bronchpneumonia.
• Interstitial pneumonia.
• Septic pneumonia.

811. Localized type of purulent inflammation is called:


• Phlegmon.
• Empyema.
• Abscess.
• Furuncle.

812. Diffuse type of purulent inflammation is called:


• Phlegmon.
• Empyema.
• Abscess.
• Furuncle.

813. Type of purulent inflammation with accumulation of pus in serosal cavities and hollow organs is called:
• Phlegmon.
• Empyema.
• Abscess.
• Furuncle.

814. Type of purulent inflammation with accumulation of pus in serosal cavities and hollow organs is called :
• Phlegmon.
• Empyema.
• Abscess.
• Furuncle.

815. Purulent inflammation of hair follicle is called:


• Phlegmon.
• Empyema.
• Abscess.
• Furuncle.

816. Which of the following are thought to mediate, many of the systemic effects of inflammation are
chemotactic and stimulate adhesion molecules:
• Interleukin-1 (IL-1) and tumor necrosis factor
• C5 a and leukotriene B-4
• C3 b.
• Leukotriene C4, D4 and E4.
817. After initiation of an acute inflammatory process third in a sequence of changes in vascular flow is:
• Vasoconstriction.
• Redness.
• Leukocytic migration.
• Vasodilation

818. The term “croupous” inflammation is synonymous with


• Catarrhal inflammation
• Fibrinous inflammation
• Sero-fibrinous inflammation
• Suppurative inflammation

819. Vascular phenomenon in acute inflammation helps in:


• Bringing the defensive cells like neutrophils from interior of the vessels to the site of irritation
• Minimizing the effect of irritant by diluting it considerably
• Limiting the inflammatory reaction to the smallest possible area
• All of these

820. Which of the complement components act as chemokines?


• C3b
• C4b
• C5a
• C4a

821. All are types of tissue macrophages, except:


• Littoral cells
• Hoffbauer cells
• Osteoclasts
• Osteoblasts

822. Main cytokines acting as mediators of inflammation are as under, except:


• Interleukin-1 (IL-1)
• Tumor necrosis factor α (TNF-α)
• Nitric oxide (NO)
• Interferon –y (IF-y)
1. Teratogens are defined as agents which induce:
Mitosis
Carcinogenesis
Birth defects
Fallot’s tetralogy
2. For chromosomal study, it is best to use the following nucleated cells:
Polymorphs
Lymphocytes
Epithelial cells
Fibroblasts
3. For chromosomal study, the dividing cells are arrested by colchicine in the following phase of cell cycle:
Prophase
Metaphase
Anaphase
Telophase
4. Denver classification divides chromosomes based on their length into the following groups:
A to C (3 groups)
A to E (5 groups)
A to G (7 groups)
A to I (9 groups)
5. Polyploidy is generally not a feature of dividing cells of the following type:
Megakaryocytes
Hepatocytes
Tubular cells
Conceptus of abortions
6. Numeric abnormality in chromosome occurs in the following conditions except:
Ph chromosome in CML
Turner’s syndrome
Klinefelter’s syndrome
Down’s syndrome
7. Mutations affecting germ cells produce:
Cancers
Inherited diseases
Congenital malformations
Aneuploidy
8. In lysosomal storage diseases, the following cells are particularly involved:
Hepatocytes
Skeletal muscle
Macrophages
White pulp of spleen
9. Out of the following glycogenosis, the following is example of lysosomal storage disease:
Von Gierke’s disease
Pompe’s disease
Forbe’s disease
Anderson’s disease
10. Blastomas are childhood tumours seen more often in the age range of:
<4 years
5-9 years
10-14 years
14-16 years
11. All of the following are X-linked recessive disorders except:
Haemophilia A and B
Chronic granulomatous disease
G-6 PD deficiency
Sickle cell anaemia
12. Gaucher cells are positive for all except:
PAS
Mucicarmine
Oil red O
Prussian blue
13. A newborn male infant is examined. There is a palpable right testis, but no left testis, in the scrotal sac. The
infant has no other abnormalities noted. Which of the following is the most likely abnormality involving the
gonads of this infant?
Agenesis
Fusion
Hypopituitarism
Incomplete descent
14. Following ovulation and fertilization of an ovum, a blastocyst develops. Growth and differentiation occurs
following implantation. Which of the following will develop into the placenta?
Epimere
Neural plate
Sclerotome
Trophoblast
15. A 5-year-old child fatigues easily. On examination a machinery-like murmur is auscultated over his chest.
Echocardiography shows shunting of blood from the aorta to the pulmonary artery. An abnormality involving
which of the following structures is most likely present in this boy?
Ductus arteriosus
Endocardial cushion
Sinus venosus
Third aortic arch
16. A 3500 gm infant born at term is doing well and gaining weight normally until the fourth week of life. He then
exhibits feeding difficulty with forceful vomiting. Now, on physical examination, no external anomalies are
noted. However, there is a firm mass palpable in the upper mid abdominal region. Which of the following
conditions is the infant most likely to have?
Duodenal atresia
Hepatoblastoma
Hirschsprung disease
Pyloric stenosis
17. An infant born at term is examined, and there is abnormal fusion of the 3rd, 4th, and 5th digits of the left hand.
A radiograph of the left hand shows transformation of metacarpals I, II, III, and V to short, carpal-like bones. A
mutation involving which of the following genes most likely led to these findings?
CFTR
FGFR3
HOXD13
SHH
18. An infant born at term has a radiograph taken. No congenital abnormalities are noted. The upper heart borders
are not visualized. Which of the following structures is most likely to obscure the heart borders in this infant's
radiograph?
Esophagus
Lymph node
Stomach
Thymus
19. A secondary oocyte is developing normally. At which of the following times does the second maturation division
of meiosis become completed?
In the ovarian cortex
During ovulation
At fertilization
In the blastocyst
20. During the 3rd week of embryonic development, splanchnic mesoderm differentiates into blood islands with
angioblasts. In what developing tissue site does this process occur?
Heart
Liver
Spleen
Thymus
21. An ovum is fertilized. After 3 weeks a blastocyst implanted on the endometrium has an inner embryoblast
and an outer trophoblast. Which of the following is the next step in forming a 3-layered embryonic disk?
Decidualization
Gastrulation
Lateralizaton
Neurulation
22. In the developing embryo, pharyngeal arches are formed. The second (branchial) arch gives rise to the
stapedius, orbicularis oris, and posterior belly of the digastric muscles. Which of the following cranial nerves
is also derived from this arch?
III
V
VII
IX
23. In the normally developing embryo, aortic arches are apparent by the end of the fourth week. Which of the
following arteries is derived from the third pair of aortic arches?
Carotid
Pulmonary
Stapedial
Subclavian
24. A fetus at 18 weeks gestation is normally formed and appropriately sized. No fetal or placental anomalies are
present. Which of the following parts of the fetal vasculature is most likely to have the greatest oxygen
concentration?
Aorta
Ductus arteriosus
Pulmonary vein
Umbilical vein
25. An embryonic eye is developing normally. A retina is present on the posterior aspect of the globe. What is
the embryologic origin of the portion of retina containing nerve cell bodies?
Diencephalon
Mesoderm
Notochord
Pharyngeal arch
26. A newborn examination is performed. The tympanic membranes are visualized bilaterally. What is the
embryologic origin of the structure that forms the canal connecting the external ear to the tympanic
membrane?
Bony labyrinth
First pharyngeal cleft
Endolymphatic duct
Otic pit
27. A 20-year-old woman is unable to conceive and undergoes an infertility workup. A pelvic ultrasound reveals
a bicornuate uterus. Which of the following is the most likely explanation for her anomalous uterus?
Excess circulating maternal androgens while in utero
Failure of primordial germ cell migration
Failure of the paramesonephric ducts to fuse
Failure of the urorectal septum to reach the perineum
28. In a developing embryo, neural crest cells form adjacent to the neural tube. These neural crest cells then
undergo migration. Which of the following structures is derived from these migrating cells?
Dorsal root ganglion
Dura mater
Epidermis covering the back
Intervertebral disk
29. A neonate undergoes a check of health status. On examination a faint cardiac murmur is auscultated.
Echocardiography is performed and shunting of blood is noted from the left atrium to the right atrium. An
abnormality in embryonic development of which of the following structures has most likely occurred in this
neonate?
Ductus arteriosus
Endocardial cushion
Fourth aortic arch
Sinus venosus
30. The two superior and two inferior parathyroid glands are found adjacent to the posterior aspect of the
thyroid gland. From which of the following are these parathyroid glands derived in embryologic
development?
Foramen cecum
Mandibular swellings
Maxillary processes
Pharyngeal pouches
31. A 31-year-old woman has noted no fetal movement by 16 weeks gestation. A fetal ultrasound shows the
fetal head contains a brain with thin cortex, single enlarged ventricle, and fused thalami. Which of the
following developing regions of the brain is most likely abnormal in this fetus?
Cranial vault
Mesencephalon
Prosencephalon
Rhombencephalon
32. Following fertilization of the ovum, the ovarian corpus luteum continues to produce progesterone. If the
corpus luteum regresses in the first trimester, abortion occurs. Which of the following hormones maintains
this capability of the corpus luteum?
Alpha-fetoprotein
Estradiol
Human chorionic gonadotropin
Human placental lactogen
33. Following coitus, spermatozoa migrate to the fallopian tubes. Within the tubes, the acrosomal head of
spermatozoa undergoes reduction of glycoprotein coating with increasing permeability to calcium. Which of
the following functions of spermatozoa is facilitated by this process?
Blocking other sperm
Fertilization
Motility
Mitosis
34. A 24-year-old primigravida has felt no fetal movement at 16 weeks gestation. A screening fetal ultrasound
shows anhydramnios and a single poorly formed lower extremity. An abnormality involving which of the
following structures most likely led to these findings?
Aorta
Dermis
Neural crest
Splanchnic mesoderm
35. A neonate born at term after an uncomplicated pregnancy is noted to pass pale coloured stool and dark
urine. The infant becomes progressively jaundiced in the next 2 days. On examination the infant is at the
50th percentile for weight. Icterus is present. The serum IgM antibody titer to cytomegalovirus is increased.
Which of the following developmental abnormalities most likely occurred in utero?
Accessory pancreas
Biliary atresia
Gallbladder duplication
Hepatic agenesis
36. Following birth, circulation from placenta to fetus ceases. Which of the following fetal structures that
conducts blood to the inferior vena cava becomes atretic in postnatal life?
Azygous vein
Ductus arteriosus
Ductus venosus
Foramen ovale
37. A newborn male infant is examined. There is a palpable right testis, but no left testis, in the scrotal sac. The
infant has no other abnormalities noted. Which of the following is the most likely abnormality involving the
gonads of this infant?
Agenesis
Fusion
Hypopituitarism
Incomplete descent
38. How soon after fertilization occurs within the uterine tube does the blastocyst begin implantation?
Within minutes
By 12 hours
By day 1
By day 7
39. Where does the blastocyst normally implant?
Functional layer of the cervix
Functional layer of the endometrium
Basal layer of the endometrium
Myometrium
40. Which of the following events is involved in the cleavage of the zygote during week 1 of development?
A series of meiotic divisions forming blastomeres
Production of highly differentiated blastomeres
An increased cytoplasmic content of blastomeres
A decrease in size of blastomeres
41. Which of the following structures must degenerate for blastocyst implantation to occur?
Endometrium in progestational phase
Zona pellucida
Syncytiotrophoblast
Cytotrophoblast
42. Which of the following is the origin of the mitochondrial DNA of all human adult cells?
Paternal only
Maternal only
A combination of paternal and maternal
Either paternal or maternal
43. Individual blastomeres were isolated from a blastula at the 4-cell stage. Each blastomere was cultured in
vitro to the blastocyst stage and individually implanted into four pseudopregnant foster mothers. Which of
the following would you expect to observe 9 months later?
Birth of one baby
Birth of four genetically different babies
Birth of four genetically identical babies
Birth of four grotesquely deformed babies
44. During the later stages of pregnancy, maternal blood is separated from fetal blood by the
Syncytiotrophoblast only
Cytotrophoblast only
Syncytiotrophoblast and cytotrophoblast
Syncytiotrophoblast and fetal endothelium
45. The maternal and fetal components of the placenta are
Decidua basalis and secondary chorionic villi
Decidua capsularis and secondary chorionic villi
Decidua parietalis and tertiary chorionic villi
Decidua basalis and villous chorion
46. The intervillous space of the placenta contains
Maternal blood
Fetal blood
Maternal and fetal blood
Amniotic fluid
47. A young insulin-dependent diabetic woman in her first pregnancy is concerned that her daily injection of
insulin will cause a congenital malformation in her baby. What should the physician tell her?
Insulin is highly teratogenic; discontinue treatment
Insulin does not cross the placental membrane
Insulin crosses the placental membrane but is degraded rapidly
Insulin will benefit her baby by increasing glucose metabolism
48. What is a normal amount of amniotic fluid at term?
50 mL
500 mL
1000 mL
1500 mL
49. Which of the following does not pass through the primitive umbilical ring?
Allantois
Amnion
Yolk sac
Connecting stalk
50. Which of the following best describes the placental components of dizygotic twins?
One placenta, two amniotic sacs, one chorion
One placenta, two amniotic sacs, two chorions
Two placentas, two amniotic sacs, one chorion
Two placentas, two amniotic sacs, two chorions
51. A 26-year-old pregnant woman experiences repeated episodes of bright red vaginal bleeding at week 28,
week 32, and week 34 of pregnancy. The bleeding spontaneously subsided each time. Use of ultrasound
shows that the placenta is located in the lower right portion of the uterus over the internal os. What is the
diagnosis?
Hydatidiform mole
Vasa previa
Placenta previa
Placental abruption
52. A 19-year-old woman in week 32 of a complication-free pregnancy is rushed to the emergency department
because of profuse vaginal bleeding. The bleeding subsides, but afterward no fetal heart sounds can be
heard, indicating intrauterine fetal death. The woman goes into labor and delivers a stillborn infant. On
examination of the afterbirth, a velamentous placenta is detected. Although not much can be done at this
point, what is the diagnosis?
Placenta previa
Vasa previa
Hydatidiform mole
Premature rupture of the amniochorionic membrane
53. A 32-year-old pregnant woman at 30 weeks of gestation comes to her physician because of excess weight
gain in a 2-week period. Ultrasonography reveals polyhydramnios. Which fetal abnormality is most likely
responsible for the polyhydramnios?
Bilateral kidney agenesis
Umbilical cord knots
Velamentous placenta
Esophageal atresia
54. A 25-year-old pregnant woman at 17 weeks of gestation comes to her OB/GYN for a normal examination.
During routine blood tests, her serum α-fetoprotein (AFP) concentration is found to be markedly decreased
for her gestational age. Which abnormality will the physician need to rule out based on these low AFP levels?
Spina bifida
Anencephaly
Omphalocele
Esophageal atresia
55. Which of the following time intervals best describes the maximum susceptibility period?
Week 1
Weeks 3–8
Weeks 9–38
Weeks 15-17
56. Which of the following time intervals best describes the resistant period?
Week 1
Weeks 3–8
Weeks 9–38
Weeks 15-17
57. The most common viral infection is
Cytomegalovirus
Rubella virus
Herpes virus type 2
Varicella zoster virus
58. Which of the following is a parasite found in cats?
Treponema pallidum
Toxoplasma gondii
Rubella virus
Cytomegalovirus
59. Warfarin falls into which category of drugs?
Category X drugs
Category D drugs
XX
Not either
60. Valium falls into which category of drugs?
Category X drugs
Category D drugs
XX
Not either
61. Teratogens are defined as agents which induce:
Mitosis
Carcinogenesis
Birth defects
Fallot’s tetralogy
62. For chromosomal study, it is best to use the following nucleated cells:
Polymorphs
Lymphocytes
Epithelial cells
Fibroblasts
63. For chromosomal study, the dividing cells are arrested by colchicine in the following phase of cell cycle:
Prophase
Metaphase
Anaphase
Telophase
64. Denver classification divides chromosomes based on their length into the following groups:
A to C (3 groups)
A to E (5 groups)
A to G (7 groups)
A to I (9 groups)
65. Polyploidy is generally not a feature of dividing cells of the following type:
Megakaryocytes
Hepatocytes
Tubular cells
Conceptus of abortions
66. Numeric abnormality in chromosome occurs in the following conditions except:
Ph chromosome in CML
Turner’s syndrome
Klinefelter’s syndrome
Down’s syndrome
67. Mutations affecting germ cells produce:
Cancers
Inherited diseases
Congenital malformations
Aneuploidy
68. In lysosomal storage diseases, the following cells are particularly involved:
Hepatocytes
Skeletal muscle
Macrophages
White pulp of spleen
69. Out of the following glycogenosis, the following is example of lysosomal storage disease:
Von Gierke’s disease
Pompe’s disease
Forbe’s disease
Anderson’s disease
70. Blastomas are childhood tumours seen more often in the age range of:
<4 years
5-9 years
10-14 years
14-16 years
71. All of the following are X-linked recessive disorders except:
Haemophilia A and B
Chronic granulomatous disease
G-6 PD deficiency
Sickle cell anaemia
72. Gaucher cells are positive for all except:
PAS
Mucicarmine
Oil red O
Prussian blue
73. hich type of pneumoconiosis is associated with inhalation of carbon particles?
Anthracosis.
Silicosis.
Asbestosis.
Siderosis.
74. Which type of pneumoconiosis is associated with inhalation of iron particles?
Anthracosis
Silicosis.
Asbestosis.
Siderosis.
75. Inhalation of foreign particles leads to development in the lung:
Acute purulent pneumonia.
Obstructive emphysema.
Chronic inflammation and fibrosis.
Acute abscess
76. Exogenic pigmentation as a result of inhalation of the foreign particles is:
Argyria
Asbestosis.
Carotenemia.
Lead poisoning.
77. Exogenic pigmentation as a result of inhalation of the foreign particles is:
Argyria.
Anthracosis.
Lead poisoning.
Carotenemia.
78. Exogenic pigmentation as a result of inhalation of the foreign particles is:
Silicosis.
Tattoing.
Carotenemia.
Lead poisoning.
79. Exogenic pigmentation as a result of inhalation of the foreign particles is:
Carotenemia.
Tattoing.
Siderosis.
Lead poisoning.
80. Exogenic pigmentation as a result of inhalation of the foreign particles is:
Carotenemia.
Tattoing.
Argyria.
Talcosis.
81. Exogenic pigmentation as a result of ingestion of the foreign substances is:
Argyria.
Silicosis.
Asbestosis.
Siderosis.
82. Exogenic pigmentation as a result of ingestion of the foreign substances is:
Anthracosis.
Silicosis.
Carotenemia.
Siderosis.
83. Exogenic pigmentation as a result of ingestion of the foreign substances is:
Anthracosis.
Lead poisoning.
Silicosis.
Siderosis.
84. Exogenic pigmentation as a result of injection of the foreign substances is:
Anthracosis.
Lead poisoning.
Silicosis.
Tattoing.
85. Argyria is as a result of ingestion of the:
Iron.
Silver.
Gold.
Aluminium
86. Characteristic blue lines on teeth at the gumline occurs in:
Chronic lead poisoning.
Chronic iron poisoning.
Chronic silver poisoning.
Chronic gold poisoning.
87. Brownish pigmentation in the skin, bowel and kidney occurs in:
Options
Argyria
Tattoing
Carotenemia.
Lead poisoning.
88. Yellowish-red coloration of the skin occurs in:
Options
Argyria.
Tattoing.
Carotenemia.
Lead poisoning.
89. Tattoing is a result of introduction into the dermis:
Options
Iron.
Silver.
Copper.
Carbon.
90. Tattoing is a result of introduction into the dermis:
Options
Iron.
Silver
Copper.
Cinnabar.
91. Tattoing is a result of introduction into the dermis:
Options
Iron
India ink.
Copper.
Asbestos
92. Exogenic pigmentation is:
Options
Hemosiderosis.
Melanosis.
Lipofuscinosis.
Anthracosis.
93. Exogenic pigmentation is:
Options
Hemosiderosis.
Melanosis.
Lipofuscinosis.
Asbestosis.
94. Exogenic pigmentation is:
Options
Siderosis.
Melanosis.
Lipofuscinosis.
Jaundice.
95. Exogenic pigmentation is:
Options
Argyria.
Ferritinemia.
Porphyria.
Jaundice.
96. Exogenic pigmentation is:
Options
Porphyria.
Ferritinemia
Carotenemia.
Jaundice.
97. Exogenic pigmentation is:
Options
Porphyria.
Ferritinemia
Tattoing.
Jaundice
98. Endogenic pigmentation is:
Options
Argyria
Ferritinemia.
Carotenemia.
Tattoing.
99. Endogenic pigmentation is:
Options
Argyria
Silicosis.
Carotenemia
Hemosiderosis.
100. Endogenic pigmentation is:
Options
Argyria.
Silicosis.
Carotenemia.
Jaundice.
101. Endogenic pigmentation is:
Options
Argyria.
Silicosis.
Siderosis.
Melanosis.
102. Endogenic pigmentation is:
Options
Argyria.
Silicosis.
Siderosis.
Lipofuscinosis.
103. Hemosiderin histologically appears as:
Options
Golden-yellow to brown pigment.
Pinkish to red pigment
Pale-green to dark-green pigment.
Dark-blue to violet pigment.
104. “Heart failure cells” are the alveolar macrophages, containing pigment which is called:
Options
Hemosiderin.
Hematin.
Ferritin.
Bilirubin.
105. Cause of generalized hemosiderosis is:
Options
Intracerebral hemorrhage.
Pulmonary hemorrhagic infarct.
Brown induration of lungs.
Increased absorbtions of dietary iron.
106. Cause of generalized hemosiderosis is:
Options
Intracerebral hemorrhage.
Pulmonary hemorrhagic infarct.
Brown induration of lungs.
Impaired utilization of iron.
107. Cause of generalized hemosiderosis is:
Options
Intracerebral hemorrhage
Pulmonary hemorrhagic infarct.
Brown induration of lungs.
Repeated transfusions of blood.
108. Hereditary disease which associated with excessive intestinal absorbtion of iron and deposition of
hemosiderin is called:
Options
Hemosiderosis.
Lipofuscinosis
Melanosis
Hemachromatosis.
109. Morphological features of hemachromatosis are all the following, EXCEPT:
Options
Pigmentary liver cirrhosis.
Myocardial infarction.
Pigmentary cardiomyopathy.
Diabetes mellitus
110. Cause of prehepatic jaundice is:
Options
Hepatitis.
Liver cirrhosis.
Obstruction of bile ducts by stones.
Intoxications
111. Cause of prehepatic jaundice is:
Options
Hepatitis
Liver cirrhosis.
Obstruction of bile ducts by stones.
Infections.
112. Cause of hepatocellular jaundice is:
Options
Obstruction of bile ducts by stones.
Compression of bile ducts by tumors.
Heterohemotransfusions.
Hepatosis.
113. Cause of posthepatic jaundice is:
Options
Atresia of bile ducts.
Hepatitis
Heterohemotransfusions.
Liver cirrhosis.
114. Cause of posthepatic jaundice is:
Options
Stenosis of bile ducts.
Hepatitis.
Heterohemotransfusions.
Liver cirrhosis.
115. Pigment which is a result of reaction between hemoglobin and hydrochloric acid is called:
Options
Hemosiderin.
Hemin.
Ferritin.
Bilirubin.
116. Pigment which is a result of reaction between hemoglobin and malarial toxin is called:
Options
Hemosiderin.
Hemin.
Hemomelanin.
Bilirubin.
117. Histologically hematin may be seen as:
Options
Yellow granules.
Red granules.
Green granules.
Black granules
118. Pigment which is formed in tissues where a good oxygen supply is lacking in dead tissues is called:
Options
Hemosiderin.
Hemin.
Hemomelanin.
Hematoidin.
119. Histologically hematoidin may be seen as amorphous:
Options
Yellow granules.
Red granules.
Green granules.
Black granules.
120. Pigment which is associated with highly photosensitivity and damage of skin is called:
Options
Hemosiderin.
Hemin.
Hemomelanin.
Porphyrin.
121. Generalized acquired hyperpigmentation of melanin occurs in:
Options
Addison’s disease.
Melanosis coli.
Lentigo.
Nevus.
122. Generalized acquired hyperpigmentation of melanin occurs in:
Options
Avitaminosis.
Melanosis coli.
Lentigo.
Nevus.
123. Generalized acquired hyperpigmentation of melanin occurs in:
Options
Melanosis coli.
Cachexia.
Lentigo.
Nevus.
124. Generalized acquired hyperpigmentation of melanin occurs in:
Options
Melanosis coli.
Chronic arsenical poisoning.
Lentigo.
Nevus
125. Generalized congenital hyperpigmentation of melanin occurs in:
Options
Melanosis coli
Chronic arsenical poisoning.
Xeroderma pigmentosum.
Nevus.
126. Focal acquired hyperpigmentation of melanin occurs in:
Options
Addison’s disease.
Cachexia.
Melanosis coli.
Nevus.
127. Focal congenital hyperpigmentation of melanin occurs in:
Options
Addison’s disease
Cachexia.
Avitaminosis.
Nevus.
128. Focal congenital hyperpigmentation of melanin occurs in:
Options
Addison’s disease.
Cachexia.
Avitaminosis.
Melanoma.
129. General hypopigmentation of melanin occurs in:
Options
Leukoderma.
Albinism.
Vitiligo.
Cachexia.
130. Focal acquired hypopigmentation of melanin is called:
Options
Albinism.
Nevus
Leukoderma.
Cachexia.
131. Focal congenital hypopigmentation of melanin is called:
Options
Albinism.
Nevus.
Vitiligo
Cachexia.
132. Albinism is associated with all the following signs, EXCEPT:
Options
Blond hair.
White skin.
Poor vision.
Arterial hypertension.
133. Leukoderma is a sign of:
Options
Syphilis
Typhoid fever.
Yellow fever.
Antrax.
134. Leukoderma is a sign of all the following diseases, EXCEPT:
Options
Syphilis.
Leprosy.
Radiation dermatitis.
Gonorrhea.
135. Distrophic calcification occurs in:
Options
Hyperparathyroidism.
Hypervitaminosis D.
Hyperthyroidismv
Atheromas of advanced atherosclerosis.
136. Distrophic calcification occurs in:
Options
Hyperparathyroidism.
Hypervitaminosis D.
Hyperthyroidism
Old thrombi.
137. Distrophic calcification occurs in:
Options
Hyperparathyroidism.
Hypervitaminosis D.
Hyperthyroidism.
Dead animal parasites.
138. Metastatic calcification occurs in:
Options
Hyperthyroidism.
Damaged heart valves.
Atheromas.
Necrosis.
139. Metastatic calcification occurs in:
Options
Atheromas.
Damaged heart valves.
Hypervitaminosis D.
Necrosis.
140. Metastatic calcification occurs in:
Options
Atheromas.
Damaged heart valves.
Old thrombi
Increased bone catabolism.
141. Metastatic calcification occurs in:
Options
Atheromas.
Damaged heart valves.
Decreased bone formation.
Necrosis.
142. Common localization of metastatic calcification is in all the following organs, EXCEPT:
Options
Blood vessels.
Brain.
Kidneys.
Lungs
143. Derangement of nucleoproteids metabolism is associated with:
Options
Gout.
Atheroma.
Nevus.
Hemachromatosis.
144. Derangement of nucleoproteids metabolism is associated with:
Options
Urolithiasis.
Atheroma.
Nevus.
Hemachromatosis.
145. Derangement of nucleoproteids metabolism is associated with:
Options
Cerebral infarction.
Myocardial infarction.
Urate infarction of newborn
Pulmonary hemorrhagic infarction.
146. Chemical substances which occurs in urinary stones are of all the following, EXCEPT:
Options
Cholesterol.
Cystine.
Magnesium.
Calcium.
147. Congenital incomplete development of thymus is called:
Options
Aplasia.
Hypoplasia.
Dysplasia
Atrophy.
148. Aquired decrease in weight of thymus is called:
Options
Aplasia.
Hypoplasia.
Dysplasia.
Atrophy.
149. Abnormal development of thymus is called:
Options
Aplasia.
Hypoplasia.
Dysplasia.
Atrophy.
150. Decrease in size and weight of thymus under different stress situations including infectious diseases,
intoxications, traumas is called:
Options
Aplasia.
Hypoplasia.
Dysplasia.
Accidental involution.
151. Increase in size and weight of thymus more than age level with normal histological structure is
called:
Options
Aplasia.
Hypoplasia.
Thymomegaly.
Atrophy.
152. Collection of B-lymphocytes and plasma cells with formation of lymphoid follicles in the intralobular
perivascular spaces of thymus parenchyma is called:
Options
Aplasia.
Hypoplasia.
Thymomegaly.
Hyperplasia with lymphoid follicles
153. Changes in peripheric lymphoid tissue under antigene stimulation include are all the following
processes, EXCEPT:
Options
Macrophageal reaction.
Hyperplasia of lymphocytes.
Plasma cell transformation.
Infarction.
154. Changes in peripheric lymphoid tissue under antigene stimulation include are all the following
processes, EXCEPT:
Options
Macrophageal reaction.
Hyperplasia of lymphocytes.
Dysplasia of lymphocytes.
Plasma cell transformation.
155. Changes in peripheric lymphoid tissue under antigene stimulation include are all the following
processes, EXCEPT:
Options
Macrophageal reaction.
Anaplastic transformation of lymphocytes.
Hyperplasia of lymphocytes.
Plasma cell transformation.
156. Changes in lymph nodes under antigene stimulation include are all the following processes, EXCEPT:
Options
Hyperemia.
Edema.
Accumulation of neutrophils.
Accumulation of macrophages.
157. Changes in lymph nodes under antigene stimulation include are all the following processes, EXCEPT:
Options
Hyperemia.
Ischemia.
Edema.
Accumulation of plasma cells.
158. Changes in spleen under antigene stimulation include are all the following processes, EXCEPT:
Options
Hypoplasia of red pulp
Hyperplasia of red pulp.
Plasmatization of red pulp.
Accumulaton of macrophages.
159. Changes in spleen under antigene stimulation include are all the following processes, EXCEPT:
Options
Hyperplasia of red pulp.
Hypoplasia of follicles.
Plasmatization of red pulp.
Accumulaton of macrophages.
160. Changes in spleen under antigene stimulation include are all the following processes, EXCEPT:
Options
Hyperplasia of red pulp.
Plasmatization of red pulp.
Accumulaton of giant cells.
Accumulaton of macrophages.
161. Changes in bone marrow under antigene stimulation include are all the following processes, EXCEPT:
Options
Hyperplasia.
Macrophageal-plasma cell transformation.
Myeloid metaplasia.
Hypoplasia.
162. Changes in bone marrow under antigene stimulation include are all the following processes, EXCEPT:
Options
Hyperplasia.
Macrophageal-plasma cell transformation.
Myeloid metaplasia.
Myeloid dysplasia.
163. Changes in peripheric lymphoid tissue in congenital immunodeficiency syndromes are characterized
by:
Options
Decrease in size of follicles of spleen.
Increase in size of follicles of spleen.
Hyperplasia of lymph nodes
Myeloid metaplasia of bone marrow.
164. Changes in peripheric lymphoid tissue in congenital immunodeficiency syndromes are characterized
by:
Options
Increase in size of follicles of spleen.
Hyperplasia of lymph nodes.
Absence of B-zones in cortex of lymph nodes.
Myeloid metaplasia of bone marrow.
165. Changes in peripheric lymphoid tissue in congenital immunodeficiency syndromes are characterized
by:
Options
Increase in size of follicles of spleen.
Absence of germinal centers in follicles of spleen.
Hyperplasia of lymph nodes.
Myeloid metaplasia of bone marrow.
166. Immediate type of hypersensitivity morphologically occurs like:
Options
Acute immune inflammation
Purulent inflammation.
Hemorragic inflammation.
Chronic immune inflammation.
167. Delayed type of hypersensitivity morphologically occurs as:
Options
Acute immune inflammation.
Purulent inflammation.
Chronic immune inflammation.
Hemorragic inflammation.
168. Signs of acute immune inflammation are all the following processes, EXCEPT:
Options
Fast development.
Prevalence of alterative changes.
Slow development
Prevalence of exudative changes.
169. Morphologically immediate type of hypersensitivity occurs as:
Options
Fibrinoid necrosis.
Lympho-hystiocytic infiltration.
Macrophageal infiltration.
Granulomatosis.
170. Morphologically immediate type of hypersensitivity occurs as:
Options
Lympho-hystiocytic infiltration.
Macrophageal infiltration.
Granulomatosis
Plasmatic saturation.
171. Morphologically immediate type of hypersensitivity occurs as:
Options
Lympho-hystiocytic infiltration.
Mucoid changes.
Macrophageal infiltration.
Granulomatosis.
172. Morphologically immediate type of hypersensitivity occurs as:
Options
Lympho-hystiocytic infiltration.
Fibrinoid changes.
Macrophageal infiltration.
Granulomatosis.
173. Morphologically immediate type of hypersensitivity occurs as:
Options
Lympho-hystiocytic infiltration.
Fibrinous-hemorrhagic exudate.
Macrophageal infiltration.
Granulomatosis.
174. Morphological signs of immediate type of hypersensitivity are all the following, EXCEPT:
Options
Lympho-hystiocytic infiltration.
Mucoid and Fibrinoid changes.
Plasmatic saturation.
Fibrinoid necrosis.
175. Morphological signs of immediate type of hypersensitivity are all the following, EXCEPT:
Options
Mucoid and Fibrinoid changes.
Plasmatic saturation.
Granulomatosis.
Fibrinoid necrosis.
176. Morphological signs of immediate type of hypersensitivity are all the following, EXCEPT:
Options
Mucoid and Fibrinoid changes.
Plasmatic saturation.
Fibrinoid necrosis.
Macrophageal infiltration.
177. Which disease is associated with immediate type of hypersensitivity?
Options
Myocardial infarction.
Liver cirrhosis.
Аrthus reaction.
Purulent meningitis.
178. Which disease is associated with immediate type of hypersensitivity?
Options
Myocardial infarction.
Glomerulonephritis.
Liver cirrhosis.
Purulent meningitis.
179. Which disease is associated with immediate type of hypersensitivity?
Options
Syphilis.
Myocardial infarction.
Liver cirrhosis.
Purulent meningitis.
180. Which disease is associated with immediate type of hypersensitivity?
Options
Myocardial infarction
Liver cirrhosis.
Purulent meningitis.
Rheumatic fever.
181. Which disease is associated with immediate type of hypersensitivity?
Options
Myocardial infarction
Liver cirrhosis.
Purulent meningitis.
Systemic lupus erytematosus.
182. Which disease is associated with immediate type of hypersensitivity?
Options
Myocardial infarction.
Nodular periarteritis.
Purulent meningitis.
Liver cirrhosis.
183. Which disease is not associated with immediate type of hypersensitivity?
Options
Myocardial infarction.
Nodular periarteritis
Glomerulonephritis.
Syphilis.
184. Which disease is not associated with immediate type of hypersensitivity?
Options
Liver cirrhosis
Nodular periarteritis
Glomerulonephritis.
Syphilis.
185. Which disease is not associated with immediate type of hypersensitivity?
Options
Glomerulonephritis.
Nodular periarteritis.
Arterial hypertension.
Syphilis.
186. Reagin reactions are associated with action of which type of allergic reagin-antibodies?
Options
IgA.
IgB.
IgC.
IgE.
187. Reagin reactions are associated with formation of which type of exudate?
Options
Basophylic.
Eosinophylic.
Neutrophylic.
Hemorrhagic.
188. Which disease is associated with action of allergic reagin-antibodies?
Options
Chronic bronchitis.
Pulmonary carcinoma.
Atopic bronchial asthma.
Bronchiectasis.
189. Morphologically delayed type of hypersensitivity occurs as:
Options
Mucoid and Fibrinoid changes.
Plasmatic saturation.
Fibrinoid necrosis
Cytolysis.
190. Morphologically delayed type of hypersensitivity occurs as:
Options
Lympho-hystiocytic infiltration.
Mucoid and Fibrinoid changes.
Plasmatic saturation.
Fibrinoid necrosis.
191. Morphologically delayed type of hypersensitivity occurs as:
Options
Mucoid and Fibrinoid changes
Plasmatic saturation.
Granulomatosis
Fibrinoid necrosis.
192. Morphologically delayed type of hypersensitivity occurs as:
Options
Mucoid and Fibrinoid changes.
Plasmatic saturation.
Fibrinoid necrosis.
Macrophageal infiltration.
193. Morphologically delayed type of hypersensitivity occurs as:
Options
Mucoid and Fibrinoid changes.
Plasmatic saturation.
Fibrinoid necrosis.
Cytoplasmic bridges between lymphocytes and macrophages.
194. Morphological signs of delayed type of hypersensitivity are all the following, EXCEPT:
Options
Fibrinoid necrosis.
Lympho-hystiocytic infiltration.
Macrophageal infiltration.
Granulomatosis.
195. Morphological signs of delayed type of hypersensitivity are all the following, EXCEPT:
Options
Lympho-hystiocytic infiltration.
Macrophageal infiltration.
Granulomatosis.
Plasmatic saturation.
196. Morphological signs of delayed type of hypersensitivity are all the following, EXCEPT:
Options
Lympho-hystiocytic infiltration.
Mucoid changes.
Macrophageal infiltration.
Granulomatosis.
197. Morphological signs of delayed type of hypersensitivity are all the following, EXCEPT:
Options
Lympho-hystiocytic infiltration.
Fibrinous-hemorrhagic exudate.
Macrophageal infiltration.
Granulomatosis.
198. Which disease is associated with delayed type of hypersensitivity?
Options
Myocardial infarction.
Liver cirrhosis.
Purulent meningitis.
Contact dermatitis.
199. Which disease is associated with delayed type of hypersensitivity?
Options
Viral hepatitis.
Liver cirrhosis.
Chronic cholecystitis.
Arterial hypertension.
200. Which disease is associated with delayed type of hypersensitivity?
Options
Typhoid fever.
Tuberculosis.
Syphilis.
Purulent meningitis.
201. Which disease is associated with delayed type of hypersensitivity?
Options
Typhoid fever.
Brucellosis.
Syphilis.
Purulent meningitis.
202. Which disease is not associated with delayed type of hypersensitivity?
Options
Contact dermatitis.
Brucellosis.
Syphilis.
Tuberculosis.
203. Which disease is not associated with delayed type of hypersensitivity?
Options
Contact dermatitis.
Typhoid fever.
Brucellosis.
Tuberculosis.
204. Which disease is not associated with delayed type of hypersensitivity?
Options
Purulent meningitis.
Contact dermatitis.
Brucellosis.
Tuberculosis.
205. Which disease is not associated with delayed type of hypersensitivity?
Options
Viral hepatitis.
Contact dermatitis.
Brucellosis.
Sepsis.
206. Morphological signs of transplant rejection are all the following, EXCEPT:
Options
Lympho-hystiocytic infiltration.
Edema of transplant.
Macrophageal infiltration.
Granulomatosis.
207. Morphological signs of transplant rejection are all the following, EXCEPT:
Options
Lympho-hystiocytic infiltration.
Edema of transplant.
Macrophageal infiltration.
Cytoplasmic bridges between lymphocytes and macrophages.
208. Morphological signs of transplant rejection are all the following, EXCEPT:
Options
Lympho-hystiocytic infiltration.
Edema of transplant.
Macrophageal infiltration.
Eosinophylic infiltration.
209. Morphological signs of transplant rejection are all the following, EXCEPT:
Options
Lympho-hystiocytic infiltration.
Edema of transplant.
Macrophageal infiltration.
Coagulative necrosis.
210. Morphological signs of transplant rejection are all the following, EXCEPT:
Options
Lympho-hystiocytic infiltration.
Edema of transplant.
Hyperemia of transplant.
Macrophageal infiltration.
211. Predisposing factor in pathogenesis of autoimmunization is:
Options
Genes of HLA system.
Viral infections.
Bacterial infections.
Phisical and chemical influences to immune organs.
212. Predisposing factor in pathogenesis of autoimmunization is:
Options
Phisical and chemical influences to immune organs.
Viral infections.
Bacterial infections.
Hormonal disbalance.
213. Predisposing factor in pathogenesis of autoimmunization is:
Options
Phisical and chemical influences to immune organs.
Viral infections
Bacterial infections.
Genetic peculiarities of target cells.
214. Initiated factor in pathogenesis of autoimmunization is:
Options
Genes of HLA system.
Viral infections.
Hormonal disbalance
Decrease of supressive activity of T-lymphocytes.
215. Initiated factor in pathogenesis of autoimmunization is:
Options
Genes of HLA system.
Bacterial infections.
Hormonal disbalance
Decrease of supressive activity of T-lymphocytes.
216. Initiated factor in pathogenesis of autoimmunization is:
Options
Genes of HLA system.
Hormonal disbalance.
Phisical influences to immune organs.
Decrease of supressive activity of T-lymphocytes.
217. Initiated factor in pathogenesis of autoimmunization is:
Options
Genes of HLA system.
Hormonal disbalance.
Chemical influences to immune organs.
Decrease of supressive activity of T-lymphocytes.
218. Basic factor in pathogenesis of autoimmunization is:
Options
Genes of HLA system.
Hormonal disbalance.
Chemical influences to immune organs.
Decrease of supressive activity of T-lymphocytes.
219. Organ specific immune disease is:
Options
Hashimoto thyroiditis.
Rheumatoid arthritis
Systemic Lupus Erythematosus
Scleroderma.
220. Organ specific immune disease is:
Options
Rheumatoid arthritis.
Systemic Lupus Erythematosus.
Scleroderma.
Encephalomyelitis.
221. Organ specific immune disease is:
Options
Rheumatoid arthritis.
Systemic Lupus Erythematosus.
Scleroderma.
Polyneuritis.
222. Organ specific immune disease is:
Options
Rheumatoid arthritis.
Disseminated sclerosis of CNS.
Systemic Lupus Erythematosus.
Scleroderma.
223. Organ specific immune disease is:
Options
Rheumatoid arthritis
Aspermatogenesis.
Systemic Lupus Erythematosus.
Scleroderma.
224. Organ non-specific immune disease is:
Options
Hashimoto thyroiditis.
Rheumatoid arthritis.
Encephalomyelitis.
Polyneuritis.
225. Organ non-specific immune disease is:
Options
Hashimoto thyroiditis.
Encephalomyelitis.
Systemic Lupus Erythematosus.
Polyneuritis.
226. Organ non-specific immune disease is:
Options
Scleroderma.
Encephalomyelitis.
Hashimoto thyroiditis.
Polyneuritis.
227. Organ non-specific immune disease is:
Options
Polyneuritis.
Encephalomyelitis.
Hashimoto thyroiditis.
Dermatomyositis.
228. Organ non-specific immune disease is:
Options
Polyneuritis.
Encephalomyelitis.
Hashimoto thyroiditis.
Secondary thrombocytopenia.
229. Autoimmune disease of intermediate type is:
Options
Myastenia gravis.
Systemic Lupus Erythematosus.
Scleroderma.
Polyneuritis.
230. Autoimmune disease of intermediate type is:
Options
Hashimoto thyroiditis.
Diabetes mellitus-type I.
Scleroderma.
Polyneuritis.
231. Autoimmune disease of intermediate type is:
Options
Hashimoto thyroiditis.
Scleroderma.
Thyrotoxicosis.
Polyneuritis.
232. Autoimmune disease of intermediate type is:
Options
Hashimoto thyroiditis.
Scleroderma.
Polyneuritis.
Sjogren’s syndrome.
233. Autoimmune disease of intermediate type is:
Options
Hashimoto thyroiditis.
Scleroderma.
Polyneuritis.
Goodpasture’s syndrome.
234. Variant of primary immunodeficiency syndrome occurs:
Options
In leukemia.
Under radial therapy.
In aplasia of thymus.
In infections.
235. Variant of primary immunodeficiency syndrome occurs:
Options
In leukemia.
Under radial therapy.
In sarcoidosis.
In hypoplasia of thymus.
236. Variant of secondary immunodeficiency syndrome occurs in all the following diseases, EXCEPT:
Options
Leukemia.
Thymoma.
Sarcoidosis.
Hypoplasia of thymus.
237. A leiyoma of the uterus is considered benign or malignant?
Options
Benign
Malignant
Neither
Tumor-like
238. Is a lipoma considered to be malignant or benign?
Options
Malignant
Benign
Tumor-like
Hamartoma
239. Is a melanoma considered to be malignant or benign?
Options
Malignant
Benign
Tumor-like
Hamartoma
240. Is a lymphoma considered to be malignant or benign?
Options
Malignant
Benign
Tumor-like
Hamartoma
241. Is a rhabdomyosarcoma considered to be malignant or benign?
Options
Malignant
Benign
Tumor-like
Hamartoma
242. Is a adenoma considered to be malignant or benign?
Options
Malignant
Benign
Tumor-like
Hamartoma
243. Is a leukemia considered to be malignant or benign?
Options
Malignant
Benign
Tumor-like
Hamartoma
244. Benign tumor of cartilage is called an:
Options
Osteoma
Chondroma
Leiomyoma
Lipoma
245. Benign tumor of osseous tissue is called an:
Options
Osteoma
Chondroma
Leiomyoma
Lipoma
246. Benign tumor in smooth muscle is called a:
Options
Osteoma
Leiomyoma
Lipoma
Chondroma
247. Benign tumor in fat tissue is called a:
Options
Osteoma
Lipoma
Leimyoma
Chondroma
248. Malignant tumor in fibrous tissue is called a:
Options
Liposarcoma
Osteosarcoma
Fibrosarcoma
Rhabdomyosarcoma
249. Malignant tumor in fat tissue is called a:
Options
Fibrosarcoma
Liposarcoma
Osteosarcoma
Rhabdomyosarcoma
250. Malignant tumor in bone tissue is called a:
Options
Fibrosarcoma
Rhabdomyosarcoma
Osteosarcoma
Liposarcoma
251. Malignant tumor in skeletal muscle is called a:
Options
Fibrosarcoma
Liposarcoma
Osteosarcoma
Rhabdomyosarcoma
252. What is a benign tumor called that is derived from all 3 germ cell layers?
Options
Myoma
Fibroma
Teratoma
Lymphoma
253. Most common location of teratoma?
Options
Uterus
Ovaries
Kidney
Liver
254. Gross specimen of teratoma may contain?
Options
Teeth
Hair
Cartilage
All of the above
255. Which brain structure may be a site for teratoma formation?
Options
Right hemispheire
Pineal gland
Medulla
Pons
256. Which of the following is the tumor-like condition?
Options
Choristoma
Chondroma
Hepatoma
Melanoma
257. What is the definition of choristoma?
Options
Heterotopic rest
Bening tumor of ovaries
Benign tumor of liver
Benign tumor of cartilage
258. Most common malignant bone tumor is?
Options
Osteogenic sarcoma
Chondrosarcoma
Giant cell tumor
Synovioma
259. Malignant tumor originating from plasma cells?
Options
Multiple myeloma
Chondroma
Fibroma
Lieomyoma
260. Benign tumor is:
Options
Noninvasive
Invasive
Able to spread
Metastasize
261. Mixed tumor is found in the following organ:
Options
Salivary gland
Liver
Kidney
Parathyroid
262. Seminoma arising from testicles are:
Options
Benign
Malignant
Tumor-like
Hamartoma
263. Hamartoma is defined as:
Options
Heperotopic rest
Tumor originating from indigenous site
Malignant tumor
Benign tumor
264. Malignant tumor from connective tissues are called:
Options
Adenomas
Papillomas
Carcinomas
Sarcomas
265. Malignant tumor from connective tissues commonly located in:
Options
Brain
Heart
Liver
Lower extremity
266. Tumor arising from all 3 germ cell layers are called:
Options
Choristoma
Hamartoma
Teratoma
Papilloma
267. Tumor containing teeth, hair, cartilage, and adipose tissue are called:
Options
Choristoma
Hamartoma
Teratoma
Papilloma
268. Hamartoma refers to:
Options
Tumour differentiating towards more than one cell line
Tumour arising from totipotent cells
Mass of disorganised but mature cells indigenous to the part
Mass of ectopic rests of normal tissue
269. Increased number of normal mitoses may be present in the following tissues except:
Options
Bone marrow cells
Nails
Hepatocytes
Intestinal epithelium
270. A tumour is termed medullary when it is almost entirely composed of:
Options
Amyloid stroma
Large areas of necrosis
Abundant lymphoid tissue
Parenchymal cells
271. All the following malignant tumours metastasise except:
Options
Synovial sarcoma
Malignant mesothelioma
Glioma
Neuroblastoma
272. The following malignant tumours frequently spread through haematogenous route except:
Options
Bronchogenic carcinoma
Renal cell carcinoma
Follicular carcinoma thyroid
Seminoma testis
273. Degradation of ECM is brought about by the following except:
Options
Proteases
Metalloproteinases
Free radicals
Cathepsin D
274. Grading of tumours depends upon the following except:
Options
Degree of anaplasia
Metastatic spread
Rate of growth of cells
Degree of differentiation
275. Patients of xeroderma pigmentosum are prone to develop the following cancers except:
Options
Basal cell carcinoma
Sweat gland carcinoma
Malignant melanoma
Squamous cell carcinoma
276. The primary target of reactive electrophiles is as under:
Options
Cytochrome P-450
RNA
DNA
Mitochondria
277. Carcinogenic influence of radiation appears after:
Options
< 2 years
2-5 years
5-10 years
> 10 years
278. The following hereditary diseases have higher incidence of cancers due to inherited defect in DNA
repair mechanism except:
Options
Ataxia telangiectasia
Xeroderma pigmentosum
Familial polyposis coli
Bloom’s syndrome
279. The following form of ionising radiation exposure is associated with highest risk of cancer:
Options
A-rays
B-rays
G-rays
X-rays
280. Women receiving oestrogen therapy have an increased risk of developing the following cancers
except:
Options
Breast cancer
Endometrial carcinoma
Gallbladder cancer
Hepatocellular carcinoma
281. Important cyclins in cell cycle include the following except:
Options
Cyclin A
Cyclin B
Cyclin C
Cyclin D
282. Bittner milk factor is a transmissible agent belonging to the following category:
Options
Acute transforming virus
Slow transforming virus
HTLV-I
HTLV-II
283. Important examples of tumour suppressor genes implicated in human cancers include the following
except:
Options
RB gene
TP53
APC
ERB-B
284. An example of tumour-associated antigen (TAA) is:
Options
Testis specific antigen (MAGE)
Alpha-fetoprotein (AFP)
Carcinoembryonic antigen (CEA)
Prostate specific antigen (PSA)
285. Hypercalcaemia as a paraneoplastic syndrome is observed in the following tumours except:
Options
Squamous cell carcinoma lung
Small cell carcinoma lung
Renal cell carcinoma
Breast cancer
286. Lymphocytic infiltrate is frequently present in the following tumours indicative of host immune
response except:
Options
Seminoma testis
Medullary carcinoma breast
Papillary carcinoma thyroid
Malignant melanoma
287. The following antibody-stain is used in immunohistochemistry to identify epithelial cells:
Options
Desmin
Vimentin
Cytokeratin
Neurofilaments
288. Which of the following viral infection is not known to produce any human tumour?
Options
Polyoma virus
EBV
HSV
HTLV
289. All are autosomal dominant inherited cancer syndromes except:
Options
Retinoblastoma
Xeroderma pigmentosum
HNPCC
Neurofibromatosis
290. Phosphorylation of retinoblastoma gene:
Options
Inhibits cell replication
Promotes cellular quiescence
Stops cell cycle progression
Promotes cell division
291. P53:
Options
Activates cyclins
Activates BAX
Activates CDKs
Activates bcl2
292. All are matrix metalloproteinases except:
Options
Collagenase
Gelatinase
Stromelysin
Elastase
293. All are anti-angiogenesis factors except:
Options
Thrombospondin-1
Basic fibroblast growth factor (bFGF)
Endostatin
Angiostatin
294. Which of the following is a test for mutagenicity?
Options
Kveim’s test
Ame’s test
Schilling’s test
Mantoux test
295. All are autosomal dominant inherited cancer syndromes except:
Options
Retinoblastoma
Xeroderma pigmentosum
HNPCC
Neurofibromatosis
296. DNA extraction is a pre-requisite for the following molecular techniques except:
Options
PCR technique
In situ hybridisation
Western blot technique
Southern blot technique
297. All are methods of cell proliferation analysis except:
Options
Microspectrophotometry
Flow cytometry
PCR
Immunohistochemistry
298. A 54-year-old woman who has been diagnosed with early-stage breast cancer undergoes surgery for
a lumpectomy to remove a small tumor detected by mammography. The pathology report confirms the
early stage of the cancer and further comments on the fact that there is significant desmoplasia in the
surrounding tissue. The term desmoplasia refers to
Options
An irregular accumulation of blood vessels.
Maturation and spatial arrangement of cells.
Metastatic involvement of surrounding tissue.
Proliferation of non-neoplastic fibrous connective tissue.
299. A 24-year-old woman with a history of heavy and painful menstrual periods has been having
difficulty conceiving despite months of trying to become pregnant. Further workup includes a bimanual
pelvic examination and an ultrasound, which demonstrates a mass in the uterus that is presumed to be a
leiomyoma. This mass is a
Options
Benign tumor of mesenchymal tissue.
Benign tumor of surface epithelium.
Malignant tumor of epithelial tissue.
Malignant tumor of glandular epithelium.
300. A 68-year-old man has a long history of prostate cancer that was metastatic at the time of diagnosis.
Over the past 2 months, he has had significant weight loss, loss of appetite, and loss of energy. His current
spectrum of conditions can be attributed to which of the following?
Options
Platelet-derived growth factor
Fibroblast growth factor
Interleukin-2
Tumor necrosis factor-α
1. A 58-year-old man with a 700-pack-peryear smoking history presents to the emergency department with
shortness of breath and hemoptysis. Portable chest radiography demonstrates a large mass centrally located
within the left lung field. The serum calcium is 13.0 mg/dL (normal 8.5 to 10.2). The metabolic abnormality
described here is likely due to elaboration of which substance?

Options

Adrenocorticotropic hormone–like substance

Antidiuretic hormone

Carcinoembryonic antigen

Parathyroid-related hormone

2. An 8-year-old boy is referred to the dermatologist for numerous “suspicious” pigmented lesions on the face
and neck. Further history reveals that the patient has had difficulty seeing out of his right eye; he is referred
to the ophthalmologist, who diagnoses an ocular melanoma. Based on the patient’s symptoms, the diagnosis
of xeroderma pigmentosum is considered. This condition results from

Options

Aberrant expression of a receptor tyrosine kinase.

An inborn defect in DNA repair.

Chemical carcinogenesis.

DNA viral infection.

3. A 46-year-old woman with prominent splenomegaly presents with a 3-month history of malaise, easy
fatigability, weakness, weight loss, and anorexia. A complete blood count and differential demonstrates a
white blood cell count of 250,000/mm3 (normal 3,000 to 10,000/mm3) with a predominance of myelocytes,
metamyelocytes, band cells, and segmented neutrophils. Cytogenetic analysis is most likely to reveal which
of the following translocations?

Options

T(8;14)

T(9;22)

T(11;22)

T(14;18)
4. An 18-year-old patient presents with renal cell carcinoma. Given that this is typically a tumor of older adults,
what translocation might you expect to find? This translocation is also seen in which mesenchymal
malignancy?

Options

T(X;18), synovial sarcoma

T(X;17), alveolar soft part sarcoma

T(9;22), leiomyosarcoma

T(14;18), leiomyosarcoma

5. A 63-year-old woman discovers a lump in her right breast. Mammography confirms the presence of a
suspicious “lump,” and a needle core biopsy is performed to determine whether the mass is malignant. The
pathology report confirms that the mass is indeed cancerous and that the tissue demonstrates amplification
of the Her-2/neuoncogene. The gene product of Her-2/neu is what kind of protein?

Options

GTPase

GTPase-activating protein

Nuclear transcription factor

Receptor tyrosine kinase

6. A 27-year-old woman has recently been diagnosed with a glioma (a malignant brain tumor). Further family
history reveals that her 4-year-old son has been diagnosed with leukemia and has been undergoing
chemotherapy. In addition, the patient’s mother died at 36 years of age due to metastatic breast cancer. Li-
Fraumeni syndrome is suspected, given the familial clustering of this group of malignancies. The gene
mutated in Li-Fraumeni syndrome normally functions in what capacity?

Options

Activates the GTPase activity of the gene product of the Ras oncogene

Excises ultraviolet light–induced thymidine dimers

Functions as a cytoplasmic tyrosine kinase

Halts the cell cycle if DNA damage is detected

7. An 8-year-old child is evaluated by the pediatrician, who notes what appear to be 10 small café-au-lait spots
on the child’s torso. In addition, on close inspection of the eyes, the presence of Lisch nodules is noted. The
patient is diagnosed with von Recklinghausen neurofibromatosis type 1. The protein that is mutated in this
disorder normally

Options

Activates the GTPase activity of Ras.

Cleaves cellular proteins during apoptosis.

Functions as a regulator of the cell cycle.

Promotes angiogenesis in the growing tumor mass.


8. A 78-year-old Navy veteran with a 600-pack-per-year history of cigarette smoking presents with cancer.
During his military career, he was involved in fireproofing naval combat ships with asbestos insulation. Given
his environmental exposure to both tobacco and asbestos, to which cancer do both of these carcinogens
contribute?

Options

Bladder cancer

Bronchogenic cancer

Cancer of the throat

Esophageal cancer

9. A 40-year-old woman presents with endometrial carcinoma. Her family history reveals that her mother died
of endometrial cancer at age 50, while her 42-year-old brother was recently diagnosed with colon cancer.
You begin to suspect a familial cancer syndrome. What gene is most likely to be mutated in this family?

Options

WT-1

APC

MSH2

P53

10. What does the proliferation of neoplastic cells lead to?

Options

Neoplasia

Tumors

Atrophy

Freckles

11. What is the term that means "new growth?

Options

Anaplasia

Metaplasia

Neoplasia

Hyperplasia

12. What kind of tumors have a limited growth potential and a good outcome?

Options

Malignant

Hypertrophic
Hypotrophic

Benign

13. Who determines the definitive diagnosis of tumors?

Options

Oncologist

Physician

Your mom

Pathologist

14. What kind of tumors resemble the tissue from which they have arisen?

Options

Hypertrophic

Malignant

Benign

Tumor-like

15. What does pleomorphism mean?

Options

Uncontrolled mitosis

Multiple nuclei

Variability in shape and size

Nucleus – cytoplasm ration

16. The cells are different from where they arose from What is a normal N/C ratio?

Options

1:3

1:8

3:6

1:5

17. IN a malignant tumor the N/C ratio most commonly exhibited is:

Options

1:2

1:1

3:6
1:8

18. What is the process called by which cells move from one site to another?

Options

Transportation

Biotransformation

Metastasis

Metrostatic

19. Which of the following is NOT a pathway in which malignant cells spread (metastasize)?

Options

Lymph

Saliva

Blood

None

20. What is an example of lymphatic metastasis?

Options

Renal cell carcinoma

Adrenal adenoma

Breast cancer

Leiyoma

21. What is an example of metastasis occurring as a direct extension of the primary tumor?

Options

Breast cancer developing over a course of 3 months

Leukemia developing at a very young age

Renal cell carcinoma spreading to the adrenal gland

HIV developing into AIDs

22. Is a metastatic malignant melanoma of the vertebra considered to be malignant or benign?

Options

Malignant

Benign

Choristoma-like

Tumor-like
23. What is a metastatic adenocarcinoma of the stomach that specifically goes to the ovary called? (be specific).

Options

Metastatic adenocarcinoma

Melanoma

Krukenberg tumor

Wilson's tumor

24. What would you be worried about if a 45-year-old woman comes into your ED with massive weight gain
(fluid) over a short period of time?

Options

Metastatic ovarian carcinoma

Metastatic vaginal carcinoma

Cervical cancer

Cholecistitis

25. If a tumor is benign and of squamous origin, what would it be called?

Options

Malignant

Adenoma

Papilloma

Carcinoma

26. If a tumor is benign and glandular in origin, what is it called?

Options

Malignant

Adenoma

Carcinoma

Papilloma

27. What is it called when the nucleus are pushed off to one side due to abundant mucin?

Options

Hypertrophy

Benign

Krukenburgs sign

Signet-rings
28. Who does the grading of a tumor?

Options

Physician

Pharmacist

Pathologist

Oncologist

29. What grade is a moderately differentiated tumor?

Options

II

30. Who does the staging of the tumor?

Options

Pathologist

Physician

Oncologist

Dr. Fischione

31. What does TNM stand for?

Options

Tumor size, lymph node, malignancy

Tumor size, leimyoma, malignancy

Tumor shape, lymph node, metastasis

Tumor size, lymph node, metastasis

32. A 40-year-old man has a positive stool guaiac test during a routine physical examination. A colonoscopy is
performed and a 0.9-cm, circumscribed, pedunculated mass on a short stalk is found in the upper rectum.
Which of the following terms best describes this lesion?

Options

Adenoma

Carcinoma

Choristoma

Hamartoma
33. A Pap smear obtained from a 29-year-old woman dur ing a routine health maintenance examination is
abnormal. She is currently asymptomatic. She has a history of multiple sexual partners. Cervical biopsy
specimens are obtained and the microscopic appearance is shown in the figure. Which of the following is the
most likely diagnosis?

Options

Adenocarcinoma

Carcinoma in situ

Dysplasia

Squamous cell carcinoma

34. A 69-year-old woman has experienced increasing mal aise and a 10-kg weight loss over the past year. She
dies of massive pulmonary thromboembolism. The gross appearance of the liver at autopsy is shown in the
figure. Which of the fol lowing best describes the lesions seen in her liver?

Options

Invasive angiosarcoma

Hepatocellular carcinoma

Leukemic infiltration

Metastatic adenocarcinoma

35. A 66-year-old man with chronic cough has an episode of hemoptysis. On physical examination, there are no
abnormal findings. A chest radiograph shows a 6-cm mass in the right lung. A sputum cytologic analysis
shows neoplastic squamous cells. Metastases from his lung lesion are most likely to be found at which of the
following sites?

Options

Cerebral hemisphere

Chest wall muscle

Hilar lymph nodes

Splenic red pulp

36. An epidemiologic study of cancer deaths recorded in the last half of the 20th century is conducted. The
number of deaths for one particular type of cancer had been decreas ing in developed nations, despite the
absence of widespread screening and prevention programs. Which of the following neoplasms was most
likely to be identified by this study?

Options

Cerebral glioma

Gastric adenocarcinoma

Hepatic angiosarcoma

Leukemia
37. An epidemiologic study of cancer deaths recorded in the last half of the 20th century is conducted. The
number of deaths for one particular cancer had increased markedly in developed nations. More than 30% of
cancer deaths in men, and more than 24% of cancer deaths in women, were caused by this neoplasm in
1998. In some nations, prevention strate gies reduced deaths from this cancer. Which of the following
neoplasms was most likely identified by this study?

Options

Cerebral glioma

Bronchogenic carcinoma

Hepatocellular carcinoma

Colonic adenocarcinoma

38. An epidemiologic study analyzes health care benefits of cancer screening techniques applied to persons
more than 50 years of age. Which of the following diagnostic screening techniques used in health care is
most likely to have the great est impact on reduction in cancer deaths in Europe and North America?

Options

Chest radiograph

Mammography

Pap smear

Serum tumor markers

39. A 34-year-old sexually active woman undergoes a rou tine physical examination. There are no abnormal
findings. A Pap smear is obtained as part of the pelvic examination. Cyto logically, the cells obtained on the
smear from the cervix show severe epithelial dysplasia (high-grade squamous intraepithe lial lesion). Which
of the following therapeutic options is most appropriate for this woman?

Options

Antibiotic therapy

Excision

Ovarian removal

Screening of family members

40. A 70-year-old woman reported a 4-month history of a 4-kg weight loss and increasing generalized icterus. On
physi cal examination, she has midepigastric tenderness on palpa tion. An abdominal CT scan shows a 5-cm
mass in the head of the pancreas. Fine-needle aspiration of the mass is performed. On biochemical analysis,
the neoplastic cells show continued activation of cytoplasmic kinases. Which of the following genes is most
likely to be involved in this process?

Options

APC

MYC

P53
RAS

41. A 22-year-old man has a raised, pigmented lesion on his forearm that has increased in size and become more
irregular in color over the past 4 months. Physical examination shows a 0.5 × 1.2 cm black-to-brown
asymmetric lesion with irregular borders. An excisional biopsy specimen shows clusters of pleo morphic
pigmented cells that extend into the reticular dermis. Family history indicates that the patient’s maternal
uncle died from a similar tumor. His grandfather required enucleation of the left eye because of a “dark
brown” retinal mass. Which of the following genes is most likely to have undergone mutation to produce
these findings in this family?

Options

BCL2 (anti-apoptosis gene)

C-MYC (transcription factor gene)

IL2 (growth factor gene)

P16 (cell cycle inhibition)

42. A 3-year-old child has exhibited difficulty with vision in her right eye. On physical examination, there is
leukocoria of the right eye, consistent with a mass in the posterior chamber. MR imaging shows a mass that
nearly fills the globe. The child undergoes enucleation of the right eye. Molecular analysis of the neoplastic
cells indicates absence of both copies of a gene that contributes to control of the cell cycle. Which of the
following genes has most likely undergone mutation in this neoplasm?

Options

BCR-ABL

BCL2

HMSH2

RB

43. A 76-year-old man has experienced abdominal pain for the past year. On physical examination, there is an
epigastric mass. An abdominal CT scan shows a 10-cm mass in the body of the pancreas. A fine-needle
biopsy specimen of this mass shows a moderately differentiated adenocarcinoma. Mutational analysis of the
carcinoma cells shows inactivation of cyclin-dependent kinase inhibitor with loss of growth-suppression.
Regulatory pathways controlled by which of the following genes are most likely altered in this man’s
carcinoma?

Options

BCL2

β-Catenin

MYC

TGF-β

44. A 55-year-old man has had hemoptysis and worsening cough for the past month. On physical examination,
wheezes are auscultated over the right lung posteriorly. A chest radiograph shows a 6-cm right perihilar
mass. A fine-needle aspiration biop sy is performed and yields cells with the microscopic appearance of non–
small cell bronchogenic carcinoma. Molecular analysis of the neoplastic cells shows a p53 gene mutation.
Which of the following mechanisms has most likely produced the neoplastic transformation?
Options

Inability to hydrolyze GTP

Growth factor receptor activation

Loss of cell cycle arrest

Microsatellite instability

45. A 26-year-old man with a family history of colon carci noma undergoes a surveillance colonoscopy. It reveals
hun dreds of polyps in the colon, and two focal 0.5-cm ulcerated areas. A biopsy specimen from an ulcer
reveals irregularly shaped glands that have penetrated into the muscular layer. Which of the following
molecular events is believed to occur very early in the evolution of his colonic disease process?

Options

Activation of the WNT signaling pathway

Inability to hydrolyze GTP-bound RAS

Loss of heterozygosity affecting the p53 gene

Mutations in mismatch repair genes.

46. A 63-year-old man has a cough with hemoptysis for 10 days. He has a 65 pack-year history of smoking. A
chest CT scan shows a 5-cm right hilar mass. Bronchoscopy is per formed, and lung biopsy specimens show
small cell anaplastic lung carcinoma. His family history shows three first-degree maternal relatives who
developed leukemia, sarcoma, and carcinoma before age 40 years. Which of the following gene products is
most likely to have been altered by mutation to produce these findings?

Options

APC (tumor suppressor)

BCL2 (anti-apoptosis)

K-RAS (GTP binding)

P53 (DNA damage response)

47. A 30-year-old man has a 15-year history of increasing numbers of benign skin nodules. On physical
examination, the firm, nontender, subcutaneous nodules average 0.5 to 1 cm. Further examination shows
numerous oval 1- to 5- cm flat, light brown skin macules. Ophthalmoscopic examination shows
hamartomatous nodules on the iris. A biopsy specimen of one skin nodule shows that it is attached to a
peripheral nerve. Which of the following molecular abnormalities is most likely related to his clinical
presentation?

Options

Decreased susceptibility to apoptosis

Impaired functioning of mismatch repair

Increased production of epidermal growth factor

Persistent activation of the RAS gene


48. A 53-year-old man diagnosed with oral cancer and treated with radiation and chemotherapy 1 year ago now
has a positron emission tomography (PET) scan of his neck that shows a single focus of increased uptake.
This focus is resected and microscopic examination shows that it is a metastasis. Mo lecular analysis of this
cancer shows p53, PTEN, and c-MYC gene mutations. Which of the following metabolic pathways is most
likely up-regulated to promote his cancer cell survival and proliferation?

Options

Aerobic glycolysis

Gluconeogenesis

Hexose monophosphate shunt

Oxidative phosphorylation

49. An experiment involving carcinoma cells grown in cul ture studies the antitumor surveillance effects of the
innate immune system. These carcinoma cells fail to express MHC class I antigens. It is observed, however,
that carcinoma cells are lysed when an immune cell that has been activated by IL-2 is added to the culture.
Which of the following immune cells is most likely to function in this manner?

Options

CD4+ lymphocyte

CD8+ lymphocyte

Macrophage

NK cell

50. A 33-year-old man has experienced occasional head aches for the past 3 months. He suddenly has a
generalized seizure. CT scan of the head shows a periventricular 3-cm mass in the region of the right
thalamus. A stereotactic biop sy of the mass yields large lymphoid cells positive for B cell markers. Which of
the following underlying diseases is most likely to be found in this patient?

Options

Diabetes mellitus

HIV infection

Hypertension

Multiple sclerosis

51. A 40-year-old man has a history of intravenous drug use. Physical examination shows needle tracks in his left
an tecubital fossa. He has mild scleral icterus. Serologic studies for HBsAg and anti-HCV are positive. He
develops hepatocel lular carcinoma 15 years later. Which of the following viral characteristics best explains
why this patient developed hepa tocellular carcinoma?

Options

Viral integration in the vicinity of proto-oncogenes

Viral capture of proto-oncogenes from host cellular DNA

Viral inflammatory changes with genomic damage


Viral inactivation of RB and p53 gene expression

52. A 61-year-old man with a history of chronic viral hep atitis has noted a 6-kg weight loss over the past 5
months. Physical examination shows no masses or palpable lymphade nopathy. An abdominal CT scan shows
a nodular liver with a 10-cm mass in the right lobe. A stool guaiac test result is nega tive. An elevation in
which of the following laboratory tests is most likely to be present in this man?

Options

Alpha-fetoprotein

CA-19-9

Calcitonin

Carcinoembryonic antigen

53. A 59-year-old man has noticed blood in his urine for the past week. Cystoscopy shows a 4-cm exophytic mass
involving the right bladder mucosa near the trigone. After biopsy specimens are obtained, he undergoes a
radical cystectomy. Examination of the excised specimen shows an anaplastic car cinoma that has infiltrated
the bladder wall. Which of the fol lowing techniques applied to the cells from his neoplasm is most likely to
categorize the cell of origin?

Options

Chromosomal karyotyping

Cytologic smear

DNA microarray

Immunohistochemistry

54. A 69-year-old man has noted a chronic cough for the past 3 months. On physical examination, there is mild
stridor on inspiration over the right lung. A chest radiograph shows a 5-cm right hilar lung mass, and a fine-
needle aspiration biopsy specimen of the mass shows cells consistent with squamous cell carcinoma. If
staging of this neoplasm is denoted as T2N1M1, which of the following findings is most likely in this man?

Options

Brain metastases

Elevated corticotropin level

Infiltration of the chest wall

Obstruction of a mainstem bronchus

55. A 44-year-old woman notes a lump in her left breast while taking a shower. The nurse practitioner palpates a
3 cm firm, irregular, non-movable mass in the upper outer quadrant of her left breast on physical
examination. A fine needle aspiration of this mass is performed, and cytologically the cells are consistent
with infiltrating ductal carcinoma. The mass is removed with lumpectomy along with an axillary lymph node
dissection. Which of the following findings will best predict a better prognosis for this patient?

Options

Tumor cells strongly estrogen receptor positive

No metastases in the sampled lymph nodes


Flow cytometric analysis with aneuploidy and a high S-phase

One relative who had a similar type of breast cancer

56. A change in bowel habits prompts a 53-year-old woman to see her physician. On physical examination there
are no lesions noted on digital rectal examination, but her stool is positive for occult blood. A colonoscopy is
performed and reveals a 6 cm friable exophytyic mass in the cecum. A biopsy of this mass is performed and
microscopic examination shows a moderately differentiated adenocarcinoma. Which of the following
laboratory findings is most likely to be present in this patient?

Options

K-RAS mutation in the neoplastic cells

Neoplastic cells positive for vimentin

Stool culture with Shigella flexneri

Presence of HIV-1 RNA

57. An experiment is conducted in which proliferating cells are subjected to ionizing radiation. The ionizing
radiation leads to arrest in a checkpoint that monitors completion of DNA replication. It is observed that
there are increased numbers of chromosomal abnormalities in these cells. Which of the following is the
checkpoint affected by the ionizing radiation?

Options

G0/G1

G1/S

S/G2

G2/M

58. A clinical study is performed to determine the incidence of cancers in different countries. The data show that
persons born in Japan and continuing to reside there have an increased risk for cancer. Which of the
following cancers is most likely seen with increased frequency in this population?

Options

Breast

Colon

Lung

Stomach

59. A 48-year-old woman has a routine physical examination. A 4 cm diameter non-tender mass is palpated in
her right breast. The mass appears fixed to the chest wall. Another 2 cm non-tender mass is palpable in the
left axilla. A chest radiograph reveals multiple 0.5 to 2 cm nodules in both lungs. Which of the following TNM
classifications best indicates the stage of her disease?

Options

T1 N1 M0

T1 N0 M1
T2 N1 M0

T4 N1 M1

60. A study is performed to analyze characteristics of malignant neoplasms in biopsy specimens. The biopsies
were performed on patients who had palpable mass lesions on digital rectal examination. Of the following
microscopic findings, which is most likely to indicate that the neoplasm is malignant?

Options

Pleomorphism

Atypia

Invasion

Increased nuclear/cytoplasmic ratio

61. A child is born with a single functional allele of a tumor suppressor gene. At the age of five the remaining
normal allele is lost through a point mutation. As a result, the ability to inhibit cell cycle progression until the
cell is ready to divide is lost. Which of the following neoplasms is most likely to arise via this mechanism?

Options

Breast ductal carcinoma

Pulmonary small cell anaplastic carcinoma

Ocular retinoblastoma

Cerebral astrocytoma

62. A 50-year-old man has felt vague abdominal discomfort for the past 4 months. On physical examination he
has no lymphadenopathy, and no abdominal masses or organomegaly can be palpated. Bowel sounds are
present. An abdominal CT scan shows a 20 cm retroperitoneal soft tissue mass obscuring the left psoas
muscle. A stool specimen tested for occult blood is negative. Which of the following neoplasms is this man
most likely to have?

Options

Melanoma

Hamartoma

Adenocarcinoma

Liposarcoma

63. A clinical study is performed of oncogenesis in human neoplasms. It is observed that some neoplasms
appear to develop from viral oncogenesis, with serologic confirmation of past viral infection. Which of the
following neoplasms is most likely to arise in this manner?

Options

Retinoblastoma

Small cell anaplastic carcinoma

T-cell leukemia
Prostatic adenocarcinoma

64. An experiment is designed to study the genetics of cancer. The study will link the appearance of cancer to
specific gene abnormalities. Which of the following forms of cell molecular analysis is most useful to identify
gene alterations involved in carcinogenesis?

Options

Florescence in situ hybridization

Flow cytometry

Immunohistochemistry

Single nucleotide polymorphisms

65. A 14-year-old healthy girl has a 0.3 cm reddish, slightly raised nodule on the skin of the upper part of her
chest found on a routine physical examination. She states that this lesion has been present for years and has
not appreciably changed in size or color. Which of the following neoplasms is this nodule most likely to be?

Options

Hemangioma

Melanoma

Carcinoma

Lymphoma

66. A 60-year-old man who has a 90 pack year history of cigarette smoking has had a chronic cough for the past
10 years. He has begun to lose weight (3 kg) during the past year. No abnormal findings are noted on
physical examination. He has a chest radiograph that reveals a right hilar mass. A sputum cytology shows
atypical, hyperchromatic squamous cells. What is the most common initial pathway for metastases from this
lesion?

Options

Bloodstream

Pleural cavity

Contiguous spread to chest wall

Lymphatics

67. A 55-year-old man has had malaise and a 4 kg weight loss over the past 6 months. On physical examination
his stool is positive for occult blood. An abdominal CT scan shows his liver contains multiple tumor masses
from 2 to 5 cm in size with central necrosis. The surrounding hepatic parenchyma appears normal. Which of
the following characteristics of neoplasia is best illustrated by these findings?

Options

Multicentric origin

High tumor grade

Primary neoplasm in the stomach


Advanced stage

68. A 59-year-old man has had a worsening cough with chest pain for the past 6 months. On physical
examination he has no remarkable findings. A chest x-ray shows a 3 cm left lung mass. A sputum cytology
specimen yields cells diagnosed as a squamous cell carcinoma. A mediastinoscopy is performed and reveals
metastases in a lymph node. He is given radiation therapy, and the mass diminishes in size. Which of the
following cellular mechanisms is most likely to account for this tumor response?

Options

Point mutations in DNA

Generation of free radicals

Loss of the blood supply

Secondary inflammation

69. A 61-year-old woman has a firm mass with irregular borders felt in her left breast on a routine physical
examination. A fine needle aspiration is performed and microscopic examination shows malignant cells. A
left mastectomy with axillary lymph node dissection is performed. A tissue sample of this neoplasm is
submitted for analysis by flow cytometry. Which of the following does flow cytometric analysis most likely
provide?

Options

Analysis of the karyotype

Detection of gene mutations

Determination of aneuploidy

Distinguishing carcinoma from sarcoma

70. A 35-year-old healthy woman had a firm nodule palpable on the dome of the uterus six years ago recorded
on routine physical examination. The nodule has slowly increased in size and now appears to be about twice
the size it was when first discovered. By ultrasound scan it is solid and circumscribed. She remains
asymptomatic. Which of the following neoplasms is she most likely to have?

Options

Adenocarcinoma

Leiomyosarcoma

Hemangioma

Leiomyoma

71. A 27-year-old woman in excellent health has a routine health maintenance examination. A 2 cm firm,
rounded mass is palpable beneath the skin of the left forearm. She has no difficulty using the arm and there
is no associated pain with the mass, either in movement or on palpation. The overlying skin appears normal.
The mass does not change in size over the next year. Which of the following neoplasms is she most likely to
have?

Options

Metastatic carcinoma
Melanoma

Rhabdomyosarcoma

Lipoma

72. A 45-year-old woman has noted a lump on her left shoulder that has enlarged over the past 4 months. On
physical examination there is a palpable non-tender supraclavicular lymph node. A biopsy of the node is
performed and on microscopic examination there is a metastatic neoplasm. Which of the following is the
most likely primary for this neoplasm?

Options

Cerebral glioma

Adenocarcinoma of the stomach

Fibroadenoma of the breast

Liposarcoma of the retroperitoneum

73. A 52-year-old woman feels a lump in her right breast. On physical examination there is a 3 cm right breast
mass fixed to the chest wall. This mass is biopsied and on microscopic examination shows nests of cells with
marked hyperchromatism and pleomorphism. These cells are estrogen receptor positive. Flow cytometry is
performed. Compared with surrounding non-neoplastic stromal cells, the neoplastic cells are more likely to
be in which of the following phases of the cell cycle?

Options

G0

G1

G2

74. In an experiment, it is observed that chronic, increased exposure to ionizing radiation results in damage to
cellular DNA. As a consequence, a protein is now absent that would arrest the cell in the G1 phase of the cell
cycle. Subsequent to this, the cell is transformed to acquire the property of unregulated growth. The absent
protein is most likely the product of which of the following genes?

Options

RAS

TP53

MYC

ABL

75. An epidemiologic study is performed to find risk factors for development of malignant neoplasms. A
statistical analysis of pre-existing medical conditions is done. Some pre-existing chronic medical conditions
are observed to precede development of malignant neoplasms, while others do not. Which of the following
conditions is most likely to be statistically related to development of a malignancy?

Options
Essential hypertension

Coronary artery disease

Chronic bronchitis

Ulcerative colitis

76. A 35-year-old man has noted several 1 to 2 cm reddish purple, nodular lesions present on the skin of his
right arm which have increased in size and number over the past 3 months. The lesions do not itch and are
not painful. He has had a watery diarrhea for the past month. On physical examination he has generalized
lymphadenopathy and oral thrush. Which of the following infections is most likely to be related to the
appearance of these skin lesions?

Options

Candida albicans

Human herpesvirus 8

Mycobacterium tuberculosis

Pseudomonas aeruginosa

77. A 44-year-old woman who has had multiple sexual partners for the past 30 years has an abnormal Pap smear
with cytologic changes suggesting human papillomavirus (HPV) infection. Without treatment, she is most
likely to develop which of the following lesions?

Options

Squamous cell carcinoma

Non-Hodgkin's lymphoma

Kaposi sarcoma

Adenocarcinoma

78. A healthy 22-year-old woman undergoes a routine physical examination. A discrete, firm, rubbery, movable
mass is found in the left breast. She has no axillary lymphadenopathy. The skin overlying the breast and the
nipple appear normal. Which of the following neoplasms is most likely to be present in this woman?

Options

Lipoma

Intraductal carcinoma

Malignant lymphoma

Fibroadenoma

79. An epidemiologic study is performed involving patients of East Asian ancestry with long-standing Epstein-
Barr virus (EBV) infection. It is observed that these patients have an increased risk for development of
malignant neoplasms in adulthood. Which of the following neoplasms is most likely to be found in these
patients?

Options
Kaposi sarcoma of skin

Small cell anaplastic carcinoma of lung

Osteosarcoma of bone

Nasopharyngeal carcinoma

80. An experiment is conducted in which proliferating cells are subjected to ionizing radiation. The ionizing
radiation leads to arrest in a checkpoint that monitors completion of DNA replication. It is observed that
there are increased numbers of chromosomal abnormalities in these cells. Which of the following is the
checkpoint affected by the ionizing radiation?

Options

G0/G1

G1/S

S/G2

G2/M

81. A clinical study is performed to determine the incidence of cancers in different countries. The data show that
persons born in Japan and continuing to reside there have an increased risk for cancer. Which of the
following cancers is most likely seen with increased frequency in this population?

Options

Breast

Colon

Lung

Stomach

82. A 25-year-old man presents 1 week after discovering that his left testicle is twice the normal size. Physical
examination reveals a nontender, testicular mass that cannot be transilluminated. Serum levels of alpha-
fetoprotein and human chorionic gonadotropin are normal. A hemiorchiectomy is performed, and histologic
examination of the surgical specimen shows embryonal carcinoma. Compared to normal adult somatic cells,
this germ cell neoplasm would most likely show high levels of expression of which of the following proteins?

Options

Desmin

Dystrophin

Cytochrome c

Telomerase

83. A 62-year-old woman presents with a breast lump that she discovered 6 days ago. A breast biopsy shows
lobular carcinoma in situ. Compared to normal epithelial cells of the breast lobule, these malignant cells
would most likely show decreased expression of which of the following proteins?

Options
Desmin

E-cadherin

Lysyl hydroxylase

P selectin

84. An 80-year-old man complains of lower abdominal pain, increasing weakness, and fatigue. He has lost 16 lb
(7.3 kg) in the past 6 months. The prostate-specifi c antigen test is elevated (8.5 ng/mL). Rectal examination
reveals an enlarged and nodular prostate. A needle biopsy of the prostate discloses invasive prostatic
adenocarcinoma. Histologic grading of this patient’s carcinoma is based primarily on which of the following
criteria?

Options

Capsular involvement

Extent of regional lymph nodes involvement

Pulmonary metastases

Resemblance to normal tissue of origin

85. A 65-year-old man complains of muscle weakness and a dry cough for 4 months. He has smoked two packs
of cigarettes daily for 45 years. A chest X-ray shows a 4-cm central, left lung mass. Laboratory studies reveal
hyperglycemia and hypertension. A transbronchial biopsy is diagnosed as small cell carcinoma. Metastases
to the liver are detected by CT scan. Which of the following might account for the development of
hyperglycemia and hypertension in this patient?

Options

Adrenal metastases

Paraneoplastic syndrome

Pituitary adenoma

Pituitary metastases

86. Which of the following potent carcinogens was most likely involved in the pathogenesis of lung cancer in the
patient described in Question 8?

Options

Afl atoxin B1

Asbestos

Azo dyes

Polycyclic aromatic hydrocarbons

87. A 33-year-old woman discovers a lump in her left breast on self-examination. Her mother and sister both
had breast cancer. A mammogram demonstrates an ill-defined density in the outer quadrant of the left
breast, with microcalcifications. Needle aspiration reveals the presence of malignant, ductalepithelial cells.
Genetic screening identifies a mutation inBRCA1. In addition to cell cycle control, BRCA1 protein promotes
which of the following cellular functions?
Options

Apoptosis

Cell adhesion

DNA repair

Gene transcription

88. A 60-year-old man who worked for 30 years in a chemical factory complains of blood in his urine. Urine
cytology discloses dysplastic cells. A bladder biopsy demonstrates transitional cell carcinoma. Which of the
following carcinogens was most likely involved in the pathogenesis of bladder cancer in this patient?

Options

Aniline dyes

Arsenic

Benzene

Cisplatinum

89. A 45-year-old man presents with a 9-month history of a reddish nodule on his foot. Biopsy of the nodule
discloses a poorly demarcated lesion composed of fi broblasts and endothelial-like cells lining vascular
spaces. Further work-up identifies similar lesions in the lymph nodes and liver. The tumor cells contain
sequences of human herpesvirus-8 (HHV-8). This patient most likely has which of the following diseases?

Options

Acquired immunodefi ciency

Ataxia telangiectasia

Li-Fraumeni syndrome

Neurofi bromatosis type I

90. During a routine checkup, a 50-year-old man is found to have blood in his urine. He is otherwise in excellent
health. An abdominal CT scan reveals a 2-cm right renal mass. You inform the patient that staging of this
tumor is key to selecting treatment and evaluating prognosis. Which of the following is the most important
staging factor for this patient?

Options

Histologic grade of the tumor

Metastases to regional lymph nodes

Proliferative capacity of the tumor cells

Somatic mutations in the p53 tumor suppressor gene

91. A 58-year-old woman with colon cancer presents with 3 months of increasing shortness of breath. A chest X-
ray reveals numerous, bilateral, round masses in both lungs. Histologic examination of an open-lung biopsy
discloses malignant gland-like structures, which are nearly identical to the colon primary. Which of the
following changes in cell behavior was the fi rst step in the process leading to tumor metastasis from the
colon to the lung in this patient?
Options

Arrest within the circulating blood or lymph

Exit from the circulation into a new tissue

Invasion of the underlying basement membrane

Penetration of vascular or lymphatic channels

92. A 68-year-old man complains of recent changes in bowel habits and blood-tinged stools. Colonoscopy
reveals a 3- cm mass in the sigmoid colon. Biopsy of the mass shows infi ltrating malignant glands. These
neoplastic cells have most likely acquired a set of mutations that cause which of the following changes in cell
behavior?

Options

Decreased cellular motility

Enhanced stem cell differentiation

Increased cell-cell adhesion

Loss of cell cycle restriction point control

93. A 35-year-old woman complains of nipple discharge and irregular menses of 5 months duration. Physical
examination reveals a milky discharge from both nipples. MRI shows an enlargement of the anterior
pituitary. Which of the following is the most likely histologic diagnosis of this patient’s pituitary tumor?

Options

Adenoma

Choristoma

Hamartoma

Papilloma

94. A 52-year-old woman presents with a 1-year history of upper truncal obesity and moderate depression.
Physical examination shows hirsutism and moon facies. A CT scan of the thorax displays a hilar mass. A
transbronchial lung biopsy discloses small cell carcinoma. Electron microscopy of this patient’s lung tumor
will most likely reveal which of the following cytologic features?

Options

Councilman bodies

Hyperplasia of endoplasmic reticulum

Mitochondrial calcifi cation

Neuroendocrine granules

95. Cytogenetic studies in a 40-year-old woman with follicular lymphoma demonstrate a t(14;18) chromosomal
translocation involving the bcl-2 gene. Constitutive expression of the protein encoded by the bcl-2 gene
inhibits which of the following processes in this patient’s transformed lymphocytes?

Options
Apoptosis

DNA excision repair

G1-to-S cell cycle progression

Oxidative phosphorylation

96. A 59-year-old woman presents with increasing pigmentation of the skin. Physical examination shows
hyperkeratosis and hyperpigmentation of the axilla, neck, fl exures, and anogenital region. Endocrinologic
studies reveal normal serum levels of adrenal corticosteroids and glucocorticoids. If this patient’s skin
pigmentation represents a paraneoplastic syndrome, the primary tumor would most likely be found in which
of the following anatomic locations?

Options

Bladder

Cervix

Esophagus

Stomach

97. A 65-year-old man dies after a protracted battle with metastatic colon carcinoma. At autopsy, the liver is fi
lled with multiple nodules of cancer, many of which display central necrosis (umbilication). Which of the
following best explains the pathogenesis of tumor umbilication in this patient?

Options

Biphasic tumor

Chronic infl ammation

Granulomatous infl ammation

Ischemia and infarction

98. A 59-year-old man complains of progressive weakness. He reports that his stools are very dark. Physical
examination demonstrates fullness in the right lower quadrant. Laboratory studies show iron defi ciency
anemia, with a serum hemoglobin level of 7.4 g/dL. Stool specimens are positive for occult blood.
Colonoscopy discloses an ulcerating lesion of the cecum. Which of the following serum tumor markers is
most likely to be useful for following this patient after surgery?

Options

Alpha-fetoprotein

Carcinoembryonic antigen

Chorionic gonadotropin

Chromogranin

99. A 20-year-old woman has an ovarian tumor removed. The surgical specimen is 10 cm in diameter and cystic.
The cystic cavity is found to contain black hair and sebaceous material. Histologic examination of the cyst
wall reveals a variety of benign differentiated tissues, including skin, cartilage, brain, and mucinous glandular
epithelium. What is the diagnosis?
Options

Adenoma

Chondroma

Hamartoma

Teratoma

100. A 42-year-old man presents with upper gastrointestinal bleeding. Upper endoscopy and biopsy
reveal gastric adenocarcinoma. Which country of the world has the highest incidence of this malignant
neoplasm?

Options

Argentina

Canada

Japan

Mexico

101. An 8-year-old girl with numerous hypopigmented, ulcerated, and crusted patches on her face and
forearms develops an indurated, crater-like, skin nodule on the back of her left hand. Biopsy of this skin
nodule discloses a squamous cell carcinoma. Molecular biology studies reveal that this patient has germline
mutations in the gene encoding a nucleotide excision repair enzyme. What is the appropriate diagnosis?

Options

Ataxia telangiectasia

Hereditary albinism

Li-Fraumeni syndrome

Xeroderma pigmentosum

102. A 59-year-old woman complains of “feeling light-headed” and losing 5 kg (11 lb) in the last month. A
CBC reveals a normocytic, normochromic anemia. The patient subsequently dies of metastatic cancer. Based
on current epidemiologic data for cancer-associated mortality in women, which of the following is the most
likely primary site for this patient’s malignant neoplasm?

Options

Brain

Breast

Colon

Lung

103. The parents of a 6-month-old girl palpate a mass on the left side of the child’s abdomen. Urinalysis
shows high levels of vanillylmandelic acid. A CT scan reveals an abdominal tumor and bony metastases. The
primary tumor is surgically resected. Histologic examination of the surgical specimen discloses
neuroblastoma. Evaluation of the Nmyc protooncogene in this child’s tumor will most likely demonstrate
which of the following genetic changes?
Options

Chromosomal translocation

Exon deletion

Expansion of a trinucleotide repeat

Gene amplifi cation

104. A 58-year-old woman undergoes routine colonoscopy. A 2-cm submucosal nodule is identifi ed in the
appendix. Biopsy of the nodule shows nests of cells with round, uniform nuclei. Electron microscopy reveals
numerous neuroendocrine granules in the cytoplasm. This patient’s neoplastic disease is associated with
which of the following clinical features?

Options

Congestive heart failure

Flushing and wheezing

Muscular dystrophy

Progressive systemic sclerosis

105. A 45-year-old woman presents with abdominal pain and vaginal bleeding. A hysterectomy is
performed and shows a benign tumor of the uterus derived from a smooth muscle cell. What is the
appropriate diagnosis?

Options

Angiomyolipoma

Leiomyoma

Leiomyosarcoma

Myxoma

106. Cytogenetic studies in a 70-year-old woman with chronic myelogenous leukemia (CML) demonstrate
a t(9;22) chromosomal translocation. Which of the following best explains the role of this translocation in
the pathogenesis of leukemia in this patient?

Options

Altered DNA methylation status

Enhanced expression of telomerase gene

Expansion of a trinucleotide repeat

Protooncogene activation

107. A 33-year-old woman presents with a diffuse scaly skin rash of 4 weeks duration. Biopsy of lesional
skin reveals a cutaneous T-cell lymphoma (mycosis fungoides). Which of the following immunohistochemical
markers would be most useful for identifying malignant cells in the skin of this patient?

Options

Calcitonin
CD4

Desmin

HMB-45

108. A 63-year-old woman with chronic bronchitis presents with shortness of breath. A chest X-ray
reveals a 2-cm “coin lesion” in the upper lobe of the left lung. A CT-guide lung biopsy is obtained. Which of
the following describes the histologic features of this lesion if the diagnosis is hamartoma?

Options

Benign neoplasm of epithelial origin

Disorganized normal tissue

Ectopic islands of normal tissue

Granulation tissue

109. A 2-year-old boy is found to have bilateral retinal tumors. Molecular studies demonstrate a germline
mutation in one allele of the Rb gene. Which of the following genetic events best explains the mechanism of
carcinogenesis in this patient?

Options

Balanced translocation

Expansion of trinucleotide repeat

Gene amplifi cation

Loss of heterozygosity

110. A 48-year-old nulliparous woman complains that her menstrual blood fl ow is more abundant than
usual. An ultrasound examination reveals a polypoid mass in the uterine fundus. The patient subsequently,
undergoes a hysterectomy, which reveals a poorly differentiated endometrial adenocarcinoma. The
development of this neoplasm was preceded by which of the following histopathologic changes in the
glandular epithelium?

Options

Atrophy

Hydropic swelling

Hyperplasia

Hypertrophy

111. Which type of adaptation is able to convert to dysplasia and further to adenocarcinoma?

Options

Hypertrophy

Atrophy

Metaplasia
Hamartoma

112. Metaplasia of distal end of esophagus leads to which type of cancer?

Options

Adenocarcinoma

Sarcoma

Hamartoma

Choristoma

113. Endometrial hyperplasia may lead to which cancer type?

Options

Adenocarcinoma

Sarcoma

Hamartoma

Choristoma

114. Increase of number of cells is termed:

Options

Hypertrophy

Hyperplasia

Atrophy

Aplasia

115. Change in cell polarity, which is reversible on treatment is -

Options

Metaplasia

Dysplasia

Anaplasia

None

116. Reversible disorder with variability in size, shape and polarity of cells is-

Options

Metaplasia

Dysplasia

Anaplasia

Hyperplasia
117. Hyperplasia is -

Options

Results in increased cell size

Results in increased cell numbers

Occurs in corneal endothelium

Occurs in retinal pigment epithelium

118. The term atrophy implies:

Options

Increase in size and number of cells

Decrease in size of cells

Decrease in number of cells

Increase in size and number of cells

119. True hypertrophy characterized by –

Options

Enlargement of organ due to foreign bodies

Connective tissue growth

Accumulation of fluid in the cavities

Hyperplasia and hypertrophy of cells or organelles

120. In atrophy, the cells are:

Options

Dead cells

Shrunken cells

Irreversibly injured cells

Reversibly injured cells

121. For metaplasia the following holds true:

Options

It is a disordered growth

It affects only epithelial tissues

It is a reversible change

It is an irreversible and progressive change


122. Histologic sections of an enlarged tonsil from a 9-year-old female reveal an increased number of
reactive folliclescontaining germinal centers with proliferating B-lymphocytes. Which one of the listed terms
best describes this pathologic process?

Options

B-lymphocyte hypertrophy

Follicular dysplasia

Follicular hyperplasia

Germinal center atrophy

123. Causes of pathological atrophy are all of the following, except:

Options

Starvation atrophy

Ischemic atrophy

Disuse atrophy

Hypertrophy of smooth muscle

124. Causes of pathological atrophy are all of the following, except:

Options

Starvation atrophy

Ischemic atrophy

Disuse atrophy

Hypertrophy of cardiac muscle

125. Causes of pathological atrophy are all of the following, except:

Options

Starvation atrophy

Ischemic atrophy

Disuse atrophy

Hypertrophy of skeletal muscle

126. Causes of pathological atrophy are all of the following, except:

Options

Starvation atrophy

Ischemic atrophy

Disuse atrophy

Compensatory hypertrophy
127. Causes of pathological atrophy are all of the following, except:

Options

Starvation atrophy

Ischemic atrophy

Disuse atrophy

Enlarged size of the uterus in pregnancy

128. Causes of pathological atrophy are all of the following, except:

Options

Starvation atrophy

Ischemic atrophy

Disuse atrophy

Hormonal hyperplasia

129. Causes of pathological atrophy are all of the following, except:

Options

Starvation atrophy

Ischemic atrophy

Disuse atrophy

Compensatory hyperplasia

130. Causes of pathological atrophy are all of the following, except:

Options

Neuropathic atrophy

Ischemic atrophy

Disuse atrophy

Hypertrophy of smooth muscle

131. Causes of pathological atrophy are all of the following, except:

Options

Neuropathic atrophy

Ischemic atrophy

Disuse atrophy

Hypertrophy of cardiac muscle

132. Causes of pathological atrophy are all of the following, except:


Options

Neuropathic atrophy

Ischemic atrophy

Disuse atrophy

Hypertrophy of skeletal muscle

133. Causes of pathological atrophy are all of the following, except:

Options

Neuropathic atrophy

Ischemic atrophy

Disuse atrophy

Compensatory hypertrophy

134. Causes of pathological atrophy are all of the following, except:

Options

Neuropathic atrophy

Ischemic atrophy

Disuse atrophy

Enlarged size of the uterus in pregnancy

135. Causes of pathological atrophy are all of the following, except:

Options

Neuropathic atrophy

Ischemic atrophy

Disuse atrophy

Hormonal hyperplasia

136. Causes of pathological atrophy are all of the following, except:

Options

Neuropathic atrophy

Ischemic atrophy

Disuse atrophy

Compensatory hyperplasia

137. Causes of pathological atrophy are all of the following, except:

Options
Neuropathic atrophy

Endocrine atrophy

Disuse atrophy

Hypertrophy of smooth muscle

138. Causes of pathological atrophy are all of the following, except:

Options

Neuropathic atrophy

Endocrine atrophy

Disuse atrophy

Hypertrophy of cardiac muscle

139. Causes of pathological atrophy are all of the following, except:

Options

Neuropathic atrophy

Endocrine atrophy

Disuse atrophy

Hypertrophy of skeletal muscle

140. Causes of pathological atrophy are all of the following, except:

Options

Neuropathic atrophy

Endocrine atrophy

Disuse atrophy

Compensatory hypertrophy

141. Causes of pathological atrophy are all of the following, except:

Options

Neuropathic atrophy

Endocrine atrophy

Disuse atrophy

Enlarged size of the uterus in pregnancy

142. Causes of pathological atrophy are all of the following, except:

Options

Neuropathic atrophy
Endocrine atrophy

Disuse atrophy

Hormonal hyperplasia

143. Causes of pathological atrophy are all of the following, except:

Options

Neuropathic atrophy

Endocrine atrophy

Disuse atrophy

Compensatory hyperplasia

144. Causes of pathological atrophy are all of the following, except:

Options

Neuropathic atrophy

Endocrine atrophy

Pressure atrophy

Hypertrophy of smooth muscle

145. Causes of pathological atrophy are all of the following, except:

Options

Neuropathic atrophy

Endocrine atrophy

Pressure atrophy

Hypertrophy of cardiac muscle

146. Causes of pathological atrophy are all of the following, except:

Options

Neuropathic atrophy

Endocrine atrophy

Pressure atrophy

Hypertrophy of skeletal muscle

147. Causes of pathological atrophy are all of the following, except:

Options

Neuropathic atrophy

Endocrine atrophy
Pressure atrophy

Compensatory hypertrophy

148. Causes of pathological atrophy are all of the following, except:

Options

Neuropathic atrophy

Endocrine atrophy

Pressure atrophy

Enlarged size of the uterus in pregnancy

149. Causes of pathological atrophy are all of the following, except:

Options

Neuropathic atrophy

Endocrine atrophy

Pressure atrophy

Hormonal hyperplasia

150. Causes of pathological atrophy are all of the following, except:

Options

Neuropathic atrophy

Endocrine atrophy

Pressure atrophy

Compensatory hyperplasia

151. Causes of pathological atrophy are all of the following, except:

Options

Starvation atrophy

Ischemic atrophy

Disuse atrophy

Autolysis

152. Causes of pathological atrophy are all of the following, except:

Options

Starvation atrophy

Ischemic atrophy

Disuse atrophy
Necrosis

153. Causes of pathological atrophy are all of the following, except:

Options

Starvation atrophy

Ischemic atrophy

Disuse atrophy

Coagulative necrosis

154. Causes of pathological atrophy are all of the following, except:

Options

Starvation atrophy

Ischemic atrophy

Disuse atrophy

Liquefaction necrosis

155. Causes of pathological atrophy are all of the following, except:

Options

Starvation atrophy

Ischemic atrophy

Disuse atrophy

Caseous necrosis

156. Causes of pathological atrophy are all of the following, except:

Options

Starvation atrophy

Ischemic atrophy

Disuse atrophy

Fat necrosis

157. Causes of pathological atrophy are all of the following, except:

Options

Starvation atrophy

Ischemic atrophy

Disuse atrophy

Fibrinoid necrosis
158. Causes of pathological atrophy are all of the following, except:

Options

Neuropathic atrophy

Ischemic atrophy

Disuse atrophy

Gangrene

159. Causes of pathological atrophy are all of the following, except:

Options

Neuropathic atrophy

Ischemic atrophy

Disuse atrophy

Dry gangrene

160. Causes of pathological atrophy are all of the following, except:

Options

Neuropathic atrophy

Ischemic atrophy

Disuse atrophy

Wet gangrene

161. Causes of pathological atrophy are all of the following, except:

Options

Neuropathic atrophy

Ischemic atrophy

Disuse atrophy

Gas gangrene

162. Causes of pathological atrophy are all of the following, except:

Options

Neuropathic atrophy

Ischemic atrophy

Disuse atrophy

Squamous metaplasia in bronchus

163. Causes of pathological atrophy are all of the following, except:


Options

Neuropathic atrophy

Ischemic atrophy

Disuse atrophy

Squamous metaplasia in uterine endocervix

164. Causes of pathological atrophy are all of the following, except:

Options

Neuropathic atrophy

Ischemic atrophy

Disuse atrophy

Squamous metaplasia in gallbladder

165. Causes of pathological atrophy are all of the following, except:

Options

Neuropathic atrophy

Endocrine atrophy

Disuse atrophy

Squamous metaplasia in prostate

166. Causes of pathological atrophy are all of the following, except:

Options

Neuropathic atrophy

Endocrine atrophy

Disuse atrophy

Squamous metaplasia in renal pelvis

167. Causes of pathological atrophy are all of the following, except:

Options

Neuropathic atrophy

Endocrine atrophy

Disuse atrophy

Squamous metaplasia in urinary bladder

168. Causes of pathological atrophy are all of the following, except:

Options
Neuropathic atrophy

Endocrine atrophy

Disuse atrophy

Columnar (interstinal) metaplasia in healed chronic gastric ulcer

169. Causes of pathological atrophy are all of the following, except:

Options

Neuropathic atrophy

Endocrine atrophy

Disuse atrophy

Columnar metaplasia in Barrett's esophagus

170. Causes of pathological atrophy are all of the following, except:

Options

Neuropathic atrophy

Endocrine atrophy

Disuse atrophy

Osseous metaplasia

171. Causes of pathological atrophy are all of the following, except:

Options

Neuropathic atrophy

Endocrine atrophy

Disuse atrophy

Cartilaginous metaplasia

172. Causes of pathological atrophy are all of the following, except:

Options

Neuropathic atrophy

Endocrine atrophy

Pressure atrophy

Dysplasia

173. Causes of pathological atrophy are all of the following, except:

Options

Neuropathic atrophy
Endocrine atrophy

Pressure atrophy

Renal edema in nephrotic syndrome

174. Causes of pathological atrophy are all of the following, except:

Options

Neuropathic atrophy

Endocrine atrophy

Pressure atrophy

Edema in nephritic syndrome

175. Causes of pathological atrophy are all of the following, except:

Options

Neuropathic atrophy

Endocrine atrophy

Pressure atrophy

Edema in acute tubular injury

176. Causes of pathological atrophy are all of the following, except:

Options

Neuropathic atrophy

Endocrine atrophy

Pressure atrophy

Cardiac edema

177. Causes of pathological atrophy are all of the following, except:

Options

Neuropathic atrophy

Endocrine atrophy

Pressure atrophy

Pulmonary edema

178. Causes of pathological atrophy are all of the following, except:

Options

Neuropathic atrophy

Endocrine atrophy
Pressure atrophy

Cerebral edema

179. Causes of pathological hypertrophy are all of the following, except:

Options

Starvation atrophy

Hypertrophy of cardiac muscle in systemic hypertension

Hypertrophy of cardiac muscle in aortic valve disease

Hypertrophy of cardiac muscle in mitral insufficiency

180. Causes of pathological hypertrophy are all of the following, except:

Options

Ischemic atrophy

Hypertrophy of cardiac muscle in systemic hypertension

Hypertrophy of cardiac muscle in aortic valve disease

Hypertrophy of cardiac muscle in mitral insufficiency

181. Causes of pathological hypertrophy are all of the following, except:

Options

Disuse atrophy

Hypertrophy of cardiac muscle in systemic hypertension

Hypertrophy of cardiac muscle in aortic valve disease

Hypertrophy of cardiac muscle in mitral insufficiency

182. Causes of pathological hypertrophy are all of the following, except:

Options

Neuropathic atrophy in poliomyelitis

Hypertrophy of cardiac muscle in systemic hypertension

Hypertrophy of cardiac muscle in aortic valve disease

Hypertrophy of cardiac muscle in mitral insufficiency

183. Causes of pathological hypertrophy are all of the following, except:

Options

Starvation atrophy

Hypertrophy of smooth muscle in cardiac achalasia (in esophagus)

Hypertrophy of cardiac muscle in aortic valve disease


Hypertrophy of cardiac muscle in mitral insufficiency

184. Causes of pathological hypertrophy are all of the following, except:

Options

Ischemic atrophy

Hypertrophy of smooth muscle in cardiac achalasia (in esophagus)

Hypertrophy of cardiac muscle in aortic valve disease

Hypertrophy of cardiac muscle in mitral insufficiency

185. Causes of pathological hypertrophy are all of the following, except:

Options

Disuse atrophy

Hypertrophy of cardiac muscle in systemic hypertension

Hypertrophy of smooth muscle in cardiac achalasia (in esophagus)

Hypertrophy of cardiac muscle in mitral insufficiency

186. Causes of pathological hypertrophy are all of the following, except:

Options

Neuropathic atrophy in poliomyelitis

Hypertrophy of cardiac muscle in systemic hypertension

Hypertrophy of smooth muscle in cardiac achalasia (in esophagus)

Hypertrophy of cardiac muscle in mitral insufficiency

187. Causes of pathological hypertrophy are all of the following, except:

Options

Starvation atrophy

Hypertrophy of smooth muscle in cardiac achalasia (in esophagus)

Hypertrophy of cardiac muscle in aortic valve disease

Hypertrophy of skeletal muscle in athletes and manual labourers

188. Causes of pathological hypertrophy are all of the following, except:

Options

Ischemic atrophy

Hypertrophy of smooth muscle in cardiac achalasia (in esophagus)

Hypertrophy of cardiac muscle in aortic valve disease

Hypertrophy of skeletal muscle in athletes and manual labourers


189. Causes of pathological hypertrophy are all of the following, except:

Options

Disuse atrophy

Hypertrophy of cardiac muscle in systemic hypertension

Hypertrophy of smooth muscle in cardiac achalasia (in esophagus)

Hypertrophy of skeletal muscle in athletes and manual labourers

190. Causes of pathological hypertrophy are all of the following, except:

Options

Neuropathic atrophy in poliomyelitis

Hypertrophy of cardiac muscle in systemic hypertension

Hypertrophy of smooth muscle in cardiac achalasia (in esophagus)

Hypertrophy of skeletal muscle in athletes and manual labourers

191. Causes of pathological hypertrophy are all of the following, except:

Options

Starvation atrophy

Compensatory hypertrophy

Hypertrophy of cardiac muscle in aortic valve disease

Hypertrophy of skeletal muscle in athletes and manual labourers

192. Causes of pathological hypertrophy are all of the following, except:

Options

Ischemic atrophy

Compensatory hypertrophy

Hypertrophy of cardiac muscle in aortic valve disease

Hypertrophy of skeletal muscle in athletes and manual labourers

193. Causes of pathological hypertrophy are all of the following, except:

Options

Disuse atrophy

Compensatory hypertrophy

Hypertrophy of smooth muscle in cardiac achalasia (in esophagus)

Hypertrophy of skeletal muscle in athletes and manual labourers

194. Causes of pathological hypertrophy are all of the following, except:


Options

Neuropathic atrophy in poliomyelitis

Compensatory hypertrophy

Hypertrophy of smooth muscle in cardiac achalasia (in esophagus)

Hypertrophy of skeletal muscle in athletes and manual labourers

195. Causes of pathological hyperplasia are all of the following, except:

Options

Pseudocarcinomatous hyperplasia of the skin

Endometrial hyperplasia following estrogen excess

Hypertrophy of cardiac muscle in systemic hypertension

In wound healing, there is formation of granulation tissue due proliferation of fibroblasts and endothelial cells

196. Causes of pathological hyperplasia are all of the following, except:

Options

Pseudocarcinomatous hyperplasia of the skin

Endometrial hyperplasia following estrogen excess

Hypertrophy of skeletal muscle in athletes and manual labourers

In wound healing, there is formation of granulation tissue due proliferation of fibroblasts and endothelial cells

197. Causes of pathological hyperplasia are all of the following, except:

Options

Pseudocarcinomatous hyperplasia of the skin

Endometrial hyperplasia following estrogen excess

Starvation atrophy

In wound healing, there is formation of granulation tissue due proliferation of fibroblasts and endothelial cells

198. Causes of pathological hyperplasia are all of the following, except:

Options

Pseudocarcinomatous hyperplasia of the skin

Endometrial hyperplasia following estrogen excess

Compensatory hypertrophy

In wound healing, there is formation of granulation tissue due proliferation of fibroblasts and endothelial cells

199. Causes of pathological hyperplasia are all of the following, except:

Options
Pseudocarcinomatous hyperplasia of the skin

Endometrial hyperplasia following estrogen excess

Neuropathic atrophy in poliomyelitis

In wound healing, there is formation of granulation tissue due proliferation of fibroblasts and endothelial cells

200. Causes of pathological hyperplasia are all of the following, except:

Options

Pseudocarcinomatous hyperplasia of the skin

Endometrial hyperplasia following estrogen excess

Hypertrophy of smooth muscle in cardiac achalasia (in esophagus)

In wound healing, there is formation of granulation tissue due proliferation of fibroblasts and endothelial cells

201. Causes of pathological hyperplasia are all of the following, except:

Options

Pseudocarcinomatous hyperplasia of the skin

Endometrial hyperplasia following estrogen excess

Epithelial metaplasia

In wound healing, there is formation of granulation tissue due proliferation of fibroblasts and endothelial cells

202. Causes of pathological hyperplasia are all of the following, except:

Options

Pseudocarcinomatous hyperplasia of the skin

Endometrial hyperplasia following estrogen excess

Mesenchymal metaplasia

In wound healing, there is formation of granulation tissue due proliferation of fibroblasts and endothelial cells

203. Causes of pathological hyperplasia are all of the following, except:

Options

Pseudocarcinomatous hyperplasia of the skin

Endometrial hyperplasia following estrogen excess

Dysplasia

In wound healing, there is formation of granulation tissue due proliferation of fibroblasts and endothelial cells

204. Select the option of cells in chronic inflammation:

Options

Monocytes.
Myelocytes.

Erythrocytes.

Monocytes.

205. Select a specific variant of cells in chronic inflammation:

Options

Plasma cells.

Neutrophils.

Erythrocytes

Myelocytes

206. Find and specify the variant of cells in chronic inflammation:

Options

Neutrophils.

Erythrocytes.

Astrocytes.

Giant cells.

207. Are there any chronic inflammation cells in the list?

Options

Neutrophils.

Erythrocytes.

Mast cells.

Epithelial cells.

208. Select a specific variant of cells in chronic inflammation:

Options

Neutrophils.

Plasma cells.

Erythrocytes.

Myelocytes.

209. Are there any chronic inflammation cells in the list? Underscore:

Options

Neutrophils.

Erythrocytes.
Mast cells.

Epithelial cells.

210. Find and specify the variant of cells in chronic inflammation:

Options

Neutrophils.

Erythrocytes.

Astrocytes.

Giant cells.

211. Cells of chronic inflammation are:

Options

Neutrophils.

Epitelioid cells.

Epithelial cells.

Astrocytes.

212. Are there any chronic inflammation cells in the list

Options

Neutrophils.

Erythrocytes.

Mast cells.

Epithelial cells.

213. Select cells of chronic inflammation in the list:

Options

Fibrolasts.

Osteoblasts.

Chondroblasts.

Lipoblasts.

214. Production of mediators is associated with certain cells:

Options

Lymphocytes.

Plasma cells.

Eosinophils.
Must cells.

215. Which cells are characteristically found in inflammatory sites around animal parasites?

Options

Lymphocytes.

Plasma cells.

Must cells.

Eosinophils.

216. Which cells are central players in anaphylactic shock?

Options

Lymphocytes.

Plasma cells.

Eosinophils.

Must cells.

217. Select the type of cells that produce collagen.

Options

Lymphocytes.

Plasma cells.

Eosinophils.

Fibroblasts.

218. Infectious granulomas occur in the case of:

Options

Tuberculosis.

Rheumatic fever.

Rheumatoid arthritis.

Sarcoidosis

219. Infectious granulomas occur in the case of:

Options

Rheumatic fever

Syphilis.

Rheumatoid arthritis.

Sarcoidosis
220. Infectious granulomas occur in the case of:

Options

Rheumatic fever.

Rheumatoid arthritis.

Leprosy.

Sarcoidosis

221. Infectious granulomas occur in the case of:

Options

Rheumatic fever.

Rheumatoid arthritis.

Sarcoidosis

Typhoid fever

222. In any case, meet allergic granuloma?

Options

Tuberculosis

Syphilis.

Rheumatic fever

Typhoid fever

223. In any case, meet allergic granuloma?

Options

Tuberculosis

Syphilis.

Rheumatoid arthritis.

Typhoid fever.

224. Granulomas of unknown etiology can occur only when:

Options

Tuberculosis.

Syphilis.

Rheumatic fever.

Sarcoidosis.

225. Granulomas of unknown etiology can occur only when:


Options

Tuberculosis.

Syphilis.

Rheumatic fever.

Regional ileitis.

226. Interstitial inflammation is determined by characteristic changes:

Options

Formation of granulomas.

Inflammatory infiltration of the stroma of inner organs.

Formation of pseudopolyps.

Cell degeneration of inner organs.

227. Where is the most common location of infectious polyps?

Options

Small intestine.

Large intestine.

Rectum.

Duodenum.

228. Where is the most common location of infectious polyps?

Options

Stomach.

Small intestine.

Large intestine.

Duodenum.

229. Where is the most common location of infectious polyps?

Options

Oral cavity

Nasal cavity.

Pleural cavity.

Peritoneal cavity.

230. In chronic non-specific inflammation, there are certain morphological changes:

Options
Neutrophils, lymphocytes and liquefaction necrosis.

Neutrophils, macrophages and fibrosis.

Lymphocytes, plasma cells and fibrosis.

Giant cells, macrophages and coagulative necrosis

231. In what ways are accumulated macrophages in chronic inflammation?

Options

Continuous inflow of monocytes from circulation maintained by chemotactic factors

Local proliferation of macrophages by mitotic division

Prolonged immobilization of macrophages within the site of inflammation

A and B

232. Exactly how is formed a granuloma with inflammation?

Options

Type I hypersensitivity reaction

Type II hypersensitivity reaction

Type III hypersensitivity reaction

Type IV hypersensitivity reaction

233. What is most often associated with the formation of granulomas?

Options

The healing process.

Acute inflammation

Wound contraction.

A persistent irritant

234. Specify the option of chronic inflammation of the myoma:

Options

Chronic cervicitis

Chronic gastritis

Chronic cholecystitis

None of these

235. Where you can find the epithelioid cells in tuberculous granuloma?

Options

Neutrophils
Eosinophils

Lymphocytes

Macrophages

236. A possible variant of the Epithelial cell modified-specify.

Options

Lymphocytes

Macrophages

Mast cell

Eosinophils

237. In any options found in a granuloma, except:

Options

TB

Yersinia

Mycoplasma

Leprosy

238. Specify a variant of atypical mycobacteria in inflammation.

Options

Mycobacterium microti

Mycobacterium canneti

Mycobacterium africanum

Mycobacterium ulcerans

239. How IgM antibodies are used PGL-1 is used for diagnosis?

Options

Leprosy

Tuberculosis

Syphilis

Brucellosis

240. What kind of leprosy is not included in the classification of Ridley-Jopling?

Options

Mid borderline leprosy

Borderline tuberculoid leprosy


Inderterminate leprosy

Tuberculoid polar leprosy

241. In which case we see the Drug hepar lobatum, specify where:

Options

Primary syphilis

Secondary syphilis

Tertiary syphilis

Congenital syphilis

242. The mechanism of killing of M. tuberculosis which grows inside the macrophage:

Options

By reactive oxygen species

By oxygen-independent bactericidal mechanism

By nitric oxide mechanism

By hydrolytic enzymes

243. The mechanism of the lesion tubercular bacilli - specify option:

Options

Elaboration of endotoxin

Elaboration of exotoxin

Type IV hypersensitivity

Direct cytotoxicity

244. What is true for tuberculosis Bacillus, with the exception of:

Options

A.Tubercle bacilli can be cultured

Tubercle bacilli are anaerobe

Tubercle bacilli thrive best in the apex of lung

M.smegmatis is not pathogenic to man

245. How tubercle bacilli in caseous lesions are best demonstrated in:

Options

Caseous centre

Margin of necrosis with viable tissue

Epithelioid cells
Langhans′ giant cells

246. Specify these options for the Bacilli of leprosy:

Options

Not acid fast

As acid fast as tubercle bacilli

Less acid fast compared to tubercle bacilli

More acid fast compared to tubercle bacilli

247. Lepromin test may be positive when:

Options

Lepromatous leprosy

Borderline lepromatous leprosy

Tuberculoid leprosy

Inderterminate leprosy

248. Where the most difficult to demonstrate Spirochetes in Spirochetes?

Options

Primary syphilis

Secondary syphilis

Tertiary syphilis

Congenital syphilis

249. Specify in which cases Actinomycosis is caused:

Options

Fungus

Gram-negative bacteria

Anaerobic bacteria

Acid fast bacteria

250. Sarcoid granuloma has the following features, except:

Options

Non caseating granuloma

Giant cells have cytoplasmic inclusions

Peripheral mantle of lymphocytes

Fibroblastic proliferation at the periphery of a granuloma


251. Granulomas consists of:

Options

Cholesterol clefts

Collagen

Endothelial cells and fibroblasts

Epithelioid cells

252. Finding acid-fast bacilli within peripheral nerves is most suggestive of

Options

Relapsing fever

Syphilis

Leprosy

Tuberculosis

253. In the study of sputum in a young patient reveals the presence of rare acid-resistant organisms.
Maybe it's the

infection

Options

K. pneumoniae--

L. pneumophila

Mycobacterium avium-intracellulare

Mycobacterium tuberculosis

254. How reliable histopathological evidence of chroniclesthe inflammatory process in the organs is?

Options

Hemorrhages

Leucocytic infiltrates

Blood vessels destruction

Interstitial fibrosis

255. In the microscopic part of the ovary removed during surgery, a large accumulation of epithelioid cells
observed your diagnosis:

Options

Granulation tissue

Pyogenic granuloma

Granulosa cell tumor


Granuloma

256. In primary tuberculosis Granuloma consists mainly of the following:

Options

Fibroblasts

Epithelioid cells

Eosinophils

Plasma cells

257. In primary syphilis Granulomatous infiltration cell consists of the following:

Options

Neutrophils

Monocytes/macrophages

Plasma cells

Eosinophils

258. What type of inflammation is characteristic of Miliary pulmonary tuberculosis?

Options

Granulomatous

Serous

Fibrinous

Suppurative

259. Chronic inflammation is most reliably characterized by the following:

Options

Infiltration with mononuclear cell including macrophages, lymphocytes, and plasma cells

Tissue destruction

Healing by connective tissue with angiogenesis and fibrosis

All of these

260. The causes of chronic inflammation are all of the following, except:

Options

Persistent infections by certain microorganisms

Prolonged exposure to potentially toxic agents, either exogenous or endogenous

Autoimunity (autoimmune diseases)

Complete phagocytosis
261. Determine which cells play an important role in chronic inflammation of tuberculosis?

Options

Macrophages

Leucocytes

Eosinophils

Erythrocytcs

262. All cells can be found in chronic inflammation infiltrate, except for these:

Options

Lymphocytes

Platelets

Macrophages

Plasma cells

263. Granulomatous inflammation develops in all these diseases, except:

Options

Tuberculosis

Leprosy

Syphilis

Budd-Chiary syndrome

264. Specify which of the types of necrosis can be detected in granuloma of tuberculosis?

Options

Coagulation necrosis

Liquefactive necrosis

Caseous necrosis

Enzymatic fat necrosis

265. Tuberculosis granuloma can be characterized by all of the following except:

Options

Plasma cells

Area of central necrosis

Epithelioid cells

Langhans-type giant cells

266. Another name for Syphilis granuloma:


Options

Fibroma

Gumma

Tuberculoma

Leproma

267. Granuloma of syphilis is characterized by all the following except:

Options

Area of central necrosis

Plasma cell infiltrate

Lymphocyte infiltrate

Platelet infiltrate

268. In granulomatous inflammation Macrophages can be transformed into which of the cells:

Options

Monocytes

Epithelial cells

Epithelioid cells

Plasma cells

269. Where it can be found in Gummatous Infiltration of tertiary syphilis?

Options

Aorta

Testes

Liver

Bones and joints

270. In the examination, syphilitic Gumma is characterized by all the following features, except:

Options

White-gray

Rubbery

Solitary

Red-brown

271. Products of activated macrophages in tissue injury include all of the following, except:

Options
Fibrogenic cytokines

Toxic oxygen metabolites

Collagenases

Neutrophile chemotactive factors

272. Products of activated macrophages in tissue injury include all of the following, except

Options

Growth factors

Fibrogenic cytokines

Angiogenesis factors

Proteases

273. What are the lung with multiple tubercular granulomas?

Options

Tuberculous pneumonia

Brown induration of lungs

Miliary tuberculosis

Cavitary fibrocaseous tuberculosis

274. The pathologic changes of vasa vasorum of aorta in syphilitic mesaortitis are characterized by which
of the following:

Options

Migratory thrombophlebitis

Productive vasculitis (obliterative endoarteritis)

Thromboangitis obliterans

Necrotizing arteriolitis

275. What can lead to Medial destruction of the aorta in tertiary syphilis?

Options

Aneurismal dilatation of the aorta

Marian's syndrome

Atherosclerosis

Takayasu's arteritis

276. What is associated with the development of sarcoidosis?

Options
Mycobaclerium leprae

Mycobacterium tuberculosis

Treponema pallidum

Unknown

277. Foreign body granulomas can be caused by all the following reasons except:

Options

Paniculate matter

Synthetic material

Gram-negative bacillus

Vegetable matter

278. Macrophage functions can include the following, except:

Options

Phagocytosis

Degradation of engulfed particles

Forming of giant cells

Producing of mediators.

279. Macrophage functions can include the following, except:

Options

Phagocytosis

Degradation of engulfed particles.

Tissue destruction

Syntesis og collagen

280. Macrophage functions can include the following, except:

Options

Phagocytosis

Degradation of engulfed particles

Producing of antibodies

Tissue destruction.

281. Macrophage functions can include the following, except:

Options

Phagocytosis.
Regeneration of epithelial cells

Degradation of engulfed particles

Tissue destruction

282. Localization of interstitial inflammation is the following, except:

Options

Liver

Kidneys.

Heart.

Spleen.

283. Localization of interstitial inflammation is the following, except:

Options

Liver.

Kidneys.

Heart.

Lymph nodes.

284. Infarction it is necrosis:

Options

With different etiology.

With curtain localization.

With vascular genesis (due to blood circulation disturbances).

In organ due to stopping of arterial blood flow.

285. What is not a morphological type of infarction?

Options

White.

Red.

Mixed.

White with red rim.

286. Call the most often cause of infarction development:

Options

Venous congestion.

Arterial thrombosis.
Thrombosis of large veins

Microcirculatory bed embolism.

287. The most main condition of hemorrhagic infarction development is:

Options

Massive blood loss.

Arterial thrombosis.

Venous congestion.

Anemia.

288. Red infarction is usual for:

Options

Myocardium.

Lung.

Spleen.

Kidney.

289. White infarction with red rim is usual for:

Options

Intestine.

Skin.

Brain.

Myocardium.

290. White infarction is usual for:

Options

Spleen.

Intestine.

Lung.

Liver.

291. What is wrong about lung infarction?

Options

It has pyramidal form.

Dark-red colour.

Develops in venous congestion.


Cyst – is its result.

292. What is wrong about myocardial infarction?

Options

It has pyramidal form.

It has whitish-yellowish colour

It has red rim.

Its consistency is dense.

293. What is wrong about kidney infarction?

Options

It has pyramidal form.

It has whitish-yellowish colour.

It has red rim.

It has soggy mass consistency.

294. What is wrong about spleen infarction?

Options

It has pyramidal form.

Its outcome is connective tissue scar

Its consistency is dense.

Its colour is red.

295. What is wrong about brain infarction?

Options

It has pyramidal form.

It has whitish-grayish colour

Its consistency is soft.

Localizes into subcortical nuclei.

296. Infarction of what organ has the most severe results?

Options

Spleen.

Kidney.

Brain.

Lung.
297. Unfavorable outcome of infarction is:

Options

Organization.

Petrifaction.

Cyst formation.

Suppuration.

298. What does develop as myocardial infarction outcome usually?

Options

Cyst.

Abscess.

Scar.

Hemosiderosis.

299. What does develop as brain infarction outcome usually?

Options

Cyst.

Abscess

Scar.

Hemosiderosis.

300. What does develop as kidney (spleen) infarction outcome usually?

Options

Cyst.

Abscess.

Hemosiderosis.

Scar.

1. Venous emboli are most often lodged in:

Options

Intestines.

Kidney.

Lungs.

Heart.
2. Pathologic changes between sudden decompression from high pressure to normal levels and decompression
from low pressure to normal levels are:

Options

More marked in the former.

More marked in the latter.

No difference between the two.

Acute form is more marked in the latter.

3. The infarct of following organ is invariably hemorrhagic:

Options

Infarct kidney.

Infarct spleen.

Infarct lung.

Infarct hear.

4. Which of the following is true?

Options

Arterial thrombi are white and occlusive.

Venous thrombi are white and occlusive

Arterial thrombi are white and mural.

Venous thrombi are red and mural.

5. Call the type of shock:

Options

Acute.

Hypovolemic.

Reversible.

Irreversible.

6. What does develop in kidney in shock?

Options

Acute tubular necrosis.

Inflammation.

Hemosiderosis.

Petrifaction.
7. What is morphological change in kidney in shock observed?

Options

Tubular atrophy.

Tubular necrosis.

Stromal sclerosis.

Inflammation.

8. What does develop in lung in shock?

Options

Necrosis.

Fat degeneration.

Disappearance of glycogen.

Edema.

9. What is morphological change in lung in shock observed?

Options

Degeneration.

Necrotic foci.

Edema.

Inflammation.

10. What does develop in lung in shock?

Options

Hyperemia.

Hemorrhage.

Edema.

All enumerated.

11. What does develop in liver in shock?

Options

Ischemia.

Necrotic foci.

Hemosiderosis.

Sclerosis.

12. What does develop in myocardium in shock?


Options

Petrifaction.

Hemosiderosis.

Necrosis of cardiomyocites.

Sclerosis.

13. In what organ ulcers and erosions develop in shock more often?

Options

Stomach.

Esophagus.

Oral cavity.

Rectum.

14. Pale infarction is not seen in -

Options

Intestines

Liver

Spleen

Heart

15. What is main in DIC-syndrome development?

Options

Thrombocytopenia.

Anemia.

Insufficiency of fibrinogen synthesis

Increased intravascular blood coagulation.

16. What is the starting moment in the DIC-syndrome development?

Options

Coagulation of fibrinogen with the formation of fibrin

Appearance lot of thromboplastin in blood.

Hypofibrinogenemia

Increased formation of thrombin from plasma prothrombin.

17. Disseminated intravascular coagulation (DIC) is characterized by the following except:

Options
Thrombocytopenia.

Microangiopathic hemolytic anemia.

Presence of FDPs (fibrin degradation products) in the blood.

Normal prothrombin time.

18. Most common manifestation of DIC (disseminated intravascular coagulation) is:

Options

Bleeding.

Thrombosis.

Microangiopathic hemolytic anemia.

Organ damage.

19. A 25-year-old female presents with a history of losing four pregnancies in the past 5 years. She also has
history of recurrent pains in her legs secondary to recurrent thrombosis. Her symptoms are most likely due
to a deficiency of:

Options

PA inhibitiors.

Protein C.

Plasmin.

Thrombin.

20. Procoagulant factors produced by endothelial cells include:

Options

Thrombomodulin.

Prostacyclin.

Won Willebrand factor.

Thromboxane A2.

21. A postmortem clot is most likely to

Options

Grossly display features of recanalization.

Grossly have lines of Zahn.

Grossly have the appearance of «chicken fat» overlying «currant jelly».

Microscopically appear attached to the blood vesse

22. What is the most common site of origin of thrombotic pulmonary emboli?

Options
Deep leg veins.

Lumen of left ventricle.

Lumen of right ventricle

Mesenteric veins.

23. A 9-year-old boy suddenly develops severe testicular pain. He is taken to the emergency room, where he is
evaluated and immediately taken to surgery. There his left testis is found to be markedly hemorrhagic due to
testicular torsion. This abnormality causes a hemorrhagic infarction because of

Options

Arterial occlusion.

Septic infarction

The collateral blood supply of the testis.

Venous occlusion.

24. Red infarction is caused by all of the following events, except:

Options

Coronary artery thrombosis.

Pulmonary embolism

Torsion of the testis

Superior mesenteric artery thromboembolism.

25. Shock is commonly associated with all of the following conditions, except:

Options

Escherichia coli sepsis.

Myocardial infarction.

Cholera.

Cerebral infarction.

26. The fate of the thrombus may be all of the following, except:

Options

Dissolution.

Recanalization.

Organization.

Malignization.

27. The causes of infarction include all of the following pathologic conditions, except:

Options
Trombotic events.

Embolic events.

Arterial occlusion.

Hemophilia.

28. Red infarct occurs in all of the following pathologic conditions, except:

Options

Venous occlusion.

Coronary occlusion.

Loose tissue.

Tissues with dual circulation.

29. White infarct results from which of the following:

Options

Venous occlusion.

Arterial occlusion

Loose tissue.

Tissues with dual circulation.

30. White infarct commonly occurs in all organs, except:

Options

Lung.

Spleen.

Kidney.

Heart.

The formation of a thrombus is favored by all of the following, except:

Options

Anemia.

Endothelial injury.

Pregnancy.

Stases.

The type of tissue necrosis commonly associated with myocardial infarction is which of the following:

Options

Caseous necrosis.
Coagulation necrosis.

Enzymatic fat necrosis.

Gangrenous necrosis.

Pulmonary emboli may originate from all of the following sites, except:

Options

Deep leg veins.

Inferior vena cava.

Pelvic veins.

Portal vein.

The development of endothelial-lined blood channels that reestablish blood flow through a vascular thrombus is
known as:

Options

Collateral circulation.

Recanalization.

Organization.

Hyalinization.

The initial step of the thrombus formation is which of the following:

Options

Activation ofthrombin

Development of fibrin plugs.

Endothelial injury.

Marginationofleukocytes.

Disseminated intravascular coagulation is characterized by each of the following, except:

Options

Decreased fibrinolysis.

Decreased plasma fibrinogen.

Hemolysis.

Prolonged partial thromboplastin time.

Each of the following conditions favors the development of thrombosis, except:

Options

Endothelial injury.
Polycythemia.

Stases.

Thrombocytopenia.

The type of necrosis most often caused by sudden ischemia from vascular occlusion is:

Options

Apoptosis.

Caseous necrosis.

Coagulation necrosis.

Fat necrosis.

All the following disorders are associated with disseminated intravascular coagulation, except:

Options

Infections.

Neoplasms.

Massive tissue injury.

Malnutritions.

The usual source of pulmonary emboli is -

Options

Thrombi in the deep veins of the lower extremities.

Thrombi in the veins of upper extremities.

Thrombi in the vena porta.

Thrombi in the left atrial appendage

Emboli in the main pulmonary artery often cause:

Options

Infarction of the lung.

Sudden death.

Infarction of the heart.

Disseminated intravascular coagulation.

The origin of emboli include all of the following, except:

Options

Thrombi.

Fat droplets.
Air bubbles.

Amyloid masses.

What is shock accompanied with often?

Options

Nephrotic syndrome.

DIC-syndrome.

Hepatico-renal syndrome.

Hepatico-lienal syndrome.

The forms of embolism include all of the following, except:

Options

Pulmonary thromboembolism.

Fat embolism.

Viral embolism.

Bacterial embolism.

The selective stain, used to identify fat in the fat emboli syndrome is:

Options

Sudan III.

Toluidin blue.

Congo red.

PAS reaction.

What is hematoma?

Options

Accumulation of blood inside serous cavities.

Accumulation of blood in tissues without their destruction.

Accumulation of blood in tissues with their destruction.

Bruise.

When does bleeding develop due to blood vessel wall erosion?

Options

In purulent inflammation.

In chronic venous congestion.

In acute venous congestion.


In hypertensive crisis.

When does bleeding develop due to blood vessel rupture?

Options

In purulent inflammation.

In chronic venous congestion.

In acute venous congestion.

In hypertensive crisis.

When does bleeding develop due to diapedesis?

Options

In traumas.

In chronic venous congestion.

In tumours.

In tubal pregnancy.

Hemorrhage which is associated with tissue necrosis is called:

Options

Hemorrhagia.

Hematoma.

Hemorrhagic saturation.

Petechia.

What can be outcome of hemorrhage?

Options

Hematoma.

Organization.

Necrosis.

Petechia.

One of the causes of general venous hyperemia, except:

Options

Congenital valvular heart disease.

Acquired valvular heart disease.

Fibroelastosis of endocardium.

Acute myocardial infarction.


Hemorrhage into pleural cavity is -

Options

Hemopericardium.

Metrorrhagia

Hematomesis

Hemothorax

Brown induration is a type of chronic venous congestion occurs in

Options

Liver

Heart

Spleen

Lung

Choose the definition of the stasis:

Options

Decreased arterial blood flow.

Blood viscous increasing.

Difficulties of blood outflow.

Stopping of blood flow in microcirculatory bed.

The most severe result of long stasis is:

Options

Sludge phenomenon

Perivascular edema.

Plasmorrhagia.

Necrosis of parenchymatous cells.

What does develop in stasis?

Options

Sludge phenomenon.

Erythrocytes diapedesis.

Necrosis of tissue elements.

All enumerated.

Morphologic changes in the lungs due to chronic venous hyperemia –


Options

Edema

Congestive hyperemia.

Brown induration.

Congestive induration.

What does sludge phenomenon mean?

Options

Adhesion of blood cells to each other.

Erythrocytes agglutination.

Increasing of blood cells number.

Increasing of blood viscous.

Edema it is:

Options

Increased blood filling of organ, tissue.

Increased containment of interstitial fluid.

Difficulties of venous blood outflow.

Exudate accumulation.

Generalized edema as a result of renal dysfunction or nephrotic syndrome is localized in:

Options

Periorbital tissue.

Lung

All parts of the body.

Brain

The local vacant hyperemia occurs due to -

Options

Compression of arteries by tumor.

Decreasing of barometric pressure.

Obturation of arteries by thrombus.

Irritation of vasorelaxant nerves.

In nephrotic syndrome edemas are:

Options
Hydrostatic.

Oncotic.

Membranogenic.

Electrolyte

In acute glomerulonephritis edemas are:

Options

Hydrostatic

Oncotic

Membranogenic

Electrolyte

What is leading there in edemas development in chronic cardiac failure?

Options

Increased hydrostatic pressure.

Decreasing of colloid-osmotic pressure.

Increased aldosterone secretion.

Damage of endothelium and basement membranes of capillars.

What is observed in lung edema?

Options

Increasing of lungs sizes.

Increasing of lungs weight.

Flabby consistency of lungs.

All enumerated.

Pulmonary edema commonly results from all of the following, except:

Options

Left ventricular failure.

Renal failure.

Systemic hypertension.

Appendicitis

What is observed in lung edema?

Options

Increasing of lungs sizes.


Decreasing of lungs weight.

Increased air filling of lungs.

Dense consistency of lungs.

What does develop in lung edema?

Options

Extension of alveolars spaces.

Accumulation of edematous fluid in alveolars spaces.

Sclerosis of interalveolar septas.

All enumerated.

Organ in arterial hyperemia is characterized by:

Options

Enlarged, red, hot.

Enlarged, blue, cold.

Small, red, cold.

Small, brown, hot.

Hemorrhage from nasal cavity is called:

Options

Maelena

Haemoptoe

Haematemesis

Epistaxis

Nutmeg liver is characterized by:

Options

Enlarged, soft, yellow-grey.

Enlarged, hard, dark-red.

Enlarged, hard, yellow-grey with dark-red dots.

Small, hard, reddish-yellow.

A transudate is characterized by the fallowing feature:

Options

Muddy

Bad smelling
Contains proteins less than 2%.

All enumerated.

Microscopically, acute pulmonary congestion is characterized by all of the following, except:

Options

Engorged alveolar capillaries.

Thickened and fibrotic alveolar septa.

Edema fluid in alveoli.

Focal intra-alveolar hemorrhage.

What does develop in lung edema?

Options

Hyperemia of capillars.

Accumulation of edematous fluid in alveolars spaces.

Erythrocytes diapedesis.

All enumerated.

Cyanotic induration of kidney is characterized by:

Options

Enlarged, hard, brown.

Enlarged, hard, blue-red.

Small. soft, red.

Small, hard, red-brown.

Call the certain variant of edematous fluid containment increasing:

Options

Hematoma

Ascites

Petechia

Hemothorax

Microscopically, brain hemorrhage is characterized by all of the following, except:

Options

Fatty changes of damaged neurons.

Central core of clotted blood.

Edema of adjacent brain tissue.


Dystrophy of survived neurons.

What is the cause of thrombus formation?

Options

Damage of blood vessel wall.

Number of erythrocytes decreasing.

Number of thrombocytes decreasing.

Plasmorrhagia

The most common underlying cause of primary brain parenchymal hemorrhage is which of the following:

Options

Systemic coagulation disorders.

Vasculitis.

Systemic hypertension.

Neoplasms

Macroscopically, brain hemorrhage is characterized by all of the following, except:

Options

Local extravasation of blood.

Local damage of brain tissue.

Narrowed cerebral suici.

Cavitary destruction of brain.

Call stages of thrombus formation:

Options

Agglutination of thrombocytes.

Fibrinogen coagulation.

Plasma proteins precipitation.

All enumerated.

Signs of chronic venous hyperemia are all of the following, EXCEPT:

Options

Brown induration of lungs.

Waxy kidneys.

Cyanotic induration of spleen.

Cyanotic induration of kidneys.


What morphological type of thrombus is non-existent?

Options

Red

White

Mixed

White with red rim

More often white thrombi form in:

Options

Veins

Arteries

Aneurysm cavity.

Capillars

More often red thrombi form in:

Options

Veins

Arteries

Capillars

Heart chambers.

More often hyaline thrombi form in:

Options

Veins

Arteries

Capillars

Heart chambers.

One of unfavorable thrombus formation outcomes is:

Options

Organization

Thromboembolism

Petrifaction

Vascularization

Obstructive thrombus of artery can cause:


Options

Venous congestion.

Arterial hyperemia.

Infarction

Thromboembolism

Obstructive thrombus of vein can cause:

Options

Venous congestion

Arterial hyperemia.

Infarction.

Petrifaction

Rupture of the heart wall may be result of:

Options

Inflammation

Myocardial infarction.

Invasion of cancers.

Hypoxia

Corrosion of the vessel wall may be result of:

Options

Atherosclerosis

Inflammation

Vascular abnormalities.

Hypoxia

Favorable outcome of thrombus formation is:

Options

Septic autolysis.

Suppuration

Organization

Thromboembolism

Thrombus which is consisting of alternating red thrombus particles with white thrombus particles is called:

Options
Red

White

Mixed

Hyaline

Thrombus which contains lot of erythrocytes is called:

Options

Red

White

Mixed

Flaky

Thrombus which contains lot of leukocytes and fibrin is called:

Options

Red

White

Mixed

Hyaline

Clinical significance of hemorrhage depends on all the following, EXCEPT:

Options

Site of hemorrhage.

Volume of blood loss.

Blood group of individual.

Diameter of damaged vessel.

Large (more than 2 cm) hemorrhage into skin is called:

Options

Hematoma.

Hemorrhagic infiltration.

Bruise

Purpura

Gross characteristics of a thrombus include:

Options

Rough surface.
Contains lot of fluid.

It is not attached to blood vessel wall.

All enumerated is right.

Gross characteristics of a thrombus include:

Options

Rough surface.

Crimped surface.

It is attached to blood vessel wall.

All enumerated is right.

Thromboembolism of pulmonary trunk and its large brunches results as:

Options

Pulmonocoronary reflex.

Lung infarction.

Athelectasis.

Shock.

Call the localization of thrombi in pulmonary thromboembolism:

Options

Valvulars of the left part of heart.

Aorta

Arteries of large circulation.

Veins of large circulation.

Call the localization of thrombi in large circulation arteries thromboembolism:

Options

Valvulars of the left part of heart.

Valvulars of the right part of heart.

Veins of lesser circulation.

Veins of large circulation.

Fat embolism of what organs capillars is most dangerous?

Options

Kidney

Liver
Lungs

Intestine

Call the outcome of large circulation arteries thromboembolism:

Options

Hyperemia of inner organs.

Infarctions in organs.

Edema

Cachexia

Pulmonocoronary reflex develops in:

Options

Fat embolism of lung blood vessels.

Amniotic fluid embolism.

Microbe embolism of lung blood vessels.

Pulmonary trunk thromboembolism.

Fat embolism is possible in:

Options

Ulceration and disattachment of atherosclerotic plaque particles.

Massive traumas of subcutaneous fat tissue.

Mistaken intramuscular injections of oil-based drugs.

All enumerated.

Lipids that can accumulate intracellularly are:

Options

Triglycerides

Derived lipids

Miscellaneous lipids

Lecithin

Lipids that can accumulate intracellularly are:

Options

Cholesterol

Derived lipids

Miscellaneous lipids
Lecithin

Which type of lipids accumulates at fatty change in liver cells?

Options

Triglycerides

Cholesterol

Complex lipids

Lipogialin

Which type of lipids accumulate in atherosclerosis?

Options

Triglycerides

Cholesterol

Complex lipids

Lipogialin

Substance accumulating in tissues as a result is fatty change:

Options

Water

Cholesterol

Glycogen

Triglycerides

Give some example of intracellular hyaline degeneration:

Options

Tubular epithelium of kidney

Keloid

Renal glomeritis in chronic nephritis

Pyelonephritis

Give some example of intracellular hyaline degeneration:

Options

Mallory body in alcoholic liver cells

Keloid

Renal glomeritis in chronic nephritis

Pyelonephritis
Following are the features of reversible injury, except:

Options

Swelling of mitochondria and endoplasmic reticulum

Detachment of ribosomes from endoplasmic reticulum

Clumping of nuclear chromatin

Pyknosis and karyolysis of nucleus

One of cause of intracellular accumulation of metabolic substances is:

Options

Genetic defects

Inflammation

Embolism

Necrosis

The most common cause of fatty change is:

Options

Inflammation

Neoplasia

Hypoxia

Amyloidosis

Dystrophy is defined as the process:

Options

Degeneration of tissue

Inflammation

Edema

Adaptation

Dystrophies are based on following mechanism, except one:

Options

Decomposition

Transformation

Regeneration

Infiltration

One of manifestations of metabolic derangements in cells is:


Options

Apoptosis

Atrophy

Hypertrophy

Intracellular accumulation of abnormal amounts of various substances.

Fatty change in the heart is characterized by:

Options

Tiger heart.

Small size heart

Red heart

Solid heart

Xantoma is a pathological process with accumulation of:

Options

Cholesterol

Lipoproteins

Phospholipids

Lipohyalin

Mallory’s bodies may be found in:

Options

Hepatocytes

Neurons

Cardiomyocytes

Epithelial cells of stomach

The fatty liver has all pathologic features, except:

Options

Enlarged

Yellow

Soft

Red

Diabetes mellitus is characterized by accumulation of glycogen in all cells, except:

Options
Epithelial cells of the proximal tubules.

Liver cells.

Smooth muscle cells.

B-cells of the islets of Langerhans.

Hyaline droplets in renal tubular epithelial cells are seen in:

Options

Proteinuria

Lipiduria

Hematuria

Cylindruria

What substances are accumulated within parenchymal cells in fatty changes?

Options

Cholesterol

Apoproteins

Triglycerides

Vitamins

The cells accumulating fat within the intimal layer of arteries in atherosclerotic plaques are called:

Options

Fibroblasts

Epithelial cells

Foam cells

Lymphocytes

Tumor arising from epithelial cells with accumulation of mucin is called:

Options

Mucinous teratoma

Mucinous carcinoma

Mucinous melanoma.

Mucinous neuroblastoma

Fatty change in the heart is characterized by:

Options

Red heart.
Small size heart

Tiger heart

Solid heart

Simple hyaline occurs in:

Options

Diabetes mellitus.

Systemic lupus erythematosus.

Atherosclerosis

Rheumatic fever

Fatty change is often seen in all of the following organs, except:

Options

Lung

Heart

Kidney

Muscles

Accumulation of cholesterol and cholesterol esters with formation of tumorous masses is called:

Options

Atheroma

Xantoma

Adenoma

Papilloma

Colloid degeneration is occurs in

Options

Thyroid gland

Mammary gland

Salivary gland

Pituitary gland

The fatty liver has all pathologic features, except:

Options

Greasy

Red
Yellow

Enlarged

The stain used to identify fat is:

Options

Hematoxylin and eosin stain

Metachromatic stain

PAS reaction

Red oil O

The reversible process caused by accumulation of glycosaminoglycans in extracellular matrix is:

Options

Amyloidosis

Hyalinosis

Mucoid changes

Fibrinoid changes

Lipohyalin occurs in:

Options

Diabetes mellitus.

Arterial hypertension

Atherosclerosis

Rheumatic fever

Specific method for diagnosis of amyloid in fresh tissue is:

Options

Virchow test

Rokitansky test

Ewing test

Masson test

Substance accumulating in tissues as a result is mucoid change:

Options

Water

Cholesterol

Protein
Glycogen

Compound hyalin occurs in:

Options

Diabetes mellitus

Arterial hypertension

Atherosclerosis

Rheumatic fever

Foam cells are characterized by accumulation of:

Options

Neutral fat

Triglycerides

Cholesterol

Phospholipids

All the pathological processes are irreversible, except:

Options

Mucoid changes.

Fibrinoid changes

Amyloidosis

Apoptosis

Variant of systemic amyloidosis is:

Options

Senile cardiac

Senile cerebral

Heredofamilial

Endocrine

Causes of hyalinosis are all of the following, except:

Options

Fibrinoid changes

Inflammation

Apoptosis

Sclerosis
The substance with fibrillar structure, which forms under the pathological conditions is:

Options

Reabsorption droplets

Russell bodies

Lipids

Amyloid

Variant of systemic amyloidosis is:

Options

Senile cardiac.

Senile cerebral

Endocrine

Hemodialysis-associated

Simple hyaline occurs in:

Options

Diabetes mellitus

Arterial hypertension

Rheumatic fever

Rheumatoid arthritis

The pathologic proteinaceous substance, accumulating only between cells in various tissues and organs of the body
is:

Options

Glycogen

Hyaline

Water

Amyloid

Variant of localized amyloidosis is:Variant of localized amyloidosis is:

Options

Primary

Secondary

Heredofamilial

Endocrine
Cause of obesity, which associated with excessive nutrition, is called:

Options

Primary

Alimentary

Cerebral

Endocrine

Grossly the organs affected by amyloidosis are characterized by all of the following, except:

Options

Enlarged

Hard

Smooth

Soft

Variant of localized amyloidosis is:

Options

Primary

Secondary

Heredofamilial

Tumor-forming

Type of obesity with unknown cause is called:

Options

Primary

Alimentary

Cerebral

Endocrine

On autopsy of the 58-year-old man it is revealed: mitral valve is deformed, thickened, not totally closed.
Microscopically: centers of collagen fibers are eosinophilic, have positive fibrin reaction. The most likely it is:

Options

Fibrinoid swelling

Mucoid swelling

Hyalinosis

Amyloidosis
The stain used to identify mucoid changes is:

Options

Hematoxylin and eosin stain.

Metachromatic stain

Congo red stain.

PAS reaction.

Most common localization of mucoid changes is:

Options

Reproductive system

Cardiovascular system

Urinary system

Blood system.

Simple hyaline occurs in:

Options

Arterial hypertension

Rheumatic fever

Diabetes mellitus

Rheumatoid arthritis

Simple hyaline occurs in:

Options

Diabetes mellitus.

Systemic lupus erythematosus

Atherosclerosis

Rheumatoid arthritis

Lipohyalin occurs in

Options

Arterial hypertension

Diabetes mellitus

Atherosclerosis

Rheumatic fever

The abnormal protein substance, accumulating in extracellular space in various tissues and organs of the body is:
Options

Glycogen

Amyloid

Hyaline

Water

Type of systemic amyloidosis is:

Options

Senile cardiac

Senile cerebral

Heredofamilial

Endocrine

Type of systemic amyloidosis is:

Options

Senile cardiac

Senile cerebral

Endocrine

Hemodialysis-associated

Type of localized amyloidosis is:

Options

Senile cardiac

Primary

Secondary

Heredofamilial

Typeof localized amyloidosis is:

Options

Primary

Secondary

Senile cerebral

Heredofamilial

Type of localized amyloidosis is:

Options
Primary

Secondary

Heredofamilial

Endocrine

Type of localized amyloidosis is:

Options

Primary

Secondary

Heredofamilial

Tumor-forming

Localization of per collagenous amyloidosis is:

Options

Liver

Spleen

Heart

Kidneys

Localization of per collagenous amyloidosis is:

Options

Liver

Spleen

Bowel

Kidneys

Localization of per collagenous amyloidosis is:

Options

Liver

Spleen

Nerves

Kidneys

Localization of per reticular amyloidosis is:

Options

Heart
Tongue

Nerves

Kidneys

Localization of per reticular amyloidosis is:

Options

Heart

Tongue

Nerves

Liver

Localization of per reticular amyloidosis is:

Options

Heart

Tongue

Nerves

Spleen

Obesity with cause of excessive nutrition is called:

Options

Alimentary

Primary

Cerebral

Endocrine

Type of obesity with unknown cause is called:

Options

Alimentary

Cerebral

Endocrine

Primary

Variant of systemic amyloidosis is:

Options

Senile cardiac

Senile cerebral
Endocrine

Secondary

Obesity with deposition of fat in abdomen area is called:

Options

Upper type

Lower type

Symmetric type

Middle type

Obesity with deposition of fat in area of face and neck is called:

Options

Diffuse type

Middle type

Lower type

Upper type

Systemic amyloidoses affect :

Options

More than one body organ or system

Only one body organ or tissue type

Heredofamilial

Endocrine

Localised amyloidoses affect:

Options

A.Senile cardiac

Senile cerebral

Only one body organ or tissue type

Tumor-forming

All examples of extracellular hyaline are change, except:

Options

Mallory’s hyaline

Old scar

Hyaline arteriolosclerosis
Chronic glomerulonephritis

In amyloidosis macroscopically liver changed in the following way:

Options

Enlarged, pale, waxy and firm

Dense and sharply reduced

Enlarged and yellow color

Soft and enlarged

Spleen changed by amyloidosis in the following way:

Options

Cut surface is translucent pale and waxy

Dense and sharply reduced

Cut surface – map-like areas of amyloid

True (A+C)

Causes of hyalinosis are all of the following, EXCEPT:

Options

Fibrinoid changes

Inflammation

Apoptosis

Necrosis

Most specific histological sign in mucoid changes is:

Options

Metachromasia

Metaplasia

Metastasis

Metamorphosis

In the process of repair, parenchymal regeneration indicates there generation of:

Options

Labile cells

Stable cells

Permanent cell

A+b
Wound healing is the summation of following processes, except:

Options

Coagulation

Matrix synthesis

Angiogenesis

Fibrolysis

Organization is - ?

Options

Change of area of necrosis or thrombus by connective tissue

Regeneration of tissue

Vascularization of necrotic mass

Autolysis of necrotic mass

What of collagen type is normally present in the skin, bone and tendons?

Options

Type I collagen

Type II collagen

Type III collagen

Type IV collagen

Keloid scars is made up of -

Options

Dense collagen

Loose fibrous tissue

Granulomatous tissue

Loose areolar tissue

Contraction of wounds is completed by the -

Options

Two weeks

Six weeks

Six months

Two years
A young man of 20 got a lacerated wound on his left arm, stitched - 1 week later sutures were remained-healing
continued but the site became disfigured by prominent raised irregular nodular scar, in next 2 months which of the
following best describes the process

Options

Organization

Dehiscence

Resolution

Keloid formation

The most characteristic feature of granulation tissue is:

Options

Growth of fibroblasts and new capillaries

Resemblance to a granuloma

Character of the exudate

Granular scar that results

Granulation tissue:

Options

Is a feature of wound healing

Contains fibroblasts

Contains thin-walled capillaries

Often contains granuloma

The following holds true for stable cells in cell cycle:

Options

They remain in cell cycle from one mitosis to the next

They are in resting phase but can be stimulated to enter the cell cycle

They have left the cell cycle

They do not have capacity to multiply in response to stimuli throughout adult life

Connective tissue in scar is formed by the following types of fibrillar collagen:

Options

Type II, III, IV

Type I, III, V

Type I, II, V

Type III, V, VII


Basement membrane consists of:

Options

Type I collagen

Type V collagen

Type III collagen

Type IV collagen

Which of the following is non-fibrillar collagen?

Options

Type V

Type I

Type III

Type VI

Which is false about primary union?

Options

Exuberant granulation tissue to fill the gap

Clear margins

Uninfected

Lead to neat linear scar

A patient present with a large wound to his right forearm that is the result of a chain saw accident. You treat his
wound appropriately and follow him in your surgery clinic at routine intervals. Initially his wound is filled with
granulation tissue, which is composed of proliferating fibroblasts and proliferating new blood vessels (angiogenesis).
A growth factor that is capable of inducing all the steps necessary for angiogenesis is:

Options

Epidermal growth factor (EGF)

Transforming growth factor α (TGF-α)

Platelet-derived factor (PDGF)

Basic fibroblast growth factor (FGF)

Systemic factors that influence wound healing are all of the following, except:

Options

Nutrition

Metabolic status

Blood group
Hormones (glucocorticoids)

Local factors that influence wound healing are all of the following, except:

Options

Wound infection

Mechanical factors

Foreign bodies

Hormones (glucocorticoids)

What type of wound healing can be termed as “keloid”?

Options

Atrophic scar

Scar with dysplasia

Hyperplastic scar

Hypertrophic scar

The main difference between slowly dividing and rapidly dividing cells is the duration of the next phase

Options

M (mitosis) phase: Phase of mitosis.

G1 (gap 1) phase: The daughter cell enters G1 phase after mitosis.

S (synthesis) phase: During this phase, the synthesis of nuclear DNA takes place.

G2 (gap 2) phase: After completion of nuclear DNA duplication, the cell enters G2 phase

These cells continue to multiply throughout life under normal physiologic conditions:

Options

Labile cells

Stable cells

Permanent cells

None

These cells decrease or lose their ability to proliferate after adolescence but retain the capacity to multiply in
response to stimuli throughout adult life:

Options

Labile cells

Stable cells

Permanent cells
None

These cells lose their ability to proliferate around the time of birth:

Options

Labile cells

Stable cells

Permanent cells

None

Labile cells include following cells:

Options

Epithelial cells of the alimentary tract

Parenchymal cells of live

Neurons of nervous system

A+d

Stable cells include following cells:

Options

Epithelial cells of the alimentary tract

Parenchymal cells of liver

Neurons of nervous system

Hematopoietic cells of bone marrow

Permanent cells include following cells:

Options

Epithelial cells of the alimentary tract

Parenchymal cells of liver

Neurons of nervous system

Hematopoietic cells of bone marrow

What is the replacement of injured tissue by fibrosis tissue?

Options

Regeneration

Repair

Metaplasia

Necrosis
The following phases are observed in the formation of granulation tissue:

Options

Phase of inflammation

Phase of clearance

Phase of ingrowths of granulation tissue

A+b+c

Healing by first intention has the following characteristics, except:

Options

Surgically incised

Without much loss of cells and tissue

Clean and uninfected

Open with a large tissue defect, at times infected

Healing by second intention has the following characteristics, except:

Options

Open with a large tissue defect, at times infected

Clean and uninfected

Edges of wound are approximated by surgical sutures

B+c

The following factors influence the wound healing is local, except:

Options

Infection

Poor blood supply to wound

Exposure ultraviolet light

Age

The following factors influence the wound healing is systemic, except:

Options

Nutrition

Systemic infection

Administration of glucocorticoids

Exposure to ionizing radiation

Steps involved in the formation of procallus are as follows:


Options

Hematoma → Local inflammatory response → Ingrowths of granulation tissue → Callus composed of woven bone
and cartilage

Hematoma → Ingrowths of granulation tissue → Local inflammatory response → Callus composed of woven bone
and cartilage

Callus composed of woven bone and cartilage → Ingrowths of granulation tissue → Hematoma → Local

inflammatory response

Ingrowths of granulation tissue → Local inflammatory response → Callus composed of woven bone and

cartilage → Hematoma

Destruction of which organs and tissues is replaced by fibrous tissue?

Options

Epithelial of skin

Heart muscle

Liver with intact basement membrane

Bone marrow

Which proteoglican is distributed in basement membranes?

Options

Chondroitin sulphate

Heparan sulphate

Dermatan sulphate

Keratan sulphate

Labile cells are:

Options

Cells of stratified squamous epithelium

Parenchymal cells of liver

Parenchymal cells of kidney

Skeletal muscle cells

Labile cells are:

Options

Cells of columnar epithelium

Parenchymal cells of liver


Parenchymal cells of kidney

Cells of neurons

Labile cells are:

Options

Parenchymal cells of pancreas

Cells of lymph nodes

Parenchymal cells of kidne

Skeletal muscle cells

Labile cells are:

Options

Parenchymal cells of pancreas

Hematopoietic cells of bone marrow

Parenchymal cells of kidney

Skeletal muscle cells

Stable cells are:

Options

Cells of stratified squamous epithelium

Hematopoietic cells of bone marrow

Parenchymal cells of kidney

Cells of columnar epithelium

Stable cells are:

Options

Cells of stratified squamous epithelium

Hematopoietic cells of bone marrow

Parenchymal cells of liver

Cells of columnar epithelium

Stable cells are:

Options

Cells of stratified squamous epithelium

Cells of columnar epithelium

Hematopoietic cells of bone marrow


Smooth muscles cells

Stable cells are:

Options

Cells of stratified squamous epithelium

Cells of columnar epithelium

Hematopoietic cells of bone marrow

Bone cells

Stable cells are:

Options

Cells of stratified squamous epithelium

Cells of columnar epithelium

Cells of transitional epithelium

Cartilage cells

Stable cells are:

Options

Cells of stratified squamous epithelium

Cells of columnar epithelium

Hematopoietic cells of bone marrow

Vascular endothelium

Permanent cells are:

Options

Cells of stratified squamous epithelium

Smooth muscles cells

Cardiac muscle cells

Bone cells

Permanent cells are:

Options

Cells of stratified squamous epithelium

Cells of columnar epithelium

Cells of neurons

Bone cells
Polypeptide growth factors circulating in the serum are, except:

Options

Epidermal growth factor

Mesodermal growth factor

Platelet derived growth factor

Transforming growth factor

Polypeptide growth factors circulating in the serum are, except:

Options

Epidermal growth factor

Lymphocyte derived growth factor

Platelet derived growth factor

Fibroblast growth factor

Polypeptide growth factors circulating in the serum are, except:

Options

Epidermal growth factor

Platelet derived growth factor

Chondroblast growth factor

Transforming growth factor

Functions of growth factors are, except:

Options

Stimulation of cell division

Proliferation of cells

Synthesis of collagen

Depression of cell division

Functions of growth factors are, except:

Options

Functions of growth factors are, except:

Proliferation of cells

Synthesis of collagen

Metaplasia of cells

Functions of growth factors are, except:


Options

Stimulation of cell division

Proliferation of cells

Synthesis of collagen

Atrophy of cells

Mechanisms of wound healing are, except:

Options

Regeneration of parenchymal cells

Proliferation of connective tissue cells

Proliferation of hematopoietic cells

Remodeling of connective tissue

Mechanisms of wound healing are, except:

Options

Regeneration of parenchymal cells

Proliferation of connective tissue cells

Remodeling of bone tissue

Remodeling of connective tissue

Sign of primary union of wound healing is:

Options

Large amount of granulation tissue

Small amount of granulation tissue

Slow healing

Wound contraction

Sign of primary union of wound healing is:

Options

Large amount of granulation tissue

Rapid healing

Slow healing

Wound contraction

Sign of primary union of wound healing is:

Options
Large amount of granulation tissue

Less scar formation

More scar formation

Wound contraction

Signs of primary union of wound healing are, except:

Options

Small amount of granulation tissue

Less scar formation

Rapid healing

More scar formation

Sign of secondary union of wound healing is:

Options

Small amount of granulation tissue

Less scar formation

More scar formation

Rapid healing

Signs of secondary union of wound healing are, except:

Options

Large amount of granulation tissue

More scar formation

Less inflammatory reaction

More inflammatory reaction

Signs of secondary union of wound healing are, except:

Options

Large amount of granulation tissue

Rapid healing

Wound contraction

More inflammatory reaction

Signs of secondary union of wound healing are, except:

Options

Large amount of granulation tissue


Less scar formation

More scar formation

Wound contraction

Local factor that adversely affect wound healing is:

Options

Infection

Protein malnutrition

Severe anemia

Zink deficiency

Local factor that adversely affect wound healing is:

Options

Infection

Protein malnutrition

Severe anemia

Zink deficiency

Local factor that adversely affect wound healing is:

Options

Poor blood supply

Protein malnutrition

Severe anemia

Corticosteroid excess

Systemic factor that adversely affect wound healing is:

Options

Infection

Poor blood supply

Presence of necrotic tissue

Protein malnutrition

Systemic factor that adversely affect wound healing is:

Options

Infection

Poor blood supply


Presence of necrotic tissue

Diabetes mellitus

All of the following cells are stable, except:

Options

Parenchymal cells of liver

Parenchymal cells of pancreas

Parenchymal cells of kidneys

Surface epithelial cells of the epidermis

All of the following cells are stable, except:ll of the following cells are stable, except:

Options

Parenchymal cells of liver

Parenchymal cells of pancreas

Parenchymal cells of thyroid

Surface epithelial cells of the alimentary tract

All of the following cells are stable, except:

Options

Parenchymal cells of liver

Parenchymal cells of pancreas

Parenchymal cells of kidneys

Surface epithelial cells of the respiratory tract

All of the following cells are stable, except:

Options

Parenchymal cells of liver

Parenchymal cells of pancrea

Parenchymal cells of thyroid

Hematopoietic cells of bone marrow

All of the following cells are stable, except:

Options

Parenchymal cells of liver

Parenchymal cells of pancreas

Parenchymal cells of kidneys


Hematopoietic cells of bone marrow

All of the following cells are stable, except:

Options

Parenchymal cells of liver

Parenchymal cells of pancreas

Parenchymal cells of thyroid

Skeletal muscle cells

All of the following cells are stable, except:

Options

Parenchymal cells of liver

Parenchymal cells of pancreas

Parenchymal cells of kidneys

Cardiac muscle cells

All of the following cells are labile, except:

Options

Surface epithelial cells of the epidermis

Surface epithelial cells of the alimentary tract

Surface epithelial cells of the respiratory tract

Parenchymal cells of liver

All of the following cells are labile, except:

Options

Surface epithelial cells of the epidermis

Surface epithelial cells of the alimentary tract

Parenchymal cells of pancreas

Hematopoietic cells of bone marrow

All of the following cells are labile, except:

Options

Surface epithelial cells of the epidermis

Surface epithelial cells of the alimentary tract

Parenchymal cells of pancreas

Hematopoietic cells of bone marrow


All of the following cells are labile, except:

Options

Surface epithelial cells of the epidermis

Surface epithelial cells of the alimentary tract

Surface epithelial cells of the respiratory tract

Parenchymal cells of kidneys

All of the following cells are labile, except:

Options

Surface epithelial cells of the epidermis

Surface epithelial cells of the alimentary tract

Surface epithelial cells of the respiratory tract

Parenchymal cells of thyroid

All of the following cells are labile, except:

Options

Surface epithelial cells of the epidermis

Surface epithelial cells of the alimentary tract

Surface epithelial cells of the respiratory tract

Neurons of nervous system

All of the following cells are labile, except:

Options

Surface epithelial cells of the epidermis

Surface epithelial cells of the alimentary tract

Skeletal muscle cells

Hematopoietic cells of bone marrow

All of the following cells are labile, except:

Options

Surface epithelial cells of the epidermis

Surface epithelial cells of the alimentary tract

Surface epithelial cells of the respiratory tract

Cardiac muscle cells

The objects of the study of pathological anatomy are:


Options

Cadaveric material

Substrates obtained from patients during life

Experimental material

Correct answers are ABC

Depending on the method of obtaining the diagnostic material, a biopsy can be:

Options

Invasive

Traction

Aspiration

Expiring

Indicate the term " autopsy":

Options

Digestion of cells and tissues of the body

Examination of the cadaver

Intravital tissue taking for diagnostic purposes

Intravital decrease in the size of cells, tissues, organs

The term "etiology" as applied to the disease means:

Options

Complications

The mechanism of death

The cause of development

Development mechanism

Indicate the levels of study of the pathological process, except:

Options

Tissue

Organic

Cellular

Population

Groups of objects studied by the pathologoanatomist, EXCEPT:

Options
Cadaveric material

Experimental material

Substrates from living patients

Data from sociological survey

Pathoanatomical autopsy is performed in case of death from:

Options

Drowning

Disease

Hanging

Poisoning

Methods for examining cadaveric material, EXCEPT:

Options

Biochemical

X-ray

Microbiological

Electrocardiographic

In the course of systemic pathanatomy study:

Options

Adaptation processes

Kidney disease

Inflammation and healing

Circulatory disorders

Taking a piece of organs tissue with a diagnostic purpose during the life of the patient is called:

Options

Biopsy

Autopsy

Necropsy

Electrocardiography

Immunohistochemical examination is used for, EXCEPT:

Options

Detection of autoantigens
Determining the sex of the child

Determination of tumor histogenesis

Establishing the type of infectious agent

Methods of molecular biology:

Options

In situ hybridization

Chromosome analysis

Gene analysis

Scanning electron microscopy

For quantitatively analyze the DNA content in tumor cells used:

Options

In situ hybridization

Chromosome analysis

Flow cytometry

Scanning electron microscopy

The area of using the hybridization method in situ is:

Options

Diagnosis of tumors

Cloning of cells and tissues

Detection of tissue necrosis zone

Detection of tissue necrosis zone

Basophilic structure in tissues is:

Options

Cytoplasm of cells

Erythrocytes

Collagen fibers

Deposits of calcium salts

Picrofuxin staining by Van Gieson selectively reveals:

Options

Nerve fibers

Smooth muscle cells


Macrophage of connective tissue

Collagen fibers of connective tissue

Electron microscopy is needed to identify everything except:

Options

Viruses in tissues

Bacteria in the tissues

Immune complexes with pemphigoid

B-cell and T-cell lymphomas

Flow cytometry for DNA content analysis allows you to determine everything EXCEPT:

Options

Aneuploidy

Diploidity

Presence of pathogens

Number of dividing cells

The hybridization technique is used for everything, EXCEPT:

Options

Identification of viral DNA

Diagnosis of certain anemia

Studying the genome with its congenital disorders

Differentiation of breast cancer and ovarian cancer

The term "pathogenesis" as applied to the disease means:

Options

Complications

Cause of development

Clinical manifestations

The development mechanism

The term "tanatogenesis" as applied to the disease means:

Options

Complications

Cause of development

The development mechanism


Mechanism of death

The term "pathomorphosis" as applied to the disease means:

Options

Complications

Cause of development

Variability

The development mechanism

Selective for glycogen staining:

Options

Sudan III

Hematoxylin and eosin

Picrofuxin

Schiff (PAS) -reaction

Lipid-selective staining:

Options

Sudan III

Hematoxylin and eosin

Picrofuxin

Schiff (PAS) -reaction

The terminal state with absence of cardiac activity without signs of irreversible changes in the gray matter of the
brain indicates:

Options

Sudden death

Clinical death

Biological death

Death from disease

Beloglazov's sign at death is:

Options

Brownish spots on the sclera

Opacity of the cornea

Dilated pupils
"cat's eye"

Selective staining for amyloid:

Options

Sudan III

Hematoxylin and eosin

Congo red

Schiff (PAS) -reaction

Selective staining for mucus:

Options

Picrofuxin

Sudan III

Congo red

Mucicarmine

A reliable sign of death:

Options

Decrease in body temperature less than 22°C

Absent of independent breathing

Absent of pulse

Absent of heart beats

Unreliable sign of death:

Options

Stable areflexion

Cadaveric rigor mortis

Lack of independent breathing

Cadaveric spots

Fibrinoid necrosis is typical for:

Options

Syphilis

Lymphogranulomatosis

Rheumatism

Tuberculosis
The primary localization of Liquefactive necrosis is:

Options

Heart

Spleen

Brain

Kidney

Favorable outcome of necrosis is:

Options

Trombosis

Encapsulation

Suppuration

Embolism
Is a adenoma considered to be malignant or benign?

Options

Malignant

Benign

Tumor-like

Hamartoma

Who does the grading of a tumor?

Options

Physician

Pharmacist

Pathologist

Oncologist

What grade is a moderately differentiated tumor?

Options

.I

II

Is a leukemia considered to be malignant or benign?

Options

Malignant

Benign

Tumor-like

Hamartoma

Benign tumor of cartilage is called an:

Options

Osteoma

Chondroma

Leiomyoma

Lipoma

Who does the staging of the tumor?


Options

Pathologist

Physician

Oncologist

Dr. Fischione

Benign tumor of osseous tissue is called an:

Options

Osteoma

Chondroma

Leiomyoma

Lipoma

What does TNM stand for?

Options

Tumor size, lymph node, malignancy

Tumor size, leimyoma, malignancy

Tumor shape, lymph node, metastasis

Tumor size, lymph node, metastasis

Benign tumor in smooth muscle is called a:

Options

Osteoma

Leiomyoma

Lipoma

Chondroma

A 40-year-old man has a positive stool guaiac test during a routine physical examination. A colonoscopy is performed
and a 0.9-cm, circumscribed, pedunculated mass on a short stalk is found in the upper rectum. Which of the
following terms best describes this lesion?

Options

Adenoma

Carcinoma

Choristoma

Hamartoma

Benign tumor in fat tissue is called a:


Options

Osteoma

Lipoma

Leimyoma

Chondroma

A Pap smear obtained from a 29-year-old woman dur ing a routine health maintenance examination is abnormal.
She is currently asymptomatic. She has a history of multiple sexual partners. Cervical biopsy specimens are obtained
and the microscopic appearance is shown in the figure. Which of the following is the most likely diagnosis?

Options

Adenocarcinoma

Carcinoma in situ

Dysplasia

Squamous cell carcinoma

Malignant tumor in fibrous tissue is called a:

Options

Liposarcoma

Osteosarcoma

Fibrosarcoma

Rhabdomyosarcoma

A 69-year-old woman has experienced increasing mal aise and a 10-kg weight loss over the past year. She dies of
massive pulmonary thromboembolism. The gross appearance of the liver at autopsy is shown in the figure. Which of
the following best describes the lesions seen in her liver?

Options

Invasive angiosarcoma

Hepatocellular carcinoma

Leukemic infiltration

Metastatic adenocarcinoma

Malignant tumor in fat tissue is called a:

Options

Fibrosarcoma

Liposarcoma

Osteosarcoma
Rhabdomyosarcoma

A 66-year-old man with chronic cough has an episode of hemoptysis. On physical examination, there are no
abnormal findings. A chest radiograph shows a 6-cm mass in the right lung. A sputum cytologic analysis shows
neoplastic squamous cells. Metastases from his lung lesion are most likely to be found at which of the following
sites?

Options

. Cerebral hemisphere

Chest wall muscle

Hilar lymph nodes

Splenic red pulp

Malignant tumor in bone tissue is called a:

Options

Fibrosarcoma

Rhabdomyosarcoma

Osteosarcoma

Liposarcoma

An epidemiologic study of cancer deaths recorded in the last half of the 20th century is conducted. The number of
deaths for one particular type of cancer had been decreas ing in developed nations, despite the absence of
widespread screening and prevention programs. Which of the following neoplasms was most likely to be identified
by this study?

Options

Cerebral glioma

Gastric adenocarcinoma

Hepatic angiosarcoma

Leukemia

An epidemiologic study of cancer deaths recorded in the last half of the 20th century is conducted. The number of
deaths for one particular cancer had increased markedly in developed nations. More than 30% of cancer deaths in
men, and more than 24% of cancer deaths in women, were caused by this neoplasm in 1998. In some nations,
prevention strate gies reduced deaths from this cancer. Which of the following neoplasms was most likely identified
by this study?

Options

Cerebral glioma

Bronchogenic carcinoma

Hepatocellular carcinoma

Colonic adenocarcinoma
Malignant tumor in skeletal muscle is called a:

Options

Fibrosarcoma

Liposarcoma

Osteosarcoma

Rhabdomyosarcoma

An epidemiologic study analyzes health care benefits of cancer screening techniques applied to persons more than
50 years of age. Which of the following diagnostic screening techniques used in health care is most likely to have the
great est impact on reduction in cancer deaths in Europe and North America?

Options

Chest radiograph

Mammography

Pap smear

Serum tumor markers

What is a benign tumor called that is derived from all 3 germ cell layers?

Options

Myoma

Fibroma

Teratoma

Lymphoma

. A 34-year-old sexually active woman undergoes a rou tine physical examination. There are no abnormal findings. A
Pap smear is obtained as part of the pelvic examination. Cyto logically, the cells obtained on the smear from the
cervix show severe epithelial dysplasia (high-grade squamous intraepithe lial lesion). Which of the following
therapeutic options is most appropriate for this woman?

Options

Antibiotic therapy

Excision

Ovarian removal

Screening of family members

Most common location of teratoma?

Options

Uterus

Ovaries
Kidney

Liver

A 70-year-old woman reported a 4-month history of a 4-kg weight loss and increasing generalized icterus. On physi
cal examination, she has midepigastric tenderness on palpa tion. An abdominal CT scan shows a 5-cm mass in the
head of the pancreas. Fine-needle aspiration of the mass is performed. On biochemical analysis, the neoplastic cells
show continued activation of cytoplasmic kinases. Which of the following genes is most likely to be involved in this
process?

Options

APC

MYC

P53

RAS

A 22-year-old man has a raised, pigmented lesion on his forearm that has increased in size and become more
irregular in color over the past 4 months. Physical examination shows a 0.5 × 1.2 cm black-to-brown asymmetric
lesion with irregular borders. An excisional biopsy specimen shows clusters of pleo morphic pigmented cells that
extend into the reticular dermis. Family history indicates that the patient’s maternal uncle died from a similar tumor.
His grandfather required enucleation of the left eye because of a “dark brown” retinal mass. Which of the following
genes is most likely to have undergone mutation to produce these findings in this family?

Options

BCL2 (anti-apoptosis gene)

C-MYC (transcription factor gene)

IL2 (growth factor gene)

P16 (cell cycle inhibition)

A 3-year-old child has exhibited difficulty with vision in her right eye. On physical examination, there is leukocoria of
the right eye, consistent with a mass in the posterior chamber. MR imaging shows a mass that nearly fills the globe.
The child un dergoes enucleation of the right eye. Molecular analysis of the neoplastic cells indicates absence of both
copies of a gene that contributes to control of the cell cycle. Which of the following genes has most likely undergone
mutation in this neoplasm?

Options

BCR-ABL

BCL2

HMSH2

RB

A 76-year-old man has experienced abdominal pain for the past year. On physical examination, there is an epigastric
mass. An abdominal CT scan shows a 10-cm mass in the body of the pancreas. A fine-needle biopsy specimen of this
mass shows a moderately differentiated adenocarcinoma. Mutational analysis of the carcinoma cells shows
inactivation of cyclin-dependent kinase inhibitor with loss of growth-suppression. Regulatory pathways controlled by
which of the following genes are most likely altered in this man’s carcinoma?
Options

BCL2

β-Catenin

MYC

TGF-β

A 55-year-old man has had hemoptysis and worsening cough for the past month. On physical examination, wheezes
are auscultated over the right lung posteriorly. A chest radiograph shows a 6-cm right perihilar mass. A fine-needle
aspiration biop sy is performed and yields cells with the microscopic appearance of non–small cell bronchogenic
carcinoma. Molecular analysis of the neoplastic cells shows a p53 gene mutation. Which of the following
mechanisms has most likely produced the neoplastic transformation?

Options

Inability to hydrolyze GTP

Growth factor receptor activation

Loss of cell cycle arrest

Microsatellite instability

A 26-year-old man with a family history of colon carci noma undergoes a surveillance colonoscopy. It reveals hun
dreds of polyps in the colon, and two focal 0.5-cm ulcerated areas. A biopsy specimen from an ulcer reveals
irregularly shaped glands that have penetrated into the muscular layer. Which of the following molecular events is
believed to occur very early in the evolution of his colonic disease process?

Options

Activation of the WNT signaling pathway

Inability to hydrolyze GTP-bound RAS

Loss of heterozygosity affecting the p53 gene

Mutations in mismatch repair genes.

A 63-year-old man has a cough with hemoptysis for 10 days. He has a 65 pack-year history of smoking. A chest CT
scan shows a 5-cm right hilar mass. Bronchoscopy is per formed, and lung biopsy specimens show small cell
anaplastic lung carcinoma. His family history shows three first-degree maternal relatives who developed leukemia,
sarcoma, and carcinoma before age 40 years. Which of the following gene products is most likely to have been
altered by mutation to produce these findings?

Options

APC (tumor suppressor)

BCL2 (anti-apoptosis)

K-RAS (GTP binding)

P53 (DNA damage response)

A 30-year-old man has a 15-year history of increasing numbers of benign skin nodules. On physical examination, the
firm, nontender, subcutaneous nodules average 0.5 to 1 cm. Further examination shows numerous oval 1- to 5- cm
flat, light brown skin macules. Ophthalmoscopic examination shows hamartomatous nodules on the iris. A biopsy
specimen of one skin nodule shows that it is attached to a peripheral nerve. Which of the following molecular
abnormalities is most likely related to his clinical presentation?

Options

Decreased susceptibility to apoptosis

Impaired functioning of mismatch repair

Increased production of epidermal growth factor

Persistent activation of the RAS gene

A 53-year-old man diagnosed with oral cancer and treated with radiation and chemotherapy 1 year ago now has a
positron emission tomography (PET) scan of his neck that shows a single focus of increased uptake. This focus is
resected and microscopic examination shows that it is a metastasis. Mo lecular analysis of this cancer shows p53,
PTEN, and c-MYC gene mutations. Which of the following metabolic pathways is most likely up-regulated to promote
his cancer cell survival and proliferation?

Options

Aerobic glycolysis

Gluconeogenesis

Hexose monophosphate shunt

Oxidative phosphorylation

An experiment involving carcinoma cells grown in cul ture studies the antitumor surveillance effects of the innate
immune system. These carcinoma cells fail to express MHC class I antigens. It is observed, however, that carcinoma
cells are lysed when an immune cell that has been activated by IL-2 is added to the culture. Which of the following
immune cells is most likely to function in this manner?

Options

CD4+ lymphocyte

CD8+ lymphocyte

Macrophage

NK cell

Gross specimen of teratoma may contain?

Options

Teeth

Hair

Cartilage

All of the above

Which brain structure may be a site for teratoma formation?

Options
Right hemispheire

Pineal gland

Medulla

Pons

Which of the following is the tumor-like condition?

Options

Choristoma

Chondroma

Hepatoma

Melanoma

What is the definition of choristoma?

Options

Heterotopic rest

Bening tumor of ovaries

Benign tumor of liver

Benign tumor of cartilage

Most common malignant bone tumor is?

Options

Osteogenic sarcoma

Chondrosarcoma

Giant cell tumor

Synovioma

Malignant tumor originating from plasma cells?

Options

Multiple myeloma

Chondroma

Fibroma

Lieomyoma

Benign tumor is:

Options

Noninvasive
Invasive

Able to spread

Metastasize

Mixed tumor is found in the following organ:

Options

Salivary gland

Liver

Kidney

Parathyroid

Seminoma arising from testicles are:

Options

Benign

Malignant

Tumor-like

Hamartoma

A 33-year-old man has experienced occasional head aches for the past 3 months. He suddenly has a generalized
seizure. CT scan of the head shows a periventricular 3-cm mass in the region of the right thalamus. A stereotactic
biop sy of the mass yields large lymphoid cells positive for B cell markers. Which of the following underlying diseases
is most likely to be found in this atient?

Options

Diabetes mellitus

HIV infection

Hypertension

Multiple sclerosis

A 40-year-old man has a history of intravenous drug use. Physical examination shows needle tracks in his left an
tecubital fossa. He has mild scleral icterus. Serologic studies for HBsAg and anti-HCV are positive. He develops
hepatocel lular carcinoma 15 years later. Which of the following viral characteristics best explains why this patient
developed hepa tocellular carcinoma?

Options

Viral integration in the vicinity of proto-oncogenes

Viral capture of proto-oncogenes from host cellular DNA

Viral capture of proto-oncogenes from host cellular DNA

Viral inactivation of RB and p53 gene expression


A 61-year-old man with a history of chronic viral hep atitis has noted a 6-kg weight loss over the past 5 months.
Physical examination shows no masses or palpable lymphade nopathy. An abdominal CT scan shows a nodular liver
with a 10-cm mass in the right lobe. A stool guaiac test result is nega tive. An elevation in which of the following
laboratory tests is most likely to be present in this man?

Options

Alpha-fetoprotein

CA-19-9

Calcitonin

Carcinoembryonic antigen

A 59-year-old man has noticed blood in his urine for the past week. Cystoscopy shows a 4-cm exophytic mass involv
ing the right bladder mucosa near the trigone. After biopsy specimens are obtained, he undergoes a radical
cystectomy. Examination of the excised specimen shows an anaplastic car cinoma that has infiltrated the bladder
wall. Which of the fol lowing techniques applied to the cells from his neoplasm is most likely to categorize the cell of
origin?

Options

Chromosomal karyotyping

Cytologic smear

DNA microarray

Immunohistochemistry

A 69-year-old man has noted a chronic cough for the past 3 months. On physical examination, there is mild stri dor
on inspiration over the right lung. A chest radiograph shows a 5-cm right hilar lung mass, and a fine-needle aspi
ration biopsy specimen of the mass shows cells consistent with squamous cell carcinoma. If staging of this neoplasm
is denoted as T2N1M1, which of the following findings is most likely in this man?

Options

Brain metastases

Elevated corticotropin level

Infiltration of the chest wall

Obstruction of a mainstem bronchus

A 44-year-old woman notes a lump in her left breast while taking a shower. The nurse practitioner palpates a 3 cm
firm, irregular, non-movable mass in the upper outer quadrant of her left breast on physical examination. A fine
needle aspiration of this mass is performed, and cytologically the cells are consistent with infiltrating ductal
carcinoma. The mass is removed with lumpectomy along with an axillary lymph node dissection. Which of the
following findings will best predict a better prognosis for this patient?

Options

Tumor cells strongly estrogen receptor positive

No metastases in the sampled lymph nodes

Flow cytometric analysis with aneuploidy and a high S-phase


One relative who had a similar type of breast cancer

A change in bowel habits prompts a 53-year-old woman to see her physician. On physical examination there are no
lesions noted on digital rectal examination, but her stool is positive for occult blood. A colonoscopy is performed and
reveals a 6 cm friable exophytyic mass in the cecum. A biopsy of this mass is performed and microscopic examination
shows a moderately differentiated adenocarcinoma. Which of the following laboratory findings is most likely to be
present in this patient?

Options

K-RAS mutation in the neoplastic cells

Neoplastic cells positive for vimentin

Stool culture with Shigella flexneri

Presence of HIV-1 RNA

An experiment is conducted in which proliferating cells are subjected to ionizing radiation. The ionizing radiation
leads to arrest in a checkpoint that monitors completion of DNA replication. It is observed that there are increased
numbers of chromosomal abnormalities in these cells. Which of the following is the checkpoint affected by the
ionizing radiation?

Options

G0/G1

G1/S

S/G2

G2/M

Hamartoma is defined as:

Options

Heperotopic rest

Tumor originating from indigenous site

Malignant tumor

Benign tumor

A clinical study is performed to determine the incidence of cancers in different countries. The data show that
persons born in Japan and continuing to reside there have an increased risk for cancer. Which of the following
cancers is most likely seen with increased frequency in this population?

Options

Breast

Colon

Lung

Stomach
A 48-year-old woman has a routine physical examination. A 4 cm diameter non-tender mass is palpated in her right
breast. The mass appears fixed to the chest wall. Another 2 cm non-tender mass is palpable in the left axilla. A chest
radiograph reveals multiple 0.5 to 2 cm nodules in both lungs. Which of the following TNM classifications best
indicates the stage of her disease?

Options

T1 N1 M0

T1 N0 M1

T2 N1 M0

T4 N1 M1

Malignant tumor from connective tissues are called:

Options

Adenomas

Papillomas

Carcinomas

Sarcomas

A study is performed to analyze characteristics of malignant neoplasms in biopsy specimens. The biopsies were
performed on patients who had palpable mass lesions on digital rectal examination. Of the following microscopic
findings, which is most likely to indicate that the neoplasm is malignant?

Options

Pleomorphism

Atypia

Invasion

Increased nuclear/cytoplasmic ratio

A child is born with a single functional allele of a tumor suppressor gene. At the age of five the remaining normal
allele is lost through a point mutation. As a result, the ability to inhibit cell cycle progression until the cell is ready to
divide is lost. Which of the following neoplasms is most likely to arise via this mechanism?

Options

Breast ductal carcinoma

Pulmonary small cell anaplastic carcinoma

Ocular retinoblastoma

Cerebral astrocytoma

A 50-year-old man has felt vague abdominal discomfort for the past 4 months. On physical examination he has no
lymphadenopathy, and no abdominal masses or organomegaly can be palpated. Bowel sounds are present. An
abdominal CT scan shows a 20 cm retroperitoneal soft tissue mass obscuring the left psoas muscle. A stool specimen
tested for occult blood is negative. Which of the following neoplasms is this man most likely to have?
Options

Melanoma

Hamartoma

Adenocarcinoma

Liposarcoma

Malignant tumor from connective tissues commonly located in:

Options

Brain

Heart

Liver

Lower extremity

A clinical study is performed of oncogenesis in human neoplasms. It is observed that some neoplasms appear to
develop from viral oncogenesis, with serologic confirmation of past viral infection. Which of the following neoplasms
is most likely to arise in this manner?

Options

Retinoblastoma

Small cell anaplastic carcinoma

T-cell leukemia

Prostatic adenocarcinoma

An experiment is designed to study the genetics of cancer. The study will link the appearance of cancer to specific
gene abnormalities. Which of the following forms of cell molecular analysis is most useful to identify gene alterations
involved in carcinogenesis?

Options

Florescence in situ hybridization

Flow cytometry

Immunohistochemistry

Single nucleotide polymorphisms

Tumor arising from all 3 germ cell layers are called:

Options

Choristoma

Hamartoma

Teratoma
Papilloma

A 14-year-old healthy girl has a 0.3 cm reddish, slightly raised nodule on the skin of the upper part of her chest found
on a routine physical examination. She states that this lesion has been present for years and has not appreciably
changed in size or color. Which of the following neoplasms is this nodule most likely to be?

Options

Hemangioma

Melanoma

Carcinoma

Lymphoma

Tumor containing teeth, hair, cartilage, and adipose tissue are called:

Options

Choristoma

Hamartoma

Hamartoma

Papilloma

A 60-year-old man who has a 90 pack year history of cigarette smoking has had a chronic cough for the past 10
years. He has begun to lose weight (3 kg) during the past year. No abnormal findings are noted on physical
examination. He has a chest radiograph that reveals a right hilar mass. A sputum cytology shows atypical,
hyperchromatic squamous cells. What is the most common initial pathway for metastases from this lesion?

Options

Bloodstream

Pleural cavity

Contiguous spread to chest wall

Lymphatics

A 55-year-old man has had malaise and a 4 kg weight loss over the past 6 months. On physical examination his stool
is positive for occult blood. An abdominal CT scan shows his liver contains multiple tumor masses from 2 to 5 cm in
size with central necrosis. The surrounding hepatic parenchyma appears normal. Which of the following
characteristics of neoplasia is best illustrated by these findings?

Options

Multicentric origin

High tumor grade

Primary neoplasm in the stomach

Advanced stage

A 59-year-old man has had a worsening cough with chest pain for the past 6 months. On physical examination he has
no remarkable findings. A chest x-ray shows a 3 cm left lung mass. A sputum cytology specimen yields cells
diagnosed as a squamous cell carcinoma. A mediastinoscopy is performed and reveals metastases in a lymph node.
He is given radiation therapy, and the mass diminishes in size. Which of the following cellular mechanisms is most
likely to account for this tumor response?

Options

Point mutations in DNA

Generation of free radicals

Loss of the blood supply

Secondary inflammation

A 61-year-old woman has a firm mass with irregular borders felt in her left breast on a routine physical examination.
A fine needle aspiration is performed and microscopic examination shows malignant cells. A left mastectomy with
axillary lymph node dissection is performed. A tissue sample of this neoplasm is submitted for analysis by flow
cytometry. Which of the following does flow cytometric analysis most likely provide?

Options

. Analysis of the karyotype

Detection of gene mutations

Determination of aneuploidy

Distinguishing carcinoma from sarcoma

A 35-year-old healthy woman had a firm nodule palpable on the dome of the uterus six years ago recorded on
routine physical examination. The nodule has slowly increased in size and now appears to be about twice the size it
was when first discovered. By ultrasound scan it is solid and circumscribed. She remains asymptomatic. Which of the
following neoplasms is she most likely to have?

Options

Adenocarcinoma

Leiomyosarcoma

Hemangioma

Leiomyoma

A 27-year-old woman in excellent health has a routine health maintenance examination. A 2 cm firm, rounded mass
is palpable beneath the skin of the left forearm. She has no difficulty using the arm and there is no associated pain
with the mass, either in movement or on palpation. The overlying skin appears normal. The mass does not change in
size over the next year. Which of the following neoplasms is she most likely to have?

Options

Metastatic carcinoma

Melanoma

Rhabdomyosarcoma

Lipoma
A 45-year-old woman has noted a lump on her left shoulder that has enlarged over the past 4 months. On physical
examination there is a palpable non-tender supraclavicular lymph node. A biopsy of the node is performed and on
microscopic examination there is a metastatic neoplasm. Which of the following is the most likely primary for this
neoplasm?

Options

Cerebral glioma

Adenocarcinoma of the stomach

Fibroadenoma of the breast

Liposarcoma of the retroperitoneum

Hamartoma refers to:

Options

Tumour differentiating towards more than one cell line

Tumour arising from totipotent cells

Mass of disorganised but mature cells indigenous to the part

Mass of ectopic rests of normal tissue

A 52-year-old woman feels a lump in her right breast. On physical examination there is a 3 cm right breast mass fixed
to the chest wall. This mass is biopsied and on microscopic examination shows nests of cells with marked
hyperchromatism and pleomorphism. These cells are estrogen receptor positive. Flow cytometry is performed.
Compared with surrounding non-neoplastic stromal cells, the neoplastic cells are more likely to be in which of the
following phases of the cell cycle?

Options

G0

G1

G2

Increased number of normal mitoses may be present in the following tissues except:

Options

Bone marrow cells

Nails

Hepatocytes

Intestinal epithelium

In an experiment, it is observed that chronic, increased exposure to ionizing radiation results in damage to cellular
DNA. As a consequence, a protein is now absent that would arrest the cell in the G1 phase of the cell cycle.
Subsequent to this, the cell is transformed to acquire the property of unregulated growth. The absent protein is
most likely the product of which of the following genes?
Options

RAS

TP53

MYC

ABL

An epidemiologic study is performed to find risk factors for development of malignant neoplasms. A statistical
analysis of pre-existing medical conditions is done. Some pre-existing chronic medical conditions are observed to
precede development of malignant neoplasms, while others do not. Which of the following conditions is most likely
to be statistically related to development of a malignancy?

Options

Essential hypertension

Coronary artery disease

Chronic bronchitis

Ulcerative colitis

A tumour is termed medullary when it is almost entirely composed of:

Options

Amyloid stroma

Large areas of necrosis

Abundant lymphoid tissue

Parenchymal cells

A 35-year-old man has noted several 1 to 2 cm reddish purple, nodular lesions present on the skin of his right arm
which have increased in size and number over the past 3 months. The lesions do not itch and are not painful. He has
had a watery diarrhea for the past month. On physical examination he has generalized lymphadenopathy and oral
thrush. Which of the following infections is most likely to be related to the appearance of these skin lesions?

Options

Candida albicans

Human herpesvirus 8

Mycobacterium tuberculosis

Pseudomonas aeruginosa

. A 44-year-old woman who has had multiple sexual partners for the past 30 years has an abnormal Pap smear with
cytologic changes suggesting human papillomavirus (HPV) infection. Without treatment, she is most likely to develop
which of the following lesions?

Options

Squamous cell carcinoma


Non-Hodgkin's lymphoma

Kaposi sarcoma

Adenocarcinoma

All the following malignant tumours metastasise except:

Options

Synovial sarcoma

Malignant mesothelioma

Glioma

Neuroblastoma

A healthy 22-year-old woman undergoes a routine physical examination. A discrete, firm, rubbery, movable mass is
found in the left breast. She has no axillary lymphadenopathy. The skin overlying the breast and the nipple appear
normal. Which of the following neoplasms is most likely to be present in this woman?

Options

Lipoma

Intraductal carcinoma

Malignant lymphoma

Fibroadenoma

The following malignant tumours frequently spread through haematogenous route except:

Options

Bronchogenic carcinoma

Renal cell carcinoma

Follicular carcinoma thyroid

Seminoma testis

An epidemiologic study is performed involving patients of East Asian ancestry with long-standing Epstein-Barr virus
(EBV) infection. It is observed that these patients have an increased risk for development of malignant neoplasms in
adulthood. Which of the following neoplasms is most likely to be found in these patients?

Options

Kaposi sarcoma of skin

Small cell anaplastic carcinoma of lung

Osteosarcoma of bone

Nasopharyngeal carcinoma

Degradation of ECM is brought about by the following except:

Options
Proteases

Metalloproteinases

Free radicals

Cathepsin D

An experiment is conducted in which proliferating cells are subjected to ionizing radiation. The ionizing radiation
leads to arrest in a checkpoint that monitors completion of DNA replication. It is observed that there are increased
numbers of chromosomal abnormalities in these cells. Which of the following is the checkpoint affected by the
ionizing radiation?

Options

G0/G1

G1/S

S/G2

G2/M

Grading of tumours depends upon the following except:

Options

Degree of anaplasia

Metastatic spread

Rate of growth of cells

Degree of differentiation

A clinical study is performed to determine the incidence of cancers in different countries. The data show that
persons born in Japan and continuing to reside there have an increased risk for cancer. Which of the following
cancers is most likely seen with increased frequency in this population?

Options

Breast

Colon

Lung

Stomach

A 25-year-old man presents 1 week after discovering that his left testicle is twice the normal size. Physical
examination reveals a nontender, testicular mass that cannot be transilluminated. Serum levels of alpha-fetoprotein
and human chorionic gonadotropin are normal. A hemiorchiectomy is performed, and histologic examination of the
surgical specimen shows embryonal carcinoma. Compared to normal adult somatic cells, this germ cell neoplasm
would most likely show high levels of expression of which of the following proteins?

Options

Desmin

Dystrophin
Cytochrome c

Telomerase

Patients of xeroderma pigmentosum are prone to develop the following cancers except:

Options

Basal cell carcinoma

Sweat gland carcinoma

Malignant melanoma

Squamous cell carcinoma

A 62-year-old woman presents with a breast lump that she discovered 6 days ago. A breast biopsy shows lobular
carcinoma in situ. Compared to normal epithelial cells of the breast lobule, these malignant cells would most likely
show decreased expression of which of the following proteins?

Options

Desmin

E-cadherin

Lysyl hydroxylase

P selectin

The primary target of reactive electrophiles is as under:

Options

Cytochrome P-450

RNA

DNA

Mitochondria

An 80-year-old man complains of lower abdominal pain, increasing weakness, and fatigue. He has lost 16 lb (7.3 kg)
in the past 6 months. The prostate-specifi c antigen test is elevated (8.5 ng/mL). Rectal examination reveals an
enlarged and nodular prostate. A needle biopsy of the prostate discloses invasive prostatic adenocarcinoma.
Histologic grading of this patient’s carcinoma is based primarily on which of the following criteria?

Options

Capsular involvement

Extent of regional lymph nodes involvement

Pulmonary metastases

Resemblance to normal tissue of origin

A 65-year-old man complains of muscle weakness and a dry cough for 4 months. He has smoked two packs of
cigarettes daily for 45 years. A chest X-ray shows a 4-cm central, left lung mass. Laboratory studies reveal
hyperglycemia and hypertension. A transbronchial biopsy is diagnosed as small cell carcinoma. Metastases to the
liver are detected by CT scan. Which of the following might account for the development of hyperglycemia and
hypertension in this patient?

Options

Adrenal metastases

Paraneoplastic syndrome

Pituitary adenoma

Pituitary metastases

Which of the following potent carcinogens was most likely involved in the pathogenesis of lung cancer in the patient
described in Question 8?

Options

Afl atoxin B1

Asbestos

Azo dyes

Polycyclic aromatic hydrocarbons

A 33-year-old woman discovers a lump in her left breast on self-examination. Her mother and sister both had breast
cancer. A mammogram demonstrates an ill-defined density in the outer quadrant of the left breast, with
microcalcifications. Needle aspiration reveals the presence of malignant, ductalepithelial cells. Genetic screening
identifi es a mutation inBRCA1. In addition to cell cycle control, BRCA1 protein promotes which of the following
cellular functions?

Options

Apoptosis

Cell adhesion

DNA repair

Gene transcription

A 60-year-old man who worked for 30 years in a chemical factory complains of blood in his urine. Urine cytology
discloses dysplastic cells. A bladder biopsy demonstrates transitional cell carcinoma. Which of the following
carcinogens was most likely involved in the pathogenesis of bladder cancer in this patient?

Options

Aniline dyes

Arsenic

Benzene

Cisplatinum

A 45-year-old man presents with a 9-month history of a reddish nodule on his foot. Biopsy of the nodule discloses a
poorly demarcated lesion composed of fi broblasts and endothelial-like cells lining vascular spaces. Further work-up
identifi es similar lesions in the lymph nodes and liver. The tumor cells contain sequences of human herpesvirus-8
(HHV-8). This patient most likely has which of the following diseases?

Options

Acquired immunodefi ciency

Ataxia telangiectasia

Li-Fraumeni syndrome

Neurofi bromatosis type I

During a routine checkup, a 50-year-old man is found to have blood in his urine. He is otherwise in excellent health.
An abdominal CT scan reveals a 2-cm right renal mass. You inform the patient that staging of this tumor is key to
selecting treatment and evaluating prognosis. Which of the following is the most important staging factor for this
patient?

Options

Histologic grade of the tumor

Histologic grade of the tumor

Proliferative capacity of the tumor cells

Somatic mutations in the p53 tumor suppressor gene

A 58-year-old woman with colon cancer presents with 3 months of increasing shortness of breath. A chest X-ray
reveals numerous, bilateral, round masses in both lungs. Histologic examination of an open-lung biopsy discloses
malignant gland-like structures, which are nearly identical to the colon primary. Which of the following changes in
cell behavior was the fi rst step in the process leading to tumor metastasis from the colon to the lung in this patient?

Options

Arrest within the circulating blood or lymph

Exit from the circulation into a new tissue

Invasion of the underlying basement membrane

Penetration of vascular or lymphatic channels

A 68-year-old man complains of recent changes in bowel habits and blood-tinged stools. Colonoscopy reveals a 3- cm
mass in the sigmoid colon. Biopsy of the mass shows infi ltrating malignant glands. These neoplastic cells have most
likely acquired a set of mutations that cause which of the following changes in cell behavior?

Options

Decreased cellular motility

Enhanced stem cell differentiation

Increased cell-cell adhesion

Loss of cell cycle restriction point control


A 35-year-old woman complains of nipple discharge and irregular menses of 5 months duration. Physical
examination reveals a milky discharge from both nipples. MRI shows an enlargement of the anterior pituitary. Which
of the following is the most likely histologic diagnosis of this patient’s pituitary tumor?

Options

Adenoma

Choristoma

Hamartoma

Papilloma

A 52-year-old woman presents with a 1-year history of upper truncal obesity and moderate depression. Physical
examination shows hirsutism and moon facies. A CT scan of the thorax displays a hilar mass. A transbronchial lung
biopsy discloses small cell carcinoma. Electron microscopy of this patient’s lung tumor will most likely reveal which
of the following cytologic features?

Options

Councilman bodies

Hyperplasia of endoplasmic reticulum

Mitochondrial calcifi cation

Neuroendocrine granules

Cytogenetic studies in a 40-year-old woman with follicular lymphoma demonstrate a t(14;18) chromosomal
translocation involving the bcl-2 gene. Constitutive expression of the protein encoded by the bcl-2 gene inhibits
which of the following processes in this patient’s transformed lymphocytes?

Options

Apoptosis

DNA excision repair

G1-to-S cell cycle progression

Oxidative phosphorylation

A 59-year-old woman presents with increasing pigmentation of the skin. Physical examination shows hyperkeratosis
and hyperpigmentation of the axilla, neck, fl exures, and anogenital region. Endocrinologic studies reveal normal
serum levels of adrenal corticosteroids and glucocorticoids. If this patient’s skin pigmentation represents a
paraneoplastic syndrome, the primary tumor would most likely be found in which of the following anatomic
locations?

Options

Bladder

Cervix

Esophagus

Stomach
A 65-year-old man dies after a protracted battle with metastatic colon carcinoma. At autopsy, the liver is fi lled with
multiple nodules of cancer, many of which display central necrosis (umbilication). Which of the following best
explains the pathogenesis of tumor umbilication in this patient?

Options

Biphasic tumor

Chronic infl ammation

Granulomatous infl ammation

Ischemia and infarction

Carcinogenic influence of radiation appears after:

Options

< 2 years

2-5 years

5-10 years

> 10 years

A 59-year-old man complains of progressive weakness. He reports that his stools are very dark. Physical examination
demonstrates fullness in the right lower quadrant. Laboratory studies show iron defi ciency anemia, with a serum
hemoglobin level of 7.4 g/dL. Stool specimens are positive for occult blood. Colonoscopy discloses an ulcerating
lesion of the cecum. Which of the following serum tumor markers is most likely to be useful for following this patient
after surgery?

Options

Alpha-fetoprotein

Carcinoembryonic antigen

Chorionic gonadotropin

Chromogranin

The following hereditary diseases have higher incidence of cancers due to inherited defect in DNA repair mechanism
except:

Options

Ataxia telangiectasia

Xeroderma pigmentosum

Familial polyposis coli

Bloom’s syndrome

A 20-year-old woman has an ovarian tumor removed. The surgical specimen is 10 cm in diameter and cystic. The
cystic cavity is found to contain black hair and sebaceous material. Histologic examination of the cyst wall reveals a
variety of benign differentiated tissues, including skin, cartilage, brain, and mucinous glandular epithelium. What is
the diagnosis?
Options

Adenoma

Chondroma

Hamartoma

Teratoma

A 42-year-old man presents with upper gastrointestinal bleeding. Upper endoscopy and biopsy reveal gastric
adenocarcinoma. Which country of the world has the highest incidence of this malignant neoplasm?

Options

Argentina

Canada

Japan

Mexico

The following form of ionising radiation exposure is associated with highest risk of cancer:

Options

A-rays

B-rays

G-rays

X-rays

An 8-year-old girl with numerous hypopigmented, ulcerated, and crusted patches on her face and forearms develops
an indurated, crater-like, skin nodule on the back of her left hand. Biopsy of this skin nodule discloses a squamous
cell carcinoma. Molecular biology studies reveal that this patient has germline mutations in the gene encoding a
nucleotide excision repair enzyme. What is the appropriate diagnosis?

Options

Ataxia telangiectasia

Hereditary albinism

Li-Fraumeni syndrome

Xeroderma pigmentosum

Women receiving oestrogen therapy have an increased risk of developing the following cancers except:

Options

Breast cancer

Endometrial carcinoma

Gallbladder cancer
Hepatocellular carcinoma

A 59-year-old woman complains of “feeling light-headed” and losing 5 kg (11 lb) in the last month. A CBC reveals a
normocytic, normochromic anemia. The patient subsequently dies of metastatic cancer. Based on current
epidemiologic data for cancer-associated mortality in women, which of the following is the most likely primary site
for this patient’s malignant neoplasm?

Options

Brain

Breast

Colon

Lung

Important cyclins in cell cycle include the following except:

Options

Cyclin A

Cyclin B

Cyclin C

Cyclin D

The parents of a 6-month-old girl palpate a mass on the left side of the child’s abdomen. Urinalysis shows high levels
of vanillylmandelic acid. A CT scan reveals an abdominal tumor and bony metastases. The primary tumor is surgically
resected. Histologic examination of the surgical specimen discloses neuroblastoma. Evaluation of the Nmyc
protooncogene in this child’s tumor will most likely demonstrate which of the following genetic changes?

Options

Chromosomal translocation

Exon deletion

Expansion of a trinucleotide repeat

Gene amplifi cation

Bittner milk factor is a transmissible agent belonging to the following category:

Options

Acute transforming virus

Slow transforming virus

HTLV-I

HTLV-II

A 58-year-old woman undergoes routine colonoscopy. A 2-cm submucosal nodule is identifi ed in the appendix.
Biopsy of the nodule shows nests of cells with round, uniform nuclei. Electron microscopy reveals numerous
neuroendocrine granules in the cytoplasm. This patient’s neoplastic disease is associated with which of the following
clinical features?
Options

Congestive heart failure

Flushing and wheezing

Muscular dystrophy

Progressive systemic sclerosis

A 45-year-old woman presents with abdominal pain and vaginal bleeding. A hysterectomy is performed and shows a
benign tumor of the uterus derived from a smooth muscle cell. What is the appropriate diagnosis?

Options

Angiomyolipoma

Leiomyoma

Leiomyosarcoma

Myxoma

Important examples of tumour suppressor genes implicated in human cancers include the following except:

Options

RB gene

TP53

APC

ERB-B

Cytogenetic studies in a 70-year-old woman with chronic myelogenous leukemia (CML) demonstrate a t(9;22)
chromosomal translocation. Which of the following best explains the role of this translocation in the pathogenesis of
leukemia in this patient?

Options

Altered DNA methylation status

Enhanced expression of telomerase gene

Expansion of a trinucleotide repeat

Protooncogene activation

An example of tumour-associated antigen (TAA) is:

Options

Testis specific antigen (MAGE)

Alpha-fetoprotein (AFP)

Carcinoembryonic antigen (CEA)

Prostate specific antigen (PSA)


A 33-year-old woman presents with a diffuse scaly skin rash of 4 weeks duration. Biopsy of lesional skin reveals a
cutaneous T-cell lymphoma (mycosis fungoides). Which of the following immunohistochemical markers would be
most useful for identifying malignant cells in the skin of this patient?

Options

Calcitonin

CD4

Desmin

HMB-45

Hypercalcaemia as a paraneoplastic syndrome is observed in the following tumours except:

Options

Squamous cell carcinoma lung

Small cell carcinoma lung

Renal cell carcinoma

Breast cancer

A 63-year-old woman with chronic bronchitis presents with shortness of breath. A chest X-ray reveals a 2-cm “coin
lesion” in the upper lobe of the left lung. A CT-guide lung biopsy is obtained. Which of the following describes the
histologic features of this lesion if the diagnosis is hamartoma?

Options

Benign neoplasm of epithelial origin

Disorganized normal tissue

Ectopic islands of normal tissue

Granulation tissue

Lymphocytic infiltrate is frequently present in the following tumours indicative of host immune response except:

Options

Seminoma testis

Medullary carcinoma breast

Papillary carcinoma thyroid

Malignant melanoma

A 2-year-old boy is found to have bilateral retinal tumors. Molecular studies demonstrate a germline mutation in one
allele of the Rb gene. Which of the following genetic events best explains the mechanism of carcinogenesis in this
patient?

Options

Balanced translocation
Expansion of trinucleotide repeat

Gene amplifi cation

Loss of heterozygosity

The following antibody-stain is used in immunohistochemistry to identify epithelial cells:

Options

Desmin

Vimentin

Cytokeratin

Neurofilaments

A 48-year-old nulliparous woman complains that her menstrual blood fl ow is more abundant than usual. An
ultrasound examination reveals a polypoid mass in the uterine fundus. The patient subsequently, undergoes a
hysterectomy, which reveals a poorly differentiated endometrial adenocarcinoma. The development of this
neoplasm was preceded by which of the following histopathologic changes in the glandular epithelium?

Options

Atrophy

Hydropic swelling

Hyperplasia

Hypertrophy

Which of the following viral infection is not known to produce any human tumour?

Options

Polyoma virus

EBV

HSV

HTLV

Which type of adaptation is able to convert to dysplasia and further to adenocarcinoma?

Options

Hypertrophy

Atrophy

Metaplasia

Hamartoma

All are autosomal dominant inherited cancer syndromes except:

Options
Retinoblastoma

Xeroderma pigmentosum

HNPCC

Neurofibromatosis

Metaplasia of distal end of esophagus leads to which type of cancer?

Options

Adenocarcinoma

Sarcoma

Hamartoma

Choristoma

Phosphorylation of retinoblastoma gene:

Options

Inhibits cell replication

Promotes cellular quiescence

Stops cell cycle progression

Promotes cell division

Endometrial hyperplasia may lead to which cancer type?

Options

Adenocarcinoma

Sarcoma

Hamartoma

Choristoma

P53:

Options

Activates cyclins

Activates BAX

Activates CDKs

Activates bcl2

All are matrix metalloproteinases except:

Options

Collagenase
Gelatinase

Stromelysin

Elastase

All are anti-angiogenesis factors except:

Options

Thrombospondin-1

Basic fibroblast growth factor (bFGF)

Endostatin

Angiostatin

Which of the following is a test for mutagenicity?

Options

Kveim’s test

Ame’s test

Schilling’s test

Mantoux test

All are autosomal dominant inherited cancer syndromes except:

Options

Retinoblastoma

Xeroderma pigmentosum

HNPCC

Neurofibromatosis

DNA extraction is a pre-requisite for the following molecular techniques except:

Options

PCR technique

In situ hybridisation

Western blot technique

Southern blot technique

All are methods of cell proliferation analysis except:

Options

Microspectrophotometry

Flow cytometry
PCR

Immunohistochemistry

A 54-year-old woman who has been diagnosed with early-stage breast cancer undergoes surgery for a lumpectomy
to remove a small tumor detected by mammography. The pathology report confirms the early stage of the cancer
and further comments on the fact that there is significant desmoplasia in the surrounding tissue. The term
desmoplasia refers to

Options

An irregular accumulation of blood vessels.

Maturation and spatial arrangement of cells.

Metastatic involvement of surrounding tissue.

Proliferation of non-neoplastic fibrous connective tissue.

A 24-year-old woman with a history of heavy and painful menstrual periods has been having difficulty conceiving
despite months of trying to become pregnant. Further workup includes a bimanual pelvic examination and an
ultrasound, which demonstrates a mass in the uterus that is presumed to be a leiomyoma. This mass is a

Options

Benign tumor of mesenchymal tissue.

Benign tumor of surface epithelium.

Malignant tumor of epithelial tissue.

Malignant tumor of glandular epithelium.

A 68-year-old man has a long history of prostate cancer that was metastatic at the time of diagnosis. Over the past 2
months, he has had significant weight loss, loss of appetite, and loss of energy. His current spectrum of conditions
can be attributed to which of the following?

Options

Platelet-derived growth factor

Fibroblast growth factor

Interleukin-2

Tumor necrosis factor-α

A 58-year-old man with a 700-pack-peryear smoking history presents to the emergency department with shortness
of breath and hemoptysis. Portable chest radiography demonstrates a large mass centrally located within the left
lung field. The serum calcium is 13.0 mg/dL (normal 8.5 to 10.2). The metabolic abnormality described here is likely
due to elaboration of which substance?

Options

Adrenocorticotropic hormone–like substance

Antidiuretic hormone

Carcinoembryonic antigen
Parathyroid-related hormone

An 8-year-old boy is referred to the dermatologist for numerous “suspicious” pigmented lesions on the face and
neck. Further history reveals that the patient has had difficulty seeing out of his right eye; he is referred to the
ophthalmologist, who diagnoses an ocular melanoma. Based on the patient’s symptoms, the diagnosis of xeroderma
pigmentosum is considered. This condition results from

Options

Aberrant expression of a receptor tyrosine kinase.

An inborn defect in DNA repair.

Chemical carcinogenesis.

DNA viral infection.

A 46-year-old woman with prominent splenomegaly presents with a 3-month history of malaise, easy fatigability,
weakness, weight loss, and anorexia. A complete blood count and differential demonstrates a white blood cell count
of 250,000/mm3 (normal 3,000 to 10,000/mm3) with a predominance of myelocytes, metamyelocytes, band cells,
and segmented neutrophils. Cytogenetic analysis is most likely to reveal which of the following translocations?

Options

T(8;14)

T(9;22)

T(11;22)

T(14;18)

An 18-year-old patient presents with renal cell carcinoma. Given that this is typically a tumor of older adults, what
translocation might you expect to find? This translocation is also seen in which mesenchymal malignancy?

Options

T(X;18), synovial sarcoma

T(X;17), alveolar soft part sarcoma

T(9;22), leiomyosarcoma

T(14;18), leiomyosarcoma

A 63-year-old woman discovers a lump in her right breast. Mammography confirms the presence of a suspicious
“lump,” and a needle core biopsy is performed to determine whether the mass is malignant. The pathology report
confirms that the mass is indeed cancerous and that the tissue demonstrates amplification of the Her-
2/neuoncogene. The gene product of Her-2/neu is what kind of protein?

Options

GTPase

GTPase-activating protein

Nuclear transcription factor

Receptor tyrosine kinase


A 27-year-old woman has recently been diagnosed with a glioma (a malignant brain tumor). Further family history
reveals that her 4-year-old son has been diagnosed with leukemia and has been undergoing chemotherapy. In
addition, the patient’s mother died at 36 years of age due to metastatic breast cancer. Li-Fraumeni syndrome is
suspected, given the familial clustering of this group of malignancies. The gene mutated in Li-Fraumeni syndrome
normally functions in what capacity?

Options

Activates the GTPase activity of the gene product of the Ras oncogene

Excises ultraviolet light–induced thymidine dimers

Functions as a cytoplasmic tyrosine kinase

Halts the cell cycle if DNA damage is detected

An 8-year-old child is evaluated by the pediatrician, who notes what appear to be 10 small café-au-lait spots on the
child’s torso. In addition, on close inspection of the eyes, the presence of Lisch nodules is noted. The patient is
diagnosed with von Recklinghausen neurofibromatosis type 1. The protein that is mutated in this disorder normally

Options

Activates the GTPase activity of Ras.

Cleaves cellular proteins during apoptosis.

Functions as a regulator of the cell cycle.

Promotes angiogenesis in the growing tumor mass

A 78-year-old Navy veteran with a 600-pack-per-year history of cigarette smoking presents with cancer. During his
military career, he was involved in fireproofing naval combat ships with asbestos insulation. Given his environmental
exposure to both tobacco and asbestos, to which cancer do both of these carcinogens contribute?

Options

Bladder cancer

Bronchogenic cancer

Cancer of the throat

Esophageal cancer

A 40-year-old woman presents with endometrial carcinoma. Her family history reveals that her mother died of
endometrial cancer at age 50, while her 42-year-old brother was recently diagnosed with colon cancer. You begin to
suspect a familial cancer syndrome. What gene is most likely to be mutated in this family?

Options

WT-1

APC

MSH2

P53

Teratogens are defined as agents which induce:


Options

Mitosis

Carcinogenesis

Birth defects

Fallot’s tetralogy

For chromosomal study, it is best to use the following nucleated cells:

Options

Polymorphs

Lymphocytes

Epithelial cells

Fibroblasts

For chromosomal study, the dividing cells are arrested by colchicine in the following phase of cell cycle:

Options

Prophase

Metaphase

Anaphase

Telophase

Denver classification divides chromosomes based on their length into the following groups:

Options

A to C (3 groups)

A to E (5 groups)

A to G (7 groups)

A to I (9 groups)

Polyploidy is generally not a feature of dividing cells of the following type:

Options

Megakaryocytes

Hepatocytes

Tubular cells

Conceptus of abortions

Numeric abnormality in chromosome occurs in the following conditions except:

Options
Ph chromosome in CML

Turner’s syndrome

Klinefelter’s syndrome

Down’s syndrome

Mutations affecting germ cells produce:

Options

Cancers

Inherited diseases

Congenital malformations

Aneuploidy

In lysosomal storage diseases, the following cells are particularly involved:

Options

Hepatocytes

Skeletal muscle

Macrophages

White pulp of spleen

Out of the following glycogenosis, the following is example of lysosomal storage disease:

Options

Von Gierke’s disease

Pompe’s disease

Forbe’s disease

Anderson’s disease

Blastomas are childhood tumours seen more often in the age range of:

Options

<4 years

5-9 years

10-14 years

14-16 years

All of the following are X-linked recessive disorders except:

Options

Haemophilia A and B
Chronic granulomatous disease

G-6 PD deficiency

Sickle cell anaemia

Gaucher cells are positive for all except:

Options

PAS

Mucicarmine

Oil red O

Prussian blue

A newborn male infant is examined. There is a palpable right testis, but no left testis, in the scrotal sac. The infant
has no other abnormalities noted. Which of the following is the most likely abnormality involving the gonads of this
infant?

Options

Agenesis

Fusion

Hypopituitarism

Incomplete descent

Following ovulation and fertilization of an ovum, a blastocyst develops. Growth and differentiation occurs following
implantation. Which of the following will develop into the placenta?

Options

Epimere

Neural plate

Sclerotome

Trophoblast

A 5-year-old child fatigues easily. On examination a machinery-like murmur is auscultated over his
chest.Echocardiography shows shunting of blood from the aorta to the pulmonary artery. An abnormality involving
which of the following structures is most likely present in this boy?

Options

Ductus arteriosus

Endocardial cushion

Sinus venosus

Third aortic arch


A 3500 gm infant born at term is doing well and gaining weight normally until the fourth week of life. He then
exhibits feeding difficulty with forceful vomiting. Now, on physical examination, no external anomalies are noted.
However, there is a firm mass palpable in the upper mid abdominal region. Which of the following conditions is the
infant most likely to have?

Options

Duodenal atresia

Hepatoblastoma

Hirschsprung disease

Pyloric stenosis

An infant born at term is examined, and there is abnormal fusion of the 3rd, 4th, and 5th digits of the left hand. A
radiograph of the left hand shows transformation of metacarpals I, II, III, and V to short, carpal-like bones. A
mutation involving which of the following genes most likely led to these findings?

Options

CFTR

FGFR3

HOXD13

SHH

An infant born at term has a radiograph taken. No congenital abnormalities are noted. The upper heart borders are
not visualized. Which of the following structures is most likely to obscure the heart borders in this infant's
radiograph?

Options

Esophagus

Lymph node

Stomach

Thymus

A secondary oocyte is developing normally. At which of the following times does the second maturation division of
meiosis become completed?

Options

In the ovarian cortex

During ovulation

At fertilization

In the blastocyst

During the 3rd week of embryonic development, splanchnic mesoderm differentiates into blood islands with
angioblasts. In what developing tissue site does this process occur?

Options
Heart

Liver

Spleen

Thymus

An ovum is fertilized. After 3 weeks a blastocyst implanted on the endometrium has an inner embryoblast and an
outer trophoblast. Which of the following is the next step in forming a 3-layered embryonic disk?

Options

Decidualization

Gastrulation

Lateralizaton

Neurulation

A 55-year-old man has noted sharp pain in his right lower extremity for the past 2 months. MR imaging of his spine
shows impingement on a spinal nerve root by a herniated structure located between L5 and S1. From which of the
following embryonic derivatives does this herniated structure most likely arise?

Options

Amnion

Cloaca

Lateral plate mesoderm

Neural crest

In the developing embryo, pharyngeal arches are formed. The second (branchial) arch gives rise to the stapedius,
orbicularis oris, and posterior belly of the digastric muscles. Which of the following cranial nerves is also derived
from this arch?

Options

III

VII

IX

In the normally developing embryo, aortic arches are apparent by the end of the fourth week. Which of the following
arteries is derived from the third pair of aortic arches?

Options

Carotid

Pulmonary

Stapedial
Subclavian

A fetus at 18 weeks gestation is normally formed and appropriately sized. No fetal or placental anomalies are
present. Which of the following parts of the fetal vasculature is most likely to have the greatest oxygen
concentration?

Options

Aorta

Ductus arteriosus

Pulmonary vein

Umbilical vein

An embryonic eye is developing normally. A retina is present on the posterior aspect of the globe. What is the
embryologic origin of the portion of retina containing nerve cell bodies?

Options

Diencephalon

Mesoderm

Notochord

Pharyngeal arch

A newborn examination is performed. The tympanic membranes are visualized bilaterally. What is the embryologic
origin of the structure that forms the canal connecting the external ear to the tympanic membrane?

Options

Bony labyrinth

First pharyngeal cleft

Endolymphatic duct

Otic pit

A 20-year-old woman is unable to conceive and undergoes an infertility workup. A pelvic ultrasound reveals a
bicornuate uterus. Which of the following is the most likely explanation for her anomalous uterus?

Options

Excess circulating maternal androgens while in utero

Failure of primordial germ cell migration

Failure of the paramesonephric ducts to fuse

Failure of the urorectal septum to reach the perineum

In a developing embryo, neural crest cells form adjacent to the neural tube. These neural crest cells then undergo
migration. Which of the following structures is derived from these migrating cells?

Options
Dorsal root ganglion

Dura mater

Epidermis covering the back

Intervertebral disk

A neonate undergoes a check of health status. On examination a faint cardiac murmur is


auscultated.Echocardiography is performed and shunting of blood is noted from the left atrium to the right atrium.
An abnormality in embryonic development of which of the following structures has most likely occurred in this
neonate?

Options

Ductus arteriosus

Endocardial cushion

Fourth aortic arch

Sinus venosus

The two superior and two inferior parathyroid glands are found adjacent to the posterior aspect of the thyroid gland.
From which of the following are these parathyroid glands derived in embryologic development?

Options

Foramen cecum

Mandibular swellings

Maxillary processes

Pharyngeal pouches

A 31-year-old woman has noted no fetal movement by 16 weeks gestation. A fetal ultrasound shows the fetal head
contains a brain with thin cortex, single enlarged ventricle, and fused thalami. Which of the following developing
regions of the brain is most likely abnormal in this fetus?

Options

Cranial vault

Mesencephalon

Prosencephalon

Rhombencephalon

Following fertilization of the ovum, the ovarian corpus luteum continues to produce progesterone. If the corpus
luteum regresses in the first trimester, abortion occurs. Which of the following hormones maintains this capability of
the corpus luteum?

Options

Alpha-fetoprotein

Estradiol
Human chorionic gonadotropin

Human placental lactogen

Following coitus, spermatozoa migrate to the fallopian tubes. Within the tubes, the acrosomal head of spermatozoa
undergoes reduction of glycoprotein coating with increasing permeability to calcium. Which of the following
functions of spermatozoa is facilitated by this process?

Options

Blocking other sperm

Fertilization

Motility

Mitosis

A 24-year-old primigravida has felt no fetal movement at 16 weeks gestation. A screening fetal ultrasound shows
anhydramnios and a single poorly formed lower extremity. An abnormality involving which of the following
structures most likely led to these findings?

Options

Aorta

Dermis

Neural crest

Splanchnic mesoderm

A neonate born at term after an uncomplicated pregnancy is noted to pass pale coloured stool and dark urine. The
infant becomes progressively jaundiced in the next 2 days. On examination the infant is at the 50th percentile for
weight. Icterus is present. The serum IgM antibody titer to cytomegalovirus is increased. Which of the following
developmental abnormalities most likely occurred in utero?

Options

Accessory pancreas

Biliary atresia

Gallbladder duplication

Hepatic agenesis

Following birth, circulation from placenta to fetus ceases. Which of the following fetal structures that conducts blood
to the inferior vena cava becomes atretic in postnatal life?

Options

Azygous vein

Ductus arteriosus

Ductus venosus

Foramen ovale
A newborn male infant is examined. There is a palpable right testis, but no left testis, in the scrotal sac. The infant
has no other abnormalities noted. Which of the following is the most likely abnormality involving the gonads of this
infant?

Options

Agenesis

Fusion

Hypopituitarism

Incomplete descent

How soon after fertilization occurs within the uterine tube does the blastocyst begin implantation?

Options

Within minutes

By 12 hours

By day 1

By day 7

Where does the blastocyst normally implant?

Options

Functional layer of the cervix

Functional layer of the endometrium

Basal layer of the endometrium

Myometrium

Which of the following events is involved in the cleavage of the zygote during week 1 of development?

Options

A series of meiotic divisions forming blastomeres

Production of highly differentiated blastomeres

An increased cytoplasmic content of blastomeres

A decrease in size of blastomeres

Which of the following structures must degenerate for blastocyst implantation to occur?

Options

Endometrium in progestational phase

Zona pellucida

Syncytiotrophoblast

Cytotrophoblast
Which of the following is the origin of the mitochondrial DNA of all human adult cells?

Options

Paternal only

Maternal only

A combination of paternal and maternal

Either paternal or maternal

Individual blastomeres were isolated from a blastula at the 4-cell stage. Each blastomere was cultured in vitro to the
blastocyst stage and individually implanted into four pseudopregnant foster mothers. Which of the following would
you expect to observe 9 months later?

Options

Birth of one baby

Birth of four genetically different babies

Birth of four genetically identical babies

Birth of four grotesquely deformed babies

During the later stages of pregnancy, maternal blood is separated from fetal blood by the

Options

Syncytiotrophoblast only

Cytotrophoblast only

Syncytiotrophoblast and cytotrophoblast

Syncytiotrophoblast and fetal endothelium

The maternal and fetal components of the placenta are

Options

Decidua basalis and secondary chorionic villi

Decidua capsularis and secondary chorionic villi

Decidua parietalis and tertiary chorionic villi

Decidua basalis and villous chorion

The intervillous space of the placenta contains

Options

Maternal blood

Fetal blood

Maternal and fetal blood

Amniotic fluid
A young insulin-dependent diabetic woman in her first pregnancy is concerned that her daily injection of insulin will
cause a congenital malformation in her baby. What should the physician tell her?

Options

Insulin is highly teratogenic; discontinue treatment

Insulin does not cross the placental membrane

Insulin crosses the placental membrane but is degraded rapidly

Insulin will benefit her baby by increasing glucose metabolism

What is a normal amount of amniotic fluid at term?

Options

50 mL

500 mL

1000 mL

1500 mL

Which of the following does not pass through the primitive umbilical ring?

Options

Allantois

Amnion

Yolk sac

Connecting stalk

Which of the following best describes the placental components of dizygotic twins?

Options

One placenta, two amniotic sacs, one chorion

One placenta, two amniotic sacs, two chorions

Two placentas, two amniotic sacs, one chorion

Two placentas, two amniotic sacs, two chorions

A 26-year-old pregnant woman experiences repeated episodes of bright red vaginal bleeding at week 28, week 32,
and week 34 of pregnancy. The bleeding spontaneously subsided each time. Use of ultrasound shows that the
placenta is located in the lower right portion of the uterus over the internal os. What is the diagnosis?

Options

Hydatidiform mole

Vasa previa

Placenta previa
Placental abruption

A 19-year-old woman in week 32 of a complication-free pregnancy is rushed to the emergency department because
of profuse vaginal bleeding. The bleeding subsides, but afterward no fetal heart sounds can be heard, indicating
intrauterine fetal death. The woman goes into labor and delivers a stillborn infant. On examination of the afterbirth,
a velamentous placenta is detected. Although not much can be done at this point, what is the diagnosis?

Options

Placenta previa

Vasa previa

Hydatidiform mole

Premature rupture of the amniochorionic membrane

A 32-year-old pregnant woman at 30 weeks of gestation comes to her physician because of excess weight gain in a 2-
week period. Ultrasonography reveals polyhydramnios. Which fetal abnormality is most likely responsible for the
polyhydramnios?

Options

Bilateral kidney agenesis

Umbilical cord knots

Velamentous placenta

Esophageal atresia

A 25-year-old pregnant woman at 17 weeks of gestation comes to her OB/GYN for a normal examination. During
routine blood tests, her serum α-fetoprotein (AFP) concentration is found to be markedly decreased for her
gestational age. Which abnormality will the physician need to rule out based on these low AFP levels?

Options

Spina bifida

Anencephaly

Omphalocele

Esophageal atresia

Which of the following time intervals best describes the maximum susceptibility period?

Options

Week 1

Weeks 3–8

Weeks 9–38

Weeks 15-17

Which of the following time intervals best describes the resistant period?

Options
Week 1

Weeks 3–8

Weeks 9–38

Weeks 15-17

The most common viral infection is

Options

Cytomegalovirus

Rubella virus

Herpes virus type 2

Varicella zoster virus

Which of the following is a parasite found in cats?

Options

Treponema pallidum

Toxoplasma gondii

Rubella virus

Cytomegalovirus

Warfarin falls into which category of drugs?

Options

Category X drugs

Category D drugs

XX

Not either

Valium falls into which category of drugs?

Options

Category X drugs

Category D drugs

XX

Not either

Teratogens are defined as agents which induce:

Options

Mitosis
Carcinogenesis

Birth defects

Fallot’s tetralogy

For chromosomal study, it is best to use the following nucleated cells:

Options

Polymorphs

Lymphocytes

Epithelial cells

Fibroblasts

For chromosomal study, the dividing cells are arrested by colchicine in the following phase of cell cycle:

Options

Prophase

Metaphase

Anaphase

Telophase

Denver classification divides chromosomes based on their length into the following groups:

Options

A to C (3 groups)

A to E (5 groups)

A to G (7 groups)

A to I (9 groups)

Polyploidy is generally not a feature of dividing cells of the following type:

Options

Megakaryocytes

Hepatocytes

Tubular cells

Conceptus of abortions

Numeric abnormality in chromosome occurs in the following conditions except:

Options

Ph chromosome in CML

Turner’s syndrome
Klinefelter’s syndrome

Down’s syndrome

Mutations affecting germ cells produce:

Options

Cancers

Inherited diseases

Congenital malformations

Aneuploidy

In lysosomal storage diseases, the following cells are particularly involved:

Options

Hepatocytes

Skeletal muscle

Macrophages

White pulp of spleen

Out of the following glycogenosis, the following is example of lysosomal storage disease:

Options

Von Gierke’s disease

Pompe’s disease

Forbe’s disease

Anderson’s disease

Blastomas are childhood tumours seen more often in the age range of:

Options

<4 years

5-9 years

10-14 years

14-16 years

All of the following are X-linked recessive disorders except:

Options

Haemophilia A and B

Chronic granulomatous disease

G-6 PD deficiency
Sickle cell anaemia

Gaucher cells are positive for all except:

Options

PAS

Mucicarmine

Oil red O

Prussian blue

The outcome of the Liquefactive necrosis of the brain can be:

Petrification

Ossification

Cyst formation

Scarring
The organization is a favorable outcome of:

Options

Embolism

Necrosis

Pathological accumulation (dystrophy)

Induration

The organization is the outcome of necrosis, characterized by:

Options

Replacement with connective tissue

Formation of a capsule

Deposition of calcium

Suppuration

Encapsulation is the outcome of necrosis, characterized by:

Options

Replacement with connective tissue

Formation of a capsule

Deposition of calcium

Formation of a cyst

Petrification is the outcome of necrosis, characterized by:

Options

Replacement with connective tissue

Formation of a capsule

Deposition of calcium

Formation of a cyst

Outcome of necrosis can be:

Options

Organization

Embolism

Infarction

Thrombosis

Definition of "necrosis":
Options

Anemia of the organ

Dystrophy

Death of an organism

Necrosis of tissues in a living body

Microscopic signs of cell necrosis:

Options

Fatty degeneration

Karyopicnosis

Karyokinesis

Degranulation

Favorable outcome of dry necrosis:

Options

Organization

Septic autolysis

Regeneration

Cyst formation

Type of gangrene:

Options

Dense

Dry

Partial

Toxic

Definition of the concept of "necrobiosis":

Options

Spasm of blood vessels

Anemia of organs

Irreversible dystrophic processes

Autolysis

Microscopic signs of cell necrosis:

Options
Plasmorrhagia

Plasmatization

Karyorexis

Karyokinesis

Characteristic outcome of wet necrosis:

Options

Encapsulation

Petification

Mummification

Cyst

In which organs the development of gangrene is possible:

Options

The brain

The spleen

Intestine

Heart

Select the stages of the necrotic process, EXCEPT:

Options

Plasmolysis

Plasmorexis

Necrobiosis

Karyorexis

Enzyme that take part in the autolysis of cell nuclei:

Options

Acidic phosphatase

DNAase

Alkaline phosphatase

Cytochrome oxidase

Etiological type of necrosis:

Options

Infarction
Dry

Sequestration

Trophoneurotic

Characteristic of tissues with dry gangrene:

Options

Edema

Mummification

Anemia

Fullness

Characteristic of the infarction:

Options

Traumatic necrosis

Direct necrosis

Vascular necrosis

Caseous necrosis

Cell ultrastructure that determine autolytic enzymatic processes in a cell:

Options

Golgi apparatus

Mitochondria

Lysosomes

Endoplasmic reticulum

Causes of direct necrosis:

Options

Nerve damage

Standstill of blood flow

The effect of acids

Rupture of blood vessels

Frequent localization of dry gangrene:

Options

Extremities

Intestine
Heart

Liver

Characteristic of vascular necrosis:

Options

Direct

Traumatic

Indirect

Trophoneurotic

Characteristics of bedsore:

Options

Traumatic necrosis

A kind of gangrene

Toxic necrosis

Myocardial infarction

What is formed as a result of necrosis of cells and extracellular substance:

Options

Pigments

Polysaccharides

Hemoglobin

Tissue detritus

What necrosis developed under the action of chemical and physical factors:

Options

Mechanical

Vascular

Trophoneurotic

Traumatic

Frequent localization of wet gangrene:

Options

Intestine

Spinal cord

Umbilical cord
Spleen

What is infarction:

Options

Toxic necrosis

Allergic necrosis

Direct necrosis

Vascular necrosis

What develops around the focus of necrosis in living tissues:

Options

Autolysis

Tissue anemia

Demarcation inflammation

Necrobiosis

Cause of Vascular Necrosis:

Options

Arterial hyperemia

Anemia

Thrombosis of the vessel

Hemorrhage

The process that constitute the essence of necrosis:

Options

Chronic venous congestion

Tissue anemia

Protein coagulation

Sclerosis

The substance that determines the color of gangrenous tissues:

Options

Hemomelanin

Lipofuscin

Melanin

Sulphurous iron
What is necrosis:

Options

Death of an organism

Death of organs and tissues in the living body

Calcification of tissues

Standstill of breathing

Adverse outcome of necrosis:

Options

Suppuration

Ossification

Organization

Petification

Frequent localization of Liquefactive necrosis:

Options

Liver

Brain

Myocardium

Kidneys

Classification of necrosis by the mechanism of action of the etiologic factor:

Options

Direct

Gangrene

Sequestration

Infarction

What necrosis develops with the predominance of colliquation processes:

Options

Coagulation

Dry

Wet

Caseous

Diseases in which fibrinoid necrosis often develops:


Options

Intestinal infections

Rheumatic diseases

Influenza

Leukemia

Synonym of vascular necrosis:

Options

Infarction

Dystonic

Allergic

Dystrophic

Clinico - morphological forms of necrosis:

Options

Direct

Indirect

Infarction

Vascular

What necrosis develops in a sensitized organism:

Options

Trophoneurotic

Toxic

Allergic

Traumatic

Characteristic of the demarcation zone:

Options

Zone of ischemia

Accumulation of leukocytes and hyperemia

Accumulation of erythrocytes

Vascular spasm

The disease, accompanied by the development of caseous necrosis:

Options
Intestinal infections

Atherosclerosis

Tuberculosis

Rheumatism

Choose the favorable outcome of necrosis:

Options

Sepsis

Septic melting

Encapsulation

Mummification

What necrosis develops from exposure to low and high temperatures:

Options

Direct

Indirect

Trophoneurotic

Vascular

A variety of coagulation necrosis:

Options

Sequestration

Caseous

Colliquative

Bedsore

What etiological kind of necrosis develops under the phenomenon of Arthus:

Options

Vascular

Toxic

Traumatic

Allergic

Typical localization of wet necrosis:

Options

Myocardium
Brain

Smooth muscles

Striated musculature

Changes in the stroma of the organ with necrosis:

Options

Ischemia

Hyalinosis

Amyloidosis

Fibrinoid necrosis

A synonym of focal colliquative necrosis of the cell:

Options

Fatty degeneration

Granular degeneration

Balloon dystrophy

Hyaline-drop dystrophy

Frequent localization of infarction:

Options

Lower limbs

Heart

Liver

Stomach

At what disease often there is a vascular necrosis:

Options

Tuberculosis

Atherosclerosis

Influenza

Anemia

What is sequestration:

Options

A kind of gangrene

Etiological kind of necrosis


A kind of calcification

Area of necrosis not subjected to autolysis

Frequent localization of fibrinoid necrosis:

Options

Muscles

Nervous tissue

Bone tissue

The wall of the vessel

Typical localization of Liquefactive necrosis:

Options

Bone tissue

Connective tissue

Brain

Myocardium

What necrosis develops with tuberculosis and syphilis:

Options

Waxy

Caseous

Vascular

Traumatic

A type of cell damage is:

Options

Metaplasia

Atrophy

Dysplasia

Necrosis

At autopsy performed on the 3rd day after the death of a patient with myocardial infarction, signs of autolysis in all
organs were found macroscopically. For differential diagnosis between necrosis and post-mortem autolysis, a
histological sign can be used:

Options

Caryolysis

Karyorexis
Plasmolysis

Demarcation inflammation

Histologically, in necrotic cells:

Options

Nuclei stained with eosin more

Cytoplasm is less basophilic than usual

Nuclei stained with hematoxylin more

Cytoplasm is stained with hematoxylin more

The level of serum creatine kinase increases with necrosis of:

Options

Kidneys

Myocardium

Brain

Pancreas

Select the sign of apoptosis:

Options

Cariolysis

Activation of endonucleases

Activation of DNA synthesis

Demarcation inflammation

The programmed cell death, which occurs normally in the fetal organs, is called:

Options

Apoptosis

Heterolysis

Autolysis

Heterophagy

Marker of irreversible damage to cardiomyocytes during ischemia is:

Options

Cell swelling

Condensation of chromatin

Calcium deposits in the mitochondria


Disappearance of glycogen from the cytoplasm

Red infarction develops as a result of:

Options

Splenic artery thrombosis

Carotid thrombosis

Coronary artery thrombosis

Embolism of the pulmonary artery branch

Geometric form of myocardial infarction:

Options

Round

Oval

Triangular

Irregular

The demarcation line with dry gangrene is:

Options

Clear

Unclear

Absent

Irregular

The demarcation line with wet gangrene is:

Options

Clear

Unclear

Absent

Hemorragic

Wet gangrene of the foot often develops in patients with:

Options

Vasculitis

Pancreatitis

Atherosclerosis

Diabetes mellitus
Dry gangrene of the foot often develops in the patient with:

Options

Pneumonia

Pancreatitis

Atherosclerosis

Diabetes mellitus

Fatty (enzymatic) necrosis most often develops in patients with:

Options

Hepatitis

Enteritis

Gastritis

Pancreatitis

Lymph nodes in tuberculosis are:

Options

Dark red and enlarged

White and yellow and enlarged

Dark red and reduced

White and yellow and reduced

In the myocardium with ischemia disappear:

Options

Drops of lipids

Granules of glucose

Granules of Glycogen

Lipofuscin granules

Apoptotic bodies are exposed by:

Options

Autolysis

Heterolysis

Encapsulation

Phagocytosis

Changes in the nucleus of the cell during apoptosis occur under the action of:
Options

Aminotransferases

Hydrolytic enzymes

Proteolytic enzymes

Ca-Mg-dependent endonucleases

Cells phagocytizing apoptotic bodies are:

Options

Macrophages

Fibroblasts

Lymphocytes

Eosinophils

Component of apoptotic bodies is:

Options

Nucleus with nucleolus

Lipid vacuoles

Giant mitochondria

Condensed fragments of chromatin

Apoptotic bodies histologically look like:

Options

Granular balls

Basophilic blots

Basophilic oval corpuscles

Eosinophilic rounded masses

Apoptotic bodies in the tissue are best seen in stainig:

Options

Sudan III

Hematoxylin and eosin

Picrofuxin

Schiff (PAS) -reaction

Type of cell death is:

Options
Necrosis

Hyperhidrosis

Mummification

Swelling

The main morphological signs of apoptosis are all, EXEPT:

Options

Cell shrinkage

Damage to organelles

Preservation of integrity of organelles

Formation of apoptic bodies

Physical agents that cause cell damage are all, EXEPT:

Options

Radiation

Mechanical trauma

Electric current

Acid and alkali

Histological signs of necrosis after total myocardial ischemia appear through:

Options

24 hours

10 min

10 - 12 hours

20 - 60 minutes

The most important mechanism of cell damage and death:

Options

Lipolysis

Phagocytosis

Depletion of ATP stocks

Vacuolation of the cytoplasm

The most common cause of ischemic and hypoxic damage is:

Options

Vasodilation
Blockage of veins

Occlusion of the artery

Development of collaterals

If the infarct reaches the surface of the organ, covered with a serous membrane, it develops

Options

Edema

Hyalinosis

Granulation tissue

Fibrinous inflammation

Choose positions that are correct with respect to gangrene:

Options

Gangrene - necrosis of tissues in contact with the external environment

Sequestration - a kind of gangrene

Gangrene of the intestine is always dry

Gangrene of the extremities can be only wet

Choose the positions that are correct for necrosis:

Options

The cytoplasm of necrotic cells is less eosinophilic

Pycnotic nuclei stained with hematoxylin more

Fat necrosis in the pancreas in pancreatitis is calcified

With caseous necrosis, cells retain their shape

Each type of necrosis is correctly correlated with the organs for which it is typical, except for

Options

Coagulation necrosis - heart, kidneys

Liquefactive necrosis - brain

Caseous necrosis - a variety of organs

Fat necrosis - muscles of the anterior wall of the abdomen

What histological sign can be used for differential diagnosis between necrosis and post mortem autolysis:

Options

Karyolysis

Plasmorexis
Karyorexis

Demarcation inflammation

Indicate reversible changes in the structure of the cell in response to damage:

Options

Dissociation of the polisomes

Loss of microvilli

Mitochondrial swelling

Violation of the integrity of cytoplasmic membranes

Mechanism of pathogenic action of clostridium perfringens is:

Options

Damage to glycolysis

Activation of glycolysis

Inactivation of cytochrome oxidase

Development of "aggressive" phospholipids

Gangrene can develop in all, EXEPT:

Options

Kidney

Uterus

Intestine

Lungs

The development of both coagulation and liquefactive necrosis is possible in:

Options

Kidney

Intestine

Lungs

Soft tissues of extremities

Which type of pneumoconiosis is associated with inhalation of carbon particles?

Options

Anthracosis.

Silicosis.

Asbestosis.
Siderosis.

Which type of pneumoconiosis is associated with inhalation of iron particles?

Options

Anthracosis.

Silicosis

Asbestosis

Siderosis

Inhalation of foreign particles leads to development in the lung:

Options

Acute purulent pneumonia.

Obstructive emphysema.

Chronic inflammation and fibrosis

Acute abscess.

Exogenic pigmentation as a result of inhalation of the foreign particles is:

Options

Argyria.

Asbestosis.

Carotenemia.

Lead poisoning.

Exogenic pigmentation as a result of inhalation of the foreign particles is:

Options

Argyria.

Tattoing.

Carotenemia.

Anthracosis.

Exogenic pigmentation as a result of inhalation of the foreign particles is:

Options

Silicosis.

Tattoing.

Carotenemia.

Lead poisoning.
Exogenic pigmentation as a result of inhalation of the foreign particles is:

Options

Carotenemia.

Tattoing

Siderosis

Lead poisoning.

Exogenic pigmentation as a result of inhalation of the foreign particles is:

Options

Carotenemia.

Tattoing.

Argyria.

Talcosis.

Exogenic pigmentation as a result of ingestion of the foreign substances is:

Options

Argyria.

Silicosis.

Asbestosis.

Siderosis.

Exogenic pigmentation as a result of ingestion of the foreign substances is:

Options

Anthracosis.

Silicosis.

Carotenemia.

Siderosis.

Exogenic pigmentation as a result of ingestion of the foreign substances is:

Options

Anthracosis.

Lead poisoning.

Silicosis.

Siderosis.

Exogenic pigmentation as a result of injection of the foreign substances is:


Options

Anthracosis.

Lead poisoning.

Silicosis.

Tattoing.

Argyria is as a result of ingestion of the:

Options

Iron

Silver

Gold

Aluminium

Characteristic blue lines on teeth at the gumline occurs in:

Options

Chronic lead poisoning.

Chronic iron poisoning.

Chronic silver poisoning.

Chronic gold poisoning.

Brownish pigmentation in the skin, bowel and kidney occurs in:

Options

Argyria.

Tattoing.

Carotenemia.

Lead poisoning.

Yellowish-red coloration of the skin occurs in:

Options

Argyria.

Tattoing.

Carotenemia.

Lead poisoning.

Tattoing is a result of introduction into the dermis:

Options
Iron.

Silver.

Copper.

Carbon.

Tattoing is a result of introduction into the dermis:

Options

Iron

Silver

Copper

Cinnabar

Tattoing is a result of introduction into the dermis:

Options

Iron

India ink.

Copper

Asbestos

Exogenic pigmentation is:

Options

Hemosiderosis

Melanosis

Lipofuscinosis

Anthracosis

Exogenic pigmentation is:

Options

Hemosiderosis.

Melanosis.

Lipofuscinosis.

Asbestosis

Exogenic pigmentation is:

Options

Siderosis
Melanosis

Lipofuscinosis

Jaundice

Exogenic pigmentation is:

Options

Argyria

Ferritinemia

Porphyria

Jaundice

Exogenic pigmentation is:

Options

Porphyria

Ferritinemia

Carotenemia

Jaundice

Exogenic pigmentation is:

Options

Porphyria

Ferritinemia

Tattoing

Jaundice

Endogenic pigmentation is:

Options

Argyria

Ferritinemia

Carotenemia

Tattoing

Endogenic pigmentation is:

Options

Argyria

Silicosis
Carotenemia

Hemosiderosis

Endogenic pigmentation is:

Options

Argyria

Silicosis

Carotenemia

Jaundice

Endogenic pigmentation is:

Options

Argyria

Silicosis

Siderosis

Melanosis

Endogenic pigmentation is:

Options

Argyria

Silicosis

Siderosis

Lipofuscinosis

Hemosiderin histologically appears as:

Options

Golden-yellow to brown pigment.

Pinkish to red pigment.

Pale-green to dark-green pigment.

Dark-blue to violet pigment.

“Heart failure cells” are the alveolar macrophages, containing pigment which is called:

Options

Hemosiderin

Hematin

Ferritin
Bilirubin

Cause of generalized hemosiderosis is:

Options

Intracerebral hemorrhage.

Pulmonary hemorrhagic infarct.

Brown induration of lungs.

Increased absorbtions of dietary iron.

Cause of generalized hemosiderosis is:

Options

Intracerebral hemorrhage.

Pulmonary hemorrhagic infarct.

Brown induration of lungs.

Impaired utilization of iron.

Cause of generalized hemosiderosis is:

Options

Intracerebral hemorrhage.

Pulmonary hemorrhagic infarct.

Brown induration of lungs.

Repeated transfusions of blood.

Hereditary disease which associated with excessive intestinal absorbtion of iron and deposition of hemosiderin is
called:

Options

Hemosiderosis.

Lipofuscinosis.

Melanosis.

Hemachromatosis.

Morphological features of hemachromatosis are all the following, EXCEPT:

Options

Pigmentary liver cirrhosis.

Myocardial infarction.

Pigmentary cardiomyopathy.
Diabetes mellitus.

Cause of prehepatic jaundice is:

Options

Hepatitis.

Liver cirrhosis.

Obstruction of bile ducts by stones.

Intoxications.

Cause of prehepatic jaundice is:

Options

Hepatitis.

Liver cirrhosis.

Obstruction of bile ducts by stones.

Infections.

Cause of hepatocellular jaundice is:

Options

Obstruction of bile ducts by stones.

Compression of bile ducts by tumors.

Heterohemotransfusions.

Hepatosis

Cause of posthepatic jaundice is:

Options

Atresia of bile ducts.

Hepatitis.

Heterohemotransfusions.

Liver cirrhosis.

Cause of posthepatic jaundice is:

Options

Stenosis of bile ducts.

Hepatitis.

Heterohemotransfusions.

Liver cirrhosis.
Pigment which is a result of reaction between hemoglobin and hydrochloric acid is called:

Options

Hemosiderin.

Hemin

Ferritin

Bilirubin

Pigment which is a result of reaction between hemoglobin and malarial toxin is called:

Options

Hemosiderin

Hemin

Hemomelanin

Bilirubin

Histologically hematin may be seen as:

Options

Yellow granules.

Red granules.

Green granules.

Black granules.

Pigment which is formed in tissues where a good oxygen supply is lacking in dead tissues is called:

Options

Hemosiderin.

Hemin.

Hemomelanin.

Hematoidin.

Histologically hematoidin may be seen as amorphous:

Options

Yellow granules.

Red granules.

Green granules.

Black granules.

Pigment which is associated with highly photosensitivity and damage of skin is called:
Options

Hemosiderin.

Hemin.

Hemomelanin.

Porphyrin.

Generalized acquired hyperpigmentation of melanin occurs in:

Options

Addison’s disease.

Melanosis coli.

Lentigo.

Nevus.

Generalized acquired hyperpigmentation of melanin occurs in:

Options

Avitaminosis.

Melanosis coli.

Lentigo.

Nevus.

Generalized acquired hyperpigmentation of melanin occurs in:

Options

Melanosis coli.

Cachexia.

Lentigo

Nevus

Generalized acquired hyperpigmentation of melanin occurs in:

Options

Melanosis coli.

Chronic arsenical poisoning.

Lentigo.

Nevus.

Generalized congenital hyperpigmentation of melanin occurs in:

Options
Melanosis coli.

Chronic arsenical poisoning.

Xeroderma pigmentosum.

Nevus.

Focal acquired hyperpigmentation of melanin occurs in:

Options

Addison’s disease.

Cachexia.

Melanosis coli.

Nevus.

Focal congenital hyperpigmentation of melanin occurs in:

Options

Addison’s disease.

Cachexia.

Avitaminosis.

Nevus.

Focal congenital hyperpigmentation of melanin occurs in:

Options

Addison’s disease.

Cachexia.

Avitaminosis.

Melanoma.

General hypopigmentation of melanin occurs in:

Options

Leukoderma.

Albinism.

Vitiligo.

Cachexia.

Focal acquired hypopigmentation of melanin is called:

Options

Albinism
Nevus

Leukoderma

Cachexia

Focal congenital hypopigmentation of melanin is called:

Options

Albinism.

Nevus.

Vitiligo.

Cachexia.

Albinism is associated with all the following signs, EXCEPT:

Options

Blond hair.

White skin.

Poor vision.

Arterial hypertension.

Leukoderma is a sign of:

Options

Syphilis.

Typhoid fever.

Yellow fever.

Antrax.

Leukoderma is a sign of all the following diseases, EXCEPT:

Options

Syphilis

Leprosy

Radiation dermatitis.

Gonorrhea

Distrophic calcification occurs in:

Options

Hyperparathyroidism.

Hypervitaminosis D.
Hyperthyroidism.

Atheromas of advanced atherosclerosis.

Distrophic calcification occurs in:

Options

Hyperparathyroidism.

Hypervitaminosis D.

Hyperthyroidism.

Old thrombi.

Distrophic calcification occurs in:

Options

Hyperparathyroidism.

Hypervitaminosis D.

Hyperthyroidism.

Dead animal parasites.

Metastatic calcification occurs in:

Options

Hyperthyroidism

Damaged heart valves.

Atheromas.

Necrosis.

Metastatic calcification occurs in:

Options

Atheromas.

Damaged heart valves.

Old thrombi.

Hypervitaminosis D.

Metastatic calcification occurs in:

Options

Atheromas.

Damaged heart valves.

Old thrombi.
Increased bone catabolism.

Metastatic calcification occurs in:

Options

Atheromas.

Damaged heart valves.

Old thrombi.

Decreased bone formation.

Common localization of metastatic calcification is in all the following organs, EXCEPT:

Options

Blood vessels.

Brain.

Kidneys.

Lungs.

Derangement of nucleoproteids metabolism is associated with:

Options

Gout.

Atheroma.

Nevus

Hemachromatosis.

Derangement of nucleoproteids metabolism is associated with:

Options

Urolithiasis.

Atheroma.

Hemachromatosis

Jaundice.

Derangement of nucleoproteids metabolism is associated with:

Options

Cerebral infarction.

Myocardial infarction.

Urate infarction of newborn.

Pulmonary hemorrhagic infarction.


Chemical substances which occurs in urinary stones are of all the following, EXCEPT:

Options

Uric acid.

Calcium.

Magnesium.

Cholesterol.

Сongenital absence of thymus is called:

Options

Aplasia.

Hypoplasia.

Dysplasia

Atrophy.

Congenital incomplete development of thymus is called:

Options

Aplasia.

Hypoplasia.

Dysplasia.

Atrophy.

Aquired decrease in weight of thymus is called:

Options

Aplasia.

Hypoplasia.

Dysplasia.

Atrophy.

Abnormal development of thymus is called:

Options

Aplasia.

Hypoplasia.

Dysplasia.

Atrophy.
Decrease in size and weight of thymus under different stress situations including infectious diseases, intoxications,
traumas is called:

Options

Aplasia.

Hypoplasia.

Dysplasia.

Accidental involution.

Increase in size and weight of thymus more than age level with normal histological structure is called:

Options

Aplasia.

Hypoplasia.

Thymomegaly.

Atrophy.

Collection of B-lymphocytes and plasma cells with formation of lymphoid follicles in the intralobular perivascular
spaces of thymus parenchyma is called:

Options

Aplasia.

Hypoplasia.

Thymomegaly.

Hyperplasia with lymphoid follicles.

Changes in peripheric lymphoid tissue under antigene stimulation include are all the following processes, EXCEPT:

Options

Macrophageal reaction.

Hyperplasia of lymphocytes.

Plasma cell transformation.

Infarction.

Changes in peripheric lymphoid tissue under antigene stimulation include are all the following processes, EXCEPT:

Options

Macrophageal reaction.

Hyperplasia of lymphocytes.

Dysplasia of lymphocytes.

Plasma cell transformation.


Changes in peripheric lymphoid tissue under antigene stimulation include are all the following processes, EXCEPT:

Options

Macrophageal reaction.

Anaplastic transformation of lymphocytes.

Hyperplasia of lymphocytes.

Plasma cell transformation.

Changes in lymph nodes under antigene stimulation include are all the following processes, EXCEPT:

Options

Hyperemia.

Edema.

Accumulation of plasma cells.

Accumulation of neutrophils.

Changes in lymph nodes under antigene stimulation include are all the following processes, EXCEPT:

Options

Hyperemia.

Ischemia.

Edema.

Accumulation of plasma cells.

Changes in spleen under antigene stimulation include are all the following processes, EXCEPT:

Options

Hypoplasia of red pulp.

Hyperplasia of red pulp.

Plasmatization of red pulp

Accumulaton of macrophages.

Changes in spleen under antigene stimulation include are all the following processes, EXCEPT:

Options

Hyperplasia of red pulp.

Hypoplasia of follicles.

Plasmatization of red pulp.

Accumulaton of macrophages.

Changes in spleen under antigene stimulation include are all the following processes, EXCEPT:
Options

Hyperplasia of red pulp.

Plasmatization of red pulp.

Accumulaton of giant cells.

Accumulaton of macrophages.

Changes in bone marrow under antigene stimulation include are all the following processes, EXCEPT:

Options

Hyperplasia.

Macrophageal-plasma cell transformation.

Myeloid metaplasia.

Hypoplasia.

Changes in bone marrow under antigene stimulation include are all the following processes, EXCEPT:

Options

Hyperplasia.

Macrophageal-plasma cell transformation.

Myeloid metaplasia.

Myeloid dysplasia.

Changes in peripheric lymphoid tissue in congenital immunodeficiency syndromes are characterized by:

Options

Decrease in size of follicles of spleen.

Increase in size of follicles of spleen.

Hyperplasia of lymph nodes.

Myeloid metaplasia of bone marrow.

Changes in peripheric lymphoid tissue in congenital immunodeficiency syndromes are characterized by:

Options

Increase in size of follicles of spleen.

Hyperplasia of lymph nodes.

Absence of B-zones in cortex of lymph nodes.

Myeloid metaplasia of bone marrow.

Changes in peripheric lymphoid tissue in congenital immunodeficiency syndromes are characterized by:

Options
Increase in size of follicles of spleen.

Absence of germinal centers in follicles of spleen.

Hyperplasia of lymph nodes.

Myeloid metaplasia of bone marrow.

Immediate type of hypersensitivity morphologically occurs like:

Options

Acute immune inflammation

Purulent inflammation.

Hemorragic inflammation.

Chronic immune inflammation.

Delayed type of hypersensitivity morphologically occurs as:

Options

Acute immune inflammation.

Purulent inflammation.

Chronic immune inflammation.

Hemorragic inflammation.

Signs of acute immune inflammation are all the following processes, EXCEPT:

Options

Fast development.

Prevalence of alterative changes.

Slow development.

Prevalence of exudative changes.

Morphologically immediate type of hypersensitivity occurs as:

Options

Fibrinoid necrosis.

Lympho-hystiocytic infiltration.

Macrophageal infiltration.

Granulomatosis.

Morphologically immediate type of hypersensitivity occurs as:

Options

Lympho-hystiocytic infiltration.
Macrophageal infiltration.

Granulomatosis.

Plasmatic saturation.

Morphologically immediate type of hypersensitivity occurs as:

Options

Lympho-hystiocytic infiltration.

Mucoid changes.

Macrophageal infiltration.

Granulomatosis.

Morphologically immediate type of hypersensitivity occurs as:

Options

Lympho-hystiocytic infiltration.

Fibrinoid changes.

Macrophageal infiltration.

Granulomatosis.

Morphologically immediate type of hypersensitivity occurs as:

Options

Lympho-hystiocytic infiltration.

Fibrinous-hemorrhagic exudate.

Macrophageal infiltration.

Granulomatosis.

Morphological signs of immediate type of hypersensitivity are all the following, EXCEPT:

Options

Lympho-hystiocytic infiltration.

Mucoid and Fibrinoid changes.

Plasmatic saturation.

Fibrinoid necrosis.

Morphological signs of immediate type of hypersensitivity are all the following, EXCEPT:

Options

Mucoid and Fibrinoid changes.

Plasmatic saturation.
Granulomatosis.

Fibrinoid necrosis.

Morphological signs of immediate type of hypersensitivity are all the following, EXCEPT:

Options

Mucoid and Fibrinoid changes.

Plasmatic saturation.

Fibrinoid necrosis.

Macrophageal infiltration.

Which disease is associated with immediate type of hypersensitivity?

Options

Myocardial infarction.

Liver cirrhosis.

Аrthus reaction.

Purulent meningitis.

Which disease is associated with immediate type of hypersensitivity?

Options

Myocardial infarction.

Glomerulonephritis.

Liver cirrhosis.

Purulent meningitis.

Which disease is associated with immediate type of hypersensitivity?

Options

Syphilis.

Myocardial infarction.

Liver cirrhosis.

Purulent meningitis.

Which disease is associated with immediate type of hypersensitivity?

Options

Myocardial infarction.

Liver cirrhosis.

Purulent meningitis.
Rheumatic fever.

Which disease is associated with immediate type of hypersensitivity?

Options

Myocardial infarction.

Liver cirrhosis.

Purulent meningitis.

Systemic lupus erytematosus.

Which disease is associated with immediate type of hypersensitivity?

Options

Myocardial infarction.

Nodular periarteritis.

Purulent meningitis.

Liver cirrhosis.

Which disease is not associated with immediate type of hypersensitivity?

Options

Myocardial infarction.

Nodular periarteritis

Glomerulonephritis

Syphilis.

Which disease is not associated with immediate type of hypersensitivity?

Options

Liver cirrhosis.

Nodular periarteritis

Glomerulonephritis

Syphilis

Which disease is not associated with immediate type of hypersensitivity?

Options

Glomerulonephritis

Nodular periarteritis.

Arterial hypertension.

Syphilis
Reagin reactions are associated with action of which type of allergic reagin-antibodies?

Options

IgA

IgB

IgD

IgE

Reagin reactions are associated with formation of which type of exudate?

Options

Basophylic

Eosinophylic

Neutrophylic

Macrofageal

Which disease is associated with action of allergic reagin-antibodies?

Options

Chronic bronchitis.

Pulmonary carcinoma.

Atopic bronchial asthma.

Bronchiectasis

Morphologically delayed type of hypersensitivity occurs as:

Options

Mucoid and Fibrinoid changes

Plasmatic saturation.

Fibrinoid necrosis.

Cytolysis

Morphologically delayed type of hypersensitivity occurs as:

Options

Lympho-hystiocytic infiltration.

Mucoid and Fibrinoid changes.

Plasmatic saturation.

Fibrinoid necrosis.

Morphologically delayed type of hypersensitivity occurs as:


Options

Mucoid and Fibrinoid changes.

Plasmatic saturation.

Granulomatosis

Fibrinoid necrosis.

Morphologically delayed type of hypersensitivity occurs as:

Options

Mucoid and Fibrinoid changes.

Plasmatic saturation

Fibrinous-hemorrhagic exudate.

Macrophageal infiltration.

Morphologically delayed type of hypersensitivity occurs as:

Options

Mucoid and Fibrinoid changes

Plasmatic saturation.

Fibrinoid necrosis.

Cytoplasmic bridges between lymphocytes and macrophages.

Morphological signs of delayed type of hypersensitivity are all the following, EXCEPT:

Options

Fibrinoid necrosis.

Lympho-hystiocytic infiltration.

Macrophageal infiltration.

Granulomatosis

Morphological signs of delayed type of hypersensitivity are all the following, EXCEPT:

Options

Fibrinoid necrosis.

Lympho-hystiocytic infiltration.

Macrophageal infiltration.

Granulomatosis

Morphological signs of delayed type of hypersensitivity are all the following, EXCEPT:

Options
Lympho-hystiocytic infiltration.

Macrophageal infiltration.

Granulomatosis

Plasmatic saturation.

Morphological signs of delayed type of hypersensitivity are all the following, EXCEPT:

Options

Lympho-hystiocytic infiltration.

Mucoid changes.

Macrophageal infiltration.

Granulomatosis.

Morphological signs of delayed type of hypersensitivity are all the following, EXCEPT:

Options

Lympho-hystiocytic infiltration.

Fibrinous-hemorrhagic exudate.

Macrophageal infiltration.

Granulomatosis

Which disease is associated with delayed type of hypersensitivity?

Options

Myocardial infarction.

Liver cirrhosis.

Purulent meningitis.

Contact dermatitis.

Which disease is associated with delayed type of hypersensitivity?

Options

Viral hepatitis.

Liver cirrhosis.

Chronic cholecystitis.

Arterial hypertension.

Which disease is associated with delayed type of hypersensitivity?

Options

Typhoid fever.
Tuberculosis

Syphilis

Purulent meningitis.

Which disease is associated with delayed type of hypersensitivity?

Options

Typhoid fever.

Brucellosis

Syphilis

Purulent meningitis.

Which disease is not associated with delayed type of hypersensitivity?

Options

Contact dermatitis.

Brucellosis

Syphilis

Tuberculosis

Which disease is not associated with delayed type of hypersensitivity?

Options

Contact dermatitis.

Typhoid fever.

Brucellosis

Tuberculosis

Which disease is not associated with delayed type of hypersensitivity?

Options

Purulent meningitis.

Contact dermatitis.

Brucellosis

Tuberculosis

Which disease is not associated with delayed type of hypersensitivity?

Options

Viral hepatitis.

Contact dermatitis.
Brucellosis.

Sepsis

Morphological signs of transplant rejection are all the following, EXCEPT:

Options

Lympho-hystiocytic infiltration.

Edema of transplant.

Macrophageal infiltration.

Granulomatosis

Morphological signs of transplant rejection are all the following, EXCEPT:

Options

Lympho-hystiocytic infiltration.

Edema of transplant.

Macrophageal infiltration.

Cytoplasmic bridges between lymphocytes and macrophages.

Morphological signs of transplant rejection are all the following, EXCEPT:

Options

Lympho-hystiocytic infiltration.

Edema of transplant.

Macrophageal infiltration.

Eosinophylic infiltration.

Morphological signs of transplant rejection are all the following, EXCEPT:

Options

Lympho-hystiocytic infiltration.

Edema of transplant.

Macrophageal infiltration.

Coagulative necrosis.

Morphological signs of transplant rejection are all the following, EXCEPT:

Options

Lympho-hystiocytic infiltration.

Edema of transplant.

Macrophageal infiltration.
Hyperemia of transplant.

Predisposing factor in pathogenesis of autoimmunization is:

Options

Genes of HLA system.

Viral infections.

Bacterial infections.

Phisical and chemical influences to immune organs.

Predisposing factor in pathogenesis of autoimmunization is:

Options

Phisical and chemical influences to immune organs.

Viral infections.

Bacterial infections.

Hormonal disbalance.

Predisposing factor in pathogenesis of autoimmunization is:

Options

Phisical and chemical influences to immune organs.

Viral infections.

Bacterial infections.

Genetic peculiarities of target cells.

Initiated factor in pathogenesis of autoimmunization is:

Options

Genes of HLA system.

Viral infections.

Hormonal disbalance

Decrease of supressive activity of T-lymphocytes.

Initiated factor in pathogenesis of autoimmunization is:

Options

Genes of HLA system.

Bacterial infections.

Hormonal disbalance

Decrease of supressive activity of T-lymphocytes.


Initiated factor in pathogenesis of autoimmunization is:

Options

Genes of HLA system.

Hormonal disbalance.

Phisical influences to immune organs.

Decrease of supressive activity of T-lymphocytes.

Initiated factor in pathogenesis of autoimmunization is:

Options

Genes of HLA system.

Hormonal disbalance.

Chemical influences to immune organs.

Decrease of supressive activity of T-lymphocytes.

Basic factor in pathogenesis of autoimmunization is:

Options

Genes of HLA system.

Hormonal disbalance.

Decrease of supressive activity of T-lymphocytes.

Genetic peculiarities of target cells.

Organ specific immune disease is:

Options

Hashimoto thyroiditis.

Rheumatoid arthritis.

Scleroderma

Secondary thrombocytopenia.

Organ specific immune disease is:

Options

Rheumatoid arthritis.

Systemic Lupus Erythematosus.

Secondary thrombocytopenia.

Encephalomyelitis

Organ specific immune disease is:


Options

Rheumatoid arthritis.

Systemic Lupus Erythematosus.

Polyneuritis

Secondary thrombocytopenia.

Organ specific immune disease is:

Options

Rheumatoid arthritis.

Disseminated sclerosis of CNS.

Systemic Lupus Erythematosus.

Scleroderma

Organ specific immune disease is:

Options

Rheumatoid arthritis

Aspermatogenesis

Systemic Lupus Erythematosus

Scleroderma

Organ non-specific immune disease is:

Options

Hashimoto thyroiditis.

Rheumatoid arthritis.

Encephalomyelitis

Polyneuritis

Organ non-specific immune disease is:

Options

Hashimoto thyroiditis.

Encephalomyelitis

Systemic Lupus Erythematosus.

Polyneuritis

Organ non-specific immune disease is:

Options
Scleroderma

Encephalomyelitis

Hashimoto thyroiditis.

Polyneuritis

Organ non-specific immune disease is:

Options

Polyneuritis

Encephalomyelitis

Hashimoto thyroiditis.

Dermatomyositis

Organ non-specific immune disease is:

Options

Polyneuritis

Encephalomyelitis

Hashimoto thyroiditis.

Secondary thrombocytopenia.

Autoimmune disease of intermediate type is:

Options

Myastenia gravis.

Systemic Lupus Erythematosus.

Scleroderma

Polyneuritis

Autoimmune disease of intermediate type is:

Options

Hashimoto thyroiditis.

Diabetes mellitus-type I.

Scleroderma

Polyneuritis

Autoimmune disease of intermediate type is:

Options

Hashimoto thyroiditis.
Scleroderma

Thyrotoxicosis

Polyneuritis

Autoimmune disease of intermediate type is:

Options

Hashimoto thyroiditis.

Scleroderma

Polyneuritis

Sjogren’s syndrome.

Autoimmune disease of intermediate type is:

Options

Hashimoto thyroiditis.

Scleroderma

Polyneuritis

Goodpasture’s syndrome.

Variant of primary immunodeficiency syndrome occurs:

Options

In leukemia.

Under radial therapy.

In aplasia of thymus.

In sarcoidosis.

Variant of secondary immunodeficiency syndrome occurs in all the following diseases, EXCEPT:

Options

Leukemia

Thymoma

Sarcoidosis

Hypoplasia of thymus.

Variant of secondary immunodeficiency syndrome occurs in all the following diseases, EXCEPT:

Options

Leukemia

Thymoma
Sarcoidosis

Aplasia of thymus.

Variant of secondary immunodeficiency syndrome occurs in all the following diseases, EXCEPT:

Options

Fibroma

Leukemia

Thymoma

Sarcoidosis

Variant of secondary immunodeficiency syndrome occurs in all the following diseases, EXCEPT:

Options

Hemangioma

Leukemia

Thymoma

Sarcoidosis

Variant of secondary immunodeficiency syndrome is:

Options

Lui-Bar syndrome.

Neseloff syndrome.

DiGeorge syndrome.

AIDS

Variant of secondary immunodeficiency syndrome occurs in all the following situations, EXCEPT:

Options

Radial therapy.

Corticosteroid therapy.

Immunodepressant therapy.

Antibiotic therapy.

Variant of secondary immunodeficiency syndrome occurs in all the following situations, EXCEPT:

Options

Radial therapy.

Corticosteroid therapy.

Immunodepressant therapy.
Physiotherapy

Complication of immunodeficiency syndromes is:

Options

Arterial hypertension.

Myocardial infarction.

Typhoid fever.

Sepsis

Complication of immunodeficiency syndromes is:

Options

Recurrence of bronchial asthma.

Recurrence of chronic hepatitis.

Recurrence of chronic cholecystitis.

Recurrence of tuberculosis.

Complication of immunodeficiency syndromes is:

Options

Arterial hypertension.

Myocardial infarction.

Typhoid fever.

Purulent pneumonia.

Complication of immunodeficiency syndromes is:

Options

Pulmonary abscess

Myocardial infarction.

Typhoid fever.

Chronic peptic gastric ulcer.

Complications of immunodeficiency syndromes are all the following, EXCEPT:

Options

Pulmonary abscess.

Purulent pneumonia.

Myocardial infarction.

Sepsis.
Complications of immunodeficiency syndromes are all the following, EXCEPT:

Options

Pulmonary abscess.

Purulent pneumonia.

Liver cirrhosis.

Sepsis

Complications of immunodeficiency syndromes are all the following, EXCEPT:

Options

Pulmonary abscess.

Purulent pneumonia.

Sepsis

Chronic peptic gastric ulcer.

Complications of immunodeficiency syndromes are all the following, EXCEPT:

Options

Pulmonary abscess.

Purulent pneumonia.

Sepsis

Arterial hypertension.

What does the proliferation of neoplastic cells lead to?

Options

Neoplasia

Tumors

Atrophy

Freckles

What is the term that means "new growth?

Options

Anaplasia

Metaplasia

Neoplasia

Hyperplasia

What kind of tumors have a limited growth potential and a good outcome?
Options

Malignant

Hypertrophic

Hypotrophic

Benign

Who determines the definitive diagnosis of tumors?

Options

Oncologist

Physician

Your mom

Pathologist

What kind of tumors resemble the tissue from which they have arisen?

Options

Hypertrophic

Malignant

. Benign

Tumor-like

What does pleomorphism mean?

Options

Uncontrolled mitosis

Multiple nuclei

Variability in shape and size

Nucleus – cytoplasm ration

The cells are different from where they arose from What is a normal N/C ratio?

Options

1:3

1:8

3:6

1:5

IN a malignant tumor the N/C ratio most commonly exhibited is:

Options
1:2

1:1

3:6

1:8

What is the process called by which cells move from one site to another?

Options

Transportation

Biotransformation

Metastasis

Metrostatic

A leiyoma of the uterus is considered benign or malignant?

Options

Benign

Malignant

Neither

Tumor-like

Which of the following is NOT a pathway in which malignant cells spread (metastasize)?

Options

. Lymph

Saliva

Blood

Metrostatic

What is an example of lymphatic metastasis?

Options

Renal cell carcinoma

Adrenal adenoma

Breast cancer

. Leiyoma

What is an example of metastasis occurring as a direct extension of the primary tumor?

Options

Breast cancer developing over a course of 3 months


Leukemia developing at a very young age

Renal cell carcinoma spreading to the adrenal gland

HIV developing into AIDs

Is a metastatic malignant melanoma of the vertebra considered to be malignant or benign?

Options

Malignant

Benign

Choristoma-like

Tumor-like

What is a metastatic adenocarcinoma of the stomach that specifically goes to the ovary called? (be specific).

Options

Metastatic adenocarcinoma

Melanoma

Krukenberg tumor

Wilson's tumor

Is a lipoma considered to be malignant or benign?

Options

Malignant

Benign

Tumor-like

Hamartoma

What would you be worried about if a 45-year-old woman comes into your ED with massive weight gain (fluid)

over a short period of time?

Options

Metastatic ovarian carcinoma

Metastatic vaginal carcinoma

Cervical cancer

Cholecistitis

Is a melanoma considered to be malignant or benign?

Options

Malignant
Benign

Tumor-like

Hamartoma

If a tumor is benign and of squamous origin, what would it be called?

Options

Malignant

Adenoma

Papilloma

Carcinoma

Is a lymphoma considered to be malignant or benign?

Options

Malignant

Benign

Tumor-like

Hamartoma

If a tumor is benign and glandular in origin, what is it called?

Options

Malignant

Adenoma

Carcinoma

Papilloma

Is a rhabdomyosarcoma considered to be malignant or benign?

Options

Malignant

Benign

Tumor-like

Hamartoma

What is it called when the nucleus are pushed off to one side due to abundant mucin?

Options

Hypertrophy

Benign
Krukenburgs sign

Signet-rings

Vascular changes associated with acute inflammation include ____ (from histamine and NO) and ____ vascular
permeability.

Options

Vasoconstriction; Decreased

Vasoconstriction; Increased

Vasodilation; Decreased

Vasodilation; Increased
Mediators such as histamine, thrombin, and platelet activating factor (PAF) stimulate the redistribution of which
ofthe following from its normal intracellular stores in granules (Weibel-Palade bodies) to the cell surface?

Options

P-selectin

E-selectin

ICAM-1

VCAM-1

Cytokine activation of endothelium (with neutrophils) involves which of the following?

Options

IL-1 and IL-2

IL-3

IL-4 and IL-5

IL-1 and TNF

Which of the following is NOT a general principle of the chemical mediators of inflammation?

Options

Mediators originate either from plasma or from cells

The production of active mediators is triggered by microbial products or by host proteins

Most mediators perform their biologic activity by initially binding to specific receptors on target cells

Once activated and released from the cell, most of these mediators last a long time (long-lived)

Nitric oxide (NO) is synthesized from what amino acid by the enzyme nitric oxide synthase (NOS)?

Options

Alanine

Arginine

Asparagine

Lysine

NO causes vasodilation by what mechanism?

Options

Constriction of smooth muscle

Relaxation of smooth muscle

Constriction of striated muscle

Relaxation of striated muscle


Which of the following is NOT true regarding NO?

Options

NO acts on target cells through induction of GMP

NO reduces platelet adhesion

NO reduces leukocyte adhesion

NO promotes leukocyte rolling

Which of the following is NOT true regarding contribution to inflammation?

Options

Lysosomal constituents increase vascular permeability and tissue damage

Oxygen free radicals amplify the cascade that elicits the inflammatory response

Neuropeptides help initiate and propagate the inflammatory response

The response to hypoxia decreases vascular permeability

One possible outcome of acute inflammation is resolution, with the other outcomes being chronic inflammation and
fibrosis (loss of function). Which of the following is NOT associated with resolution?

Options

Clearance of injurious stimuli

Clearance of mediators and acute inflammatory cells

Replacement of injured cells

Angiogenesis

Which of the following appears, histologically, as an eosinophilic meshwork of threads or sometimes as an


amorphous coagulum?

Options

Serous inflammation

Fibrinous inflammation

Suppurative or purulent inflammation

Ulcers

Which of the following is characterized by the production of large amounts of pus consisting of neutrophils, necrotic
cells, and edema fluid?

Options

Serous inflammation

Fibrinous inflammation

Suppurative or purulent inflammation


Ulcers

Which of the following is marked by the outpouring of a thin fluid that, depending on the size of injury, is derived
from either the plasma or the secretions of mesothelial cells lining the peritoneal, pleural, and pericardial cavities?

Options

Serous inflammation

Fibrinous inflammation

Suppurative or purulent inflammation

Ulcers

Unlike acute inflammation, chronic inflammation is characterized by tissue destruction, induced by the persistent
offending agent or by the inflammatory cells. It is also characterized by the cells involved, which include all of the
following EXCEPT:

Options

Lymphocytes

Plasma cells

Macrophages

Neutrophils

A granuloma is a focus of chronic inflammation consisting of a microscopic aggregation of macrophages. Which of


the following is an immune granuloma and NOT a foreign body granuloma?

Options

Talc

Sutures

Microbes

Soil

A patient presents with an infection of the hand and red streaks along the arm and into the axilla, with painful
nodules in the axilla. This is due to a secondary inflammation of the ____, which ____ in flow due to the infection.

Options

Blood; Increased

Blood; Decreased

Lymph; Increased

Lymph; Decreased

Which of the following is NOT a systemic effect of inflammation?

Options

Fever
Increased acute-phase proteins

Leukocytosis

Decreased pulse and blood pressure

Which of the following would be associated with defective inflammation?

Options

Infections

Allergies

Asthma

Psoriasis

Which of the following is NOT capable of regeneration?

Options

Epithelial tissue

Cardiac tissue

Skin

Liver

Which of the following is the correct order of the cell cycle?

Options

G1 => S => G2 => M

G2 => S => G1 => M

G1 => M => G2 => S

G2 => M => G1 => S

Which of the following is NOT a cell cycle checkpoint?

Options

Restriction checkpoint

G0 checkpoint

G2 checkpoint

Anaphase checkpoint

Which part of the cell cycle has the most redundancies, is tightly regulated by proteins called cyclins, and associated
enzymes called cyclin-dependent kinases (CDKs)?

Options

Between G0 and G1
Between G1 and S

Between S and G2

Between G2 and M

Which of the following cell types is pluripotent?

Options

Quiescent

Labile

Permanent

Embryonic stem cell

Which of the following growth factors comes from mesenchymal cells and functions to replicate hepatocytes?

Options

Epidermal Growth Factor (EGF)

Transforming Growth Factor-α (TGF-α)

Hepatocyte Growth Factor (HGF)

Vascular Endothelial Growth Factor (VEGF)

Which of the following growth factors comes from platelets, macrophages, endothelial cells, and smooth muscle and
functions as a chemotactic for PMNs, macrophages, fibronblast, and smooth muscle? It also functions to stimulate
production of MMPs, stimulate angiogenesis, and wound contraction.

Options

Epidermal Growth Factor (EGF)

Hepatocyte Growth Factor (HGF)

Vascular Endothelial Growth Factor (VEGF)

Platelet-Derived Growth Factor (PDGF)

Which of the following growth factors comes from macrophages, mast cells, and T cells and functions in
angiogenesis, wound repair, and hematopoiesis?

Options

Epidermal Growth Factor (EGF)

Transforming Growth Factor-α (TGF-α)

Vascular Endothelial Growth Factor (VEGF)

Fibroblast Growth Factor (FGF)

Which of the following growth factors comes from platelets, macrophages, saliva,urine, milk, and plasma and
functions to stimulate keratinocyte migration and granule tissue formation?

Options
Epidermal Growth Factor (EGF)

Transforming Growth Factor-α (TGF-α)

Hepatocyte Growth Factor (HGF)

Vascular Endothelial Growth Factor (VEGF)

Which of the following growth factors comes from mesenchymal cells and increases vascular permeability and
vasculogenesis?

Options

Epidermal Growth Factor (EGF)

Transforming Growth Factor-α (TGF-α)

Hepatocyte Growth Factor (HGF)

Vascular Endothelial Growth Factor (VEGF)

Which of the following growth factors comes from mesenchymal cells and enhances proliferation of epithelial and
endothelial cells?

Options

Epidermal Growth Factor (EGF)

Transforming Growth Factor-α (TGF-α)

Hepatocyte Growth Factor (HGF)

Vascular Endothelial Growth Factor (VEGF)

Which of the following growth factors comes from macrophages, mast cells, and lymphocytes and functions in
chemotaxis and regulation of cytokines?

Options

Tumor Nectosis Factor (TNF)

Transforming Growth Factor-β (TGF-β)

Keratinocyte Growth Factor (KGF)

Interleukin-1 (IL-1)

Which of the following growth factors comes from fibroblasts and stimulates keratinocyte migration, proliferation,
and differentiation?

Options

Tumor Nectosis Factor (TNF)

Transforming Growth Factor-β (TGF-β)

Keratinocyte Growth Factor (KGF)

Insulin-like Growth Factor (IGF-1)


Which of the following growth factors comes from macrophages and T cells and activates macrophages as well as
regulates cytokines?

Options

Tumor Nectosis Factor (TNF)

Transforming Growth Factor-β (TGF-β)

Keratinocyte Growth Factor (KGF)

Insulin-like Growth Factor (IGF-1)

Which of the following growth factors comes from platelets, T cells, macrophages, fibroblast, and smooth muscle
cells and is a growth inhibitor for most epithelial cells?

Options

Tumor Nectosis Factor (TNF)

Transforming Growth Factor-β (TGF-β)

Keratinocyte Growth Factor (KGF)

Insulin-like Growth Factor (IGF-1)

Which of the following growth factors comes from lymphocytes and fibroblasts, activates macrophages, and inhibits
fibroblast proliferation?

Options

Tumor Nectosis Factor (TNF)

Transforming Growth Factor-β (TGF-β)

Keratinocyte Growth Factor (KGF)

Interferons

Which of the following growth factors comes from macrophages and fibroblasts and stimulates synthesis of
proteoglycans?

Options

Tumor Nectosis Factor (TNF)

Transforming Growth Factor-β (TGF-β)

Keratinocyte Growth Factor (KGF)

Insulin-like Growth Factor (IGF-1)

Which of the following is true regarding target cell signal transduction by growth factors?

Options

It stimulates the transcription of genes that were silent in the resting cells

It blocks the transcription of genes that were active in the resting cells

The genes regulate the entry of the cells into the cell cycle
A&C

Which of the following signaling modes uses blood vessels and is associated with several cytokines?

Options

Autocrine signaling

Paracrine signaling

Endocrine signaling

None of the above

Which of the following describes the JAK/STAT signal transduction pathway?

Options

Receptors with intrinsic tyrosine kinase activity

Receptors lacking intrinsic tyrosine kinase activity that recruit kinases

Seven transmembrane G-protein-coupled receptors (GPCRs)

Steroid hormone receptors

Which of the following signal transduction pathways includes cytokines such as interleukin-2 (IL-2)?

Options

Receptors with intrinsic tyrosine kinase activity

Receptors lacking intrinsic tyrosine kinase activity that recruit kinases

Seven transmembrane G-protein-coupled receptors (GPCRs)

Steroid hormone receptors

Which of the following is associated with the cAMP signal transduction pathway?

Options

Receptors with intrinsic tyrosine kinase activity

Receptors lacking intrinsic tyrosine kinase activity that recruit kinases

Seven transmembrane G-protein-coupled receptors (GPCRs)

Steroid hormone receptors

Vitamin D utilizes which of the following signal transduction pathways?

Options

Receptors with intrinsic tyrosine kinase activity

Receptors lacking intrinsic tyrosine kinase activity that recruit kinases

Seven transmembrane G-protein-coupled receptors (GPCRs)

Steroid hormone receptors


Which of the following is associated with the IP3 signal transduction pathway?

Options

Receptors with intrinsic tyrosine kinase activity

Receptors with intrinsic tyrosine kinase activity

Seven transmembrane G-protein-coupled receptors (GPCRs)

Steroid hormone receptors

Which of the following is NOT true regarding regeneration of mammalian tissue?

Options

Quiescent cells such as cardiac myotubes reenter the cell cycle

Stem cells in the area of injury differentiate efficiently

There is a rapid fibroproliferative response and scar formation after wounding

Regeneration is complete (not compensatory growth)

Which of the following extracellular matrix (ECM) fibrous structural proteins is the most common protein in the
animal world and is composed of a triple helix of three polypeptide chains?

Options

Collagen

Elastin

Fibrillin

Elastic fibers

The ECM contains cell adhesion molecules (CAMs). Which of the following families of CAMs are generally involved in
calcium-dependent homotypic interactions?

Options

Immunoglobulin

Immunoglobulin

Integrins

Selectins

Which of the following families of CAMs participate in both homotypic and heterotypic cell-to-cell interactions due
to the types of ligands they bind?

Options

Immunoglobulin

Cadherins

Integrins
Selectins

Which of the following families of CAMs have broader ligand specificity and are responsible for many events
involving cell adhesion?

Options

Immunoglobulin

Cadherins

Integrins

Selectins

Which of the following families of CAMs function in adhesion of leukocytes to endothelial cells?

Options

Immunoglobulin

Cadherins

Integrins

Selectins

Hyaluronic acid (HA) is a component of the ECM and is bound by what surface glycoprotein on leukocytes, allowing T
cells to remain bound to endothelium at sites of inflammation?

Options

CD3

CD4

CD34

CD44

Proteoglycans, such as heparan sulfate, chondroitin sulfate, and dermatan sulfate, are a component of the ECM and
made from glycosaminoglycans (GAGs). Which of the following is NOT a function of proteoglycans?

Options

They can be integral member proteins

They bind with fibroblast growth factor

They bind leukocyte surface markers

They modulate cell growth

Which of the following growth factors appears to be the most important in scar formation during the fibroblast
migration and proliferation stage?

Options

EFG

PDGF
FGF

TGF-α

Which of the following is the most true regarding scar formation (net collagen accumulation)?

Options

Depends on increased collagen synthesis only

Depends on decreased degradation only

Depends on decreased degradation only

Depends on both increased collagen synthesis and decreased degradation

What is the function of matrix metalloproteinases (MMPs) in scar formation and tissue remodeling?

Options

Remodels collagen

Degrades collagen

Degrades other non-collagen ECM proteins

Both B & C

Which of the following is true when comparing healing by second intention (separated edges) to healing by first
intention (opposed edges)?

Options

Significant granulation and wound contraction are seen in first intention

A scab forms within 24 hours in first intention

A fibrous union is seen in first intention, weeks after injur

In first intention, mitosis near the wound is seen in 3-7 days

Which of the following contributes to wound tensile strength toward the end of healing?

Options

Increased collagen synthesis

Decreased collagen degradation

Structural collagen fiber modification

Decreased collagen cross-linking

Which of the following drugs would NOT retard wound healing?

Options

Steroids

Cytotoxic medications
Vitamins (A, C)

Intensive antibiotic therapy

Which of the following would NOT retard wound healing?

Options

Obesity

Malnutrition

Protective dressings

Keloidosis

Which of the following wound healing complications can lead to wound dehiscence and ulceration?

Options

Deficient scar formation

Excessive formation of the repair components

Formation of contractures

Allof the above

Which of the following differentiates fibrosis from normal wound healing?

Options

Cutaneous scar-cell proliferation

Cell-cell interactions

Cell-matrix interactions

Immune and autoimmune reactions

What is the term for extravasation of water into the interstitial space?

Options

Hyperemia

Hemorrhage

Edema

Embolism

Anasarca is a ____ and ____ edema with subcutaneous tissue swelling.

Options

Mild; Localized

Mild; Generalized

Severe; Localized
Severe; Generalized

Which of the following would NOT cause edema?

Options

Decreased hydrostatic pressure

Reduced plasma colloid osmotic pressure (hypoproteinemia)

Lymphatic obstruction

Sodium retention

Finger pressure over substantially edematous subcutaneous tissue displaces the interstitial fluid and leaves a
fingershaped depression. This is called:

Options

Lower limb edema

Sacral edema

Periorbital edema

Pitting edema

Which of the following types of edema is more commonly known as ascites?

Options

Hydrothorax

Hydrocephalus

Hydrosalpinx

Hydroperitoneum

In heart failure, which of the following would NOT increase, leading to edema?

Options

Renin and angiotensin II

Aldosterone

Renal Na+ reabsorption and retention

Cardiac output

In congestive heart failure (CHF) of the left ventricle, edema will develop in the ____ system. In CHF of the right
ventricle, edema will develop in the ____ system.

Options

Circulatory; Pulmonary

Pulmonary; Circulatory

Circulatory; Hepatic
Hepatic; Circulatory

A deep vein thrombosis (DVT) causes edema by what mechanism?

Options

Inflammation

Lymphatic obstruction

Reduced plasma colloid osmotic pressure

Increased hydrostatic pressure

During surgery, an anesthesiologist notices the patient’s blood pressure has dropped. Which of the following could
be given to increase plasma colloid osmotic pressure?

Options

Globulins

Vitamin K

Albumins

Steroids

The parasite filariasis (mosquito vectored) induces lower limb and external genitalia elephantiasis by which of the
following mechanisms?

Options

Inflammation

Sodium retention

Reduced plasma colloid osmotic pressure

Lymphatic obstruction

In hyperemia, which is caused by arterial dilation such as during exercise or inflammation, tissues are ____ because
of increased ____ blood. In congestion, such as during CHF or venous obstruction, tissues are ____ because of
increased ____ blood.

Options

Red; Oxygenated; Blue; Deoxygenated

Red; Deoxygenated; Blue; Oxygenated

Blue; Oxygenated; Red; Deoxygenated

Blue; Deoxygenated; Red; Oxygenated

Which of the following is most likely associated with skin purpura (>3mm hemorrhages) as opposed to petechiae (1-
2mm hemorrhages)?

Options

Thrombocytopenia
Amyloidosis

Defective platelet function

Increased intravascular

Which clotting cascade factor converts prothrombin to thrombin and is the beginning of the common pathway (from
intrinsic and extrinsic)?

Options

II

XII

Which of the following factors initiates the intrinsic coagulation pathway?

Options

Fibrin

Thrombin

Tissue Factor

Hageman Factor

Which of the following factors initiates the extrinsic coagulation pathway?

Options

Fibrin

Thrombin

Tissue Factor

Fibrinogen

Which of the following factors is the end result of the coagulation common pathway, leading to cross-linked fibrin?

Options

Ia

Ia

IIa

Vitamin K converts glutamyl residues in precursors to gamma carboxygltuamates and is essential to prevent
hemorrhages. Which of the following factors is NOT dependent on vitamin K?

Options

XII
X

VII

Which of the following is NOT part of the Virchow thrombosis triad?

Options

Abnormal blood flow

Hypercoagulability

Reduction of clotting factors

Endothelial injury

Mutations to which of the following clotting factors cause hypercoagulable states most commonly?

Options

I and II

II and XII

XII and V

V and II

Of the secondary (acquired) causes of thrombosis, which of the following is NOT considered high risk?

Options

Prolonged bed rest

Atrial fibrillation

Prosthetic cardiac valves

Smoking

Lines of Zahn can be most commonly seen (microscopically) in thrombi of which organ?

Options

Liver

Kidney

Brain

Heart

Which of the following post-thrombus processes involves the thrombi inducing inflammation and fibrosis?

Options

Propagation

Embolization
Dissolution

Organization and recanalization

Which of the following is commonly associated with arterial thrombosis and not venous thrombosis?

Options

Inactivity

Atherosclerosis

Cardiac failure stasis

Prothrombin gene mutation

Disseminated intravascular coagulation (DIC) is common in patient with gramnegative sepsis and acute
promyelocytic leukemia. DIC has a high mortality rate and is a potential complication of any disease with widespread
activation of which of the following?

Options

Fibrin

Thrombin

Tissue Factor

Fibrinogen

Pulmonary emboli are not commonly fatal and arise from DVT in > 95% of cases. What percentage of pulmonary
circulation must be obstructed before right-sided heart failure (sudden death) occurs?

Options

> 95%

> 90%

> 80%

> 60%

Systemic (arterial) emboli commonly arise from ____ and most commonly lodge in the lower limbs and sometimes
the brain.

Options

Cardiac mural emboli

Cardiac valvular emboli

Deep vein emboli

Renal emboli

Amniotic fluid emboli arise from the infusion of amniotic fluid or fetal tissue into the maternal circulation via a tear
in the placental membranes or rupture of uterine veins. They are associated with a mortality rate of:

Options
0 – 20%

20 – 40%

40 – 60%

60 – 80%

How much air is required (in general) to have a clinical effect as an air emboli?

Options

0.1 cc

1 cc

10 cc

100 cc

Caisson disease, a chronic form of decompression sickness, commonly affects all of the following EXCEPT:

Options

Femur (heads)

Tibia

Humerus

Scapula

SCUBA divers who surface too quickly can get decompression sickness (DCS). According to Henry’s Law, as the diver
surfaces, the partial pressure of gas in their blood ____ and thus the amount of gas in their blood must decrease and
____ the vasculature.

Options

Decrease; Enter

Decrease; Exit

Increase; Enter

Increase; Exit

Which of the following is NOT a clinical finding in fat embolisms, which have a 10% fatality?

Options

Pulmonary insufficiency

Neurologic symptoms

Thrombocytopenia and anemia

Bradycardia

Which of the following is the most likely to cause a fat emboli?

Options
Setting a shoulder dislocation

Healing of a chemical burn

Healing of a heat burn

A broken femur

Which of the following is most likely associated with anemic (white) infarction and not hemorrhagic (red)

infarction?

Options

Venous occlusions (ovarian torsion)

Loose tissues (lungs)

Tissues with dual circulations (small intestine, lungs)

Solid organs with end-arterial circulation (heart, spleen, kidney)

Patients who are in septic shock will have systemic _____ and thus a(n) _____ in peripheral vascular resistance.
These patients will also have decreased myocardial contractility, although their cardiac index (CI) may be normal
(compensatory).

Options

Vasodilation; Decrease

Vasodilation; Increase

Vasoconstriction; Decrease

Vasocontriction; Increase

Which of the following organs is least likely to be affected by shock, with necrosis being a common complication?

Options

Spleen

Brain

Lungs

Kidneys

Which of the following is a sign of septic shock and NOT a sign of hypovolemic or cardiogenic shock?

Options

Hypotension

Weak, thready pulse

Tachycardia

Vasodilation
Cardiogenic shock associated with extensive myocardial infarction and gramnegative shock carry mortality rates of
up to:

Options

0.05

0.25

0.5

0.75

All are true about Fanconi anemia, except:


Defect in DNA repair
Bone marrow hyper function
Congenital anomaly present
Increased chances of cancer

HER2/neu receptor plays a role in


Predicting therapeutic response
Diagnosis of breast cancer
Screening of breast cancer
Recurrence of tumor

The most common secondary malignancy in a patient having retinoblastoma is:


Osteosarcoma
Renal cell carcinoma
Pineoblastoma
Osteoblastoma

Regarding Fanconi anemia, the wrong statement is:


Autosomal dominant
Bone marrow show pancytopenia
Usually aplastic anemia
It is due to defective DNA repair

True statements about p53 gene are all except:


Arrests cell cycle at G1 phase
Product is 53 kD protein
Located on chromosome 17
Wild/non-mutated form is associated with increased risk of childhood tumors

Growth factor oncogene is:


Myc
Fos
Sis
Jun

Rosettes are characteristically seen in


Retinoblastoma
Melanoma
Dysgerminoma
Lymphoma

The normal cellular counterparts of oncogenes are important for the following functions, except:
Promotion of cell cycle progression
Inhibition of apoptosis
Promotion of DNA repair
Promotion of nuclear transcription

An example of a tumor suppressor gene is:


Myc
Fos
ras
Rb

Lynch syndrome is associated with cancers of the:


Breast, colon, ovary
Breast, endometrium, ovary
Breast, colon, endometrium
Colon, endometrium, ovary

Loss of heterozygosity associated with:


Acute myeloid leukemia
ALL
Retinoblastoma
Promyelocytic leukemia
Which is not a tumor suppressor gene?
WT-1
Rb
P53
Ras

The inheritance pattern of familial Retinoblastoma is:


Autosomal recessive
Autosomal dominant
X-linked dominant
X-linked recessive

Which of the following is known as the “guardian of the genome”?


P53
Mdm2
P14
ATM

The following statements are true about Tumor Suppressor Gene p53, except:
It regulates certain genes involved in cell cycle regulation
Its increased levels can induce apoptosis
Its activity in the cells decreases following UV irradiation and stimulates cell cycle
Mutations of the p53 gene are most common genetic alteration seen in human cancer

In the mitogen activated protein kinase pathway, the activation of RAS is counteracted by:
Protein kinase C
GTPase activating protein
Phosphatidyl inositol
Inositol triphosphate

Which of the following mutations in a tumor suppressor agent causes breast carcinoma?
p43
P53
p73
P83

True about proto-oncogenes is:


Important for normal cell growth
May get converted into oncogenes
C-myc over-expression causes lymphoma
Their mutation causes retinoblastoma
True about oncogene is:
Present in normal cell
They are of viral origin
They are transduced from virus infected cells
P53 is most common oncogene mutation causing malignancy

Cancer cell survival is enhanced by:


Suppression of p53 protein
Over expression of p53 gene
bcl-2
Bax

Following are required for normal growth


Proto-oncogenes
Tumor suppressor genes
Oncogenes
DNA repair genes

Xeroderma pigmentosum is caused due to a group of closely related abnormalities in:


Mismatch repair
Base excision repair
Nucleotide excision repair
SOS repair

Increased expression of which of the following causes oncogenesis


IGF receptor
EGF receptor
GH receptor
Aldosterone receptor

Tumor suppressor genes are all, except


APC
p53
Rb
C-myc

Angiogenesis is
Formation of the new blood vessels
Repair by connective tissues
Formation of the blood clot
All of the above

Medullary carcinoma of thyroid is associated with mutation in:


RET
RAS
NF
Rb

APC gene is located on which chromosome


Chromosome 5
Chromosome 6
Chromosome 9
Chromosome 11

Proto-oncogene erb-B is not related to:


Breast carcinoma
Small cell lung carcinoma
Non-small cell lung carcinoma
Ovarian carcinoma

Most common genetic mutation in carcinogenesis involves:


p53
Rb
HPC
PTEN

Which of the following is a true statement regarding the above gene?


It is associated with the
It is called as
It is active in hypopohosphorylated form
It is active in hyperphosphorylated form

Aisha, a 51 year old woman discovers a lump in her left breast on a weekly self-examination. Mammography is
performed which confirms the presence of a suspicious “mass”, and needle core biopsy is performed to determine
whether the mass is malignant. Dr. Devesh, the pathologist confirmed the mass to be malignant and said that the
tissue demonstrates amplification of her-2/ neu oncogene. What kind of protein is the gene product of Her-2/neu

GTPase
GTPase-activating protein
Receptor tyrosine kinase
Retinoic acid receptor protein
A patient Madhu undergoes total thyroidectomy for a mass lesion of the thyroid. During the surgery it is found
thatthe parathyroid glands appeared enlarged. The thyroid lesion shows neuroendocrine-type cells and
amyloiddeposition. This patient’s thyroid and parathyroid lesions may be related to which of the following
oncogenes?
Bcl-2
C-myc
Ret
L-myc

Dr. Marwah, a pediatrician, performing an ophthalmoscopic examination on a four-year-old boy, notices


severalsmall pigmented nodules in his irises. He also notices six light brown macules on the trunk of the child of
variablesizes. This boy may have a propensity to develop tumors in which of the following structures?
Bladder
Colon
Peripheral nerve
Skin

RET gene mutation is associated with which malignancy?


Pheochromocytoma
Medullary carcinoma thyroid
Lymphoma
Renal cell carcinoma

Endometrial carcinoma is associated with which of the following tumor suppression gene mutation?
P53
Rb
PTEN
APC

The tumor suppressor gene p 53 induces cell cycle arrest at:


G2
S
G1 – S phase
G0 phase

MYC gene is:


Protein kinase inhibitor
Growth factor inhibitor
GTPase
Transcription activator
Retinoblastoma is associated with which of the following tumours?
Osteoclastoma
Hepatocellular cancer
Squamous cell cancer
Osteosarcoma

An example of a tumour suppressor gene is:


Myc
Fos
Ras
RB

Which of the following is DNA repair defect?


Retinoblastoma
Neurofibromatosis
Xeroderma pigmentosum
MEN-I

Which of the following gene defect is associated with development of medullary carcinoma of thyroid:
RET Proto Oncogene
Fap gene
Rb gene
BRCA 1 gene

All of the following are tumor markers, except:


Beta-2 macroglobulin
HCG
Alpha-fetoprotein
CEA

Knudson two hit hypothesis is seen with


Melanoma
Retinoblastoma
Ulcerative colitis
Crohn disease
Retinoblastomas arising in the context of germ-line mutations not only may be bilateral, but also may be
associatedwith (so called “trilateral” retinoblastoma)
Medulloblastoma
Pinealoblastoma
Neuroblastoma
Hemangioblastoma

Post transplant lymphoma is caused by which of the following?


CMV
EBV
Herpes simplex
HHV-6

H. pylori infection is associated with development of which malignancy:


MALTomas
Atherosclerosis
Sarcoma
Gastrointestinal stromal tumor (GIST)

Helicobacter pylori infection is associated with all of the following conditions, except:
Peptic ulcer disease
Gastric adenocarcinoma
B cell lymphoma
Burkitt’s lymphoma

Tumors associated with organisms are all except:


Hepatocellular cancer
Non-small Cell Carcinoma of Lung
Gastric cancer
Nasopharyngeal cancer

Which of the following is essential for tumor metastasis?

Angiogenesis
Tumorogenesis
Apoptosis
Inhibition of tyrosine kinase activity

Which of the following statements about carcinogenesis is false?


Asbestos exposure increases the incidence of lung cancer
Papilloma viruses produce tumors in animals but not in humans
Exposure to aniline dyes predisposes to cancer of the urinary bladder
Hepatitis B virus has been implicated in hepatocellular carcinoma

Smoking is a risk factor for all carcinomas , except;


Oral
Bronchial
Bladder
Thyroid
Workers exposed to polyvinyl chloride may develop following liver malignancy
Cholangiocarcinoma
Fibrolamellar carcinoma
Angiosarcoma
All of the above

Which among the following is not a neoplastic virus:


Cytomegalovirus
Hepatitis B virus
Human papilloma virus
All of these

A 37-year-old man, Gagan presents with increasing abdominal pain and jaundice. He gives a history of intake
ofgroundnuts which did not taste appropriate. Physical examination reveals a large mass involving the right side of
hisliver, and a biopsy specimen from this mass confirms the diagnosis of liver cancer (hepatocellular
carcinoma).Which of the following substances is most closely associated with the pathogenesis of this tumor?
Aflaxotin B1
Direct-acting alkylating agents
Vinyl chloride
Azo dyes

Biopsy of an ulcerated gastric lesion of a 26-year-old smoker Akki demonstrates glands containing cells withenlarged,
hyperchromatic nuclei below the muscularis mucosa. Two tripolar mitotic figures are noted. With which ofthe
following infectious agents has this type of lesion been most strongly associated?
Epstein-Barr virus
Helicobacter pylori
Human papilloma virus
Molluscum contagiosum virus

A man Alok Nath contracts HTLV-1 infection through sexual contact. Twenty-one years later he developsgeneralized
lymphadenopathy with hepatosplenomegaly, a skin rash, hypercalcemia, and an elevated white bloodcount. This
man has most likely developed which of the following?
AIDS
Autoimmunity
Delayed hypersensitivity reaction
Leukemia

Thorium induced tumor is which of the following?


Renal cell carcinoma
Lymphoma
Angiosarcoma of liver
Astrocytoma

Radiation exposure during infancy has been linked to which one of the following carcinoma?
Breast
Melanoma
Thyroid
Lung
The following parasitic infections predispose to malignancies?
Paragonimus westermani
Guinea worm infection
Clonorchiasis
Schistosomiasis

Kaposi’s sarcoma is seen with


HCV
HPV
HSV
HHV

UV radiation has which of the following effects on the cells?


Prevents formation of pyrimidine dimers
Stimulates formation of pyrimidine dimers
Prevents formation of purine dimers
All of the above

The most radiosensitive cells are:


Neutrophils
Lymphocytes
Erythrocytes
Megakaryocytes

The SI unit of radiation absorbed dose is


Rad
Becquerel
Gray
Sievert

One of the following leukemia almost never develops after radiation?


Acute myeloblastic leukemia
Chronic myeloid leukemia
Acute lumphoblastic leukemia
Chronic lymphocytic leukemia

A 20 year old female was diagnosed with granulose cell tumor of the ovary. Which of the following bio
markerswould be most useful for follow-up of patient?
CA 19-9
CA50
Inhibin
Neuron – specific enolase

Alpha fetoprotein is a marker of:


Hepatoblastoma
Seminoma
Renal cell carcinoma
Choriocarcinoma
Hyperglycemia associated with:
Multiple myeloma
Ewing sarcoma
Osteosarcoma
Chondroblastoma

Which of the following is Not associated with thymoma?


SIADH
Myasthenia gravis
Polymyositis
Hypogammaglobinemia

Which of the following is not true about Neuroblastoma?


Most common extracranial solid tumor in childhood
>50% patients present with metastasis at time of diagnosis
Lung metastases are common
Involve aorta and its branches early

Migratory thrombophlebitis is associated with all of t


Medullary carcinoma of thyroid
Pancreatic neuroendocrine tumor
Pheochromocytoma
Gastrointestinal stromal tumor

All of the following are examples of tumor markers, except:


Alpha-HCG (a-HCG)
Alpha-Feto protein
Thyroglobulin
Beta 2-microglobulin

Which of the following tumors have an increased elevation of placental alkaline phosphatase in the serum as well
asa positive immunohistochemical staining for placental alkaline phosphatase?
Seminoma
Hepatoblastoma
Hepatocellular carcinoma
Peripheral neuroectodermal tumor
In tumor lysis syndrome, all of the following are seen, except:
Hypernatremia
Hypercalcemia
Hyperkalemia
Hyperphosphatemia

Uses of tumor marker are:


Screening of a cancer
Follow up of a cancer patient, esp. for knowing about recurrence
Confirmation of a diagnosed cancer
For monitoring the treatment of a cancer

True about Carcinoembryonic antigen (CEA):


Useful for screening of carcinoma colon
Gives confirmative evidence of Ca. colon
Helpful for follow-up after resection
Levels decrease immediately after resection of tumor

CA·125 is associated with:


Colon ca
Breast ca
Ovarian ca
Bronchogenic ca

Secondaries are common in all, except:


Skull
Hand and feet bones
Proximal limb bones
Pelvic
Hybridoma refers to
Collision tumor
A tumor of brown fat
A hamartoma
A technique for raising monoclonal antibodies

BCL2 is a marker for:


Follicular lymphoma
Mycosis fungoides
B-cell lymphoma
Mantle cell lymphoma

alfa-fetoproteins are a marker of:


Secondaries in liver
Cholangiocarcinoma
Hepatoma
None of the above

Increased level of alpha fetoprotein is found in


Yolk sac tumor
Seminoma
Teratoma
Choriocarcinoma

Migratory thrombophlebitis is seen in:


Disseminated cancer
Rheumatic heart disease
Libman-Sachs endocarditis
All of the above

A 65 years old male diagnosed by biopsy a case of lung carcinoma, with paraneoplastic syndrome and increased
calcium. Probable cause is

Parathyroid hormone
Parathyroid hormone related peptide
Calcitonin
Calcitonin related peptide

Which is associated with polycythemia:


Gastric carcinoma
Fibrosarcoma
Cerebellar hemangioblastoma
All
Serum AFP is increased in all, except:
Acute hepatitis
Hepatocellular carcinoma
Hepatoma
Bladder carcinoma

Carcinoembryonic antigen is elevated in all, except:


Alcoholic cirrhosis
Ca colon
Ulcerative colitis
Emphysema
Desmoid tumor arises from
Wall of the intestine
Anterior abdominal wall
Submucosa
Appendix

Alpha-fetoprotein is a tumor marker of:


Carcinoma ovary
Liver malignancies
Endodermal sinus tumor of testis
Both (b) and (c)

A-fetoprotein is seen in all except:


Hepatocellular carcinoma
Carcinoma colon
Pancreatic carcinoma
Germ cells of testes

The diagnostic tumor marker of liver carcinoma is:


CEA
AFP
CA - 125
All of the above

Spontaneous regression of tumor is seen in:


Wilm’s tumor
Neuroblastoma
Acute monocytic leukemia
Hepatoblastoma

All of the following about tumor markers are properly matched, except:
Prostate cancer - PSA
Colon cancer - CEA
Ovarian cancer – CA 125
Cholangiocarcinoma - AFP

Popcorn calcification is seen in:


Chondrosarcoma
Fibrous dysplasia
Osteoblastoma
Wilms’ tumor

Which one of the following is a frequent cause of serum alpha- fetoprotein level greater than 10 times the
normalupper limit?
Seminoma
Hepatocellular carcinoma of liver
Cirrhosis of liver
Oat cell tumor of lung

Rise of AFP is noted in all except:


Hepatocellular carcinoma
Cirrhosis
Germ cell tumor
Kidney tumor

Catecholamines are increased in:


Neuroblastoma
Retinoblastoma
Medulloblastoma
Nephroblastoma

A 60-year-old man, Shibu is found to have a 3.5-cm mass in the right upper lobe of his lung. A biopsy of this mass
isdiagnosed as a moderately differentiated squamous cell carcinoma. Workup reveals that no bone metastases
arepresent, but laboratory examination reveals that the man’s serum calcium levels are 11.5 mg/dL. This
patient’sparaneoplastic syndrome is most likely the result of the ectopic production of which of the following
substances?
Parathyroid hormone
Parathyroid hormone-related peptide
Calcitonin
Calcitonin-related peptide

During a routine physical examination, a 45-year-old woman Nusheen is noted to have a ruddy complexion.
Herhematocrit is 52%. Her lungs are clear and she does not smoke. Serum erythropoietin levels are elevated. Cancer
ofwhich of the following organs is the most likely cause of her increased hematocrit?
Breast
Colon
Kidney
Stomach

A 62 year-old woman Omvati with advanced, metastatic lung cancer develops profound fatigue and weakness
andalternating diarrhea and constipation. Physical examination demonstrates hyperpigmentation of skin, even in
areasprotected from the sun. Tumor involvement of which endocrine organ is most strongly suggested by this
patient’spresentation?
Adrenal gland
Endocrine pancreas
Ovaries
Pituitary gland

An old man Velu presents with complaints of abdominal and back pain, malaise, nausea, 8 kg weight loss
andweakness, which have been present for 3 or 4 months. His history also reveals several episodes of unilateral
legswelling, which have involved both legs at different times. These findings are most consistent with which of
thefollowing diagnoses?
Pancreatic cancer
Primary sclerosing cholangitis
Splenic infarction
Reflux esophagitis

A 65-year-old woman Ramkali presents to the emergency room with a pathologic fracture of the shaft of her
humerus. X-ray studies demonstrate multiple lytic and blastic bone lesions. Biopsy of one of these lesions
showsadenocarcinoma. Which of the following is the most likely source of the primary tumor?
Breast
Colon
Kidney
Lung

Tumor that follows rule of 10 is:


Pheochromocytoma
Oncocytoma
Lymphoma
Renal cell carcinoma

Which of the followingis a squamous cell carcinoma marker?


Vimentin
Desmin
Cytokeratin
Glial fibrillary acid protein
Marker for ovarian carcinoma in serum is:
CA-125
Fibronectin
Acid Phosphatase
PSA
Which of the following is a marker for carcinoma of lung and breast?
CEA
AEP
HCG
CA-15-3

Secondaries of all the following cause osteolytic lesions except:


Prostate
Kidney
Bronchus
Thyroid

Sacrococcygeal teratoma, marker is:


CEA
β- HCG
S100
CA-125

Which of the following mutation is seen in malignant melanoma


N-myc
CDKN2A
RET
Rb

Marker of small cell cancer of lung is:


Chromogranin
Cytokeratin
Desmin
Vimentin

Which of the following is tumor marker of seminoma?


AFP
LDH
PLAP
HCG

Which of the following is a special stain for rhabdomyosarcoma?


Cytokeratin
Synaptophysin
Desmin
Myeloperoxidase

The most common cause of malignant adrenal mass


Adrenocortical carcinoma
Malignant Phaeochromocytoma
Lymphoma
Metastasis from another solid tissue tumor

Commonest cancer in which metastasis is seen in brain in


Breast
Lung
Kidney
Intestines

Which of the following is incorrect about neuro- blastoma ?


Most common abdominal tumor in infants
X-ray abdomen shows calcification
Can show spontaneous regression
Urine contains 5H.I.A.A

Colloid degeneration is occurs in:


Salivary gland
Prostate gland.
Thyroid gland
Pituitary gland

An 87-year-old male develops worsening heart failure. Workup reveals decreased left ventricular filling due to
decreased compliance of the left ventricle. Two months later the patient dies, and postmortem sections reveal
deposits of eosinophilic, Congo red positive material in the interstitial of his heart. When viewed under polarized
light. This material displays an apple-green birefringence. What is the correct diagnosis?
Amyloidosis
Glycogenosis
Hemochromatosis
Sarcoidosis
In cases of renal failure on long-term hemodialysis, there is development of following type of amyloid:
Amyloid light chain (AL)
Amyloid-associated protein (AA)
Amyloid β2 microglobulin (Aβ2m)
β amyloid protein (Aβ)
The most common form of amyloid in third world countries is:
Primary
Secondary
Hereditary
Localized

Cardiac amyloidosis often produces:


Dilated cardiomyopathy
Constrictive cardiomyopathy
Restrictive cardiomyopathy
Ischemic cardiomyopathy

In senile cardiac amyloidosis, the biochemical form of amyloid is:


AL
AA
ATTR
Aβ2m

Health risk in obesity is due to weight in excess of the following for age and sex:
0.1
0.2
0.3
0.4

Obesity is due to:


Hyperplasia of adipocytes only
Hypertrophy of adipocytes only
Hyperplasia as well as hypertrophy of adipocytes
Fatty change in liver only

Most often secondary amyloidosis occurs when the following pathologies:


Chronic suppuration
Acute inflammation
Cellular necrosis
Hyaline degeneration

The substance with fibrillar structure, which forms under the pathological conditions, is:
Reabsorption droplets
Russell bodies
Lipids
Amyloid
The substance giving red color with the Congo red stain is:
Lipid
Hyaline
Water
Amyloid

In long-standing hypertension and diabetes mellitus, the walls of arterioles, especially in the kidney, become:
Serous
Thined
Hyalinized
Ulcered

The pathologic proteinaceous substance, accumulating only between cells in various tissues and organs of the
bodyis:
Glycogen
Hyaline
Water
Amyloid

The pathologic proteinaceous substance, accumulating both within cells and in the extracellular matrix in
varioustissues and organs of the body is:
Glycogen
Hyaline
Water
Amyloid
The characteristics of amyloid fibrils include all of the following, except:
Fibril composed of paired filaments
Nonbranching fibrils
Fibrils with an indefinite diameter
Fibrils with definite length

The organ affected in both primary and secondary amyloidosis is:


Kidney
Stomach
Uterus
Brain

The deposits of amyloid sees in all tissues, except:


Mesangium and capillary loops
Basement membranes of blood vessels
Tubular basement membranes
Epithelium in proximal renal tubules

All of the following pathological processes are reversible, except:


Mucoid changes
Cellular swelling
Amyloidosis
Edema

The organ most commonly and seriously damaged in amyloidosis is:


Stomach
Kidney
Lung
Liver

Reactive systemic amyloidosis knows to be all of the following, except:


Secondary amyloidosis
Complication of tuberculosis
Hereditary amyloidosis
Complication of osteomyelitis
Amyloid in primary amyloidosis is usually systemic and belongs to the following biochemical type:
AL type
AA type
Aβ type
ATTR type

Reactive systemic amyloidosis occurs in association with all of the following diseases, except:
Tuberculosis
Bronchiectasis
Chronic osteomyelitis
Hepatitis B

Reactive systemic amyloidosis occurs in association with all of the following diseases, except
Rheumatoid arthritis
Ankylosing spondylitis
Chronic appendicitis
Myeloma

Secondary amyloidosis damages all of the following organs, except:


Kidneys
Liver
Spleen
Brain
Macroscopically the organs affected by amyloidosis are characterized by all of the following, except:
Enlarged
Firm
Waxy in appearance
Soft

The common cause of death in patients with secondary amyloidosis is insufficiency of:
Kidneys
Heart
Liver
Lung

The organs that should be histological examined in patients with amyloidosis are all of the following, except:
Kidney
Eye
Rectum
Gingiva

The reversible process caused by accumulation of glycosaminoglycans in extracellular matrix due to the increase
ofvascular permeability is:
Amyloidosis
Glycogenoses
Hyalinosis
Mucoid changes

The irreversible process caused by accumulation of proteins with high molecular weight associated with
thedestruction of connective tissue is:
Amyloidosis
Glycogenoses
Hyalinosis
Fibrinoid changes

All the pathological processes are irreversible, except


Mucoid changes
Fibrinoid changes
Amyloidosis
Apoptosis
In amyloidosis macroscopically kidney changed in the following way:
Cut surface is pale and translucent
Dense and sharply reduced
Enlarged and dense
True a + c

All examples of extracellular hyaline are change, except:


Old scar
Hyaline arteriolosclerosis
Chronic glomerulonephritis
Mallory’s hyaline

In amyloidosis macroscopically liver changed in the following way:


Enlarged, pale, waxy and firm
Dense and sharply reduced
Enlarged and yellow color
Soft and enlarged

In amyloidosis macroscopically spleen changed in the following way:


Cut surface is translucent pale and waxy
Dense and sharply reduced
Cut surface – map-like areas of amyloid
True a + c

Causes of hyalinosis are all of the following, EXCEPT:


Fibrinoid changes
Inflammation
Apoptosis
Sclerosis

Most specific histological sign in mucoid changes is:


Metachromasia.
Metaplasia.
Metastasis.
Metakinesis.

Most common localization of mucoid changes is


Nervous system.
Cardiovascular system
Urinary system
Blood system

Simple hyaline occurs in:


Diabetes mellitus.
Arterial hypertension
Rheumatic fever
Rheumatoid arthritis

Simple hyaline occurs in


Diabetes mellitus
systemic lupus erythematosus
Atherosclerosis.
Rheumatic fever.

Lipohyalin occurs in:


Diabetes mellitus.
Arterial hypertension
Atherosclerosis
Rheumatic fever

Compound hyalin occurs in:


Diabetes mellitus.
Arterial hypertension
Atherosclerosis.
Rheumatic fever.

Compound hyalin occurs in


Diabetes mellitus
Arterial hypertension
Atherosclerosis
Rheumatoid arthritis

The stain used to identify amyloid is


Hematoxylin and eosin stain.
Metachromatic stain
Congo-red stain
Sudan III stain.
The pathologic proteinaceous substance, accumulating only between cells in various tissues and organs of the bodyis
Glycogen.
Hyaline.
Water
Amyloid.

Specific method for diagnosis of amyloid in fresh tissue is:


Virchow test.
Rokitansky test.
Ewing test
Masson test

Variant of systemic amyloidosis is:


Senile cardiac.
Secondary
Senile cerebral.
Endocrine.

Variant of systemic amyloidosis is


Senile cardiac.
Senile cerebral
Heredofamilial
Endocrine

Variant of systemic amyloidosis is


Senile cardiac
Senile cerebral
Endocrine.
Hemodialysis-associated
Variant of localized amyloidosis is:
Senile cardiac.
Primary
Secondary
Heredofamilial

Variant of localized amyloidosis is


Primary
Secondary.
Senile cerebral
Heredofamilial
Variant of localized amyloidosis is
Primary
Secondary.
Heredofamilial.
Endocrine

Variant of localized amyloidosis is?


Primary.
Secondary
Heredofamilial
Tumor-forming

Localization of pericollagenous amyloidosis is:


Liver
Spleen
Heart.
Kidneys

Localization of pericollagenous amyloidosis is


Liver
Spleen
Bowel.
Kidneys

Localization of pericollagenous amyloidosis is:


Liver
Spleen
Nerves
Kidneys

Localization of perireticulin amyloidosis is:


Heart
Tongue.
Nerves
Kidneys

Localization of perireticulin amyloidosis is:


Heart
Tongue
Nerves
Liver
Localization of perireticulin amyloidosis is :
Heart
Tongue
Nerves
Spleen

Cause of obesity which associated with excessive nutrition is called:


Primary.
Alimentary
Cerebral
Endocrine

Type of obesity with unknown cause is called:


Primary
Alimentary
Cerebral
Endocrine

1st degree of obesity is associated with increasing of body weight over than normal by:
20-29%.
10-20%.
15-25%.
20-35%

2nd degree of obesity is associated with increasing of body weight over than normal by:
30-40%
30-49%
25-45%.
30-55%.

3rd degree of obesity is associated with increasing of body weight over than normal by:
50-99%.
45-85%
50-75%
60-90%.

4th degree of obesity is associated with increasing of body weight over than normal by
90% and more
85% and more.
120% and more.
100% and more

Deposition of fat in abdomen area in obesity is called:


Upper type.
Middle type.
Lower type.
Simmetric type

Deposition of fat in area of face and neck in obesity is called:


Upper type.
Middle type
Lower type
Simmetric type

A 38-year-old female presents with intermittent pelvic pain. Physical examination reveals a 3-cm mass in the area of
her right ovary. Histological sections from this ovarian mass reveal a papillary tumor with multiple, scattered
small,round, and laminated calcifications. These structures are most likely the result of
Apoptosis
Dystrophic calcification
Enzymatic necrosis
Hyperparathyroidism

The following pigments are stainable by Prussian blue reaction except:


Hemosiderin
Ferritin
Hematin
Hemochromatosis

Idiopathic calcinosis cutis is an example of:


Necrotizing inflammation
Dystrophic calcification
Metastatic calcification
Calcified thrombi in veins

Risk factors implicated in the etiology of cholesterol gallstones include the following except:
Family history
Obesity
Hemolytic anemia
Oral contraceptives
The following type of gallstones is generally unassociated with changes in the gallbladder wall:
Cholesterol
Mixed
Combined
Pigment

The following type of renal calculi is radiolucent:


Calcium oxalate
Struvite
Uric acid
Calcium phosphate

The following type of renal calculi is infection-induced:


Calcium oxalate
Struvite
Uric acid
Cystine

The color of organs hemosiderin is:


Black
Sky-blue
Yellow
Brown

The pigment hematoidin is most identical with:


Porphyrin
Bilirubin
Hemosiderin
Hemozoin

Lipofuscin, the golden yellow pigment sees in heart muscle


Hypertrophy
Atrophy
Hyperplasia
Metaplasia

What stain is specific for iron:


Hematoxylin and eosin
Sudan III
Prussian blue reaction
PAS-reaction
Hemosiderin has all of the following features, except:
Hemoglobin-derived
Golden yellow-to-brown
Granular or crystalline
Synthesized by enzyme tyrosinase

Hemosiderin in the lung accumulates in:


Leukocytes
Lymphocytes
Macrophages
Fibroblasts

Idiopathic pulmonary hemosiderosis characterizes by all of the following pathologic symptoms, except:
Productive cough
Hemophtysis
Anemia
Heavy proteinuria

The lungs in pulmonary hemosiderosis are:


Enlarged with turbid fluid exuding from cut surface
Increased in weight with areas of red-brown consolidation
Diminished with areas of red-brown consolidation
Diminished with turbid fluid exuding from cut surface

The color of hemosiderin granules stained with Prussian blue reaction is:
Yellow
Brown
Orange-red
Blue-black

Hemosiderosis sees in all pathologic processes, except:


Inflammation
Hereditary increased absorption of dietary iron
Impaired use of iron
Hemolytic anemia

Morphologic changes in genetic hemochromatosis characterizes by all of the following, except:


Metastatic calcification of many organs
Deposition of hemosiderin in many organs
Deposition of hemosiderin in the skin
Liver cirrhosis

Melanin has all of the following features, except:


Localized endogenous
Formed of ferritin
Non-hemoglobin-derived
Black-brown

An increased amount of melanin in melanocytes and within basal keratinocytes is also known as:
Vacuolization
Vitiligo
Hyperpigmentation (melanosis)
Albinism

Lipofuscin has all of the following features, except:


Aging pigment
Most often seen in kidney
Yellow-brown
Noninjurious to the cells or their function

Lipofuscin pigment granules in cells result from:


Hemosiderosis
Accumulation of protein in cytoplasm
Accumulation of lipids in cytoplasm
Intracellular lipid peroxidation

Lipofuscin granules in cells see in:


Necrosis
Denervation atrophy
Brown atrophy
Atrophy from pressure

Heart and liver of a patient with cancer cachexia macroscopically sees as:
Diminished and brown
Diminished and brown
Enlarged and brown
Enlarged and yellow

Lipofuscin in the liver may founds in:


Unchanged cells
Cells with ballooning degeneration
Cells with hyaline droplets
Cells with regressive changes

Bilirubin has all of the following features, except:


The end product of hem degradation
Derived from breakdown erythrocytes
Brilliant-yellow
Stained in blue-black color with Prussian-blue reaction

Jaundice occurs in all of the following pathologic processes, except:


Increased hepatocellular excretion
Excessive production of bilirubin
Reduced hepatocyte uptake
Impaired conjugation of bilirubin

Biliary ducts obstruction by gallstones may lead to:


Liver hemosiderosis
Liver steatosis
Ballooning degeneration of hepatocytes
Cholestasis and jaundice

Complications or well-established associations of gallstones include all of the following, except:


Biliary obstruction
Brown atrophy of the liver
Pancreatitis
Intestinal obstruction

Which of the following sites is an example of metastatic calcification?


The kidney in nephrocalcinosis
The mitral valve in mitral stenosis of rheumatic organ
The left anterior ascending coronary artery affected by atheromatous plaques
The lung involved by metastatic carcinoma

Point out the posttuberculosis lung lesion:


Granuloma
Cavity
Fibrocalcific scar
Caseation in lymph node
Dystrophic calcification encounters in all of the following areas, except:
Coagulative necrosis
Intracellular fat accumulation
Caseous necrosis
Liquefactive necrosis

Initiation of intracellular calcification occurs in:


Cytoplasm
Lysosome
Nucleus
Mitochondria

The causes of metastatic calcification are all of the following, except:


Diabetes mellitus
Increased secretion of parathyroid hormone
Destruction of bone tissue
Vitamin D-related disorders

Metastatic calcification may occur in all of the following organs, except:


Stomach (gastric mucosa)
Kidneys
Lungs
Liver

Deficiency of vitamin D tends to cause:


Hypercalcemia
Hypocalcemia
Hyperpigmentation
Hypopigmentation

Deficiency of vitamin D in the adults leads to:


Osteosclerosis
Osteomyelitis
Osteomalati
Osteonecrosis

Hemoglobin derivative pigment which occurs in normal conditions:


Hemosiderin
Hematoidin
Hemin
Hemomelanin
Hemoglobin derivative pigment which occurs in normal conditions
Ferritin
Hematoidin.
Hemin
Hemomelanin.

Hemoglobin derivative pigment which occurs in normal conditions :


Bilirubin
Hematoidin
Hemin.
Hemomelanin

Hemoglobin derivative pigment which occurs only in pathologic conditions:


Hemosiderin
Hematoidin
Ferritin
Bilirubin

Hemoglobin derivative pigment which occurs only in pathologic conditions:


Hemosiderin
Porphyrin
Ferritin.
Bilirubin.

Hemoglobin derivative pigment which occurs only in pathologic conditions


Hemosiderin
Hematin
Ferritin
Bilirubin

Iron-containing pigment is:


Bilirubin
Hematoidin
Hemosiderin
Porphyrin

Iron-containing pigment is
Bilirubin
Hematoidin
Ferritin
Porphyrin

Iron-containing pigment is:


Bilirubin.
Hematoidin.
Hematin
Porphyrin

The stain used to identify iron-containing pigments is:


Prussian-blue reaction.
PAS reaction
Congo-red stain
Sudan III stain

In hemosiderosis organs become:


Enlarged, brown, hard.
Small, red, soft
Enlarged, yellow, hard.
Enlarged, brown, soft.

Cause of generalized hemosiderosis is:


Intracerebral hemorrhage.
Intoxications.
Brown induration of lungs
Pulmonary hemorrhagic infarct

Cause of generalized hemosiderosis is


Intracerebral hemorrhage
Heterohemotransfusions
Brown induration of lungs
Pulmonary hemorrhagic infarct

Cause of generalized hemosiderosis is ?


Typhoid fever
Yellow fever
Malaria
Diphteria

Cause of localized hemosiderosis is:


Intoxications
Heterohemotransfusions.
Malaria
Brown induration of lungs

Cause of localized hemosiderosis is :


Intoxications
Heterohemotransfusions
Malaria
Intracerebral hemorrhage

Cause of localized hemosiderosis is:


Intoxications
Heterohemotransfusions
Malaria
Pulmonary hemorrhagic infarct.

Accumulation of which pigment may leads to decrease of blood pressure?


Ferritin
Hemosiderin
Bilirubin
Hemomelanin

Accumulation of which pigment may leads to jaundice?


Ferritin.
Hemosiderin
Bilirubin.
Hemomelanin

Cause of prehepatic jaundice is:


Hepatitis.
Liver cirrhosis
Obstruction of bile ducts by stones
Hemolytic disease of newborns

Cause of prehepatic jaundice is :


Hepatitis
Liver cirrhosis
Obstruction of bile ducts by stones.
Heterohemotransfusions
Cause of hepatocellular jaundice is:
Obstruction of bile ducts by stones
Compression of bile ducts by tumors
Heterohemotransfusions
Hepatitis

Cause of hepatocellular jaundice is :


Obstruction of bile ducts by stones
Compression of bile ducts by tumors
Heterohemotransfusions.
Liver cirrhosis

Cause of posthepatic jaundice is:


Obstruction of bile ducts by stones
Hepatitis
Heterohemotransfusions.
Liver cirrhosis

Cause of posthepatic jaundice is :


Compression of bile ducts by tumors
Hepatitis.
Heterohemotransfusions.
Liver cirrhosis

Which pigment is “tear and wear”:


Bilirubin
Hematoidin
Hematin.
Lipofuscin

Which pigment is tyrosin-derived:


Bilirubin
Hematoidin
Hematin.
Melanin

Generalized hyperpigmentation of melanin occurs in:


Addison’s disease
Melanosis coli
Lentigo
Nevus.
Focal hyperpigmentation of melanin occurs in:
Addison’s disease
Cachexia.
Avitaminosis
Nevus.

General hypopigmentation of melanin occurs in


Leukoderma.
Albinism
Vitiligo
Cachexia

Focal hypopigmentation of melanin occurs in:


Albinism
Nevus
Vitiligo
Cachexia

Distrophic calcification occurs in:


Hyperparathyroidism
Hypervitaminosis D
Hyperthyroidism
Necrosis.

Metastatic calcification occurs in:


Hyperparathyroidism.
Damaged heart valves
Atheromas.
Necrosis.

Gross appearance of calcificate is:


Hard, greywish-white.
Soft, yellow
Hard, greywish-red
Soft, greywish-white.

Area of calcification in histological examination with hematoxylin and eosin staining is:
Red
Black.
Brown
Blue.

Сongenital absence of thymus is called:


Aplasia
Hypoplasia
Dysplasia
Atrophy

Congenital incomplete development of thymus is called:


Aplasia
Hypoplasia
Dysplasia
Atrophy

Aquired decrease in weight of thymus is called:


Aplasia
Hypoplasia
Dysplasia
Atrophy

Abnormal development of thymus is called:


Aplasia
Hypoplasia
Dysplasia
Atrophy

Morphologically immediate type of hypersensitivity occurs as:


Fibrinoid necrosis
Lympho-hystiocytic infiltration
Macrophageal infiltration
Granulomatosis

Morphologically immediate type of hypersensitivity occurs as


Lympho-hystiocytic infiltration
Macrophageal infiltration
Granulomatosis
Plasmatic saturation
Morphologically immediate type of hypersensitivity occurs as
Lympho-hystiocytic infiltration
Mucoid changes
Macrophageal infiltration
Granulomatosis

Morphologically immediate type of hypersensitivity occurs as:


Lympho-hystiocytic infiltration
Fibrinoid changes
Macrophageal infiltration
Granulomatosis

Morphologically immediate type of hypersensitivity occurs as:


Lympho-hystiocytic infiltration
Fibrinous-hemorrhagic exudate
Macrophageal infiltration
Granulomatosis

Morphological signs of immediate type of hypersensitivity are all the following, except:
Lympho-hystiocytic infiltration
Mucoid and Fibrinoid changes
Plasmatic saturation
Fibrinoid necrosis

Morphological signs of immediate type of hypersensitivity are all the following, except:
Mucoid and Fibrinoid changes
Plasmatic saturation
Granulomatosis
Fibrinoid necrosis

Morphological signs of immediate type of hypersensitivity are all the following, except:
Mucoid and Fibrinoid changes
Plasmatic saturation
Fibrinoid necrosis
Macrophage infiltration

Morphologically delayed type of hypersensitivity occurs as:


Lympho-hystiocytic infiltration
Mucoid and Fibrinoid changes
Plasmatic saturation
Fibrinoid necrosis
Morphologically delayed type of hypersensitivity occurs as:
Mucoid and Fibrinoid changes
Plasmatic saturation
Granulomatosis
Fibrinoid necrosis

Morphologically delayed type of hypersensitivity occurs as:


Mucoid and Fibrinoid changes
Plasmatic saturation
Fibrinoid necrosis
Macrophageal infiltration

Morphologically delayed type of hypersensitivity occurs as:


Mucoid and Fibrinoid changes
Plasmatic saturation
Fibrinoid necrosis
Cytoplasmic bridges between lymphocytes and macrophages

Morphological signs of delayed type of hypersensitivity are all the following, except:
Fibrinoid necrosis
Lympho-hystiocytic infiltration
Macrophageal infiltration
Granulomatosis

Morphological signs of delayed type of hypersensitivity are all the following, except:
Lympho-hystiocytic infiltration
Macrophageal infiltration
Granulomatosis
Plasmatic saturation

Morphological signs of delayed type of hypersensitivity are all the following, except:
Lympho-hystiocytic infiltration
Mucoid changes
Macrophageal infiltration
4.Granulomatosis

Morphological signs of delayed type of hypersensitivity are all the following, except:
Lympho-hystiocytic infiltration
Fibrinous-hemorrhagic exudate
Macrophageal infiltration
Granulomatosis

Morphological signs of transplant rejection are all the following, except:


Lympho-hystiocytic infiltration
Edema of transplant
Macrophageal infiltration
4.Granulomatosis

Morphological signs of transplant rejection are all the following, except ?


Lympho-hystiocytic infiltration
Edema of transplant
Macrophageal infiltration
Cytoplasmic bridges between lymphocytes and macrophages

Organ specific immune disease is:


Hashimoto thyroiditis
Rheumatoid arthritis
Systemic Lupus Erythematosus
Scleroderma

Organ specific immune disease is :


Rheumatoid arthritis
Systemic Lupus Erythematosus
Scleroderma
Encephalomyelitis

Non-organ specific immune disease is:


Hashimoto thyroiditis
Rheumatoid arthritis
Encephalomyelitis
Polyneuritis

Non-organ specific immune disease is :


Hashimoto thyroiditis
Encephalomyelitis
Systemic Lupus Erythematosus.
Polyneuritis.

Variant of primary immunodeficiency syndrome occurs:


In leukemia
Under radial therapy
In aplasia of thymus
In infections

Variant of primary immunodeficiency syndrome occurs :


In leukemia
Under radial therapy.
In sarcoidosis
In hypoplasia of thymus.
Variant of secondary immunodeficiency syndrome is:
Lui-Bar syndrome
Neseloff syndrome
DiGeorge syndrome
AIDS

Complication of immunodeficiency syndromes is:


Arterial hypertension
Myocardial infarction
Typhoid fever
Sepsis

Complication of immunodeficiency syndromes is :


Recurrence of bronchial asthma
Recurrence of chronic hepatitis
Recurrence of chronic cholecystitis
Recurrence of tuberculosis

Complication of immunodeficiency syndromes is:


Arterial hypertension
Myocardial infarction
Typhoid fever
Purulent pneumonia

A 22-year-old woman nursing her newborn develops a tender erythematous area around the nipple of her left
breast. A thick, yellow fluid is observed to drain from an open fissure. Examination of this breast fluid under the
lightmicroscope will most likely reveal an abundance of which of the following inflammatory cells?
B lymphocytes
Eosinophils
Mast cells
Neutrophils
A 63-year-old man becomes febrile and begins expectorating large amounts of mucopurulent sputum.
Sputumcultures are positive for Gram-positive diplococci. Which of the following mediators of inflammation
providespotent chemotactic factors for the directed migration of inflammatory cells into the alveolar air spaces of
thispatient?
Bradykinin
Histamine
Myeloperoxidase
N-formylated peptides

A 59-year-old man suffers a massive heart attack and expires 24 hours later due to ventricular arrhythmia.
Histologicexamination of the affected heart muscle at autopsy would show an abundance of which of the
followinginflammatory cells?
Fibroblasts
Lymphocytes
Macrophages
Neutrophils

A 5-year-old boy punctures his thumb with a rusty nail. Four hours later, the thumb appears red and swollen.
Initialswelling of the boy’s thumb is primarily due to which of the following mechanisms?
Decreased intravascular hydrostatic pressure
Decreased intravascular oncotic pressure
Increased capillary permeability
Increased intravascular oncotic pressure

An 80-year-old woman presents with a 4-hour history of fever, shaking chills, and disorientation. Her blood
pressureis 80/40 mm Hg. Physical examination shows diffuse purpura on her upper arms and chest. Blood cultures
arepositive for Gram negative organisms. Which of the following cytokines is primarily involved in the pathogenesis
ofdirect vascular injury in this patient with septic shock?
Interferon-g
Interleukin-1
Platelet-derived growth factor
Tumor necrosis factor-a

A 24-year-old intravenous drug abuser develops a 2-day history of severe headache and fever. His temperature
is38.7°C (103°F). Blood cultures are positive for Gram-positive cocci The patient is given intravenous antibiotics,
buthe deteriorates rapidly and dies. A cross section of the brain at autopsy (shown in the image) reveals
twoencapsulated cavities. Which of the following terms best characterizes this pathologic finding?
Chronic inflammation
Fibrinoid necrosis
Granulomatous inflammation
Suppurative inflammation
A 36-year-old woman with pneumococcal pneumonia develops a right pleural effusion. The pleural fluid displays
ahigh specific gravity and contains large numbers of polymorphonuclear (PMN) leukocytes. Which of the
followingbest characterizes this pleural effusion?
Fibrinous exudate
Lymphedema
Purulent exudate
Serosanguineous exudate

A 33-year-old man presents with a 5-week history of calf pain and swelling and low-grade fever. Serum levels
ofcreatine kinase are elevated. A muscle biopsy reveals numerous eosinophils. What is the most likely etiology of
thispatient’s myalgia?
Autoimmune disease
Bacterial infection
Muscular dystrophy
Parasitic infection

A 10-year-old boy with a history of recurrent bacterial infections presents with fever and a productive
cough.Biochemical analysis of his neutrophils demonstrates that he has an impaired ability to generate reactive
oxygenspecies. This patient most likely has inherited mutations in the gene that encodes which of the following
proteins?
Catalase
Cytochrome P450
Myeloperoxidase
NADPH oxidase

A 25-year-old woman presents with a history of recurrent shortness of breath and severe wheezing.
Laboratorystudies demonstrate that she has a deficiency of C1 inhibitor, an esterase inhibitor that regulates the
activation of theclassical complement pathway. What is the diagnosis?
Chronic granulomatous disease
Hereditary angioedema
Myeloperoxidase deficiency
Selective IgA deficiency

A 40-year-old man complains of a 2-week history of increasing abdominal pain and yellow discoloration of hissclera.
Physical examination reveals right upper quadrant pain. Laboratory studies show elevated serum levels ofalkaline
phosphatase (520 U/dL) and bilirubin (3.0 mg/dL). A liver biopsy shows portal fibrosis, with scatteredforeign bodies
consistent with schistosome eggs. Which of the following inflammatory cells is most likely topredominate in the
portal tracts in the liver of this patient?
Basophils
Eosinophils
Macrophages
Monocytes
A 41-year-old woman complains of excessive menstrual bleeding and pelvic pain of 4 months. She uses
anintrauterine device for contraception. Endometrial biopsy (shown in the image) reveals an excess of plasma
cells(arrows) and macrophages within the stroma. The presence of these cells and scattered lymphoid follicles within
theendometrial stroma is evidence of which of the following conditions?
Acute inflammation
Chronic inflammation
Granulation tissue
Granulomatous inflammation

A 62-year-old woman undergoing chemotherapy for breast cancer presents with a 3-day history of fever and
chestpain. Cardiac catheterization reveals a markedly reduced ejection fraction with normal coronary blood flow.
Amyocardial biopsy is obtained, and a PCR test for coxsackievirus is positive. Histologic examination of this
patient’smyocardium will most likely reveal an abundance of which of the following inflammatory cells?
Eosinophils
Lymphocytes
Macrophages
Mast cells

A 58-year-old woman with long-standing diabetes and hypertension develops end-stage renal disease and dies
inuremia. A shaggy fi brin-rich exudate is noted on the visceral pericardium at autopsy (shown in the image).
Whichof the following best explains the pathogenesis of this fibrinous exudate?
Antibody binding and complement activation
Chronic passive congestion
Injury and increased vascular permeability
Margination of segmented neutrophils

A 68-year-old man presents with fever, shaking chills, and shortness of breath. Physical examination shows rales
anddecreased breath sounds over both lung fields. The patient exhibits grunting respirations, 30 to 35 breaths
perminute, with flaring of the nares. The sputum is rusty yellow and displays numerous polymorphonuclear
leukocytes.Which of the following mediators of inflammation is chiefly responsible for the development of fever in
this patient?
Arachidonic acid
Interleukin-1
Leukotriene B4
Prostacyclin (PGI2)

A 35-year-old woman presents with a 5-day history of a painful sore on her back. Physical examination reveals a 1-
cm abscess over her left shoulder. Biopsy of the lesion shows vasodilation and leukocyte margination (shown in
theimage). What glycoprotein mediates initial tethering of segmented neutrophils to endothelial cells in this skin
lesion?
Cadherin
Entactin
Integrin
Selectin

A 14-year-old boy receives a laceration on his forehead during an ice hockey game. When he is first attended to
bythe medic, there is blanching of the skin around the wound. Which of the following mechanisms accounts for
thistransient reaction to neurogenic and chemical stimuli at the site of injury?
Constriction of postcapillary venules
Constriction of precapillary arterioles
Dilation of postcapillary venules
Dilation of precapillary arterioles

An 8-year-old girl with asthma presents with respiratory distress. She has a history of allergies and upper
respiratorytract infections. She also has history of wheezes associated with exercise. Which of the following
mediators ofinflammation is the most powerful stimulator of bronchoconstriction and vasoconstriction in this
patient?
Bradykinin
Complement proteins
Interleukin-1
Leukotrienes

A 75-year-old woman complains of recent onset of chest pain, fever, and productive cough with rust-coloredsputum.
A chest X-ray reveals an infiltrate in the right middle lobe. Sputum cultures are positive for
Streptococcuspneumoniae. Phagocytic cells in this patient’s affected lung tissue generate bacteriocidal hypochlorous
acid using

which of the following enzymes?

Catalase
Cyclooxygenase
Myeloperoxidase
NADPH oxidase

A 28-year-old woman cuts her hand while dicing vegetables in the kitchen. The wound is cleaned and sutured.
Fivedays later, the site of injury contains an abundance of chronic inflammatory cells that actively secrete
interleukin-1,tumor necrosis factor-, interferon-, numerous arachidonic acid derivatives, and various enzymes. Name
these cells
B lymphocytes
Macrophages
Plasma cells
Smooth muscle cells

A 68-year-old man with prostate cancer and bone metastases presents with shaking chills and fever. The
peripheralWBC count is 1,000/L (normal = 4,000 to 11,000/L). Which of the following terms best describes this
hematologicfinding?
Leukocytosis
Leukopenia
Neutrophilia
Pancytopenia

A 25-year-old machinist is injured by a metal sliver in his left hand. Over the next few days, the wounded
areabecomes reddened, tender, swollen, and feels warm to the touch. Redness at the site of injury in this patient is
causedprimarily by which of the following mechanisms?
Hemorrhage
Hemostasis
Neutrophil margination
Vasodilation

A 37-year-old man with AIDS is admitted to the hospital with a 3-week history of chest pain and shortness of
breath.An X-ray film of the chest shows bilateral nodularities of the lungs. A CT-guided lung biopsy is shown in the
image.The multinucleated cell in the center of this field is most likely derived from which of the following
inflammatorycells?
Basophils
Capillary endothelial cells
Macrophages
Myofi broblasts

A 10-year-old girl presents with a 2-week history of puffiness around her eyes and swelling of the legs and
ankles.Laboratory studies show hypoalbuminemia and proteinuria. The urinary sediment contains no inflammatory
cells orred blood cells. Which of the following terms describes this patient’s peripheral edema?
Effusion
Exudate
Hydropic change
Transudate

A 25-year-old woman develops a sore, red, hot, swollen left knee. She has no history of trauma and no
familialhistory of joint disease. Fluid aspirated from the joint space shows an abundance of segmented
neutrophils.Transendothelial migration of acute inflammatory cells into this patient’s joint space was mediated
primarily bywhich of the following families of proteins?
Entactins
Fibrillins
Fibronectins
Integrins
A 50-year-old woman is discovered to have metastatic breast cancer. One week after receiving her first dose
ofchemotherapy, she develops bacterial pneumonia. Which of the following best explains this patient’s susceptibility
tobacterial infection?
Depletion of serum complement
Impaired neutrophil respiratory burst
Inhibition of clotting factor activation
Neutropenia

A 53-year-old man develops weakness, malaise, cough with bloody sputum, and night sweats. A chest X-ray
revealsnumerous apical densities bilaterally. Exposure to Mycobacterium tuberculosis was documented 20 years
ago, and M. tuberculosis I identified in the sputum. The patient subsequently dies of respiratory insufficiency. The
lungs areexamined at autopsy (shown in the image). Which of the following best characterizes the histopathologic
features ofthis pulmonary lesion?
Acute suppurative inflammation
Chronic inflammation
Fat necrosis
Granulomatous inflammation

A 59-year-old man experiences acute chest pain and is rushed to the emergency room. Laboratory studies and
ECGdemonstrate an acute myocardial infarction; however, coronary artery angiography performed 2 hours later
does notshow evidence of thrombosis. Intravascular thrombolysis that occurred in this patient was mediated by
plasminogenactivators that were released by which of the following cells?
Cardiac myocytes
Endothelial cells
Macrophages
Segmented neutrophils

A 68-year-old coal miner with a history of smoking and emphysema develops severe air-flow obstruction andexpires.
Autopsy reveals a “black lung,” with coal-dust nodules scattered throughout the parenchyma and a centra area of
dense fibrosis. The coal dust entrapped within this miner’s lung was sequestered primarily by which of thefollowing
cells?
Endothelial cells
Fibroblasts
Lymphocytes
Macrophages

A 40-year-old man presents with 5 days of productive cough and fever. Pseudomonas aeruginosa is isolated from
apulmonary abscess. The CBC shows an acute effect characterized by marked leukocytosis (50,000 WBC/L), and
thedifferential count reveals numerous immature cells (band forms). Which of the following terms best describes
thesehematologic findings?
Leukemoid reaction
Leukopenia
Myeloid metaplasia
Myeloproliferative disease
A 19-year-old woman presents with 5 days of fever (38°C/101°F) and sore throat. She reports that she has
feltfatigued for the past week and has difficulty swallowing. A physical examination reveals
generalizedlymphadenopathy. If this patient has a viral infection, a CBC will most likely show which of the
followinghematologic findings?
Eosinophilia
Leukopenia
Lymphocytosis
Neutrophilia

A 40-year-old woman presents with an 8-month history of progressive generalized itching, weight loss, fatigue,
andyellow sclerae. Physical examination reveals mild jaundice. The antimitochondrial antibody test is positive. A
liverbiopsy discloses periductal inflammation and bile duct injury (shown in the image). Which of the
followinginflammatory cells is the principal mediator of destructive cholangitis in this patient?
Eosinophils
B lymphocytes
T lymphocytes
Mast cells

A 25-year-old woman presents with a 2-week history of febrile illness and chest pain. She has an
erythematous,macular facial rash and tender joints, particularly in her left wrist and elbow. A CBC shows mild
anemia andthrombocytopenia. Corticosteroids are prescribed for the patient. This medication induces the synthesis
of aninhibitor of which of the following enzymes in inflammatory cells?
Lipoxygenase
Myeloperoxidase
Phospholipase A2
Phospholipase C

A 22-year-old man develops marked right lower quadrant abdominal pain over the past day. On physicalexamination
there is rebound tenderness on palpation over the right lower quadrant. Laparoscopic surgery isperformed, and the
appendix is swollen, erythematous, and partly covered by a yellowish exudate. It is removed, anda microscopic
section shows infiltration with numerous neutrophils. The pain experienced by this patient ispredominantly the
result of which of the following two chemical mediators?
Complement C3b and IgG
Interleukin-1 and tumor necrosis factor
Histamine and serotonin
Prostaglandin and bradykinin
A 40-year-old woman had laparoscopic surgery 3 months ago. Now she has a small 0.5 cm nodule beneath the skinat
the incision site that was sutured. Which of the following cell types is most likely to be most characteristic of
theinflammatory response in this situation?
Mast cell
Eosinophil
Giant cell
Neutrophil
A 39-year-old man incurs a burn injury to his hands and arms while working on a propane furnace. Over the next
3weeks, the burned skin heals without the need for skin grafting. Which of the following is the most critical factor
indetermining whether the skin in the region of the burn will regenerate?
Good cardiac output with tissue perfusion
Persistence of skin appendages
Maintenance of underlying connective tissue
Diminished edema and erythema

A 58-year-old woman has had a cough with fever for 3 days. A chest radiograph reveals infiltrates in the right
lowerlobe. A sputum culture grows Streptococcus pneumoniae. The clearance of these organisms from the
lungparenchyma would be most effectively accomplished through generation of which of the following substances
by themajor inflammatory cell type responding to this infection?
Platelet activating factor
Prostaglandin E2
Kallikrein
Hydrogen peroxide

A clinical study is performed of patients with pharyngeal infections. The most typical clinical course averages 3
daysfrom the time of onset until the patient sees the physician. Most of these patients experience fever and chills.
Onphysical examination, the most common findings include swelling, erythema, and pharyngeal purulent
exudate.Which of the following types of inflammation did these patients most likely have?
Granulomatous
Acute
Gangrenous
Resolving

A 56-year-old man has had increasing dyspnea for 6 years. He has no cough or fever. He had chronic exposure
toinhalation of silica dust for many years in his job. A chest x-ray now shows increased interstitial markings
anparenchymal 1 to 3 cm solid nodules. His pulmonary problems are most likely to be mediated through which of
thefollowing inflammatory processes?
Neutrophilic infiltrates producing leukotrienes
Foreign body giant cell formation
Plasma cell synthesis of immunoglobulins
Macrophage elaboration of cytokines

A 22-year-old woman has premature labor with premature rupture of fetal membranes at 20 weeks gestation. Prior
tothat time, the pregnancy had been proceeding normally. A stillbirth occurs two days later. Microscopic
examinationof the normal-sized placenta reveals numerous neutrophils in the amnion and chorion, but no villitis.
The prematurelabor was most likely mediated by the effects from release of which of the following substances?
Immunoglobulin
Prostaglandin
Complement
Fibrinogen
After two weeks in the hospital following a fall in which she incurred a fracture of her left femoral trochanter, a 76-
year-old woman now has a left leg that is swollen, particularly her lower leg below the knee. She experiences painon
movement of this leg, and there is tenderness to palpation. Which of the following complications is most likely
tooccur next after these events?
Gangrenous necrosis of the foot
Hematoma of the thigh
Disseminated intravascular coagulation
Pulmonary thromboembolism

A 43-year-old woman has had a chronic cough with fever and weight loss for the past month. A chest
radiographreveals multiple nodules from 1 to 4 cm in size, some of which demonstrate cavitation in the upper lobes.
A sputumsample reveals the presence of acid fast bacilli. Which of the following cells is the most important in
thedevelopment her lung lesions?
Macrophage
Fibroblast
Neutrophil
Mast cell

A 20-year-old man has experienced painful urination for 4 days. A urethritis is suspected, and Neisseria gonorrheaeis
cultured. Numerous neutrophils are present in a smear of the exudate from the penile urethra. These
neutrophilsundergo diapedesis to reach the organisms. Release of which of the following chemical mediators is most
likely todrive neutrophil exudation?
Histamine
Prostaglandin
Hageman factor
Complement

An episode of marked chest pain lasting 4 hours brings a 51-year-old man to the emergency room. He is found
tohave an elevated serum creatine kinase. An angiogram reveals a complete blockage of the left circumflex artery
2cm from its origin. Which of the following substances would you most expect to be elaborated around the region
oftissue damage in the next 3 days as an initial response to promote healing?
Histamine
Immunogloblulin G
Complement component C3b
Vascular endothelial growth factor

A 94-year-old woman has developed a fever and cough over the past 2 days. Staphylococcus aureus is cultured
fromher sputum. She receives a course of antibiotic therapy. Two weeks later she no longer has a productive cough,
butshe still has a fever. A chest radiograph reveals a 3 cm rounded density in the right lower lobe whose
liquefiedcontents form a central air-fluid level. There are no surrounding infiltrates. Which of the following is the
bestdescription for this outcome of her pneumonia?
Hypertrophic scar
Abscess formation
Regeneration
Bronchogenic carcinoma

A 36-year-old woman has been taking acetylsalicylic acid (aspirin) for arthritis for the past 4 years. Her joint pain
istemporarily reduced via this therapy. However, she now has occult blood identified in her stool. Which of
thefollowing substances is most likely inhibited by aspirin to cause this complication?
Leukotriene B4
Interleukin-1
Thromboxane
Bradykinin

A small sliver of wood becomes embedded in the finger of a 25-year-old man. He does not remove it, and over then
next 3 days the area around the sliver becomes red, swollen, and tender. Neutrophils migrate into the injured
tissue.Expression of which of the following substances on endothelial cells is most instrumental in promoting
thisinflammatory reaction?
Interferon gamma
Hageman factor
Lysozyme
E-selectin

An inflammatory process that has continued for 3 months includes the transformation of tissue macrophages
toepithelioid cells. There are also lymphocytes present. Over time, fibroblasts lay down collagen as the focus
ofinflammation heals. These events are most likely to occur as an inflammatory response to which of the
followinginfectious agents?
Mycobacterium tuberculosis
Pseudomonas aeruginosa
Cytomegalovirus
Giardia lamblia

A 37-year-old man has had nausea and vomiting for 5 weeks. He experienced an episode of hematemesis
yesterday.On physical examination he has no abnormal findings. Upper GI endoscopy is performed, and there is a
1.5 c diameter lesion in the gastric antrum with loss of the epithelial surface. These findings are most typical for
which ofthe following pathologic processes?
Abscess
Serositis
Granuloma
Ulcer

A 17-year-old truck driver is involved in a collision. He incurs blunt force abdominal trauma. In response to thisinjury,
cells in tissues of the abdomen are stimulated to enter the G1 phase of the cell cycle from the G0 phase.Which of the
following cell types is most likely to remain in G0 following this injury?
Smooth muscle
Endothelium
Skeletal muscle
Fibroblast

A 19-year-old woman who works indoors spends a day outside gardening. She does not wear a hat or sunscreen.That
evening her partner remarks that her face appears red. Which of the following dermal changes most likelyaccounts
for her red appearance?
Neutrophil aggregation
Hemorrhage
Edema
Vasodilation

A 45-year-old woman has had a chronic, non-productive cough for 3 months, along with intermittent fever. She hasa
chest radiograph that reveals multiple small parenchymal nodules along with hilar and cervical lymphadenopathy.A
cervical lymph node biopsy is performed. Microscopic examination of the biopsy shows noncaseatinggranulomatous
inflammation. Cultures for bacterial, fungal, and mycobacterial organisms are negative. Which of thefollowing
chemical mediators is most important in the development of her inflammatory response?
Interferon gamma
Bradykinin
Complement C5a
Histamine

A 55-year-old man has a history of hypercholesterolemia with coronary artery disease and suffered a
myocardialinfarction 2 years ago. He now presents with crushing substernal chest pain. Which of the following
laboratory testsis most useful in diagnosing the cause of his chest pain?
Increased white blood cell count
Elevated sedimentation rate
Decreased serum complement
Increased serum troponin

A 15-year-old girl has had episodes of sneezing with watery eyes and runny nose for the past 2 weeks. On
physicalexamination she has red, swollen nasal mucosal surfaces. She has had similar episodes each Spring and
Summerwhen the amount of pollen in the air is high. Her symptoms are most likely to be mediated by the release of
which ofthe following chemical mediators?
Complement C3b
Platelet activating factor (PAF)
Tumor necrosis factor (TNF)
Histamine

A 45-year-old man has been working hard all day long carrying loads of bricks to build a wall. He takes a
nonsteroidalanti-inflammatory drug (ibuprofen). Which of the following processes is this drug most likely to
diminishin his arms?
Thrombosis
Pain
Necrosis
Fibrinolysis

Within minutes following a bee sting, a 37-year-old man develops marked respiratory stridor with dyspnea
andwheezing. He also develops swelling and erythema seen in his arms and legs. An injection of epinephrine helps
toreverse these events and he recovers within minutes. Which of the following chemical mediators is most
importantin the pathogenesis of this man's condition?
Bradykinin
Complement C5a
Nitric oxide
Histamine

A 72-year-old woman did not get a 'flu' shot in the fall as recommended for older persons. In the wintertime,
shebecame ill, as many people in her community did, with a respiratory illness that lasted for 3 weeks. During
thisillness, she had a fever with a non-productive cough, mild chest pain, myalgias, and headache. What was her
chestradiograph most likely to have shown during this illness?
Hilar mass
Interstitial infiltrates
Hilar lymphadenopathy
Lobar consolidation

In an experiment, Enterobacter cloacae organisms are added to a solution containing leukocytes and blood
plasma.Engulfment and phagocytosis of the microbes is observed to occur. Next a substance is added which
enhancesengulfment, and more bacteria are destroyed. Which of the following substances in the plasma is most
likely toproduce this effect?
Complement C3b
Glutathione peroxidase
Immunoglobulin M
P-selectin

A 43-year-old woman has had nausea with vomiting persisting for the past 5 weeks. On physical examination
thereare no abnormal findings. She undergoes an upper GI endoscopy and gastric biopsies are obtained. The
microscopicappearance of these biopsies shows mucosal infiltration by lymphocytes, macrophages, and plasma cells.
Which ofthe following most likely caused her findings?
Staphylococcus aureus septicemia
Ingestion of chili peppers
Diabetes mellitus
Infection with Helicobacter pylori

In an experiment, lymphatic channels are observed in normal soft tissue preparations. Staphylococcus
aureusorganisms are innoculated into the tissues and the immunologic response observed over the next 24 hours.
Which ofthe following functions is most likely to be served by these lymphatics to produce a specific immune
response tothese organisms?
Carry lymphocytes to peripheral tissue sites
Remove extravascular tissue fluid
Transport antigen presenting cells
Serve as a route for dissemination of infection

In an experiment, surgical wound sites are observed following suturing. An ingrowth of new capillaries is observedto
occur within the first week. A substance elaborated by macrophages is found at the wound site to stimulate
thiscapillary proliferation. Which of the following substances is most likely to have this function?
Platelet-derived growth factor
Phospholipase C-gamma
Fibronectin
Fibroblast growth factor

A 55-year-old man with a history of ischemic heart disease has worsening congestive heart failure. He has
notedincreasing dyspnea and orthopnea for the past 2 months. On physical examination there is dullness to
percussion atlung bases. A chest x-ray shows bilateral pleural effusions. A left thoracentesis is performed, and 500
mL of fluid isobtained. Which of the following characteristics of this fluid would most likely indicate that it is a
transudate?
Cloudy appearance
High protein content
<3 lymphocytes/microliter
Presence of fibrin

In a clinical study, patients undergoing laparoscopic cholecystectomy are followed to document the post-
surgicalwound healing process. The small incisions are closed with sutures. Over the 4 weeks following surgery, the
woundsare observed to regain tensile strength and there is re-epithelialization. Of the following substances, which is
mostlikely found to function intracellularly in cells involved in this wound healing process?
Fibronectin
Laminin
Tyrosine kinase
Hyaluronic acid

A 31-year-old woman has a laparotomy performed for removal of an ovarian cyst. She recovers uneventfully, with no
complications. At the time of surgery, a 12 cm long midline abdominal incision was made. The tensile strength inthe
surgical scar will increase so her normal activities can be resumed. Most of the tensile strength will likely beachieved
in which of the following time periods?
One week
One month
Three months
Six months
A 9-year-old girl sustains a small 0.5 cm long laceration to her right index finger while playing 'Queen of Swords'with
a letter opener. Which of the following substances, on contact with injured vascular basement membrane,activates
both the coagulation sequence and the kinin system as an initial response to this injury?
Thromboxane
Plasmin
Platelet activating factor
Hageman factor

A 65-year-old woman has had a fever for the past day. On physical examination her temperature is 39°C and
bloodpressure 90/50 mm Hg with heart rate of 106/minute. Laboratory studies show a WBC count of
12,510/microliterand WBC differential count of 78 segs, 8 bands, 11 lymphs, and 3 monos. A blood culture is positive
for Escherichiacoli. Her central venous pressure falls markedly. She goes into hypovolemic shock as a result of the
widespreadinappropriate release of a chemical mediator derived from macrophages. She develops multiple organ
failure. Whichof the following mediators is most likely to produce these findings?
Nitric oxide
Bradykinin
Histamine
Prostacyclin

A 20-year-old woman sustains an injury to her right calf in a mountain biking accident. On physical examination
shehas a 5 cm long laceration on the right lateral aspect of her lower leg. This wound is closed with sutures.
Woundhealing proceeds over the next week. Which of the following factors will be most likely to aid and not inhibit
woun healing in this patient?
Commensal bacteria
Decreased tissue perfusion
Presence of sutures
Corticosteroid therapy

A 24-year-old primigravida is late in the second trimester of pregnancy. She experiences the sudden onset of
somecramping lower abdominal pain. This is immediately followed by passage of some fluid per vagina along with
afoul-smelling discharge. The fetus is stillborn two days later. Examination of the placenta demonstrates
extensiveneutrophilic infiltrates in the chorion and amnion. Which of the following organisms is most likely to be
responsiblefor these findings?
Mycobacterium tuberculosis
Herpes simplex virus
Escherichia coli
Treponema pallidum

A 19-year-old man incurs a stab wound to the chest. The wound is treated in the emergency room. Two months
laterthere is a firm, 3 x 2 cm nodular mass with intact overlying epithelium in the region of the wound. On
examinationthe scar is firm, but not tender, with no erythema. This mass is excised and microscopically shows
fibroblasts withabundant collagen. Which of the following mechanisms has most likely produced this series of
events?
Keloid formation
Development of a fibrosarcoma
Poor wound healing from diabetes mellitus
Foreign body response from suturing

A 45-year-old man has had a fever and dry cough for 3 days, and now has difficulty breathing and a coughproductive
of sputum. On physical examination his temperature is 38.5°C. Diffuse rales are auscultated over lowerlung fields. A
chest radiograph shows a right pleural effusion. A right thoracentesis is performed. The fluid obtainedhas a cloud
appearance with a cell count showing 15,500 leukocytes per microliter, 98% of which are neutrophils.Which of the
following terms best describes his pleural process?
Serous inflammation
Purulent inflammation
Fibrinous inflammation
Chronic inflammation

A 52-year-old woman with no major medical problems takes a long airplane flight across the Pacific Ocean.
Uponarrival at Sydney's Kingsford Smith airport following the flight from Los Angeles, she cannot put her shoes back
on There is no pain or tenderness. Which of the following is the most likely explanation for this phenomenon?
Activation of Hageman factor has led to bradykinin production.
A lot of drinks were served in the first class section.
Femoral vein thrombosis developed
Venous hydrostatic pressure became increased.

In an experiment, a lung tissue preparation is exposed to Mycobacterium tuberculosis organisms. Over the
nextweek, it is observed that granulomas form in the lung. Within the granuloma are found inflammatory
cellsexpressing class II MHC antigens. These cells elaborate cytokines that promote fibroblastic production of
collagenwithin the granulomas. From which of the following peripheral blood leukocytes are these cells bearing class
IIantigen most likely to be derived?
Neutrophils
B cells
Monocytes
NK cells

A 56-year-old man has had increasing difficulty breathing for the past week. On physical examination he is
afebrile.Auscultation of his chest reveals diminished breath sounds and dullness to percussion bilaterally. There is 2+
pittingedema present to the level of his thighs. A chest radiograph reveals bilateral pleural effusions. Which of
thefollowing laboratory test findings is he most likely to have?
Hypoalbuminemia
Glucosuria
Neutrophilia
Anemia
A 72-year-old man presents with a 3-day history of progressively worsening productive cough, fever, chills, andsigns
of toxicity. Prominent physical findings include signs of consolidation and rales over the right lung base.Sputum
culture is positive for Streptococcus pneumoniae. An intra-alveolar exudate filling the alveoli of theinvolved portion
of the lung is present. Which of the following types of inflammatory cells is most likely aprominent feature of this
exudate?
Basophils
Eosinophils
Lymphocytes
Neutrophils

A routine complete blood count performed on a 22-year-old medical student reveals an abnormality in
thedifferential leukocyte count. She has been complaining of frequent sneezing and “watery” eyes during the
pastseveral weeks and reports that she frequently had such episodes in the spring and summer. Which of the
followingcell types is most likely to be increased?
Basophils
Eosinophils
Lymphocytes
Monocytes

A 16-year-old boy presents with a 24-hour history of severe abdominal pain, nausea, vomiting, and low-grade
fever.The pain is initially periumbilical in location but has migrated to the right lower quadrant of the abdomen,
withmaximal tenderness elicited at a site one-third of the way between the crest of the ileum and the
umbilicus(McBurney point). The leukocyte count is 14,000/mm3, with 74% segmented neutrophils and 12% bands.
Surgery isperformed. Which of the following describes the expected findings at the affected site?
Fistula (abnormal duct or passage) connecting to the abdominal wall
Granulation tissue (new vessels and young fibroblasts) with a prominent infiltrate of eosinophils
Granulomatous inflammation with prominent aggregates of epithelioid cells and multinucleated giant cells
Prominent areas of edema, congestion, and a purulent reaction with localized areas of abscess formation

A 2-year-old boy presents with recurrent infections involving multiple organ systems. Extensive investigation
resultsin a diagnosis of chronic granulomatous disease of childhood. Which of the following most closely
characterizes theabnormality in this patient’s phagocytic cells?
Decreased killing of microorganisms because of enhanced production of hydrogen peroxide
Deficiency of NADPH oxidase activity
Impaired chemotaxis and migration caused by abnormal microtubule formation
Inability to kill streptococci

A laboratory experiment is performed to evaluate the chemotactic potential of a group of potential mediators.
Whichof the following substances most likely has the greatest affinity for neutrophils?
C5a
Fucosyl transferase
β2-Integrin
P-selectin
A 26-year-old African-American woman has bilateral hilar adenopathy, and radiography reveals multiple reticular
densities in both lung fields. A bronchoscopic biopsy reveals granulomatous inflammation with multiple giant cellsof
the Langhans type and no evidence of caseous necrosis. Which of the following is the most likely diagnosis?

Aspergillosis
Coccidioidomycosis
Histoplasmosis
Sarcoidosis

In a laboratory exercise for medical students, an unknown compound is studied. The students are informed that
thecompound has been isolated from endothelial cells and that its synthesis can be inhibited by aspirin. In
thelaboratory, the students demonstrate that the compound is a potent vasodilator and platelet antiaggregant.
Giventhese findings, the substance is most likely which of the following mediators?
5-HPETE
LTC4
LXA4
PGI2

A 70-year-old man presents with the sudden onset of left-sided weakness, spasticity, and hyperactive and
pathologicreflexes. The most serious consequences of this disorder are the result of damage to which of the
following celltypes?
Labile cells
Multipotent adult progenitor cells
Permanent cells
Stable cells

Inflammation of stomach is called:


Gastritis.
Enteritis.
Colitis.
Proctitis.

Inflammation of liver is called:


Gastritis.
Hepatitis
Colitis.
Proctitis.

Inflammation of kidney is called:


Gastritis.
Hepatitis.
Nephritis.
Proctitis.
Inflammation of small intestine is called:
Gastritis.
Enteritis.
Colitis.
Proctitis.

Inflammation of large intestine is called:


Gastritis.
Enteritis.
Colitis.
Proctitis.

Inflammation of rectum is called:


Gastritis.
Enteritis.
Colitis.
Proctitis.

Inflammation of skin is called:


Gastritis.
Hepatitis.
Nephritis.
Dermatitis.

Inflammation of blood vessels is called:


Gastritis.
Hepatitis.
Nephritis.
Vasculitis.

Inflammation of lung tissue is called:


Pneumonia.
Angina.
Furuncle.
Carbuncle.

Inflammation of throat is called:


Pneumonia.
Angina.
Furuncle.
Carbuncle.

Inflammation of hair follicle is called:


Pneumonia.
Angina.
Furuncle.
Carbuncle.

Inflammation of brain is called:


Gastritis.
Hepatitis.
Enteritis.
Encephalitis.

Local sign of inflammation “rubor” means:


Heat.
Pain.
Redness.
Swelling.

Local sign of inflammation “calor” means:


Heat
Pain.
Redness.
Swelling.

Local sign of inflammation “dolor” means:


Heat.
Pain.
Redness.
Swelling.

Local sign of inflammation “tumor” means:


Heat.
Pain.
Redness.
Swelling.

Local sign of inflammation which associated with pressure on nerve endings by exudate is:
Rubor.
Calor.
Dolor.
Tumor.

Local sign of inflammation which associated with accumulation of exudate is:


Rubor.
Calor.
Dolor.
Tumor.

Calor as a local sign of inflammation is associated with:


Arterial hyperemia.
Pressure of nerve endings.
Exudation
Interference with enzyme function

Tumor as a local sign of inflammation is associated with:


Arterial hyperemia.
Pressure of nerve endings.
Exudation.
Interference with enzyme function.

Dolor as a local sign of inflammation is associated with:


Arterial hyperemia.
Pressure of nerve endings.
Exudation.
Local pain.

Rubor as a local sign of inflammation is associated with:


Arterial hyperemia.
Pressure of nerve endings.
Exudation.
Interference with enzyme function.

Component of alteratio stage of inflammation is:


Necrosis.
Arterial hyperemia.
Phagocytosis.
Regeneration of epithelial cells.
Component of alteratio stage of inflammation is :
Degeneration of cells.
Arterial hyperemia.
Phagocytosis.
Regeneration of epithelial cells

Component of exudation stage of inflammation is:


Degeneration of cells.
Arterial hyperemia.
Necrosis.
Regeneration of epithelial cells.

Component of exudation stage of inflammation is:


Degeneration of cells
Necrosis.
Formation of cell infiltrate
Regeneration of epithelial cells.

Component of exudation stage of inflammation is:


Degeneration of cells.
Necrosis.
Regeneration of epithelial cells.
Phagocytosis.

Component of exudation stage of inflammation is:


Degeneration of cells.
Necrosis.
Pinocytosis
Regeneration of epithelial cells.

Component of exudation stage of inflammation is:


Degeneration of cells.
Necrosis.
Regeneration of epithelial cells
Leakage of blood plasma from vessels.

Component of exudation stage of inflammation is:


Degeneration of cells.
Necrosis.
Regeneration of epithelial cells.
Emigration of blood cells from vessels.
Component of proliferation stage of inflammation is:
Degeneration of cells.
Necrosis.
Regeneration of epithelial cells.
Emigration of blood cells from vessels

Component of proliferation stage of inflammation is :


Degeneration of cells.
Necrosis.
Regeneration of fibroblasts.
Emigration of blood cells from vessels.

Type of acute inflammation which associated with formation of protein-poor fluid is called:
Serous.
Fibrinous.
Purulent
Hemorrhagic.

Type of acute inflammation which associated with formation of hair-like deposits is called:
Serous.
Fibrinous.
Purulent.
Hemorrhagic.

Type of acute inflammation which associated with formation of pseudomembrane on mucosal surfaces is called:
Serous.
Fibrinous.
Purulent.
Hemorrhagic.

Type of acute inflammation which associated with formation of thick turbid yellow-green fluid is called:
Serous.
Fibrinous.
Purulent.
Catarrhal.

Which type of pneumonia acute is associated with formation of fibrinous exudate:


Lobar pneumonia.
Bronchpneumonia.
Interstitial pneumonia.
Septic pneumonia.

Localized type of purulent inflammation is called:


Phlegmon.
Empyema.
Abscess.
Furuncle.

Diffuse type of purulent inflammation is called:


Phlegmon.
Empyema.
Abscess.
Furuncle.

Type of purulent inflammation with accumulation of pus in serosal cavities and hollow organs is called:
Phlegmon.
Empyema.
Abscess.
Furuncle.

Type of purulent inflammation with accumulation of pus in serosal cavities and hollow organs is called :
Phlegmon.
Empyema.
Abscess.
Furuncle.

Purulent inflammation of hair follicle is called:


Phlegmon.
Empyema.
Abscess.
Furuncle.

Which of the following are thought to mediate, many of the systemic effects of inflammation are chemotactic
andstimulate adhesion molecules:
Interleukin-1 (IL-1) and tumor necrosis factor
C5 a and leukotriene B-4
C3 b.
Leukotriene C4, D4 and E4.
After initiation of an acute inflammatory process third in a sequence of changes in vascular flow is:
Vasoconstriction.
Redness.
Leukocytic migration.
Vasodilation

The term “croupous” inflammation is synonymous with


Catarrhal inflammation
Fibrinous inflammation
Sero-fibrinous inflammation
Suppurative inflammation

Vascular phenomenon in acute inflammation helps in:


Bringing the defensive cells like neutrophils from interior of the vessels to the site of irritation
Minimizing the effect of irritant by diluting it considerably
Limiting the inflammatory reaction to the smallest possible area
All of these

Which of the complement components act as chemokines?


C3b
C4b
C5a
C4a

All are types of tissue macrophages, except:


Littoral cells
Hoffbauer cells
Osteoclasts
Osteoblasts

Main cytokines acting as mediators of inflammation are as under, except:


Interleukin-1 (IL-1)
Tumor necrosis factor α (TNF-α)
Nitric oxide (NO)
Interferon –y (IF-y)
MCQ – Introduction to Pathologic Anatomy (25 q)

1.Cause of disease is:

A. Etiology.

B. Pathogenesis.

C. Morphogenesis.

D. Pathomorphism.

E. Thanatogenesis.

2.Mechanism of disease is:

A. Etiology.

B. Pathogenesis.

C. Morphogenesis.

D. Pathomorphism.

E. Thanatogenesis.

3.Morphologic stages of disease is:

A. Etiology.

B. Pathogenesis.

C. Morphogenesis.

D. Pathomorphism.

E. Thanatogenesis.

4.Changes in course and manifestations of disease is:

A. Etiology.

B. Pathogenesis.

C. Morphogenesis.

D. Pathomorphism.

E. Thanatogenesis.
5.Mechanism of death is:

A. Etiology.

B. Pathogenesis.

C. Morphogenesis.

D. Pathomorphism.

E. Thanatogenesis.

6.Pathology of treatment is:

A. Etiology.

B. Pathogenesis.

C. Morphogenesis.

D. Iatrogeny.

E. Thanatogenesis.

7.Sign of biologic death which associated with decrease of body temperature is called:

A. Cooling.

B. Rigor Mortis.

C. Drying.

D. Cadaveric spots.

E. Cadaveric decomposition.

8.Sign of biologic death which associated with hardning of muscles is called:

A. Cooling.

B. Rigor mortis.

C .Drying.

D. Cadaveric spots.

E. Cadaveric decomposition.

9.Sign of biologic death which associated with evaporation of fluid is called:


A. Cooling.

B. Rigor mortis.

C. Drying.

D. Cadaveric spots.

E. Cadaveric decomposition.

10.Sign of biologic death which associated with enzymatic destruction of tissues is called:

A. Cooling.

B. Rigor mortis.

C. Drying.

D. Cadaveric spots.

E. Cadaveric decomposition.

11.Sign of clinical death is:

A. Cooling.

B. Rigor mortis.

C. Stop of breath.

D. Drying.

E. Cadaveric spots.

12.Sign of clinical death is:

A. Cooling.

B. Stop of blood circulation.

C. Rigor mortis.

D. Drying.

E. Cadaveric spots.

13. The concept of clinicopathologic correlation (CPC) by study of morbid anatomy was introduced by:

A. Hippocrates

B. Virchow
C. John Hunter

D. Morgagni

E. Laennec

14. The first ever museum of pathologic anatomy was developed by:

A. John Hunter

B. Rokitansky

C. Rudolf Virchow

D. Morgagni

E. Hippocrates

15. An ABO human blood group system was first described by:

A. Edward Jenner

B. Karl Landsteiner

C. Hippocrates

D. Laennec

E. Leeuwenhoek

16. Frozen section was first introduced by:

A. Cohnheim

B. Ackerman

C. Virchow

D. Feulgen

E. Laennec

17. Electron microscope was first developed by:

A. Barbara mcclintock

B. Watson and crick

C. Tijo and levan

D. Ruska and lorries


E. Nowell and hagerford

18. Structure of DNA of the cell was described by:

A. Watson and crick

B. Tijo and levan

C. Ruska and lorries

D. Barbara mcclintock

E. Nowell and hagerford

19. Flexibilty and dynamism of DNA was invented by:

A. Watson and crick

B. Tijo and levan

C. Ruska and lorries

D. Barbara mcclintock

E. Nowell and hagerford

20. Father of cellular pathology is:

A. Karl rokitansky

B. Rudolf virchow

C. G. Morgagni

D. Ft schwann

E. Barbara Mcclintock

21. Human’s genome consists of following number of genes:

A. 20,000

B. 30,000

C. 50,000

D. 70,000

E. 100,000
22. Stem cell research consists of:

A. Human cells grown in vitro

B. Plant cells grown in vitro

C. Cadaver cells grown in vitro

D. Synonymous with pcr (polymerase chain reaction)

E. Human cells grown in vivo

23. PCR (polymerase chain reaction) technique was introduced by:

A. Ian wilmut

B. Watson

C. Nowell hagerford

D. Kary mullis

E. Barbara mcclintock

24. Human genome project was completed in:

A. 2001

B. 2002

C. 2003

D. 2004

E. 2005

25. A postmortem clot is most likely to

A. Grossly display features of recanalization

B. Grossly have lines of zahn

C. Grossly have the appearance of «chicken fat» overlying «currant jelly»

D. Microscopically appear attached to the blood vessel

E. Microscopically have alternating layers of cells and platelets

Hemodynamic disorders – I (Kozmina 52 q)


The essential difference between plasma and interstitial fluid compartment is:
Glucose is higher in the former
Urea is higher in the former
Protein content is higher in the former
Potassium is higher in the former
Bicarbonate is higher in the former

2. Osmotic pressure exerted by the chemical constituents of the body fluids has the following features except:

Crystalloid osmotic pressure comprises minor portion of total osmotic pressure


Oncotic pressure constitutes minor portion of total osmotic pressure
Oncotic pressure of plasma is higher
Oncotic pressure of interstitial fluid is lower

3. For causation of oedema by decreased osmotic pressure, the following factor is most important:

Fall in albumin as well as globulin


Fall in globulin level
Fall in albumin level
Fall in fibrinogen level

4. Transsudate differs from exudates in having the following except:

No inflammatory cells
Low glucose content
Low protein content
Low specific gravity

5. Nephritic oedema differs from nephrotic oedema in having the following except:

Mild oedema
Distributed on face, eyes
Heavy proteinuria
Occurs in acute glomerulonephritis

6. The following type of oedema is characteristically dependent oedema:

Nephrotic oedema
Nephritic oedema
Pulmonary oedema
Cardiac oedema

7. Pulmonary oedema appears due to elevated pulmonary hydrostatic pressure when the fluid accumulation is:

Two fold
Four fold
Eight fold
Ten fold
Six fold

8. Active hyperemia is result of:


Dilatation of capillaries
Dilatation of arterioles
Venous engorgement
Lymphatic obstruction
Venous obstruction

9. Sectioned surface of lung shows brown induration in:

Pulmonary embolism
Pulmonary hemorrhage
Pulmonary infarction
CVC lung
Pneumonia

10. Histologic sections of lung tissue from a 68-year-old female with congestive heart failure and progressive
breathing problems reveal numerous hemosiderin-laden cells within the alveoli. Theses “heart failure cells” originate
from alveolar

Endothelial cells
Eosinophils
Lymphocytes
Macrophages
Pneumocytes

11. Which one of the listed changes correctly describes the pathophysiology involved in the production of pulmonary
oedema in patients with congestive heart failure?

Decreased plasma oncotic pressure


Endothelial damage
Increased hydrostatic pressure
Increased vascular permeability
Lymphatic obstruction

12. Which one of the listed clinical scenarios best illustrates the concept of active hyperemia?

A 22-year-old second-year medical student who develops a red face after being asked a question during a lecture
A 37-year-old male who develops massive swelling of the scrotum due to infection with Wuchereria bancrofti
A 69-year-old male who dies secondary to progressive heart failure and at autopsy is found to have a “nutmeg” liver
A 6-year-old boy who develops the sudden onset of intense scrotal pain due to testicular torsion
A 71-year-old female who develops perifollicular hemorrhages due to a deficiency of vitamin C.

13. The local vacant hyperemia occurs due to –

Compression of arteries by tumor


Decreasing of barometric pressure
Obturation of arteries by thrombus
Irritation of vasorelaxant nerves
Increasing of barometric pressure

14. Hemorrhage into the abdominal cavity –

Hemopericardium
Hemothorax
Metrorrhagia
Hematomesis
Hemoperitoneum

15. Local angioneurotic hyperemia occurs due to –

Decreasing of barometric pressure


Increasing of barometric pressure
Compression of arteries by tumor
Obturation of arteries by thrombus
Irritation of vasorelaxant nerves

16. One of the causes of general venous hyperemia, except:

Congenital valvular heart disease


Acquired valvular heart disease
Acute myocardial infarction
Fibroelastosis of endocardium
Cardiomyopathy

17. The most important cause of oedema in nephrotic syndrome is –

Hypoproteinaemia
Increased capillary permeability
increased hydrostatic pressure of blood
Lymphatic obstruction
Hyperkaliemia

18. Brown induration is a type of chronic venous congestion occurs in

Liver
Heart
Lung
Kidney
Spleen

19. Hemorrhage into pleural cavity is –

Hemopericardium
Hemothorax
Metrorrhagia
Hematomesis
Hemoperitoneum

20. Hemorrhage into the pericardial cavity is –

Hemopericardium
Hemothorax
Metrorrhagia
Hematomesis
Hemoperitoneum

21. Collateral hyperemia occurs due to

Inflammation
Irritation of vasorelaxant nerves
Decrease of barometric pressure
Overload of physical work
Difficulty of blood circulation in the main artery

22. The pathogenesis of local venous hyperemia

Difficulty of venous outflow


Difficulty of arterial inflow
Increase of inflow and outflow
Decrease of inflow and outflow
Increase of arterial inflow

23. Morphologic changes in the lungs due to chronic venous hyperemia –

Edema
Congestive hyperemia
Congestive induration
Brown induration
Nutmeg cirrhosis

24. Minute 1 – 2 mm hemorrhages are –

Hematoma
Hemorrhagic infiltration
Petechiae
Ecchymosis
Hemopericard

25. Hemorrhage is –

Accumulation of blood in serous cavities


Escape of blood from blood vessels or heart cavities to the environment, body cavities, tissues or organs
Accumulation of blood in the tissues with their rupture
Accumulation of blood in the tissues without their rupture
Accumulation of blood in the organs

26. Fluid collections in different body cavities depending on the site are all of the following, except:

Hydrothorax
Hydropericardium
Hydroperitoneum
Hydrocephalus
Hydrocele
27. A severe and generalized edema with profound subcutaneous tissue swelling is called:

Anasarca
Ascites
Hydrothorax
Hydropericardium
Hydrocele

28. Pathophysiologic categories of edema are all of the following, except:

Inflammation
Sodium retention
Calcium retention
Hypoproteinemia
Increased hydrostatic pressure

29. Conditions resulting in edema due to increased hydrostatic pressure are all of the following, except:

Congestive heart failure


Inflammation
Ischemic heart disease
Systemic hypertension
Venous obstruction or compression

30. The serum protein responsible for maintaining colloid osmotic pressure is called:

Fibrinogen
Albumin
Globulin
Plasminogen
Prothrombin

31. Edema resulting from the congestive heart failure develops in all of the following, except:

Lung
Brain
Liver
Lower extremities
Upper extremities

32. Generalized edema as a result of renal dysfunction or nephrotic syndrome is localized in:

Periorbital tissue
All parts of the body
Lung
Brain
Liver

33. Pulmonary edema commonly results from all of the following, except:
Left ventricular failure
Renal failure
Systemic hypertension
Appendicitis
Myocardial infarction

34. The following pathologic processes commonly occur together, except:

Congestion and edema


Capillary rupture and hemorrhage
Breakdown of the red cells and hemosiderosis
Active hyperemia and anemia
Venous obstruction and cyanosis

35. Microscopically, acute pulmonary congestion is characterized by all of the following, except:

Engorged alveolar capillaries


Thickened and fibrotic alveolar septa
Edema fluid in alveoli
Focal intra-alveolar hemorrhage
Foci of atelectasis

36. Microscopically, chronic pulmonary congestion is characterized by all of the following, except:

Engorged alveolar capillaries


Thickened and fibrotic alveolar septa
Focal intra-alveolar hemorrhage
Numerous “heart failure cells” in lung tissue
Abscesses in lung tissue

37. Microscopically, acute hepatic congestion is characterized by all of the following, except:

Engorged central veins


Engorged central sinusoids
Destroyed central hepatocyles
Unchanged periportal sinusoids
Diminished periportal hepatocytes

38. Microscopically, chronic passive congestion of the liver is characterized by all of the following, except:

Centrolobular necrosis of hepatocytes


Centrolobular hemorrhages
Engorged periportal sinusoids
Capillarization of sinusoids
Fatty changes of periportal hepatocytes

39. Rupture of a large artery or vein is commonly due to all of the following, except:

Trauma
Atherosclerosis
Inflammatory erosion
Neoplastic erosion
Increased permeability

40. Hemorrhages enclosed within a tissue with its destruction are referred to as:

Hematomas
Petechiae
Purpura
Ecchymoses
Hemothorax

41. Minute (1- to 2-mm) hemorrhages into skin, mucous membranes or serosal surfaces are called:

Hematomas
Petechiae
Purpura
Ecchymoses
Hemothorax

42. Small (0.3 to 1.0 cm) hemorrhages into skin, mucous membranes or serosal surfaces are called:

Hematomas
Petechiae
Purpura
Ecchymoses
Hemothorax

43. Large (>1 to 2 cm) subcutaneous hemorrhages are called:

Hematomas
Petechiae
Purpura
Ecchymoses
Hemothorax

44. Large accumulations of blood in one or another of the body cavities are termed as all of the following, except:

Hemothorax
Hemopericardium
Hemosiderosis
Hemoperitoncum
Hemarthrosis

45. The clinical significance of hemorrhage depends on all of the following, except:

Blood group of individual


Volume of blood loss
Rate of blood loss
The site of hemorrhage
The diameter of damaged vessel

46. The most common underlying cause of primary brain parenchymal hemorrhage is which of the following:

Systemic coagulation disorders


Vasculitis
Systemic hypertension
Neoplasms
Vascular malformations

47. Macroscopically, brain hemorrhage is characterized by all of the following, except:

Local extravasation of blood


Local damage of brain tissue
Narrowed cerebral suici
Distended cerebral gyri
Cavitary destruction of brain

48. Macroscopically, chronic passive congestion of the liver is characterized by all of the following, except:

Hemorrhagic and wet cut surface


Red-brown central areas of the hepatic lobules
Goose liver
Tan periportal areas (fatty changes)
Nutmeg liver

49. Microscopically, brain hemorrhage is characterized by all of the following, except:

Fatty changes of damaged neurons


Central core of clotted blood
Edema of adjacent brain tissue
Dystrophy of survived neurons
Reactive proliferation of astrocytes

50. The markers of myocardial ischemia with irreversible injured myofibers are which of the following:

Cytoplasmic fatty change


Glycogen depletion of the cytoplasm
Cellular swelling
Clumping of nuclear chromatin
Contraction bands in the cytoplasm

51. Pathologic lesions resulting from passive congestion include all of the following, except:

Nutmeg liver
Brown induration of the lungs
Cyanotic induration of the spleen
Strawberry gallbladder
Stasis dermatitis of the legs
52. Ischemic injury in the central nervous system results in:

Liquefaclive necrosis
Coagulalive necrosis
Caseous necrosis
Gangrenous necrosis
Fat necrosis

EDEMA. HYPEREMIA. HEMORRHAGE (Davydov – 40 q )

1.Accumulation of fluid in the pleural cavities is called:

Hydropericardium.
Hydrothorax.
Ascites.
Anasarca.
Hydrocele.

2.Accumulation of fluid in the pericardial cavity is called:

Hydropericardium.
Hydrothorax.
Ascites.
Anasarca.
Hydrocele.

3.Accumulation of fluid in the peritoneal cavity is called:

Hydropericardium.
Hydrothorax.
Ascites.
Anasarca.
Hydrocele.

4.Generalized edema with profound subcutaneous tissue swelling is called:

Hydropericardium.
Hydrothorax.
Ascites.
Anasarca.
Hydrocele.

5.Pitting edema occurs in:

Hydropericardium.
Hydrothorax.
Ascites.
Anasarca.
Hydrocele.

6.Mechanisms of edema are all of the following, EXCEPT:

Increased hydrostatic pressure.


Hypoproteinemia.
Sodium retention.
Calcium retention.
Inflammation.

7.Mechanisms of edema are all of the following, EXCEPT:

Increased hydrostatic pressure.


Hypoproteinemia.
Hyperproteinemia.
Sodium retention.
Inflammation.

8.Mechanisms of edema are all of the following, EXCEPT:

Increased hydrostatic pressure.


Hypoproteinemia.
Sodium retention.e
Inflammation.
Neoplasia.

9.Organ in arterial hyperemia is characterized by:

Enlarged, red, hot.


Enlarged, blue, cold.
Enlarged, brown, hot.
Small, red, cold.
Small, brown, hot.

10.Organ in acute venous hyperemia is characterized by:

Enlarged, red, hot.


Enlarged, blue-red, cold.
Enlarged, brown, hot.
Small, red, cold.
Small, brown, hot.

11.Signs of chronic venous hyperemia are all of the following, EXCEPT:

Brown induration of lungs.


Nutmeg liver.
Cyanotic induration of spleen.
Cyanotic induration of kidneys.
Fatty liver.
12.Signs of chronic venous hyperemia are all of the following, EXCEPT:

Cyanosis of skin.
Pale skin.
Brown induration of lungs.
Nutmeg liver.
Cyanotic induration of spleen and kidneys.

13.Signs of chronic venous hyperemia are all of the following, EXCEPT:

Brown induration of lungs.


Nutmeg liver.
Cyanotic induration of spleen.
Cyanotic induration of kidneys.
Waxy kidneys.

14.Nutmeg liver is characterized by:

Enlarged, hard, yellow-grey with dark-red dots.


Enlarged, hard, dark-red.
Enlarged, soft, yellow-grey.
Small, hard, reddish-yellow.
Small. soft, yellow-red.

15.Cyanotic induration of kidney is characterized by:

Enlarged, hard, brown.


Small. soft, red.
Enlarged, hard, blue-red.
Small, hard, red-brown.
Enlarged, soft, brown.

16.Which pigment is accumulated in the lungs in brown induration:

Hemin
Hemosiderin.
Hemomelanin
Melanin.
Bilirubin.

17.Hemorrhage from nasal cavity is called:

Epistaxis.
Haemoptoe.
Haematemesis.
Maelena.
Metrorrhagia.

18.Hemorrhage from respiratory system is called:

Epistaxis.
Haemoptoe.
Haematemesis.
Maelena.
Metrorrhagia.

19.Hemorrhage from stomach is called:

Epistaxis.
Haemoptoe.
Haematemesis.
Maelena.
Metrorrhagia.

20.Hemorrhage from intestine is called:

Epistaxis.
Haemoptoe.
Haematemesis.
Maelena.
Metrorrhagia.

21.Hemorrhage from urinary system is called:

Epistaxis.
Haemoptoe.
Haematemesis.
Haematuria.
Metrorrhagia.

22.Hemorrhage from ear is called:

Epistaxis.
Haemoptoe.
Haematemesis.
Haematuria.
Otorrhagia.

23.Accumulation of blood in the pericardial cavity is called:

Hemopericardium.
Hemothorax.
Hemoperitoneum.
Hemarthrosis.
Hematoma.

24.Accumulation of blood in the pleural cavities is called:

Hemopericardium.
Hemothorax.
Hemoperitoneum.
Hemarthrosis.
Hematoma.

25.Accumulation of blood in the peritonal cavity is called:

Hemopericardium.
Hemothorax.
Hemoperitoneum.
Hemarthrosis.
Hematoma.

26.Accumulation of blood in the joint cavity is called:

Hemopericardium.
Hemothorax.
Hemoperitoneum.
Hemarthrosis.
Hematoma.

27.Hemorrhage with regular borders and destruction of tissue is called:

Hematoma.
Hemorrhagic infiltration.
Petechiae.
Purpura.
Ecchymoses.

28.Hemorrhage with irregular borders and accumulation of blood in the interstitial tissue is called:

Hematoma.
Hemorrhagic infiltration.
Petechiae.
Purpura.
Ecchymoses.

29.Minute hemorrhages (1-2 mm) into skin, mucous membranes or serosal surfaces are called:

Hematoma.
Hemorrhagic infiltration.
Petechiae
Purpura.
Ecchymoses.
30.Small (3-5 mm) hemorrhages into skin, mucous membranes or serosal surfaces are called:

A. Hematoma.

B. Hemorrhagic infiltration.

C. Petechiae.

D. Purpura.

E. Ecchymoses.

31.Large (1-2 cm) subcutaneous hemorrhages are called:

A. Hematoma.

B. Hemorrhagic infiltration.

C. Petechiae.

D. Purpura.

E. Ecchymoses.

32.Large (more than 2 cm) hemorrhage into skin is called:

A. Hematoma.

B. Hemorrhagic infiltration.

C. Bruise.

D. Purpura.

E. Ecchymoses.

33.Rupture of the vessel wall may be result of:

A. Atherosclerosis.

B. Inflammation.

C. Invasion of cancer.

D. Hypoxia.

E. Intoxication.

34.Rupture of the heart wall may be result of:


A. Inflammation.

B. Myocardial infarction.

C. Invasion of cancers.

D. Hypoxia.

E. Intoxication.

35.Corrosion of the vessel wall may be result of:

A. Atherosclerosis.

B. Inflammation.

C. Vascular abnormalities.

D. Hypoxia.

E. Intoxication.

36.Corrosion of the vessel wall may be result of:

A. Atherosclerosis.

B. Vascular abnormalities.

C. Invasion of cancer.

D. Hypoxia.

E. Intoxication.

37.Increase of vascular permeability may be result of:

A. Atherosclerosis.

B. Vascular hyalinosis.

C. Invasion of cancer.

D. Hypoxia.

E. Inflammation.

38.Increase of vascular permeability may be result of:

A. Atherosclerosis.

B. Vascular hyalinosis.
C. Invasion of cancer.

D. Inflammation.

E. Intoxication.

39.Increase of vascular permeability may be result of:

A. Atherosclerosis.

B. Vascular abnormalities.

C. Invasion of cancer.

D. Inflammation.

E. Intoxication.

40.Clinical significance of hemorrhage depends on all the following, EXCEPT:

A. Blood group of individual.

B. Volume of blood loss.

C. Rate of blood loss.

D. Site of hemorrhage.

E. Diameter of damaged vessel.

Hemodynamic disorders-II: thrombosis, embolism, infarction, shock, DIC. (Kozmina - 60)

1. In septic shock, pathogenesis of endothelial cell injury involves the following mechanisms except:

Lipopolysaccharide from lysed bacteria injures the endothelium


Interleukin-1 causes endothelial cell injury
TNF-α causes direct cytotoxicity
Adherence of PMNs to endothelium causes endothelial cell injury

2. An intact endothelium elaborates the following anti-thrombotic factors except:

Thrombomodulin
ADPase
Tissue plasminogen activator
Thromboplastin

3. The most common cause of arterial thromboemboli is:

Cardiac thrombi
Aortic aneurysm
Pulmonary veins
Aortic atherosclerotic plaques

4. Venous emboli are most often lodged in:

Intestines
Kidney
Lungs
Heart
Brain

5. Pathologic changes between sudden decompression from high pressure to normal levels and decompression from
low pressure to normal levels are:

More marked in the former


More marked in the latter
No difference between the two
Acute form is more marked in the latter
Chronic form is more marked in the latter

6. The infarct of following organ is invariably hemorrhagic:

Infarct kidney
Infarct spleen
Infarct lung
Infarct heart
Infarct liver

7. Milroy’s disease is:

Cerebral oedema
Pulmonary oedema
Hereditary lymphoedema
Postural oedema
Ascites

8. Pick the correct sequence:

Renin - Angiotensin II - Angiotensin I – Angiotensinogen - Aldosterone


Angiotensinogen – Renin - Angiotensin II - Angiotensin I - Aldosterone
Renin – Angiotensinogen - Angiotensin I - Angiotensin II – Aldosterone
Aldosterone – Renin – Angiotensinogen - Angiotensin II - Angiotensin I
Angiotensin I - Aldosterone – Renin – Angiotensinogen - Angiotensin II
9. Which of the following is true?

Arterial thrombi are white and occlusive


Venous thrombi are white and occlusive
Arterial thrombi are white and mural
Venous thrombi are red and mural
Venous thrombi are white and mural

10. Disseminated intravascular coagulation (DIC) is characterized by the following except:

Thrombocytopenia
Microangiopathic hemolytic anemia
Presence of FDPs (fibrin degradation products) in the blood
Normal prothrombin time
Thrombin time is prolonged

11. Which of the following is not included in TTP (thrombotic thrombocytopenic purpura) triad?

DIC (disseminated intravascular coagulation)


Thrombocytopenia
Microangiopathic hemolytic anemia
Fibrin microthrombi
Leucocytosis

12. Most common manifestation of DIC (disseminated intravascular coagulation) is:

Bleeding
Thrombosis
Microangiopathic hemolytic anemia
Organ damage
High temperature

13. A 25-year-old female presents with a history of losing four pregnancies in the past 5 years. She also has s history
of recurrent pains in her legs secondary to recurrent thrombosis. Her symptoms are most likely due to a deficiency
of:

PA inhibitiors
Protein C
Plasmin
Thrombin
C’1 inactivator

14. Procoagulant factors produced by endothelial cells include

Thrombomodulin
Prostacyclin
won Willebrand factor
Thromboxane A2
Fibrinogen
15. Which one of the listed laboratory findings is most consistent with an individual who is not taking any medication
but has a familial deficiency of coagulation factor VII, assuming all other coagulation factors to be within normal
limits?

Prothrombin time (PT) Partial Thromboplastin time (PTT)

A. Prolonged Normal

B. Normal Prolonged

C. Shortened Normal

D. Normal Shortened

E. Shortened Prolonged

16. A postmortem clot is most likely to

Grossly display features of recanalization


Grossly have lines of Zahn
Grossly have the appearance of «chicken fat» overlying «currant jelly»
Microscopically appear attached to the blood vessel
Microscopically have alternating layers of cells and platelets

17. What is the most common site of origin of thrombotic pulmonary emboli?

Deep leg veins


Lumen of left ventricle
Lumen of right ventricle
Mesenteric veins
Superficial leg veins

18. A 9-year-old boy suddenly develops severe testicular pain. He is taken to the emergency room, where he is
evaluated and immediately taken to surgery. There his left testis is found to be markedly hemorrhagic due to
testicular torsion. This abnormality causes a hemorrhagic infarction because of

Arterial occlusion
Septic infarction
The collateral blood supply of the testis
The dual blood supply of the testis
Venous occlusion

19. Red infarction is caused by all of the following events, except:

Coronary artery thrombosis


Pulmonary embolism
Torsion of the testis
Superior mesenteric artery thromboembolism
Portal vein thrombosis

20. Shock is commonly associated with all of the following conditions, except:
Escherichia coli sepsis
Myocardial infarction
Cholera
Acute pancreatitis
Cerebral infarction

21. The fate of the thrombus may be all of the following, except:

Dissolution
Recanalization
Organization
Embolization
Malignization

22. The causes of infarction include all of the following pathologic conditions, except:

Trombotic events
Embolic events
Arterial occlusion
Local vasospasm
Hemophilia

23. Red infarct occurs in all of the following pathologic conditions, except:

Venous occlusion
Coronary occlusion
Loose tissue
Tissues with dual circulation
Previously congested with blood tissues

24. White infarct results from which of the following:

Venous occlusion
Arterial occlusion
Loose tissue
Tissues with dual circulation
Previously congested with blood tissues

25. Red infarct occurs in which of the following organs:

Lung
Heart
Spleen
Kidney
Lower extremity

26. White infarct commonly occurs in all organs, except:

Lung
Spleen
Kidney
Heart
Brain

27. The formation of a thrombus is favored by all of the following, except:

Anemia
Endothelial injury
Pregnancy
Stases
Thrombocytosis

28. The type of tissue necrosis commonly associated with myocardial infarction is which of the following:

Caseous necrosis
Coagulation necrosis
Enzymatic fat necrosis
Gangrenous necrosis
Fibrinoid necrosis

29. Pulmonary emboli may originate from all of the following sites, except:

Deep leg veins


Inferior vena cava
Pelvic veins
Portal vein
Right atrial appendage

30. The development of endothelial-lined blood channels that reestablish blood flow through a vascular thrombus is
known as:

Collateral circulation
Recanalization
Organization
Hyalinization
Incapsulation

31. The initial step of the thrombus formation is which of the following:

Activation ofthrombin
Development of fibrin plugs
Endothelial injury
Marginationofleukocytes
Trapping of red cells

32. The type of tissue necrosis associated with renal infarction is:

Caseous necrosis
Coagulation necrosis
Enzymatic fat necrosis
Gangrenous necrosis
Liquefactive necrosis
33. Disseminated intravascular coagulation is characterized by each of the following, except:

Decreased fibrinolysis
Decreased plasma fibrinogen
Hemolysis
Prolonged partial thromboplastin time
Thrombocytopenia

34. Each of the following conditions favors the development of thrombosis, except:

Endothelial injury
Polycythemia
Stases
Thrombocytopenia
Congestion

35. The type of necrosis most often caused by sudden ischemia from vascular occlusion is:

Apoptosis
Caseous necrosis
Coagulation necrosis
Fat necrosis
Fibrinoid necrosis

36. All the following disorders are associated with disseminated intravascular coagulation, except:

Infections
Neoplasms
Massive tissue injury
Malnutritions
Obstetric complications

37. The usual source of pulmonary emboli is -

Thrombi in the deep veins of the lower extremities


Thrombi in the veins of upper extremities
Thrombi in the vena porta
Thrombi in the left atrial appendage
Thrombi in the uterus arteries

38. Emboli in the main pulmonary artery often cause:

Infarctionofthe lung
Sudden death
Infarction of the heart
Disseminated intravascular coagulation
Acute pneumonia
39. The origin of emboli include all of the following, except:

Thrombi
Fat droplets
Air bubbles
Amyloid masses
Microorganisms

40. The forms of embolism include all of the following, except:

Pulmonary thromboembolism
Fat embolism
Viral embolism
Bacterial embolism
Air embolism

41. The selective stain, used to identify fat in the fat emboli syndrome is:

Sudan III
Toluidin blue
Congo red
PAS reaction
Hematoxylin and eosin

42. Acute tubular necrosis develops in which of the following conditions:

Amyloidosis
Acute glomenilonephritis
Kidney infarct
Systemic thromboembolism
Hypovolemic shock

43. The commonest site of thrombosis is

Vein
Artery
Heart
Capillary
Lymphatics

44. The pulmonary thrombus is different from pulmonary thromboembolus all, except:

Locally formed
In small arteries and branches
Firmly adherent to vessel wall
Head pale, tail red
Lying free to vessel wall

45. Which of the following is true of fat embolism, except:


Trauma of bones
Trauma of soft tissue
Diabetes mellitus
Extensive burns
Operation on head

46. Air embolism occurs due to trauma of -

Large arteries
Neck veins
Soft tissue
Lungs
Heart

47. Retrograde embolism occurs in -

Aorta
Femoral artery
Portal vein
Microcirculatory vessels
Heart

48. Pale infarction is not seen in -

Intestines
Liver
Spleen
Heart
Brine

49. The thrombus in compare with postmortem blood clot is?

Laying free in the vessel


With smooth surface
Attached to the vessel’s wall
Elastic
Gelatinous and soft

50. The histological features of shock includes following, except:

ATN (acute tubular necrosis)


Pulmonary congestion
Depletion of lipids in adrenal cortex
Hepatic necrosis
Depletion of lymphocytes

51. Fat embolism may be recognized with - ?

Sudan III
Hematoxiiin-eosin
Congo-red
By Van-Guison
Methilen blue

52. Which of the following is most frequent site of venous thrombosis?

Veins of lower extremity


Pelvic veins
Portal vein
Hepatic vein
Pulmonary veins

53. Stasis of blood is –

Accumulation of blood in the microcirculatory vessel


Deficiency of blood in the microcirculatory vessel
Stopping of blood flow in the microcirculatory vessel
Stopping of blood flow in the venous system
Accumulation of blood in the aorta

54. In a state of shock there is:

A decreased hydrostatic pressure and increased osmotic


Cardiovascular collapse.
Active process leading to increased volume of blood.
Decreased pulse rate.
Fever

55. Unfavorable outcome of thrombosis is –

Aseptic autolysis
Canalization
Vascularisation
Thromboembolism
Petrification

56. The cause of ischemic infarct is –

Stasis of blood
Lymphostasis
Venus hyperemia
Arterial thrombosis
Arterial hyperemia

57. In which disease occur the paradoxal embolisms?

Left ventricle aneurism


Aortic coarctation
Pulmonary artery stenosis
Defect of the intraventricular septa
Tricuspid stenosis

58. In shock characteristic feature is –


Cardiac failure
Poor perfusion of tissues
Cyanosis
Oedema
Acute adrenal hemorrhagic necrosis

59. All of the following conditions may predispose to pulmonary embolism, except

Protein S deficiency
Malignancy
Obesity
Progesterone therapy
Bed-ridden patients

60. Thrombus in the aneurisms is –

Obturative
Congestive
Dilative
Mural
Ball

THROMBOSIS. EMBOLISM. INFARCTION. SHOCK (Davydov-40)

1.Favourable significance of thrombosis may be in:

A. Coronary artery in atherosclerosis.

B. Parietal thrombus of left ventricle in myocardial infarction.

C. Thrombosis of vena femoralis in phlebitis.

D. Cerebral arteries in atherosclerosis.

E. Arterial thrombosis in adges of traumatic wound.

2.Red thrombus usually occurs in:

A. Arteries.

B. Veins.

C. Aorta.

D. Capillaries.

E. Lymphatic vessels.
3.White thrombus usually occurs in:

A. Arteries.

B. Veins.

C. Venules.

D. Capillaries.

E. Lymphatic vessels.

4.Mixed thrombus consists of:

A. Head, neck, body.

B. Head, body, extremities.

C. Head, body, tail.

D. Head, neck, tail.

E. Neck, body, tail.

5.Sign of thrombus is:

A. Smooth surface.

B. Soft.

C. Homogenous cut surface.

D. Attached to vessel wall.

E. Not attached to vessel wall.

6.Sign of thrombus is:

A. Rough surface.

B. Soft.

C. Gomogenous cut surface.

D. Smooth surface.

E. Not attached to vessel wall.

7.Sign of thrombus is:


A. Smooth surface.

B. Soft, wet.

C. Homogenous cut surface.

D. Hard, dry.

E. Not attached to vessel wall.

8.Sign of thrombus is:

A. Smooth surface.

B. Laminated cut surface.

C. Homogenous cut surface.

D. Soft, wet.

E. Not attached to vessel wall.

9.Sign of postmortem clot is:

A. Ruph surface.

B. Laminated cut surface.

C. Homogenous cut surface.

D. Hard, dry.

E. Attached to vessel wall.

10.Sign of postmortem clot is:

A. Ruph surface.

B. Smooth surface.

C. Homogenous cut surface.

D. Hard, dry.

E. Attached to vessel wall.

11.Sign of postmortem clot is:

A. Ruph surface.

B. Hard, dry.
C. Laminated cut surface.

D. Not attached to vessel wall.

E. Attached to vessel wall.

12.Sign of postmortem clot is:

A. Ruph surface.

B. Hard, dry.

C. Laminated cut surface.

D. Soft, wet.

E. Attached to vessel wall.

13.Ball thrombus is localized in:

A. Left ventricle.

B. Right ventricle.

C. Left atrium.

D. Right atrium.

E. Aorta.

14.Localization of thrombotic vegetations is:

A. Aorta.

B. Arteries

C. Veins.

D. Heart valves.

E. Capillaries.

15.Grouth of connective tissue in thrombotic masses is called:

A. Organization.

B. Calcification.

C. Recanalization.
D. Propagation.

E. Embolization.

16.Deposition of calcium salts to thrombotic masses is called:

A. Organization.

B. Calcification.

C. Recanalization.

D. Propagation.

E. Embolization.

17.Detachtment of thrombus from vessel wall is called:

A. Organization.

B. Calcification.

C. Recanalization.

D. Propagation.

E. Embolization.

18.Paradoxical embolism may occurs in:

A. Aneurism of left ventricle.

B. Coarctation of aorta.

C. Stenosis of pulmonary artery.

D. Ventricular septal defect.

E. Atresia of aorta.

19.Paradoxical embolism may occurs in:

A. Aneurism of left ventricle.

B. Coarctation of aorta.

C. Stenosis of pulmonary artery.

D. Atresia of aorta.

E. Atrial septal defect.


20.Exogenic type of embolism is:

A. Thromboembolism.

B. Cell embolism.

C. Amniotic fluid embolism.

D. Gas embolism.

E. Foreign body embolism.

21.Generalization of infection is a result of:

A. Thromboembolism.

B. Microbial embolism.

C. Amniotic fluid embolism.

D. Gas embolism.

E. Foreign body embolism.

22.Metastasis of malignant tumors is a result of:

A. Thromboembolism.

B. Microbial embolism.

C. Cell embolism.

D. Air embolism.

E. Foreign body embolism.

23.Fractures of bones may lead to:

A. Thromboembolism.

B. Microbial embolism.

C. Fat embolism.

D. Gas embolism.

E. Amniotic fluid embolism.

24.Postpartum period may be complicated with:


A. Thromboembolism.

B. Microbial embolism.

C. Amniotic fluid embolism.

D. Gas embolism.

E. Foreign body embolism.

25.Common localization of ischemic infarction is:

A. Brain.

B. Lungs.

C. Small intestine.

D. Large intestine.

E. Liver.

26.Common localization of hemorrhagic infarction is:

A. Brain.

B. Lungs.

C. Small intestine.

D. Large intestine.

E. Liver.

27.Cause of hemorrhagic infarction is:

A. Thrombosis of artery.

B. Embolism of artery.

C. Venous hyperemia.

D. Vasospasm.

E. Compression of artery.

28.Causes of ischemic infarction are all of the following, EXCEPT:

A. Thrombosis of artery.

B. Embolism of artery.
C. Vasospasm.

D. Compression of artery.

E. Venous hyperemia.

29.Morphologically infarction of brain occurs as:

A. Coagulative necrosis.

B. Liquefactive necrosis.

C. Gangrene.

D. Fat necrosis.

E. Fibrinoid necrosis.

30.Morphologically myocardial infarction occurs as:

A. Coagulative necrosis.

B. Liquefactive necrosis.

C. Gangrene.

D. Fat necrosis.

E. Fibrinoid necrosis.

31.Unfourable outcome of infarction is:

A. Organization.

B. Incapsulation.

C. Cyst formation.

D. Suppuration.

E. Hemosiderosis.

32.Typical outcome of hemorrhagic infarction is:

A. Organization.

B. Incapsulation.

C. Cyst formation.
D. Suppuration.

E. Hemosiderosis.

33.Typical outcome of cerebral infarction is:

A. Organization.

B. Incapsulation.

C. Cyst formation.

D. Suppuration.

E. Hemosiderosis.

34.Type of shock which associated with action of strong painful irritant is:

A. Cardiogenic.

B. Traumatic.

C. Hypovolemic.

D. Neurogenic.

E. Anaphylactic.

35.Type of shock which associated with action of decrease of cardiac output is:

A. Cardiogenic.

B. Traumatic.

C. Hypovolemic.

D. Neurogenic.

E. Anaphylactic.

36.Type of shock which associated with loss of blood:

A. Cardiogenic.

B. Traumatic.

C. Hypovolemic.
D. Neurogenic.

E. Anaphylactic.

37.Type of shock which associated with severe burns is:

A. Cardiogenic.

B. Traumatic.

C. Endotoxic.

D. Hypovolemic.

E. Anaphylactic.

38.Type of shock which associated with infectious diseases is:

A. Cardiogenic.

B. Traumatic.

C. Hypovolemic.

D. Neurogenic.

E. Endotoxic.

39.Type of shock which associated with injury of spinal cord is:

A. Cardiogenic.

B. Traumatic.

C. Hypovolemic.

D. Neurogenic.

E. Anaphylactic.

40.Type of shock which associated with generalized reactions of hypersensitivity is:

A. Cardiogenic.

B. Traumatic.

C. Hypovolemic.

D. Neurogenic.

E. Anaphylactic.
Immunopathology (32 q)

1. Сongenital absence of thymus is called:

Aplasia
Hypoplasia
Dysplasia
Atrophy
Thymomegaly

2. Congenital incomplete development of thymus is called:

Aplasia
Hypoplasia
Dysplasia
Atrophy
Thymomegaly

3. Aquired decrease in weight of thymus is called:

Aplasia
Hypoplasia
Dysplasia
Atrophy
Thymomegaly

4. Abnormal development of thymus is called:

Aplasia
Hypoplasia
Dysplasia
Atrophy
Thymomegaly

5. Morphologically immediate type of hypersensitivity occurs as:

Fibrinoid necrosis
Lympho-hystiocytic infiltration
Macrophageal infiltration
Granulomatosis
Cytoplasmic bridges between lymphocytes and macrophages

6. Morphologically immediate type of hypersensitivity occurs as:


Lympho-hystiocytic infiltration
Macrophageal infiltration
Granulomatosis
Cytoplasmic bridges between lymphocytes and macrophages
Plasmatic saturation

7. Morphologically immediate type of hypersensitivity occurs as:

Lympho-hystiocytic infiltration
Mucoid changes
Macrophageal infiltration
Granulomatosis
Cytoplasmic bridges between lymphocytes and macrophages

8. Morphologically immediate type of hypersensitivity occurs as:

Lympho-hystiocytic infiltration
Fibrinoid changes
Macrophageal infiltration
Granulomatosis
Cytoplasmic bridges between lymphocytes and macrophages

9. Morphologically immediate type of hypersensitivity occurs as:

Lympho-hystiocytic infiltration
Fibrinous-hemorrhagic exudate
Macrophageal infiltration
Granulomatosis
Cytoplasmic bridges between lymphocytes and macrophages

10. Morphological signs of immediate type of hypersensitivity are all the following, except:

Lympho-hystiocytic infiltration
Mucoid and Fibrinoid changes
Plasmatic saturation
Fibrinoid necrosis
Fibrinous-hemorrhagic exudate

11. Morphological signs of immediate type of hypersensitivity are all the following, except:

Mucoid and Fibrinoid changes


Plasmatic saturation
Granulomatosis
4.Fibrinoid necrosis
5.Fibrinous-hemorrhagic exudate

12. Morphological signs of immediate type of hypersensitivity are all the following, except:

Mucoid and Fibrinoid changes


Plasmatic saturation
Fibrinoid necrosis
Fibrinous-hemorrhagic exudate
Macrophage infiltration

13. Morphologically delayed type of hypersensitivity occurs as:

Lympho-hystiocytic infiltration
Mucoid and Fibrinoid changes
Plasmatic saturation
Fibrinoid necrosis
Fibrinous-hemorrhagic exudate

14. Morphologically delayed type of hypersensitivity occurs as:

Mucoid and Fibrinoid changes


Plasmatic saturation
Granulomatosis
Fibrinoid necrosis
Fibrinous-hemorrhagic exudate

15. Morphologically delayed type of hypersensitivity occurs as:

Mucoid and Fibrinoid changes


Plasmatic saturation
Fibrinoid necrosis
Fibrinous-hemorrhagic exudate
Macrophageal infiltration

16. Morphologically delayed type of hypersensitivity occurs as:

Mucoid and Fibrinoid changes


Plasmatic saturation
Fibrinoid necrosis
Fibrinous-hemorrhagic exudate
Cytoplasmic bridges between lymphocytes and macrophages

17. Morphological signs of delayed type of hypersensitivity are all the following, except:

Fibrinoid necrosis
Lympho-hystiocytic infiltration
Macrophageal infiltration
Granulomatosis
Cytoplasmic bridges between lymphocytes and macrophages

18. Morphological signs of delayed type of hypersensitivity are all the following, except:

Lympho-hystiocytic infiltration
Macrophageal infiltration
Granulomatosis
Cytoplasmic bridges between lymphocytes and macrophages
Plasmatic saturation

19. Morphological signs of delayed type of hypersensitivity are all the following, except:
Lympho-hystiocytic infiltration
Mucoid changes
Macrophageal infiltration
4.Granulomatosis
Cytoplasmic bridges between lymphocytes and macrophages

20. Morphological signs of delayed type of hypersensitivity are all the following, except:

Lympho-hystiocytic infiltration
Fibrinous-hemorrhagic exudate
Macrophageal infiltration
Granulomatosis
Cytoplasmic bridges between lymphocytes and macrophages

21. Morphological signs of transplant rejection are all the following, except:

Lympho-hystiocytic infiltration
Edema of transplant
Macrophageal infiltration
Granulomatosis
Leukocytic infiltration

22. Morphological signs of transplant rejection are all the following, except:

Lympho-hystiocytic infiltration
Edema of transplant
Macrophageal infiltration
Cytoplasmic bridges between lymphocytes and macrophages
Leukocytic infiltration

23. Organ specific immune disease is:

Hashimoto thyroiditis
Rheumatoid arthritis
Systemic Lupus Erythematosus
Scleroderma
Secondary thrombocytopenia

24. Organ specific immune disease is:

Rheumatoid arthritis
Systemic Lupus Erythematosus
Scleroderma
Secondary thrombocytopenia
Encephalomyelitis

25. Non-organ specific immune disease is:

Hashimoto thyroiditis.
Rheumatoid arthritis.
Encephalomyelitis.
Polyneuritis.
Disseminated sclerosis of CNS (central nervous system).

26. Non-organ specific immune disease is:

Hashimoto thyroiditis.
Encephalomyelitis.
Systemic Lupus Erythematosus.
Polyneuritis.
Disseminated sclerosis of CNS (central nervous system).

27. Variant of primary immunodeficiency syndrome occurs:

In leukemia.
Under radial therapy.
In aplasia of thymus.
In infections.
In sarcoidosis.

28. Variant of primary immunodeficiency syndrome occurs:

In leukemia.
Under radial therapy.
In sarcoidosis.
In hypoplasia of thymus.
In malignant lymphoma.

29. Variant of secondary immunodeficiency syndrome is:

Lui-Bar syndrome
Neseloff syndrome
DiGeorge syndrome
Bruton syndrome
AIDS

30. Complication of immunodeficiency syndromes is:

Arterial hypertension
Myocardial infarction
Typhoid fever
Sepsis
Chronic peptic gastric ulcer

31. Complication of immunodeficiency syndromes is:

Recurrence of bronchial asthma


Recurrence of chronic hepatitis
Recurrence of chronic cholecystitis
Recurrence of chronic gastric peptic ulcer
Recurrence of tuberculosis
32. Complication of immunodeficiency syndromes is:

Arterial hypertension
Myocardial infarction
Typhoid fever
Purulent pneumonia
Chronic peptic gastric ulcer

Acute Inflammation

1 A 22-year-old woman nursing her newborn develops a tender erythematous area around the nipple of her
left breast. A thick, yellow fluid is observed to drain from an open fissure. Examination of this breast fluid
under the light microscope will most likely reveal an abundance of which of the following inflammatory
cells?

B lymphocytes
Eosinophils
Mast cells
Neutrophils
Plasma cells

2 A 63-year-old man becomes febrile and begins expectorating large amounts of mucopurulent sputum.
Sputum cultures are positive for Gram-positive diplococci. Which of the following mediators of
inflammation provides potent chemotactic factors for the directed migration of inflammatory cells into the
alveolar air spaces of this patient?

Bradykinin
Histamine
Myeloperoxidase
N-formylated peptides
Plasmin

3 A 59-year-old man suffers a massive heart attack and expires 24 hours later due to ventricular arrhythmia.
Histologic examination of the affected heart muscle at autopsy would show an abundance of which of the
following inflammatory cells?

Fibroblasts
Lymphocytes
Macrophages
Neutrophils
Plasma cells

4 A 5-year-old boy punctures his thumb with a rusty nail. Four hours later, the thumb appears red and
swollen. Initial swelling of the boy’s thumb is primarily due to which of the following mechanisms?

Decreased intravascular hydrostatic pressure


Decreased intravascular oncotic pressure
Increased capillary permeability
Increased intravascular oncotic pressure
Vasoconstriction of arterioles

5 An 80-year-old woman presents with a 4-hour history of fever, shaking chills, and disorientation. Her
blood pressure is 80/40 mm Hg. Physical examination shows diffuse purpura on her upper arms and chest.
Blood cultures are positive for Gram negative organisms. Which of the following cytokines is primarily
involved in the pathogenesis of direct vascular injury in this patient with septic shock?

Interferon-g
Interleukin-1
Platelet-derived growth factor
Transforming growth factor-a
Tumor necrosis factor-a

6 A 24-year-old intravenous drug abuser develops a 2-day history of severe headache and fever. His
temperature is 38.7°C (103°F). Blood cultures are positive for Gram-positive cocci The patient is given
intravenous antibiotics, but he deteriorates rapidly and dies. A cross section of the brain at autopsy (shown
in the image) reveals two encapsulated cavities. Which of the following terms best characterizes this
pathologic finding?

Chronic inflammation
Fibrinoid necrosis
Granulomatous inflammation
Reactive gliosis
Suppurative inflammation

7 A 36-year-old woman with pneumococcal pneumonia develops a right pleural effusion. The pleural fluid
displays a high specific gravity and contains large numbers of polymorphonuclear (PMN) leukocytes. Which
of the following best characterizes this pleural effusion?

Fibrinous exudate
Lymphedema
Purulent exudate
Serosanguineous exudate
Transudate

8 A 33-year-old man presents with a 5-week history of calf pain and swelling and low-grade fever. Serum
levels of creatine kinase are elevated. A muscle biopsy reveals numerous eosinophils. What is the most
likely etiology of this patient’s myalgia?

Autoimmune disease
Bacterial infection
Muscular dystrophy
Parasitic infection
Viral infection

9 A 10-year-old boy with a history of recurrent bacterial infections presents with fever and a productive
cough. Biochemical analysis of his neutrophils demonstrates that he has an impaired ability to generate
reactive oxygen species. This patient most likely has inherited mutations in the gene that encodes which of
the following proteins?

Catalase
Cytochrome P450
Myeloperoxidase
NADPH oxidase
Superoxide dismutase

10 A 25-year-old woman presents with a history of recurrent shortness of breath and severe wheezing.
Laboratory studies demonstrate that she has a deficiency of C1 inhibitor, an esterase inhibitor that regulates
the activation of the classical complement pathway. What is the diagnosis?
Chronic granulomatous disease
Hereditary angioedema
Myeloperoxidase deficiency
Selective IgA deficiency
Wiskott-Aldrich syndrome

11 A 40-year-old man complains of a 2-week history of increasing abdominal pain and yellow discoloration
of his sclera. Physical examination reveals right upper quadrant pain. Laboratory studies show elevated
serum levels of alkaline phosphatase (520 U/dL) and bilirubin (3.0 mg/dL). A liver biopsy shows portal
fibrosis, with scattered foreign bodies consistent with schistosome eggs. Which of the following
inflammatory cells is most likely to predominate in the portal tracts in the liver of
this patient?

Basophils
Eosinophils
Macrophages
Monocytes
Plasma cells

12 A 41-year-old woman complains of excessive menstrual bleeding and pelvic pain of 4 months. She uses
an intrauterine device for contraception. Endometrial biopsy (shown in the image) reveals an excess of
plasma cells (arrows) and macrophages within the stroma. The presence of these cells and scattered
lymphoid follicles within the endometrial stroma is evidence of which of the following conditions?

Acute inflammation
Chronic inflammation
Granulation tissue
Granulomatous inflammation
Menstruation

13 A 62-year-old woman undergoing chemotherapy for breast cancer presents with a 3-day history of fever
and chest pain. Cardiac catheterization reveals a markedly reduced ejection fraction with normal coronary
blood flow. A myocardial biopsy is obtained, and a PCR test for coxsackievirus is positive. Histologic
examination of this patient’s myocardium will most likely reveal an abundance of which of the following
inflammatory cells?

Eosinophils
Lymphocytes
Macrophages
Mast cells
Neutrophils

14 A 58-year-old woman with long-standing diabetes and hypertension develops end-stage renal disease and
dies in uremia. A shaggy fi brin-rich exudate is noted on the visceral pericardium at autopsy (shown in the
image). Which of the following best explains the pathogenesis of this fibrinous exudate?
Antibody binding and complement activation
Chronic passive congestion
Injury and increased vascular permeability
Margination of segmented neutrophils
Thrombosis of penetrating coronary arteries

15 A 68-year-old man presents with fever, shaking chills, and shortness of breath. Physical examination
shows rales and decreased breath sounds over both lung fields. The patient exhibits grunting respirations, 30
to 35 breaths per minute, with flaring of the nares. The sputum is rusty yellow and displays numerous
polymorphonuclear leukocytes. Which of the following mediators of inflammation is chiefly responsible
for the development of fever in this patient?

Arachidonic acid
Interleukin-1
Leukotriene B4
Prostacyclin (PGI2)
Thromboxane A2

16 A 35-year-old woman presents with a 5-day history of a painful sore on her back. Physical examination
reveals a 1-cm abscess over her left shoulder. Biopsy of the lesion shows vasodilation and leukocyte
margination (shown in the image). What glycoprotein mediates initial tethering of segmented neutrophils to
endothelial cells in this skin lesion?
Cadherin
Entactin
Integrin
Laminin
Selectin

17 A 14-year-old boy receives a laceration on his forehead during an ice hockey game. When he is first
attended to by the medic, there is blanching of the skin around the wound. Which of the following
mechanisms accounts for this transient reaction to neurogenic and chemical stimuli at the site of injury?

Constriction of postcapillary venules


Constriction of precapillary arterioles
Dilation of postcapillary venules
Dilation of precapillary arterioles
Ischemic necrosis

18 An 8-year-old girl with asthma presents with respiratory distress. She has a history of allergies and upper
respiratory tract infections. She also has history of wheezes associated with exercise. Which of the following
mediators of inflammation is the most powerful stimulator of bronchoconstriction and vasoconstriction in
this patient?

Bradykinin
Complement proteins
Interleukin-1
Leukotrienes
Tumor necrosis factor-

19 A 75-year-old woman complains of recent onset of chest pain, fever, and productive cough with rust-
colored sputum. A chest X-ray reveals an infiltrate in the right middle lobe. Sputum cultures are positive for
Streptococcus pneumoniae. Phagocytic cells in this patient’s affected lung tissue generate bacteriocidal
hypochlorous acid using which of the following enzymes?

Catalase
Cyclooxygenase
Myeloperoxidase
NADPH oxidase
Superoxide dismutase

20 A 28-year-old woman cuts her hand while dicing vegetables in the kitchen. The wound is cleaned and
sutured. Five days later, the site of injury contains an abundance of chronic inflammatory cells that actively
secrete interleukin-1, tumor necrosis factor- , interferon- , numerous arachidonic acid derivatives, and
various enzymes. Name these cells.

B lymphocytes
Macrophages
Plasma cells
Smooth muscle cells
T lymphocytes

21 A 68-year-old man with prostate cancer and bone metastases presents with shaking chills and fever. The
peripheral WBC count is 1,000/ L (normal = 4,000 to 11,000/ L). Which of the following terms best
describes this hematologic finding?

Leukocytosis
Leukopenia
Neutrophilia
Pancytopenia
Leukemoid reaction

22 A 25-year-old machinist is injured by a metal sliver in his left hand. Over the next few days, the wounded
area becomes reddened, tender, swollen, and feels warm to the touch. Redness at the site of injury in this
patient is caused primarily by which of the following mechanisms?

Hemorrhage
Hemostasis
Neutrophil margination
Vasoconstriction
Vasodilation

23 A 37-year-old man with AIDS is admitted to the hospital with a 3-week history of chest pain and
shortness of breath. An X-ray film of the chest shows bilateral nodularities of the lungs. A CT-guided lung
biopsy is shown in the image. The multinucleated cell in the center of this field is most likely derived from
which of the following inflammatory cells?
Basophils
Capillary endothelial cells
Macrophages
Myofi broblasts
Smooth muscle cells

24 A 10-year-old girl presents with a 2-week history of puffiness around her eyes and swelling of the legs
and ankles. Laboratory studies show hypoalbuminemia and proteinuria. The urinary sediment contains no
inflammatory cells or red blood cells. Which of the following terms describes this patient’s peripheral
edema?

Effusion
Exudate
Hydropic change
Lymphedema
Transudate

25 A 25-year-old woman develops a sore, red, hot, swollen left knee. She has no history of trauma and no
familial history of joint disease. Fluid aspirated from the joint space shows an abundance of segmented
neutrophils. Transendothelial migration of acute inflammatory cells into this patient’s joint space was
mediated primarily by which of the following families of proteins?

Entactins
Fibrillins
Fibronectins
Integrins
Laminins

26 A 50-year-old woman is discovered to have metastatic breast cancer. One week after receiving her first
dose of chemotherapy, she develops bacterial pneumonia. Which of the following best explains this patient’s
susceptibility to bacterial infection?

Depletion of serum complement


Impaired neutrophil respiratory burst
Inhibition of clotting factor activation
Lymphocytosis
Neutropenia

27 A 53-year-old man develops weakness, malaise, cough with bloody sputum, and night sweats. A chest X-
ray reveals numerous apical densities bilaterally. Exposure to Mycobacterium tuberculosis was documented
20 years ago, and M. tuberculosis is identified in the sputum. The patient subsequently dies of respiratory
insufficiency. The lungs are examined at autopsy (shown in the image). Which of the following best
characterizes the histopathologic features of this pulmonary lesion?

Acute suppurative inflammation


Chronic inflammation
Fat necrosis
Fibrinoid necrosis
Granulomatous inflammation

28 A 59-year-old man experiences acute chest pain and is rushed to the emergency room. Laboratory studies
and ECG demonstrate an acute myocardial infarction; however, coronary artery angiography performed 2
hours later does not show evidence of thrombosis. Intravascular thrombolysis that occurred in this patient
was mediated by plasminogen activators that were released by which of the following cells?
Cardiac myocytes
Endothelial cells
Macrophages
Segmented neutrophils
Vascular smooth muscle cells

29 A 68-year-old coal miner with a history of smoking and emphysema develops severe air-flow obstruction
and expires. Autopsy reveals a “black lung,” with coal-dust nodules scattered throughout the parenchyma
and a central area of dense fibrosis. The coal dust entrapped within this miner’s lung was sequestered
primarily by which of the following cells?

Endothelial cells
Fibroblasts
Lymphocytes
Macrophages
Plasma cells
30 A 40-year-old man presents with 5 days of productive cough and fever. Pseudomonas aeruginosa is
isolated from a pulmonary abscess. The CBC shows an acute effect characterized by marked leukocytosis
(50,000 WBC/ L), and the differential count reveals numerous immature cells (band forms). Which of the
following terms best describes these hematologic findings?

Leukemoid reaction
Leukopenia
Myeloid metaplasia
Myeloproliferative disease
Neutrophilia

31 A 19-year-old woman presents with 5 days of fever (38°C/101°F) and sore throat. She reports that she
has felt fatigued for the past week and has difficulty swallowing. A physical examination reveals generalized
lymphadenopathy. If this patient has a viral infection, a CBC will most likely show which of the following
hematologic findings?

Eosinophilia
Leukopenia
Lymphocytosis
Neutrophilia
Thrombocythemia

32 A 40-year-old woman presents with an 8-month history of progressive generalized itching, weight loss,
fatigue, and yellow sclerae. Physical examination reveals mild jaundice. The antimitochondrial antibody test
is positive. A liver biopsy discloses periductal inflammation and bile duct injury (shown in the image).
Which of the following inflammatory cells is the principal mediator of destructive cholangitis in this
patient?

Eosinophils
B lymphocytes
T lymphocytes
Mast cells
Neutrophils
33 A 25-year-old woman presents with a 2-week history of febrile illness and chest pain. She has an
erythematous, macular facial rash and tender joints, particularly in her left wrist and elbow. A CBC shows
mild anemia and thrombocytopenia. Corticosteroids are prescribed for the patient. This medication induces
the synthesis of an inhibitor of which of the following enzymes in inflammatory cells?

Lipoxygenase
Myeloperoxidase
Phospholipase A2
Phospholipase C
Superoxide dismutase

34 A 22-year-old man develops marked right lower quadrant abdominal pain over the past day. On physical
examination there is rebound tenderness on palpation over the right lower quadrant. Laparoscopic surgery is
performed, and the appendix is swollen, erythematous, and partly covered by a yellowish exudate. It is
removed, and a microscopic section shows infiltration with numerous neutrophils. The pain experienced by
this patient is predominantly the result of which of the following two chemical mediators?

Complement C3b and IgG


Interleukin-1 and tumor necrosis factor
Histamine and serotonin
Prostaglandin and bradykinin
Leukotriene and HPETE

35 A 40-year-old woman had laparoscopic surgery 3 months ago. Now she has a small 0.5 cm nodule
beneath the skin at the incision site that was sutured. Which of the following cell types is most likely to be
most characteristic of the inflammatory response in this situation?

A. Mast cell
B. Eosinophil
C. Giant cell
D. Neutrophil
E. Plasma cell

36 A 39-year-old man incurs a burn injury to his hands and arms while working on a propane furnace. Over
the next 3 weeks, the burned skin heals without the need for skin grafting. Which of the following is the
most critical factor in determining whether the skin in the region of the burn will regenerate?

Good cardiac output with tissue perfusion


Persistence of skin appendages
Maintenance of underlying connective tissue
Diminished edema and erythema
Granulation tissue formation
37 A 58-year-old woman has had a cough with fever for 3 days. A chest radiograph reveals infiltrates in the
right lower lobe. A sputum culture grows Streptococcus pneumoniae. The clearance of these organisms from
the lung parenchyma would be most effectively accomplished through generation of which of the following
substances by the major inflammatory cell type responding to this infection?

Platelet activating factor


Prostaglandin E2
Kallikrein
Leukotriene B4
E. Hydrogen peroxide

38 A clinical study is performed of patients with pharyngeal infections. The most typical clinical course
averages 3 days from the time of onset until the patient sees the physician. Most of these patients experience
fever and chills. On physical examination, the most common findings include swelling, erythema, and
pharyngeal purulent exudate. Which of the following types of inflammation did these patients most likely
have?

A. Granulomatous
B. Acute
C. Gangrenous
D. Resolving
E. Chronic

39 A 56-year-old man has had increasing dyspnea for 6 years. He has no cough or fever. He had chronic
exposure to inhalation of silica dust for many years in his job. A chest x-ray now shows increased interstitial
markings and parenchymal 1 to 3 cm solid nodules. His pulmonary problems are most likely to be mediated
through which of the following inflammatory processes?

A. Neutrophilic infiltrates producing leukotrienes


B. Foreign body giant cell formation
C. Plasma cell synthesis of immunoglobulins
D. Mast cell histamine release
E. Macrophage elaboration of cytokines

40 A 22-year-old woman has premature labor with premature rupture of fetal membranes at 20 weeks
gestation. Prior to that time, the pregnancy had been proceeding normally. A stillbirth occurs two days later.
Microscopic examination of the normal-sized placenta reveals numerous neutrophils in the amnion and
chorion, but no villitis. The premature labor was most likely mediated by the effects from release of which
of the following substances?
A. Immunoglobulin
B. Prostaglandin
C. Complement
D. Fibrinogen
E. Lymphokines

41 After two weeks in the hospital following a fall in which she incurred a fracture of her left femoral
trochanter, a 76-year-old woman now has a left leg that is swollen, particularly her lower leg below the
knee. She experiences pain on movement of this leg, and there is tenderness to palpation. Which of the
following complications is most likely to occur next after these events?

A. Gangrenous necrosis of the foot


B. Hematoma of the thigh
C. Disseminated intravascular coagulation
D. Pulmonary thromboembolism
E. Soft tissue sarcoma

42 A 43-year-old woman has had a chronic cough with fever and weight loss for the past month. A chest
radiograph reveals multiple nodules from 1 to 4 cm in size, some of which demonstrate cavitation in the
upper lobes. A sputum sample reveals the presence of acid fast bacilli. Which of the following cells is the
most important in the development her lung lesions?

A. Macrophage
B. Fibroblast
C. Neutrophil
D. Mast cell
E. Platelet

43 A 20-year-old man has experienced painful urination for 4 days. A urethritis is suspected, and Neisseria
gonorrheae is cultured. Numerous neutrophils are present in a smear of the exudate from the penile urethra.
These neutrophils undergo diapedesis to reach the organisms. Release of which of the following chemical
mediators is most likely to drive neutrophil exudation?

A. Histamine
B. Prostaglandin
C. Hageman factor
D. Bradykinin
E. Complement

44 An episode of marked chest pain lasting 4 hours brings a 51-year-old man to the emergency room. He is
found to have an elevated serum creatine kinase. An angiogram reveals a complete blockage of the left
circumflex artery 2 cm from its origin. Which of the following substances would you most expect to be
elaborated around the region of tissue damage in the next 3 days as an initial response to promote healing?

A. Histamine
B. Immunogloblulin G
C. Complement component C3b
D. Leukotriene B4
E. Vascular endothelial growth factor

45 A 94-year-old woman has developed a fever and cough over the past 2 days. Staphylococcus aureus is
cultured from her sputum. She receives a course of antibiotic therapy. Two weeks later she no longer has a
productive cough, but she still has a fever. A chest radiograph reveals a 3 cm rounded density in the right
lower lobe whose liquefied contents form a central air-fluid level. There are no surrounding infiltrates.
Which of the following is the best description for this outcome of her pneumonia?

A. Hypertrophic scar
B. Abscess formation
C. Regeneration
D. Bronchogenic carcinoma
E. Chronic inflammation
F. Granulomatous cavitation

46 A 36-year-old woman has been taking acetylsalicylic acid (aspirin) for arthritis for the past 4 years. Her
joint pain is temporarily reduced via this therapy. However, she now has occult blood identified in her stool.
Which of the following substances is most likely inhibited by aspirin to cause this complication?

A. Leukotriene B4
B. Interleukin-1
C. Thromboxane
D. Bradykinin
E. Hageman factor

47 A small sliver of wood becomes embedded in the finger of a 25-year-old man. He does not remove it, and
over the next 3 days the area around the sliver becomes red, swollen, and tender. Neutrophils migrate into
the injured tissue. Expression of which of the following substances on endothelial cells is most instrumental
in promoting this inflammatory reaction?

A. Interferon gamma
B. Hageman factor
C. Lysozyme
D. E-selectin
E. Prostacyclin

48 An inflammatory process that has continued for 3 months includes the transformation of tissue
macrophages to epithelioid cells. There are also lymphocytes present. Over time, fibroblasts lay down
collagen as the focus of inflammation heals. These events are most likely to occur as an inflammatory
response to which of the following infectious agents?

A. Mycobacterium tuberculosis
B. Pseudomonas aeruginosa
C. Cytomegalovirus
D. Giardia lamblia
E. Treponema pallidum

49 A 37-year-old man has had nausea and vomiting for 5 weeks. He experienced an episode of hematemesis
yesterday. On physical examination he has no abnormal findings. Upper GI endoscopy is performed, and
there is a 1.5 cm diameter lesion in the gastric antrum with loss of the epithelial surface. These findings are
most typical for which of the following pathologic processes?

A. Abscess
B. Serositis
C. Granuloma
D. Gangrene
E. Ulcer

50 A 17-year-old truck driver is involved in a collision. He incurs blunt force abdominal trauma. In response
to this injury, cells in tissues of the abdomen are stimulated to enter the G1 phase of the cell cycle from the
G0 phase. Which of the following cell types is most likely to remain in G0 following this injury?

A. Smooth muscle
B. Endothelium
C. Skeletal muscle
D. Fibroblast
E. Hepatocyte

51 A 19-year-old woman who works indoors spends a day outside gardening. She does not wear a hat or
sunscreen. That evening her partner remarks that her face appears red. Which of the following dermal
changes most likely accounts for her red appearance?

A. Neutrophil aggregation
B. Hemorrhage
C. Edema
D. Hemolysis
E. Vasodilation

52 A 45-year-old woman has had a chronic, non-productive cough for 3 months, along with intermittent
fever. She has a chest radiograph that reveals multiple small parenchymal nodules along with hilar and
cervical lymphadenopathy. A cervical lymph node biopsy is performed. Microscopic examination of the
biopsy shows noncaseating granulomatous inflammation. Cultures for bacterial, fungal, and mycobacterial
organisms are negative. Which of the following chemical mediators is most important in the development of
her inflammatory response?

A. Interferon gamma
B. Bradykinin
C. Complement C5a
D. Histamine
E. Prostaglandin E2

53 A 55-year-old man has a history of hypercholesterolemia with coronary artery disease and suffered a
myocardial infarction 2 years ago. He now presents with crushing substernal chest pain. Which of the
following laboratory tests is most useful in diagnosing the cause of his chest pain?

A. Increased white blood cell count


B. Elevated sedimentation rate
C. Decreased serum complement
D. Increased serum troponin
E. Decreased platelet count

54 A 15-year-old girl has had episodes of sneezing with watery eyes and runny nose for the past 2 weeks.
On physical examination she has red, swollen nasal mucosal surfaces. She has had similar episodes each
Spring and Summer when the amount of pollen in the air is high. Her symptoms are most likely to be
mediated by the release of which of the following chemical mediators?

A. Complement C3b
B. Platelet activating factor (PAF)
C. Tumor necrosis factor (TNF)
D. Histamine
E. Immunoglobulin G

55 A 45-year-old man has been working hard all day long carrying loads of bricks to build a wall. He takes a
non-steroidal anti-inflammatory drug (ibuprofen). Which of the following processes is this drug most likely
to diminish in his arms?
A. Thrombosis
B. Pain
C. Necrosis
D. Fibrinolysis
E. Scar formation

56 Within minutes following a bee sting, a 37-year-old man develops marked respiratory stridor with
dyspnea and wheezing. He also develops swelling and erythema seen in his arms and legs. An injection of
epinephrine helps to reverse these events and he recovers within minutes. Which of the following chemical
mediators is most important in the pathogenesis of this man's condition?

A. Bradykinin
B. Complement C5a
C. Nitric oxide
D. Tumor necrosis factor
E. Histamine

57 A 72-year-old woman did not get a 'flu' shot in the fall as recommended for older persons. In the
wintertime, she became ill, as many people in her community did, with a respiratory illness that lasted for 3
weeks. During this illness, she had a fever with a non-productive cough, mild chest pain, myalgias, and
headache. What was her chest radiograph most likely to have shown during this illness?

A. Hilar mass
B. Interstitial infiltrates
C. Hilar lymphadenopathy
D. Lobar consolidation
E. Pleural effusions

58 In an experiment, Enterobacter cloacae organisms are added to a solution containing leukocytes and
blood plasma. Engulfment and phagocytosis of the microbes is observed to occur. Next a substance is added
which enhances engulfment, and more bacteria are destroyed. Which of the following substances in the
plasma is most likely to produce this effect?

A. Complement C3b
B. Glutathione peroxidase
C. Immunoglobulin M
D. P-selectin
E. NADPH oxidase
59 A 43-year-old woman has had nausea with vomiting persisting for the past 5 weeks. On physical
examination there are no abnormal findings. She undergoes an upper GI endoscopy and gastric biopsies are
obtained. The microscopic appearance of these biopsies shows mucosal infiltration by lymphocytes,
macrophages, and plasma cells. Which of the following most likely caused her findings?

A. Staphylococcus aureus septicemia


B. Ingestion of chili peppers
C. Diabetes mellitus
D. Tuberculosis
E. Infection with Helicobacter pylori

60 In an experiment, lymphatic channels are observed in normal soft tissue preparations. Staphylococcus
aureus organisms are innoculated into the tissues and the immunologic response observed over the next 24
hours. Which of the following functions is most likely to be served by these lymphatics to produce a specific
immune response to these organisms?

A. Carry lymphocytes to peripheral tissue sites


B. Remove extravascular tissue fluid
C. Transport antigen presenting cells
D. Serve as a route for dissemination of infection
E. Provide an emigration point for neutrophils

61 In an experiment, surgical wound sites are observed following suturing. An ingrowth of new capillaries is
observed to occur within the first week. A substance elaborated by macrophages is found at the wound site
to stimulate this capillary proliferation. Which of the following substances is most likely to have this
function?

A. Platelet-derived growth factor


B. Phospholipase C-gamma
C. Fibronectin
D. Fibroblast growth factor
E. Epidermal growth factor

62 A 55-year-old man with a history of ischemic heart disease has worsening congestive heart failure. He
has noted increasing dyspnea and orthopnea for the past 2 months. On physical examination there is dullness
to percussion at lung bases. A chest x-ray shows bilateral pleural effusions. A left thoracentesis is
performed, and 500 mL of fluid is obtained. Which of the following characteristics of this fluid would most
likely indicate that it is a transudate?

A. Cloudy appearance
B. High protein content
C. <3 lymphocytes/microliter
D. Presence of fibrin
E. Large size of the effusion

63 In a clinical study, patients undergoing laparoscopic cholecystectomy are followed to document the post-
surgical wound healing process. The small incisions are closed with sutures. Over the 4 weeks following
surgery, the wounds are observed to regain tensile strength and there is re-epithelialization. Of the following
substances, which is most likely found to function intracellularly in cells involved in this wound healing
process?

A. Fibronectin
B. Laminin
C. Tyrosine kinase
D. Hyaluronic acid
E. Collagen

64 A 31-year-old woman has a laparotomy performed for removal of an ovarian cyst. She recovers
uneventfully, with no complications. At the time of surgery, a 12 cm long midline abdominal incision was
made. The tensile strength in the surgical scar will increase so her normal activities can be resumed. Most of
the tensile strength will likely be achieved in which of the following time periods?

A. One week
B. One month
C. Three months
D. Six months
E. One year

65 A 9-year-old girl sustains a small 0.5 cm long laceration to her right index finger while playing 'Queen of
Swords' with a letter opener. Which of the following substances, on contact with injured vascular basement
membrane, activates both the coagulation sequence and the kinin system as an initial response to this injury?

A. Thromboxane
B. Plasmin
C. Platelet activating factor
D. Hageman factor
E. Histamine

66 A 65-year-old woman has had a fever for the past day. On physical examination her temperature is 39°C
and blood pressure 90/50 mm Hg with heart rate of 106/minute. Laboratory studies show a WBC count of
12,510/microliter and WBC differential count of 78 segs, 8 bands, 11 lymphs, and 3 monos. A blood culture
is positive for Escherichia coli. Her central venous pressure falls markedly. She goes into hypovolemic
shock as a result of the widespread inappropriate release of a chemical mediator derived from macrophages.
She develops multiple organ failure. Which of the following mediators is most likely to produce these
findings?

A. Nitric oxide
B. Bradykinin
C. Histamine
D. Prostacyclin
E. Complement C3a

67 A 20-year-old woman sustains an injury to her right calf in a mountain biking accident. On physical
examination she has a 5 cm long laceration on the right lateral aspect of her lower leg. This wound is closed
with sutures. Wound healing proceeds over the next week. Which of the following factors will be most
likely to aid and not inhibit wound healing in this patient?

A. Commensal bacteria
B. Decreased tissue perfusion
C. Presence of sutures
D. Corticosteroid therapy
E. Hypoalbuminemia

68 A 24-year-old primigravida is late in the second trimester of pregnancy. She experiences the sudden
onset of some cramping lower abdominal pain. This is immediately followed by passage of some fluid per
vagina along with a foul-smelling discharge. The fetus is stillborn two days later. Examination of the
placenta demonstrates extensive neutrophilic infiltrates in the chorion and amnion. Which of the following
organisms is most likely to be responsible for these findings?

A. Mycobacterium tuberculosis
B. Herpes simplex virus
C. Escherichia coli
D. Treponema pallidum

69 A 19-year-old man incurs a stab wound to the chest. The wound is treated in the emergency room. Two
months later there is a firm, 3 x 2 cm nodular mass with intact overlying epithelium in the region of the
wound. On examination the scar is firm, but not tender, with no erythema. This mass is excised and
microscopically shows fibroblasts with abundant collagen. Which of the following mechanisms has most
likely produced this series of events?

A. Keloid formation
B. Development of a fibrosarcoma
C. Poor wound healing from diabetes mellitus
D. Foreign body response from suturing
E. Staphyloccocal wound infection

70 A 45-year-old man has had a fever and dry cough for 3 days, and now has difficulty breathing and a
cough productive of sputum. On physical examination his temperature is 38.5°C. Diffuse rales are
auscultated over lower lung fields. A chest radiograph shows a right pleural effusion. A right thoracentesis is
performed. The fluid obtained has a cloudy appearance with a cell count showing 15,500 leukocytes per
microliter, 98% of which are neutrophils. Which of the following terms best describes his pleural process?

A. Serous inflammation
B. Purulent inflammation
C. Fibrinous inflammation
D. Chronic inflammation
E. Granulomatous inflammation

71 A 52-year-old woman with no major medical problems takes a long airplane flight across the Pacific
Ocean. Upon arrival at Sydney's Kingsford Smith airport following the flight from Los Angeles, she cannot
put her shoes back on. There is no pain or tenderness. Which of the following is the most likely explanation
for this phenomenon?

A. Activation of Hageman factor has led to bradykinin production.


B. A lot of drinks were served in the first class section.
C. Femoral vein thrombosis developed.
D. A cellulitis developed in her legs.
E. Venous hydrostatic pressure became increased.

72 In an experiment, a lung tissue preparation is exposed to Mycobacterium tuberculosis organisms. Over


the next week, it is observed that granulomas form in the lung. Within the granuloma are found
inflammatory cells expressing class II MHC antigens. These cells elaborate cytokines that promote
fibroblastic production of collagen within the granulomas. From which of the following peripheral blood
leukocytes are these cells bearing class II antigen most likely to be derived?

A. Neutrophils
B. B cells
C. Monocytes
D. NK cells
E. Basophils

73 A 56-year-old man has had increasing difficulty breathing for the past week. On physical examination he
is afebrile. Auscultation of his chest reveals diminished breath sounds and dullness to percussion bilaterally.
There is 2+ pitting edema present to the level of his thighs. A chest radiograph reveals bilateral pleural
effusions. Which of the following laboratory test findings is he most likely to have?

A. Hypoalbuminemia
B. Glucosuria
C. Neutrophilia
D. Anemia
E. Hypernatremia

73. A 72-year-old man presents with a 3-day history of progressively worsening productive cough, fever,
chills, and signs of toxicity. Prominent physical findings include signs of consolidation and rales over the
right lung base. Sputum culture is positive for Streptococcus pneumoniae. An intra-alveolar exudate filling
the alveoli of the involved portion of the lung is present. Which of the following types of inflammatory cells
is most likely a prominent feature of this exudate?

Basophils
Eosinophils
Lymphocytes
Monocytes–macrophages
Neutrophils

75. A routine complete blood count performed on a 22-year-old medical student reveals an abnormality in
the differential

leukocyte count. She has been complaining of frequent sneezing and “watery” eyes during the past several
weeks and reports that she frequently had such episodes in the spring and summer. Which of the following
cell types is most likely to be increased?

Basophils
Eosinophils
Lymphocytes
Monocytes
Neutrophils

76. A 16-year-old boy presents with a 24-hour history of severe abdominal pain, nausea, vomiting, and low-
grade fever. The pain is initially periumbilical in location but has migrated to the right lower quadrant of the
abdomen, with maximal tenderness elicited at a site one-third of the way between the crest of the ileum and
the umbilicus (McBurney point). The leukocyte count is 14,000/mm3, with 74% segmented neutrophils and
12% bands. Surgery is performed. Which of the following describes the expected findings at the affected
site?

Fistula (abnormal duct or passage) connecting to the abdominal wall


Granulation tissue (new vessels and young fibroblasts) with a prominent infiltrate of eosinophils
Granulomatous inflammation with prominent aggregates of epithelioid cells and multinucleated giant cells
Massive infiltration of lymphocytes and plasma cells
Prominent areas of edema, congestion, and a purulent reaction with localized areas of abscess formation
77. A 2-year-old boy presents with recurrent infections involving multiple organ systems. Extensive
investigation results in a diagnosis of chronic granulomatous disease of childhood. Which of the following
most closely characterizes the abnormality in this patient’s phagocytic cells?

Decreased killing of microorganisms because of enhanced production of hydrogen peroxide


Deficiency of NADPH oxidase activity
Impaired chemotaxis and migration caused by abnormal microtubule formation
Inability to kill streptococci
Increased myeloperoxidase–halidemediated killing of catalase-positive organisms when compared with
catalase-negative organisms
78. A laboratory experiment is performed to evaluate the chemotactic potential of a group of potential
mediators. Which of the following substances most likely has the greatest affinity for neutrophils?

A. C5a
Fucosyl transferase
β2-Integrin
P-selectin
TNF-α

79. A 26-year-old African-American woman has bilateral hilar adenopathy, and radiography reveals
multiple reticular densities in both lung fields. A bronchoscopic biopsy reveals granulomatous inflammation
with
multiple giant cells of the Langhans type and no evidence of caseous necrosis. Which of the following is the
most likely diagnosis?

Aspergillosis
Coccidioidomycosis
Histoplasmosis
Sarcoidosis
Tuberculosis

80. In a laboratory exercise for medical students, an unknown compound is studied. The students are
informed that the compound has been isolated from endothelial cells and that its synthesis can be inhibited
by aspirin. In the laboratory, the students demonstrate that the compound is a potent vasodilator and platelet
antiaggregant. Given these findings, the substance is most likely which of the following mediators?

5-HPETE
LTC4
LXA4
PGI2
TxA2

81. A 70-year-old man presents with the sudden onset of left-sided weakness, spasticity, and hyperactive and
pathologic reflexes. The most serious consequences of this disorder are the result of damage to which of the
following cell types?

Labile cells
Multipotent adult progenitor cells
Permanent cells
Stable cells

ACUTE INFLAMMATION (Davydov) – 42 Qs

1.Inflammation of stomach is called:


A. Gastritis.
B. Enteritis.
C. Colitis.
D. Proctitis.
E. Duodenitis.

2.Inflammation of liver is called:


A. Gastritis.
B. Hepatitis.
C. Colitis.
D. Proctitis.
E. Duodenitis.

3.Inflammation of kidney is called:


A. Gastritis.
B. Hepatitis.
C. Nephritis.
D. Proctitis.
E. Duodenitis.

4.Inflammation of small intestine is called:


A. Gastritis.
B. Enteritis.
C. Colitis.
D. Proctitis.
E. Duodenitis.

5.Inflammation of large intestine is called:


A. Gastritis.
B. Enteritis.
C. Colitis.
D. Proctitis.
E. Duodenitis.

6.Inflammation of rectum is called:


A. Gastritis.
B. Enteritis.
C. Colitis.
D. Proctitis.
E. Duodenitis.
7.Inflammation of skin is called:
A. Gastritis.
B. Hepatitis.
C. Nephritis.
D. Proctitis.
E. Dermatitis.

8.Inflammation of blood vessels is called:


A. Gastritis.
B. Hepatitis.
C. Nephritis.
D. Vasculitis.
E. Dermatitis.

9.Inflammation of lung tissue is called:


A. Pneumonia.
B. Angina.
C. Furuncle.
D. Carbuncle.
E. Empyema.

10.Inflammation of throat is called:


A. Pneumonia.
B. Angina.
C. Furuncle.
D. Carbuncle.
E. Empyema.

11.Inflammation of hair follicle is called:


A. Pneumonia.
B. Angina.
C. Furuncle.
D. Carbuncle.
E. Empyema.

12.Inflammation of brain is called:


A. Gastritis.
B. Hepatitis.
C. Enteritis.
D. Epididimitis.
E. Encephalitis.

13.Local sign of inflammation “rubor” means:


A. Heat.
B. Pain.
C. Redness.
D. Swelling.
E. Loss of function.

14.Local sign of inflammation “calor” means:


A. Heat.
B. Pain.
C. Redness.
D. Swelling.
E. Loss of function.

15.Local sign of inflammation “dolor” means:


A. Heat.
B. Pain.
C. Redness.
D. Swelling.
E. Loss of function.

16.Local sign of inflammation “tumor” means:


A. Heat.
B. Pain.
C. Redness.
D. Swelling.
E. Loss of function.

17.Local sign of inflammation which associated with pressure on nerve endings


by exudate is:
A. Rubor.
B. Calor.
C. Dolor.
D. Tumor.
E.Functio laesa.

18.Local sign of inflammation which associated with accumulation of exudate is:


A. Rubor.
B. Calor.
C. Dolor.
D. Tumor.
E. Functio laesa.

19.Calor as a local sign of inflammation is associated with:


A. Arterial hyperemia.
B. Pressure of nerve endings.
C. Exudation.
D. Interference with enzyme function.
E. Local pain.
20.Tumor as a local sign of inflammation is associated with:
A. Arterial hyperemia.
B. Pressure of nerve endings.
C. Exudation.
D. Interference with enzyme function.
E. Local pain.

21.Dolor as a local sign of inflammation is associated with:


A. Arterial hyperemia.
B. Pressure of nerve endings.
C. Exudation.
D. Interference with enzyme function.
E. Local pain.

22.Rubor as a local sign of inflammation is associated with:


A. Arterial hyperemia.
B. Pressure of nerve endings.
C. Exudation.
D. Interference with enzyme function.
E. Local pain.

23.Component of alteratio stage of inflammation is:


A. Necrosis.
B. Arterial hyperemia.
C. Phagocytosis.
D. Regeneration of epithelial cells.
E. Regeneration of fibroblasts.

24.Component of alteratio stage of inflammation is:


A. Degeneration of cells.
B. Arterial hyperemia.
C. Phagocytosis.
D. Regeneration of epithelial cells.
E. Regeneration of fibroblasts.

25.Component of exudation stage of inflammation is:


A. Degeneration of cells.
B. Arterial hyperemia.
C. Necrosis.
D. Regeneration of epithelial cells.
E. Regeneration of fibroblasts.

26.Component of exudation stage of inflammation is:


A. Degeneration of cells.
B. Necrosis.
C. Formation of cell infiltrate.
D. Regeneration of epithelial cells.
E. Regeneration of fibroblasts.

27.Component of exudation stage of inflammation is:


A. Degeneration of cells.
B. Necrosis.
C. Regeneration of epithelial cells.
D. Phagocytosis.
E. Regeneration of fibroblasts.

28.Component of exudation stage of inflammation is:


A. Degeneration of cells.
B. Necrosis.
C. Pinocytosis.
D. Regeneration of epithelial cells.
E. Regeneration of fibroblasts.

29.Component of exudation stage of inflammation is:


A. Degeneration of cells.
B. Necrosis.
C. Regeneration of epithelial cells.
D. Regeneration of fibroblasts.
E. Leakage of blood plasma from vessels.

30.Component of exudation stage of inflammation is:


A. Degeneration of cells.
B. Necrosis.
C. Regeneration of epithelial cells.
D. Regeneration of fibroblasts.
E. Emigration of blood cells from vessels.

31.Component of proliferation stage of inflammation is:


A. Degeneration of cells.
B. Necrosis.
C. Regeneration of epithelial cells.
D. Emigration of blood cells from vessels.
E. Leakage of blood plasma from vessels.

32.Component of proliferation stage of inflammation is:


A. Degeneration of cells.
B. Necrosis.
C. Regeneration of fibroblasts.
D. Emigration of blood cells from vessels.
E. Leakage of blood plasma from vessels.
33.Type of acute inflammation which associated with formation of protein-poor fluid
is called:
A. Serous.
B. Fibrinous.
C. Purulent.
D. Hemorrhagic.
E. Catarrhal.

34.Type of acute inflammation which associated with formation of hair-like deposits


is called:
A. Serous.
B. Fibrinous.
C. Purulent.
D. Hemorrhagic.
E. Catarrhal.

35.Type of acute inflammation which associated with formation of pseudomembrane on mucosal surfaces is
called:
A. Serous.
B. Fibrinous.
C. Purulent.
D. Hemorrhagic.
E. Catarrhal.

36.Type of acute inflammation which associated with formation of thick turbid


yellow-green fluid is called:
A. Serous.
B. Fibrinous.
C. Purulent.
D. Hemorrhagic.
E. Catarrhal.

37.Which type of pneumonia acute is associated with formation of fibrinous exudate:


A. Lobar pneumonia.
B. Bronchpneumonia.
C. Interstitial pneumonia.
D. Septic pneumonia.
E. Atypical pneumonia.

38.Localized type of purulent inflammation is called:


A. Phlegmon.
B. mpyema.
C. Abscess.
D. Furuncle.
E. Carbuncle.

39.Diffuse type of purulent inflammation is called:


A. Phlegmon.
B. Empyema.
C .Abscess.
D. Furuncle.
E. Carbuncle.

40.Type of purulent inflammation with accumulation of pus in serosal cavities and hollow organs is called:
A. Phlegmon.
B. Empyema.
C. Abscess.
D. Furuncle.
E. Carbuncle.

41.Type of purulent inflammation with accumulation of pus in serosal cavities and hollow organs is called:
A. Phlegmon.
B. Empyema.
C. Abscess.
D .Furuncle.
E. Carbuncle.

42.Purulent inflammation of hair follicle is called:


A. Phlegmon.
B. Empyema.
C. Abscess.
D. Furuncle.
E. Carbuncle.

1. Which of the following are thought to mediate, many of the systemic effects of inflammation are
chemotactic and stimulate adhesion molecules:

Interleukin-1 (IL-1) and tumor necrosis factor.


C5 a and leukotriene B-4.
C3 b.
Leukotriene C4, D4 and E4.
Bradykinin.

2. After initiation of an acute inflammatory process third in a sequence of changes in vascular flow is:

Vasoconstriction.
Redness.
Leukocytic migration.
Vasodilation.
Slowing of the circulation.

3. The term “croupous” inflammation is synonymous with

Catarrhal inflammation
Fibrinous inflammation
Sero-fibrinous inflammation
Suppurative inflammation
None of these

4. Vascular phenomenon in acute inflammation helps in:

Bringing the defensive cells like neutrophils from interior of the vessels to the site of irritation
Minimizing the effect of irritant by diluting it considerably
Limiting the inflammatory reaction to the smallest possible area
Early healing of the affected tissue
All of these

5. Which of the complement components act as chemokines?

C3b
C4b
C5a
C4a

6. All are types of tissue macrophages, except:

Littoral cells
Hoffbauer cells
Osteoclasts
Osteoblasts

7. Main cytokines acting as mediators of inflammation are as under, except:

Interleukin-1 (IL-1)
Tumor necrosis factor α (TNF-α)
Nitric oxide (NO)
Interferon-γ (IF-γ)

8. Typhoid fever is an example of:

Acute inflammation
Chronic nonspecific inflammation
Chronic granulomatous inflammation
Chronic suppurative inflammation

9. Which one of the listed statements is the best histologic definition of an abscess?

A circumscribed collection of neutrophils with necrotic cellular debris


A localized defect that results from the sloughing of necrotic of necrotic inflammatory tissue from the
surface of an organ
A localized proliferation of fibroblasts and small blood vessels
An aggregate of two or more activated macrophages
The excessive secretion of mucus from a mucosal surface

10. The cardinal sign of inflammation called rubor is mainly the result of:

Decreased interstitial hydrostatic pressure


Decreased vascular permeability of capillaries
Increased vascular permeability of venules
Vasoconstriction of muscular arteries
Vasodilatation of arterioles

11. During the early stages of the inflammatory response, histamine-induced increased vascular permeability
is most likely to occur in

Arteries
Precapillary arterioles
Capillaries
Postcapillary venules
Veins

12. Which one of the listed statements best describes the process called chemotaxis?

Abnormal fusion of phagosomes to primary lysosomes


Attachment of chemicals to extracellular material to increase phagocytosis
Dilation of blood vessels by chemotherapeutic drugs
Movement of cells toward a certain site or source
Transmigration of cells from blood vessel into tissue

13. During acute inflammation, histamine-induced increased vascular permeability causes the formation of
exudates (inflammatory edema). Which one of the listed cell types is the most likely source of the histamine
that causes the increased vascular permeability?

Endothelial cells
Fibroblasts
Lymphocytes
Mast cells
Neutrophils

14. What type of leukocyte actively participates in acute inflammatory processes and contains
myeloperoxidase within its primary (azurophilic) granules and alkaline phosphatase in its secondary
(specific) granules?

Neutrophils
Eosinophils
Monocytes
Lymphocytes
Plasma cells

15. During the inflammatory response, the proper order of white cell events is:

Endothelial adherence, margination, phagocytosis. chemotaxis


Margination, leukodiapedesis, chemotaxis, phagocytosis
Marginalion, migration, chemotaxis, stasis
Stasis, leukodiapedesis, margination phagocytosis
Leukodiapedesis, stasis, degranulation

16. The most reliable evidence of purulent inflammation is the presence in tissue of which of the following:

Lymphocytes
Cellular necrosis
Intracellular pigment accumulations
Neutrophils
Plasma cells

17. Fluid that collects during acute inflammation and that has a protein content exceeding 3 g/dl and a
specific gravity exceeding 1.015 is referred to as:

Oedema
Effusion
Transudate
Serum
Exudate

18. The adherence of neutrophils and monocytes to the vascular endothelium prior to movement into the
extravascular space is called:

Margination
Diapedesis
Pavementing
Emigration
Clotting

19. Cells that are capable of phagocytosis of particulate matter include which of the following:

Neutrophils, macrophages
Lymphocytes, mast cells
T-cells, NK-cells
Basophils, stem cells
Endothelial cells, plasma cells
20. The unidirectional migration of leukocytes toward a target is referred to as:

Diapedesis
Chemotaxis
Opsonization
Endocytosis
Margination

21. All of the following statements describing leukocyte emigration from vessels in areas of inflammation
are true, except:

Leukocytes pass through gaps between the vascular endothelial cells


Neutrophils are the first cells to emigrate
Lcukocyles develop pseudopods to aid iemigration
Lipofuscin accumulation accompanies leukocyte emigration
Accompanying loss of red cells is passive

22. In an inflammatory response, neutrophils release molecules that induce all of the following effects,
except:

Chemotaxis of monocytes
Chemotaxis of lymphocytes
Degranulation of mast cells
Increased vascular permeability independent of histamine release
Connective tissue digestion

23. Mediators of increased vascular permeability in acute inflammatory responses include all of the
following, except:

Leukotriene E4
Complement complex C5b-9
Leukotriene C4
Bradykinin
Platelet-activating factor

24. The first cells to arrive at the injured area in the inflammatory response are which of the following:

Neutrophils
Fibroblasts
Lymphocytes
Macrophages
Frythrocytes

25. Lysosomes within neutrophils contain all of the following enzymes, except:
Myeloperoxidase
Acid hydrolases
Proteases
Interleukin-2
Cationic proteins

26. Acute Inflammation is characterized by all of the following features, except:

Relatively short duration


Immigration of lymphocytes into the injured area
Immigration of leukocytes into the injured area
Exudation of fluid
Exudation of plasma proteins

27. Major functions of macrophages are all of the following, except:

Phagocyte function
IL-1-synthesizing function
Secretory function
Antibody-synthesizing function
Healing and repair function

28. The inflammatory response leads to all of the following, except:

Isolation of infected tissues


Inactivation of causative agents
Neutralization of toxins
Removal of devitalized tissue debris
Obesity

29. Acute inflammation includes all of the following types, except:

Purulent
Fibrinous
Granulomatous
Serous
Catarrhal

30. Cardinal signs of acute inflammation include all of the following, except:

Local heat
Redness
Pallor
Swelling
Pain
31. "Hairy heart" is an example of which of the following types of inflammation:

Purulent
Fibrinous
Serous
Catarrhal
Hemorrhagic

32. Purulent inflammation is characterized by which of the following:

Neutrophil infiltration with tissue lysis


Fibrin deposition
Mucus production
Lymphocyte infiltration
Neutrophil infiltration

33. The most common formation in the body cavities and the spinal fluid is which of the following:

Fibrinous inflammation
Serous inflammation
Catarrhal inflammation
Hemorrhagic inflammation
Granulomatous inflammation

34. Fibrinous pericarditis occurs in all diseases, except:

Acute rheumatic fever


Systemic lupus erythematosus
Uremia
Atherosclerosis
Acute myocardial infarction

35. Lobar pneumonia is an example of which of the following types of inflammation:

Purulent
Fibrinous
Serous
Catarrhal
Granulomatous

36. Abscess of the lung is an example of which of the following types of inflammation:

Purulent
Fibrinous
Serous
Catarrhal
Granulomatous

37. Acute gastritis with abundant mucus production is an example of which of the following types of
inflammation:

Purulent
Fibrinous
Serous
Catarrhal
Granulomatous

СHRONIC INFLAMMATION (Davydov – 32 Qs)

1.Cells of chronic inflammation are:

A. Neutrophils.
B. Erythrocytes.
C. Monocytes.
D. Myelocytes.
E. Adipocytes.

2.Cells of chronic inflammation are:

A. Lymphocytes.
B. Erythrocytes.
C. Neutrophils.
D. Myelocytes.
E. Adipocytes.

3.Cells of chronic inflammation are:

A. Neutrophils.
B. Plasma cells.
C. Erythrocytes.
D. Myelocytes.
E. Adipocytes.

4.Cells of chronic inflammation are:


A. Neutrophils.
B. Erythrocytes.
C. Myelocytes.
D. Adipocytes.
E. Eosinophils.

5.Cells of chronic inflammation are:

A. Neutrophils.
B. Erythrocytes.
C. Astrocytes.
D. Giant cells.
E. Osteocytes.

6.Cells of chronic inflammation are:

A. Neutrophils.
B. Epitelioid cells.
C. Epithelial cells.
D. Astrocytes.
E. Osteocytes.

7.Cells of chronic inflammation are:

A. Neutrophils.
B. Erythrocytes.
C. Mast cells.
D. Epithelial cells.
E. Chondrocytes.

8.Cells of chronic inflammation are:

A. Fibrolasts.
B. Osteoblasts.
C. Chondroblasts.
D. Lipoblasts.
E. Lymphoblasts.

9.Functions of macrophages are all the following, EXCEPT:

A. Phagocytosis.
B. Degradation of engulfed particles.
C. Tissue destruction.
D. Forming of giant cells.
E. Producing of mediators.

10.Functions of macrophages are all the following, EXCEPT:

A. Phagocytosis.
B. Degradation of engulfed particles.
C. Tissue destruction.
D. Syntesis og collagen.
E. Forming of giant cells.

11.Functions of macrophages are all the following, EXCEPT:

A. Phagocytosis.
B. Degradation of engulfed particles.
C. Producing of antibodies.
D. Tissue destruction.
E. Forming of giant cells.

12.Functions of macrophages are all the following, EXCEPT:

A. Phagocytosis.
B. Regeneration of epithelial cells.
C. Degradation of engulfed particles.
D. Tissue destruction.
E. Forming of giant cells.

13.Which cells are associated with producing of mediators?

A. Lymphocytes.
B. Plasma cells.
C. Eosinophils.
D. Must cells.
E. Fibroblasts.

14.Which cells are characteristically found in inflammatory sites around animal parasites?

A. Lymphocytes.
B. Plasma cells.
C. Eosinophils.
D. Must cells.
E. Fibroblasts.

15.Which cells are central players in anaphylactic shock?

A. Lymphocytes.
B. Plasma cells.
C. Eosinophils.
D. Must cells.
E. Fibroblasts.

16.Which cells are collagen producing?

A. Lymphocytes.
B. Plasma cells.
C. Eosinophils.
D. Must cells.
E. Fibroblasts.

17.Infective granulomas occur in:

A. Tuberculosis.
B. Rheumatic fever.
C. Rheumatoid arthritis.
D. Sarcoidosis
E. Regional ileitis.

18.Infective granulomas occur in:

A. Rheumatic fever.
B. Syphilis.
C. Rheumatoid arthritis.
D. Sarcoidosis
E. Regional ileitis.

19.Infective granulomas occur in:

A. Rheumatic fever.
B. Rheumatoid arthritis.
C. Leprosy.
D. Sarcoidosis
E. Regional ileitis.

20.Infective granulomas occur in:

A. Rheumatic fever.
B. Rheumatoid arthritis.
C. Sarcoidosis.
D. Typhoid fever.
E. Regional ileitis.

21.Allergic granulomas occur in:

A. Tuberculosis.
B. Syphilis.
C. Rheumatic fever.
D. Typhoid fever.
E. Leprosy.

22. Allergic granulomas occur in:

A. Tuberculosis.
B. Syphilis.
C. Rheumatoid arthritis.
D. Typhoid fever.
E. Leprosy.

23.Granulomas with unknoun etiology occur in:

A. Tuberculosis.
B. Syphilis.
C. Rheumatic fever.
D. Sarcoidosis.
E. Leprosy.

24.Granulomas with unknoun etiology occur in:

A. Tuberculosis.
B. Syphilis.
C. Rheumatic fever.
D. Regional ileitis.
E. Leprosy.

25.Interstitial inflammation is characterized by:

A. Formation of granulomas.
B. Inflammatory infiltration of the stroma of inner organs.
C. Formation of pseudopolyps.
D. Cell degeneration of inner organs.
E. Formation of condylomas.

26.Typical localization of interstitial inflammation is all the following, EXCEPT:

A. Liver.
B. Kidneys.
C. Heart.
D. Lungs.
E. Spleen.

27.Typical localization of interstitial inflammation is all the following, EXCEPT:

A. Liver.
B. Kidneys.
C. Heart.
D. Lungs.
E. Lymph nodes.

28.Typical localization of interstitial inflammation is all the following, EXCEPT:

A. Liver.
B. Kidneys.
C. Heart.
D. Lungs.
E. Stomach.

29.Typical localization of inflammatory polyps is:

A. Small intestine.
B. Large intestine.
C. Rectum.
D. Duodenum.
E. Appendix.

30.Typical localization of inflammatory polyps is:

A. Stomach.
B. Small intestine.
C. Large intestine.
D. Duodenum.
E. Appendix.

31.Typical localization of inflammatory polyps is:

A. Oral cavity.
B. Nasal cavity.
C. Pleural cavity.
D. Peritoneal cavity.
E. Joint cavity.

32.Inflammatory wart-like lesion is called:

A. Papilloma.
B. Adenoma.
C. Condyloma.
D. Pseudopolyp.
E. Carcinoma.
(Kozmina - 48)

1. Morphologic changes seen in chronic non-specific inflammation include an increase in:

Neutrophils, lymphocytes and liquefaction necrosis.


Neutrophils, macrophages and fibrosis.
Lymphocytes, plasma cells and fibrosis.
Giant cells, macrophages and coagulative necrosis.

2. In chronic inflammation, macrophages accumulate in large number in the following ways:

Continuous inflow of monocytes from circulation maintained by chemotactic factors.


Local proliferation of macrophages by mitotic division
Prolonged immobilization of macrophages within the site of inflammation
A and B
All the three

3. Formation of granuloma is:

Type I hypersensitivity reaction


Type II hypersensitivity reaction
Type III hypersensitivity reaction
Type IV hypersensitivity reaction
All of these

4. Granuloma formation is most frequently associated with:

The healing process.


Acute inflammation.
Wound contraction.
Fibroblasts and neovascularization.
A persistent irritant.

5. An example of chronic fibrinoid inflammation is:

Chronic cervicitis
Chronic gastritis
Chronic cholecystitis
None of these
All of these
6. Epithelioid cells in tubercular granuloma are derived from:

Neutrophils
Eosinophils
Lymphocytes
Monocytes
Macrophages

7. Epithelioid cell is modified –

Lymphocytes
Macrophages
Mast cell
Eosinophils
Neutrophils

8. Granuloma is seen in all, except:

TB
Yersinia
Mycoplasma
Leprosy
Syphilis

9. Which of the following is atypical mycobacterium?

Mycobacterium microti
Mycobacterium canneti
Mycobacterium africanum
Mycobacterium ulcerans

10. IgM antibody against PGL-1 antigen is used for the diagnosis of:

Leprosy
Tuberculosis
Syphilis
Brucellosis
Mycoplasmosis

11. Which category of leprosy is not included in Ridley-Jopling classification?

Mid borderline leprosy


Borderline tuberculoid leprosy
Inderterminate leprosy
Tuberculoid polar leprosy
Borderline lepromatous

12. Hepar lobatum is seen in:

Primary syphilis
Secondary syphilis
Tertiary syphilis
Congenital syphilis

13. Killing of M.tuberculosis that grows within the macrophage is brought about by the following
mechanisms:

By reactive oxygen species


By oxygen-independent bactericidal mechanism
By nitric oxide mechanism
By hydrolytic enzymes

14. Tubercle bacilli cause lesions by the following mechanisms:

Elaboration of endotoxin
Elaboration of exotoxin
Type IV hypersensitivity
Direct cytotoxicity

15. The following statements are correct for tubercle bacilli, except:

Tubercle bacilli can be cultured


Tubercle bacilli are anaerobe
Tubercle bacilli thrive best in the apex of lung
M.smegmatis is not pathogenic to man

16. Tubercle bacilli in caseous lesions are best demonstrated in:

Caseous centre
Margin of necrosis with viable tissue
Epithelioid cells
Langhans′ giant cells

17. Leprosy bacilli are:

Not acid fast


As acid fast as tubercle bacilli
Less acid fast compared to tubercle bacilli
More acid fast compared to tubercle bacilli
18. Lepromin test is always positive in:

Lepromatous leprosy
Borderline lepromatous leprosy
Tuberculoid leprosy
Inderterminate leprosy

19. Spirochetes are most difficult to demonstrate in:

Primary syphilis
Secondary syphilis
Tertiary syphilis
Congenital syphilis

20. Actinomycosis is caused by:

Fungus
Gram-negative bacteria
Anaerobic bacteria
Acid fast bacteria

21. Typically, sarcoid granuloma has the following features, except:

Non caseating granuloma


Giant cells have cytoplasmic inclusions
Peripheral mantle of lymphocytes
Fibroblastic proliferation at the periphery of a granuloma

22. By definition, granulomas are composed of

Cholesterol clefts
Collagen
Endothelial cells and fibroblasts
Epithelioid cells
Hemosiderin-laden macrophages

23. Finding acid-fast bacilli within peripheral nerves is most suggestive of

Relapsing fever
Syphilis
Leprosy
Tuberculosis
Weil′s disease
24. A 21-year-old college athlete presents with a nagging cough and a 20-lb weight loss. In addition to the
chronic cough and weight loss, his main symptoms consist of fever, night sweats, and chest pains.
Examination of his sputum reveals the presence of rare acid-fast organisms. His symptoms are most likely
due to an infection with

K. pneumoniae
L. pneumophila
Mycobacterium avium-intracellulare
Mycobacterium tuberculosis
Mycoplasma pneumonia

25. The most reliable histopathologic evidence of chronicity in an inflammatory process in organs is which
of the following:

Hemorrhages
Leucocytic infiltrates
Blood vessels destruction
Interstitial fibrosis
Councilman's bodies

26. A large aggregate of epithelioid cells is seen in a microscopic section of an ovary removed at surgery.
Your diagnosis is:

Granulation tissue
Pyogenic granuloma
Granulosa cell tumor
Granulocytosis
Granuloma

27. The granuloma in primary tuberculosis is composed predominantly of which of the following:

Fibroblasts
Epithelioid cells
Eosinophils
Plasma cells
Neutrophils

28. The granulomatous cell infiltrate in primary syphilis is composed predominantly of which of the
following:

Neutrophils
Monocytes/macrophages
Plasma cells
Eosinophils
Lymphocyles

29. The miliary lung tuberculosis is characterized by which type of inflammation:

Granulomatous
Serous
Fibrinous
Suppurative
Hemorrhagic

30. Most comprehensively the chronic inflammation is characterized by which of the following:

Infiltration with mononuclear cell including macrophages, lymphocytes, and plasma cells
Tissue destruction
Healing by connective tissue with angiogenesis and fibrosis
All of these
None of these

31. The possible causes of chronicity of inflammation are all of the following, except:

Persistent infections by certain microorganisms


Prolonged exposure to potentially toxic agents, either exogenous or endogenous
Autoimunity (autoimmune diseases)
Complete phagocytosis
Resistance of the etiologic agent

32. What cells play the most important role in chronic tuberculosis inflammation?

Macrophages
Leucocytes
Eosinophils
Erythrocytcs
Plasma cells

33. All of these cells can be found in chronic inflammation infiltrate, except:

Lymphocytes
Platelets
Macrophages
Plasma cells
Eosinophils

34. Granulomatous inflammation develops in all of the following diseases, except:

Tuberculosis
Leprosy
Syphilis
Cat-scratch disease
Budd-Chiary syndrome

35. What type of necrosis can be found in tuberculosis granuloma?

Coagulation necrosis
Liquefactive necrosis
Caseous necrosis
Enzymatic fat necrosis
Fibrinoid necrosis

36. Typical tyberculous granuloma is characterized by all of the following, except:

Plasma cells
Area of central necrosis
Epithelioid cells
Langhans-type giant cells
Lymphocytes

37. Syphilis granuloma is also called as:

Fibroma
Gumma
Tuberculoma
Leproma
Hepatoma

38. Typical syphilis granuloma is characterized by all of the following, except:

Area of central necrosis


Plasma cell infiltrate
Lymphocyte infiltrate
Productive vasculitis
Platelet infiltrate

39. Macrophages in granulomatous inflammation can transform into which of the following cells:

Monocytes
Epithelial cells
Epithelioid cells
Plasma cells
Lymphocytes
40. Gummatous infiltrate in tertiary syphilis can be found in which of the following organs:

Aorta
Testes
Liver
Bones and joints
Skin and subcutaneous tissue

41. On gross inspection, syphilitic gumma is characterized by all of the following features, except:

White-gray
Rubbery
Solitary
Red-brown
Tumor-like

42. Products released by the activated macrophages that lead to tissue injury, include all of the following,
except:

Fibrogenic cytokines
Toxic oxygen metabolites
Collagenases
Neutrophile chemotactive factors
Elastase

43. Products released by the activated macrophages, that lead to fibrosis include all of the following, except:

Growth factors
Fibrogenic cytokines
Angiogenesis factors
Fibronectin
Proteases

44. The lungs with multiple tuberculous granulomas are called as:

Tuberculous pneumonia
Brown induration of lungs
Miliary tuberculosis
Cavitary fibrocaseous tuberculosis
Tuberculoma

45. The pathologic changes of vasa vasorum of aorta in syphilitic mesaortitis are characterized by which of
the following:

Migratory thrombophlebitis
Productive vasculitis (obliterative endoarteritis)
Thromboangitis obliterans
Necrotizing arteriolitis
Thrombotic microangiopathy

46. The medial destruction of aorta in tertiary syphilis may lead to which of the following:

Aneurismal dilatation of the aorta


Marian's syndrome
Atherosclerosis
Takayasu's arteritis
Giant cell arteritis

47. Sarcoidisis development is associated with which of the following:

Mycobaclerium leprae
Mycobacterium tuberculosis
Treponema pallidum
Gram-negative bacillis
Unknown

48. The foreign-body granulomas are caused by all the following, except:

Paniculate matter
Synthetic material
Gram-negative bacillus
Vegetable matter
Beryllium particles

ADAPTATION (Davydov - 24 Qs)

1.Type of cellular adaptation which occurs in decrease of cell size is called:

A. Atrophy.

B. Hypertrophy.

C. Hyperplasia.

D. Metaplasia.

E. Dysplasia.
2.Type of cellular adaptation which occurs in increase of cell size is called:

A. Atrophy.

B. Hypertrophy.

C. Hyperplasia.

D. Metaplasia.

E. Dysplasia.

3.Type of cellular adaptation which occurs in increase in cell number is called:

A. Atrophy.

B. Hypertrophy.

C. Hyperplasia.

D. Metaplasia.

E. Dysplasia.

4.Type of cellular adaptation which occurs in change of cell type is called:

A. Atrophy.

B. Hypertrophy.

C. Hyperplasia.

D. Metaplasia.

E. Dysplasia.

5.Mechanisms of adaptive response involve all of the following, EXCEPT:

A. Up-regulation of specific cellular receptors.

B. Down-regulation of specific cellular receptors.

C. Alteration in signal for protein synthesis.

D. Induction of new protein synthesis by the target cell.

E. Reduction of new protein synthesis by the target cell.

6.Mechanisms of adaptive response involve all of the following, EXCEPT:

A. Up-regulation of specific cellular receptors.


B. Down-regulation of specific cellular receptors.

C. Alteration in signal for protein synthesis.

D. Alteration in signal for lipid synthesis.

E. Induction of new protein synthesis by the target cell.

7.One of variants of physiologic atrophy is:

A. Atrophy of brain in atherosclerosis.

B. Atrophy of thymus in adults.

C. Atrophy of skeletal muscle by immobilized broken limb.

D. Atrophy of kidney from pressure by stones.

E. Atrophy of thyroid gland in disfunction of pituitary gland.

8.Mechanisms of adaptive response involve all of the following, EXCEPT:

A. Up-regulation of specific cellular receptors.

B. Down-regulation of specific cellular receptors.

C. Alteration in signal for protein synthesis.

D. Alteration in signal for carbohydrate synthesis.

E. Induction of new protein synthesis by the target cell.

9.Atrophy of organ due to prolonged diminished of functional activity is called:

A. Neuropathic atrophy.

B. Endocrine atrophy.

C. Ischemic atrophy.

D. Disuse atrophy.

E. Pressure atrophy.

10.Atrophy of organ due to gradual diminished of blood supply is called:

A. Neuropathic atrophy.

B. Endocrine atrophy.
C. Ischemic atrophy.

D. Disuse atrophy.

E. Pressure atrophy.

11.Atrophy of organ with no obvious cause is called:

A. Idiopathic atrophy.

B. Endocrine atrophy.

C. Ischemic atrophy.

D. Disuse atrophy.

E. Pressure atrophy.

12.Which pigment can be found in cytoplasm of heart and liver in general atrophy?

A. Melanin.

B. Hemosiderin.

C. Bilirubin.

D. Ferritin.

E. Lipofuscin.

13.Which pigment can be accumulated in subcutaneous fat in general atrophy?

A. Lipochrome.

B. Lipofuscin.

C. Bilirubin.

D. Ferritin.

E. Hemosiderin.

14.General atrophy (cachexia) due to brain pathology is called:

A. Alimentary.

B. Endocrine.

C. Cerebral.
D. Cancerous.

E. Infectious.

15.General atrophy (cachexia) due to starvation is called:

A. Alimentary.

B. Endocrine.

C. Cerebral.

D. Cancerous.

E. Infectious.

16.Morphological signs of atrophy are, EXCEPT:

A. Shrinkage in cell size.

B. Reduction of myofilaments.

C. Reduction of endoplasmic reticulum.

D. Cell dead.

E. Reduction of mitochondria.

17.One of variants of physiologic hypertrophy is:

A. Enlargement of uterus in leiomyoma.

B. Enlargement of uterus in leiomyosarcoma.

C. Enlargement of uterus in ovarian tumors.

D. Enlargement of uterus in pregnancy.

E. Enlargement of uterus in endomyometritis.

18. Concentric cardiac hypertrophy is characterized by all of the following, EXCEPT:

A. Hypertrophy of left ventricle.

B. Increased wall thickness.

C. Normal left cavity diameter.

D. Reduced left cavity diameter.


E. Dilated left cavity diameter.

19. Excentric cardiac hypertrophy is characterized by all of the following, EXCEPT:

A. Hypertrophy of left ventricle.

B. Increased wall thickness.

C. Normal left cavity diameter.

D. Dilated borders of heart.

E. Reduced left cavity diameter.

20.Hypertrophy of one of paired organ after removal of another organ is called:

A. Concentric.

B. Excentric.

C. Controlateral.

D. Endocrine.

E. Overworkloading.

21.General signs of hypertrophy are, EXCEPT:

A. Enlargement of cells.

B. Increased synthesis of DNA.

C. Decreased synthesis of RNA.

D. Increased protein synthesis.

E. Increased number of organelles.

22.Endometrial hyperplasia is a result of:

A. Chronic inflammation.

B. Hormonal stimulation by estrogens.

C. Hormonal stimulation by thyroid stimulating immunoglobulins.

D. Acute inflammation.

E. Hormonal stimulation by progesteron.


23. Hyperplasia of epidermis and formation of skin warts is a result of:

A. Hormonal stimulation by estrogens.

B. Hormonal stimulation by progesteron.

C. Hormonal stimulation by thyroid stimulating immunoglobulins.

D. Chronic inflammation.

E. Acute inflammation.

24. Hyperplasia of hepatic cells that occurs after partial hepatoectomy is an example of:

A. Compensatory hyperplasia.

B. Hormonal hyperplasia.

C. Inflammatory hyperplasia.

D. Physiologic hyperplasia.

E. Controlateral hyperplasia.

(Kozmina – 24 qs )

1. The cellular adaptation without cell proliferation includes all of the following, EXCEPT:

Apoptosis
Atrophy
Hyperplasia
Hypertrophy
Metaplasia

2. Hyperplasia is characterized by which of the following:

Increase in the size of cells


Increase in the number of cells
Increase in the number of nuclei in cells
Shrinkage in the size of cells
Atypia of cells

3. The proliferation of the glandular epithelium of a female during pregnancy is an example of:

Compensatory hyperplasia
Pathologic hyperplasia
Hormonal hyperplasia
Compensatory hypertrophy
Hormonal hypertrophy

4. The hyperplasia of hepatic cells that occurs after partial hepatoectomy is an example of:

Pathologic hyperplasia
Hormonal hyperplasia
Hormonal hypertrophy
Compensatory hyperplasia
Compensatory hypertrophy

5. Conditions leading to endometrial hyperplasia include all of the following, EXCEPT:

Polycystic ovarian disease


Functioning granulosa cell tumors of the ovary
Prolonged administration of estrogenic substances
Excessive ovarian cortical function
Prolonged administration of analgesic substances

6. The most common clinical manifestation of endometrial hyperplasia is:

Abnormal uterine bleeding


Menses stopping
Pains
Purulent discharges
Mucous discharges

7. Simple endometrial hyperplasia is characterized by all of the following, EXCEPT:

Increase in the number and size of endometrial glands


Atypia of gland cells
Complex endometrial glands
Increase in gland-to-stroma ratio
Dilated endometrial glands

8. Hypoplasia is characterized by all of the following, EXCEPT:

Incomplete development of an organ


Decreased number of cells
Increased number of cells
Underdevelopment of an organ
Decreased function of an organ
9. Dysplasia is characterized by all of the following, EXCEPT:

Abnormal organization of cells


Loss in the uniformity of individual cells
Loss in cell architectural organization
Replacement of one adult cell type by another adult cell type
Variation of cells in size and shape

10. Which pathologic process results from dysplasia?

Aplasia
Hypoplasia
Hyperpigmentation
Calcification
Neoplasia

11. Hypertrophy as a process is characterized by which of the following:

Increase in the size of cells and of the organ


Shrinkage in the size of cells and of the organ
Increase in the number of cells
Abnormal organization of cells
Variation of cells in size and shape

12. The massive growth of the gravid uterus with large plump cells is an example of:

Pathologic hypertrophy
Hormone induced physiologic hypertrophy
Pathologic hyperplasia
Dysplasia
Metaplasia

13. Hypertrophy as an adaptive response is characterized by which of the following:

Pathologic hypertrophy of the breast during lactation


Hypertrophy of the skeletal muscle cells in a body-builder
Pathologic hypertrophy of the uterus during pregnancy
Hypertrophy of the skeletal muscle cells in 1 patient with immobilized broken limb
Hypertrophy of the endometrium due to ovarian tumor
14. Cardiac hypertrophy is characterized by all pathologic changes, EXCEPT:

Increased number of myocardial cells


Increased mass and size of the heart
Increased protein synthesis
Interstitial cardiac fibrosis
Inadequate vasculature

15. Atrophy is characterized by which of the following:

Variation of cells in size and shape


Increase in the size of cells
Shrinkage in the size of cells by loss of cell substance
Increase in the number of cells
Abnormal organization of cells

16. One of the variants of physiologic atrophy is:

Atrophy of skeletal muscle by the immobilized broken limb


Atrophy of uterus after parturition
Kidney atrophy from pressure (hydronephrosis)
Atrophy of the endometrium by ovarian tumor
Atrophy of the brain in atherosclerosis

17. The causes of pathologic atrophy are all of following, EXCEPT:

Loss of endocrine stimulation


Loss of innervation
Diminished blood supply
Decreased workload
Increased workload

18. The causes of pathologic atrophy are all of the following, EXCEPT:

Aging
Intracellular fat accumulation
Pressure
Inadequate nutrition
Denervation

19. What pigment can be found in the cytoplasm of heart and muscle cells in aging atrophy?

Lipofuscin
Melanin
Hemosiderin
Bilirubin
Ferritin

20. Metaplasia is characterized by which of the following:

Reversible increase in the size of cells


Irreversible change in which one adult cell type is replaced by another adult cell type
Reversible change in which one adult cell type is replaced by another adult cell type
Reversible abnormal organization of cells
Reversible increase in the number of cells

21. What type of metaplasia may occur in the respiratory tract in habitual cigarette smoker?

Epithelial metaplasia; squamous to columnar


Metaplasia to undifferentiated mesenchymal cells
Connective tissue metaplasia
Epithelial metaplasia: columnar to squamosus
None of these

22. The replacement of the normal secretory columnar epithelium by the nonfunctioning-stratified squamous
epithelium may occur in all organs of the following, EXCEPT:

Bile ducts of the liver


Excretory ducts of the salivary glands
Excretory ducts of the pancreas
Respiratory epithelium of the bronchi
Tubular epithelium of the kidney

23. Hydronephrosis is characterized by all of the following, EXCEPT:

Thickening of the renal parenchyma


Thinning of the renal parenchyma
Dilatation of the renal pelvis
Dilatation of the renal calyces
Progressive atrophy of the kidney

24. In urinary tract obstruction all pathologic processes can be found, EXCEPT:

Dilatation of the pelvis and calyces


Acute tubular necrosis
Interstitial inflammation
Interstitial fibrosis
Glomerular and tubular atrophy

Tissue Repair (Tugolbai - 18 Qs)

1 A 74-year-old woman presents with acute chest pain and shortness of breath. Cardiac catheterization demonstrates
occlusion of the left anterior descending coronary artery. Laboratory studies and ECG are consistent with acute
myocardial infarction. Which of the following is the most likely pathologic finding in the affected heart muscle 4
weeks later?

Capillary-rich granulation tissue


Collagen-rich scar tissue
Granulomatous inflammation
Neutrophils and necrotic debris
Vascular congestion and edema

2 A 4-year-old boy falls on a rusty nail and punctures his skin. The wound is cleaned and covered with sterile gauze.
Which of the following is the initial event in the healing process?

Accumulation of acute inflammatory cells


Deposition of proteoglycans and collagen
Differentiation and migration of myofibroblasts
Formation of a fibrin clot
Macrophage-mediated phagocytosis of cellular debris

3 An 82-year-old man dies 4 years after developing congestive heart failure. He had a history of multiple myocardial
infarcts over the past 10 years. A trichrome stain of heart muscle at autopsy is shown in the image. What is the
predominant type of collagen found in this mature scar tissue?

Type I
Type II
Type IV
Type V
Type VI

4 A 25-year-old woman sustains a deep, open laceration over her right forearm in a motorcycle accident. The wound is
cleaned and sutured. Which of the following cell types mediates contraction of the wound to facilitate healing?

Endothelial cells
Fibroblasts
Macrophages
Myofibroblasts
Smooth muscle cells

5 A 70-year-old woman with diabetes develops an ulcer on her right leg (shown in the image). The ulcer bed is
covered with granulation tissue. Which of the following are the principle cellular components found in the bed of this
wound of this wound?

Fibroblasts and endothelial cells


Myofibroblasts and eosinophils
Neutrophils and lymphocytes
Plasma cells and macrophages
Smooth muscle cells and Merkel cells

6 A 68-year-old man presents for repair of an abdominal aortic aneurysm. Severe complicated atherosclerosis is noted
at surgery, prompting concern for embolism of atheromatous material to the kidneys and other organs. If the patient
were to develop a renal cortical infarct as a result of surgery, which of the following would be the most likely
outcome?

Chronic inflammation
Granulomatous inflammation
Hemangioma formation
Repair and regeneration
Scar formation
7 A 40-year-old woman presents with a painless lesion on her right ear lobe (shown in the image). She reports that her
ears were pierced 4 months ago. Which of the following best explains the pathogenesis of this lesion?

Clonal expansion of smooth muscle cells


Exuberant formation of granulation tissue
Increased growth of capillary endothelial cells
Increased turnover of extracellular matrix proteoglycans
Maturation arrest of collagen assembly

8 A 58-year-old woman undergoes lumpectomy for breast cancer. One month following surgery, she notices a firm
0.3-cm nodule along one edge of the surgical incision. Biopsy of this nodule reveals chronic inflammatory cells,
multinucleated giant cells, and extensive fibrosis. The multinucleated cells in this nodule most likely formed in
response to which of the following pathogenic stimuli?

Bacterial infection
Foreign material
Lymphatic obstruction
Neoplastic cells
Viral infection

9 A 57-year-old man with a history of alcoholism presents with yellow discoloration of his skin and sclerae.
Laboratory studies show elevated serum levels of liver enzymes (AST and ALT). A trichrome stain of a liver biopsy is
shown in the image. A similar pattern of regeneration and fibrosis would be expected in the liver of a patient with
which of the following conditions?
Acute toxic liver injury
Chronic viral hepatitis
Fulminant hepatic necrosis
Hepatocellular carcinoma
Thrombosis of the portal vein

10 A 10-year-old boy trips at school and scrapes the palms of his hands. The wounds are cleaned and covered with
sterile gauze. Which of the following terms best characterizes the healing of these superficial abrasions?

Fibrosis
Granulation tissue
Primary intention
Regeneration
Secondary intention

11 A 34-year-old woman has a benign nevus removed from her back under local anesthesia. Which of the following
families of cell adhesion molecules is the principal component of the “provisional matrix” that forms during early
wound healing?

Cadherins
Fibronectins
Integrins
Laminins
Selectins

12 A 29-year-old carpenter receives a traumatic laceration to her left arm. Which of the following is the most
important factor that determines whether this wound will heal by primary or secondary intention?

Apposition of edges
Depth of wound
Metabolic status
Skin site affected
Vascular supply

13 A 9-year-old boy receives a deep laceration over his right eyebrow playing ice hockey. The wound is cleaned and
sutured. Which of the following describes the principal function of macrophages that are present in the wound 24 to
48 hours after injury?

Antibody production
Deposition of collagen
Histamine release
Phagocytosis
Wound contraction

14 A 16-year-old boy suffers a concussion during an ice hockey game and is rushed to the emergency room. A CT
scan of the brain reveals a cerebral contusion of the left frontal lobe. The boy lies comatose for 3 days but eventually
regains consciousness. Which of the following cells is the principal mediator of scar formation in the central nervous
system of this patient?

Fibroblasts
Glial cells
Neurons
Oligodendrocytes
Schwann cells

15 A 30-year-old fire fighter suffers extensive third-degree burns over his arms and hands. This patient is at high risk
for developing which of the following complications of wound healing?
Contracture
Dehiscence
Incisional hernia
Keloid
Traumatic neuroma

16 A 23-year-old man suffers a crush injury of his foot, which becomes secondarily infected. He undergoes a below-
the-knee amputation. Six months later, the patient complains of chronic pain at the site of amputation. A firm nodule
is identified at the scar site. A biopsy of the nodule demonstrates haphazard growth of nerves (shown in the image).
Which of the following is the most likely diagnosis?
Ganglioma
Ganglioneuroma
Hamartoma
Neural nevus
Neuroma

17 A 34-year-old man presents with a 5-day history of a painful sore on his hand. Physical examination reveals a 0.5-
cm abscess on the extensor surface of the left hand that drains a thick, purulent material. Diapedesis of leukocytes into
and around this patient’s infected wound occurs primarily at which of the following anatomic locations?

Lymphatic capillaries
Postcapillary venules
Precapillary arterioles
Small dermal arteries
Small dermal veins

18 A 35-year-old pregnant woman with a history of chronic gastritis presents to the emergency room complaining of
acute abdominal pain. Physical examination reveals hepatomegaly, ascites, and mild jaundice. The patient
subsequently develops acute hepatic failure and expires. Autopsy reveals thrombosis of the hepatic veins (Budd-Chiari
syndrome). During the autopsy, a lesion is identified in the distal stomach and examined by light microscopy (shown
in the image). Which of the following best describes this incidental finding at autopsy?

Carcinoma
Contracture
Diverticulum
Granuloma
Ulcer

Neoplasia

1. The term of a benign tumor is constructed by combining the word designating the tumor cell origin plus
which of the following endings or words:

A - genic

B - emia

C - oma

D - itis

E - osis(-asis)

2. Lack of differentiation is characteristic of which of the following:

A Hyperplasia

B Hypoplasia

C Anaplasia

D Hypertrophy

E Dysplasia

3. Growth of tumor without regular borderline is called:

A.Exophytic.

B.Unicentric.

C.Endophytic.

D.Expansive.

E.Invasive.
4. Growth of tumor with regular borderline is called:

A.Exophytic.

B.Unicentric.

C.Expansive.

D.Invasive.

E.Endophytic.

5. Growth of tumor into the lumen of hollow organ is called:

A.Exophytic.

B.Unicentric.

C.Expansive.

D.Invasive.

E.Endophytic.

6. Growth of tumor into the wall of hollow organ is called:

A.Exophytic.

B.Unicentric.

C.Expansive.

D.Invasive.

E.Endophytic.

7. Spreding of malignant tumors through the bloodstream is called:

A.Seeding.

B.Hematogenous metastasis.

C.Lymphogenic metastasis.

D.Perineural metastasis.

E.Through the natural passages.

8. Spreding of malignant tumors through the serosal surfaces is called:


A.Seeding.

B.Hematogenous metastasis.

C.Lymphogenic metastasis.

D.Perineural metastasis.

E.Through the natural passages.

9.Cancer is the:

A.First leading cause of death.

B.Second leading cause of death.

C.Third leading cause of death.

D.Fourth leading cause of death.

E.Fifth leading cause of death.

10. Nomenclature of tumors is based on which of the following:

A.Parenchymal component histogenesis.

B.Stromal component.

C.Localization.

D.Vascular component.

E.Inflammatory changes.

11. Neoplastic cells are characterized by all of the following, EXCEPT:

A.Loss of responsiveness to normal growth control.

B.Behave as parasites.

C.They always need in endocrine support.

D.They increase in size regardless of their local invironment.

E.They are depend on host for their nutrition and blood supply.

12. Neoplastic cells are characterized by all of the following, EXCEPT:

A.Do not loss of responsiveness to normal growth control.


B.Behave as parasites.

C.They continue to grow regardless of normal regulatory control.

D.They increase in size regardless of their local invironment.

E.They are depend on host for their nutrition and blood supply.

13. Morphologic atypia is characterized by all of the following, EXCEPT:

A Cell pleomorphism

B Metaplasia

C Alteration of parenchyma/ stroma ratio

D Atypical mitoses

E Enlarged hyperchromatic nuclei

14. Dysplastic cells are characterized by all of the following, EXCEPT:

A.Loss of cell uniformity.

B.Formation of tumor giant cells.

C.Hyperchromatic enlarged nuclei.

D.Architectural anarchy.

E.Increased mitotic figures.

15. Dysplastic cells are characterized by all of the following, EXCEPT:

A.Loss of cell uniformity.

B.Hyperchromatic enlarged nuclei.

C.Increased normal mitotic figures.

D.Mitoses are numerous and atypical.

E.Architectural anarchy.

16. Dysplastic cells are characterized by all of the following, EXCEPT:

A.Hyperchromatic enlarged nuclei.

B.Loss of cell uniformity.


C.Increased normal mitotic figures.

D.Architectural anarchy.

E.Marked pleomorphism.

17. Dysplastic cells are characterized by all of the following, EXCEPT:

A.Loss of cell uniformity.

B.Hyperchromatic enlarged nuclei.

C.Nuclei are variable and bizarre in size and shape.

D.Increased normal mitotic figures.

E.Architectural anarchy.

18. Lack of cell differentiation (anaplasia) in malignant tumor is characterized by all of the following,
EXCEPT:

A Cellular and nuclear pleomorphism

B Hyperchromatic nuclei

C Increased nuclear-to-cytoplasmic ratio

D Coarsely clumped chromatin

E Metastases

19. Anaplastic cells are characterized by all of the following, EXCEPT:

A.Marked pleomorphism.

B.Mitoses are numerous and atypical.

C.Nuclei are variable and bizarre in size and shape.

D.Cells resemble very closely their normal counterparts.

E.Nucleoli are of astounding size.

20. Anaplastic cells are characterized by all of the following, EXCEPT:

A.Marked pleomorphism.

B.Mitoses are numerous and atypical.


C.Nuclei are variable and bizarre in size and shape.

D.Formation of tumor giant cells.

E.Formation of Langhans giant cells.

21. Anaplasia is characterized by all of the following, EXCEPT:

A Cellular and nuclear pleomorphism

B Formation of minor giant cells

C Formation of atypical mitotic figures

D Formation of Langhans' giant cells

E Hyperchromatic nuclei

22. Anaplastic cells are characterized by:

A.Nuclear-cytoplasmic ratio is 1:1.

B.Nuclear-cytoplasmic ratio is 1:3.

C.Nuclear-cytoplasmic ratio is 1:5.

D.Nuclear-cytoplasmic ratio is 1:7.

E.Nuclear-cytoplasmic ratio is 1:9.

23. Anaplastic cells are characterized by all of the following, EXCEPT:

A.Marked pleomorphism.

B.Mitoses are numerous and atypical.

C.Nuclei are variable and bizarre in size and shape.

D.Loss of functions is not correlated with degree of anaplasia.

E.Loss of functions is correlated with degree of anaplasia.

24. Tumor cell invasion into the extracellular matrix can be characterized by all of the following, EXCEPT:

A Detachment of the tumor cells from each other

B Intravasation

C Attachment to matrix components


D Degradation of extracellular matrix

E Migration of tumor cells

25. All of the following statements correctly describe cell oncogenes, EXCEPT:

A Oncogenes are derived from viral DNA that has been incorporated into the genome

B Oncogenes encode proteins that resemble the products of normal genes

C Some oncogene products are the analogues of growth factors

D Some oncogene products are the analogues of growth factor receptors

E Some oncogene products activate nuclear transcription

26. Signs of benign tumors are all of the following, EXCEPT:

A.Well differentiated.

B.Few mitoses.

C.Slowly growing.

D.Invasive type of growth.

E.Expansive type of growth.

27. Signs of benign tumors are all of the following, EXCEPT:

A.Well differentiated.

B.Few mitoses.

C.Presence of metastasis.

D.Slowly growing.

E.Expansive type of growth.

28. Signs of benign tumors are all of the following, EXCEPT:

A.Rapidly growing.

B.Few mitoses.

C.Slowly growing.

D.Well differentiated.
E.Expansive type of growth.

29. Signs of benign tumors are all of the following, EXCEPT:

A.Well differentiated.

B.Pleomorphic cells.

C.Few mitoses.

D.Slowly growing.

E.Expansive type of growth.

30. Sign of benign tumors is:

A.Pleomorphic cells.

B.Rapidly growing.

C.Atypical mitoses.

D.Slowly growing.

E.Invasive type of growth.

31. Sign of benign tumors is:

A.Pleomorphic cells.

B.Rapidly growing.

C.Atypical mitoses.

D.Expansive type of growth.

E.Invasive type of growth.

32. The growth of neoplasms is critically dependent on which of the following:

A Localization

B Tumor stroma

C Neutrophil immigration

D Lymphatic drainage

E Inflammatory reaction
33. The most characteristic feature of a malignant neoplasm in contrast to a reactive or inflammatory
overgrowth is which of the following:

A Growth factor production

B Necrosis

C Localization

D Autonomous growth following the removal of all provoking factors

E Adjacent tissue compression

34. The sequence of events in the tumor cell invasion into the basement membranes includes all of the
following, EXCEPT:

A Loosening of intercellular junctions

B Attachment to the basement membrane

C Accumulation of the neutrophils

D Degradation of the basement membrane

E Tumor cell migration

35. Signs of malignant tumors are all of the following, EXCEPT:

A.Pleomorphic cells.

B.Rapidly growing.

C.Atypical mitoses.

D.Slowly growing.

E.Invasive type of growth.

36. Signs of malignant tumors are all of the following, EXCEPT:

A.Pleomorphic cells.

B.Rapidly growing.

C.Atypical mitoses.

D.Invasive type of growth.


E.Expansive type of growth.

37. Sign of malignant tumors is:

A.Well differentiated.

B.Few mitoses.

C.Slowly growing.

D.Invasive type of growth.

E.Expansive type of growth.

38. Sign of malignant tumors is:

A.Pleomorphic cells.

B.Few mitoses.

C.Slowly growing.

D.Regular cells.

E.Expansive type of growth.

39. A malignant tumor is characterized by all of the following, EXCEPT:

A Increased abnormal tissue mass

B Uncoordinated invasive growth

C Relatively autonomous growth

D Decreased abnormal tissue mass

E Metastases

40. The most characteristic manifestation of malignant tumor is which of the following:

A Cellular atypia and pleomorphism

B Compression of surrounding tissue

C Large size

D. Necrosis

E Metastases
41. Criteria, by which benign tumors can be differentiated from malignant ones, are all of the following,
EXCEPT:

A Maturity

B Rate and character of growth

C Local invasion

D Edema

E Metastases

42. The most important feature to distinguish the malignant tumor from a benign one is which of the
following:

A Lack of encapsulation

B High mitotic rate

C Necrosis

D Metastases

E Nuclear pleomorphism

43. The criteria by which benign tumors can be differentiated from the malignant ones are all of the
following, EXCEPT:

A Maturity

B Rate and character of growth

C Localisation

D Anaplasia

E Metastases

44. Signs of tumors with intermediate malignancy are all of the following, EXCEPT:

A.Pleomorphic cells.

B.Slowly growing.

C.Absence of metastasis.

D.Invasive type of growth.


E.Expansive type of growth.

45. Signs of tumors with intermediate malignancy are all of the following, EXCEPT:

A.Pleomorphic cells.

B.Slowly growing.

C.Rapidly growing.

D.Absence of metastasis.

E.Invasive type of growth.

46. Histologic origin of tumors is called:

A.Morphogenesis.

B.Histogenesis.

C.Thanatogenesis.

D.Pathogenesis.

E.Cancerogenesis.

47. Oncogenic virus is:

A.Influenza virus.

B.Parainfluenza virus.

C.Chikenpox virus.

D.AIDS virus.

E.Human papilloma virus.

48. Select from the proposed options oncogenic virus:

A.Influenza virus.

B.Parainfluenza virus.

C.Chikenpox virus.

D.AIDS virus.

E.Hepatitis B virus.
49. All of the following viruses proved to be capable of producing malignancies in human beings, EXCEPT

A Human papillomavirus

B Cytomegalovirus

C Epstein - Barr virus

D Hepatitis B virus

E Hepatitis C virus

50. Secondary lesions in malignant tumors are all of the following, EXCEPT:

A.Hemorrhage.

B.Necrosis.

C.Mucin accumulation.

D.Calcification.

E.Amyloidosis.

51. Classification of neoplasms by WHO is based on:

A.Etiology of tumors.

B.Morphogenesis of tumors.

C.Localization of tumors.

D.Histogenesis of tumors.

E.Differentiation of tumors.

52. The factor assessed in the histological grading of a malignant tumor is which of the following:

A The number of lymph node metastases

B The size (diameter) of the primary tumor

C The extent of invasion of the primary tumor into surrounding structures

D The degree of cytological differentiation of the primary tumor

E The presence or absence of liver metastases


53. The important factors associated with the increasing incidence of tumors are all of the following,
EXCEPT:

A Age

B Diet

C Environment

D Acute inflammation

E Genetic make up

54. Most common localization of cancer in men in Western countries is:

A.Stomach.

B.Prostate gland.

C.Thyroid gland.

D.Lungs.

E.Large intestine.

55. Most common localization of cancer in women in Western countries is:

A.Uterus.

B.Ovaries.

C.Thyroid gland.

D.Lungs.

E.Breasts.

56. Most common localization of cancer in men in Kyrgyzstan is:

A.Esophagus.

B.Stomach.

C.Thyroid gland.

D.Lungs.

E.Large intestine.
57. Most common localization of cancer in women in Kyrgyzstan is:

A.Uterus.

B.Ovaries.

C.Breasts.

D.Thyroid gland.

E.Lungs.

58. Tumor with intermediate malignancy is:

A.Ameloblastoma.

B.Neuroblastoma.

C.Glioblastoma.

D.Osteoblastoma.

E.Chondroblastoma

59. Negative effects resulting from neoplasia are all of the following, EXCEPT:

A Compression of adjacent tissues

B Cachexia

C Destruction of adjacent tissues

D Paraneoplastic syndrome

E Cancer obesity

60. All of the following neoplasms are malignant, EXCEPT:

A. Adenocarcinoma

B. Melanoma

C. Seminoma

D. Chorionepithelioma

E. Papillary cystadenoma
1. A malignant tumor is characterized by all of the following, EXCEPT:
1. Increased abnormal tissue mass

2. Uncoordinated invasive growth

3. Relatively autonomous growth

4. Decreased abnormal tissue mass

5. Metastases

2. The growth of neoplasms is critically dependent on which of the following:


1. Localization

2. Tumor stroma

3. Neutrophil immigration

4. Lymphatic drainage

5. Inflammatory reaction

3. The nomenclature of tumors is based on which of the following:

1. Stromal component

2. Localization

3. Inflammatory changes

4. Vascular component

5. Parenchymal component histogenesis

4. The term of a benign mesenchymal tumor is constructed by combining the word designating
the tumor cell origin plus which of the following endings or words:

1. - sarcoma

2. - carcinoma

3. -oma

4. - itis

5. - osis(-asis)
4

9. The important factors associated with the increasing incidence of tumors are all of the
following, EXCEPT:
1. Age

2. Diet

3. Environment

4. Acute inflammation

5. Genetic make up

10. The criteria by which benign tumors can be differentiated from the malignant ones are all of
the following, EXCEPT:

1. Maturity

2. Rate and character of growth

3. Localisation

4. Anaplasia

5. Metastases

11. Morphologic atypia is characterized by all of the following, EXCEPT:

1. Cell pleomorphism

2. Metaplasia

3. Alteration of parenchyma/ stroma ratio

4. Atypical mitoses

5. Enlarged hyperchromatic nuclei

12. Lack of cell differentiation (anaplasia) in malignant tumor is characterized by all of the
following, EXCEPT:

1. Cellular and nuclear pleomorphism

2. Hyperchromatic nuclei

3. Increased nuclear-to-cytoplasmic ratio


4. Coarsely clumped chromatin

5. Metastases

13. Anaplasia is characterized by all of the following, EXCEPT:


1. Cellular and nuclear pleomorphism

2. Formation of minor giant cells

3. Formation of atypical mitotic figures

4. Formation of Langhans' giant cells

5. Hyperchromatic nuclei

14. Dysplasia is characterized by all of the following, EXCEPT:

1. Loss of cell uniformity

2. Hyperchromatic enlarged nuclei

3. Appearance of mitotic figures

4. Loss of cell architectural orientation

5. Formation of tumor giant cells

15. Negative effects resulting from neoplasia are all of the following, EXCEPT:

1. Compression of adjacent tissues

2. Cachexia

3. Destruction of adjacent tissues

4. Paraneoplastic syndrome

5. Cancer obesity

16. The sequence of events in the tumor cell invasion into the basement membranes includes
all of the following, EXCEPT:

1. Loosening of intercellular junctions

2. Attachment to the basement membrane


3. Accumulation of the neutrophils

4. Degradation of the basement membrane

5. Tumor cell migration

17. Tumor cell invasion into the extracellular matrix can be characterized by all of the following,
EXCEPT:

1. Detachment of the tumor cells from each other

2. Intravasation

3. Attachment to matrix components

4. Degradation of extracellular matrix

5. Migration of tumor cells

18. The most characteristic feature of a malignant neoplasm in contrast to a reactive or


inflammatory overgrowth is which of the following:

1. Growth factor production

2. Necrosis

3. Localization

4. Autonomous growth following the removal of all provoking factors

5. Adjacent tissue compression

19. The most characteristic manifestation of malignant tumor is which of the following:

1. Cellular atypia and pleomorphism

2. Compression of surrounding tissue

3. Large size

4. Necrosis

5. Metastases

4
20. The most important feature to distinguish the malignant tumor from a benign one is which
of the following:

1. Lack of encapsulation

2. High mitotic rate

3. Necrosis

4. Metastases

5. Nuclear pleomorphism

21. The factor assessed in the histological grading of a malignant tumor is which of the
following:

1. The number of lymph node metastases

2. The size (diameter) of the primary tumor

3. The extent of invasion of the primary tumor into surrounding structures

4. The degree of cytological differentiation of the primary tumor

5. The presence or absence of liver metastases

22. All of the following statements correctly describe cell oncogenes, EXCEPT:

1. Oncogenes are derived from viral DNA that has been incorporated into the genome

2. Oncogenes encode proteins that resemble the products of normal genes

3. Some oncogene products are the analogues of growth factors

4. Some oncogene products are the analogues of growth factor receptors

5. Some oncogene products activate nuclear transcription

23. All of the following viruses proved to be capable of producing malignancies in human
beings, EXCEPT

1. Human papillomavirus

2. Cytomegalovirus

3. Epstein - Barr virus


4. Hepatitis B virus

5. Hepatitis C virus

24. Criteria, by which benign tumors can be differentiated from malignant ones, are all of the
following, EXCEPT:

I. Maturity

2. Rate and character of growth

3. Local invasion

4. Edema

5. Metastases

25. Lack of differentiation is characteristic of which of the following:

1. Hyperplasia

2. Hypoplasia

3. Anaplasia

4. Hypertrophy

5. Dysplasia

EPITHELIAL TUMORS
1. The term of a benign epithelial tumor is constructed by combining the word designating the tumor cell
origin plus which of the following endings or words:

A sarcoma

B carcinoma

C oma

D itis

E osis(-asis)

2. The term of a malignant epithelial tumor is constructed by combining the word designating the tumor
cell origin plus which of the following endings or words:

A sarcoma

B carcinoma

C oma

D itis

E osis(-asis)

3. Benign tumor arising from squamous epithelium is called:

A.Papilloma.

B.Adenoma.

C.Lipoma.

D.Fibroma.

E.Osteoma.

4. Benign epithelial neoplasm producing finger-like projection from epithelial surfaces is referred to as:

A Papilloma

B Adenoma

C Fibroma

D Sarcoma
E Teratoma

5. Select the signs inherent in the papilloma:

A tissue atypism

B cellular atypism

C metastasis

D invasive growth

E cancer pearls

6. Papilloma can become complicated by all: EXCEPT:

A bleeding

B inflammation

C resorption

D malignancy with the development of squamous cell carcinoma

E relapse after removal

7.Squamous cell papilloma is localized in:

A.Pelvis of kidney.

B.Calyces of kidney.

C.Vagina.

D.Ureter.

C.Urethra.

8. Squamous cell papilloma is localized in:

A.Pelvis of kidney.

B.Calyces of kidney.

C.Ureter.

D.Mouth.

E.Urethra.
9. Transitional cell papilloma is localized in:

A.Mouth.

B.Esophagus.

C.Skin.

D.Ureter.

E.Vagina.

10. Transitional cell papilloma is localized in:

A.Mouth.

B.Esophagus.

C.Vagina.

D.Gall bladder.

E.Urinary bladder.

11. Benign tumor arising from columnar epithelium is called:

A.Papilloma.

B.Adenoma.

C.Lipoma.

D.Fibroma.

E.Osteoma.

12. Benign tumors arising from epithelial tissue are referred to as:
A Sarcomas

B Adenocarcinomas

C Papillomas

D Adenoma

E Polyps

13. Adenoma is localized in:


A.Mouth.

B.Esophagus.

C.Vagina.

D.Gall bladder.

E.Urinary bladder.

14. Adenoma is localized in:

A.Mouth.

B.Esophagus.

C.Stomach.

D.Ureter.

E.Vagina.

15. Which histologic type of adenoma arise from glandular parenchyma?

A.Acinar.

B.Tubular.

C.Trabecular.

D.Papillary.

E.Cystadenoma.

16. Which histologic type of adenoma arise from ducts?

A.Acinar.

B.Tubular.

C.Trabecular.

D.Papillary.

E.Cystadenoma.

17. Which histologic type of adenoma is characterized with predominance of stroma?

A.Acinar.
B.Tubular.

C.Trabecular.

D.Papillary.

E.Fibroadenoma.

18. Which type of adenoma projected under surface of mucous membrane?

A.Acinar adenoma.

B.Tubular adenoma.

C.Trabecular adenoma.

D.Adenomatous polyp.

E.Fibroadenoma.

19. In what organs and tissues can develop adenoma?

A skeletal muscles

B pituitary gland

C substance of the brain

D spleen

E bones

20. Precancerous disease of the thyroid gland is:

A papilloma

B polyp

C serous cystadenoma

D follicle

E adenoma

21. Malignant tumor arising from epithelial cells is called:

A.Papilloma.

B.Adenoma.
C.Carcinoma.

D.Sarcoma.

E.Teratoma.

22. The earliest way of metastasizing a malignant tumor from the epithelium is:

A hematogenous

B perineural

C lymphogenic

D implantation

E Intracanacular

23. Which histologic type of carcinoma is characterized by absence of invasive growth?

A.Adenocarcinoma.

B.Carcinoma in citu.

C.Mucous carcinoma.

D.Solid carcinoma.

E.Medullary carcinoma.

24. Which histologic type of carcinoma is characterized by formation of cancerous pearls?

A.Adenocarcinoma.

B.Nonkeratinizing squamous cell carcinoma.

C.Keratinizing squamous cell carcinoma.

D.Transitional cell carcinoma.

E.Medullary carcinoma.

25. Which histologic type of carcinoma is characterized by formation of trabecular structures?

A.Adenocarcinoma.

B.Mucous carcinoma.

C.Solid carcinoma.
D.Scirrhous carcinoma.

E.Medullary carcinoma.

26. Which histologic type of carcinoma is undifferentiated?

A.Carcinoma in citu.

B.Adenocarcinoma.

C.Solid carcinoma.

D.Nonkeratinizing squamous cell carcinoma.

E.Keratinizing squamous cell carcinoma.

27. Carcinoma histologically - trabecular structures with stroma. Which type of carcinomas is it?

A scirrhous carcinoma

B solid carcinoma

C mucous or colloid carcinoma

D medullary carcinoma

E small cell carcinoma

28. Which histologic type of carcinoma is characterized by production of excessive amount of mucus?

A.Adenocarcinoma.

B.Colloid carcinoma.

C.Solid carcinoma.

D.Scirrhous carcinoma.

E.Medullary carcinoma.

29. Carcinoma histologically - appear signet ring cells (excessive mucin accumulates in the cell and the
nucleus is pushed towards the periphery). Which type of carcinomas is it?

A scirrhous carcinoma

B solid carcinoma

C mucous or colloid carcinoma


D medullary carcinoma

E small cell carcinoma

30. In which histologic type of carcinoma stroma predominates anaplastic parenchyma?

A.Adenocarcinoma.

B.Colloid carcinoma.

C.Solid carcinoma.

D.Scirrhous carcinoma.

E.Medullary carcinoma.

31. Which histologic type of carcinoma is undifferentiated?

A.Carcinoma in citu.

B.Scirrhous carcinoma.

C.Adenocarcinoma.

D.Nonkeratinizing squamous cell carcinoma.

E.Keratinizing squamous cell carcinoma.

32. Carcinoma histologically - stroma predominates anaplastic parenchyma. Which type of carcinomas is it?

A scirrhous carcinoma

B solid carcinoma

C mucous or colloid carcinoma

D medullary carcinoma

E small cell carcinoma

33. In which histologic type of carcinoma parenchyma predominates stroma?

A.Adenocarcinoma.

B.Colloid carcinoma.

C.Solid carcinoma.

D.Scirrhous carcinoma.
E.Medullary carcinoma.

34. Carcinoma histologically - parenchyma predominates stroma; grossly - consistence is soft. Which type
of carcinomas is it?

A scirrhous carcinoma

B solid carcinoma

C mucous or colloid carcinoma

D medullary carcinoma

E small cell carcinoma

35. Mixt histologic type of carcinoma is:

A.Adenosquamous carcinoma.

B.Colloid carcinoma.

C.Solid carcinoma.

D.Scirrhous carcinoma.

E.Medullary carcinoma.

36. Carcinomas with epidermis cell differentiation are called:


A Sarcomas

B Adenocarcinomas

C Squamous cell carcinomas

D Papillomas

E Cystadenomas

37. Frequent localization of squamous cell carcinoma:

A skin

B liver

C brain

D intestine
E stomach

38. Malignant tumor, arising from squamous cell, is named:

A squamous cell carcinoma

B acinar adenocarcinoma

C papillary adenocarcinoma

D transitional cell carcinoma

E papilloma

39. Which histologic type of carcinoma is characterized by formation of irregular glandular structures?

A.Adenocarcinoma.

B.Nonkeratinizing squamous cell carcinoma.

C.Keratinizing squamous cell carcinoma.

D.Transitional cell carcinoma.

E.Medullary carcinoma.

40. What differentiated malignant tumor can develop from adenoma:

Asolid carcinoma

B scirrhous carcinoma

C adenocarcinoma

D small cell carcinoma

E mucous or colloid carcinoma

41. Malignant tumors arising from epithelial tissue are referred to as:
A Sarcomas

B Adenocarcinomas

C Papillomas

D Cystadenomas

E Polyps
42. The first site of metastasis for adenocarcinonia of the colon would most likely be to which of the
following:

A Brain

B Liver

C Lung

D Lymph nodes

E Spleen

43. Carcinomas forming glandular structures called:

A Sarcomas

B Papillomas

C Adenocarcinomas

D Cystadenomas

E Polyps

44. Adenocarcinomas property:

A develops from connective tissue

B develops from the glandular epithelium

C organ-specific

D there is no atypia

E does not give metastasis

45. All of the following morphologic features characterize the adenocarcinoma cells, EXCEPT:

A Variation in size and shape

B Hyperchromatic nuclei

C Enlarged nucleoli

D Hypochromatic nuclei

E Atypical mitoses
46. Which histologic type of carcinoma is undifferentiated?

A.Adenocarcinoma.

B.Carcinoma in citu.

C.Nonkeratinizing squamous cell carcinoma.

D.Keratinizing squamous cell carcinoma.

E.Small cell carcinoma.

47. Carcinoma histologically consists of small lymphocyte-like cells. Which type of carcinomas is it?

A scirrhous carcinoma

B solid carcinoma

C mucous or colloid carcinoma

D medullary carcinoma

E small cell carcinoma

48. Which of the following is a benign tumor?

A Chondroblastic osteosarcoma of the bone

B Granuloma of the soft tissue

C Tuberculoma

D Papillary serous cystadenoma ot the ovary

E Papillary carcinoma of the thyroid

49. All of the following neoplasms are malignant, EXCEPT:

A Adenocarcinoma

B Melanoma

C Seminoma

D Chorionepithelioma

E Papillary cystadenoma
50. Which of the following defeats of the reproductive system of women is a precancerous?

A Cystic atrophy of the endomethum

B Endocervical polyps

C Condyloma acuminatum of the vulva

D Endometrial polyps

E Atypical hyperplasia of the endometrium

51. The important prognostic features in invasive breast cancer are all of the following, EXCEPT:

A Histologic type of the tumor

B Grade of the tumor

C Size of the tumor

D Secondary necrosis

E Presence or absence of estrogen receptors on tumor cells

52. The most common benign tumor of the ovary is which of the following:

A Papilloma

B Fibroma

C Cystadenoma

D Adenocarcinoma

E Melanoma

53. The most common benign tumor of the female breast is which of the following:
A Cystadenoma

B Fibroadenoma

C Sarcoma

D Fibroma

E Adenocarcinoma
54. How called benign tumor from the glandular epithelium with a significantly developed stroma:

A fibroma

B adenoma

C fibrosarcoma

D papilloma

E fibroadenoma

55. Carcinoma histologically consists of two patterns: adenosquamous carcinoma (adenocarcinoma +


squamous cell carcinoma). Which type of carcinomas is it?

A scirrhous carcinoma

B solid carcinoma

C mucous or colloid carcinoma

D dimorphous carcinoma

E small cell carcinoma

56. Carcinoma histologically - stroma predominates anaplastic parenchyma. Which type of carcinomas is it?

A fibrous cancer

B solid carcinoma

C mucous or colloid carcinoma

D medullary carcinoma

E small cell carcinoma

57. Malignant tumor, arising from transitional cell, is named:

A squamous cell carcinoma

B acinar adenocarcinoma

C papillary adenocarcinoma

D transitional cell carcinoma

E papilloma
58. Origin of chorionepithelioma:

A endometrium

B placenta

C myometrium

D umbilical cord

E teka – tissue

59. What is the characteristic of chorionepithelioma:

A develops from uterine tissue

B Expansive growth

C feature of stroma structure - many collagen fibers

D the primary way of metastasis is hematogenous

E do not metastasize

60. Select the organ-specific tumor of the pituitary gland:

A Scirrhous carcinoma

B eosinophilic adenoma

C pheochromocytoma

D Small cell carcinoma

E papilloma

61. In which organ the pheochromocytoma develops:

A pituitary gland

B ovary

C adrenal gland

D Thyroid gland

E pancreas
Cell Injury (Davydov-14 q)

1.Variant of reversible cell injury is:

A. Coagulative necrosis.

B. Gangrene.

C. Cellular swelling.

D. Fat necrosis.

E. Apoptosis.

2.Variant of reversible cell injury is:

A. Coagulative necrosis.

B. Fatty change.

C. Caseous necrosis.

D. Fat necrosis.

E. Apoptosis.

3.Programmed cell death is:

A. Coagulative necrosis.

B. Gangrene.

C. Cellular swelling.

D. Fat necrosis.

E. Apoptosis.

4.Type of necrosis which associated with formation of yellow-white hard areas is:

A. Coagulative necrosis.

B. Liquefactive necrosis.

C. Gangrene.

D. Fat necrosis.
E. Fibrinoid necrosis.

5.Type of necrosis which usually occurs in brain and spinal cord is:

A. Coagulative necrosis.

B. Liquefactive necrosis.

C. Gangrene.

D. Fat necrosis.

E. Fibrinoid necrosis.

6.Type of necrosis which usually occurs in tuberculous infction is:

A. Coagulative necrosis.

B. Liquefactive necrosis.

C. Gangrene.

D. Caseous necrosis.

E. Fibrinoid necrosis.

7.Type of necrosis which usually occurs in acute pancreatitis and appears like chalky white plaques in adipose tissue
is:

A. Coagulative necrosis.

B. Liquefactive necrosis.

C. Fat necrosis.

D. Caseous necrosis.

E. Fibrinoid necrosis.

8.Type of necrosis which usually occurs in connective tissue and blood vessels wall is:

A. Coagulative necrosis.

B. Liquefactive necrosis.

C. Fat necrosis.

D. Caseous necrosis.

E. Fibrinoid necrosis.
9.Type of necrosis which occurs in extremities due to arterial obstruction is:

A. Coagulative necrosis.

B. Liquefactive necrosis.

C. Fat necrosis.

D. Gangrene.

E. Fibrinoid necrosis.

10.Type of necrosis which is caused by Clostridium perfringens is:

A. Coagulative necrosis.

B. Liquefactive necrosis.

C. Fat necrosis.

D. Dry gangrene.

E. Gas gangrene.

11.Outcome of necrosis with formation of scar is called:

A. Organization.

B. Calcification.

C. Ossification.

D. Cyst formation.

E. Suppuration.

12.Outcome of necrosis with formation of bone tissue is called:

A. Organization.

B. Calcification.

C. Ossification.

D. Cyst formation.

E. Suppuration.

13.Outcome of necrosis with formation of cavity filled with serous fluid is called:

A. Organization.
B. Calcification.

C. Ossification.

D. Cyst formation.

E. Suppuration.

14.Outcome of necrosis which associated with bacterial infection is called:

A. Organization.

B. Calcification.

C. Ossification.

D. Cyst formation.

E. Suppuration.

Cell Injury (Kozmina- 32q)

15. Hypoxia decreases cellular levels of ATP and inhibits the normal function of the plasma membrane ouabain-
sensitive Na-K-ATPase pump. Which one of the listed changes will results from decreased function of this membrane
ion pump?

Sodium Ion Changes Potassium Ion Changes

Decreased sodium ions inside the cell Decreased potassium ions outside the cell

Decreased sodium ions inside the cell Increased potassium ions outside the cell

Increased sodium ions inside the cell Increased potassium ions outside the cell

Increased sodium ions outside the cell Increased potassium ions inside the cell

Increased sodium ions outside the cell Decreased potassium ions inside the cell

16. A 49-year-old man develops an acute myocardial infarction because of the sudden occlusion of the left anterior
descending coronary artery. The areas of myocardial necrosis within the ventricle can best be describe as

Coagulative necrosis
Liquefactive necrosis
Fat necrosis
Caseous necrosis
Fibrinoid necrosis

17. The degradation of intracellular organelles through the process in which autosomes combine with primary
lysosomes to form autophagolysosomes are called

Autophagy
Heterophagy
Heteroplasmy
Homophagy
Endocytosis

18. Enzymatic digestion is the predominant event in the following type of necrosis:

Coagulative necrosis
Liquefactive necrosis
Caseous necrosis
Fat necrosis
Gangrene

19. Apoptosis has the following features except:

There is cell shrinkage in apoptosis


There are no acute inflammatory cells surrounding apoptosis
There may be single cell loss or affect clusters of cells
Apoptosis is seen in pathologic processes only
Apoptosis is a form of «programmed cell death»

20. Diabetic foot is an example of:

Dry gangrene
Wet gangrene
Gas gangrene
Necrotizing inflammation
Coagulative necrosis

21. The causative organism of gas gangrene is

Staphylococcus
Streptococcus
Leptospira
Mycobacterium
None of these

22. A red infarct is least likely to occur in

Kidney
Lung
Heart
Brain
All of these to
23. The process of programmed gene directed cell death characterized by cell shrinkage, nuclear condensation and
fragmentation is known as –

Apoptosis
Chromatolysis
Pyknosis
Necrosis
Karyorrhexis

24. Localized area of ischemic necrosis is mostly associated with

Ascitese.
Petechiae.
Infarction.
Emboli formation.
Hematoma.

25. One of manifestations of metabolic derangements in cells is:

Apoptosis
The intracellular accumulation of abnormal amounts of various substances
Hypertrophy
Metaplasia
Atrophy

26. Early potentially reversible changes in myocardial cells induced by anoxia include all of the following conditions,
except:

Failure of oxidative phosphorylation


Depletion of ATP
Inhibition of anaerobic glycolysis and glycogenolysis
Decrease in cellular pH
Increase in intracellular Na and water and loss of intracellular K

27. Which group of factors is most important in the cellular pathogenesis of irreversible cell injury?

Lipid deposition, reduced protein synthesis, nuclear damage


Mitochondrial condensation, glycolysis, sodium cell loss
Mitochondrial hyperplasia, lysozyme release, membrane injury
Reduced ATP, increased calcium influx, membrane injury
Ribosome detachment, glycolysis, nuclear damage

28. Which of the following events is most important in the development of irreversible cell injury?

Cell membrane damage


Increased cell water
Myelin figure accumulation
Loss of ribosomes
Swelling of mitochondria
29 The reversible process caused by accumulation of glycosaminoglycans in extracellular matrix due to the increase
of vascular permiability is:

Amyloidosis
Glycogenoses
Hyalinosis
Mucoid changes
Fibrinoid changes

30. All the pathological processes are irreversible, except:

Mucoid changes
Fibrinoid changes
Amyloidosis
Apoptosis
Necrosis

31. The following features characterize reversible cell injury, except:

Blebs
Endoplasmic reticulum swelling
Dispersion of ribosomes
Myelin figures
Autophagy

32. The following features characterize irreversible cell injury, except:

Nucleous pyknosis
Karyolysis
Karyorrhaxis
Endoplasmic reticulum swelling
Mitochondrial swelling

33. Coagulative necrosis is characterized by all of the following pathologic features, except:

Denaturation of cytoplasmic proteins


Karyorrhaxis
Karyopiknosis
Breakdown of cell organelles
Lipid deposition

34. Digestion of the cell by lysosome enzymes of immigrant leukocytes is termed:

Autolysis
Apoptosis
Heterolysis
Inflammation
Metaplasia

35. Necrotic cells are characterized by all of the following features, except:

Increased eosinophilia
Hyaline-like droplets in the cytoplasm
Glassy appearance
Lysis of cytoplasm
Fragmentation of cytoplasm

36. One of the following changes in cells is apoptotic:

Karyolysis
Nuclear pyknosis
Plasmolysis
Breakdown ofmilochondrias
Cellul.ir swelling

37. One of the following variants of necrosis can be found in tuberculosis:

Caseous necrosis
Gangrenous necrosis
Liquefaclive necrosis
Fat necrosis
Fibrinoid necrosis

38. One of the following variants of necrosis is associated with acute pancreatitis:

Coagulative necrosis
Liquefactive necrosis
Caseous necrosis
Gangrenous necrosis
Fat necrosis

39. Ischemic injury in the central nervous system results in:

Liquefactive necrosis
Coagulative necrosis
Caseous necrosis
Gangrenous necrosis
Fat necrosis

40. One of the following variants of necrosis can be found in wet gangrene:

Coagulative necrosis
Liquefactive necrosis
Caseous necrosis
Gangrenous necrosis
Fat necrosis

41. One of the following variants of necrosis can be found in gangrene of lower extremities:

Coagulative necrosis
Liquefactive necrosis
Caseous necrosis
Gangrenous necrosis
Fat necrosis
42. Apoptosis is characterized by all of the following pathologic features, except:

Nuclear chromatin condensation


Nuclear fragmentation
Cytoplasmic budding
Organellar swelling
Phagocytosis of the apoptotic bodies

43. Apoptosis occurs in all of the following events, except:

Aging
Cell death in tumors
Death of immune cells
Pathologic atrophy
Cell autolysis

44. Apoptotic body is characterized by all of the following, except:

Eosinophilic cytoplasm
Tightly packed organelles
Presence of nuclear fragments
Absence of nuclear fragments
Protein droplets in cytoplasm

45. Apoptotic cell is characterized by all of the following hislological features, except:

Round form
Oval form
Eosinophilic cytoplasm
Basophilic cytoplasm
Dense nuclear chromatin fragments

Intracellular accumulations (Kozmina -34q)

1. Increased lipolysis of fat stores, which can results from starvations, diabetes mellitus, or corticosteroid use, it most
likely to cause steatosis (fatty liver) through which one of the listed mechanisms?

Decreased free fatty acid excretion from the liver leads of free fatty acid accumulation in hepatocytes
Excess NADH (high NADH/NNAD ratio) causes excess production of lactate from pyruvate, which accumulates in
hepatocytes
Increased free fatty acid delivery to the liver leads to triglyceride accumulation in hepatocytes
Inhibition of apoprotein synthesis by the liver leads to phospholipids accumulation in hepatocytes
Inhibition of HMG-GoA reductasa activity leads to cholesterol accumulation in hepatocytes
2. A 48-year-old male who has a long history of excessive drinking presents with sings of alcoholic hepatitis.
Microscopic examination of a biopsy of this patient’s liver reveals irregular eosinophilic hyaline inclusions within the
cytoplasm of the hepatocytes. These eosinophilic inclusions are composed of

Immunoglobulin
Excess plasma proteins
Prekeratin intermediate filaments
Basement membrane material
Lipofuscin

3. In fatty liver to chronic alcoholism, the following mechanisms are involved except:

Increased free fatty acid synthesis


Decreased triglyceride utilization
Increased α-glycerophosphate
Block in lipoprotein excretion
Decreased free fatty acid synthesis

4. One of the possible causes of intracellular accumulation of metabolic substances is:

Genetic defects
Inflammation
Embolism
Necrosis
Activation of oncogenes

5. What substances are accumulating within parenchymal cells in steatosis?

Cholesterol
Apoproleins
Triglycerides
Vitamins
Ketone bodies

6. Fatty change is often seining in all of the following organs, except:

Liver
Heart
Kidney
Muscles
Lung

7. The causes of steatosis include all of the following pathologic states, except:

Obesity
Anoxia
Inflammation
Protein malnutrition
Intoxication

8. The stain used to identify fat is:


Hematoxylin and eosin stain
Sudan III stain
Congo red stain
PAS reaction
Metachromatic stain

9. The stain used to identify glycogen is:

Hematoxylin and eosin stain


Sudan III stain
Congo red stain
PAS reaction
Metachromatic stain

10. The fatty liver has all pathologic features, except:

Enlarged
2 Yellow
Red
Soft
Greasy

11. Fatty change is seen by light microscopy as:

Intracellular granules
Basophilic granules
Extracellular granules
Vacuoles in the cytoplasm around the nucleus
Eosinophilic granules

12. The most common cause of fatty change in the heart is:

Inflammation
Neoplasia
Hypoxia
Amyloidosis
Autoimmune diseases

13. Fatty change in the heart characterized by:

Red heart
Small size heart
Tiger heart
Solid heart
Goose heart

14. Cells, which can accumulate cholesterol and cholesterol esters in atherosclerotic plaque, are:

Macrophages and smooth muscle cells


Leucocytes and fibroblasts
Lymphocytes and erythrocytes
Fibroblasts and leucocytes
Erythrocytes and lymphocytes

15. The cells accumulating fat within the intimae layer of the aorta and large arteries in atherosclerotic plaques are
called:

Fibroblasts
Epithelial cells
Foam cells
Lymphocytes
Leukocytes

16. All of the following mechanisms cause intracellular accumulation, except:

Abnormal metabolism
Protein mutation
Inflammation
Enzyme deficiency
Ingestion of indigestible materials

17. Ballooning degeneration of hepatocytes is caused by:

Accumulation of water and cellular swelling


Retaining of biliary material
Accumulation of iron or copper substances
Accumulation of fat droplets
Accumulation of lipofuscin

18. Ballooning degeneration of hepatocytes results from:

Viral hepatitis
Alcoholic liver disease
Biliary material accumulation
Obesity
Diabetes mellitus

19. All of the following features characterize the ballooning degeneration of hepatocytes, except:

Swollen cells
Edematous appearance
Clumped cytoplasm
Large droplets of fat
Large clear spaces

20. Diabetes mellitus is characterized by the accumulation of glycogen in all cells, except:

Epithelial cells of the proximal tubules


Liver cells
b-cells of the islets of Langerhans
Smooth muscle cells
Heart muscle cells
21.One of manifestations of metabolic derangements in cells is:

Apoptosis.
B. Intracellular accumulation of abnormal amounts of various substances.

Hypertrophy.
Metaplasia.
Atrophy.

22.Mallory’s bodies may be found in:

Neurons.
Cardiomyocytes.
Hepatocytes.
Epithelial cells of stomach.
Epithelial cells of renal tubules.

23.Hyaline droplets in renal tubular epithelial cells are seen in:

Proteinuria.
Lipiduria.
Hematuria.
Cylindruria.
Disproteinemia.

24.Russel’s bodies may be found in:

Epithelioid cells.
Plasma cells.
Lymphocytes.
Histiocytes.
Mast cells.

25.Alzheimer disease is associated with intracellular accumulations of proteins in:

Liver.
Heart.
Spinal cord.
Brain.
Kidneys.

26.The stain used to identify fat is:


Hematoxylin and eosin stain.
Red oil O.
Congo red stain.
PAS reaction.
Metachromatic stain.

27.Accumulation of cholesterol and cholesterol esthers with formation of tumorous masses is called:

Atheroma.
Xantoma.
Adenoma.
Papilloma.
Teratoma.

28.Xantoma is a pathological process with accumulation of:

Neutral fat.
Lipoids.
Cholesterol.
Phospholipids.
Lipoproteins.

29.Foam cells are characterized by accumulation of:

Neutral fat.
Triglycerides.
Cholesterol.
Phospholipids.
Lipoprotein.

30.Derangement of glycogen metabolism is seen in:

Atherosclerosis.
Diabetes mellitus.
Viral hepatitis.
Arterial hypertension.
Goiter.

31.Glycogen storage disease is caused by:

Genetic disorders.
Hypoxia.
Infection.
Immune disorders.
Intoxication.

32.Mucinous degeneration is occurs in:

Inflammation of heart.
Inflammation of brain.
Inflammation of bronchi.
Inflammation of spinal cord.
Inflammation of liver.

33.Tumor arising from epithelial cells with accumulation of mucin is called:

Mucinous teratoma.
Mucinous sarcoma.
Mucinous melanoma.
Mucinous neuroblastoma.
Mucinous carcinoma.
34.Colloid degeneration is occurs in:

Salivary gland.
Prostate gland.
Thyroid gland.
Pituitary gland.
Parathyroid gland.

Extracellular accumulations (Kozmina – 65 q)

1. An 87-year-old male develops worsening heart failure. Workup reveals decreased left ventricular filling due to
decreased compliance of the left ventricle. Two months later the patient dies, and postmortem sections reveal
deposits of eosinophilic, Congo red positive material in the interstitial of his heart. When viewed under polarized
light. This material displays an apple-green birefringence. What is the correct diagnosis?

Amyloidosis
Glycogenosis
Hemochromatosis
Sarcoidosis
Senile atrophy

2. In cases of renal failure on long-term hemodialysis, there is development of following type of amyloid:

Amyloid light chain (AL)


Amyloid-associated protein (AA)
Amyloid β2 microglobulin (Aβ2m)
β amyloid protein (Aβ)
Immunoglobulin heavy chain amyloid (AH)

3. The most common form of amyloid in third world countries is:

Primary
Secondary
Hereditary
Localized
Endocrine amyloidosis

4. Cardiac amyloidosis often produces:

Dilated cardiomyopathy
Constrictive cardiomyopathy
Restrictive cardiomyopathy
Ischemic cardiomyopathy
Idiopathic cardiomyopathy

5. In senile cardiac amyloidosis, the biochemical form of amyloid is:


AL
AA
ATTR
Aβ2m
AL + AA

6. Health risk in obesity is due to weight in excess of the following for age and sex:

10%
20%
30%
40%
50%

7. Obesity is due to:

Hyperplasia of adipocytes only


Hypertrophy of adipocytes only
Hyperplasia as well as hypertrophy of adipocytes
Fatty change in liver only
Fatty change in liver and hart only

8. Most often secondary amyloidosis occurs when the following pathologies:

Chronic suppuration
Acute inflammation
Cellular necrosis
Hyaline degeneration
c. Ischemia

9. The substance with fibrillar structure, which forms under the pathological conditions, is:

Reabsorption droplets
Russell bodies
Lipids
Amyloid
Calcificates

10. The substance giving red color with the Congo red stain is:

Lipid
Hyaline
Water
Amyloid
Glycogen

11. In long-standing hypertension and diabetes mellitus, the walls of arterioles, especially in the kidney, become:

Serous
Thined
Hyalinized
Ulcered
Pigmented

12. The pathologic proteinaceous substance, accumulating only between cells in various tissues and organs of the
body is:

Glycogen
Hyaline
Water
Amyloid
Lipid

13. The pathologic proteinaceous substance, accumulating both within cells and in the extracellular matrix in various
tissues and organs of the body is:

Glycogen
Hyaline
Water
Amyloid
Lipid

14. The characteristics of amyloid fibrils include all of the following, except:

Fibril composed of paired filaments


Nonbranching fibrils
Fibrils with an indefinite diameter
Fibrils with indefinite length
Fibrils with definite length

15. The organ affected in both primary and secondary amyloidosis is:

Kidney
Stomach
Uterus
Brain
Lung

16. The deposits of amyloid sees in all tissues, except:

Mesangium and capillary loops


Basement membranes of blood vessels
Tubular basement membranes
Epithelium in proximal renal tubules
Interstitium of the kidneys

17. All of the following pathological processes are reversible, except:

Mucoid changes
Cellular swelling
Amyloidosis
Edema
Lipid accumulation in hepatocytes

18. The organ most commonly and seriously damaged in amyloidosis is:

Stomach
Kidney
Lung
Liver
Uterus

19. Reactive systemic amyloidosis knows to be all of the following, except:

Secondary amyloidosis
Complication of tuberculosis
Hereditary amyloidosis
Complication of osteomyelitis
Complication of bronchoectatic disease

20. Amyloid in primary amyloidosis is usually systemic and belongs to the following biochemical type:

AL type
AA type
Aβ type
ATTR type
Aβ2m type

21. Reactive systemic amyloidosis occurs in association with all of the following diseases, except:

Tuberculosis
Bronchiectasis
Chronic osteomyelitis
Rheumatoid arthritis
Hepatitis B

22. Reactive systemic amyloidosis occurs in association with all of the following diseases, except:

Rheumatoid arthritis
Ankylosing spondylitis
Chronic appendicitis
Myeloma
Tuberculosis

23. Secondary amyloidosis damages all of the following organs, except:

Kidneys
Liver
Spleen
Adrenals
Brain
24. Macroscopically the organs affected by amyloidosis are characterized by all of the following, except:

Enlarged
Firm
Waxy in appearance
Smooth
Soft

25. The common cause of death in patients with secondary amyloidosis is insufficiency of:
Kidneys
Heart
Liver
Lung
Adrenals

26. The organs that should be histological examined in patients with amyloidosis are all of the following, except:

Kidney
Eye
Rectum
Gingiva
Skin

27. The reversible process caused by accumulation of glycosaminoglycans in extracellular matrix due to the increase
of vascular permeability is:

Amyloidosis
Glycogenoses
Hyalinosis
Mucoid changes
Fibrinoid changes

28. The irreversible process caused by accumulation of proteins with high molecular weight associated with the
destruction of connective tissue is:

Amyloidosis
Glycogenoses
Hyalinosis
Mucoid changes
Fibrinoid changes

29. All the pathological processes are irreversible, except:

Mucoid changes
Fibrinoid changes
Amyloidosis
Apoptosis
Necrosis

30. In amyloidosis macroscopically kidney changed in the following way:

Cut surface is pale and translucent


Dense and sharply reduced
Enlarged and dense
Soft and enlarged
True a + c

31. All examples of extracellular hyaline are change, except:

Old scar
Hyaline arteriolosclerosis
Chronic glomerulonephritis
Mallory’s hyaline
Cornora amylacea in the brain in old age

32. In amyloidosis macroscopically liver changed in the following way:

Enlarged, pale, waxy and firm


Dense and sharply reduced
Enlarged and yellow color
Soft and enlarged
True b + c

33. In amyloidosis macroscopically spleen changed in the following way:

Cut surface is translucent pale and waxy


Dense and sharply reduced
Cut surface – map-like areas of amyloid
Soft and enlarged
True a + c

34.Causes of hyalinosis are all of the following, EXCEPT:

Fibrinoid changes.
Inflammation.
Apoptosis.
Sclerosis.
Necrosis.

35.Most specific histological sign in mucoid changes is:

Metachromasia.
Metaplasia.
Metastasis.
Metakinesis.
Metamorphosis.

36.Most common localization of mucoid changes is:

A. Nervous system.

B. Cardiovascular system.

C. Urinary system.
D. Blood system.

E. Endocrine system.

37.Simple hyaline occurs in:

Diabetes mellitus.
Arterial hypertension.
Rheumatic fever.
Rheumatoid arthritis.
Systemic lupus erythematosus.

38.Simple hyaline occurs in:

Diabetes mellitus.
Systemic lupus erythematosus.
Atherosclerosis.
Rheumatic fever.
Rheumatoid arthritis.

39.Lipohyalin occurs in:

A. Diabetes mellitus.

Arterial hypertension.
Atherosclerosis.
Rheumatic fever.
Rheumatoid arthritis.

40.Compound hyalin occurs in:

Diabetes mellitus.
Arterial hypertension.
Atherosclerosis.
D. Rheumatic fever.

E. Vasculitis.

41.Compound hyalin occurs in:

Diabetes mellitus.
Arterial hypertension.
Atherosclerosis.
Vasculitis.
E. Rheumatoid arthritis.

42.The stain used to identify amyloid is:


Hematoxylin and eosin stain.
Metachromatic stain.
Congo-red stain.
Sudan III stain.
Toluidin blue stain.
C

43.The pathologic proteinaceous substance, accumulating only between cells in various tissues and

organs of the body is:

Glycogen.
Hyaline.
Water.
D. Amyloid.

E. Lipid.

44.Specific method for diagnosis of amyloid in fresh tissue is:

Virchow test.
Rokitansky test.
Ewing test.
Masson test.
Papanicolaou test.

45.Variant of systemic amyloidosis is:

Senile cardiac.
Secondary.
Senile cerebral.
Endocrine.
Tumor-forming.

46.Variant of systemic amyloidosis is:

Senile cardiac.
Senile cerebral.
Heredofamilial.
Endocrine.
Tumor-forming.

47.Variant of systemic amyloidosis is:

Senile cardiac.
Senile cerebral.
Endocrine.
Hemodialysis-associated.
Tumor-forming.

48.Variant of localized amyloidosis is:

Senile cardiac.
Primary.
Secondary.
Heredofamilial.
Hemodialysis-associated.

49.Variant of localized amyloidosis is:

Primary.
Secondary.
Senile cerebral.
Heredofamilial.
Hemodialysis-associated.

50.Variant of localized amyloidosis is:

Primary.
Secondary.
Heredofamilial.
Endocrine.
Hemodialysis-associated.

51.Variant of localized amyloidosis is:

Primary.
Secondary.
Heredofamilial.
Hemodialysis-associated.
Tumor-forming.

52.Localization of pericollagenous amyloidosis is:

Liver.
Spleen.
Heart.
Kidneys.
Suprarenal glands.

53.Localization of pericollagenous amyloidosis is:

Liver.
Spleen.
Bowel.
Kidneys.
Suprarenal glands.

54. Localization of pericollagenous amyloidosis is:

Liver.
Spleen.
Nerves.
Kidneys.
Suprarenal glands.
55.Localization of perireticulin amyloidosis is:

Heart.
Tongue.
Nerves.
Kidneys.
Bowel.

56.Localization of perireticulin amyloidosis is:

Heart.
Tongue.
Nerves.
Liver.
Bowel.

57.Localization of perireticulin amyloidosis is:

Heart.
Tongue.
Nerves.
Spleen.
Bowel.

58.Cause of obesity which associated with excessive nutrition is called:

Primary.
Alimentary.
Cerebral.
Endocrine.
Hereditary.
B

59.Type of obesity with unknown cause is called:

Primary.
Alimentary.
Cerebral.
Endocrine.
Hereditary.

60.1st degree of obesity is associated with increasing of body weight over than normal by:

20-29%.
10-20%.
15-25%.
20-35%.
15-29%.

61.2nd degree of obesity is associated with increasing of body weight over than normal by:
30-40%.
30-49%.
25-45%.
30-55%.
35-55%.

62.3rd degree of obesity is associated with increasing of body weight over than normal by:

50-99%.
45-85%.
50-75%.
60-90%.
50-90%.

63.4th degree of obesity is associated with increasing of body weight over than normal by:

90% and more.


85% and more.
120% and more.
95% and more.
100% and more.

64.Deposition of fat in abdomen area in obesity is called:

Upper type.
Middle type.
Lower type.
Simmetric type.
Diffuse type.

65.Deposition of fat in area of face and neck in obesity is called:

Upper type.
Middle type.
Lower type.
Simmetric type.
Diffuse type.

MESENCHYMAL TUMORS

1.Malignant tumor arising from mesenchymal cells is called:

A.Leukemia.

B.Adenoma.

C.Carcinoma.

D.Sarcoma.
E.Teratoma.

2. Malignant tumors arising from mesenchymal tissue are referred to as:

A. Sarcomas

B. Adenocarcinomas

C. Papillomas

D. Cystadenomas

E. Polyps

3.Benign tumor arising from connective tissue is called:

A.Hybernoma.

B.Leiomyoma.

C.Lipoma.

D.Fibroma.

E.Osteoma.

4. Benign tumor arising from fibrous tissue called:

A. Leiomyoma

B. Myoma

C. Chondroma

D. Rhabdomyoma

E. Fibroma

5.Benign tumor arising from fatty tissue is called:

A.Hybernoma.

B.Leiomyoma.

C.Lipoma.

D.Fibroma.

E.Osteoma.
6. Select the name of a benign tumor originating from adipose tissue:

A. Adenoma

B. Lipoma

C. Chondroma

D. Rhabdomyoma

E. Liposarcoma

7. Tumor histological consisting of lobules of mature adipose cells separated by delicate fibrous septa,
called:

A. Chondroma

B. Rhabdomyoma

C. Leiomyoma

D. Lipoma

E. Fibroma

8.Benign tumor arising from brown fat is called:

A.Hybernoma.

B.Leiomyoma.

C.Lipoma.

D.Fibroma.

E.Osteoma.

9. Select the name of a benign tumor originating from brown fat:

A. Hybernoma

B. Liposarcoma

C. Chondroma

D. Rhabdomyoma

E. Fibrosarcoma
10.Benign tumor arising from smooth muscles is called:

A.Hybernoma.

B.Leiomyoma.

C.Rhabdomyoma.

D.Fibroma.

E.Osteoma.

11. Leiomyomas of the uterine are characterized by all of the following, EXCEPT:

A. Arise from uterine muscle

B. May undergo degenerative changes

C. Are hormone-dependent tumors

D. Usually are multiple tumors

E. Always undergo malignization

12.Benign tumor arising from striated muscles is called:

A.Hybernoma.

B.Leiomyoma.

C.Rhabdomyoma.

D.Fibroma.

E.Osteoma.

13.Benign tumor arising from blood vessels is called:

A.Hybernoma.

B.Leiomyoma.

C.Rhabdomyoma.

D.Hemangioma.

E.Lymphangioma.

14. The microscopical features of the cavernous hemangioma are all of the following, EXCEPT:
A. Sharply defined

B. Partly or completely filled with blood

C. Cavernous vascular spaces, sometimes with thrombosis

D. Micrometastases

E. Separated by a scant connective tissue stroma

15.Benign tumor arising from lymph vessels is called:

A.Hybernoma.

B.Leiomyoma.

C.Rhabdomyoma.

D.Hemangioma.

E.Lymphangioma.

16.Benign tumor arising from bone is called:

A.Osteoma.

B.Chondroma.

C.Synovioma.

D.Mesothelioma.

E.Osteosarcoma.

17. Benign tumor arising from osteoblasts is called:

A. Chondrosarcoma

B. Adenoma

C. Osteoma

D. Fibroma

E. Papilloma

18.Benign tumor arising from cartilage is called:

A.Osteoma.
B.Chondroma.

C.Synovioma.

D.Mesothelioma.

E.Chondrosarcoma.

19. Select the name of a benign tumor originating from cartilaginous tissue:

A. Chondroma

B. Adenoma

C. Osteoma

D. Fibroma

E. Papilloma

20.Benign tumor arising from inner layer of joints is called:

A.Osteoma.

B.Chondroma.

C.Synovioma.

D.Mesothelioma.

E.Hybernoma.

21. Malignant tumor arising from cartilaginous tissue is called:

A. Chondrosarcoma

B. Adenocarcinoma

C. Osteosarcoma

D. Fibrosarcoma

E. Papilloma

22.Malignant tumor arising from connective tissue is called:

A.Angiosarcoma.

B.Leiomyosarcoma.
C.Liposarcoma.

D.Fibrosarcoma.

E.Osteosarcoma.

23. Malignant tumor arising from fibrous tissue called:

A. Leiomyosarcoma

B. Papilloma

C. Chondrsarcoma

D. Rhabdomyoma

E. Fibrosarcoma

24.Malignant tumor arising from fatty tissue is called:

A.Angiosarcoma.

B.Leiomyosarcoma.

C.Liposarcoma.

D.Fibrosarcoma.

E.Osteosarcoma.

25. Select the name of a malignant tumor originating from adipose tissue:

A. Lipoma

B. Liposarcoma

C. Chondroma

D. Rhabdomyosarcoma

E. Fibroma

26.Malignant tumor arising from bones is called:

A.Angiosarcoma.

B.Leiomyosarcoma.

C.Liposarcoma.
D.Fibrosarcoma.

E.Osteosarcoma.

27. Malignant tumor arising from osteoblasts is called:

A. Chondrosarcoma

B. Adenocarcinoma

C. Osteosarcoma

D. Fibrosarcoma

E. Papilloma

28.Malignant tumor arising from blood vessels is called:

A.Angiosarcoma.

B.Leiomyosarcoma.

C.Liposarcoma.

D.Fibrosarcoma.

E.Osteosarcoma.

29.Malignant tumor arising from smooth muscles is called:

A.Angiosarcoma.

B.Leiomyosarcoma.

C.Liposarcoma.

D.Fibrosarcoma.

E.Osteosarcoma.

30. Select the name of a malignant tumor originating from smooth muscle cells:

A. Leiomyosarcoma

B. Papilloma

C. Chondrsarcoma

D. Rhabdomyoma
E. Fibrosarcoma

31.Malignant tumor arising from striated muscles is called:

A.Angiosarcoma.

B.Leiomyosarcoma.

C.Liposarcoma.

D.Fibrosarcoma.

E.Rhabdomyosarcoma.

32. Select the name of a malignant tumor originating from striated muscle cells:

A. Mesothelioma

B. Chondrsarcoma

C. Fibrosarcoma

D. Osteosarcoma

E. Rhabdomyosarcoma

33. Tumor histological consisting of large, round or oval cells with granulated, an eosinophilic cytoplasm;
and, as a rule, transversal stripes are in some cells, called:

A. Leiomyosarcoma

B. Papilloma

C. Chondrsarcoma

D. Rhabdomyoma

E. Fibrosarcoma

34. Benign tumor arising from melanocytes cells is called:

A. Chondroma

B. Adenoma

C. Nevus

D. Fibroma
E. Papilloma

35.Commonest type of pigmented nevus is:

A.Junctional nevus.

B.Compound nevus.

C.Intradermal nevus.

D.Juvenile nevus.

E.Blue nevus.

36.Which type of nevus often quite rich in melanin pigment?

A.Junctional nevus.

B.Compound nevus.

C.Intradermal nevus.

D.Juvenile nevus.

E.Blue nevus.

37.Which type of nevus consists of epithelioid-like cells?

A.Junctional nevus.

B.Compound nevus.

C.Intradermal nevus.

D.Juvenile nevus.

E.Blue nevus.

38.Which type of nevus has increased risk of progression to malignant melanoma?

A.Junctional nevus.

B.Compound nevus.

C.Intradermal nevus.

D.Juvenile nevus.

E.Dysplastic nevus.
39. Malignant tumor arising from melanocytes cells is called:

A. Chondrosarcoma

B. Adenocarcinoma

C. Osteosarcoma

D. Fibrosarcoma

E. Melanoma

40.Which type of melanoma is essentially a malignant melanoma in situ?

A.Lentigo maligna melanoma.

B.Superficial spreading melanoma.

C.Acral lentigenous melanoma.

D.Pagetoid melanoma.

E.Nodular melanoma.

41.Which type of melanoma occurs more commonly on the soles, palms and mucosal surfaces?

A.Lentigo maligna melanoma.

B.Superficial spreading melanoma.

C.Acral lentigenous melanoma.

D.Pagetoid melanoma.

E.Nodular melanoma.

42. The microscopical features of malignant skin melanoma are all of the following, EXCEPT:
A. Brown-black cytoplasmic granules

B. Large nuclei with irregular contours

C. Clumped chromatin under nucleolar membrane

D. Glandular formation

E. Formation of the tumor nests


43. The macroscopical features of secondary liver involvement in melanoma are all of the following,
EXCEPT:

A. Enlarged liver

B. Yellow-green color of implants

C. Multiple nodular implants

D. Black-brown color of implants

E. Tuberous surface

44.Developmental tumor is:

A.Teratoma.

B.Nephroblastoma.

C.Retinoblastoma.

D.Medulloblastoma.

E.Neuroblastoma.

45.Embryonic tumor arising from embryonic tissues is:

A.Chordoma.

B.Ameloblastoma.

C.Craniopharingeoma.

D.Medulloblastoma.

E.Branchial cysts.

46.Embryonic tumor arising from embryonic tissues is:

A.Chordoma.

B.Ameloblastoma.

C.Craniopharingeoma.

D.Nephroblastoma.

E.Branchial cysts.
47.Embryonic tumor arising from embryonic tissues is:

A.Chordoma.

B.Ameloblastoma.

C.Craniopharingeoma.

D.Neuroblastoma.

E.Branchial cysts.

48.Embryonic tumor arising from embryonic tissues is:

A.Chordoma.

B.Ameloblastoma.

C.Craniopharingeoma.

D.Hepatoblastoma.

E.Branchial cysts.

49.Embryonic tumor arising from embryonic vestiges is:

A.Nephroblastoma.

B.Ameloblastoma.

C.Retinoblastoma.

D.Medulloblastoma.

E.Neuroblastoma.

50.Embryonic tumor arising from embryonic vestiges is:

A.Nephroblastoma.

B.Chordoma.

C.Retinoblastoma.

D.Medulloblastoma.

E.Neuroblastoma.

51.Embryonic tumor arising from embryonic vestiges is:


A.Nephroblastoma.

B.Craniopharingeoma.

C.Retinoblastoma.

D.Medulloblastoma.

E.Neuroblastoma.

52.Tumor arising from hindbrain is called:

A.Nephroblastoma.

B.Craniopharingeoma.

C.Retinoblastoma.

D.Medulloblastoma.

E.Neuroblastoma.

53.Tumor arising from parapituitary residues is called:

A.Nephroblastoma.

B.Craniopharingeoma.

C.Retinoblastoma.

D.Medulloblastoma.

E.Neuroblastoma.

54.Tumor arising from enamel organ is called:

A.Ameloblastoma.

B.Craniopharingeoma.

C.Retinoblastoma.

D.Medulloblastoma.

E.Neuroblastoma.

55.Tumor arising from notochord organ is called:

A.Ameloblastoma.
B.Craniopharingeoma.

C.Retinoblastoma.

D.Chordoma.

E.Neuroblastoma.

56.Origin of Wilms tumors is:

A.Heart.

B.Lung.

C.Liver.

D.Kidney.

E.Spleen.

57.Which tumor is arised from liver?

A.Nephroblastoma.

B.Hepatoblastoma.

C.Retinoblastoma.

D.Medulloblastoma.

E.Neuroblastoma.

58.Which embryonic tumor is bening?

A.Nephroblastoma.

B.Retinoblastoma.

C.Medulloblastoma.

D.Ameloblastoma.

E.Neuroblastoma.

59.Which malignant embryonic tumor may transform to benign?

A.Nephroblastoma.

B.Hepatoblastoma.
C.Melanoma.

D.Angiosarcoma.

E.Neuroblastoma.

60.Localization of Ewing sarcoma is:

A.Bone.

B.Cartilage.

C.Fat.

D.Synovium.

E.Mesothelium.

61.Malignant tumor which associated with AIDS is called:

A.Angiosarcoma.

B.Leiomyosarcoma.

C.Chondrosarcoma.

D.Kaposi’s sarcoma.

E.Osteosarcoma.

62. A tumor that tends to spread over the surfaces of organs or body cavities rather than metastasinng via
blood vessels or lymphatics is which of the following:

A. Colon carcinoma

B. Thyroid carcinoma

C. Mesothelioma

D. Renal cell carcinoma

E. Hepatocellular carcinoma

63. First hematogenous metastases of the femur soft tissue fibrosarcoma can be found in which of the
following organs:

A. Liver
B. Brain

C. Bones

D. Lungs

E. Regional lymph node

64. Listed tumors of fibrous tissue are benign, except:

A. Fibroma durum

B. Soft fibroma

C. Mesothelioma

D. Fibrolipoma

E. Elastofibroma

65. Listed educations of fibrous tissue are tumor-like lesions, except:

A. Fibrous hamartoma of infancy

B. Soft fibroma

C. Nodular fasciitis

D. Retroperitoneal fibromatosis

E. Keloid

66. Benign tumors composed of either cavernous spaces or serpentine capillary-like channels containing
blood or lymph are all of the following, EXCEPT:

A. Capillary lymphangiomas

B. Cavernous hemangiomas

C. Capillary telangiectases

D. Cavernous lymphangiomas

E. Capillary hemangiomas

67. The most frequent localization of teratomas in children:

A ovaries
B testicles

C sacrococcygeal area

D mediastinum

E cavity of the skull

68. The most characteristic tumor for childhood is:

A hemangioma

B melanoma

C glioblastoma

D chorion carcinoma

E osteosarcoma

1. All of the following neoplasms are malignant, EXCEPT:

Adenocarcinoma
Melanoma
Seminoma
Chorionepithelioma
Papillary cystadenoma

2. Malignant tumors arising from mesenchymal tissue are referred to as:

Sarcomas
Adenocarcinomas
Papillomas
Cystadenomas
Polyps

3. The term of a benign mesenchymal tumor is constructed by combining the word designating the tumor
cell origin plus which of the following endings or words:

- sarcoma
- carcinoma
-oma
- itis
- osis(-asis)

4. A tumor that tends to spread over the surfaces of organs or body cavities rather than metastasinng via
blood vessels or lymphatics is which of the following:

Colon carcinoma
Thyroid carcinoma
Mesothelioma
Renal cell carcinoma
Hepatocellular carcinoma

5. First hematogenous metastases of the femur soft tissue fibrosarcoma can be found in which of the
following organs:

Liver
Brain
Bones
Lungs
Regional lymph node

6. Listed tumors of fibrous tissue are benign, except:

Fibroma durum
Soft fibroma
Mesothelioma
Fibrolipoma
Elastofibroma

7. Listed educations of fibrous tissue are tumor-like lesions, except:

1. Fibrous hamartoma of infancy

2. Soft fibroma

3. Nodular fasciitis

4. Retroperitoneal fibromatosis

5. Keloid

8. Benign tumor arising from fibrous tissue called:

Leiomyoma
Myoma
Chondroma
Rhabdomyoma
Fibroma

9. Malignant tumor arising from fibrous tissue called:

Leiomyosarcoma
Papilloma
Chondrsarcoma
Rhabdomyoma
Fibrosarcoma

10. Leiomyomas of the uterine are characterized by all of the following, EXCEPT:

Arise from uterine muscle


May undergo degenerative changes
Are hormone-dependent tumors
Usually are multiple tumors
Always undergo malignization

11. Benign tumor arising from smooth muscle cells called:

Leiomyoma
Myoma
Chondroma
Rhabdomyoma
Fibroma

12. Benign tumor arising from striated muscle cells called:

1. Leiomyoma

2. Myoma

3. Chondroma

4. Rhabdomyoma

5. Fibroma

13. Malignant tumor arising from smooth muscle cells called:

Leiomyosarcoma
Papilloma
Chondrsarcoma
Rhabdomyoma
Fibrosarcoma

14. Malignant tumor arising from striated muscle cells called:


Mesothelioma
Chondrsarcoma
Fibrosarcoma
Osteosarcoma
Rhabdomyosarcoma

15. Tumor histological consisting of large, round or oval cells with granulated, an eosinophilic cytoplasm;
and, as a rule, transversal stripes are in some cells, called:

Leiomyosarcoma
Papilloma
Chondrsarcoma
Rhabdomyoma
Fibrosarcoma

16. Benign tumor arising from fat tissue called:

Leiomyoma
Lipoma
Chondroma
Rhabdomyoma
Fibroma

17. Benign tumor arising from brown fat called:

Hibernoma
Lipoma
Chondroma
Rhabdomyoma
Fibroma

18. Malignant tumor arising from fat tissue called:

Leiomyosarcoma
Liposarcoma
Chondroma
Rhabdomyosarcoma
Fibrosarcoma

19. Tumor histological consisting of lobules of mature adipose cells separated by delicate fibrous septa,
called:

Chondroma
Rhabdomyoma
Leiomyoma
Lipoma
Fibroma
20. Benign tumors composed of either cavernous spaces or serpentine capillary-like channels containing
blood or lymph are all of the following, EXCEPT:

Capillary lymphangiomas
Cavernous hemangiomas
Capillary telangiectases
Cavernous lymphangiomas
Capillary hemangiomas

21. The microscopical features of the cavernous hemangioma are all of the following, EXCEPT:

Sharply defined
Partly or completely filled with blood
Cavernous vascular spaces, sometimes with thrombosis
Micrometastases
Separated by a scant connective tissue stroma

22. Benign tumor arising from melanocytes cells is called:

Chondroma
Adenoma
Nevus
Fibroma
Papilloma

23. Malignant tumor arising from melanocytes cells is called:

Chondrosarcoma
Adenocarcinoma
Osteosarcoma
Fibrosarcoma
Melanoma

24. The microscopical features of malignant skin melanoma are all of the following, EXCEPT:
Brown-black cytoplasmic granules
Large nuclei with irregular contours
Clumped chromatin under nucleolar membrane
Glandular formation
Formation of the tumor nests

25. The macroscopical features of secondary liver involvement in melanoma are all of the following,
EXCEPT:

Enlarged liver
Yellow-green color of implants
Multiple nodular implants
Black-brown color of implants
Tuberous surface
26. Benign tumor arising from osteoblasts is called:

Chondroma
Adenoma
Osteoma
Fibroma
Papilloma

27. Malignant tumor arising from osteoblasts is called:

Chondrosarcoma
Adenocarcinoma
Osteosarcoma
Fibrosarcoma
Papilloma

28. Malignant tumor arising from cartilaginous tissue is called:

Chondrosarcoma
Adenocarcinoma
Osteosarcoma
Fibrosarcoma
Papilloma

29. Benign tumor arising from cartilaginous tissue is called:

Chondroma
Adenoma
Osteoma
Fibroma
Papilloma
Pigmentations. Calcifications. (Kozmina - 72)

1. A 38-year-old female presents with intermittent pelvic pain. Physical examination reveals a 3-cm mass in the area
of her right ovary. Histological sections from this ovarian mass reveal a papillary tumor with multiple, scattered
small, round, and laminated calcifications. These structures are most likely the result of

Apoptosis
Dystrophic calcification
Enzymatic necrosis
Hyperparathyroidism
Metastatic calcification

2. The following pigments are stainable by Prussian blue reaction except:

Hemosiderin
Ferritin
Hematin
Hemochromatosis
Bilirubin

3. Idiopathic calcinosis cutis is an example of:

Necrotizing inflammation
Dystrophic calcification
Metastatic calcification
Calcified thrombi in veins
Calcified blood vessels, especially on the internal elastic lamina

4. Risk factors implicated in the etiology of cholesterol gallstones include the following except:

Family history
Obesity
Hemolytic anemia
Oral contraceptives
Geography

5. The following type of gallstones is generally unassociated with changes in the gallbladder wall:

Cholesterol
Mixed
Combined
Pigment
Uric acid

6. The following type of renal calculi is radiolucent:

Calcium oxalate
Struvite
Uric acid
Calcium phosphate
Pigment

7. The following type of renal calculi is infection-induced:

Calcium oxalate
Struvite
Uric acid
Cystine
Cholesterol

8. The color of organs hemosiderin is:

Black
Sky-blue
Yellow
Broun
Golden yellow

9. The pigment hematoidin is most identical with:

Porphyrin
Bilirubin
Hemosiderin
Hemozoin
Lipochrome

10. Lipofuscin, the golden yellow pigment sees in heart muscle

Hypertrophy
Atrophy
Hyperplasia
Metaplasia
Dysplasia

11. What stain is specific for iron:

Hematoxylin and eosin


Sudan III
Prussian blue reaction
Congo red
PAS-reaction

12. Hemosiderin has all of the following features, except:


Hemoglobin-derived
Golden yellow-to-brown
Granular or crystalline
Synthesized by enzyme tyrosinase
is aggregate of ferritin

13. Hemosiderin in the lung accumulates in:


Leukocytes
Lymphocytes
Macrophages
Fibroblasts
Erythrocytes

14. Idiopathic pulmonary hemosiderosis characterizes by all of the following pathologic symptoms, except:
Productive cough
Hemophtysis
Anemia
Heavy proteinuria
Weight loss

15. The lungs in pulmonary hemosiderosis are:


Enlarged with turbid fluid exuding from cut surface
Increased in weight with areas of red-brown consolidation
Diminished with areas of red-brown consolidation
Diminished with turbid fluid exuding from cut surface
Unchanged

16. The color of hemosiderin granules stained with Prussian blue reaction is:
Yellow
Brown
Orange-red
Pink
Blue-black

17. Hemosiderosis sees in all pathologic processes, except:


Inflammation
Hereditary increased absorption of dietary iron
Impaired use of iron
Hemolytic anemia
Malaria

18. Morphologic changes in genetic hemochromatosis characterizes by all of the following, except:
Metastatic calcification of many organs
Deposition of hemosiderin in many organs
Deposition of hemosiderin in the skin
Liver cirrhosis
Pancreatic fibrosis

19. Melanin has all of the following features, except:


Localized endogenous
Formed of ferritin
Non-hemoglobin-derived
Black-brown
Formed in melanocytes

20. An increased amount of melanin in melanocytes and within basal keratinocytes is also known as:
Vacuolization
Vitiligo
Hyperpigmentation (melanosis)
Albinism
Hyperkeratosis

21. Lipofuscin has all of the following features, except:


Aging pigment
Most often seen in kidney
Yellow-brown
Noninjurious to the cells or their function
Endogenous pigment

22. Lipofuscin pigment granules in cells result from:


Hemosiderosis
Accumulation of protein in cytoplasm
Accumulation of lipids in cytoplasm
Cellular swelling
Intracellular lipid peroxidation

23. Lipofuscin granules in cells see in:


Necrosis
Denervation atrophy
Brown atrophy
Atrophy from pressure
Atrophy from diminished blood supply

24. Heart and liver of a patient with cancer cachexia macroscopically sees as:

Diminished and brown


Diminished and yellow
Enlarged and brown
Enlarged and yellow
Unchanged

25. Lipofuscin in the liver may founds in:

Unchanged cells
Cells with ballooning degeneration
Cells with hyaline droplets
Cells with regressive changes
Necrotic cells

26. Bilirubin has all of the following features, except:


The end product of hem degradation
Derived from breakdown erythrocytes
Brilliant-yellow
Stained in blue-black color with Prussian-blue reaction
Exists in two forms - conjugated and unconjugated

27. Jaundice occurs in all of the following pathologic processes, except:


Increased hepatocellular excretion
Excessive production of bilirubin
Reduced hepatocyte uptake
Impaired conjugation of bilirubin
Impaired bile flow

28. Biliary ducts obstruction by gallstones may lead to:


Liver hemosiderosis
Liver steatosis
Ballooning degeneration of hepatocytes
Cholestasis and jaundice
Hemochromatois

29. Complications or well-established associations of gallstones include all of the following, except:

Biliary obstruction
Brown atrophy of the liver
Pancreatitis
Intestinal obstruction
Malignancy

30. Which of the following sites is an example of metastatic calcification?

The kidney in nephrocalcinosis


The mitral valve in mitral stenosis of rheumatic organ
The left anterior ascending coronary artery affected by atheromatous plaques
The lung involved by metastatic carcinoma
The lung in areas of old tuberculosis foci

31. Point out the posttuberculosis lung lesion:

Granuloma
Cavity
Fibrocalcific scar
Caseation in lymph node
Caseation in lung

32. Dystrophic calcification encounters in all of the following areas, except:

Coagulative necrosis
Intracellular fat accumulation
Caseous necrosis
Liquefactive necrosis
Enzymatic necrosis of fat

33. Initiation of intracellular calcification occurs in:

Cytoplasm
Lysosome
Nucleus
Endoplasmic reticulum
Mitochondria
34. The causes of metastatic calcification are all of the following, except:

Diabetes mellitus
Increased secretion of parathyroid hormone
Destruction of bone tissue
Vitamin D-related disorders
Renal failure

35. Metastatic calcification may occur in all of the following organs, except:

Stomach (gastric mucosa)


Kidneys
Lungs
Liver
Heart, systemic arteries and pulmonary veins

36. Deficiency of vitamin D tends to cause:

Hypercalcemia
Hypocalcemia
Hyperpigmentation
Hypopigmentation
Calcification

37. Deficiency of vitamin D in the adults leads to:

Osteosclerosis
Osteomyelitis
Osteomalatia
Osteonecrosis
Osteoporosis

38.Hemoglobin derivative pigment which occurs in normal conditions:

Hemosiderin.
Hematoidin.
Hemin.
Hemomelanin.
Porphyrin.

39.Hemoglobin derivative pigment which occurs in normal conditions:

Ferritin.
Hematoidin.
Hemin.
Hemomelanin.
Porphyrin.

40.Hemoglobin derivative pigment which occurs in normal conditions:

Bilirubin.
Hematoidin.
Hemin.
Hemomelanin.
Porphyrin.

41.Hemoglobin derivative pigment which occurs only in pathologic conditions:

Hemosiderin.
Hematoidin.
Ferritin.
Bilirubin.
Melanin.

42.Hemoglobin derivative pigment which occurs only in pathologic conditions:

Hemosiderin.
Porphyrin.
Ferritin.
Bilirubin.
Melanin.

43.Hemoglobin derivative pigment which occurs only in pathologic conditions:

Hemosiderin.
Hematin.
Ferritin.
Bilirubin.
Melanin.

44.Iron-containing pigment is:

Bilirubin.
Hematoidin.
Hemosiderin.
Porphyrin.
Melanin.

45.Iron-containing pigment is:

Bilirubin.
Hematoidin.
Ferritin.
Porphyrin.
Melanin.

46.Iron-containing pigment is:

Bilirubin.
Hematoidin.
Hematin.
Porphyrin.
Melanin.
47.The stain used to identify iron-containing pigments is:
Prussian-blue reaction.
PAS reaction.
Congo-red stain.
Sudan III stain.
Toluidin blue stain.

48.In hemosiderosis organs become:

Enlarged, brown, hard.


Small, red, soft.
Enlarged, yellow, hard.
Enlarged, brown, soft.
Small, brown, hard.

49.Cause of generalized hemosiderosis is:

Intracerebral hemorrhage.
Intoxications.
Brown induration of lungs.
Pulmonary hemorrhagic infarct.
Petechial hemorrhages in skin.

50.Cause of generalized hemosiderosis is:

Intracerebral hemorrhage.
Heterohemotransfusions.
Brown induration of lungs.
Pulmonary hemorrhagic infarct.
Petechial hemorrhages in skin.

51.Cause of generalized hemosiderosis is:

Typhoid fever.
Yellow fever.
Malaria.
Diphteria.
Scarlet fever.

52.Cause of localized hemosiderosis is:

Intoxications.
Heterohemotransfusions.
Malaria.
Bone marrow diseases.
Brown induration of lungs.

53.Cause of localized hemosiderosis is:

Intoxications.
Heterohemotransfusions.
Malaria.
Bone marrow diseases.
Intracerebral hemorrhage.

54.Cause of localized hemosiderosis is:

Intoxications.
Heterohemotransfusions.
Malaria.
Bone marrow diseases.
Pulmonary hemorrhagic infarct.

55.Accumulation of which pigment may leads to decrease of blood pressure?

Ferritin.
Hemosiderin.
Bilirubin.
Hemomelanin.
Porphyrin.

56.Accumulation of which pigment may leads to jaundice?

Ferritin.
Hemosiderin.
Bilirubin.
Hemomelanin.
Porphyrin.

57.Cause of prehepatic jaundice is:

Hepatitis.
Liver cirrhosis.
Obstruction of bile ducts by stones.
Compression of bile ducts by tumors.
Hemolytic disease of newborns.

58.Cause of prehepatic jaundice is:

Hepatitis.
Liver cirrhosis.
Obstruction of bile ducts by stones.
Compression of bile ducts by tumors.
Heterohemotransfusions.

59.Cause of hepatocellular jaundice is:

Obstruction of bile ducts by stones.


Compression of bile ducts by tumors.
Heterohemotransfusions.
Hepatitis.
Hemolytic disease of newborns.
60.Cause of hepatocellular jaundice is:

Obstruction of bile ducts by stones.


Compression of bile ducts by tumors.
Heterohemotransfusions.
Liver cirrhosis.
Hemolytic disease of newborns.

61.Cause of posthepatic jaundice is:

Obstruction of bile ducts by stones.


Hepatitis.
Heterohemotransfusions.
Liver cirrhosis.
Hemolytic disease of newborns.

62.Cause of posthepatic jaundice is:

Compression of bile ducts by tumors.


Hepatitis.
Heterohemotransfusions.
Liver cirrhosis.
Hemolytic disease of newborns.

63.Which pigment is “tear and wear”:

Bilirubin.
Hematoidin.
Hematin.
Lipofuscin.
Melanin.

64.Which pigment is tyrosin-derived:

Bilirubin.
Hematoidin.
Hematin.
Lipofuscin.
Melanin.

65.Generalized hyperpigmentation of melanin occurs in:

Addison’s disease.
Melanosis coli.
Lentigo.
Nevus.
Melanoma.

66.Focal hyperpigmentation of melanin occurs in:


Addison’s disease.
Cachexia.
Avitaminosis.
Nevus.
Xeroderma pigmentosum.

67.General hypopigmentation of melanin occurs in:

Leukoderma.
Albinism.
Vitiligo.
Cachexia.
Lentigo.

68.Focal hypopigmentation of melanin occurs in:

Albinism.
Nevus.
Vitiligo.
Cachexia.
Lentigo.

69.Distrophic calcification occurs in:

Hyperparathyroidism.
Hypervitaminosis D.
Hyperthyroidism.
Necrosis.
Increased bone catabolism.

70.Metastatic calcification occurs in:

Hyperparathyroidism.
Damaged heart valves.
Atheromas.
Necrosis.
Old thrombi.

71.Gross appearance of calcificate is:

Hard, greywish-white.
Soft, yellow.
Hard, greywish-red.
Soft, greywish-white.
Hard, black.

72.Area of calcification in histological examination with hematoxylin and eosin staining is:

Red.
Black.
Brown.
Pink.
Blue.

Prenatal pathology. Perinatal pathology


1. Duration of blastogenesis is:

А. 1st-15th days of prenatal development.

В. 1st-20th days of prenatal development.

С. 5th-20th days of prenatal development.

D. 5th-25th days of prenatal development.

Е. 1st-25th days of prenatal development.

2. Duration of embryogenesis is:

А. 16th-66th days of prenatal development.

В. 15th-75th days of prenatal development.

С. 20th-80th days of prenatal development.

D. 16th-75th days of prenatal development.

Е. 16th-80th days of prenatal development.

3. Duration of early fetogenesis is:

А. 80th-180th days of prenatal development.

В. 76th-180th days of prenatal development.

С. 70th-170th days of prenatal development.

D. 76th-175th days of prenatal development.

Е. 70th-180th days of prenatal development.

4. Duration of late fetogenesis is:

А. 180th-250th days of prenatal deveelopment.

В. 170th-280th days of prenatal development.

С. 185th-285th days of prenatal development.

D. 180th-280th days of prenatal development.


Е. 181st-280th days of prenatal development.

5. Endogenic etiologic factors of kymatopathies are:

А. Genic mutations.

В. Radiation.

С. Alcohol.

D. Metals.

Е. Infections.

6. Exogenic etiologic factors of kymatopathies are:

А. Genic mutations.

В. Endocrine diseases of mother.

С. Drugs using during pregnancy.

D. Gonads overriping.

Е. Critical periods of parents age.

7. Syndromes caused by anomalies in the system of autosomes include: a) Shereshevsky-


Ter-Cher Syndrome; b) Klinefelter's syndrome; c) Down’s syndrome; d) the Patau syndrome;
e) Edward’s syndrome

A. a,b,c,d

B. a,c,e

C. b,c

D. c,d,e

E. a,b,c,d,e

8. Characteristic of Patau syndrome is:

A polydactyly
B anencephaly

C umbilical hernia

D Kidney doubling

E agyria

9. Shereshevsky-Turner’s syndrome is characterized by everything EXCEPT:

A low growth

B short folded neck

C coarctation of the aorta

D hypoplasia of the penis

E disturbances in the formation of the ovaries

10. Down’s disease - is:

А. Gametopathy.

В. Blastopathy.

С. Embryopathy.

D. Early fetopathy.

Е. Late fetopathy.

11. Ectopic pegnancy - is:

А. Gametopathy.

В. Blastopathy.

С. Embryopathy.

D. Early fetopathy.

Е. Late fetopathy.
12. Siamese twins - is:

А. Gametopathy.

В. Blastopathy.

С. Embryopathy.

D. Early fetopathy.

Е. Late fetopathy.

13. Congenital malformation - complete absence of an organ is called:

А. Agenesis.

В. Aplasia.

С. Hypoplasia.

D. Hyperplasia.

Е. Dysplasia.

14. Congenital malformation - absence of development of an organ with presence of anlage


(rudiment) is called;

А. Agenesis.

В. Aplasia.

С. Hypoplasia.

D. Hyperplasia.

Е. Dysplasia.

15. Congenital malformation - incomplete development of an organ is called:

А. Agenesis.

В. Aplasia.

С. Hypoplasia.
D. Hyperplasia.

Е. Dysplasia.

16. Congenital malformation - overdevelopment of an organ is called:

А. Agenesis.

В. Aplasia.

С. Hypoplasia.

D. Hyperplasia.

Е. Dysplasia.

17. Congenital malformation - absence of lumen in follow organs is called:

А. Atresia.

В. Stenosis.

С. Dysraphia.

D. Extrophia.

Е. Ectopia.

18. Congenital malformation - defect resulting from failure of fusion (in a middle line of
organ) is called:

А. Atresia.

В. Stenosis.

С. Dysraphia.

D. Extrophia.

Е. Ectopia.

19. Congenital malformation - abnormal location of an organ is called:


А. Atresia.

В. Stenosis.

С. Dysraphia.

D. Extrophia.

Е. Ectopia.

20. Congenital malformation - absence of brain is called:

А. Anencephaly.

В. Exencephaly.

С. Inioncephaly.

D. Microcephaly.

Е. Hydrocephaly.

21. Congenital malformation - absence of occipital bone and location of brain in posterior
cranial fossa is called:

А. Anencephaly.

В. Exencephaly.

С. Inioncephaly.

D. Microcephaly.

Е. Hydrocephaly.

22. Congenital malformation - absence of gyri and sulci of brain is called:

А. Anencephaly.

В. Agyria.

С. Microgyria.

D. Microcephaly.
Е. Porencephaly.

23. Congenital malformation - accumulation of excessive cerebrospinal fluid within the


ventricular system of the brain:

А. Anencephaly.

В. Exencephaly.

С. Inioncephaly.

D. Microcephaly.

Е. Hydrocephaly.

24. Congenital heart disease which associated with abnormal division of cardiac cavities is:

А. Common truncus arteriosus.

В. Coarctation of aorta.

С. Ventricular septal defect.

D. Patent ductus arteriosus.

Е. Transposition of great arteries.

25. Congenital heart disease which associated with abnormal division of cardiac cavities is:

А. Common truncus arteriosus.

В. Coarctation of aorta.

С. Tricameral heart.

D. Patent ductus arteriosus.

Е. Transposition of great arteries.

26. Congenital heart disease which associated with anomalies of great arteries is:

А. Atrial septal defect.


В. Ventricular septal defect.

С. Tricameral heart.

D. Bicameral heart.

Е. Transposition of great arteries.

27. Congenital heart disease which associated with anomalies of great arteries is:

А. Atrial septal defect.

В. Ventricular septal defect.

C. Patent ductus arteriosus.

D. Tricameral heart.

E. Bicameral heart.

28. Tetralogy of Fallot is characterized by all the following, EXCEPT:

А. Atrial septal defect.

В. Ventricular septal defect.

С. Stenosis of pulmonary artery.

D. Right ventricular hypertrophy.

Е. Dextraposition of aorta.

29. Noninfectious fetopathy is:

А. Syphilis.

В. Listeriosis.

С. Toxoplasmosis.

D. Mucoviscidosis.

Е. Cytomegaly.
30. The malformation of the central nervous system includes:

A. apus

B. acrania

C. syndactyly

D. hydrocele

E. piocephaly

31. The malformation of the facial skull is:

A. Syndactyly

B. simpodium

C. synoty

D. Monarchism

E. focomelia

32. The malformation of the facial skull is:

A. cyclopia

B. Syndactyly

C. hypospadia

D. amelia

E. epispadia

33. The malformation of the genito-urinary organs is:

A. Monarchism

B. acrania

C. syndactyly

D. amelia
E. focomelia

34. The malformation of the genito-urinary organs is:

A. anarchism

B. apus

C. syndactyly

D. cyclopia

E. piocephaly

35. The malformation of the genito-urinary organs is:

A. hypospadia

B. acrania

C. amelia

D. hydrocephaly

E. cyclopia

36. The malformation of the genito-urinary organs is:

A. epispadia

B. Simpus

C. simpodium

D. apus

E. focomelia

37. The malformation of the osteoarticular system includes:

A. focomelia

B. acrania
C. hydrocephaly

D. hypospadia

E. cyclopia

38. The malformation of the osteoarticular system includes:

A. epispadia

B. amelia

C. hypospadia

D. anarchism

E. Monarchism

39. Amelia is an absence of:

A. fingers

B. toes

C. fingers and toes

D. upper and lower extremities

E. Brushes and feet

40. Simpus is:

A. fusion of fingers

B. fusion of toes

C. fusion of the lower extremities

D. fusion of upper extremities

E. absence of upper and lower extremities

41. From which day antenatal phase of perinatal period is started?


А. 150th day.

В. 154th day.

С. 156th day.

D. 160th day.

Е. 180th day.

42. Body weight of premature newborn is less than:

А. 2000 gr.

В. 2300 gr.

С. 2500 gr.

D. 2600 gr.

Е. 2650 gr.

43. Body length of premature newborn is less than:

А. 35 sm.

В. 40 sm.

С. 45 sm.

D. 48 sm.

Е. 50 sm.

44. The signs of prematurity include everything except:

A fluffy body hairs

B low lying ears

C the testicles are not lowered into the scrotum

D the nucleus of ossification of the lower epiphysis of the femur is 0.2 cm.

E nail plates fully cover the nail bed


45. Morphological signs of asphyxia are all the following, EXCEPT:

А. Cyanosis of skin.

В. Petechial hemorrhages in mucosal membranes and serosal surfaces.

С. Venous hyperemia of inner organs.

D. Formation of thrombi in blood vessels.

Е. Dark-red fluid blood in heart chambers and blood vessel lumens.

46. Which variant of neonatal respiratory distress syndrome is associated with severe
blockade of gas exchange?

А. Massive aspiration of amniotic fluid.

В. Primary atelectasis.

С. Secondary atelectasis.

D. Intra-alveolar edema and hemorrhages.

Е. Hyaline membrane disease.

47. For Hyaline membrane disease is NOT typical:

A lungs are dark red color, "rubber" consistency

B atelectasis of the lungs

C inflammatory reaction in the stroma

D hyaline membrane base - fibrin

E hemorrhage in the lumen of the alveoli

48. Which variant of neonatal respiratory distress syndrome is associated with сongenital
surfactant deficiency?

А. Massive aspiration of amniotic fluid.

В. Primary atelectasis.
С. Secondary atelectasis.

D. Intra-alveolar edema and hemorrhages.

Е. Hyaline membrane disease.

49. The presence of stillborn lungs with a greenish tinge, as well as the appearance of dense
greenish masses on the incision under pressure, points to:

A bronchopulmonary dysplasia

B diseases of hyaline membranes

C syndrome of massive aspiration of amniotic fluid and meconium

D atelectasis of the lungs

E congenital bronchiectasis

50. After the completion of what week of pregnancy does the calculation of the perinatal
period begin?

A. 20 weeks

B. 22 weeks

C. 24 weeks

D. 25 weeks

E. 28 weeks

51. Classification of the perinatal period. Specify the correct sequence: а) early neonatal; b)
intranatal; c) antenatal; d) late neonatal; e) prenatal

A. c,b,a

B. e,a,d

C. b,c,a

D. a,b,c,d

E. a,d
52. At the base of the formation of spinal hernias there is a violation of:

A the multiplication of cells

B cell migration

C cell differentiation

D apoptosis in the process of normal embryogenesis

E Adhesion of tissues

53. At the base of the formation of syndactyly there is a violation of:

A the multiplication of cells

B cell migration

C cell differentiation

D apoptosis in the process of normal embryogenesis

E Adhesion of tissues

54. What characterizes the syndrome of congenital rubella:

A cataract

B defect of the interventricular septum of the heart

C deafness

D hepatosplenomegaly

E all listed

55. With icteric form of hemolytic disease from the side of the brain takes place:

A icteric encephalitis

B icteric encephalopathy

C bilirubinosis
D meningitis

E everything is possible

56. The ascending route of infection of the fetus is more:

A Staphylococcus

B viruses

C) Listeria

D anaerobic infection

E All listed

57. Infection with cytomegalovirus of the fetus most often occurs:

A fecal-oral

B hematogenous

C lymphogenically

D ascending way

E all options are equally possible

58. For birth injury is more typical:

A epidural hemorrhage

B subarachnoid hemorrhage

C hemorrhage into the brain ventricles

D hemorrhage in the subcortical nuclei

E rupture of the cerebellum

59. To pneumopathy of newborn does NOT apply:

A hyaline membrane disease


B edematous hemorrhagic syndrome

C massive aspiration syndrome

D bronchopulmonary dysplasia

E secondary atelectasis

60. The most common complication of neonatal pneumopathy is:

A bronchiectasis

B pneumonia

C emphysema

D pneumosclerosis

E tuberculosis

61. Deficiency of the surfactant leads to:

A atelectasis

B hemorrhages

C amyloidosis

D Pneumosclerosis

E fibrinoid necrosis

62. Airless dark red lungs, "rubber" consistency characterize:

A disease of hyaline membranes

B aspiration pneumonia

C congenital bronchiectasis

D primary atelectasis

E Hypoplasia of the lungs


HEMODYNAMIC DISORDERS-II:

(63 Qs)

1. Infarction it is necrosis:
A. with different etiology.
B. with curtain localization.
C. with vascular genesis (due to blood circulation disturbances).
D. due to microcirculation disturbances.
E. in organ due to stopping of arterial blood flow.

2. What is not a morphological type of infarction?


A. white.
B. red.
C. mixed.
D. white with red rim.

3. Call the most often cause of infarction development:


A. venous congestion.
B. arterial thrombosis.
C. thrombosis of large veins.
D. microcirculatory bed embolism.
E. microcirculatory bed thrombosis.

4. The most main condition of hemorrhagic infarction development is:


A. massive blood loss.
B. arterial thrombosis.
C. venous congestion.
D. anemia.
E. anastamoses insufficiency.

5. Red infarction is usual for:


A. myocardium.
B. lung.
C. spleen.
D. kidney.
E. liver.

6. White infarction with red rim is usual for:


A. intestine.
B. skin.
C. brain.
D. myocardium.
E. liver.

7. White infarction is usual for:


A. spleen.
B. intestine.
C. lung.
D. liver.
E. skin.

8. What is wrong about lung infarction?


A. it has pyramidal form.
B. dark-red colour.
C. develops in venous congestion.
D. cyst – is its result.
E. its cause is thrombosis (or thromboembolism).

9. What is wrong about myocardial infarction?


A. it has pyramidal form.
B. it has whitish-yellowish colour.
C. it has red rim.
D. its consistency is dense.
E. thrombus there is near on endocardium always.

10. What is wrong about kidney infarction?


A. it has pyramidal form.
B. it has whitish-yellowish colour.
C. it has red rim.
D. it has soggy mass consistency.
E. its cause is thrombosis (or thromboembolism).

11. What is wrong about spleen infarction?


A. it has pyramidal form.
B. its outcome is connective tissue scar.
C. its consistency is dense.
D. its cause is thrombosis (or thromboembolism).
E. its colour is red.

12. What is wrong about brain infarction?


A. it has pyramidal form.
B. it has whitish-grayish colour.
C. its consistency is soft.
D. localizes into subcortical nuclei.
E. often develops in atherosclerosis.

13. Infarction of what organ has the most severe results?


A. spleen.
B. kidney.
C. brain.
D. lung.
E. bones.

14. Unfavorable outcome of infarction is:


A. organization.
B. petrifaction.
C. cyst formation.
D. suppuration.
E. encapsulation.

15. What does develop as myocardial infarction outcome usually?


A. cyst.
B. abscess.
C. scar.
D. hemosiderosis.
E. petrifaction.

16. What does develop as brain infarction outcome usually?


A. cyst.
B. abscess.
C. scar.
D. hemosiderosis.
E. petrifaction.

17. What does develop as kidney (spleen) infarction outcome usually?


A. cyst.
B. abscess.
C. hemosiderosis.
D. scar.
E. petrifaction.

18. Venous emboli are most often lodged in:


A. Intestines.
B. Kidney.
C. Lungs.
D. Heart.
E. Brain.

19. Pathologic changes between sudden decompression from high pressure to normal levels
and decompression from low pressure to normal levels are:
A. More marked in the former.
B. More marked in the latter.
C. No difference between the two.
D. Acute form is more marked in the latter.
E. Chronic form is more marked in the latter.

20. The infarct of following organ is invariably hemorrhagic:


A. Infarct kidney.
B. Infarct spleen.
C. Infarct lung.
D. Infarct hear.
E. Infarct liver.

21. Which of the following is true?


A. Arterial thrombi are white and occlusive.
B. Venous thrombi are white and occlusive.
C. Arterial thrombi are white and mural.
D. Venous thrombi are red and mural.
E. Venous thrombi are white and mural.

22. Call the type of shock:


A. acute.
B. hypovolemic.
C. reversible.
D. irreversible.
E. all is true.

23. What does develop in kidney in shock?


A. acute tubular necrosis.
B. inflammation.
C. hemosiderosis.
D. petrifaction.
E.) urates accumulation.

24. What is morphological change in kidney in shock observed?


A. tubular atrophy.
B. tubular necrosis.
C. stromal sclerosis.
D. inflammation.
E. all is true.

25. What does develop in lung in shock?


A.) necrosis.
B. fat degeneration.
C. disappearance of glycogen.
D. edema.
E. inflammation.

26. What is morphological change in lung in shock observed?


A. degeneration.
B. necrotic foci.
C. edema.
D. inflammation.
E. all is true.

27. What does develop in lung in shock?


A. hyperemia.
B. hemorrhage.
C. edema.
D. athelectasis.
E. all enumerated.

28. What does develop in liver in shock?


A. ischemia.
B. necrotic foci.
C. hemosiderosis.
D. sclerosis.
E. inflammatory infiltration.

29. What does develop in myocardium in shock?


A. petrifaction.
B. hemosiderosis.
C. necrosis of cardiomyocites.
D. sclerosis.
E. inflammatory infiltration.
30. In what organ ulcers and erosions develop in shock more often?
A. stomach.
B.) esophagus.
C. oral cavity.
D. rectum.
E. bronchi.

31. Pale infarction is not seen in -


A. Intestines
B. Liver
C. Spleen
D. Heart
E. Brine

32. What is main in DIC-syndrome development?


A. thrombocytopenia.
B.) anemia.
C. insufficiency of fibrinogen synthesis.
D. increased intravascular blood coagulation.
E. decreased volume of circulating blood.

33. What is the starting moment in the DIC-syndrome development?


A. coagulation of fibrinogen with the formation of fibrin.
B. appearance lot of thromboplastin in blood.
C. hypofibrinogenemia.
D. increased formation of thrombin from plasma prothrombin.
E. formation of thrombi in microcirculatory bed.

34. Disseminated intravascular coagulation (DIC) is characterized by the following except:


A. Thrombocytopenia.
B. Microangiopathic hemolytic anemia.
C. Presence of FDPs (fibrin degradation products) in the blood.
D. Normal prothrombin time.
E. Thrombin time is prolonged.

35. Most common manifestation of DIC (disseminated intravascular coagulation) is:


A. Bleeding.
B. Thrombosis.
C. Microangiopathic hemolytic anemia.
D. Organ damage.
E. High temperature.

36. A 25-year-old female presents with a history of losing four pregnancies in the past 5
years. She also has history of recurrent pains in her legs secondary to recurrent thrombosis.
Her symptoms are most likely due to a deficiency of:
A. PA inhibitiors.
B. Protein C.
C. Plasmin.
D. Thrombin.
E. C’1 inactivator.

37. Procoagulant factors produced by endothelial cells include:


A. Thrombomodulin.
B. Prostacyclin.
C. won Willebrand factor.
D. Thromboxane A2.
E. Fibrinogen.

38. A postmortem clot is most likely to


A. Grossly display features of recanalization.
B. Grossly have lines of Zahn.
C. Grossly have the appearance of «chicken fat» overlying «currant jelly».
D. Microscopically appear attached to the blood vessel.
E. Microscopically have alternating layers of cells and platelets.

39. What is the most common site of origin of thrombotic pulmonary emboli?
A. Deep leg veins.
B. Lumen of left ventricle.
C. Lumen of right ventricle.
D. Mesenteric veins.
E. Superficial leg veins.

40. A 9-year-old boy suddenly develops severe testicular pain. He is taken to the emergency
room, where he is evaluated and immediately taken to surgery. There his left testis is found to
be markedly hemorrhagic due to testicular torsion. This abnormality causes a hemorrhagic
infarction because of
A. Arterial occlusion.
B. Septic infarction.
C. The collateral blood supply of the testis.
D. The dual blood supply of the testis.
E. Venous occlusion.

41. Red infarction is caused by all of the following events, except:


A. Coronary artery thrombosis.
B. Pulmonary embolism.
C. Torsion of the testis.
D. Superior mesenteric artery thromboembolism.
E. Portal vein thrombosis.

42. Shock is commonly associated with all of the following conditions, except:
A. Escherichia coli sepsis.
B. Myocardial infarction.
C. Cholera.
D. Acute pancreatitis.
E. Cerebral infarction.

43. The fate of the thrombus may be all of the following, except:
A. Dissolution.
B. Recanalization.
C. Organization.
D. Embolization.
E. Malignization.

44. The causes of infarction include all of the following pathologic conditions, except:
A. Trombotic events.
B. Embolic events.
C. Arterial occlusion.
D. Local vasospasm.
E. Hemophilia.

45. Red infarct occurs in all of the following pathologic conditions, except:
A. Venous occlusion.
B. Coronary occlusion.
C. Loose tissue.
D. Tissues with dual circulation.
E. Previously congested with blood tissues.

46. White infarct results from which of the following:


A. Venous occlusion.
B. Arterial occlusion.
C. Loose tissue.
D. Tissues with dual circulation.
E. Previously congested with blood tissues.

47. White infarct commonly occurs in all organs, except:


A. Lung.
B. Spleen.
C. Kidney.
D. Heart.
E. Brain.

48. The formation of a thrombus is favored by all of the following, except:


A. Anemia.
B. Endothelial injury.
C. Pregnancy.
D. Stases.
E. Thrombocytosis.

49. The type of tissue necrosis commonly associated with myocardial infarction is which of
the following:
A. Caseous necrosis.
B. Coagulation necrosis.
C. Enzymatic fat necrosis.
D. Gangrenous necrosis.
E. Fibrinoid necrosis.

50. Pulmonary emboli may originate from all of the following sites, except:
A. Deep leg veins.
B. Inferior vena cava.
C. Pelvic veins.
D. Portal vein.
E. Right atrial appendage.

51. The development of endothelial-lined blood channels that reestablish blood flow through
a vascular thrombus is known as:
A. Collateral circulation.
B. Recanalization.
C. Organization.
D. Hyalinization.
E. Incapsulation.

52. The initial step of the thrombus formation is which of the following:
A. Activation ofthrombin.
B. Development of fibrin plugs.
C. Endothelial injury.
D. Marginationofleukocytes.
E. Trapping of red cells.

53. The type of tissue necrosis associated with renal infarction is:
A. Caseous necrosis.
B. Coagulation necrosis.
C. Enzymatic fat necrosis.
D. Gangrenous necrosis.
E. Liquefactive necrosis.

54. Disseminated intravascular coagulation is characterized by each of the following, except:


A. Decreased fibrinolysis.
B. Decreased plasma fibrinogen.
C. Hemolysis.
D. Prolonged partial thromboplastin time.
E. Thrombocytopenia.

55. Each of the following conditions favors the development of thrombosis, except:
A. Endothelial injury.
B. Polycythemia.
C. Stases.
D. Thrombocytopenia.
E. Congestion.

56. The type of necrosis most often caused by sudden ischemia from vascular occlusion is:
A. Apoptosis.
B. Caseous necrosis.
C. Coagulation necrosis.
D. Fat necrosis.
E. Fibrinoid necrosis.

57. All the following disorders are associated with disseminated intravascular coagulation,
except:
A. Infections.
B. Neoplasms.
C. Massive tissue injury.
D. Malnutritions.
E. Obstetric complications.

58. The usual source of pulmonary emboli is -


A. Thrombi in the deep veins of the lower extremities.
B. Thrombi in the veins of upper extremities.
C. Thrombi in the vena porta.
D. Thrombi in the left atrial appendage.
E. Thrombi in the uterus arteries.

59. Emboli in the main pulmonary artery often cause:


A. Infarction of the lung.
B. Sudden death.
C. Infarction of the heart.
D. Disseminated intravascular coagulation.
E. Acute pneumonia.

60. The origin of emboli include all of the following, except:


A. Thrombi.
B. Fat droplets.
C. Air bubbles.
D. Amyloid masses.
E. Microorganisms.

61. What is shock accompanied with often?


A. nephrotic syndrome.
B. DIC-syndrome.
C. hepatico-renal syndrome.
D. hepatico-lienal syndrome.
E. chronic renal failure.

62. The forms of embolism include all of the following, except:


A. Pulmonary thromboembolism.
B. Fat embolism.
C. Viral embolism.
D. Bacterial embolism.
E. Air embolism.

63. The selective stain, used to identify fat in the fat emboli syndrome is:
A. Sudan III.
B. Toluidin blue.
C. Congo red.
D. PAS reaction.
E. Hematoxylin and eosin.

Hemodynamic disorders – I

(62 Qs)

1. What is hematoma?
А. accumulation of blood inside serous cavities.
B. accumulation of blood in tissues without their destruction.
C. accumulation of blood in tissues with their destruction.
D. bruise.
E. petechia.

2. When does bleeding develop due to blood vessel wall erosion?


A. in purulent inflammation.
B. in chronic venous congestion.
C. in acute venous congestion.
D. in hypertensive crisis.
E. in traumas.

3. When does bleeding develop due to blood vessel rupture?


A. in purulent inflammation.
B. in chronic venous congestion.
C. in acute venous congestion.
D. in hypertensive crisis.
E. in tumours.

4. When does bleeding develop due to diapedesis?


A. in traumas.
B. in chronic venous congestion.
C. in tumours.
D. in tubal pregnancy.
E. in necrosis.

5. Hemorrhage which is associated with tissue necrosis is called:


A. hemorrhagia.
B. hematoma.
C. hemorrhagic saturation.
D. petechia.
E. bruise.

6. What can be outcome of hemorrhage?


A. hematoma.
B. organization.
C. necrosis.
D. petechia.
E. functional disturbances.

7. One of the causes of general venous hyperemia, except:


A. Congenital valvular heart disease.
B. Acquired valvular heart disease.
C. Fibroelastosis of endocardium.
D. Cardiomyopathy.
E. Acute myocardial infarction.

8. Hemorrhage into pleural cavity is -


A. Hemopericardium.
B. Metrorrhagia.
C. Hematomesis.
D. Hemothorax.
E. Hemoperitoneum.

9. Brown induration is a type of chronic venous congestion occurs in


A. Liver.
B. Heart.
C. Spleen.
D. Kidney.
E. Lung.

10. Choose the definition of the stasis:


A. decreased arterial blood flow.
B. blood viscous increasing.
C. difficulties of blood outflow.
D. stopping of blood flow in microcirculatory bed.
E. stopping of arterial blood flow.

11. The most severe result of long stasis is:


A. sludge phenomenon.
B. perivascular edema.
C. plasmorrhagia.
D. erythrocytes diapedesis.
E. necrosis of parenchymatous cells.

12. What does develop in stasis?


A. sludge phenomenon.
B. erythrocytes diapedesis.
C. perivascular edema.
D. necrosis of tissue elements.
E. all enumerated.

13. Morphologic changes in the lungs due to chronic venous hyperemia –


A. Edema.
B. Congestive hyperemia.
C. Brown induration.
D. Congestive induration.
E. Nutmeg cirrhosis.

14. What does sludge phenomenon mean?


A. adhesion of blood cells to each other.
B. erythrocytes agglutination.
C. increasing of blood cells number.
D. increasing of blood viscous.
E. stopping of blood flow in microcirculatory bed.

15. Edema it is:


A. increased blood filling of organ, tissue.
B. increased containment of interstitial fluid.
C. difficulties of venous blood outflow.
D. exudate accumulation.
E. plasmatic infiltration.

16. Generalized edema as a result of renal dysfunction or nephrotic syndrome is localized in:
A. Periorbital tissue.
B. Lung.
C. All parts of the body.
D. Brain.
E. Liver.

17. The local vacant hyperemia occurs due to -


A. Compression of arteries by tumor.
B. Decreasing of barometric pressure.
C. Obturation of arteries by thrombus.
D. Irritation of vasorelaxant nerves.
E. Increasing of barometric pressure.

18. In nephrotic syndrome edemas are:


A. hydrostatic.
B. oncotic.
C. membranogenic.
D. electrolyte.
E. due to lymphostasis.

19. In acute glomerulonephritis edemas are:


A. hydrostatic.
B. oncotic.
C. membranogenic.
D. electrolyte.
E. due to lymphostasis.

20. What is leading there in edemas development in chronic cardiac failure?


A. increased hydrostatic pressure.
B. decreasing of colloid-osmotic pressure.
C. increased aldosterone secretion.
D. damage of endothelium and basement membranes of capillars.
E. increased permeability of capillars membranes.

21. What is observed in lung edema?


A. increasing of lungs sizes.
B. increasing of lungs weight.
C. flabby consistency of lungs.
D. flowing down of foamy fluid at the incision.
E. all enumerated.

22. Pulmonary edema commonly results from all of the following, except:
A. Left ventricular failure.
B. Renal failure.
C. Systemic hypertension.
D. Appendicitis.
E. Myocardial infarction.

23. What is observed in lung edema?


A. increasing of lungs sizes.
B. decreasing of lungs weight.
C. increased air filling of lungs.
D. dense consistency of lungs.
E. all enumerated.

24. What does develop in lung edema?


A. extension of alveolars spaces.
B. accumulation of edematous fluid in alveolars spaces.
C. sclerosis of interalveolar septas.
D. deposition of hemosiderin.
E. all enumerated.

25. Organ in arterial hyperemia is characterized by:


A.Enlarged, red, hot.
B.Enlarged, blue, cold.
C.Enlarged, brown, hot.
D.Small, red, cold.
E.Small, brown, hot.

26. Hemorrhage from nasal cavity is called:


A. Maelena.
B.Haemoptoe.
C.Haematemesis.
D. Epistaxis.
E.Metrorrhagia.

27. Nutmeg liver is characterized by:


A. Enlarged, soft, yellow-grey.
B.Enlarged, hard, dark-red.
C. Enlarged, hard, yellow-grey with dark-red dots.
D.Small, hard, reddish-yellow.
E.Small. soft, yellow-red.

28. A transudate is characterized by the fallowing feature:


A. muddy.
B. bad smelling.
C. contains proteins less than 2%.
D. there is lot of cells.
E. all enumerated.
29. Microscopically, acute pulmonary congestion is characterized by all of the following,
except:
A. Engorged alveolar capillaries.
B. Thickened and fibrotic alveolar septa.
C. Edema fluid in alveoli.
D. Focal intra-alveolar hemorrhage.
E. Foci of atelectasis.

30. What does develop in lung edema?


A. hyperemia of capillars.
B. accumulation of edematous fluid in alveolars spaces.
C. narrowing of alveolars spaces.
D. erythrocytes diapedesis.
E. all enumerated.

31. Cyanotic induration of kidney is characterized by:


A.Enlarged, hard, brown.
B. Enlarged, hard, blue-red.
C. Small. soft, red.
D.Small, hard, red-brown.
E.Enlarged, soft, brown.

32. Call the certain variant of edematous fluid containment increasing:


A. hematoma.
B. ascites.
C. petechia.
D. exicosis.
E. hemothorax.

33. Microscopically, brain hemorrhage is characterized by all of the following, except:


A. Fatty changes of damaged neurons.
B. Central core of clotted blood.
C. Edema of adjacent brain tissue.
D. Dystrophy of survived neurons.
E. Reactive proliferation of astrocytes.

34. What is the cause of thrombus formation?


A. damage of blood vessel wall.
B. number of erythrocytes decreasing.
C. number of thrombocytes decreasing.
D. diapedesis of erythrocytes.
E. plasmorrhagia.

35. The most common underlying cause of primary brain parenchymal hemorrhage is which
of the following:
A. Systemic coagulation disorders.
B. Vasculitis.
C. Systemic hypertension.
D. Neoplasms.
E. Vascular malformations.

36. Macroscopically, brain hemorrhage is characterized by all of the following, except:


A. Local extravasation of blood.
B. Local damage of brain tissue.
C. Narrowed cerebral suici.
D. Distended cerebral gyri.
E. Cavitary destruction of brain.

37. Call stages of thrombus formation:


A. agglutination of thrombocytes.
B. fibrinogen coagulation.
C. agglutination of erythrocytes.
D. plasma proteins precipitation.
E. all enumerated.

38. Signs of chronic venous hyperemia are all of the following, EXCEPT:
A.Brown induration of lungs.
B.Waxy kidneys.
C.Cyanotic induration of spleen.
D.Cyanotic induration of kidneys.
E. Nutmeg liver.

39. What morphological type of thrombus is non-existent?


A. red.
B. white.
C. mixed.
D. white with red rim.
E. hyaline.

40. More often white thrombi form in:


A. veins.
B. arteries.
C. aneurysm cavity.
D. capillars.
E. heart chambers.

41. More often red thrombi form in:


A. veins.
B. arteries.
C. capillars.
D. heart chambers.
E. aorta.

42. More often hyaline thrombi form in:


A. veins.
B. arteries.
C. capillars.
D. heart chambers.
E. aorta.

43. One of unfavorable thrombus formation outcomes is:


A. organization.
B. thromboembolism.
C. petrifaction.
D. vascularization.
E. recanalization.

44. Obstructive thrombus of artery can cause:


A. venous congestion.
B. arterial hyperemia.
C. infarction.
D. thromboembolism.
E. atrophy.

45. Obstructive thrombus of vein can cause:


A. venous congestion.
B. arterial hyperemia.
C. infarction.
D. petrifaction.
E. thromboembolism.
46.Rupture of the heart wall may be result of:
A.Inflammation.
B.Myocardial infarction.
C.Invasion of cancers.
D.Hypoxia.
E.Intoxication.

47.Corrosion of the vessel wall may be result of:


A.Atherosclerosis.
B.Inflammation.
C.Vascular abnormalities.
D.Hypoxia.
E.Intoxication.

48. Favorable outcome of thrombus formation is:


a) septic autolysis.
b) suppuration.
c) organization.
d) thromboembolism.
e) obstruction of blood vessel lumen.

49. Thrombus which is consisting of alternating red thrombus particles with white thrombus
particles is called:
a) red.
b) white.
c) mixed.
d) hyaline.
e) mural glomerular.

50. Thrombus which contains lot of erythrocytes is called:


A. red.
B. white.
C. mixed.
D. flaky.
E. hyaline.

51. Thrombus which contains lot of leukocytes and fibrin is called:


A. red.
B. white.
C. mixed.
d) flaky;
e) hyaline.

52.Clinical significance of hemorrhage depends on all the following, EXCEPT:


A. Site of hemorrhage.
B.Volume of blood loss.
C.Rate of blood loss.
D. Blood group of individual.
E.Diameter of damaged vessel.

53. Large (more than 2 cm) hemorrhage into skin is called:


A.Hematoma.
B.Hemorrhagic infiltration.
C.Bruise.
D.Purpura.
E.Ecchymoses.

54. Gross characteristics of a thrombus include:


A. rough surface.
B. smooth surface.
C. contains lot of fluid.
D. it is not attached to blood vessel wall.
E. all enumerated is right.

55. Gross characteristics of a thrombus include:


A. rough surface.
B. crimped surface.
C. dull surface.
D. it is attached to blood vessel wall.
E. all enumerated is right.

56. Thromboembolism of pulmonary trunk and its large brunches results as:
A. pulmonocoronary reflex.
B. lung infarction.
C. athelectasis.
D. shock.
E. DIC-syndrome.

57. Call the localization of thrombi in pulmonary thromboembolism:


A. valvulars of the left part of heart.
B. aorta.
C. arteries of large circulation.
D. veins of large circulation.
E. veins of lesser circulation.

58. Call the localization of thrombi in large circulation arteries thromboembolism:


A. valvulars of the left part of heart.
B. valvulars of the right part of heart.
C. veins of lesser circulation.
D. veins of large circulation.
E. arteries of lesser circulation.

59. Fat embolism of what organs capillars is most dangerous?


A. kidney.
B. liver.
C. lungs.
D. intestine.
E. spleen.

60. Call the outcome of large circulation arteries thromboembolism:


A. hyperemia of inner organs.
B. infarctions in organs.
C. edema.
D. exicosis.
E. cachexia.

61. Pulmonocoronary reflex develops in:


A. fat embolism of lung blood vessels.
B. amniotic fluid embolism.
C. microbe embolism of lung blood vessels.
D. pulmonary trunk thromboembolism.
E. thromboembolism of pulmonary artery small brunches.

62. Fat embolism is possible in:


A. ulceration and disattachment of atherosclerotic plaque particles.
B. massive traumas of subcutaneous fat tissue.
C. mistaken intramuscular injections of oil-based drugs.
D. amniotic fluid embolism.
E. all enumerated.
Introduction to Pathologic Anatomy

1. The objects of the study of pathological anatomy are:


A. cadaveric material
B. substrates obtained from patients during life
C. experimental material
D. Medical records
E. correct answers are ABC

2. Depending on the method of obtaining the diagnostic material, a biopsy can be:
A. invasive
B. Traction
C. aspiration
D. expiring
E. non-invasive

3. Indicate the term " autopsy":


A. digestion of cells and tissues of the body
B. examination of the cadaver
C. intravital tissue taking for diagnostic purposes
D. cell death in a living body
E. intravital decrease in the size of cells, tissues, organs

4. The term "etiology" as applied to the disease means:


A complications
B The mechanism of death
C the cause of development
D development mechanism
E clinical manifestations

5. Indicate the levels of study of the pathological process, except:


A tissue
B Organic
C cellular
D System
E Population

6. Groups of objects studied by the pathologoanatomist, EXCEPT:


A cadaveric material
B experimental material
C substrates from living patients
D biopsy data
E data from sociological survey

7. Pathoanatomical autopsy is performed in case of death from:


A drowning
B disease
C hanging
D poisoning
E gunshot wound
8. Methods for examining cadaveric material, EXCEPT:
A biochemical
B X-ray
C microbiological
D electrocardiographic
E Histological

9. In the course of systemic pathanatomy study:


A Adaptation processes
B kidney disease
C inflammation and healing
D Circulatory disorders
E pathology of the immune system

10. Taking a piece of organs tissue with a diagnostic purpose during the life of the patient is
called:
A biopsy
B autopsy
C Necropsy
D electrocardiography
E X-ray

11. Immunohistochemical examination is used for, EXCEPT:


A detection of autoantigens
B Determining the sex of the child
C determination of tumor histogenesis
D determination of the degree of differentiation of malignant tumors
E establishing the type of infectious agent

12. Methods of molecular biology:


A In situ hybridization
B chromosome analysis
C gene analysis
D Scanning electron microscopy
E Transmission electron microscopy

13. For quantitatively analyze the DNA content in tumor cells used:
A In situ hybridization
B chromosome analysis
C Flow cytometry
D Scanning electron microscopy
E Transmission electron microscopy

14. The area of using the hybridization method in situ is:


A Diagnosis of tumors
B Cloning of cells and tissues
C detection of tissue necrosis zone
D determination of the sex of the child
E Recognition of the nature of inflammation
15. Basophilic structure in tissues is:
A cytoplasm of cells
B erythrocytes
C collagen fibers
D Reticular fibers
E Deposits of calcium salts

16. Picrofuxin staining by Van Gieson selectively reveals:


A nerve fibers
B smooth muscle cells
C secreting mucus epithelium
D macrophage of connective tissue
E collagen fibers of connective tissue

17. Electron microscopy is needed to identify everything except:


A viruses in tissues
B bacteria in the tissues
C immune complexes with pemphigoid
D B-cell and T-cell lymphomas
E immune complexes with glomerulonephritis

18. Flow cytometry for DNA content analysis allows you to determine everything EXCEPT:
A aneuploidy
B Diploidity
C presence of pathogens
D number of dividing cells
E number of resting (stable) cells

19. The hybridization technique is used for everything, EXCEPT:


A identification of viral DNA
B Diagnosis of certain anemia
C Identification of T-cell lymphomas
D studying the genome with its congenital disorders
E differentiation of breast cancer and ovarian cancer

20. The term "pathogenesis" as applied to the disease means:


A complications
B cause of development
C clinical manifestations
D the development mechanism
E mechanism of death

21. The term "tanatogenesis" as applied to the disease means:


A complications
B cause of development
C clinical manifestations
D the development mechanism
E mechanism of death
22. The term "pathomorphosis" as applied to the disease means:
A complications
B cause of development
C variability
D the development mechanism
E mechanism of death

23. Selective for glycogen staining:


A Sudan III
B hematoxylin and eosin
C picrofuxin
D Schiff (PAS) -reaction
E toluidine blue

24. Lipid-selective staining:


A Sudan III
B hematoxylin and eosin
C picrofuxin
D Schiff (PAS) -reaction
E toluidine blue

25. The terminal state with absence of cardiac activity without signs of irreversible changes
in the gray matter of the brain indicates:
A sudden death
B clinical death
C biological death
D death from disease
E natural death

26. Beloglazov's sign at death is:


A brownish spots on the sclera
B opacity of the cornea
C dilated pupils
D pupillary contraction
E "cat's eye"

27. Selective staining for amyloid:


A Sudan III
B. hematoxylin and eosin
C Congo red
D Schiff (PAS) -reaction
E toluidine blue

28. Selective staining for mucus:


A picrofuxin
B Sudan III
C Congo red
D mucicarmine
E toluidine blue
29. A reliable sign of death:
A Decrease in body temperature less than 22°C
B absent of independent breathing
C absent of pulse
D absent of heart beats
E absent of both breathing and heart beats

30. Unreliable sign of death:


A stable areflexion
B cadaveric rigor mortis
C lack of independent breathing
D cadaveric spots
E Decrease in body temperature less than 22°C
Cell Injury

1. Fibrinoid necrosis is typical for:


A. syphilis
B. lymphogranulomatosis
C. rheumatism
D. tuberculosis
E. acute pancreatitis

2. The primary localization of Liquefactive necrosis is:


A. heart
B. spleen
C. brain
D. kidney
E. lower extremities

3. Favorable outcome of necrosis is:


A. trombosis
B. Encapsulation
C. suppuration
D. ulceration
E. embolism

4. The outcome of the Liquefactive necrosis of the brain can be:


A. petrification
B. ossification
C. cyst formation
D. scarring
E. Encapsulation

5. The organization is a favorable outcome of:


A. embolism
B. necrosis
C. pathological accumulation (dystrophy)
D. inflammation
E. induration

6. The organization is the outcome of necrosis, characterized by:


A. replacement with connective tissue
B. formation of a capsule
C. deposition of calcium
D. formation of a cyst
E. suppuration

7. Encapsulation is the outcome of necrosis, characterized by:


A. replacement with connective tissue
B. formation of a capsule
C. deposition of calcium
D. formation of a cyst
E. suppuration
8. Petrification is the outcome of necrosis, characterized by:
A. replacement with connective tissue
B. formation of a capsule
C. deposition of calcium
D. formation of a cyst
E. bone formation

9. Outcome of necrosis can be:


A. Organization
B. embolism
B. infarction
D. malignancy
E. thrombosis

10. Definition of "necrosis":


A anemia of the organ
B dystrophy
C the fullness of the organ
D death of an organism
E necrosis of tissues in a living body

11. Microscopic signs of cell necrosis:


A fatty degeneration
B karyopicnosis
C karyokinesis
D Degranulation
E Protein dystrophy

12. Favorable outcome of dry necrosis:


A Organization
B septic autolysis
C regeneration
D cyst formation
E formation of a sequestration

13. Type of gangrene:


A dense
B dry
C partial
D Toxic
E Vascular

14. Definition of the concept of "necrobiosis":


A spasm of blood vessels
B anemia of organs
C irreversible dystrophic processes
D autolysis
E pathobiosis
15. Microscopic signs of cell necrosis:
A plasmorrhagia
B Plasmatization
C karyorexis
D karyokinesis
E accumulation of proteins in the cytoplasm

16. Characteristic outcome of wet necrosis:


A encapsulation
B petification
C mummification
D ossification
E cyst

17. In which organs the development of gangrene is possible:


A the brain
B the spleen
C intestine
D heart
E kidney

18. Select the stages of the necrotic process, EXCEPT:


A plasmolysis
B plasmorexis
C necrobiosis
D Cariolysis
E karyorexis

19. Enzyme that take part in the autolysis of cell nuclei:


A acidic phosphotase
B DNAase
C alkaline phosphatase
D cytochrome oxidase
E transferase

20. Etiological type of necrosis:


A infarction
B dry
C sequestration
D trophoneurotic
E Wet

21. Characteristic of tissues with dry gangrene:


A edema
B mummification
C anemia
D fullness
E Flabby consistency

22. Characteristic of the infarction:


A traumatic necrosis
B direct necrosis
C Vascular necrosis
D trophoneurotic necrosis
E caseous necrosis

23. Cell ultrastructure that determine autolytic enzymatic processes in a cell:


A Golgi apparatus
B mitochondria
C lysosomes
D endoplasmic reticulum
E microsomes

24. Causes of direct necrosis:


A nerve damage
B standstill of blood flow
C spasm of blood vessels
D the effect of acids
E rupture of blood vessels

25. Frequent localization of dry gangrene:


A extremities
B intestine
C heart
D liver
E kidneys

26. Characteristic of vascular necrosis:


A direct
B traumatic
C Indirect
D trophoneurotic
E allergic

27. Characteristics of bedsore:


A traumatic necrosis
B a kind of gangrene
C toxic necrosis
D myocardial infarction
E allergic necrosis

28. What is formed as a result of necrosis of cells and extracellular substance:


A Pigments
B polysaccharides
C hemoglobin
D tissue detritus
E connective tissue

29. What necrosis developed under the action of chemical and physical factors:
A Mechanical
B vascular
C trophoneurotic
D traumatic
E allergic

30. Frequent localization of wet gangrene:


A intestine
B spinal cord
C umbilical cord
D spleen
E liver

31. What is infarction:


A toxic necrosis
B allergic necrosis
C direct necrosis
D Vascular necrosis
E Vascular spasm

32. What develops around the focus of necrosis in living tissues:


A autolysis
B tissue anemia
C demarcation inflammation
D necrobiosis
E calcification

33. Cause of Vascular Necrosis:


A arterial hyperemia
B anemia
C thrombosis of the vessel
D hemorrhage
E collateral circulation

34. The process that constitute the essence of necrosis:


A chronic venous congestion
B tissue anemia
C protein coagulation
D sclerosis
E accumulation of the polysaccharides in the tissues

35. The substance that determines the color of gangrenous tissues:


A hemomelanin
B lipofuscin
C hydrochloric acid hematin
D melanin
E sulphurous iron

36. What is necrosis:


A death of an organism
B death of organs and tissues in the living body
C calcification of tissues
D standstill of breathing
E cardiac arrest

37. Adverse outcome of necrosis:


A suppuration
B ossification
C Organization
D encapsulation
E petification

38. Frequent localization of Liquefactive necrosis:


A liver
B brain
C myocardium
D kidneys
E smooth musculature

39. Classification of necrosis by the mechanism of action of the etiologic factor:


A direct
B gangrene
C sequestration
D infarction
E bedsore

40. What necrosis develops with the predominance of colliquation processes:


A coagulation
B dry
C Wet
D caseous
C waxy

41. Diseases in which fibrinoid necrosis often develops:


A intestinal infections
B rheumatic diseases
C influenza
D leukemia
E childhood infections

42. Synonym of vascular necrosis:


A infarction
B dystonic
C allergic
D Atonic
E dystrophic

43. Clinico - morphological forms of necrosis:


A direct
B Indirect
C infarction
D vascular
E traumatic

44. What necrosis develops in a sensitized organism:


A trophoneurotic
B toxic
C allergic
D traumatic
E) infarction

45. Characteristic of the demarcation zone:


A zone of ischemia
B accumulation of leukocytes and hyperemia
C accumulation of erythrocytes
D accumulation of immunocompetent cells
E Vascular spasm

46. The disease, accompanied by the development of caseous necrosis:


A intestinal infections
B atherosclerosis
C tuberculosis
D rheumatism
E hypertensive disease

47. Choose the favorable outcome of necrosis:


A sepsis
B septic melting
C encapsulation
D mummification
E sequestration

48. What necrosis develops from exposure to low and high temperatures:
A direct
B Indirect
C Toxic
D trophoneurotic
E vascular

49. A variety of coagulation necrosis:


A sequestration
B Caseous
C colliquative
D infarction
E bedsore

50. What etiological kind of necrosis develops under the phenomenon of Arthus:
A vascular
B toxic
C traumatic
D allergic
E trophoneurotic

51. Typical localization of wet necrosis:


A myocardium
B brain
C smooth muscles
D striated musculature
E bone tissue

52. Changes in the stroma of the organ with necrosis:


A ischemia
B hyalinosis
C amyloidosis
D fibrinoid necrosis
E hemosiderosis

53. A synonym of focal colliquative necrosis of the cell:


A fatty degeneration
B granular degeneration
C balloon dystrophy
D hyaline-drop dystrophy
E mucoid changes

54. Frequent localization of infarction:


A lower limbs
B heart
C liver
D stomach
E gallbladder

55. At what disease often there is a vascular necrosis:


A tuberculosis
B atherosclerosis
C influenza
D anemia
E leprosy

56. What is sequestration:


A a kind of gangrene
B etiological kind of necrosis
C a kind of calcification
D area of necrosis not subjected to autolysis
E a kind of infarction

57. Frequent localization of fibrinoid necrosis:


A Muscles
B nervous tissue
C bone tissue
D the wall of the vessel
E adipose tissue

58. Typical localization of Liquefactive necrosis:


A Bone tissue
B connective tissue
C brain
D myocardium
E the spleen

59. What necrosis develops with tuberculosis and syphilis:


A waxy
B caseous
C colliquative
D vascular
E traumatic

60. A type of cell damage is:


A Metaplasia.
B Atrophy.
C dysplasia.
D Necrosis.
C sclerosis.

61. At autopsy performed on the 3rd day after the death of a patient with myocardial
infarction, signs of autolysis in all organs were found macroscopically. For differential
diagnosis between necrosis and post-mortem autolysis, a histological sign can be used:
A Caryolysis
B karyorexis
C Plasmolysis
D plasmorexis
E demarcation inflammation

62. Histologically, in necrotic cells:


A nuclei stained with eosin more
B cytoplasm is less basophilic than usual
C nuclei stained with hematoxylin less
D nuclei stained with hematoxylin more
E cytoplasm is stained with hematoxylin more

63. The level of serum creatine kinase increases with necrosis of:
A kidneys
B myocardium
C brain
D pancreas
E liver

64. Select the sign of apoptosis:


A Cariolysis
B activation of endonucleases
C activation of DNA synthesis
D demarcation inflammation
E reduction of free calcium in the cytosol

65. The programmed cell death, which occurs normally in the fetal organs, is called:
A apoptosis
B heterolysis
C fibrinoid necrosis
D autolysis
E heterophagy

66. Marker of irreversible damage to cardiomyocytes during ischemia is:


A cell swelling
B Condensation of chromatin
C inclusion of lipids in the cytoplasm
D Calcium deposits in the mitochondria
E disappearance of glycogen from the cytoplasm

67. Red infarction develops as a result of:


A Splenic artery thrombosis
B carotid thrombosis
C coronary artery thrombosis
D embolism of the pulmonary artery branch
E thrombosis of the femoral artery

68. Geometric form of myocardial infarction:


A Round
B Oval
C Triangular
D V-shaped
E irregular

69. The demarcation line with dry gangrene is:


A Clear
B unclear
C absent
D irregular
E vertical

70. The demarcation line with wet gangrene is:


A Clear
B unclear
C absent
D hemorragic
E vertical

71. Wet gangrene of the foot often develops in patients with:


A Vasculitis
B pancreatitis
C Atherosclerosis
D diabetes mellitus
E hypertensive disease

72. Dry gangrene of the foot often develops in the patient with:
A pneumonia
B pancreatitis
C Atherosclerosis
D diabetes mellitus
E hypertensive disease

73. Fatty (enzymatic) necrosis most often develops in patients with:


A hepatitis
B enteritis
C gastritis
D pancreatitis
E pneumonia

74. Lymph nodes in tuberculosis are:


A dark red and enlarged
B white and yellow and enlarged
C dark red and reduced
D white and yellow and reduced
E black and reduced

75. In the myocardium with ischemia disappear:


A drops of lipids
B granules of glucose
C granules of Glycogen
D Lipofuscin granules
E Cholesterol crystals

76. Apoptotic bodies are exposed by:


A autolysis
B heterolysis
C encapsulation
D phagocytosis
E petrification

77. Changes in the nucleus of the cell during apoptosis occur under the action of:
A aminotransferases
B hydrolytic enzymes
C proteolytic enzymes
D Ca-Mg-dependent endonucleases
E DNase

78. Cells phagocytizing apoptotic bodies are:


A macrophages
B fibroblasts
C lymphocytes
D eosinophils
E mast cells

79. Component of apoptotic bodies is:


A nucleus with nucleolus
B lipid vacuoles
C giant mitochondria
D Lysosomes
E Condensed fragments of chromatin

80. Apoptotic bodies histologically look like:


A granular balls
B basophilic blots
C eosinophilic crystals
D Basophilic oval corpuscles
E eosinophilic rounded masses

81. Apoptotic bodies in the tissue are best seen in stainig:


A Sudan III
B hematoxylin and eosin
C picrofuxin
D Schiff (PAS) -reaction
E toluidine blue

82. Type of cell death is:


A necrosis
B hyperhidrosis
C mummification
D swelling
E fragmentation

83. The main morphological signs of apoptosis are all, EXEPT:


A cell shrinkage
B damage to organelles
C preservation of integrity of organelles
D formation of apoptic bodies
E condensation of nuclear heterochromatin

84. Physical agents that cause cell damage are all, EXEPT:
A radiation
B mechanical trauma
C electric current
D excessive heating
E acid and alkali

85. Histological signs of necrosis after total myocardial ischemia appear through:
A 24 hours
B 10 min
C 10 - 12 hours
D 20 - 60 minutes
E 1 hour
86. The most important mechanism of cell damage and death:
A lipolysis
B phagocytosis
C depletion of ATP stocks
D vacuolation of the cytoplasm
E Preservation of the selective permeability of cell membranes

87. The most common cause of ischemic and hypoxic damage is:
A Vasodilation
B blockage of veins
C occlusion of the artery
D Development of collaterals
E decreased blood pressure

88. If the infarct reaches the surface of the organ, covered with a serous membrane, it
develops
A edema
B hyalinosis
C granulation tissue
D catarrhal inflammation
E fibrinous inflammation

89. Choose positions that are correct with respect to gangrene:


A gangrene - necrosis of tissues in contact with the external environment
B sequestration - a kind of gangrene
C gangrene of the intestine is always dry
D Gangrene of the extremities can be only wet
C Color of gangrenous tissues due to the accumulation of hydrochloric acid hematin

90. Choose the positions that are correct for necrosis:


A the cytoplasm of necrotic cells is less eosinophilic
B pycnotic nuclei stained with hematoxylin more
C fat necrosis in the pancreas in pancreatitis is calcified
D with caseous necrosis, cells retain their shape
E Liquefactive necrosis can develop in tissues due to attachment of pyogenic infection

91. Each type of necrosis is correctly correlated with the organs for which it is typical, except
for
A coagulation necrosis - heart, kidneys
B Liquefactive necrosis - brain
C caseous necrosis - a variety of organs
D fibrinoid necrosis - the walls of blood vessels
E fat necrosis - muscles of the anterior wall of the abdomen

92. What histological sign can be used for differential diagnosis between necrosis and post
mortem autolysis:
A karyolysis
B plasmorexis
C karyorexis
D demarcation inflammation
E Plasmolysis

93. Indicate reversible changes in the structure of the cell in response to damage:
A Dissociation of the polisomes
B loss of microvilli
C mitochondrial swelling
D Expansion of endoplasmic reticulum tanks
E Violation of the integrity of cytoplasmic membranes

94. Mechanism of pathogenic action of clostridium perfringens is:


A. damage to glycolysis
B activation of glycolysis
C Inactivation of cytochrome oxidase
D blockade of the citric acid cycle
E development of "aggressive" phospholipids

95. Gangrene can develop in all, EXEPT:


A kidney
B uterus
C intestine
D lungs
E Soft tissues of extremities

96. The development of both coagulation and liquefactive necrosis is possible in:
A kidney
B intestine
C lungs
D brain
E soft tissues of extremities

Pigmentations. Calcification.

1.Which type of pneumoconiosis is associated with inhalation of carbon particles?

A.Anthracosis.

B.Silicosis.

C.Asbestosis.

D.Siderosis.
E.Talcosis.

2.Which type of pneumoconiosis is associated with inhalation of iron particles?

A.Anthracosis.

B.Silicosis.

C.Asbestosis.

D.Siderosis.

E.Talcosis.

3.Inhalation of foreign particles leads to development in the lung:

A.Acute purulent pneumonia.

D.Obstructive emphysema.

C.Chronic inflammation and fibrosis.

D.Acute abscess.

E.Pulmonary infarction.

4.Exogenic pigmentation as a result of inhalation of the foreign particles is:

A.Argyria.

B.Asbestosis.

C.Carotenemia.

D.Lead poisoning.
E.Tattoing.

5.Exogenic pigmentation as a result of inhalation of the foreign particles is:

A.Argyria.

B.Tattoing.

C.Carotenemia.

D.Lead poisoning.

E.Anthracosis.

6.Exogenic pigmentation as a result of inhalation of the foreign particles is:

A.Silicosis.

B.Tattoing.

C.Carotenemia.

D.Lead poisoning.

E.Argyria.

7.Exogenic pigmentation as a result of inhalation of the foreign particles is:

A.Carotenemia.

B.Tattoing.

C.Siderosis.

D.Lead poisoning.
E.Argyria.

8.Exogenic pigmentation as a result of inhalation of the foreign particles is:

A.Carotenemia.

B.Tattoing.

C.Argyria.

D.Talcosis.

E.Lead poisoning.

9.Exogenic pigmentation as a result of ingestion of the foreign substances is:

A.Argyria.

B.Silicosis.

C.Asbestosis.

D.Siderosis.

E.Talcosis.

10.Exogenic pigmentation as a result of ingestion of the foreign substances is:

A.Anthracosis.

B.Silicosis.

C.Carotenemia.

D.Siderosis.
E.Talcosis.

11.Exogenic pigmentation as a result of ingestion of the foreign substances is:

A.Anthracosis.

B.Lead poisoning.

C.Silicosis.

D.Siderosis.

E.Talcosis.

12.Exogenic pigmentation as a result of injection of the foreign substances is:

A.Anthracosis.

B.Lead poisoning.

C.Silicosis.

D.Tattoing.

E.Siderosis.

13.Argyria is as a result of ingestion of the:

A.Iron.

B.Silver.

C.Gold.

D.Aluminium.

E.Lead.
14.Characteristic blue lines on teeth at the gumline occurs in:

A.Chronic lead poisoning.

B.Chronic iron poisoning.

C.Chronic silver poisoning.

D.Chronic gold poisoning.

D.Chronic copper poisoning.

15.Brownish pigmentation in the skin, bowel and kidney occurs in:

A.Argyria.

B.Tattoing.

C.Carotenemia.

D.Lead poisoning.

E.Anthracosis.

16.Yellowish-red coloration of the skin occurs in:

A.Argyria.

B.Tattoing.

C.Carotenemia.

D.Lead poisoning.

E.Anthracosis.
17.Tattoing is a result of introduction into the dermis:

A.Iron.

B.Silver.

C.Copper.

D.Carbon.

E.Gold.

18.Tattoing is a result of introduction into the dermis:

A.Iron.

B.Silver.

C.Copper.

D.Asbestos.

E.Cinnabar.

19.Tattoing is a result of introduction into the dermis:

A.Iron.

B.India ink.

C.Copper.

D.Asbestos.

E. Silver.
20.Exogenic pigmentation is:

A.Hemosiderosis.

B.Melanosis.

C.Lipofuscinosis.

D.Jaundice.

E.Anthracosis.

21.Exogenic pigmentation is:

A.Hemosiderosis.

B.Melanosis.

C.Lipofuscinosis.

D.Jaundice.

E.Asbestosis.

22.Exogenic pigmentation is:

A.Siderosis.

B.Melanosis.

C.Lipofuscinosis.

D.Jaundice.

E.Hemosiderosis.
23.Exogenic pigmentation is:

A.Argyria.

B.Ferritinemia.

C.Porphyria.

D.Jaundice.

E.Hemosiderosis.

24.Exogenic pigmentation is:

A.Porphyria.

B.Ferritinemia.

C.Carotenemia.

D.Jaundice.

E.Hemosiderosis.

25.Exogenic pigmentation is:

A.Porphyria.

B.Ferritinemia.

C.Tattoing.

D.Jaundice.

E.Hemosiderosis.
26.Endogenic pigmentation is:

A.Argyria.

B.Ferritinemia.

C.Carotenemia.

D.Tattoing.

E.Anthracosis.

27.Endogenic pigmentation is:

A.Argyria.

B.Silicosis.

C.Carotenemia.

D.Hemosiderosis.

E.Anthracosis.

28.Endogenic pigmentation is:

A.Argyria.

B.Silicosis.

C.Carotenemia.

D.Jaundice.

E.Anthracosis.

29.Endogenic pigmentation is:


A.Argyria.

B.Silicosis.

C.Siderosis.

D.Anthracosis.

E.Melanosis.

30.Endogenic pigmentation is:

A.Argyria.

B.Silicosis.

C.Siderosis.

D.Anthracosis.

E.Lipofuscinosis.

31.Hemosiderin histologically appears as:

A.Golden-yellow to brown pigment.

B.Pinkish to red pigment.

C.Pale-green to dark-green pigment.

D.Dark-blue to violet pigment.

E.Black pigment.

32.“Heart failure cells” are the alveolar macrophages, containing pigment which is called:
A.Hemosiderin.

B.Hematin.

C.Ferritin.

D.Bilirubin.

E.Melanin.

33.Cause of generalized hemosiderosis is:

A.Intracerebral hemorrhage.

B. Pulmonary hemorrhagic infarct.

C.Brown induration of lungs.

D.Increased absorbtions of dietary iron.

E.Petechial hemorrhages in skin.

34.Cause of generalized hemosiderosis is:

A.Intracerebral hemorrhage.

B. Pulmonary hemorrhagic infarct.

C.Brown induration of lungs.

D.Impaired utilization of iron.

E.Petechial hemorrhages in skin.

35.Cause of generalized hemosiderosis is:


A.Intracerebral hemorrhage.

B. Pulmonary hemorrhagic infarct.

C.Brown induration of lungs.

D.Repeated transfusions of blood.

E.Petechial hemorrhages in skin.

36.Hereditary disease which associated with excessive intestinal absorbtion of iron and deposition of
hemosiderin is called:

A.Hemosiderosis.

B.Lipofuscinosis.

C.Melanosis.

D.Hemachromatosis.

E.Calcinosis.

37.Morphological features of hemachromatosis are all the following, EXCEPT:

A.Pigmentary liver cirrhosis.

B.Myocardial infarction.

C.Pigmentary cardiomyopathy.

D.Diabetes mellitus.

E.Brown skin pigmentation.

38.Cause of prehepatic jaundice is:


A.Hepatitis.

B.Liver cirrhosis.

C.Obstruction of bile ducts by stones.

D.Compression of bile ducts by tumors.

E.Intoxications.

39.Cause of prehepatic jaundice is:

A.Hepatitis.

B.Liver cirrhosis.

C.Obstruction of bile ducts by stones.

D.Compression of bile ducts by tumors.

E.Infections.

40.Cause of hepatocellular jaundice is:

A.Obstruction of bile ducts by stones.

B.Compression of bile ducts by tumors.

C.Heterohemotransfusions.

D.Hepatosis.

E.Hemolytic disease of newborns.

41.Cause of posthepatic jaundice is:

A.Atresia of bile ducts.

B.Hepatitis.
C.Heterohemotransfusions.

D.Liver cirrhosis.

E.Hemolytic disease of newborns.

42.Cause of posthepatic jaundice is:

A.Stenosis of bile ducts.

B.Hepatitis.

C.Heterohemotransfusions.

D.Liver cirrhosis.

E.Hemolytic disease of newborns.

43.Pigment which is a result of reaction between hemoglobin and hydrochloric acid is called:

A.Hemosiderin.

B.Hemin.

C.Ferritin.

D.Bilirubin.

E.Melanin.

44.Pigment which is a result of reaction between hemoglobin and malarial toxin is called:

A.Hemosiderin.

B.Hemin.
C.Hemomelanin.

D.Bilirubin.

E.Melanin.

45.Histologically hematin may be seen as:

A.Yellow granules.

B.Red granules.

C.Green granules.

D.Black granules.

E.Blue granules.

46.Pigment which is formed in tissues where a good oxygen supply is lacking in dead tissues is called:

A.Hemosiderin.

B.Hemin.

C.Hemomelanin.

D.Bilirubin.

E.Hematoidin.

47.Histologically hematoidin may be seen as amorphous:

A.Yellow granules.

B.Red granules.

C.Green granules.

D.Black granules.
E.Blue granules.

48.Pigment which is associated with highly photosensitivity and damage of skin is called:

A.Hemosiderin.

B.Hemin.

C.Hemomelanin.

D.Bilirubin.

E.Porphyrin.

49.Generalized acquired hyperpigmentation of melanin occurs in:

A.Addison’s disease.

B.Melanosis coli.

C.Lentigo.

D.Nevus.

E.Melanoma.

50.Generalized acquired hyperpigmentation of melanin occurs in:

A.Avitaminosis.

B.Melanosis coli.

C.Lentigo.

D.Nevus.
E.Melanoma.

51.Generalized acquired hyperpigmentation of melanin occurs in:

A.Melanosis coli.

B.Cachexia.

C.Lentigo.

D.Nevus.

E.Melanoma.

52. Generalized acquired hyperpigmentation of melanin occurs in:

A.Melanosis coli.

B.Chronic arsenical poisoning.

C.Lentigo.

D.Nevus.

E.Melanoma.

53.Generalized congenital hyperpigmentation of melanin occurs in:

A.Melanosis coli.

B.Chronic arsenical poisoning.

C.Xeroderma pigmentosum.

D.Nevus.

E.Melanoma.
54.Focal acquired hyperpigmentation of melanin occurs in:

A.Addison’s disease.

B.Cachexia.

C.Melanosis coli.

D.Nevus.

E.Xeroderma pigmentosum.

55.Focal congenital hyperpigmentation of melanin occurs in:

A.Addison’s disease.

B.Cachexia.

C.Avitaminosis.

D.Nevus.

E.Xeroderma pigmentosum.

56.Focal congenital hyperpigmentation of melanin occurs in:

A.Addison’s disease.

B.Cachexia.

C.Avitaminosis.

D.Melanoma.

E.Xeroderma pigmentosum.
57.General hypopigmentation of melanin occurs in:

A.Leukoderma.

B.Albinism.

C.Vitiligo.

D.Cachexia.

E.Lentigo.

58.Focal acquired hypopigmentation of melanin is called:

A.Albinism.

B.Nevus.

C.Leukoderma.

D.Cachexia.

E.Lentigo.

59.Focal congenital hypopigmentation of melanin is called:

A.Albinism.

B.Nevus.

C.Vitiligo.

D.Cachexia.

E.Lentigo.
60.Albinism is associated with all the following signs, EXCEPT:

A.Blond hair.

B.White skin.

C.Poor vision.

D.Severe photophobia.

E.Arterial hypertension.

61.Leukoderma is a sign of:

A.Syphilis.

B.Typhoid fever.

C.Yellow fever.

D.Antrax.

E.Gonorrhea.

62.Leukoderma is a sign of all the following diseases, EXCEPT:

A.Syphilis.

B.Leprosy.

C.Radiation dermatitis.

D.Diabetes mellitus.

E.Gonorrhea.
63.Distrophic calcification occurs in:

A.Hyperparathyroidism.

B.Hypervitaminosis D.

C.Hyperthyroidism.

D.Atheromas of advanced atherosclerosis.

E.Increased bone catabolism.

64. Distrophic calcification occurs in:

A.Hyperparathyroidism.

B.Hypervitaminosis D.

C.Hyperthyroidism.

D.Increased bone catabolism.

E.Old thrombi.

65. Distrophic calcification occurs in:

A.Hyperparathyroidism.

B.Hypervitaminosis D.

C.Hyperthyroidism.

D.Increased bone catabolism.

E.Dead animal parasites.


66.Metastatic calcification occurs in:

A.Hyperthyroidism.

B.Damaged heart valves.

C.Atheromas.

D.Necrosis.

E.Old thrombi.

67.Metastatic calcification occurs in:

A.Atheromas.

B.Damaged heart valves.

C.Old thrombi.

D.Necrosis.

E.Hypervitaminosis D.

68.Metastatic calcification occurs in:

A.Atheromas.

B.Damaged heart valves.

C.Old thrombi.

D.Necrosis.

E.Increased bone catabolism.


69.Metastatic calcification occurs in:

A.Atheromas.

B.Damaged heart valves.

C.Old thrombi.

D.Necrosis.

E.Decreased bone formation.

70.Common localization of metastatic calcification is in all the following organs, EXCEPT:

A.Blood vessels.

B.Brain.

C.Kidneys.

D.Lungs.

E.Gastric mucosa.

71.Derangement of nucleoproteids metabolism is associated with:

A.Gout.

B.Atheroma.

C.Nevus.

D.Hemachromatosis.

E.Jaundice.

72.Derangement of nucleoproteids metabolism is associated with:


A.Urolithiasis.

B.Atheroma.

C.Nevus.

D.Hemachromatosis.

E.Jaundice.

73.Derangement of nucleoproteids metabolism is associated with:

A.Cerebral infarction.

B.Myocardial infarction.

C.Urate infarction of newborn.

D.Pulmonary hemorrhagic infarction.

E.Splenic infarction.

74.Chemical substances which occurs in urinary stones are of all the following, EXCEPT:

A.Uric acid.

B.Calcium.

C.Magnesium.

D.Cystine.

E.Cholesterol.
Immunopathology

1.Сongenital absence of thymus is called:

А.Aplasia.
B.Hypoplasia.
C.Dysplasia.
D.Atrophy.
E.Thymomegaly.

2.Congenital incomplete development of thymus is called:

А.Aplasia.
B.Hypoplasia.
C.Dysplasia.
D.Atrophy.
E.Thymomegaly.

3.Aquired decrease in weight of thymus is called:

А.Aplasia.
B.Hypoplasia.
C.Dysplasia.
D.Atrophy.
E.Thymomegaly.

4.Abnormal development of thymus is called:

А.Aplasia.
B.Hypoplasia.
C.Dysplasia.
D.Atrophy.
E.Thymomegaly.

5.Decrease in size and weight of thymus under different stress situations including
infectious diseases, intoxications, traumas is called:

А.Aplasia.
B.Hypoplasia.
C.Dysplasia.
D.Atrophy.
E.Accidental involution.
6.Increase in size and weight of thymus more than age level with normal histological
structure is called:

А.Aplasia.
B.Hypoplasia.
C.Thymomegaly.
D.Atrophy.
E.Accidental involution.

7.Collection of B-lymphocytes and plasma cells with formation of lymphoid follicles in the
intralobular perivascular spaces of thymus parenchyma is called:

А.Aplasia.
B.Hypoplasia.
C.Thymomegaly.
D.Hyperplasia with lymphoid follicles.
E.Accidental involution.

8.Changes in peripheric lymphoid tissue under antigene stimulation include are all the
following processes, EXCEPT:

A.Macrophageal reaction.
B.Hyperplasia of lymphocytes.
C.Plasma cell transformation.
D.Edema of interstitium.
E.Infarction.

9.Changes in peripheric lymphoid tissue under antigene stimulation include are all the
following processes, EXCEPT:

A.Macrophageal reaction.
B.Hyperplasia of lymphocytes.
C.Dysplasia of lymphocytes.
D.Plasma cell transformation.
E.Edema of interstitium.

10.Changes in peripheric lymphoid tissue under antigene stimulation include are all the
following processes, EXCEPT:

A.Macrophageal reaction.
B.Anaplastic transformation of lymphocytes.
C.Hyperplasia of lymphocytes.
D.Plasma cell transformation.
E.Edema of interstitium.

11.Changes in lymph nodes under antigene stimulation include are all the following
processes, EXCEPT:

A.Hyperemia.
B.Edema.
C.Accumulation of plasma cells.
D.Accumulation of neutrophils.
E.Accumulation of macrophages.

12.Changes in lymph nodes under antigene stimulation include are all the following
processes, EXCEPT:

A.Hyperemia.
B.Ischemia.
C.Edema.
D.Accumulation of plasma cells.
E.Accumulation of macrophages.

13.Changes in spleen under antigene stimulation include are all the following processes,
EXCEPT:

A.Hypoplasia of red pulp.


B.Hyperplasia of red pulp.
C.Plasmatization of red pulp.
D.Accumulaton of macrophages.
E.Hyperplasia of follicles.

14.Changes in spleen under antigene stimulation include are all the following processes,
EXCEPT:

A.Hyperplasia of red pulp.


B.Hypoplasia of follicles.
C.Plasmatization of red pulp.
D.Accumulaton of macrophages.
E.Hyperplasia of follicles.

15.Changes in spleen under antigene stimulation include are all the following processes,
EXCEPT:

A.Hyperplasia of red pulp.


B.Plasmatization of red pulp.
C.Accumulaton of giant cells.
D.Accumulaton of macrophages.
E.Hyperplasia of follicles.

16.Changes in bone marrow under antigene stimulation include are all the following
processes, EXCEPT:

A.Hyperplasia.
B.Macrophageal-plasma cell transformation.
C.Myeloid metaplasia.
D.Hypoplasia.
E.Hyperemia.

17.Changes in bone marrow under antigene stimulation include are all the following
processes, EXCEPT:

A.Hyperplasia.
B.Macrophageal-plasma cell transformation.
C.Myeloid metaplasia.
D.Hyperemia.
E.Myeloid dysplasia.

18.Changes in peripheric lymphoid tissue in congenital immunodeficiency syndromes are


characterized by:

A.Decrease in size of follicles of spleen.


B.Increase in size of follicles of spleen.
C.Hyperplasia of lymph nodes.
D.Myeloid metaplasia of bone marrow.
E.Plasmatization of red pulp of spleen.

19.Changes in peripheric lymphoid tissue in congenital immunodeficiency syndromes are


characterized by:

A.Increase in size of follicles of spleen.


B.Hyperplasia of lymph nodes.
C.Absence of B-zones in cortex of lymph nodes.
D.Myeloid metaplasia of bone marrow.
E.Plasmatization of red pulp of spleen.

20.Changes in peripheric lymphoid tissue in congenital immunodeficiency syndromes are


characterized by:
A.Increase in size of follicles of spleen.
B.Absence of germinal centers in follicles of spleen.
C.Hyperplasia of lymph nodes.
D.Myeloid metaplasia of bone marrow.
E.Plasmatization of red pulp of spleen.

21.Immediate type of hypersensitivity morphologically occurs like:

A.Acute immune inflammation.


B.Purulent inflammation.
C.Hemorragic inflammation.
D.Chronic immune inflammation.
E.Catarrhal inflammation.

22.Delayed type of hypersensitivity morphologically occurs as:

A.Acute immune inflammation.


B.Purulent inflammation.
C.Chronic immune inflammation.
D.Hemorragic inflammation.
E.Catarrhal inflammation.

23.Signs of acute immune inflammation are all the following processes, EXCEPT:

A.Fast development.
B.Prevalence of alterative changes.
C.Slow development.
D.Prevalence of exudative changes.
E.Slow reparation.

24.Morphologically immediate type of hypersensitivity occurs as:

А.Fibrinoid necrosis.
B.Lympho-hystiocytic infiltration.
C.Macrophageal infiltration.
D.Granulomatosis.
E.Cytoplasmic bridges between lymphocytes and macrophages.

25.Morphologically immediate type of hypersensitivity occurs as:

A.Lympho-hystiocytic infiltration.
B.Macrophageal infiltration.
C.Granulomatosis.
D.Cytoplasmic bridges between lymphocytes and macrophages.
E.Plasmatic saturation.

26.Morphologically immediate type of hypersensitivity occurs as:

A.Lympho-hystiocytic infiltration.
B.Mucoid changes.
C.Macrophageal infiltration.
D.Granulomatosis.
E.Cytoplasmic bridges between lymphocytes and macrophages.
27.Morphologically immediate type of hypersensitivity occurs as:

A.Lympho-hystiocytic infiltration.
B.Fibrinoid changes.
C.Macrophageal infiltration.
D.Granulomatosis.
E.Cytoplasmic bridges between lymphocytes and macrophages.

28.Morphologically immediate type of hypersensitivity occurs as:

A.Lympho-hystiocytic infiltration.
B.Fibrinous-hemorrhagic exudate.
C.Macrophageal infiltration.
D.Granulomatosis.
E.Cytoplasmic bridges between lymphocytes and macrophages.

29.Morphological signs of immediate type of hypersensitivity are all the following,


EXCEPT:

A.Lympho-hystiocytic infiltration.
B.Mucoid and Fibrinoid changes.
C.Plasmatic saturation.
D.Fibrinoid necrosis.
E.Fibrinous-hemorrhagic exudate.

30.Morphological signs of immediate type of hypersensitivity are all the following,


EXCEPT:

A.Mucoid and Fibrinoid changes.


B.Plasmatic saturation.
C.Granulomatosis.
D.Fibrinoid necrosis.
E.Fibrinous-hemorrhagic exudate.
31.Morphological signs of immediate type of hypersensitivity are all the following,
EXCEPT:

A.Mucoid and Fibrinoid changes.


B.Plasmatic saturation.
C.Fibrinoid necrosis.
D.Fibrinous-hemorrhagic exudate.
E.Macrophageal infiltration.

32.Which disease is associated with immediate type of hypersensitivity?

A.Myocardial infarction.
B.Liver cirrhosis.
C.Аrthus reaction.
D.Purulent meningitis.
E.Arterial hypertension.
33.Which disease is associated with immediate type of hypersensitivity?

A.Myocardial infarction.
B.Glomerulonephritis.
C.Liver cirrhosis.
D.Purulent meningitis.
E.Arterial hypertension.

34.Which disease is associated with immediate type of hypersensitivity?

A.Syphilis.
B.Myocardial infarction.
C.Liver cirrhosis.
D.Purulent meningitis.
E.Arterial hypertension.

35.Which disease is associated with immediate type of hypersensitivity?

A.Myocardial infarction.
B.Liver cirrhosis.
C.Purulent meningitis.
D.Rheumatic fever.
E.Arterial hypertension.

36.Which disease is associated with immediate type of hypersensitivity?

A.Myocardial infarction.
B.Liver cirrhosis.
C.Purulent meningitis.
D.Arterial hypertension.
E.Systemic lupus erytematosus.

37.Which disease is associated with immediate type of hypersensitivity?

A.Myocardial infarction.
B.Nodular periarteritis.
C.Purulent meningitis.
D.Liver cirrhosis.
E.Arterial hypertension.

38.Which disease is not associated with immediate type of hypersensitivity?

A.Myocardial infarction.
B.Nodular periarteritis
C.Glomerulonephritis.
D.Syphilis.
E.Rheumatic fever.
39.Which disease is not associated with immediate type of hypersensitivity?

A.Liver cirrhosis.
B.Nodular periarteritis
C.Glomerulonephritis.
D.Syphilis.
E.Rheumatic fever.

40.Which disease is not associated with immediate type of hypersensitivity?

A.Glomerulonephritis.
B.Nodular periarteritis.
C.Arterial hypertension.
D.Syphilis.
E.Rheumatic fever.

41.Reagin reactions are associated with action of which type of allergic reagin-antibodies?

A.IgA.
B.IgB.
C.IgC.
D.IgD.
E.IgE.

42.Reagin reactions are associated with formation of which type of exudate?

A.Basophylic.
B.Eosinophylic.
C.Neutrophylic.
D.Hemorrhagic.
E.Macrofageal.

43.Which disease is associated with action of allergic reagin-antibodies?

A.Chronic bronchitis.
B.Pulmonary carcinoma.
C.Atopic bronchial asthma.
D.Bronchiectasis.
E.Lobar pneumonia.

44.Morphologically delayed type of hypersensitivity occurs as:

A.Mucoid and Fibrinoid changes.


B.Plasmatic saturation.
C.Fibrinoid necrosis.
D.Fibrinous-hemorrhagic exudate.
E.Cytolysis.
45.Morphologically delayed type of hypersensitivity occurs as:

A.Lympho-hystiocytic infiltration.
B.Mucoid and Fibrinoid changes.
C.Plasmatic saturation.
D.Fibrinoid necrosis.
E.Fibrinous-hemorrhagic exudate.

46.Morphologically delayed type of hypersensitivity occurs as:

A.Mucoid and Fibrinoid changes.


B.Plasmatic saturation.
C.Granulomatosis.
D.Fibrinoid necrosis.
E.Fibrinous-hemorrhagic exudate.

47.Morphologically delayed type of hypersensitivity occurs as:

A.Mucoid and Fibrinoid changes.


B.Plasmatic saturation.
C.Fibrinoid necrosis.
D.Fibrinous-hemorrhagic exudate.
E.Macrophageal infiltration.

48.Morphologically delayed type of hypersensitivity occurs as:


A.Mucoid and Fibrinoid changes.
B.Plasmatic saturation.
C.Fibrinoid necrosis.
D.Fibrinous-hemorrhagic exudate.
E.Cytoplasmic bridges between lymphocytes and macrophages.

49.Morphological signs of delayed type of hypersensitivity are all the following, EXCEPT:

А.Fibrinoid necrosis.
B.Lympho-hystiocytic infiltration.
C.Macrophageal infiltration.
D.Granulomatosis.
E.Cytoplasmic bridges between lymphocytes and macrophages.

50.Morphological signs of delayed type of hypersensitivity are all the following, EXCEPT:

A.Lympho-hystiocytic infiltration.
B.Macrophageal infiltration.
C.Granulomatosis.
D.Cytoplasmic bridges between lymphocytes and macrophages.
E.Plasmatic saturation.
51.Morphological signs of delayed type of hypersensitivity are all the following, EXCEPT:

A.Lympho-hystiocytic infiltration.
B.Mucoid changes.
C.Macrophageal infiltration.
D.Granulomatosis.
E.Cytoplasmic bridges between lymphocytes and macrophages.

52.Morphological signs of delayed type of hypersensitivity are all the following, EXCEPT:

A.Lympho-hystiocytic infiltration.
B.Fibrinous-hemorrhagic exudate.
C.Macrophageal infiltration.
D.Granulomatosis.
E.Cytoplasmic bridges between lymphocytes and macrophages.

53.Which disease is associated with delayed type of hypersensitivity?

A.Myocardial infarction.
B.Liver cirrhosis.
C.Purulent meningitis.
D.Arterial hypertension.
E.Contact dermatitis.
54.Which disease is associated with delayed type of hypersensitivity?

A.Viral hepatitis.
B.Liver cirrhosis.
C.Chronic cholecystitis.
D.Arterial hypertension.
E.Alcoholic hepatitis.

55.Which disease is associated with delayed type of hypersensitivity?

A.Typhoid fever.
B.Tuberculosis.
C.Syphilis.
D.Purulent meningitis.
E.Alcoholic hepatitis.

56.Which disease is associated with delayed type of hypersensitivity?

A.Typhoid fever.
B.Brucellosis.
C.Syphilis.
D.Purulent meningitis.
E.Alcoholic hepatitis.
57.Which disease is not associated with delayed type of hypersensitivity?

A.Contact dermatitis.
B.Brucellosis.
C.Syphilis.
D.Tuberculosis.
E.Viral hepatitis.

58.Which disease is not associated with delayed type of hypersensitivity?

A.Contact dermatitis.
B.Typhoid fever.
C.Brucellosis.
D.Tuberculosis.
E.Viral hepatitis.

59.Which disease is not associated with delayed type of hypersensitivity?

A.Purulent meningitis.
B.Contact dermatitis.
C.Brucellosis.
D.Tuberculosis.
E.Viral hepatitis.
60.Which disease is not associated with delayed type of hypersensitivity?

A.Viral hepatitis.
B.Contact dermatitis.
C.Brucellosis.
D.Tuberculosis.
E.Sepsis.

61.Morphological signs of transplant rejection are all the following, EXCEPT:

A.Lympho-hystiocytic infiltration.
B.Edema of transplant.
C.Macrophageal infiltration.
D.Granulomatosis.
E.Leukocytic infiltration.

62.Morphological signs of transplant rejection are all the following, EXCEPT:

A.Lympho-hystiocytic infiltration.
B.Edema of transplant.
C.Macrophageal infiltration.
D.Cytoplasmic bridges between lymphocytes and macrophages.
E.Leukocytic infiltration.
63.Morphological signs of transplant rejection are all the following, EXCEPT:

A.Lympho-hystiocytic infiltration.
B.Edema of transplant.
C.Macrophageal infiltration.
D.Eosinophylic infiltration.
E.Leukocytic infiltration.

64.Morphological signs of transplant rejection are all the following, EXCEPT:

A.Lympho-hystiocytic infiltration.
B.Edema of transplant.
C.Macrophageal infiltration.
D.Coagulative necrosis.
E.Leukocytic infiltration.

65.Morphological signs of transplant rejection are all the following, EXCEPT:

A.Lympho-hystiocytic infiltration.
B.Edema of transplant.
C.Macrophageal infiltration.
D.Hyperemia of transplant.
E.Leukocytic infiltration.

66.Predisposing factor in pathogenesis of autoimmunization is:

A.Genes of HLA system.


B.Viral infections.
C.Bacterial infections.
D.Phisical and chemical influences to immune organs.
E.Decrease of supressive activity of T-lymphocytes.

67.Predisposing factor in pathogenesis of autoimmunization is:

A.Phisical and chemical influences to immune organs.


B.Viral infections.
C.Bacterial infections.
D.Hormonal disbalance.
E.Decrease of supressive activity of T-lymphocytes.

68.Predisposing factor in pathogenesis of autoimmunization is:

A.Phisical and chemical influences to immune organs.


B.Viral infections.
C.Bacterial infections.
D.Decrease of supressive activity of T-lymphocytes.
E.Genetic peculiarities of target cells.
69.Initiated factor in pathogenesis of autoimmunization is:

A.Genes of HLA system.


B.Viral infections.
C.Hormonal disbalance
D.Decrease of supressive activity of T-lymphocytes.
E.Genetic peculiarities of target cells.

70.Initiated factor in pathogenesis of autoimmunization is:

A.Genes of HLA system.


B.Bacterial infections.
C.Hormonal disbalance
D.Decrease of supressive activity of T-lymphocytes.
E.Genetic peculiarities of target cells.

71.Initiated factor in pathogenesis of autoimmunization is:

A.Genes of HLA system.


B.Hormonal disbalance.
C.Phisical influences to immune organs.
D.Decrease of supressive activity of T-lymphocytes.
E.Genetic peculiarities of target cells.

72.Initiated factor in pathogenesis of autoimmunization is:

A.Genes of HLA system.


B.Hormonal disbalance.
C.Chemical influences to immune organs.
D.Decrease of supressive activity of T-lymphocytes.
E.Genetic peculiarities of target cells.

73.Basic factor in pathogenesis of autoimmunization is:

A.Genes of HLA system.


B.Hormonal disbalance.
C.Chemical influences to immune organs.
D.Decrease of supressive activity of T-lymphocytes.
E.Genetic peculiarities of target cells.

74.Organ specific immune disease is:

А.Hashimoto thyroiditis.
B.Rheumatoid arthritis.
C.Systemic Lupus Erythematosus.
D.Scleroderma.
E.Secondary thrombocytopenia.
75.Organ specific immune disease is:

А.Rheumatoid arthritis.
B.Systemic Lupus Erythematosus.
C.Scleroderma.
D.Secondary thrombocytopenia.
E.Encephalomyelitis.

76.Organ specific immune disease is:

А.Rheumatoid arthritis.
B.Systemic Lupus Erythematosus.
C.Scleroderma.
D.Polyneuritis.
E.Secondary thrombocytopenia.

77.Organ specific immune disease is:


А.Rheumatoid arthritis.
B.Disseminated sclerosis of CNS.
C.Systemic Lupus Erythematosus.
D.Scleroderma.
E.Secondary thrombocytopenia.

78.Organ specific immune disease is:


А.Rheumatoid arthritis. B.Aspermatogenesis. C.Systemic Lupus Erythematosus.
D.Scleroderma. E.Secondary thrombocytopenia.

79.Organ non-specific immune disease is:

А.Hashimoto thyroiditis.
B.Rheumatoid arthritis.
C.Encephalomyelitis.
D.Polyneuritis.
E.Disseminated sclerosis of CNS.

80.Organ non-specific immune disease is:

А.Hashimoto thyroiditis.
B.Encephalomyelitis.
C.Systemic Lupus Erythematosus.
D.Polyneuritis.
E.Disseminated sclerosis of CNS.

81.Organ non-specific immune disease is:

А.Scleroderma.
B.Encephalomyelitis.
C.Hashimoto thyroiditis.
D.Polyneuritis.
E.Disseminated sclerosis of CNS.

82.Organ non-specific immune disease is:

А.Polyneuritis.
B.Encephalomyelitis.
C.Hashimoto thyroiditis.
D.Dermatomyositis.
E.Disseminated sclerosis of CNS.
83.Organ non-specific immune disease is:

А.Polyneuritis.
B.Encephalomyelitis.
C.Hashimoto thyroiditis.
D.Disseminated sclerosis of CNS.
E.Secondary thrombocytopenia.

84.Autoimmune disease of intermediate type is:

А.Myastenia gravis.
B.Systemic Lupus Erythematosus.
C.Scleroderma.
D.Polyneuritis.
E.Secondary thrombocytopenia.

85.Autoimmune disease of intermediate type is:

А.Hashimoto thyroiditis.
B.Diabetes mellitus-type I.
C.Scleroderma.
D.Polyneuritis.
E.Secondary thrombocytopenia.

86.Autoimmune disease of intermediate type is:

А.Hashimoto thyroiditis.
B.Scleroderma.
C.Thyrotoxicosis.
D.Polyneuritis.
E.Secondary thrombocytopenia.
87.Autoimmune disease of intermediate type is:

А.Hashimoto thyroiditis.
B.Scleroderma.
C.Polyneuritis.
D.Sjogren’s syndrome.
E.Secondary thrombocytopenia.

88.Autoimmune disease of intermediate type is:

А.Hashimoto thyroiditis.
B.Scleroderma.
C.Polyneuritis.
D.Secondary thrombocytopenia.
E.Goodpasture’s syndrome.

89.Variant of primary immunodeficiency syndrome occurs:

А.In leukemia.
B.Under radial therapy.
C.In aplasia of thymus.
D.In infections.
E.In sarcoidosis.

90.Variant of primary immunodeficiency syndrome occurs:

А.In leukemia.
B.Under radial therapy.
C.In sarcoidosis.
D.In hypoplasia of thymus.
E.In malignant lymphoma.

91.Variant of secondary immunodeficiency syndrome occurs in all the following diseases,


EXCEPT:

А.Leukemia.
B.Thymoma.
C.Sarcoidosis.
D.Hypoplasia of thymus.
E.Malignant lymphoma.

92.Variant of secondary immunodeficiency syndrome occurs in all the following diseases,


EXCEPT:

А.Leukemia.
B.Thymoma.
C.Sarcoidosis.
D.Aplasia of thymus.
E.Malignant lymphoma.

93.Variant of secondary immunodeficiency syndrome occurs in all the following diseases,


EXCEPT:

А.Fibroma.
B.Leukemia.
C.Thymoma.
D.Sarcoidosis.
E.Malignant lymphoma.

94.Variant of secondary immunodeficiency syndrome occurs in all the following diseases,


EXCEPT:

А.Hemangioma.
B.Leukemia.
C.Thymoma.
D.Sarcoidosis.
E.Malignant lymphoma.

95.Variant of secondary immunodeficiency syndrome is:

А.Lui-Bar syndrome.
B.Neseloff syndrome.
C.DiGeorge syndrome.
D.Bruton syndrome.
E.AIDS.

96.Variant of secondary immunodeficiency syndrome occurs in all the following situations,


EXCEPT:
A.Radial therapy.
B.Corticosteroid therapy.
C.Immunodepressant therapy.
D.Antibiotic therapy.
E.Thymusectomy.

97.Variant of secondary immunodeficiency syndrome occurs in all the following situations,


EXCEPT:
A.Radial therapy.
B.Corticosteroid therapy.
C.Immunodepressant therapy.
D.Physiotherapy.
E.Thymusectomy.

98.Complication of immunodeficiency syndromes is:

А.Arterial hypertension.
B.Myocardial infarction.
C.Typhoid fever.
D.Sepsis.
E.Chronic peptic gastric ulcer.

99.Complication of immunodeficiency syndromes is:

А.Recurrence of bronchial asthma.


B.Recurrence of chronic hepatitis.
C.Recurrence of chronic cholecystitis.
D.Recurrence of chronic gastric peptic ulcer.
E.Recurrence of tuberculosis.

100.Complication of immunodeficiency syndromes is:

А.Arterial hypertension.
B.Myocardial infarction.
C.Typhoid fever.
D.Purulent pneumonia.
E.Chronic peptic gastric ulcer.

101.Complication of immunodeficiency syndromes is:

А.Pulmonary abscess.
B.Myocardial infarction.
C.Typhoid fever.
D.Arterial hypertension.
E.Chronic peptic gastric ulcer.

102.Complications of immunodeficiency syndromes are all the following, EXCEPT:

А.Pulmonary abscess.
B.Purulent pneumonia.
C.Myocardial infarction.
D.Sepsis.
E. Recurrence of tuberculosis.
103.Complications of immunodeficiency syndromes are all the following, EXCEPT:

А.Pulmonary abscess.
B.Purulent pneumonia.
C.Liver cirrhosis.
D.Sepsis.
E. Recurrence of tuberculosis.

104.Complications of immunodeficiency syndromes are all the following, EXCEPT:


А.Pulmonary abscess.
B.Purulent pneumonia.
C.Sepsis.
D.Recurrence of tuberculosis.
E.Chronic peptic gastric ulcer.

105.Complications of immunodeficiency syndromes are all the following, EXCEPT:

А.Pulmonary abscess.
B.Purulent pneumonia.
C.Sepsis.
D.Recurrence of tuberculosis.
E.Arterial hypertension.

INTRACELLULAR ACCUMULATIONS

1. Lipids that can accumulate intracellularly are:

A. Triglycerides
B. Derived lipids
C. Miscellaneous lipids
D. Lecithin
E. Phospholipids

2. Lipids that can accumulate intracellularly are:

A. Cholesterol
B. Derived lipids
C. Miscellaneous lipids
D. Lecithin
E. Miscellaneous lipids

3. Which type of lipids accumulates at fatty change in liver cells?

A. Triglycerides
B. Cholesterol
C. Complex lipids
D. Miscellaneous lipids
E. Lipogialin

4. Which type of lipids accumulate in atherosclerosis?

A. Triglycerides
B. Cholesterol
C. Complex lipids
D. Miscellaneous lipids
E. Lipogialin
5. Substance accumulating in tissues as a result is fatty change:

A. Water
B. Cholesterol
C. Protein
D. Glycogen
E. Triglycerides

7. Give some example of intracellular hyaline degeneration:

A. Tubular epithelium of kidney


B. Keloid
C. Renal glomeritis in chronic nephritis
D. Diabetic mellitus kidney
E. Pyelonephritis

8. Give some example of intracellular hyaline degeneration:

A. Mallory body in alcoholic liver cells


B. Keloid
C. Renal glomeritis in chronic nephritis
D. Diabetic mellitus kidney
E. Pyelonephritis

9. Following are the features of reversible injury, except:


A. Swelling of mitochondria and endoplasmic reticulum
B. Detachment of ribosomes from endoplasmic reticulum
C. Clumping of nuclear chromatin
D. Appearance of myelin figures and cell blebs
E. Pyknosis and karyolysis of nucleus

10. One of cause of intracellular accumulation of metabolic substances is:

A. Genetic defects
B. Inflammation
C. Embolism
D. Necrosis
E. Activation of oncogenes

11. The most common cause of fatty change is:

A. Inflammation
B. Neoplasia
C. Hypoxia
D. Amyloidosis
E. Autoimmune diseases

12. Dystrophy is defined as the process:


A. Degeneration of tissue
B. Inflammation
C. Edema
D. Tumor
E. Adaptation

13. Dystrophies are based on following mechanism, except one:


A. Decomposition
B. Transformation
C. Regeneration
D. Infiltration
E. Abnormal synthesis

14. One of manifestations of metabolic derangements in cells is:


A. Apoptosis.
B. Atrophy.
C. Hypertrophy.
D. Metaplasia.
E. Intracellular accumulation of abnormal amounts of various substances.

15. Fatty change in the heart is characterized by:

A. Tiger heart.
B. Small size heart.
C. Red heart.
D. Solid heart.
E. Goose heart.

16. Xantoma is a pathological process with accumulation of:


A. Cholesterol.
C. Lipoproteins
D. Phospholipids.
E. Neutral fat.
B. Lipohyalin

17. Mallory’s bodies may be found in:


A. Hepatocytes.
B. Neurons.
C. Cardiomyocytes.
D. Epithelial cells of stomach.
E. Epithelial cells of renal tubules.

18. The fatty liver has all pathologic features, except:


A. Enlarged.
B. Yellow.
C. Soft.
D. Red.
E. Greasy.

19. Diabetes mellitus is characterized by accumulation of glycogen in all cells, except:


A. Epithelial cells of the proximal tubules.
B. Liver cells.
C. b-cells of the islets of Langerhans.
D. Smooth muscle cells.
E. Heart muscle cells.

20. Hyaline droplets in renal tubular epithelial cells are seen in:
A. Proteinuria.
B. Lipiduria.
C. Hematuria.
D. Cylindruria.
E. Disproteinemia.

21. What substances are accumulated within parenchymal cells in fatty changes?
A. Cholesterol.
B. Apoproteins.
C. Triglycerides.
D. Vitamins.
E. Ketone bodies.

22. The cells accumulating fat within the intimal layer of arteries in atherosclerotic plaques
are called:
A. Fibroblasts.
B. Epithelial cells.
C. Foam cells.
D. Lymphocytes.
E. Leukocytes.

23. Tumor arising from epithelial cells with accumulation of mucin is called:
A. Mucinous teratoma.
B. Mucinous carcinoma.
C. Mucinous melanoma.
D. Mucinous neuroblastoma.
E. Mucinous sarcoma.
24. Fatty change in the heart is characterized by:
A. Red heart.
B. Small size heart.
C. Tiger heart.
D. Solid heart.
E. Goose heart.

25. Simple hyaline occurs in:


A. Diabetes mellitus.
B. Systemic lupus erythematosus.
C. Atherosclerosis.
D. Rheumatic fever.
E. Rheumatoid arthritis.

26. Fatty change is often seen in all of the following organs, except:
A. Lung.
B. Heart.
C. Kidney.
D. Muscles.
E. Liver.

27. Accumulation of cholesterol and cholesterol esters with formation of tumorous masses is
called:
A. Atheroma.
B. Xantoma.
C. Adenoma.
D. Papilloma.
E. Cystadenoma

28. Colloid degeneration is occurs in:


A. Thyroid gland.
B. Mammary gland.
C. Salivary gland
D. Pituitary gland.
E. Parathyroid gland.

29. The fatty liver has all pathologic features, except:


A. Greasy.
B. Red.
C. Yellow.
D. Soft.
E. Enlarged.

30. The stain used to identify fat is:


A. Hematoxylin and eosin stain.
B. Metachromatic stain
C. Congo red stain.
D. PAS reaction.
E. Red oil O

EXTRACELLULAR ACCUMULATIONS

1. The reversible process caused by accumulation of glycosaminoglycans in extracellular


matrix is:
A. Amyloidosis.
B. Glycogenoses.
C. Hyalinosis.
D. Mucoid changes.
C. Fibrinoid changes.

2. Lipohyalin occurs in:


A. Diabetes mellitus.
B. Arterial hypertension.
C. Atherosclerosis.
D. Rheumatic fever.
E. Rheumatoid arthritis.

3. Specific method for diagnosis of amyloid in fresh tissue is:


A. Virchow test.
B. Rokitansky test.
C. Ewing test.
D. Masson test.
E. Papanicolaou test.

4. Substance accumulating in tissues as a result is mucoid change:

A. Water
B. Cholesterol
C. Protein
D. Glycogen
E. Triglycerides

5. Compound hyalin occurs in:


A. Diabetes mellitus.
B. Arterial hypertension.
C. Atherosclerosis.
D. Rheumatic fever.
E. Vasculitis.

6. Foam cells are characterized by accumulation of:


A. Neutral fat.
B. Triglycerides.
C. Cholesterol.
D. Phospholipids.
E. Lipoprotein.

7. All the pathological processes are irreversible, except:


A. Mucoid changes.
B. Fibrinoid changes.
C. Amyloidosis.
D. Apoptosis.
E. Necrosis.

8. Variant of systemic amyloidosis is:


A. Senile cardiac.
B. Senile cerebral.
C. Heredofamilial.
D. Endocrine.
E. Tumor-forming.

9. Causes of hyalinosis are all of the following, except:


A. Fibrinoid changes.
B. Inflammation.
C. Apoptosis.
D. Sclerosis.
E. Necrosis.

10. The substance with fibrillar structure, which forms under the pathological conditions is:
A. Reabsorption droplets.
B. Russell bodies.
C. Lipids.
D. Amyloid.
E. Calcificates.

11. Variant of systemic amyloidosis is:


A. Senile cardiac.
B. Senile cerebral.
C. Endocrine.
D. Hemodialysis-associated.
E. Tumor-forming.

12. Simple hyaline occurs in:


A. Diabetes mellitus.
B. Arterial hypertension.
C. Rheumatic fever.
D. Rheumatoid arthritis.
E. Systemic lupus erythematosus.

13. The pathologic proteinaceous substance, accumulating only between cells in various
tissues and organs of the body is:
A. Glycogen.
B. Hyaline.
C. Water.
D. Amyloid.
E. Lipid.

14. Variant of localized amyloidosis is:


A. Primary.
B. Secondary.
C. Heredofamilial.
D. Endocrine.
E. Hemodialysis-associated.
15. Cause of obesity, which associated with excessive nutrition, is called:
A. Primary.
B. Alimentary.
C. Cerebral.
D. Endocrine.
E. Hereditary.

16. Grossly the organs affected by amyloidosis are characterized by all of the following,
except:
A. Enlarged.
B. Hard.
C. Waxy in appearance.
D. Smooth.
E. Soft.

17. Variant of localized amyloidosis is:


A. Primary.
B. Secondary.
C. Heredofamilial.
D. Hemodialysis-associated.
E. Tumor-forming.

18. Type of obesity with unknown cause is called:


A. Primary.
B. Alimentary.
C. Cerebral.
D. Endocrine.
E. Hereditary.

19. On autopsy of the 58-year-old man it is revealed: mitral valve is deformed, thickened, not
totally closed. Microscopically: centers of collagen fibers are eosinophilic, have positive
fibrin reaction. The most likely it is:
A. Fibrinoid swelling
B. Mucoid swelling
C. Hyalinosis
D. Amyloidosis
E. Fibrinoid inflammation

20. The stain used to identify mucoid changes is:


A. Hematoxylin and eosin stain.
B. Metachromatic stain
C. Congo red stain.
D. PAS reaction.
E. Red oil O
21. Most common localization of mucoid changes is:
A. Reproductive system.
B. Cardiovascular system.
C. Urinary system.
D. Blood system.
E. Endocrine system.

22. Simple hyaline occurs in:


A. Arterial hypertension.
B. Rheumatic fever.
C. Diabetes mellitus.
D. Rheumatoid arthritis.
E. Systemic lupus erythematosus.

23. Simple hyaline occurs in:


A. Diabetes mellitus.
B. Systemic lupus erythematosus.
C. Rheumatic fever.
D. Atherosclerosis.
E. Rheumatoid arthritis.

24. Lipohyalin occurs in:


A. Arterial hypertension.
B. Diabetes mellitus.
C. Atherosclerosis.
D. Rheumatic fever.
E. Rheumatoid arthritis.

25. The abnormal protein substance, accumulating in extracellular space in various tissues
and

organs of the body is:


A. Glycogen.
B. Amyloid.
C. Hyaline
D. Water.
E. Lipid.

26. Type of systemic amyloidosis is:


A. Senile cardiac.
B. Senile cerebral.
C. Heredofamilial.
D. Endocrine.
E. Tumor-forming.

27. Type of systemic amyloidosis is:


A. Senile cardiac.
B. Senile cerebral.
C. Endocrine.
D. Hemodialysis-associated.
E. Tumor-forming.
28. Type of localized amyloidosis is:
A. Senile cardiac.
B. Primary.
C. Secondary.
D. Heredofamilial.
E. Hemodialysis-associated.

29. Typeof localized amyloidosis is:


A. Primary.
B. Secondary.
C. Senile cerebral.
D. Heredofamilial.
E. Hemodialysis-associated.

30. Type of localized amyloidosis is:


A. Primary.
B. Secondary.
C. Heredofamilial.
D. Endocrine.
E. Hemodialysis-associated.

31. Type of localized amyloidosis is:


A. Primary.
B. Secondary.
C. Heredofamilial.
D. Hemodialysis-associated.
E. Tumor-forming.

32. Localization of per collagenous amyloidosis is:


A. Liver.
B. Spleen.
C. Heart.
D. Kidneys.
E. Suprarenal glands.

33. Localization of per collagenous amyloidosis is:


A. Liver.
B. Spleen.
C. Bowel.
D. Kidneys.
E. Suprarenal glands.

34. Localization of per collagenous amyloidosis is:


A. Liver.
B. Spleen.
C. Nerves.
D. Kidneys.
E. Suprarenal glands.

35. Localization of per reticular amyloidosis is:


A. Heart.
B. Tongue.
C. Nerves.
D. Kidneys.
E. Bowel.

36. Localization of per reticular amyloidosis is:


A. Heart.
B. Tongue.
C. Nerves.
D. Liver.
E. Bowel.

37. Localization of per reticular amyloidosis is:


A. Heart.
B. Tongue.
C. Nerves.
D. Spleen.
E. Bowel.

38. Obesity with cause of excessive nutrition is called:


A. Alimentary.
B. Primary.
C. Cerebral.
D. Endocrine.
E. Hereditary.

39. Type of obesity with unknown cause is called:


A. Alimentary.
B. Cerebral.
C. Endocrine.
D. Primary.
E. Hereditary.

40. Variant of systemic amyloidosis is:


A. Senile cardiac.
B. Senile cerebral.
C. Endocrine.
D. Secondary.
E. Tumor-forming.

41. Obesity with deposition of fat in abdomen area is called:


A. Upper type.
B. Lower type.
C. Symmetric type.
D. Middle type.
E. Diffuse type.

42. Obesity with deposition of fat in area of face and neck is called:
A. Diffuse type.
B. Middle type.
C. Lower type.
D. Simmetric type.
E. Upper type

43 Systemic amyloidoses affect :


A. more than one body organ or system
B. only one body organ or tissue type
C. Heredofamilial.
D. Endocrine.
E. Tumor-forming.

44 Localised amyloidoses affect:


A. Senile cardiac
B. Senile cerebral
C. only one body organ or tissue type
D. more than one body organ or system
E. Tumor-forming

45. All examples of extracellular hyaline are change, except:


A. Mallory’s hyaline
B. Old scar
C. Hyaline arteriolosclerosis
D. Chronic glomerulonephritis
E. Corpora amylacea in the brain in old age

46. In amyloidosis macroscopically liver changed in the following way:


A. Enlarged, pale, waxy and firm
B. Dense and sharply reduced
C. Enlarged and yellow color
D. Soft and enlarged
E. Hard and red

47. Spleen changed by amyloidosis in the following way:


A. Cut surface is translucent pale and waxy
B. Dense and sharply reduced
C. Cut surface – map-like areas of amyloid
D. Soft and enlarged
E. True (A+C)

48. Causes of hyalinosis are all of the following, EXCEPT:


A. Fibrinoid changes.
B. Inflammation.
C. Apoptosis.
D. Sclerosis.
E. Necrosis.

49. Most specific histological sign in mucoid changes is:


A. Metachromasia.
B. Metaplasia.
C. Metastasis.
D. Metakinesis.
E. Metamorphosis.

Tissue Repair - новые

1. In the process of repair, parenchymal regeneration indicates there generation of:


A. Labile cells
B. Stable cells
C. Permanent cell
D. a + b
E. All of these

2. Wound healing is the summation of following processes, except:


A. Coagulation
B. Matrix synthesis
C. Angiogenesis
D. Fibrolysis

3. Organization is - ?
A. Change of area of necrosis or thrombus by connective tissue
B. Regeneration of tissue
C. Vascularization of necrotic mass
D. Autolysis of necrotic mass
E. Hyalinosis of necrotic mass

4. What of collagen type is normally present in the skin, bone and tendons?
A. Type I collagen
B. Type II collagen
C. Type III collagen
D. Type IV collagen

5. Keloid scars is made up of -


A. Dense collagen
B. Loose fibrous tissue
C. Granulomatous tissue
D. Loose areolar tissue
6. Contraction of wounds is completed by the -
A. Two weeks
B. Six weeks
C. Six months
D. Two years

7. A young man of 20 got a lacerated wound on his left arm, stitched - 1 week later sutures
were remained-healing continued but the site became disfigured by prominent raised irregular
nodular scar, in next 2 months which of the following best describes the process:
A. Organization
B. Dehiscence
C. Resolution
D. Keloid formation
E. Secondary union

8. The most characteristic feature of granulation tissue is:


A. Growth of fibroblasts and new capillaries
B. Resemblance to a granuloma
C. Character of the exudate
D. Granular scar that results
E. Presence of monocytes and fibroblasts

9. Granulation tissue:
A. Is a feature of wound healing
B. Contains fibroblasts
C. Contains thin-walled capillaries
D. Often contains granuloma
E. Leads to scar formation

10. The following holds true for stable cells in cell cycle:
A. They remain in cell cycle from one mitosis to the next
B. They are in resting phase but can be stimulated to enter the cell cycle
C. They have left the cell cycle
D. They do not have capacity to multiply in response to stimuli throughout adult life

11. Connective tissue in scar is formed by the following types of fibrillar collagen:
A. Type II, III, IV
B. Type I, III, V
C. Type I, II, V
D. Type III, V, VII

12. Basement membrane consists of:


A. Type I collagen
B. Type V collagen
C. Type III collagen
D. Type IV collagen

13. Which of the following is non-fibrillar collagen?


A. Type V
B. Type I
C. Type III
D. Type VI

14. Which is false about primary union?


A. Exuberant granulation tissue to fill the gap
B. Clear margins
C. Uninfected
D. Lead to neat linear scar

15. A patient present with a large wound to his right forearm that is the result of a chain saw
accident. You treat his wound appropriately and follow him in your surgery clinic at routine
intervals. Initially his wound is filled with granulation tissue, which is composed of
proliferating fibroblasts and proliferating new blood vessels (angiogenesis). A growth factor
that is capable of inducing all the steps necessary for angiogenesis is:
A. Epidermal growth factor (EGF)
B. Transforming growth factor α (TGF-α)
C. Platelet-derived factor (PDGF)
D. Basic fibroblast growth factor (FGF)
E. Transforming growth factor β (TGF-β)

16. Systemic factors that influence wound healing are all of the following, except:
A. Nutrition
B. Metabolic status
C. Blood group
D. Circulatory status
E. Hormones (glucocorticoids)

17. Local factors that influence wound healing are all of the following, except:
a. Wound infection
b. Mechanical factors
c. Foreign bodies
d. Hormones (glucocorticoids)
e. Size, location and type of the wound

18. What type of wound healing can be termed as “keloid”?


a. Atrophic scar
b. Scar with dysplasia
c. Hyperplastic scar
d. Hypotrophic scar
e. Hypertrophic scar

19. The main difference between slowly dividing and rapidly dividing cells is the duration of
the next phase:
a. M (mitosis) phase: Phase of mitosis.
b. G1 (gap 1) phase: The daughter cell enters G1 phase after mitosis.
c. S (synthesis) phase: During this phase, the synthesis of nuclear DNA takes place.
d. G2 (gap 2) phase: After completion of nuclear DNA duplication, the cell enters G2 phase.
e. G0 (gap 0) phase: This is the quiescent or resting phase of the cell after an M phase.

20. These cells continue to multiply throughout life under normal physiologic conditions:
a. Labile cells
b. Stable cells
c. Permanent cells
d. a + b
e. None

21. These cells decrease or lose their ability to proliferate after adolescence but retain the
capacity to multiply in response to stimuli throughout adult life:
a. Labile cells
b. Stable cells
c. Permanent cells
d. a + b
e. None

22. These cells lose their ability to proliferate around the time of birth:
a. Labile cells
b. Stable cells
c. Permanent cells
d. a + b
e. None

23. Labile cells include following cells:


a. Epithelial cells of the alimentary tract
b. Parenchymal cells of liver
c. Neurons of nervous system
d. Hematopoietic cells of bone marrow
e. a + d

24. Stable cells include following cells:


a. Epithelial cells of the alimentary tract
b. Parenchymal cells of liver
c. Neurons of nervous system
d. Hematopoietic cells of bone marrow
e. a + d

25. Permanent cells include following cells:


a. Epithelial cells of the alimentary tract
b. Parenchymal cells of liver
c. Neurons of nervous system
d. Hematopoietic cells of bone marrow
e. a + d

26. What is the replacement of injured tissue by fibrosis tissue?


a. Regeneration
b. Repair
c. Atrophy
d. Metaplasia
e. Necrosis

27. The following phases are observed in the formation of granulation tissue:
a. Phase of inflammation
b. Phase of clearance
c. Phase of ingrowths of granulation tissue
d. a + d
e. a + b + c

28. Healing by first intention has the following characteristics, except:


a. Surgically incised
b. Without much loss of cells and tissue
c. Clean and uninfected
d. Open with a large tissue defect, at times infected
e. Edges of wound are approximated by surgical sutures

29. Healing by second intention has the following characteristics, except:


a. Open with a large tissue defect, at times infected
b. Clean and uninfected
c. Edges of wound are approximated by surgical sutures
d. b + c
e. Having extensive loss of cells and tissues

30. The following factors influence the wound healing is local, except:
a. Infection
b. Poor blood supply to wound
c. Exposure ultraviolet light
d. Type, size and location of injury
e. Age

31. The following factors influence the wound healing is systemic, except:
a. Nutrition
b. Systemic infection
c. Administration of glucocorticoids
d. Bleeding disorders
e. Exposure to ionizing radiation

32. Steps involved in the formation of procallus are as follows:


a. Hematoma → Local inflammatory response → Ingrowths of granulation tissue → Callus
composed of woven bone and cartilage
b. Hematoma → Ingrowths of granulation tissue → Local inflammatory response → Callus
composed of woven bone and cartilage
c. Callus composed of woven bone and cartilage → Ingrowths of granulation tissue →
Hematoma → Local inflammatory response
d. Ingrowths of granulation tissue → Local inflammatory response → Callus composed of
woven bone and cartilage → Hematoma
e. Local inflammatory response → Hematoma → Callus composed of woven bone and
cartilage → Ingrowths of granulation tissue

33. Destruction of which organs and tissues is replaced by fibrous tissue?


a. Epithelial of skin
b. Heart muscle
c. Liver with intact basement membrane
d. Tubular epithelial cells of kidney
e. Bone marrow

34. Which proteoglican is distributed in basement membranes?


a. Chondroitin sulphate
b. Heparan sulphate
c. Dermatan sulphate
d. Keratan sulphate
e. Hyaluronic acid

35. Labile cells are:


a. Cells of stratified squamous epithelium
b. Parenchymal cells of liver
c. Parenchymal cells of kidney
d. Skeletal muscle cells
e. Cells of neurons

36. Labile cells are:


a. Cells of columnar epithelium
b. Parenchymal cells of liver
c. Parenchymal cells of kidney
d. Skeletal muscle cells
e. Cells of neurons

37. Labile cells are:


a. Parenchymal cells of pancreas
b. Cells of lymph nodes
c. Parenchymal cells of kidney
d. Skeletal muscle cells
e. Cells of neurons

38. Labile cells are:


a. Parenchymal cells of pancreas
b. Hematopoietic cells of bone marrow
c. Parenchymal cells of kidney
d. Skeletal muscle cells
e. Cells of neurons

39. Stable cells are:


a. Cells of stratified squamous epithelium
b. Hematopoietic cells of bone marrow
c. Parenchymal cells of kidney
d. Cells of columnar epithelium
e. Cells of transitional epithelium

40. Stable cells are:


a. Cells of stratified squamous epithelium
b. Hematopoietic cells of bone marrow
c. Parenchymal cells of liver
d. Cells of columnar epithelium
e. Cells of transitional epithelium

41. Stable cells are:


a. Cells of stratified squamous epithelium
b. Cells of columnar epithelium
c. Cells of transitional epithelium
d. Hematopoietic cells of bone marrow
e. Smooth muscles cells

42. Stable cells are:


a. Cells of stratified squamous epithelium
b. Cells of columnar epithelium
c. Cells of transitional epithelium
d. Hematopoietic cells of bone marrow
e. Bone cells

43. Stable cells are:


a. Cells of stratified squamous epithelium
b. Cells of columnar epithelium
c. Cells of transitional epithelium
d. Hematopoietic cells of bone marrow
e. Cartilage cells

44. Stable cells are:


a. Cells of stratified squamous epithelium
b. Cells of columnar epithelium
c. Cells of transitional epithelium
d. Hematopoietic cells of bone marrow
e. Vascular endothelium

45. Permanent cells are:


a. Cells of stratified squamous epithelium
b. Cells of columnar epithelium
c. Smooth muscles cells
d. Cardiac muscle cells
e. Bone cells

46. Permanent cells are:


a. Cells of stratified squamous epithelium
b. Cells of columnar epithelium
c. Smooth muscles cells
d. Cells of neurons
e. Bone cells

47. Polypeptide growth factors circulating in the serum are, except:


a. Epidermal growth factor
b. Mesodermal growth factor
c. Platelet derived growth factor
d. Fibroblast growth factor
e. Transforming growth factor
48. Polypeptide growth factors circulating in the serum are, except:
a. Epidermal growth factor
b. Lymphocyte derived growth factor
c. Platelet derived growth factor
d. Fibroblast growth factor
e. Transforming growth factor

49. Polypeptide growth factors circulating in the serum are, except:


a. Epidermal growth factor
b. Platelet derived growth factor
c. Fibroblast growth factor
d. Chondroblast growth factor
e. Transforming growth factor

50. Functions of growth factors are, except:


a. Stimulation of cell division
b. Proliferation of cells
c. Synthesis of collagen
d. Cell differentiation
e. Depression of cell division

51. Functions of growth factors are, except:


a. Stimulation of cell division
b. Proliferation of cells
c. Synthesis of collagen
d. Cell differentiation
e. Metaplasia of cells

52. Functions of growth factors are, except:


a. Stimulation of cell division
b. Proliferation of cells
c. Synthesis of collagen
d. Cell differentiation
e. Atrophy of cells

53. Mechanisms of wound healing are, except:


a. Regeneration of parenchymal cells
b. Proliferation of connective tissue cells
c. Proliferation of hematopoietic cells
d. Remodeling of connective tissue
e. Collagenization and acquisition of wound strength

54. Mechanisms of wound healing are, except:


a. Regeneration of parenchymal cells
b. Proliferation of connective tissue cells
c. Remodeling of bone tissue
d. Remodeling of connective tissue
e. Collagenization and acquisition of wound strength
55. Sign of primary union of wound healing is:
a. Large amount of granulation tissue
b. Small amount of granulation tissue
c. Slow healing
d. Wound contraction
e. More inflammatory reaction

56. Sign of primary union of wound healing is:


a. Large amount of granulation tissue
b. Rapid healing
c. Slow healing
d. Wound contraction
e. More inflammatory reaction

57. Sign of primary union of wound healing is:


a. Large amount of granulation tissue
b. Less scar formation
c. More scar formation
d. Wound contraction
e. More inflammatory reaction

58. Signs of primary union of wound healing are, except:


a. Small amount of granulation tissue
b. Less scar formation
c. Rapid healing
d. More scar formation
e. Minimum loss of functions

59. Sign of secondary union of wound healing is:


a. Small amount of granulation tissue
b. Less scar formation
c. More scar formation
d. Rapid healing
e. Minimum loss of functions

60. Signs of secondary union of wound healing are, except:


a. Large amount of granulation tissue
b. More scar formation
c. Slow healing
d. Less inflammatory reaction
e. More inflammatory reaction

61. Signs of secondary union of wound healing are, except:


a. Large amount of granulation tissue
b. Rapid healing
c. Slow healing
d. Wound contraction
e. More inflammatory reaction

62. Signs of secondary union of wound healing are, except:


a. Large amount of granulation tissue
b. Less scar formation
c. More scar formation
d. Wound contraction
e. More inflammatory reaction

63. Local factor that adversely affect wound healing is:


a. Infection
b. Protein malnutrition
c. Severe anemia
d. Zink deficiency
e. Corticosteroid excess

64. Local factor that adversely affect wound healing is:


a. Poor blood supply
b. Protein malnutrition
c. Severe anemia
d. Zink deficiency
e. Corticosteroid excess

65. Systemic factor that adversely affect wound healing is:


a. Infection
b. Poor blood supply
c. Presence of necrotic tissue
d. Irradiation
e. Protein malnutrition

66. Systemic factor that adversely affect wound healing is:


a. Infection
b. Poor blood supply
c. Presence of necrotic tissue
d. Irradiation
e. Diabetes mellitus

67. All of the following cells are stable, except:


A. Parenchymal cells of liver
B. Parenchymal cells of pancreas
C. Parenchymal cells of kidneys
D. Parenchymal cells of thyroid
E. Surface epithelial cells of the epidermis

68. All of the following cells are stable, except:


A. Parenchymal cells of liver
B. Parenchymal cells of pancreas
C. Parenchymal cells of kidneys
D. Parenchymal cells of thyroid
E. Surface epithelial cells of the alimentary tract

69. All of the following cells are stable, except:


A. Parenchymal cells of liver
B. Parenchymal cells of pancreas
C. Parenchymal cells of kidneys
D. Parenchymal cells of thyroid
E. Surface epithelial cells of the respiratory tract

70. All of the following cells are stable, except:


A. Parenchymal cells of liver
B. Parenchymal cells of pancreas
C. Parenchymal cells of kidneys
D. Parenchymal cells of thyroid
E. Hematopoietic cells of bone marrow

71. All of the following cells are stable, except:


A. Parenchymal cells of liver
B. Parenchymal cells of pancreas
C. Parenchymal cells of kidneys
D. Parenchymal cells of thyroid
E. Neurons of nervous system

72. All of the following cells are stable, except:


A. Parenchymal cells of liver
B. Parenchymal cells of pancreas
C. Parenchymal cells of kidneys
D. Parenchymal cells of thyroid
E. Skeletal muscle cells

73. All of the following cells are stable, except:


A. Parenchymal cells of liver
B. Parenchymal cells of pancreas
C. Parenchymal cells of kidneys
D. Parenchymal cells of thyroid
E. Cardiac muscle cells

74. All of the following cells are labile, except:


A. Surface epithelial cells of the epidermis
B. Surface epithelial cells of the alimentary tract
C. Surface epithelial cells of the respiratory tract
D. Parenchymal cells of liver
E. Hematopoietic cells of bone marrow

75. All of the following cells are labile, except:


A. Surface epithelial cells of the epidermis
B. Surface epithelial cells of the alimentary tract
C. Surface epithelial cells of the respiratory tract
D. Parenchymal cells of pancreas
E. Hematopoietic cells of bone marrow

76. All of the following cells are labile, except:


A. Surface epithelial cells of the epidermis
B. Surface epithelial cells of the alimentary tract
C. Surface epithelial cells of the respiratory tract
D. Parenchymal cells of kidneys
E. Hematopoietic cells of bone marrow

77. All of the following cells are labile, except:


A. Surface epithelial cells of the epidermis
B. Surface epithelial cells of the alimentary tract
C. Surface epithelial cells of the respiratory tract
D. Parenchymal cells of thyroid
E. Hematopoietic cells of bone marrow

78. All of the following cells are labile, except:


A. Surface epithelial cells of the epidermis
B. Surface epithelial cells of the alimentary tract
C. Surface epithelial cells of the respiratory tract
D. Neurons of nervous system
E. Hematopoietic cells of bone marrow

79. All of the following cells are labile, except:


A. Surface epithelial cells of the epidermis
B. Surface epithelial cells of the alimentary tract
C. Surface epithelial cells of the respiratory tract
D. Skeletal muscle cells
E. Hematopoietic cells of bone marrow

80. All of the following cells are labile, except:


A. Surface epithelial cells of the epidermis
B. Surface epithelial cells of the alimentary tract
C. Surface epithelial cells of the respiratory tract
D. Cardiac muscle cells
E. Hematopoietic cells of bone marrow

Chronic inflammation - новые

1. Select the option of cells in chronic inflammation:


A. Monocytes.
D. Myelocytes.
B. Erythrocytes.
C. Monocytes.
D. Neutrophils.
E. Adipocytes.

2. Select a specific variant of cells in chronic inflammation:


A. Neutrophils.
B. Plasma cells.
C. Erythrocytes.
D. Myelocytes.
E. Adipocytes.

3. Find and specify the variant of cells in chronic inflammation:


A. Neutrophils.
B. Erythrocytes.
C. Astrocytes.
D. Giant cells.
E. Osteocytes.

4. Are there any chronic inflammation cells in the list? Underscore:


A. Neutrophils.
B. Erythrocytes.
C. Mast cells.
D. Epithelial cells.
E. Chondrocytes.

5. Select a specific variant of cells in chronic inflammation:


A. Neutrophils.
B. Plasma cells.
C. Erythrocytes.
D. Myelocytes.
E. Adipocytes.

6. Are there any chronic inflammation cells in the list? Underscore:


A. Neutrophils.
B. Erythrocytes.
C. Mast cells.
D. Epithelial cells.
E. Chondrocytes.

7. Find and specify the variant of cells in chronic inflammation:


A. Neutrophils.
B. Erythrocytes.
C. Astrocytes.
D. Giant cells.
E. Osteocytes.

8. Cells of chronic inflammation are:


A. Neutrophils.
B. Epitelioid cells.
C. Epithelial cells.
D. Astrocytes.
E. Osteocytes.

9. Are there any chronic inflammation cells in the list? Underscore:


A. Neutrophils.
B. Erythrocytes.
C. Mast cells.
D. Epithelial cells.
E. Chondrocytes.

10. Select cells of chronic inflammation in the list:


A. Fibrolasts.
B. Osteoblasts.
C. Chondroblasts.
D. Lipoblasts.
E. Lymphoblasts.
E. Fibroblasts.

11. Production of mediators is associated with certain cells:. With which?


A. Lymphocytes.
B. Plasma cells.
C. Eosinophils.
D. Must cells.
E. Fibroblasts.

12. Which cells are characteristically found in inflammatory sites around animal parasites?
A. Lymphocytes.
B. Plasma cells.
C. Must cells..
D. Eosinophils.
E. Fibroblasts.

13. Which cells are central players in anaphylactic shock?


A. Lymphocytes.
B. Plasma cells.
C. Eosinophils.
D. Must cells.
E. Fibroblasts.

14. Select the type of cells that produce collagen.


A. Lymphocytes.
B. Plasma cells.
C. Eosinophils.
D. Must cells.
E. Fibroblasts.

15. Infectious granulomas occur in the case of::


A. Tuberculosis.
B. Rheumatic fever.
C. Rheumatoid arthritis.
D. Sarcoidosis
E. Regional ileitis.

16. Infectious granulomas occur in the case of::


A. Rheumatic fever.
B. Syphilis.
C. Rheumatoid arthritis.
D. Sarcoidosis
E. Regional ileitis.

17. Infectious granulomas occur in the case of::


A. Rheumatic fever.
B. Rheumatoid arthritis.
C. Leprosy.
D. Sarcoidosis
E. Regional ileitis.

18. Infectious granulomas occur in the case of::


A. Rheumatic fever.
B. Rheumatoid arthritis.
C. Sarcoidosis.
D. Typhoid fever.
E. Regional ileitis.

19. In any case, meet allergic granuloma?


A. Tuberculosis.
B. Syphilis.
C. Rheumatic fever.
D. Typhoid fever.
E. Leprosy.

20. In any case, meet allergic granuloma?


A. Tuberculosis.
B. Syphilis.
C. Rheumatoid arthritis.
D. Typhoid fever.
E. Leprosy.

21. Granulomas of unknown etiology can occur only when:


A. Tuberculosis.
B. Syphilis.
C. Rheumatic fever.
D. Sarcoidosis.
E. Leprosy.

22. Granulomas of unknown etiology can occur only when:


A. Tuberculosis.
B. Syphilis.
C. Rheumatic fever.
D. Regional ileitis.
E. Leprosy.

23. Interstitial inflammation is determined by characteristic changes:


A. Formation of granulomas.
B. Inflammatory infiltration of the stroma of inner organs.
C. Formation of pseudopolyps.
D. Cell degeneration of inner organs.
E. Formation of condylomas.
24. Where is the most common location of infectious polyps?
A. Small intestine.
B. Large intestine.
C. Rectum.
D. Duodenum.
E. Appendix.

25. Where is the most common location of infectious polyps?


A. Stomach.
B. Small intestine.
C. Large intestine.
D. Duodenum.
E. Appendix.

26. Where is the most common location of infectious polyps?


A. Oral cavity.
B. Nasal cavity.
C. Pleural cavity.
D. Peritoneal cavity.
E. Joint cavity.

27. In chronic non-specific inflammation, there are certain morphological changes::


A. Neutrophils, lymphocytes and liquefaction necrosis.
B. Neutrophils, macrophages and fibrosis.
C. Lymphocytes, plasma cells and fibrosis.
D. Giant cells, macrophages and coagulative necrosis.

28. In what ways are accumulated macrophages in chronic inflammation?


A. Continuous inflow of monocytes from circulation maintained by chemotactic factors.
B. Local proliferation of macrophages by mitotic division
C. Prolonged immobilization of macrophages within the site of inflammation
D. A and B
E. All the three

29. Exactly how is formed a granuloma with inflammation?


A. Type I hypersensitivity reaction
B. Type II hypersensitivity reaction
C. Type III hypersensitivity reaction
D. Type IV hypersensitivity reaction
E. All of these

30. What is most often associated with the formation of granulomas?


A. The healing process.
B. Acute inflammation.
C. Wound contraction.
D. Fibroblasts and neovascularization.
E. A persistent irritant.
31. Specify the option of chronic inflammation of the myoma:
A. Chronic cervicitis
B. Chronic gastritis
C. Chronic cholecystitis
D. None of these
E. All of these

32. Where you can find the epithelioid cells in tuberculous granuloma?
A. Neutrophils
B. Eosinophils
C. Lymphocytes
D. Monocytes
E. Macrophages

33. A possible variant of the Epithelial cell modified-specify.


A. Lymphocytes
B. Macrophages
C. Mast cell
D. Eosinophils
E. Neutrophils

34. In any options found in a granuloma, except:


A. TB
B. Yersinia
C. Mycoplasma
D. Leprosy
E. Syphilis

35. Specify a variant of atypical mycobacteria in inflammation.


A. Mycobacterium microti
B. Mycobacterium canneti
C. Mycobacterium africanum
D. Mycobacterium ulcerans

36. How IgM antibodies are used PGL-1 is used for diagnosis?
A. Leprosy
B. Tuberculosis
C. Syphilis
D. Brucellosis
E. Mycoplasmosis

37. What kind of leprosy is not included in the classification of Ridley-Jopling?


A. Mid borderline leprosy
B. Borderline tuberculoid leprosy
C. Inderterminate leprosy
D. Tuberculoid polar leprosy
E. Borderline lepromatous

38. In which case we see the Drug hepar lobatum, specify where:
A. Primary syphilis
B. Secondary syphilis
C. Tertiary syphilis
D. Congenital syphilis

39. The mechanism of killing of M. tuberculosis which grows inside the macrophage::
A. By reactive oxygen species
B. By oxygen-independent bactericidal mechanism
C. By nitric oxide mechanism
D. By hydrolytic enzymes

40. The mechanism of the lesion tubercular bacilli - specify option:


A. Elaboration of endotoxin
B. Elaboration of exotoxin
C. Type IV hypersensitivity
D. Direct cytotoxicity

41. What is true for tuberculosis Bacillus, with the exception of:
A. Tubercle bacilli can be cultured
B. Tubercle bacilli are anaerobe
C. Tubercle bacilli thrive best in the apex of lung
D. M.smegmatis is not pathogenic to man

42. How tubercle bacilli in caseous lesions are best demonstrated in:
A. Caseous centre
B. Margin of necrosis with viable tissue
C. Epithelioid cells
D. Langhans′ giant cells

43. Specify these options for the Bacilli of leprosy:


A. Not acid fast
B. As acid fast as tubercle bacilli
C. Less acid fast compared to tubercle bacilli
D. More acid fast compared to tubercle bacilli

44. Lepromin test may be positive when:


A. Lepromatous leprosy
B. Borderline lepromatous leprosy
C. Tuberculoid leprosy
D. Inderterminate leprosy

45. Where the most difficult to demonstrate Spirochetes in Spirochetes?


A. Primary syphilis
B. Secondary syphilis
C. Tertiary syphilis
D. Congenital syphilis

46. Specify in which cases Actinomycosis is caused:


A. Fungus
B. Gram-negative bacteria
C. Anaerobic bacteria
D. Acid fast bacteria

47. Sarcoid granuloma has the following features, except:


A. Non caseating granuloma
B. Giant cells have cytoplasmic inclusions
C. Peripheral mantle of lymphocytes
D. Fibroblastic proliferation at the periphery of a granuloma

48. Granulomas consists of:


A. Cholesterol clefts
B. Collagen
C. Endothelial cells and fibroblasts
D. Epithelioid cells
E. Hemosiderin-laden macrophages

49. Finding acid-fast bacilli within peripheral nerves is most suggestive of


A. Relapsing fever
B. Syphilis
C. Leprosy
D. Tuberculosis
E. Weil′s disease

50. In the study of sputum in a young patient reveals the presence of rare acid-resistant
organisms. Maybe it's the infection.
A. K. pneumoniae
B. L. pneumophila
C. Mycobacterium avium-intracellulare
D. Mycobacterium tuberculosis
E. Mycoplasma pneumonia

51. How reliable histopathological evidence of chroniclesthe inflammatory process in the


organs is?
A. Hemorrhages
B. Leucocytic infiltrates
C. Blood vessels destruction
D. Interstitial fibrosis
E. Councilman's bodies

52. In the microscopic part of the ovary removed during surgery, a large accumulation of
epithelioid cells observed your diagnosis:
A. Granulation tissue
B. Pyogenic granuloma
C. Granulosa cell tumor
D. Granulocytosis
E. Granuloma

53. In primary tuberculosis Granuloma consists mainly of the following:


A. Fibroblasts
B. Epithelioid cells
C. Eosinophils
D. Plasma cells
E. Neutrophils

54. In primary syphilis Granulomatous infiltration cell consists of the following:


A. Neutrophils
B. Monocytes/macrophages
C. Plasma cells
D. Eosinophils
E. Lymphocyles

55. What type of inflammation is characteristic of Miliary pulmonary tuberculosis?


A. Granulomatous
B. Serous
C. Fibrinous
D. Suppurative
E. Hemorrhagic

56. Chronic inflammation is most reliably characterized by the following:


A. Infiltration with mononuclear cell including macrophages, lymphocytes, and plasma
cells
B. Tissue destruction
C. Healing by connective tissue with angiogenesis and fibrosis
D. All of these
E. None of these

57. The causes of chronic inflammation are all of the following, except:
A. Persistent infections by certain microorganisms
B. Prolonged exposure to potentially toxic agents, either exogenous or endogenous
C. Autoimunity (autoimmune diseases)
D. Complete phagocytosis
E. Resistance of the etiologic agent

58. Determine which cells play an important role in chronic inflammation of tuberculosis?
A. Macrophages
B. Leucocytes
C. Eosinophils
D. Erythrocytcs
E. Plasma cells

59. All cells can be found in chronic inflammation infiltrate, except for these:
A. Lymphocytes
B. Platelets
C. Macrophages
D. Plasma cells
E. Eosinophils

60. Granulomatous inflammation develops in all these diseases, except:


A. Tuberculosis
B. Leprosy
C. Syphilis
D. Cat-scratch disease
E. Budd-Chiary syndrome

61. Specify which of the types of necrosis can be detected in granuloma of tuberculosis?
A. Coagulation necrosis
B. Liquefactive necrosis
C. Caseous necrosis
D. Enzymatic fat necrosis
E. Fibrinoid necrosis

62. Tuberculosis granuloma can be characterized by all of the following except:


A. Plasma cells
B. Area of central necrosis
C. Epithelioid cells
D. Langhans-type giant cells
E. Lymphocytes

63. Another name for Syphilis granuloma:


A. Fibroma
B. Gumma
C. Tuberculoma
D. Leproma
E. Hepatoma

64. Granuloma of syphilis is characterized by all the following except:


A. Area of central necrosis
B. Plasma cell infiltrate
C. Lymphocyte infiltrate
D. Productive vasculitis
E. Platelet infiltrate

65. In granulomatous inflammation Macrophages can be transformed into which of the cells:
A. Monocytes
B. Epithelial cells
C. Epithelioid cells
D. Plasma cells
E. Lymphocytes

66. Where it can be found in Gummatous Infiltration of tertiary syphilis?


A. Aorta
B. Testes
C. Liver
D. Bones and joints
E. Skin and subcutaneous tissue

67. In the examination, syphilitic Gumma is characterized by all the following features,
except:
A. White-gray
B. Rubbery
C. Solitary
D. Red-brown
E. Tumor-like

68. Products of activated macrophages in tissue injury include all of the following, except:
A. Fibrogenic cytokines
B. Toxic oxygen metabolites
C. Collagenases
D. Neutrophile chemotactive factors
E. Elastase

69. Products of activated macrophages in tissue injury include all of the following, except:
A. Growth factors
B. Fibrogenic cytokines
C. Angiogenesis factors
D. Fibronectin
E. Proteases

70. What are the lung with multiple tubercular granulomas?


A. Tuberculous pneumonia
B. Brown induration of lungs
C. Miliary tuberculosis
D. Cavitary fibrocaseous tuberculosis
E. Tuberculoma

71. The pathologic changes of vasa vasorum of aorta in syphilitic mesaortitis are
characterized by which of the following:
A. Migratory thrombophlebitis
B. Productive vasculitis (obliterative endoarteritis)
C. Thromboangitis obliterans
D. Necrotizing arteriolitis
E. Thrombotic microangiopathy

72. What can lead to Medial destruction of the aorta in tertiary syphilis?
A. Aneurismal dilatation of the aorta
B. Marian's syndrome
C. Atherosclerosis
D. Takayasu's arteritis
E. Giant cell arteritis

73. What is associated with the development of sarcoidosis?


A. Mycobaclerium leprae
B. Mycobacterium tuberculosis
C. Treponema pallidum
D. Gram-negative bacillis
E. Unknown

74. Foreign body granulomas can be caused by all the following reasons except:
A. Paniculate matter
B. Synthetic material
C. Gram-negative bacillus
D. Vegetable matter
E. Beryllium particles

75. Macrophage functions can include the following, except:


A. Phagocytosis.
B. Degradation of engulfed particles.
C. Tissue destruction.
D. Forming of giant cells.
E. Producing of mediators.

76. Macrophage functions can include the following, except:


A. Phagocytosis.
B. Degradation of engulfed particles.
C. Tissue destruction.
D. Syntesis og collagen.
E. Forming of giant cells.

77. Macrophage functions can include the following, except:


A. Phagocytosis.
B. Degradation of engulfed particles.
C. Producing of antibodies.
D. Tissue destruction.
E. Forming of giant cells.

78. Macrophage functions can include the following, except:


A. Phagocytosis.
B. Regeneration of epithelial cells.
C. Degradation of engulfed particles.
D. Tissue destruction.
E. Forming of giant cells.

79. Localization of interstitial inflammation is the following, except:


A. Liver.
B. Kidneys.
C. Heart.
D. Lungs.
E. Spleen.

80. Localization of interstitial inflammation is the following, except:


A. Liver.
B. Kidneys.
C. Heart.
D. Lungs.
E. Lymph nodes.
Adaptations - новые
1. Increase of number of cells is termed:
A. Hypertrophy
B. Hyperplasia
C. Atrophy
D. Aplasia
E. Dysplasia

2. Change in cell polarity, which is reversible on treatment is -


A. Metaplasia
B. Dysplasia
C. Anaplasia
D. None

3. Reversible disorder with variability in size, shape and polarity of cells is -


A. Metaplasia
B. Dysplasia
C. Anaplasia
D. Hyperplasia
E. Desmoplasia

4. Hyperplasia is -
A. Results in increased cell size
B. Results in increased cell numbers
C. Occurs in corneal endothelium
D. Occurs in retinal pigment epithelium
E. Occurs in myocardium

5. The term atrophy implies:


A. Increase in size and number of cells
B. Decrease in size of cells
C. Decrease in number of cells
D. Increase in size and number of cells
E. Increase in size of cells

6. True hypertrophy characterized by –


A. Enlargement of organ due to foreign bodies
B. Connective tissue growth
C. Accumulation of fluid in the cavities
D. Hyperplasia and hypertrophy of cells or organelles
E. Fat tissue growth

7. In atrophy, the cells are:


A. Dead cells
B. Shrunken cells
C. Irreversibly injured cells
D. Reversibly injured cells

8. For metaplasia the following holds true:


A. It is a disordered growth
B. It affects only epithelial tissues
C. It is a reversible change
D. It is an irreversible and progressive change

9. Histologic sections of an enlarged tonsil from a 9-year-old female reveal an increased


number of reactive follicles containing germinal centers with proliferating B-lymphocytes.
Which one of the listed terms best describes this pathologic process?
A. B-lymphocyte hypertrophy
B. Follicular dysplasia
C. Follicular hyperplasia
D. Germinal center atrophy
E. Germinal center metaplasia

10. Causes of pathological atrophy are all of the following, except:


A. Starvation atrophy
B. Ischemic atrophy
C. Disuse atrophy
D. Hypertrophy of smooth muscle

11. Causes of pathological atrophy are all of the following, except:


A. Starvation atrophy
B. Ischemic atrophy
C. Disuse atrophy
D. Hypertrophy of cardiac muscle

12. Causes of pathological atrophy are all of the following, except:


A. Starvation atrophy
B. Ischemic atrophy
C. Disuse atrophy
D. Hypertrophy of skeletal muscle

13. Causes of pathological atrophy are all of the following, except:


A. Starvation atrophy
B. Ischemic atrophy
C. Disuse atrophy
D. Compensatory hypertrophy

14. Causes of pathological atrophy are all of the following, except:


A. Starvation atrophy
B. Ischemic atrophy
C. Disuse atrophy
D. Enlarged size of the uterus in pregnancy

15. Causes of pathological atrophy are all of the following, except:


A. Starvation atrophy
B. Ischemic atrophy
C. Disuse atrophy
D. Hormonal hyperplasia

16. Causes of pathological atrophy are all of the following, except:


A. Starvation atrophy
B. Ischemic atrophy
C. Disuse atrophy
D. Compensatory hyperplasia

17. Causes of pathological atrophy are all of the following, except:


A. Neuropathic atrophy
B. Ischemic atrophy
C. Disuse atrophy
D. Hypertrophy of smooth muscle

18. Causes of pathological atrophy are all of the following, except:


A. Neuropathic atrophy
B. Ischemic atrophy
C. Disuse atrophy
D. Hypertrophy of cardiac muscle

19. Causes of pathological atrophy are all of the following, except:


A. Neuropathic atrophy
B. Ischemic atrophy
C. Disuse atrophy
D. Hypertrophy of skeletal muscle

20. Causes of pathological atrophy are all of the following, except:


A. Neuropathic atrophy
B. Ischemic atrophy
C. Disuse atrophy
D. Compensatory hypertrophy

21. Causes of pathological atrophy are all of the following, except:


A. Neuropathic atrophy
B. Ischemic atrophy
C. Disuse atrophy
D. Enlarged size of the uterus in pregnancy

22. Causes of pathological atrophy are all of the following, except:


A. Neuropathic atrophy
B. Ischemic atrophy
C. Disuse atrophy
D. Hormonal hyperplasia

23. Causes of pathological atrophy are all of the following, except:


A. Neuropathic atrophy
B. Ischemic atrophy
C. Disuse atrophy
D. Compensatory hyperplasia

24. Causes of pathological atrophy are all of the following, except:


A. Neuropathic atrophy
B. Endocrine atrophy
C. Disuse atrophy
D. Hypertrophy of smooth muscle

25. Causes of pathological atrophy are all of the following, except:


A. Neuropathic atrophy
B. Endocrine atrophy
C. Disuse atrophy
D. Hypertrophy of cardiac muscle

26. Causes of pathological atrophy are all of the following, except:


A. Neuropathic atrophy
B. Endocrine atrophy
C. Disuse atrophy
D. Hypertrophy of skeletal muscle

27. Causes of pathological atrophy are all of the following, except:


A. Neuropathic atrophy
B. Endocrine atrophy
C. Disuse atrophy
D. Compensatory hypertrophy

28. Causes of pathological atrophy are all of the following, except:


A. Neuropathic atrophy
B. Endocrine atrophy
C. Disuse atrophy
D. Enlarged size of the uterus in pregnancy

29. Causes of pathological atrophy are all of the following, except:


A. Neuropathic atrophy
B. Endocrine atrophy
C. Disuse atrophy
D. Hormonal hyperplasia

30. Causes of pathological atrophy are all of the following, except:


A. Neuropathic atrophy
B. Endocrine atrophy
C. Disuse atrophy
D. Compensatory hyperplasia

31. Causes of pathological atrophy are all of the following, except:


A. Neuropathic atrophy
B. Endocrine atrophy
C. Pressure atrophy
D. Hypertrophy of smooth muscle

32. Causes of pathological atrophy are all of the following, except:


A. Neuropathic atrophy
B. Endocrine atrophy
C. Pressure atrophy
D. Hypertrophy of cardiac muscle
33. Causes of pathological atrophy are all of the following, except:
A. Neuropathic atrophy
B. Endocrine atrophy
C. Pressure atrophy
D. Hypertrophy of skeletal muscle

34. Causes of pathological atrophy are all of the following, except:


A. Neuropathic atrophy
B. Endocrine atrophy
C. Pressure atrophy
D. Compensatory hypertrophy

35. Causes of pathological atrophy are all of the following, except:


A. Neuropathic atrophy
B. Endocrine atrophy
C. Pressure atrophy
D. Enlarged size of the uterus in pregnancy

36. Causes of pathological atrophy are all of the following, except:


A. Neuropathic atrophy
B. Endocrine atrophy
C. Pressure atrophy
D. Hormonal hyperplasia

37. Causes of pathological atrophy are all of the following, except:


A. Neuropathic atrophy
B. Endocrine atrophy
C. Pressure atrophy
D. Compensatory hyperplasia

38. Causes of pathological atrophy are all of the following, except:


A. Starvation atrophy
B. Ischemic atrophy
C. Disuse atrophy
D. Autolysis

39. Causes of pathological atrophy are all of the following, except:


A. Starvation atrophy
B. Ischemic atrophy
C. Disuse atrophy
D. Necrosis

40. Causes of pathological atrophy are all of the following, except:


A. Starvation atrophy
B. Ischemic atrophy
C. Disuse atrophy
D. Coagulative necrosis

41. Causes of pathological atrophy are all of the following, except:


A. Starvation atrophy
B. Ischemic atrophy
C. Disuse atrophy
D. Liquefaction necrosis

42. Causes of pathological atrophy are all of the following, except:


A. Starvation atrophy
B. Ischemic atrophy
C. Disuse atrophy
D. Caseous necrosis

43. Causes of pathological atrophy are all of the following, except:


A. Starvation atrophy
B. Ischemic atrophy
C. Disuse atrophy
D. Fat necrosis

44. Causes of pathological atrophy are all of the following, except:


A. Starvation atrophy
B. Ischemic atrophy
C. Disuse atrophy
D. Fibrinoid necrosis

45. Causes of pathological atrophy are all of the following, except:


A. Neuropathic atrophy
B. Ischemic atrophy
C. Disuse atrophy
D. Gangrene

46. Causes of pathological atrophy are all of the following, except:


A. Neuropathic atrophy
B. Ischemic atrophy
C. Disuse atrophy
D. Dry gangrene

47. Causes of pathological atrophy are all of the following, except:


A. Neuropathic atrophy
B. Ischemic atrophy
C. Disuse atrophy
D. Wet gangrene

48. Causes of pathological atrophy are all of the following, except:


A. Neuropathic atrophy
B. Ischemic atrophy
C. Disuse atrophy
D. Gas gangrene

49. Causes of pathological atrophy are all of the following, except:


A. Neuropathic atrophy
B. Ischemic atrophy
C. Disuse atrophy
D. Squamous metaplasia in bronchus

50. Causes of pathological atrophy are all of the following, except:


A. Neuropathic atrophy
B. Ischemic atrophy
C. Disuse atrophy
D. Squamous metaplasia in uterine endocervix

51. Causes of pathological atrophy are all of the following, except:


A. Neuropathic atrophy
B. Ischemic atrophy
C. Disuse atrophy
D. Squamous metaplasia in gallbladder

52. Causes of pathological atrophy are all of the following, except:


A. Neuropathic atrophy
B. Endocrine atrophy
C. Disuse atrophy
D. Squamous metaplasia in prostate

53. Causes of pathological atrophy are all of the following, except:


A. Neuropathic atrophy
B. Endocrine atrophy
C. Disuse atrophy
D. Squamous metaplasia in renal pelvis

54. Causes of pathological atrophy are all of the following, except:


A. Neuropathic atrophy
B. Endocrine atrophy
C. Disuse atrophy
D. Squamous metaplasia in urinary bladder

55. Causes of pathological atrophy are all of the following, except:


A. Neuropathic atrophy
B. Endocrine atrophy
C. Disuse atrophy
D. Columnar (interstinal) metaplasia in healed chronic gastric ulcer

56. Causes of pathological atrophy are all of the following, except:


A. Neuropathic atrophy
B. Endocrine atrophy
C. Disuse atrophy
D. Columnar metaplasia in Barrett's esophagus

57. Causes of pathological atrophy are all of the following, except:


A. Neuropathic atrophy
B. Endocrine atrophy
C. Disuse atrophy
D. Osseous metaplasia
58. Causes of pathological atrophy are all of the following, except:
A. Neuropathic atrophy
B. Endocrine atrophy
C. Disuse atrophy
D. Cartilaginous metaplasia

59. Causes of pathological atrophy are all of the following, except:


A. Neuropathic atrophy
B. Endocrine atrophy
C. Pressure atrophy
D. Dysplasia

60. Causes of pathological atrophy are all of the following, except:


A. Neuropathic atrophy
B. Endocrine atrophy
C. Pressure atrophy
D. Renal edema in nephrotic syndrome

61. Causes of pathological atrophy are all of the following, except:


A. Neuropathic atrophy
B. Endocrine atrophy
C. Pressure atrophy
D. Edema in nephritic syndrome

62. Causes of pathological atrophy are all of the following, except:


A. Neuropathic atrophy
B. Endocrine atrophy
C. Pressure atrophy
D. Edema in acute tubular injury

63. Causes of pathological atrophy are all of the following, except:


A. Neuropathic atrophy
B. Endocrine atrophy
C. Pressure atrophy
D. Cardiac edema

64. Causes of pathological atrophy are all of the following, except:


A. Neuropathic atrophy
B. Endocrine atrophy
C. Pressure atrophy
D. Pulmonary edema

65. Causes of pathological atrophy are all of the following, except:


A. Neuropathic atrophy
B. Endocrine atrophy
C. Pressure atrophy
D. Cerebral edema

66. Causes of pathological hypertrophy are all of the following, except:


A. Starvation atrophy
B. Hypertrophy of cardiac muscle in systemic hypertension
C. Hypertrophy of cardiac muscle in aortic valve disease
D. Hypertrophy of cardiac muscle in mitral insufficiency

67. Causes of pathological hypertrophy are all of the following, except:


A. Ischemic atrophy
B. Hypertrophy of cardiac muscle in systemic hypertension
C. Hypertrophy of cardiac muscle in aortic valve disease
D. Hypertrophy of cardiac muscle in mitral insufficiency

68. Causes of pathological hypertrophy are all of the following, except:


A. Disuse atrophy
B. Hypertrophy of cardiac muscle in systemic hypertension
C. Hypertrophy of cardiac muscle in aortic valve disease
D. Hypertrophy of cardiac muscle in mitral insufficiency

69. Causes of pathological hypertrophy are all of the following, except:


A. Neuropathic atrophy in poliomyelitis
B. Hypertrophy of cardiac muscle in systemic hypertension
C. Hypertrophy of cardiac muscle in aortic valve disease
D. Hypertrophy of cardiac muscle in mitral insufficiency

70. Causes of pathological hypertrophy are all of the following, except:


A. Starvation atrophy
B. Hypertrophy of smooth muscle in cardiac achalasia (in esophagus)
C. Hypertrophy of cardiac muscle in aortic valve disease
D. Hypertrophy of cardiac muscle in mitral insufficiency

71. Causes of pathological hypertrophy are all of the following, except:


A. Ischemic atrophy
B. Hypertrophy of smooth muscle in cardiac achalasia (in esophagus)
C. Hypertrophy of cardiac muscle in aortic valve disease
D. Hypertrophy of cardiac muscle in mitral insufficiency

72. Causes of pathological hypertrophy are all of the following, except:


A. Disuse atrophy
B. Hypertrophy of cardiac muscle in systemic hypertension
C. Hypertrophy of smooth muscle in cardiac achalasia (in esophagus)
D. Hypertrophy of cardiac muscle in mitral insufficiency

73. Causes of pathological hypertrophy are all of the following, except:


A. Neuropathic atrophy in poliomyelitis
B. Hypertrophy of cardiac muscle in systemic hypertension
C. Hypertrophy of smooth muscle in cardiac achalasia (in esophagus)
D. Hypertrophy of cardiac muscle in mitral insufficiency

74. Causes of pathological hypertrophy are all of the following, except:


A. Starvation atrophy
B. Hypertrophy of smooth muscle in cardiac achalasia (in esophagus)
C. Hypertrophy of cardiac muscle in aortic valve disease
D. Hypertrophy of skeletal muscle in athletes and manual labourers

75. Causes of pathological hypertrophy are all of the following, except:


A. Ischemic atrophy
B. Hypertrophy of smooth muscle in cardiac achalasia (in esophagus)
C. Hypertrophy of cardiac muscle in aortic valve disease
D. Hypertrophy of skeletal muscle in athletes and manual labourers

76. Causes of pathological hypertrophy are all of the following, except:


A. Disuse atrophy
B. Hypertrophy of cardiac muscle in systemic hypertension
C. Hypertrophy of smooth muscle in cardiac achalasia (in esophagus)
D. Hypertrophy of skeletal muscle in athletes and manual labourers

77. Causes of pathological hypertrophy are all of the following, except:


A. Neuropathic atrophy in poliomyelitis
B. Hypertrophy of cardiac muscle in systemic hypertension
C. Hypertrophy of smooth muscle in cardiac achalasia (in esophagus)
D. Hypertrophy of skeletal muscle in athletes and manual labourers

78. Causes of pathological hypertrophy are all of the following, except:


A. Starvation atrophy
B. Compensatory hypertrophy
C. Hypertrophy of cardiac muscle in aortic valve disease
D. Hypertrophy of skeletal muscle in athletes and manual labourers

79. Causes of pathological hypertrophy are all of the following, except:


A. Ischemic atrophy
B. Compensatory hypertrophy
C. Hypertrophy of cardiac muscle in aortic valve disease
D. Hypertrophy of skeletal muscle in athletes and manual labourers

80. Causes of pathological hypertrophy are all of the following, except:


A. Disuse atrophy
B. Compensatory hypertrophy
C. Hypertrophy of smooth muscle in cardiac achalasia (in esophagus)
D. Hypertrophy of skeletal muscle in athletes and manual labourers

81. Causes of pathological hypertrophy are all of the following, except:


A. Neuropathic atrophy in poliomyelitis
B. Compensatory hypertrophy
C. Hypertrophy of smooth muscle in cardiac achalasia (in esophagus)
D. Hypertrophy of skeletal muscle in athletes and manual labourers

82. Causes of pathological hyperplasia are all of the following, except:


A. Pseudocarcinomatous hyperplasia of the skin
B. Endometrial hyperplasia following estrogen excess
C. Hypertrophy of cardiac muscle in systemic hypertension
D. In wound healing, there is formation of granulation tissue due proliferation of fibroblasts
and endothelial cells

83. Causes of pathological hyperplasia are all of the following, except:


A. Pseudocarcinomatous hyperplasia of the skin
B. Endometrial hyperplasia following estrogen excess
C. Hypertrophy of skeletal muscle in athletes and manual labourers
D. In wound healing, there is formation of granulation tissue due proliferation of fibroblasts
and endothelial cells

84. Causes of pathological hyperplasia are all of the following, except:


A. Pseudocarcinomatous hyperplasia of the skin
B. Endometrial hyperplasia following estrogen excess
C. Starvation atrophy
D. In wound healing, there is formation of granulation tissue due proliferation of fibroblasts
and endothelial cells

85. Causes of pathological hyperplasia are all of the following, except:


A. Pseudocarcinomatous hyperplasia of the skin
B. Endometrial hyperplasia following estrogen excess
C. Compensatory hypertrophy
D. In wound healing, there is formation of granulation tissue due proliferation of fibroblasts
and endothelial cells

86. Causes of pathological hyperplasia are all of the following, except:


A. Pseudocarcinomatous hyperplasia of the skin
B. Endometrial hyperplasia following estrogen excess
C. Neuropathic atrophy in poliomyelitis
D. In wound healing, there is formation of granulation tissue due proliferation of fibroblasts
and endothelial cells

87. Causes of pathological hyperplasia are all of the following, except:


A. Pseudocarcinomatous hyperplasia of the skin
B. Endometrial hyperplasia following estrogen excess
C. Hypertrophy of smooth muscle in cardiac achalasia (in esophagus)
D. In wound healing, there is formation of granulation tissue due proliferation of fibroblasts
and endothelial cells

88. Causes of pathological hyperplasia are all of the following, except:


A. Pseudocarcinomatous hyperplasia of the skin
B. Endometrial hyperplasia following estrogen excess
C. Epithelial metaplasia
D. In wound healing, there is formation of granulation tissue due proliferation of fibroblasts
and endothelial cells

89. Causes of pathological hyperplasia are all of the following, except:


A. Pseudocarcinomatous hyperplasia of the skin
B. Endometrial hyperplasia following estrogen excess
C. Mesenchymal metaplasia
D. In wound healing, there is formation of granulation tissue due proliferation of fibroblasts
and endothelial cells

90. Causes of pathological hyperplasia are all of the following, except:


A. Pseudocarcinomatous hyperplasia of the skin
B. Endometrial hyperplasia following estrogen excess
C. Dysplasia
D. In wound healing, there is formation of granulation tissue due proliferation of fibroblasts
and endothelial cells

1. What does the proliferation of neoplastic cells lead to?


A. Neoplasia
B. Tumors
C. Atrophy
D. Freckles

2. What is the term that means "new growth?"


A. Anaplasia
B. Metaplasia
C. Neoplasia
D. Hyperplasia

3. What kind of tumors have a limited growth potential and a good outcome?
A. Malignant
B. Hypertrophic
C. Hypotrophic
D. Benign

4. Who determines the definitive diagnosis of tumors?


A. Oncologist
B. Physician
C. Your mom
D. Pathologist
5. What kind of tumors resemble the tissue from which they have arisen?
A. Hypertrophic
B. Malignant
C. Benign
D. Tumor-like

6. What does pleomorphism mean?


A. Uncontrolled mitosis
B. Multiple nuclei
C. Variability in shape and size
D. Nucleus – cytoplasm ration

7. The cells are different from where they arose from What is a normal N/C ratio?
A. 1:3
B. 1:8
C. 3:6
D. 1:5

8. IN a malignant tumor the N/C ratio most commonly exhibited is:


A. 1:2
B. 1:1
C. 3:6
D. 1:8

9. What is the process called by which cells move from one site to another?
A. Transportation
B. Biotransformation
C. Metastasis
D. Metrostatic

10. Which of the following is NOT a pathway in which malignant cells spread (metastasize)?
A. Lymph
B. Saliva
C. Blood
D.

11. What is an example of lymphatic metastasis?


A. Renal cell carcinoma
B. Adrenal adenoma
C. Breast cancer
D. Leiyoma

12. What is an example of metastasis occurring as a direct extension of the primary tumor?
A. Breast cancer developing over a course of 3 months
B. Leukemia developing at a very young age
C. Renal cell carcinoma spreading to the adrenal gland
D. HIV developing into AIDs

13. Is a metastatic malignant melanoma of the vertebra considered to be malignant or benign?


A. Malignant
B. Benign
C. Choristoma-like
D. Tumor-like

14. What is a metastatic adenocarcinoma of the stomach that specifically goes to the ovary
called? (be specific).
A. Metastatic adenocarcinoma
B. Melanoma
C. Krukenberg tumor
D. Wilson's tumor

15. What would you be worried about if a 45-year-old woman comes into your ED with
massive weight gain (fluid) over a short period of time?
A. Metastatic ovarian carcinoma
B. Metastatic vaginal carcinoma
C. Cervical cancer
D. Cholecistitis

16. If a tumor is benign and of squamous origin, what would it be called?


A. Malignant
B. Adenoma
C. Papilloma
D. Carcinoma

17. If a tumor is benign and glandular in origin, what is it called?


A. Malignant
B. Adenoma
C. Carcinoma
D. Papilloma

18. What is it called when the nucleus are pushed off to one side due to abundant mucin?
A. Hypertrophy
B. Benign
C. Krukenburgs sign
D. Signet-rings

19. Who does the grading of a tumor?


A. Physician
B. Pharmacist
C. Pathologist
D. Oncologist

20. What grade is a moderately differentiated tumor?


A. A
B. C
C. I
D. II
21. Who does the staging of the tumor?
A. Pathologist
B. Physician
C. Oncologist
D. Dr. Fischione

22. What does TNM stand for?


A. Tumor size, lymph node, malignancy
B. Tumor size, leimyoma, malignancy
C. Tumor shape, lymph node, metastasis
D. Tumor size, lymph node, metastasis

23. A 40-year-old man has a positive stool guaiac test during a routine physical examination.
A colonoscopy is performed and a 0.9-cm, circumscribed, pedunculated mass on a short stalk
is found in the upper rectum. Which of the following terms best describes this lesion?
A. Adenoma
B. Carcinoma
C. Choristoma
D. Hamartoma

24. A Pap smear obtained from a 29-year-old woman during a routine health maintenance
examination is abnormal. She is currently asymptomatic. She has a history of multiple sexual
partners. Cervical biopsy specimens are obtained and the microscopic appearance is shown in
the figure. Which of the following is the most likely diagnosis?
A. Adenocarcinoma
B. Carcinoma in situ
C. Dysplasia
D. Squamous cell carcinoma

25. A 69-year-old woman has experienced increasing malaise and a 10-kg weight loss over
the past year. She dies of massive pulmonary thromboembolism. The gross appearance of the
liver at autopsy is shown in the figure. Which of the following best describes the lesions seen
in her liver?
A. Invasive angiosarcoma
B. Hepatocellular carcinoma
C. Leukemic infiltration
D. Metastatic adenocarcinoma

26. A 66-year-old man with chronic cough has an episode of hemoptysis. On physical
examination, there are no abnormal findings. A chest radiograph shows a 6-cm mass in the
right lung. A sputum cytologic analysis shows neoplastic squamous cells. Metastases from
his lung lesion are most likely to be found at which of the following sites?
A. Cerebral hemisphere
B. Chest wall muscle
C. Hilar lymph nodes
D. Splenic red pulp
27. An epidemiologic study of cancer deaths recorded in the last half of the 20th century is
conducted. The number of deaths for one particular type of cancer had been decreasing in
developed nations, despite the absence of widespread screening and prevention programs.
Which of the following neoplasms was most likely to be identified by this study?
A. Cerebral glioma
B. Gastric adenocarcinoma
C. Hepatic angiosarcoma
D. Leukemia

28. An epidemiologic study of cancer deaths recorded in the last half of the 20th century is
conducted. The number of deaths for one particular cancer had increased markedly in
developed nations. More than 30% of cancer deaths in men, and more than 24% of cancer
deaths in women, were caused by this neoplasm in 1998. In some nations, prevention strate-
gies reduced deaths from this cancer. Which of the following neoplasms was most likely
identified by this study?
A. Cerebral glioma
B. Bronchogenic carcinoma
C. Hepatocellular carcinoma
D. Colonic adenocarcinoma

29. An epidemiologic study analyzes health care benefits of cancer screening techniques
applied to persons more than 50 years of age. Which of the following diagnostic screening
techniques used in health care is most likely to have the greatest impact on reduction in
cancer deaths in Europe and North America?
A. Chest radiograph
B. Mammography
C. Pap smear
D. Serum tumor markers

30. A 34-year-old sexually active woman undergoes a routine physical examination. There
are no abnormal findings. A Pap smear is obtained as part of the pelvic examination. Cyto-
logically, the cells obtained on the smear from the cervix show severe epithelial dysplasia
(high-grade squamous intraepithelial lesion). Which of the following therapeutic options is
most appropriate for this woman?
A. Antibiotic therapy
B. Excision
C. Ovarian removal
D. Screening of family members

31. A 70-year-old woman reported a 4-month history of a 4-kg weight loss and increasing
generalized icterus. On physical examination, she has midepigastric tenderness on palpation.
An abdominal CT scan shows a 5-cm mass in the head of the pancreas. Fine-needle aspiration
of the mass is performed. On biochemical analysis, the neoplastic cells show continued
activation of cytoplasmic kinases. Which of the following genes is most likely to be involved
in this process?
A. APC
B. MYC
C. p53
D. RAS

32. A 22-year-old man has a raised, pigmented lesion on his forearm that has increased in
size and become more irregular in color over the past 4 months. Physical examination shows
a 0.5 × 1.2 cm black-to-brown asymmetric lesion with irregular borders. An excisional
biopsy specimen shows clusters of pleomorphic pigmented cells that extend into the reticular
dermis. Family history indicates that the patient’s maternal uncle died from a similar tumor.
His grandfather required enucleation of the left eye because of a “dark brown” retinal mass.
Which of the following genes is most likely to have undergone mutation to produce these
findings in this family?
A. BCL2 (anti-apoptosis gene)
B. c-MYC (transcription factor gene)
C. IL2 (growth factor gene)
D. p16 (cell cycle inhibition)

33. A 3-year-old child has exhibited difficulty with vision in her right eye. On physical
examination, there is leukocoria of the right eye, consistent with a mass in the posterior
chamber. MR imaging shows a mass that nearly fills the globe. The child undergoes
enucleation of the right eye. Molecular analysis of the neoplastic cells indicates absence of
both copies of a gene that contributes to control of the cell cycle. Which of the following
genes has most likely undergone mutation in this neoplasm?
A. BCR-ABL
B. BCL2
C. hMSH2
D. RB

34. A 76-year-old man has experienced abdominal pain for the past year. On physical
examination, there is an epigastric mass. An abdominal CT scan shows a 10-cm mass in the
body of the pancreas. A fine-needle biopsy specimen of this mass shows a moderately
differentiated adenocarcinoma. Mutational analysis of the carcinoma cells shows inactivation
of cyclin-dependent kinase inhibitor with loss of growth-suppression. Regulatory pathways
controlled by which of the following genes are most likely altered in this man’s carcinoma?
A. BCL2
B. β-Catenin
C. MYC
D. TGF-β

35. A 55-year-old man has had hemoptysis and worsening cough for the past month. On
physical examination, wheezes are auscultated over the right lung posteriorly. A chest
radiograph shows a 6-cm right perihilar mass. A fine-needle aspiration biopsy is performed
and yields cells with the microscopic appearance of non–small cell bronchogenic carcinoma.
Molecular analysis of the neoplastic cells shows a p53 gene mutation. Which of the following
mechanisms has most likely produced the neoplastic transformation?
A. Inability to hydrolyze GTP
B. Growth factor receptor activation
C. Loss of cell cycle arrest
D. Microsatellite instability

36. A 26-year-old man with a family history of colon carcinoma undergoes a surveillance
colonoscopy. It reveals hundreds of polyps in the colon, and two focal 0.5-cm ulcerated areas.
A biopsy specimen from an ulcer reveals irregularly shaped glands that have penetrated into
the muscular layer. Which of the following molecular events is believed to occur very early
in the evolution of his colonic disease process?
A. Activation of the WNT signaling pathway
B. Inability to hydrolyze GTP-bound RAS
C. Loss of heterozygosity affecting the p53 gene
D. Mutations in mismatch repair genes.

37. A 63-year-old man has a cough with hemoptysis for 10 days. He has a 65 pack-year
history of smoking. A chest CT scan shows a 5-cm right hilar mass. Bronchoscopy is per-
formed, and lung biopsy specimens show small cell anaplastic lung carcinoma. His family
history shows three first-degree maternal relatives who developed leukemia, sarcoma, and
carcinoma before age 40 years. Which of the following gene products is most likely to have
been altered by mutation to produce these findings?
A. APC (tumor suppressor)
B. BCL2 (anti-apoptosis)
C. K-RAS (GTP binding)
D. p53 (DNA damage response)

38. A 30-year-old man has a 15-year history of increasing numbers of benign skin nodules.
On physical examination, the firm, nontender, subcutaneous nodules average 0.5 to 1 cm.
Further examination shows numerous oval 1- to 5-cm flat, light brown skin macules.
Ophthalmoscopic examination shows hamartomatous nodules on the iris. A biopsy specimen
of one skin nodule shows that it is attached to a peripheral nerve. Which of the following
molecular abnormalities is most likely related to his clinical presentation?
A. Decreased susceptibility to apoptosis
B. Impaired functioning of mismatch repair
C. Increased production of epidermal growth factor
D. Persistent activation of the RAS gene

39. A 53-year-old man diagnosed with oral cancer and treated with radiation and
chemotherapy 1 year ago now has a positron emission tomography (PET) scan of his neck
that shows a single focus of increased uptake. This focus is resected and microscopic
examination shows that it is a metastasis. Molecular analysis of this cancer shows p53,
PTEN, and c-MYC gene mutations. Which of the following metabolic pathways is most likely
up-regulated to promote his cancer cell survival and proliferation?
A. Aerobic glycolysis
B. Gluconeogenesis
C. Hexose monophosphate shunt
D. Oxidative phosphorylation

40. An experiment involving carcinoma cells grown in culture studies the antitumor
surveillance effects of the innate immune system. These carcinoma cells fail to express MHC
class I antigens. It is observed, however, that carcinoma cells are lysed when an immune cell
that has been activated by IL-2 is added to the culture. Which of the following immune cells
is most likely to function in this manner?
A. CD4+ lymphocyte
B. CD8+ lymphocyte
C. Macrophage
D. NK cell

41. A 33-year-old man has experienced occasional headaches for the past 3 months. He
suddenly has a generalized seizure. CT scan of the head shows a periventricular 3-cm mass in
the region of the right thalamus. A stereotactic biopsy of the mass yields large lymphoid cells
positive for B cell markers. Which of the following underlying diseases is most likely to be
found in this patient?
A. Diabetes mellitus
B. HIV infection
C. Hypertension
D. Multiple sclerosis

42. A 40-year-old man has a history of intravenous drug use. Physical examination shows
needle tracks in his left antecubital fossa. He has mild scleral icterus. Serologic studies for
HBsAg and anti-HCV are positive. He develops hepatocellular carcinoma 15 years later.
Which of the following viral characteristics best explains why this patient developed hepa-
tocellular carcinoma?
A. Viral integration in the vicinity of proto-oncogenes
B. Viral capture of proto-oncogenes from host cellular DNA
C. Viral inflammatory changes with genomic damage
D. Viral inactivation of RB and p53 gene expression

43. A 61-year-old man with a history of chronic viral hepatitis has noted a 6-kg weight loss
over the past 5 months. Physical examination shows no masses or palpable lymphadenopathy.
An abdominal CT scan shows a nodular liver with a 10-cm mass in the right lobe. A stool
guaiac test result is negative. An elevation in which of the following laboratory tests is most
likely to be present in this man?
A. Alpha-fetoprotein
B. CA-19-9
C. Calcitonin
D. Carcinoembryonic antigen

44. A 59-year-old man has noticed blood in his urine for the past week. Cystoscopy shows a
4-cm exophytic mass involving the right bladder mucosa near the trigone. After biopsy
specimens are obtained, he undergoes a radical cystectomy. Examination of the excised
specimen shows an anaplastic carcinoma that has infiltrated the bladder wall. Which of the
following techniques applied to the cells from his neoplasm is most likely to categorize the
cell of origin?
A. Chromosomal karyotyping
B. Cytologic smear
C. DNA microarray
D. Immunohistochemistry
45. A 69-year-old man has noted a chronic cough for the past 3 months. On physical
examination, there is mild stridor on inspiration over the right lung. A chest radiograph shows
a 5-cm right hilar lung mass, and a fine-needle aspiration biopsy specimen of the mass shows
cells consistent with squamous cell carcinoma. If staging of this neoplasm is denoted as
T2N1M1, which of the following findings is most likely in this man?
A. Brain metastases
B. Elevated corticotropin level
C. Infiltration of the chest wall
D. Obstruction of a mainstem bronchus

46. A 44-year-old woman notes a lump in her left breast while taking a shower. The nurse
practitioner palpates a 3 cm firm, irregular, non-movable mass in the upper outer quadrant of
her left breast on physical examination. A fine needle aspiration of this mass is performed,
and cytologically the cells are consistent with infiltrating ductal carcinoma. The mass is
removed with lumpectomy along with an axillary lymph node dissection. Which of the
following findings will best predict a better prognosis for this patient?
A. Tumor cells strongly estrogen receptor positive
B. No metastases in the sampled lymph nodes
C. Flow cytometric analysis with aneuploidy and a high S-phase
D. One relative who had a similar type of breast cancer

47. A change in bowel habits prompts a 53-year-old woman to see her physician. On physical
examination there are no lesions noted on digital rectal examination, but her stool is positive
for occult blood. A colonoscopy is performed and reveals a 6 cm friable exophytyic mass in
the cecum. A biopsy of this mass is performed and microscopic examination shows a
moderately differentiated adenocarcinoma. Which of the following laboratory findings is
most likely to be present in this patient?
A. K-RAS mutation in the neoplastic cells
B. Neoplastic cells positive for vimentin
C. Stool culture with Shigella flexneri
D. Presence of HIV-1 RNA

48. An experiment is conducted in which proliferating cells are subjected to ionizing


radiation. The ionizing radiation leads to arrest in a checkpoint that monitors completion of
DNA replication. It is observed that there are increased numbers of chromosomal
abnormalities in these cells. Which of the following is the checkpoint affected by the ionizing
radiation?
A. G0/G1
B. G1/S
C. S/G2
D. G2/M

49. A clinical study is performed to determine the incidence of cancers in different countries.
The data show that persons born in Japan and continuing to reside there have an increased
risk for cancer. Which of the following cancers is most likely seen with increased frequency
in this population?
A. Breast
B. Colon
C. Lung
D. Stomach

50. A 48-year-old woman has a routine physical examination. A 4 cm diameter non-tender


mass is palpated in her right breast. The mass appears fixed to the chest wall. Another 2 cm
non-tender mass is palpable in the left axilla. A chest radiograph reveals multiple 0.5 to 2 cm
nodules in both lungs. Which of the following TNM classifications best indicates the stage of
her disease?
A. T1 N1 M0
B. T1 N0 M1
C. T2 N1 M0
D. T4 N1 M1

51. A study is performed to analyze characteristics of malignant neoplasms in biopsy


specimens. The biopsies were performed on patients who had palpable mass lesions on digital
rectal examination. Of the following microscopic findings, which is most likely to indicate
that the neoplasm is malignant?
A. Pleomorphism
B. Atypia
C. Invasion
D. Increased nuclear/cytoplasmic ratio

52. A child is born with a single functional allele of a tumor suppressor gene. At the age of
five the remaining normal allele is lost through a point mutation. As a result, the ability to
inhibit cell cycle progression until the cell is ready to divide is lost. Which of the following
neoplasms is most likely to arise via this mechanism?
A. Breast ductal carcinoma
B. Pulmonary small cell anaplastic carcinoma
C. Ocular retinoblastoma
D. Cerebral astrocytoma

53. A 50-year-old man has felt vague abdominal discomfort for the past 4 months. On
physical examination he has no lymphadenopathy, and no abdominal masses or
organomegaly can be palpated. Bowel sounds are present. An abdominal CT scan shows a 20
cm retroperitoneal soft tissue mass obscuring the left psoas muscle. A stool specimen tested
for occult blood is negative. Which of the following neoplasms is this man most likely to
have?
A. Melanoma
B. Hamartoma
C. Adenocarcinoma
D. Liposarcoma

54. A clinical study is performed of oncogenesis in human neoplasms. It is observed that


some neoplasms appear to develop from viral oncogenesis, with serologic confirmation of
past viral infection. Which of the following neoplasms is most likely to arise in this manner?
A. Retinoblastoma
B. Small cell anaplastic carcinoma
C. T-cell leukemia
D. Prostatic adenocarcinoma

55. An experiment is designed to study the genetics of cancer. The study will link the
appearance of cancer to specific gene abnormalities. Which of the following forms of cell
molecular analysis is most useful to identify gene alterations involved in carcinogenesis?
A. Florescence in situ hybridization
B. Flow cytometry
C. Immunohistochemistry
D. Single nucleotide polymorphisms

56. A 14-year-old healthy girl has a 0.3 cm reddish, slightly raised nodule on the skin of the
upper part of her chest found on a routine physical examination. She states that this lesion has
been present for years and has not appreciably changed in size or color. Which of the
following neoplasms is this nodule most likely to be?
A. Hemangioma
B. Melanoma
C. Carcinoma
D. Lymphoma

57. A 60-year-old man who has a 90 pack year history of cigarette smoking has had a chronic
cough for the past 10 years. He has begun to lose weight (3 kg) during the past year. No
abnormal findings are noted on physical examination. He has a chest radiograph that reveals
a right hilar mass. A sputum cytology shows atypical, hyperchromatic squamous cells. What
is the most common initial pathway for metastases from this lesion?
A. Bloodstream
B. Pleural cavity
C. Contiguous spread to chest wall
D. Lymphatics

58. A 55-year-old man has had malaise and a 4 kg weight loss over the past 6 months. On
physical examination his stool is positive for occult blood. An abdominal CT scan shows his
liver contains multiple tumor masses from 2 to 5 cm in size with central necrosis. The
surrounding hepatic parenchyma appears normal. Which of the following characteristics of
neoplasia is best illustrated by these findings?

A. Multicentric origin
B. High tumor grade
C. Primary neoplasm in the stomach
D. Advanced stage
59. A 59-year-old man has had a worsening cough with chest pain for the past 6 months. On
physical examination he has no remarkable findings. A chest x-ray shows a 3 cm left lung
mass. A sputum cytology specimen yields cells diagnosed as a squamous cell carcinoma. A
mediastinoscopy is performed and reveals metastases in a lymph node. He is given radiation
therapy, and the mass diminishes in size. Which of the following cellular mechanisms is most
likely to account for this tumor response?
A. Point mutations in DNA
B. Generation of free radicals
C. Loss of the blood supply
D. Secondary inflammation

60. A 61-year-old woman has a firm mass with irregular borders felt in her left breast on a
routine physical examination. A fine needle aspiration is performed and microscopic
examination shows malignant cells. A left mastectomy with axillary lymph node dissection is
performed. A tissue sample of this neoplasm is submitted for analysis by flow cytometry.
Which of the following does flow cytometric analysis most likely provide?
A. Analysis of the karyotype
B. Detection of gene mutations
C. Determination of aneuploidy
D. Distinguishing carcinoma from sarcoma

61. A 35-year-old healthy woman had a firm nodule palpable on the dome of the uterus six
years ago recorded on routine physical examination. The nodule has slowly increased in size
and now appears to be about twice the size it was when first discovered. By ultrasound scan it
is solid and circumscribed. She remains asymptomatic. Which of the following neoplasms is
she most likely to have?

A. Adenocarcinoma
B. Leiomyosarcoma
C. Hemangioma
D. Leiomyoma
62. A 27-year-old woman in excellent health has a routine health maintenance examination.
A 2 cm firm, rounded mass is palpable beneath the skin of the left forearm. She has no
difficulty using the arm and there is no associated pain with the mass, either in movement or
on palpation. The overlying skin appears normal. The mass does not change in size over the
next year. Which of the following neoplasms is she most likely to have?
A. Metastatic carcinoma
B. Melanoma
C. Rhabdomyosarcoma
D. Lipoma

63. A 45-year-old woman has noted a lump on her left shoulder that has enlarged over the
past 4 months. On physical examination there is a palpable non-tender supraclavicular lymph
node. A biopsy of the node is performed and on microscopic examination there is a metastatic
neoplasm. Which of the following is the most likely primary for this neoplasm?
A. Cerebral glioma
B. Adenocarcinoma of the stomach
C. Fibroadenoma of the breast
D. Liposarcoma of the retroperitoneum

64. A 52-year-old woman feels a lump in her right breast. On physical examination there is a
3 cm right breast mass fixed to the chest wall. This mass is biopsied and on microscopic
examination shows nests of cells with marked hyperchromatism and pleomorphism. These
cells are estrogen receptor positive. Flow cytometry is performed. Compared with
surrounding non-neoplastic stromal cells, the neoplastic cells are more likely to be in which
of the following phases of the cell cycle?
A. G0
B. G1
C. S
D. G2

65. In an experiment, it is observed that chronic, increased exposure to ionizing radiation


results in damage to cellular DNA. As a consequence, a protein is now absent that would
arrest the cell in the G1 phase of the cell cycle. Subsequent to this, the cell is transformed to
acquire the property of unregulated growth. The absent protein is most likely the product of
which of the following genes?
A. RAS
B. TP53
C. MYC
D. ABL

66. An epidemiologic study is performed to find risk factors for development of malignant
neoplasms. A statistical analysis of pre-existing medical conditions is done. Some pre-
existing chronic medical conditions are observed to precede development of malignant
neoplasms, while others do not. Which of the following conditions is most likely to be
statistically related to development of a malignancy?
A. Essential hypertension
B. Coronary artery disease
C. Chronic bronchitis
D. Ulcerative colitis

67. A 35-year-old man has noted several 1 to 2 cm reddish purple, nodular lesions present on
the skin of his right arm which have increased in size and number over the past 3 months.
The lesions do not itch and are not painful. He has had a watery diarrhea for the past month.
On physical examination he has generalized lymphadenopathy and oral thrush. Which of the
following infections is most likely to be related to the appearance of these skin lesions?

A. Candida albicans
B. Human herpesvirus 8
C. Mycobacterium tuberculosis
D. Pseudomonas aeruginosa

68. A 44-year-old woman who has had multiple sexual partners for the past 30 years has an
abnormal Pap smear with cytologic changes suggesting human papillomavirus (HPV)
infection. Without treatment, she is most likely to develop which of the following lesions?
A. Squamous cell carcinoma
B. Non-Hodgkin's lymphoma
C. Kaposi sarcoma
D. Adenocarcinoma

69. A healthy 22-year-old woman undergoes a routine physical examination. A discrete, firm,
rubbery, movable mass is found in the left breast. She has no axillary lymphadenopathy. The
skin overlying the breast and the nipple appear normal. Which of the following neoplasms is
most likely to be present in this woman?
A. Lipoma
B. Intraductal carcinoma
C. Malignant lymphoma
D. Fibroadenoma

70. An epidemiologic study is performed involving patients of East Asian ancestry with long-
standing Epstein-Barr virus (EBV) infection. It is observed that these patients have an
increased risk for development of malignant neoplasms in adulthood. Which of the following
neoplasms is most likely to be found in these patients?
A. Kaposi sarcoma of skin
B. Small cell anaplastic carcinoma of lung
C. Osteosarcoma of bone
D. Nasopharyngeal carcinoma

71. An experiment is conducted in which proliferating cells are subjected to ionizing


radiation. The ionizing radiation leads to arrest in a checkpoint that monitors completion of
DNA replication. It is observed that there are increased numbers of chromosomal
abnormalities in these cells. Which of the following is the checkpoint affected by the ionizing
radiation?
A. G0/G1
B. G1/S
C. S/G2
D. G2/M

72. A clinical study is performed to determine the incidence of cancers in different countries.
The data show that persons born in Japan and continuing to reside there have an increased
risk for cancer. Which of the following cancers is most likely seen with increased frequency
in this population?
A. Breast
B. Colon
C. Lung
D. Stomach

73. A 25-year-old man presents 1 week after discovering that his left testicle is twice the
normal size. Physical examination reveals a nontender, testicular mass that cannot be
transilluminated.
Serum levels of alpha-fetoprotein and human chorionic gonadotropin are normal. A
hemiorchiectomy is performed, and histologic examination of the surgical specimen
shows embryonal carcinoma. Compared to normal adult somatic cells, this germ cell
neoplasm would most likely show high levels of expression of which of the following
proteins?
(A) Desmin
(B) Dystrophin
(C) Cytochrome c
(D) Telomerase
74. A 62-year-old woman presents with a breast lump that she discovered 6 days ago. A
breast biopsy shows lobular carcinoma in situ. Compared to normal epithelial cells of the
breast lobule,
these malignant cells would most likely show decreased expression of which of the following
proteins?
(A) Desmin
(B) E-cadherin
(C) Lysyl hydroxylase
(D) P selectin
75. An 80-year-old man complains of lower abdominal pain, increasing weakness, and
fatigue. He has lost 16 lb (7.3 kg) in the past 6 months. The prostate-specifi c antigen test is
elevated (8.5 ng/mL). Rectal examination reveals an enlarged and nodular prostate. A needle
biopsy of the prostate discloses invasive prostatic adenocarcinoma. Histologic grading of this
patient’s carcinoma is based primarily on which of the following criteria?
(A) Capsular involvement
(B) Extent of regional lymph nodes involvement
(C) Pulmonary metastases
(D) Resemblance to normal tissue of origin

76. A 65-year-old man complains of muscle weakness and a dry cough for 4 months. He has
smoked two packs of cigarettes daily for 45 years. A chest X-ray shows a 4-cm central, left
lung mass. Laboratory studies reveal hyperglycemia and hypertension. A transbronchial
biopsy is diagnosed as small cell carcinoma. Metastases to the liver are detected by CT scan.
Which of the following might account for the development of hyperglycemia and
hypertension in this patient?
(A) Adrenal metastases
(B) Paraneoplastic syndrome
(C) Pituitary adenoma
(D) Pituitary metastases

77. Which of the following potent carcinogens was most likely involved in the pathogenesis
of lung cancer in the patient described in Question 8?
(A) Afl atoxin B1
(B) Asbestos
(C) Azo dyes
(D) Polycyclic aromatic hydrocarbons

78. A 33-year-old woman discovers a lump in her left breast on self-examination. Her mother
and sister both had breast cancer. A mammogram demonstrates an ill-defined density in the
outer quadrant of the left breast, with microcalcifications. Needle aspiration reveals the
presence of malignant, ductalepithelial cells. Genetic screening identifi es a mutation
inBRCA1. In addition to cell cycle control, BRCA1 protein promotes which of the following
cellular functions?
(A) Apoptosis
(B) Cell adhesion
(C) DNA repair
(D) Gene transcription

79. A 60-year-old man who worked for 30 years in a chemical factory complains of blood in
his urine. Urine cytology discloses dysplastic cells. A bladder biopsy demonstrates
transitional cell carcinoma. Which of the following carcinogens was most likely involved in
the pathogenesis of bladder cancer in this patient?
(A) Aniline dyes
(B) Arsenic
(C) Benzene
(D) Cisplatinum

80. A 45-year-old man presents with a 9-month history of a reddish nodule on his foot.
Biopsy of the nodule discloses a poorly demarcated lesion composed of fi broblasts and
endothelial-like cells lining vascular spaces. Further work-up identifi es similar lesions in the
lymph nodes and liver. The tumor cells contain sequences of human herpesvirus-8 (HHV-8).
This patient most likely has which of the following diseases?
(A) Acquired immunodefi ciency
(B) Ataxia telangiectasia
(C) Li-Fraumeni syndrome
(D) Neurofi bromatosis type I

81. During a routine checkup, a 50-year-old man is found to have blood in his urine. He is
otherwise in excellent health. An abdominal CT scan reveals a 2-cm right renal mass. You
inform the patient that staging of this tumor is key to selecting treatment and evaluating
prognosis. Which of the following is the most important staging factor for this patient?
(A) Histologic grade of the tumor
(B) Metastases to regional lymph nodes
(C) Proliferative capacity of the tumor cells
(D) Somatic mutations in the p53 tumor suppressor gene

82. A 58-year-old woman with colon cancer presents with 3 months of increasing shortness
of breath. A chest X-ray reveals numerous, bilateral, round masses in both lungs. Histologic
examination of an open-lung biopsy discloses malignant gland-like structures, which are
nearly identical to the colon primary. Which of the following changes in cell behavior was
the fi rst step in the process leading to tumor metastasis from the colon to the lung in this
patient?
(A) Arrest within the circulating blood or lymph
(B) Exit from the circulation into a new tissue
(C) Invasion of the underlying basement membrane
(D) Penetration of vascular or lymphatic channels

83. A 68-year-old man complains of recent changes in bowel


habits and blood-tinged stools. Colonoscopy reveals a 3-cm
mass in the sigmoid colon. Biopsy of the mass shows infi ltrating
malignant glands. These neoplastic cells have most likely
acquired a set of mutations that cause which of the following
changes in cell behavior?
(A) Decreased cellular motility
(B) Enhanced stem cell differentiation
(C) Increased cell-cell adhesion
(D) Loss of cell cycle restriction point control

84. A 35-year-old woman complains of nipple discharge and irregular menses of 5 months
duration. Physical examination reveals a milky discharge from both nipples. MRI shows an
enlargement of the anterior pituitary. Which of the following is the most likely histologic
diagnosis of this patient’s pituitary tumor?
(A) Adenoma
(B) Choristoma
(C) Hamartoma
(D) Papilloma

85. A 52-year-old woman presents with a 1-year history of upper truncal obesity and
moderate depression. Physical examination shows hirsutism and moon facies. A CT scan of
the thorax displays a hilar mass. A transbronchial lung biopsy discloses small cell carcinoma.
Electron microscopy of this patient’s lung tumor will most likely reveal which of the
following cytologic features?
(A) Councilman bodies
(B) Hyperplasia of endoplasmic reticulum
(C) Mitochondrial calcifi cation
(D) Neuroendocrine granules
86. Cytogenetic studies in a 40-year-old woman with follicular lymphoma demonstrate a
t(14;18) chromosomal translocation involving the bcl-2 gene. Constitutive expression of the
protein
encoded by the bcl-2 gene inhibits which of the following processes in this patient’s
transformed lymphocytes?
(A) Apoptosis
(B) DNA excision repair
(C) G1-to-S cell cycle progression
(D) Oxidative phosphorylation

87. A 59-year-old woman presents with increasing pigmentation of the skin. Physical
examination shows hyperkeratosis and hyperpigmentation of the axilla, neck, fl exures, and
anogenital region. Endocrinologic studies reveal normal serum levels of adrenal
corticosteroids and glucocorticoids. If this patient’s skin pigmentation represents a
paraneoplastic syndrome, the
primary tumor would most likely be found in which of the following anatomic locations?
(A) Bladder
(B) Cervix
(C) Esophagus
(D) Stomach

88. A 65-year-old man dies after a protracted battle with metastatic colon carcinoma. At
autopsy, the liver is fi lled with multiple nodules of cancer, many of which display central
necrosis (umbilication). Which of the following best explains the pathogenesis of tumor
umbilication in this patient?
(A) Biphasic tumor
(B) Chronic infl ammation
(C) Granulomatous infl ammation
(D) Ischemia and infarction

89. A 59-year-old man complains of progressive weakness. He reports that his stools are very
dark. Physical examination demonstrates fullness in the right lower quadrant. Laboratory
studies show iron defi ciency anemia, with a serum hemoglobin level of 7.4 g/dL. Stool
specimens are positive for occult blood. Colonoscopy discloses an ulcerating lesion of the
cecum. Which of the following serum tumor markers is most likely to be useful for following
this patient after surgery?
(A) Alpha-fetoprotein
(B) Carcinoembryonic antigen
(C) Chorionic gonadotropin
(D) Chromogranin

90. A 20-year-old woman has an ovarian tumor removed. The surgical specimen is 10 cm in
diameter and cystic. The cystic cavity is found to contain black hair and sebaceous material.
Histologic examination of the cyst wall reveals a variety of benign differentiated tissues,
including skin, cartilage, brain, and mucinous glandular epithelium. What is the diagnosis?
(A) Adenoma
(B) Chondroma
(C) Hamartoma
(D) Teratoma

91. A 42-year-old man presents with upper gastrointestinal bleeding. Upper endoscopy and
biopsy reveal gastric adenocarcinoma. Which country of the world has the highest incidence
of this malignant neoplasm?
(A) Argentina
(B) Canada
(C) Japan
(D) Mexico

92. An 8-year-old girl with numerous hypopigmented, ulcerated, and crusted patches on her
face and forearms develops an indurated, crater-like, skin nodule on the back of her left hand.
Biopsy of this skin nodule discloses a squamous cell carcinoma. Molecular biology studies
reveal that this patient has germline mutations in the gene encoding a nucleotide excision
repair enzyme. What is the appropriate diagnosis?
(A) Ataxia telangiectasia
(B) Hereditary albinism
(C) Li-Fraumeni syndrome
(D) Xeroderma pigmentosum
93. A 59-year-old woman complains of “feeling light-headed” and losing 5 kg (11 lb) in the
last month. A CBC reveals a normocytic, normochromic anemia. The patient subsequently
dies of metastatic cancer. Based on current epidemiologic data for cancer-associated
mortality in women, which of the following is the most likely primary site for this patient’s
malignant neoplasm?
(A) Brain
(B) Breast
(C) Colon
(D) Lung

94. The parents of a 6-month-old girl palpate a mass on the left side of the child’s abdomen.
Urinalysis shows high levels of vanillylmandelic acid. A CT scan reveals an abdominal
tumor and bony metastases. The primary tumor is surgically resected. Histologic examination
of the surgical specimen discloses neuroblastoma. Evaluation of the N-myc protooncogene in
this child’s tumor will most likely demonstrate which of the following genetic changes?
(A) Chromosomal translocation
(B) Exon deletion
(C) Expansion of a trinucleotide repeat
(D) Gene amplifi cation

95. A 58-year-old woman undergoes routine colonoscopy. A 2-cm submucosal nodule is


identifi ed in the appendix. Biopsy of the nodule shows nests of cells with round, uniform
nuclei. Electron microscopy reveals numerous neuroendocrine granules in the cytoplasm.
This patient’s neoplastic disease is associated with which of the following clinical features?
(A) Congestive heart failure
(B) Flushing and wheezing
(C) Muscular dystrophy
(D) Progressive systemic sclerosis

96. A 45-year-old woman presents with abdominal pain and vaginal bleeding. A
hysterectomy is performed and shows a benign tumor of the uterus derived from a smooth
muscle cell. What is the appropriate diagnosis?
(A) Angiomyolipoma
(B) Leiomyoma
(C) Leiomyosarcoma
(D) Myxoma

97. Cytogenetic studies in a 70-year-old woman with chronic myelogenous leukemia (CML)
demonstrate a t(9;22) chromosomal translocation. Which of the following best explains the
role of this translocation in the pathogenesis of leukemia in this patient?
(A) Altered DNA methylation status
(B) Enhanced expression of telomerase gene
(C) Expansion of a trinucleotide repeat
(D) Protooncogene activation

98. A 33-year-old woman presents with a diffuse scaly skin rash of 4 weeks duration. Biopsy
of lesional skin reveals a cutaneous T-cell lymphoma (mycosis fungoides). Which of the
following immunohistochemical markers would be most useful for identifying malignant
cells in the skin of this patient?
(A) Calcitonin
(B) CD4
(C) Desmin
(D) HMB-45

99. A 63-year-old woman with chronic bronchitis presents with shortness of breath. A chest
X-ray reveals a 2-cm “coin lesion” in the upper lobe of the left lung. A CT-guide lung biopsy
is obtained. Which of the following describes the histologic features of this lesion if the
diagnosis is hamartoma?
(A) Benign neoplasm of epithelial origin
(B) Disorganized normal tissue
(C) Ectopic islands of normal tissue
(D) Granulation tissue

100. A 2-year-old boy is found to have bilateral retinal tumors. Molecular studies
demonstrate a germline mutation in one allele of the Rb gene. Which of the following genetic
events best explains the mechanism of carcinogenesis in this patient?
(A) Balanced translocation
(B) Expansion of trinucleotide repeat
(C) Gene amplifi cation
(D) Loss of heterozygosity

101. A 48-year-old nulliparous woman complains that her menstrual blood fl ow is more
abundant than usual. An ultrasound examination reveals a polypoid mass in the uterine
fundus. The patient subsequently, undergoes a hysterectomy, which reveals a poorly
differentiated endometrial adenocarcinoma. The development of this neoplasm was preceded
by which of the following
histopathologic changes in the glandular epithelium?
(A) Atrophy
(B) Hydropic swelling
(C) Hyperplasia
(D) Hypertrophy

102. Which type of adaptation is able to convert to dysplasia and further to adenocarcinoma?
A. Hypertrophy
B. Atrophy
C. Metaplasia
D. Hamartoma

103. Metaplasia of distal end of esophagus leads to which type of cancer?


A. Adenocarcinoma
B. Sarcoma
C. Hamartoma
D. Choristoma

104. Endometrial hyperplasia may lead to which cancer type?


A. Adenocarcinoma
B. Sarcoma
C. Hamartoma
D. Choristoma
1. A leiyoma of the uterus is considered benign or malignant?

A. Benign

B. Malignant

C. Neither

D. Tumor-like

2. Is a lipoma considered to be malignant or benign?

A. Malignant

B. Benign

C. Tumor-like

D. Hamartoma

3. Is a melanoma considered to be malignant or benign?

A. Malignant

B. Benign

C. Tumor-like

D. Hamartoma

4. Is a lymphoma considered to be malignant or benign?

A. Malignant

B. Benign

C. Tumor-like

D. Hamartoma
5. Is a rhabdomyosarcoma considered to be malignant or benign?

A. Malignant

B. Benign

C. Tumor-like

D. Hamartoma

6. Is a adenoma considered to be malignant or benign?

A. Malignant

B. Benign

C. Tumor-like

D. Hamartoma

7. Is a leukemia considered to be malignant or benign?

A. Malignant

B. Benign

C. Tumor-like

D. Hamartoma

8. Benign tumor of cartilage is called an:

A. Osteoma

B. Chondroma

C. Leiomyoma
D. Lipoma

9. Benign tumor of osseous tissue is called an:

A. Osteoma

B. Chondroma

C. Leiomyoma

D. Lipoma

10. Benign tumor in smooth muscle is called a:

A. Osteoma

B. Leiomyoma

C. Lipoma

D. Chondroma

11. Benign tumor in fat tissue is called a:

A. Osteoma

B. Lipoma

C. Leimyoma

D. Chondroma

12. Malignant tumor in fibrous tissue is called a:

A. Liposarcoma

B. Osteosarcoma

C. Fibrosarcoma
D. Rhabdomyosarcoma

13. Malignant tumor in fat tissue is called a:

A. Fibrosarcoma

B. Liposarcoma

C. Osteosarcoma

D. Rhabdomyosarcoma

14. Malignant tumor in bone tissue is called a:

A. Fibrosarcoma

B. Rhabdomyosarcoma

C. Osteosarcoma

D. Liposarcoma

15. Malignant tumor in skeletal muscle is called a:

A. Fibrosarcoma

B. Liposarcoma

C. Osteosarcoma

D. Rhabdomyosarcoma

16. What is a benign tumor called that is derived from all 3 germ cell layers?

A. Myoma

B. Fibroma
C. Teratoma

D. Lymphoma

17. Most common location of teratoma?

A. Uterus

B. Ovaries

C. Kidney

D. Liver

18. Gross specimen of teratoma may contain?

A. Teeth

B. Hair

C. Cartilage

D. All of the above

19. Which brain structure may be a site for teratoma formation?

A. Right hemispheire

B. Pineal gland

C. Medulla

D. Pons

20. Which of the following is the tumor-like condition?

A. Choristoma
B. Chondroma

C. Hepatoma

D. Melanoma

21. What is the definition of choristoma?

A. Heterotopic rest

B. Bening tumor of ovaries

C. Benign tumor of liver

D. Benign tumor of cartilage

22. Most common malignant bone tumor is?

A. Osteogenic sarcoma

B. Chondrosarcoma

C. Giant cell tumor

D. Synovioma

23. Malignant tumor originating from plasma cells?

A. Multiple myeloma

B. Chondroma

C. Fibroma

D. Lieomyoma
24. Benign tumor is:

A. Noninvasive

B. Invasive

C. Able to spread

D. Metastasize

25. Mixed tumor is found in the following organ:

A. Salivary gland

B. Liver

C. Kidney

D. Parathyroid

26. Seminoma arising from testicles are:

A. Benign

B. Malignant

C. Tumor-like

D. Hamartoma

27. Hamartoma is defined as:

A. Heperotopic rest

B. Tumor originating from indigenous site

C. Malignant tumor

D. Benign tumor
28. Malignant tumor from connective tissues are called:

A. Adenomas

B. Papillomas

C. Carcinomas

D. Sarcomas

29. Malignant tumor from connective tissues commonly located in:

A. Brain

B. Heart

C. Liver

D. Lower extremity

30. Tumor arising from all 3 germ cell layers are called:

A. Choristoma

B. Hamartoma

C. Teratoma

D. Papilloma

31. Tumor containing teeth, hair, cartilage, and adipose tissue are called:

A. Choristoma

B. Hamartoma
C. Teratoma

D. Papilloma

32. Hamartoma refers to:

A. Tumour differentiating towards more than one cell line

B. Tumour arising from totipotent cells

C. Mass of disorganised but mature cells indigenous to the part

D. Mass of ectopic rests of normal tissue

33. Increased number of normal mitoses may be present in the following tissues except:

A. Bone marrow cells

B. Nails

C. Hepatocytes

D. Intestinal epithelium

34. A tumour is termed medullary when it is almost entirely composed of:

A. Amyloid stroma

B. Large areas of necrosis

C. Abundant lymphoid tissue

D. Parenchymal cells

35. All the following malignant tumours metastasise except:

A. Synovial sarcoma
B. Malignant mesothelioma

C. Glioma

D. Neuroblastoma

36. The following malignant tumours frequently spread through haematogenous route except:

A. Bronchogenic carcinoma

B. Renal cell carcinoma

C. Follicular carcinoma thyroid

D. Seminoma testis

37. Degradation of ECM is brought about by the following except:

A. Proteases

B. Metalloproteinases

C. Free radicals

D. Cathepsin D

38. Grading of tumours depends upon the following except:

A. Degree of anaplasia

B. Metastatic spread

C. Rate of growth of cells

D. Degree of differentiation

39. Patients of xeroderma pigmentosum are prone to develop the following cancers except:

A. Basal cell carcinoma


B. Sweat gland carcinoma

C. Malignant melanoma

D. Squamous cell carcinoma

40. The primary target of reactive electrophiles is as under:

A. Cytochrome P-450

B. RNA

C. DNA

D. Mitochondria

41. Carcinogenic influence of radiation appears after:

A. < 2 years

B. 2-5 years

C. 5-10 years

D. > 10 years

42. The following hereditary diseases have higher incidence of cancers due to inherited defect
in DNA repair mechanism except:

A. Ataxia telangiectasia

B. Xeroderma pigmentosum

C. Familial polyposis coli

D. Bloom’s syndrome
43. The following form of ionising radiation exposure is associated with highest risk of
cancer:

A. a-rays

B. b-rays

C. g-rays

D. X-rays

44. Women receiving oestrogen therapy have an increased risk of developing the following
cancers except:

A. Breast cancer

B. Endometrial carcinoma

C. Gallbladder cancer

D. Hepatocellular carcinoma

45. Important cyclins in cell cycle include the following except:

A. Cyclin A

B. Cyclin B

C. Cyclin C

D. Cyclin D

46. Bittner milk factor is a transmissible agent belonging to the following category:

A. Acute transforming virus

B. Slow transforming virus


C. HTLV-I

D. HTLV-II

47. Important examples of tumour suppressor genes implicated in human cancers include the
following except:

A. RB gene

B. TP53

C. APC

D. ERB-B

48. An example of tumour-associated antigen (TAA) is:

A. Testis specific antigen (MAGE)

B. Alpha-fetoprotein (AFP)

C. Carcinoembryonic antigen (CEA)

D. Prostate specific antigen (PSA)

49. Hypercalcaemia as a paraneoplastic syndrome is observed in the following tumours


except:

A. Squamous cell carcinoma lung

B. Small cell carcinoma lung

C. Renal cell carcinoma

D. Breast cancer
50. Lymphocytic infiltrate is frequently present in the following tumours indicative of host
immune response except:

A. Seminoma testis

B. Medullary carcinoma breast

C. Papillary carcinoma thyroid

D. Malignant melanoma

51. The following antibody-stain is used in immunohistochemistry to identify epithelial cells:

A. Desmin

B. Vimentin

C. Cytokeratin

D. Neurofilaments

52. Which of the following viral infection is not known to produce any human tumour?

A. Polyoma virus

B. EBV

C. HSV

D. HTLV

53. All are autosomal dominant inherited cancer syndromes except:

A. Retinoblastoma

B. Xeroderma pigmentosum

C. HNPCC
D. Neurofibromatosis

54. Phosphorylation of retinoblastoma gene:

A. Inhibits cell replication

B. Promotes cellular quiescence

C. Stops cell cycle progression

D. Promotes cell division

55. p53:

A. Activates cyclins

B. Activates BAX

C. Activates CDKs

D. Activates bcl2

56. All are matrix metalloproteinases except:

A. Collagenase

B. Gelatinase

C. Stromelysin

D. Elastase

57. All are anti-angiogenesis factors except:

A. Thrombospondin-1

B. Basic fibroblast growth factor (bFGF)


C. Endostatin

D. Angiostatin

58. Which of the following is a test for mutagenicity?

A. Kveim’s test

B. Ame’s test

C. Schilling’s test

D. Mantoux test

59. All are autosomal dominant inherited cancer syndromes except:

A. Retinoblastoma

B. Xeroderma pigmentosum

C. HNPCC

D. Neurofibromatosis

60. DNA extraction is a pre-requisite for the following molecular techniques except:

A. PCR technique

B. In situ hybridisation

C. Western blot technique

D. Southern blot technique

61. All are methods of cell proliferation analysis except:

A. Microspectrophotometry

B. Flow cytometry
C. PCR

D. Immunohistochemistry

63. A 54-year-old woman who has been diagnosed with early-stage breast cancer undergoes

surgery for a lumpectomy to remove a small tumor detected by mammography. The


pathology report confirms the early stage of the cancer and further comments on the fact that
there is significant desmoplasia in the surrounding tissue. The term desmoplasia refers to

(A) an irregular accumulation of blood vessels.

(B) maturation and spatial arrangement of cells.

(C) metastatic involvement of surrounding tissue.

(D) proliferation of non-neoplastic fibrous connective tissue.

64. A 24-year-old woman with a history of heavy and painful menstrual periods has been
having difficulty conceiving despite months of trying to become pregnant. Further workup
includes a bimanual pelvic examination and an ultrasound, which demonstrates a mass in the
uterus that is presumed to be a leiomyoma. This mass is a

(A) benign tumor of mesenchymal tissue.

(B) benign tumor of surface epithelium.

(C) malignant tumor of epithelial tissue.

(D) malignant tumor of glandular epithelium.

65. A 68-year-old man has a long history of prostate cancer that was metastatic at the time of
diagnosis. Over the past 2 months, he has had significant weight loss, loss of appetite, and
loss of energy. His current spectrum of conditions can be attributed to which of the
following?

(A) Platelet-derived growth factor


(B) Fibroblast growth factor

(C) Interleukin-2

(D) Tumor necrosis factor-α

66. A 58-year-old man with a 700-pack-peryear smoking history presents to the emergency

department with shortness of breath and hemoptysis. Portable chest radiography demonstrates
a large mass centrally located within the left lung field. The serum calcium is 13.0 mg/dL
(normal 8.5 to 10.2). The metabolic abnormality described here is likely due to elaboration of
which substance?

(A) Adrenocorticotropic hormone–like substance

(B) Antidiuretic hormone

(C) Carcinoembryonic antigen

(D) Parathyroid-related hormone

67. An 8-year-old boy is referred to the dermatologist for numerous “suspicious” pigmented
lesions on the face and neck. Further history reveals that the patient has had difficulty seeing
out of his right eye; he is referred to the ophthalmologist, who diagnoses an ocular
melanoma. Based on the patient’s symptoms, the diagnosis of xeroderma pigmentosum is
considered. This condition results from

(A) aberrant expression of a receptor tyrosine kinase.

(B) an inborn defect in DNA repair.

(C) chemical carcinogenesis.

(D) DNA viral infection.


68. A 46-year-old woman with prominent splenomegaly presents with a 3-month history of
malaise, easy fatigability, weakness, weight loss, and anorexia. A complete blood count and
differential demonstrates a white blood cell count of 250,000/mm3 (normal 3,000 to
10,000/mm3) with a predominance of myelocytes, metamyelocytes, band cells, and
segmented neutrophils. Cytogenetic analysis is most likely to reveal which of the following
translocations?

(A) t(8;14)

(B) t(9;22)

(C) t(11;22)

(D) t(14;18)

69. An 18-year-old patient presents with renal cell carcinoma. Given that this is typically a
tumor of older adults, what translocation might you expect to find? This translocation is also
seen in which mesenchymal malignancy?

(A) t(X;18), synovial sarcoma

(B) t(X;17), alveolar soft part sarcoma

(C) t(9;22), leiomyosarcoma

(D) t(14;18), leiomyosarcoma

70. A 63-year-old woman discovers a lump in her right breast. Mammography confirms the
presence of a suspicious “lump,” and a needle core biopsy is performed to determine whether
the mass is malignant. The pathology report confirms that the mass is indeed cancerous and
that the tissue demonstrates amplification of the Her-2/neuoncogene. The gene product of
Her-2/neu is

what kind of protein?

(A) GTPase
(B) GTPase-activating protein

(C) Nuclear transcription factor

(D) Receptor tyrosine kinase

71. A 27-year-old woman has recently been diagnosed with a glioma (a malignant brain
tumor). Further family history reveals that her 4-year-old son has been diagnosed with
leukemia and has been undergoing chemotherapy. In addition, the patient’s mother died at 36
years of age due to metastatic breast cancer. Li-Fraumeni syndrome is suspected, given the
familial clustering of this

group of malignancies. The gene mutated in Li-Fraumeni syndrome normally functions

in what capacity?

(A) Activates the GTPase activity of the gene product of the Ras oncogene

(B) Excises ultraviolet light–induced thymidine dimers

(C) Functions as a cytoplasmic tyrosine kinase

(D) Halts the cell cycle if DNA damage is detected

72. An 8-year-old child is evaluated by the pediatrician, who notes what appear to be 10
small café-au-lait spots on the child’s torso. In addition, on close inspection of the eyes, the
presence of Lisch nodules is noted. The patient is diagnosed with von Recklinghausen
neurofibromatosis type 1. The protein that is mutated in this disorder normally

(A) activates the GTPase activity of Ras.

(B) cleaves cellular proteins during apoptosis.

(C) functions as a regulator of the cell cycle.

(D) promotes angiogenesis in the growing tumor mass.


73. A 78-year-old Navy veteran with a 600-pack-per-year history of cigarette smoking
presents with cancer. During his military career, he was involved in fireproofing naval
combat ships with asbestos insulation. Given his environmental exposure to both tobacco and
asbestos, to which cancer do both of these carcinogens contribute?

(A) Bladder cancer

(B) Bronchogenic cancer

(C) Cancer of the throat

(D) Esophageal cancer

74. A 40-year-old woman presents with endometrial carcinoma. Her family history reveals
that her mother died of endometrial cancer at age 50, while her 42-year-old brother was
recently diagnosed with colon cancer. You begin to suspect a familial cancer syndrome. What
gene is most likely to be mutated in this family?

(A) WT-1

(B) APC

(C) MSH2

(D) p53
1. Teratogens are defined as agents which induce:
A. Mitosis
B. Carcinogenesis
C. Birth defects
D. Fallot’s tetralogy

2. For chromosomal study, it is best to use the following nucleated cells:


A. Polymorphs
B. Lymphocytes
C. Epithelial cells
D. Fibroblasts

3. For chromosomal study, the dividing cells are arrested by colchicine in the following phase
of cell cycle:
A. Prophase
B. Metaphase
C. Anaphase
D. Telophase

4. Denver classification divides chromosomes based on their length into the following
groups:
A. A to C (3 groups)
B. A to E (5 groups)
C. A to G (7 groups)
D. A to I (9 groups)

5. Polyploidy is generally not a feature of dividing cells of the following type:


A. Megakaryocytes
B. Hepatocytes
C. Tubular cells
D. Conceptus of abortions

6. Numeric abnormality in chromosome occurs in the following conditions except:


A. Ph chromosome in CML
B. Turner’s syndrome
C. Klinefelter’s syndrome
D. Down’s syndrome

7. Mutations affecting germ cells produce:


A. Cancers
B. Inherited diseases
C. Congenital malformations
D. Aneuploidy

8. In lysosomal storage diseases, the following cells are particularly involved:


A. Hepatocytes
B. Skeletal muscle
C. Macrophages
D. White pulp of spleen

9. Out of the following glycogenosis, the following is example of


lysosomal storage disease:
A. von Gierke’s disease
B. Pompe’s disease
C. Forbe’s disease
D. Anderson’s disease

10. Blastomas are childhood tumours seen more often in the age
range of:
A. <4 years
B. 5-9 years
C. 10-14 years
D. 14-16 years

11. All of the following are X-linked recessive disorders except:


A. Haemophilia A and B
B. Chronic granulomatous disease
C. G-6 PD deficiency
D. Sickle cell anaemia

12. Gaucher cells are positive for all except:


A. PAS
B. Mucicarmine
C. Oil red O
D. Prussian blue

13. A newborn male infant is examined. There is a palpable right testis, but no left testis, in
the scrotal sac. The infant has no other abnormalities noted. Which of the following is the
most likely abnormality involving the gonads of this infant?
A. Agenesis
B. Fusion
C. Hypopituitarism
D. Incomplete descent

14. Following ovulation and fertilization of an ovum, a blastocyst develops. Growth and
differentiation occurs following implantation. Which of the following will develop into the
placenta?
A. Epimere
B. Neural plate
C. Sclerotome
D. Trophoblast

15. A 5-year-old child fatigues easily. On examination a machinery-like murmur is


auscultated over his chest. Echocardiography shows shunting of blood from the aorta to the
pulmonary artery. An abnormality involving which of the following structures is most likely
present in this boy?
A. Ductus arteriosus
B. Endocardial cushion
C. Sinus venosus
D. Third aortic arch

16. A 3500 gm infant born at term is doing well and gaining weight normally until the fourth
week of life. He then exhibits feeding difficulty with forceful vomiting. Now, on physical
examination, no external anomalies are noted. However, there is a firm mass palpable in the
upper mid abdominal region. Which of the following conditions is the infant most likely to
have?
A. Duodenal atresia
B. Hepatoblastoma
C. Hirschsprung disease
D. Pyloric stenosis

17. An infant born at term is examined, and there is abnormal fusion of the 3rd, 4th, and 5th
digits of the left hand. A radiograph of the left hand shows transformation of metacarpals I,
II, III, and V to short, carpal-like bones. A mutation involving which of the following genes
most likely led to these findings?
A. CFTR
B. FGFR3
C. HOXD13
D. SHH

18. An infant born at term has a radiograph taken. No congenital abnormalities are noted. The
upper heart borders are not visualized. Which of the following structures is most likely to
obscure the heart borders in this infant's radiograph?
A. Esophagus
B. Lymph node
C. Stomach
D. Thymus

19. A secondary oocyte is developing normally. At which of the following times does the
second maturation division of meiosis become completed?
A In the ovarian cortex
B During ovulation
C At fertilization
D In the blastocyst

20. During the 3rd week of embryonic development, splanchnic mesoderm differentiates into
blood islands with angioblasts. In what developing tissue site does this process occur?
A. Heart
B. Liver
C. Spleen
D. Thymus

21. An ovum is fertilized. After 3 weeks a blastocyst implanted on the endometrium has an
inner embryoblast and an outer trophoblast. Which of the following is the next step in
forming a 3-layered embryonic disk?
A. Decidualization
B. Gastrulation
C. Lateralizaton
D. Neurulation

22. A 55-year-old man has noted sharp pain in his right lower extremity for the past 2
months. MR imaging of his spine shows impingement on a spinal nerve root by a herniated
structure located between L5 and S1. From which of the following embryonic derivatives
does this herniated structure most likely arise?

A. Amnion
B. Cloaca
C. Lateral plate mesoderm
D. Neural crest

23. In the developing embryo, pharyngeal arches are formed. The second (branchial) arch
gives rise to the stapedius, orbicularis oris, and posterior belly of the digastric muscles.
Which of the following cranial nerves is also derived from this arch?
A. III
B. V
C. VII
D. IX
24. In the normally developing embryo, aortic arches are apparent by the end of the fourth
week. Which of the following arteries is derived from the third pair of aortic arches?
A. Carotid
B. Pulmonary
C. Stapedial
D. Subclavian

25. A fetus at 18 weeks gestation is normally formed and appropriately sized. No fetal or
placental anomalies are present. Which of the following parts of the fetal vasculature is most
likely to have the greatest oxygen concentration?
A. Aorta
B. Ductus arteriosus
C. Pulmonary vein
D. Umbilical vein

26. An embryonic eye is developing normally. A retina is present on the posterior aspect of
the globe. What is the embryologic origin of the portion of retina containing nerve cell
bodies?
A. Diencephalon
B. Mesoderm
C. Notochord
D. Pharyngeal arch

27. A newborn examination is performed. The tympanic membranes are visualized


bilaterally. What is the embryologic origin of the structure that forms the canal connecting
the external ear to the tympanic membrane?

A. Bony labyrinth
B. First pharyngeal cleft
C. Endolymphatic duct
D. Otic pit

28. A 20-year-old woman is unable to conceive and undergoes an infertility workup. A pelvic
ultrasound reveals a bicornuate uterus. Which of the following is the most likely explanation
for her anomalous uterus?
A. Excess circulating maternal androgens while in utero
B. Failure of primordial germ cell migration
C. Failure of the paramesonephric ducts to fuse
D. Failure of the urorectal septum to reach the perineum
29. In a developing embryo, neural crest cells form adjacent to the neural tube. These neural
crest cells then undergo migration. Which of the following structures is derived from these
migrating cells?
A. Dorsal root ganglion
B. Dura mater
C. Epidermis covering the back
D. Intervertebral disk

30. A neonate undergoes a check of health status. On examination a faint cardiac murmur is
auscultated. Echocardiography is performed and shunting of blood is noted from the left
atrium to the right atrium. An abnormality in embryonic development of which of the
following structures has most likely occurred in this neonate?
A. Ductus arteriosus
B. Endocardial cushion
C. Fourth aortic arch
D. Sinus venosus

31. The two superior and two inferior parathyroid glands are found adjacent to the posterior
aspect of the thyroid gland. From which of the following are these parathyroid glands derived
in embryologic development?
A. Foramen cecum
B. Mandibular swellings
C. Maxillary processes
D. Pharyngeal pouches

32. A 31-year-old woman has noted no fetal movement by 16 weeks gestation. A fetal
ultrasound shows the fetal head contains a brain with thin cortex, single enlarged ventricle,
and fused thalami. Which of the following developing regions of the brain is most likely
abnormal in this fetus?
A. Cranial vault
B. Mesencephalon
C. Prosencephalon
D. Rhombencephalon

33. Following fertilization of the ovum, the ovarian corpus luteum continues to produce
progesterone. If the corpus luteum regresses in the first trimester, abortion occurs. Which of
the following hormones maintains this capability of the corpus luteum?
A. Alpha-fetoprotein
B. Estradiol
C. Human chorionic gonadotropin
D. Human placental lactogen
34. Following coitus, spermatozoa migrate to the fallopian tubes. Within the tubes, the
acrosomal head of spermatozoa undergoes reduction of glycoprotein coating with increasing
permeability to calcium. Which of the following functions of spermatozoa is facilitated by
this process?
A. Blocking other sperm
B. Fertilization
C. Motility
D. Mitosis

35. A 24-year-old primigravida has felt no fetal movement at 16 weeks gestation. A screening
fetal ultrasound shows anhydramnios and a single poorly formed lower extremity. An
abnormality involving which of the following structures most likely led to these findings?
A . Aorta
B. Dermis
C. Neural crest
D Splanchnic mesoderm

36. A neonate born at term after an uncomplicated pregnancy is noted to pass pale coloured
stool and dark urine. The infant becomes progressively jaundiced in the next 2 days. On
examination the infant is at the 50th percentile for weight. Icterus is present. The serum IgM
antibody titer to cytomegalovirus is increased. Which of the following developmental
abnormalities most likely occurred in utero?
A. Accessory pancreas
B. Biliary atresia
C. Gallbladder duplication
D. Hepatic agenesis

37. Following birth, circulation from placenta to fetus ceases. Which of the following fetal
structures that conducts blood to the inferior vena cava becomes atretic in postnatal life?
A. Azygous vein
B. Ductus arteriosus
C. Ductus venosus
D. Foramen ovale

38. A newborn male infant is examined. There is a palpable right testis, but no left testis, in
the scrotal sac. The infant has no other abnormalities noted. Which of the following is the
most likely abnormality involving the gonads of this infant?
A. Agenesis
B. Fusion
C. Hypopituitarism
D. Incomplete descent
39. How soon after fertilization occurs within the uterine tube does the blastocyst begin
implantation?
(A) Within minutes
(B) By 12 hours
(C) By day 1
(D) By day 7

40. Where does the blastocyst normally implant?


(A) Functional layer of the cervix
(B) Functional layer of the endometrium
(C) Basal layer of the endometrium
(D) Myometrium

41. Which of the following events is involved in the cleavage of the zygote during week 1 of
development?
(A) A series of meiotic divisions forming blastomeres
(B) Production of highly differentiated blastomeres
(C) An increased cytoplasmic content of blastomeres
(D) A decrease in size of blastomeres

42. Which of the following structures must degenerate for blastocyst implantation to
occur?
(A) Endometrium in progestational phase
(B) Zona pellucida
(C) Syncytiotrophoblast
(D) Cytotrophoblast

43. Which of the following is the origin of the mitochondrial DNA of all human adult cells?
(A) Paternal only
(B) Maternal only
(C) A combination of paternal and maternal
(D) Either paternal or maternal

44. Individual blastomeres were isolated from a blastula at the 4-cell stage. Each blastomere
was
cultured in vitro to the blastocyst stage and individually implanted into four pseudopregnant
foster mothers. Which of the following would you expect to observe 9 months later?
(A) Birth of one baby
(B) Birth of four genetically different babies
(C) Birth of four genetically identical babies
(D) Birth of four grotesquely deformed babies

45. During the later stages of pregnancy, maternal blood is separated from fetal blood by the
(A) syncytiotrophoblast only
(B) cytotrophoblast only
(C) syncytiotrophoblast and cytotrophoblast
(D) syncytiotrophoblast and fetal endothelium

46. The maternal and fetal components of the placenta are


(A) decidua basalis and secondary chorionic villi
(B) decidua capsularis and secondary chorionic villi
(C) decidua parietalis and tertiary chorionic villi
(D) decidua basalis and villous chorion

47. The intervillous space of the placenta contains


(A) maternal blood
(B) fetal blood
(C) maternal and fetal blood
(D) amniotic fluid

48. A young insulin-dependent diabetic woman in her first pregnancy is concerned that her
daily injection of insulin will cause a congenital malformation in her baby. What should the
physician tell her?
A Insulin is highly teratogenic; discontinue treatment
B Insulin does not cross the placental membrane
C Insulin crosses the placental membrane but is degraded rapidly
D Insulin will benefit her baby by increasing glucose metabolism

49. What is a normal amount of amniotic fluid at term?


A 50 mL
B 500 mL
C 1000 mL
D 1500 mL

6. Which of the following does not pass through the primitive umbilical ring?
A Allantois
B Amnion
C Yolk sac
D Connecting stalk

50. Which of the following best describes the placental components of dizygotic twins?
A One placenta, two amniotic sacs, one chorion
B One placenta, two amniotic sacs, two chorions
C Two placentas, two amniotic sacs, one chorion
D Two placentas, two amniotic sacs, two chorions
51. A 26-year-old pregnant woman experiences repeated episodes of bright red vaginal
bleeding
at week 28, week 32, and week 34 of pregnancy. The bleeding spontaneously subsided each
time. Use of ultrasound shows that the placenta is located in the lower right portion
of the uterus over the internal os. What is the diagnosis?
A Hydatidiform mole
B. Vasa previa
C. Placenta previa
D. Placental abruption

52. A 19-year-old woman in week 32 of a complication-free pregnancy is rushed to the


emergency
department because of profuse vaginal bleeding. The bleeding subsides, but afterward
no fetal heart sounds can be heard, indicating intrauterine fetal death. The woman goes into
labor and delivers a stillborn infant. On examination of the afterbirth, a velamentous placenta
is detected. Although not much can be done at this point, what is the diagnosis?
A. Placenta previa
B. Vasa previa
C. Hydatidiform mole
D. Premature rupture of the amniochorionic membrane

53. A 32-year-old pregnant woman at 30 weeks of gestation comes to her physician because
of
excess weight gain in a 2-week period. Ultrasonography reveals polyhydramnios. Which fetal
abnormality is most likely responsible for the polyhydramnios?
A. Bilateral kidney agenesis
B. Umbilical cord knots
C. Velamentous placenta
D. Esophageal atresia

54. A 25-year-old pregnant woman at 17 weeks of gestation comes to her OB/GYN for a
normal
examination. During routine blood tests, her serum α-fetoprotein (AFP) concentration is
found to be markedly decreased for her gestational age. Which abnormality will the physician
need to rule out based on these low AFP levels?
A. Spina bifida
B. Anencephaly
C. Omphalocele
D. Esophageal atresia

55. Which of the following time intervals best describes the maximum susceptibility period?
A. Week 1
B. Weeks 3–8
C. Weeks 9–38
D. Weeks 15-17

56. Which of the following time intervals best describes the resistant period?
A. Week 1
B. Weeks 3–8
C. Weeks 9–38
D. Weeks 15-17

57. The most common viral infection is


A. cytomegalovirus
B. rubella virus
C. herpes virus type 2
D. varicella zoster virus

58. Which of the following is a parasite found in cats?


A. Treponema pallidum
B. Toxoplasma gondii
C. Rubella virus
D. Cytomegalovirus

59. Warfarin falls into which category of drugs?


A. Category X drugs
B. Category D drugs
C. XX
D. Not either

60. Valium falls into which category of drugs?


A. Category X drugs
B. Category D drugs
C. XX
D. not either

61. Teratogens are defined as agents which induce:


A. Mitosis
B. Carcinogenesis
C. Birth defects
D. Fallot’s tetralogy

62. For chromosomal study, it is best to use the following nucleated


cells:
A. Polymorphs
B. Lymphocytes
C. Epithelial cells
D. Fibroblasts

63. For chromosomal study, the dividing cells are arrested by


colchicine in the following phase of cell cycle:
A. Prophase
B. Metaphase
C. Anaphase
D. Telophase

64. Denver classification divides chromosomes based on their


length into the following groups:
A. A to C (3 groups)
B. A to E (5 groups)
C. A to G (7 groups)
D. A to I (9 groups)

65. Polyploidy is generally not a feature of dividing cells of the


following type:
A. Megakaryocytes
B. Hepatocytes
C. Tubular cells
D. Conceptus of abortions

66. Numeric abnormality in chromosome occurs in the following


conditions except:
A. Ph chromosome in CML
B. Turner’s syndrome
C. Klinefelter’s syndrome
D. Down’s syndrome

67. Mutations affecting germ cells produce:


A. Cancers
B. Inherited diseases
C. Congenital malformations
D. Aneuploidy

68. In lysosomal storage diseases, the following cells are


particularly involved:
A. Hepatocytes
B. Skeletal muscle
C. Macrophages
D. White pulp of spleen

69. Out of the following glycogenosis, the following is example of


lysosomal storage disease:
A. von Gierke’s disease
B. Pompe’s disease
C. Forbe’s disease
D. Anderson’s disease

70. Blastomas are childhood tumours seen more often in the age
range of:
A. <4 years
B. 5-9 years
C. 10-14 years
D. 14-16 years

71. All of the following are X-linked recessive disorders except:


A. Haemophilia A and B
B. Chronic granulomatous disease
C. G-6 PD deficiency
D. Sickle cell anaemia

72. Gaucher cells are positive for all except:


A. PAS
B. Mucicarmine
C. Oil red O
D. Prussian blue
1.Mi
scel
l
aneous1
MCQ–I
ntr
oduct
iont
oPat
hol
ogi
cAnat
omy(
25q)

1.
Causeofdi seaseis:
A.Etiology.
B.Pat hogenesis.
C.Mor phogenesi
s.
D.Pat homor phi
sm.
E.Thanat ogenesis.

2.
Mechanism ofdiseasei
s:
A.Eti
ology.
B.Pathogenesis.
C.Mor phogenesis.
D.Pathomor phi
sm.
E.Thanatogenesis.

3.
Mor
phol ogicstagesofdi
seasei
s:
A.Eti
ology .
B.Pathogenesi s.
C.Mor phogenesis.
D.Pathomor phism.
E.Thanat ogenesis.
4.
Changesi ncourseandmani
fest
ati
onsof
di
seaseis:
A.Et i
ology.
B.Pat hogenesis.
C.Mor phogenesi
s.
D.Pat homor phi
sm.
E.Thanat ogenesis.

5.
Mechanism ofdeathi
s:
A.Eti
ology.
B.Pathogenesis.
C.Mor phogenesi
s.
D.Pathomor phi
sm.
E.Thanatogenesis.

6.
Pat
hol
ogyoft reat
menti
s:
A.Eti
ology.
B.Pathogenesis.
C.Mor phogenesi
s.
D.Iat
rogeny.
E.Thanatogenesis.

7.
Signofbi ol
ogicdeat
hwhi chassociat
edwi
th
decreaseofbodyt emper
atureiscal
led:
A.Cool ing.
B.Ri gorMor ti
s.
C.Dr y
ing.
D.Cadav eri
cspots.
E.Cadav
eri
cdecomposi
ti
on.

8.
Signofbi ol
ogicdeat
hwhichassoci
atedwi
th
har
dningofmuscl esiscal
l
ed:
A.Cool i
ng.
B.Ri gormor ti
s.
C. Drying.
D.Cadav eri
cspots.
E.Cadav er
icdecomposit
ion.

9.
Signofbiol
ogicdeat
hwhichassoci
atedwi
th
ev
aporati
onofflui
discal
l
ed:
A.Cool i
ng.
B.Rigormor ti
s.
C.Dr yi
ng.
D.Cadav eri
cspots.
E.Cadav er
icdecomposi
tion.

10.Si
gnofbi ol
ogicdeathwhichassoci
atedwi
th
enzymati
cdest ruct
ionoft
issuesiscal
led:
A.Cool i
ng.
B.Ri gormor ti
s.
C.Dr ying.
D.Cadav eri
cspot s.
E.Cadav eri
cdecompositi
on.

11.
Signofcli
nical
deat
his:
A.Cool i
ng.
B.Ri
gormor ti
s.
C.Stopofbreath.
D.Dryi
ng.
E.Cadaver
icspots.

12.
Signofcl i
nicaldeathi
s:
A.Cool i
ng.
B.St opofbl oodci r
cul
ati
on.
C.Ri gormor tis.
D.Dr ying.
E.Cadav er
icspot s.

13.Theconceptofcl
i
nicopat
hol
ogi
ccor
rel
ati
on
(CPC)bystudyofmorbi
danatomywas
i
nt r
oducedby:
A.Hippocrat
es
B. Vir
chow
C. JohnHunter
D. Morgagni
E. Laennec

14.Thef i
rstevermuseum ofpat
hol
ogi
canat
omy
wasdev el
opedby :
A.JohnHunt er
B.Roki tansky
C.Rudol fVi
rchow
D.Mor gagni
E.Hi ppocrates
15.AnABOhumanbl oodgr
oupsy
stem wasf
ir
st
descr
ibedby:
A.Edwar dJenner
B.Kar lLandst
einer
C.Hi ppocrat
es
D.Laennec
E.Leeuwenhoek

16.Fr
ozensectionwasf
ir
sti
ntr
oducedby
:
A.Cohnhei m
B.Acker man
C.Vi r
chow
D.Feul gen
E.Laennec

17.El
ectr
onmi croscopewasf
ir
stdev
elopedby
:
A.Bar bar
amccl intock
B.Wat sonandcr ick
C.Ti j
oandl ev
an
D.Ruskaandl orr
ies
E.Nowel landhager for
d

18.St
ructur
eofDNAoft hecel
lwasdescr
ibedby
:
A.Wat sonandcrick
B.Ti j
oandlevan
C.Ruskaandl or
ries
D.Bar baramccl
intock
E.Nowel
landhager
for
d

19.Fl
exi
bil
tyanddynami sm ofDNAwasi
nvent
ed
by:
A.Wat sonandcrick
B.Ti j
oandlevan
C.Ruskaandl orr
ies
D.Bar bar
amcclintock
E.Nowel landhagerford

20.Fat
herofcellul
arpat
hol
ogyi
s:
A.Kar lroki
tansky
B.Rudol fvir
chow
C.G.Mor gagni
D.Ftschwann
E.Bar baraMcclint
ock

21.Human’sgenomeconsi
stsoff
oll
owi
ng
numberofgenes:
A.20,000
B.30,000
C.50,000
D.70,000
E.100,000

22.St
em cellr
esear
chconsi
stsof
:
A.Humancel l
sgrowni
nvitr
o
B.Pl antcel
l
sgrowninvi
tr
o
C.Cadav ercell
sgr owninv it
ro
D.Synony mouswi t
hpcr( polymerasechainreact
ion)
E.Humancel l
sgr owninv ivo
23.PCR( poly
mer asechainr eacti
on)techni
que
wasintroducedby :
A.Ianwi l
mut
B.Wat son
C.Nowel lhagerford
D.Kar ymul li
s
E.Bar baramccl i
nt ock

24.Humangenomepr
ojectwascompl
etedi
n:
A.2001
B.2002
C.2003
D.2004
E.2005

25.Apost mor tem clotismostlikelyto


A.Grossl ydi
spl ayfeatur
esofr ecanal
izati
on
B.Gr osslyhav el
inesofzahn
C.Gr ossl yhav etheappearanceof« chickenfat
»
overl
yi
ng« curr
antj ell

D.Mi croscopi cal
lyappearat
tachedt otheblood
vessel
E.Mi croscopi call
yhavealter
nat i
nglayersofcel
ls
andplatelets
Hemody
nami
cdi
sor
der
s–I(
Kozmi
na52q)

1.Theessent ialdiff
erencebetweenpl
asmaand
i
nterstit
ialfl
uidcompar t
mentis:
Glucosei shigheri ntheformer
Ureai shigherint heformer
Proteincontenti shigherinthefor
mer
Potassium ishi gherintheformer
Bi
car bonateishi gherintheformer

2.Osmot i
cpr essureexert
edbyt hechemi cal
const i
tuentsoft hebodyfluidshasthefoll
owing
featuresexcept :
Crystall
oidosmot i
cpressurecompr i
sesmi nor
portionoft otalosmoti
cpr essure
Oncot i
cpr essureconstit
utesmi norpor
tionof
totalosmot icpressure
Oncot i
cpr essureofplasmai shi
gher
Oncot i
cpr essureofinter
stit
ialfl
uidi
slower

3.Forcausati
onofoedemabydecr easedosmot
ic
pressur
e,thefoll
owingfact
ori
smosti mport
ant
:
Falli
nalbuminaswel lasgl
obul
in
Falli
nglobuli
nlevel
Falli
nalbuminlevel
Falli
nfi
brinogenlev
el
4.Tr
anssudatedif
fer
sfr
om exudat
esi
nhav
ing
thef
oll
owingexcept:
Noinfl
ammat or
ycell
s
Lowglucosecontent
Lowprotei
ncontent
Lowspecifi
cgravi
ty

5.Nephri
ticoedemadi f
fersfrom nephr
oti
c
oedemai nhavi
ngt hefol
lowingexcept:
Mildoedema
Dist
ri
butedonf ace,ey
es
Heavyprotei
nuria
Occursinacuteglomerulonephri
ti
s

6.Thef ol
lowingtypeofoedemai s
character
ist
ical
lydependentoedema:
Nephr ot
icoedema
Nephr i
ti
coedema
Pulmonar yoedema
Cardiacoedema

7.Pulmonar yoedemaappearsduetoelev
at ed
pulmonar yhydrost
ati
cpr
essurewhenthefluid
accumul ati
onis:
Twof old
Fourf ol
d
Eightfold
Tenf old
Si
xfol
d

8.Acti
vehyperemi aisresul
tof
:
Dil
atat
ionofcapi l
lar
ies
Dil
atat
ionofarteriol
es
Venousengorgement
Lymphaticobstructi
on
Venousobstruction

9.Sect
ionedsurfaceofl
ungshowsbr
own
i
ndurat
ionin:
Pulmonaryembol i
sm
Pulmonaryhemor r
hage
Pulmonaryinf
arcti
on
CVClung
Pneumonia

10.Hi stol
ogi csectionsofl ungti
ssuef r
om a
68-y
ear -
oldf emalewi t
hcongest iveheartfai
l
ure
andpr ogressi vebreathingpr obl
emsr eveal
numer oushemosi deri
n-ladencell
swithinthe
al
veol i
.Theses“ heartfailur
ecell
s”origi
natefrom
al
veol ar
Endot heli
al cel
ls
Eosinophi l
s
Lymphocy tes
Macr ophages
Pneumocy tes
11.Whichoneoft heli
stedchangescorr
ect
ly
descri
besthepat hophysiol
ogyinvol
vedi
nthe
producti
onofpul monaryoedemai npati
ent
swit
h
congesti
veheartf ai
lur
e?
Decreasedplasmaoncot i
cpressure
Endothel
ialdamage
Incr
easedhy dr
ost at
icpressur
e
Incr
easedvascularpermeabi l
it
y
Lymphaticobstructi
on

12.Whi choneoft hel istedcl inicalscenariosbest


i
ll
ust r
atest heconceptofact ivehy peremia?
A22- year-oldsecond- yearmedi cal st
udentwho
developsar edfaceaf terbei ngaskedaquest i
on
duringal ecture
A37- year-oldmal ewhodev elopsmassi ve
swel l
ingoft hescr otum duet oinfecti
onwi th
Wucher eri
abancr ofti
A69- year-oldmal ewhodi essecondar yto
progressivehear tfailureandataut opsyi sfound
tohav ea“ nutmeg”l i
ver
A6- year-ol
dboywhodev elopst hesuddenonset
ofintensescr otalpai nduet ot esti
culartorsion
A71- year-oldfemal ewhodev elopsper if
oll
icul
ar
hemor r
hagesduet oadef i
ciencyofv itaminC.

13.Thel
ocal
vacanthy
per
emi
aoccur
sduet
o–
Compr essi
onofar t
eri
esbyt umor
Decreasingofbarometri
cpr essur
e
Obturati
onofar t
eri
esbythrombus
I
rri
tati
onofv asorel
axantnerves
I
ncreasingofbaromet r
icpressure

14.Hemor rhageint
otheabdomi
nal
cav
ity–
Hemopericardium
Hemothorax
Metror
rhagia
Hematomesi s
Hemoperitoneum

15.Local angi
oneurot
ichyperemiaoccur
sduet
o

Decreasingofbarometri
cpr essur
e
I
ncreasingofbaromet r
icpressure
Compr essionofart
eri
esbyt umor
Obturati
onofar t
eri
esbythrombus
I
rri
tati
onofv asorel
axantnerves

16.Oneoft hecausesofgener al
venous
hyperemia,except:
Congenitalvalv
ularhear
tdisease
Acquiredvalvul
arheartdi
sease
Acutemy ocardiali
nfar
cti
on
Fi
broelastosi
sofendocar di
um
Car
diomy
opat
hy

17.Themosti mpor t
antcauseofoedemai
n
nephr oti
csyndromei s–
Hy poprotei
naemi a
Increasedcapil
larypermeabil
i
ty
i
ncr easedhydrostati
cpressur
eofblood
Lymphat icobstruct
ion
Hy perkal
iemia

18.Brownindur
ati
onisat
ypeofchr
oni
cvenous
congesti
onoccursi
n
Li
v er
Heart
Lung
Kidney
Spleen

19.Hemor rhageint
opl
eur
alcav
ityi
s–
Hemopericardium
Hemothorax
Metror
rhagia
Hematomesi s
Hemoperitoneum

20.Hemorr
hageint
otheper
icar
dial
cav
ityi
s–
Hemoperi
cardi
um
Hemothor
ax
Metr
orrhagia
Hematomesi s
Hemoperitoneum

21.Col l
ater
al hyperemiaoccur sduet
o
I
nflammat i
on
I
rri
tationofv asorel
axantner ves
Decreaseofbar ometricpressure
Overloadofphy sicalwor k
Dif
ficult
yofbl oodci r
culat
ioninthemainar
ter
y

22.Thepathogenesi
sofl ocalvenoushy
per
emi
a
Dif
fi
cultyofvenousoutf
low
Dif
fi
cultyofart
eri
ali
nfl
ow
I
ncreaseofinfl
owandout f
low
Decreaseofinfl
owandout flow
I
ncreaseofarteri
ali
nfl
ow

23.Morphol
ogicchangesint
hel
ungsduet
o
chroni
cvenoushy peremi
a–
Edema
Congesti
vehyperemia
Congesti
veindur
at i
on
Brownindur
ation
Nutmegcirr
hosis

24.Mi
nut
e1–2mm hemor
rhagesar
e–
Hemat oma
Hemor rhagicinf
il
tr
ati
on
Petechiae
Ecchymosi s
Hemoper icard

25.Hemorrhagei
s–
Accumulati
onofbl
oodinserouscav i
ti
es
Escapeofbloodf
rom bl
oodv esselsorheart
cavi
ti
estotheenvi
ronment,
bodycav it
ies,t
issues
ororgans
Accumulati
onofbl
oodinthet i
ssueswiththeir
rupt
ure
Accumulati
onofbl
oodinthetissueswithoutthei
r
rupt
ure
Accumulati
onofbl
oodintheor gans

26.Flui
dcollecti
onsi
ndif
fer
entbodycavi
ti
es
dependingont hesi
tear
ealloft
hefol
lowi
ng,
except:
Hydrothorax
Hydroperi
cardium
Hydroperi
toneum
Hydrocephalus
Hydrocele

27.Asever
eandgeneral
i
zededemawi t
h
prof
oundsubcut
aneousti
ssueswel
l
ingiscal
l
ed:
Anasarca
Asci
tes
Hydrothorax
Hydroperi
cardi
um
Hydrocele

28.Pat hophysi
ologiccat
egori
esofedemaar
eal
l
ofthef oll
owing,except
:
I
nflammat i
on
Sodi um ret
enti
on
Calcium retent
ion
Hypopr otei
nemi a
I
ncr easedhy dr
ostati
cpressur
e

29.Conditi
onsr esultinginedemadueto
i
ncreasedhy drost at
icpressur
eareall
ofthe
fol
lowing,except :
Congestiv
ehear tfai
lure
I
nflammat ion
I
schemi chear tdisease
Systemichy pertension
Venousobst ructionorcompr essi
on

30.Theserumprotei
nresponsi
blef
ormai
ntai
ning
coll
oidosmoti
cpressur
eiscall
ed:
Fi
brinogen
Albumin
Globuli
n
Pl
asminogen
Pr
othr
ombi n

31.Edemar esult
ingf
rom thecongest
iveheart
fai
luredev
elopsinalloft
hef ol
l
owing,except
:
Lung
Brain
Liver
Lowerextremiti
es
Upperextr
emi t
ies

32.Generali
zededemaasaresul
tofr
enal
dysfuncti
onornephr
oti
csy
ndromeisl
ocal
izedi
n:
Perior
bit
alti
ssue
Allpart
softhebody
Lung
Brain
Li
v er

33.Pulmonar yedemacommonl
yresul
tsf
rom al
l
ofthefol l
owi ng,except:
Leftventricularf
ailure
Renal f
ailure
Systemichy pert
ension
Appendi citi
s
My ocardialinfar
ction

34.Thef
oll
owi
ngpat
hol
ogi
cpr
ocesses
commonl yoccurt ogether
,except:
Congesti
onandedema
Capil
lar
yr uptur
eandhemor r
hage
Breakdownoft her edcell
sandhemosi
der
osi
s
Acti
vehy peremiaandanemi a
Venousobst ruct
ionandcy anosis

35.Mi cr
oscopi cally,
acutepul monarycongest
ion
i
schar acteri
zedbyal lofthef ol
l
owing,except
:
Engorgedal veolarcapill
aries
Thickenedandf i
br oti
calveolarsept
a
Edemaf luidinalv eoli
Focalintr
a-alveolarhemor rhage
Fociofatelectasis

36.Mi croscopi call


y,chronicpulmonar y
congest i
oni schar acter
izedbyal lofthefol
l
owi
ng,
except:
Engorgedal veolarcapil
laries
Thickenedandf i
broticalveolarsepta
Focalintra-alveolarhemor rhage
Numer ous“ heartfail
urecel l
s”inlungti
ssue
Abscessesi nlungt i
ssue

37.Micr
oscopical
l
y ,
acut
ehepat
iccongesti
oni
s
char
acter
izedbyalloft
hefol
l
owing,except
:
Engor
gedcentralvei
ns
Engorgedcentr
al sinusoids
Destroyedcent
ral hepatocyl
es
Unchangedperipor t
alsinusoi
ds
Diminishedper
ipor t
alhepatocyt
es

38.Mi croscopical
l
y ,chronicpassivecongesti
on
ofthel i
verischaract eri
zedbyal l
oft hefol
l
owing,
except:
Centrolobularnecrosi sofhepatocytes
Centrolobularhemor rhages
Engorgedper iport
al sinusoids
Capil
larizati
onofsi nusoids
Fatt
ychangesofper iportal
hepatocy tes

39.Rupt ureofal ar
gearter
yorv ei
niscommonl
y
duet oallofthef ol
l
owing,except:
Trauma
Atherosclerosis
I
nflammat oryerosi
on
Neopl asti
cer osi
on
I
ncr easedper meabil
it
y

40.Hemor rhagesenclosedwi
thi
nat
issuewi
thi
ts
destr
uct i
onarerefer
redtoas:
Hemat omas
Petechiae
Purpura
Ecchymoses
Hemot
hor
ax

41.Minute(1-t
o2-mm)hemor r
hagesintoskin,
mucousmembr anesorser
osal
surfacesare
cal
led:
Hemat omas
Petechiae
Purpura
Ecchymoses
Hemot horax

42.Smal l(
0.3t
o1.0cm)hemorrhagesint
oskin,
mucousmembr anesorser
osal
surfacesare
cal
led:
Hemat omas
Petechiae
Purpura
Ecchymoses
Hemot horax

43.Large(>1to2cm)subcut
aneous
hemor r
hagesarecal
l
ed:
Hemat omas
Petechiae
Purpura
Ecchymoses
Hemot horax
44.Largeaccumul at
ionsofbl
oodinoneor
anotheroft hebodycav i
ti
esaret
ermedasall
of
thefoll
owing, except
:
Hemot horax
Hemoper icardium
Hemosi derosis
Hemoper itoncum
Hemar thr
osi s

45.Theclinicalsignif
icanceofhemor r
hage
dependsonal lofthef ol
l
owi ng,except
:
Bloodgroupofi ndi
vidual
Volumeofbl oodloss
Rateofbloodl oss
Thesiteofhemor rhage
Thediamet erofdamagedv essel

46.Themostcommonunder l
yi
ngcauseof
pri
mar ybrai
nparenchymalhemorr
hageiswhi
ch
ofthefoll
owing:
Systemiccoagulat
iondisor
ders
Vasculi
ti
s
Systemichypert
ension
Neoplasms
Vascularmalfor
mat i
ons

47.Macr
oscopi
cal
l
y,br
ainhemor
rhagei
s
characteri
zedbyallofthefoll
owi except
ng, :
Local ext
ravasat
ionofblood
Local damageofbr ai
ntissue
Narrowedcer ebr
alsuici
Dist
endedcer ebralgy
ri
Cavitarydestr
ucti
onofbr ai
n

48.Macr oscopicall
y,chronicpassivecongesti
on
oftheli
veri scharacteri
zedbyal lofthefol
lowi
ng,
except
:
Hemorrhagi candwetcutsur f
ace
Red-br
owncent ralareasoft hehepaticl
obules
Gooseliver
Tanperipor t
alareas( f
attychanges)
Nutmegl iver

49.Mi cr
oscopicall
y,brainhemor r
hageis
character
izedbyall ofthefoll
owing,except
:
Fatt
ychangesofdamagedneur ons
Centralcoreofclottedblood
Edemaofadj acentbr ainti
ssue
Dystrophyofsurvivedneur ons
Reactiveprol
if
erati
onofast r
ocytes

50.Themar kersofmy ocardial


ischemiawi
th
i
rrever
sibl
einjur
edmy ofibersarewhichoft
he
fol
lowing:
Cytopl
asmicf at
tychange
Gly
cogendepleti
onofthecytoplasm
Cel
lul
arswell
ing
Cl
umpi ngofnucl
earchromati
n
Contr
acti
onbandsinthecytoplasm

51.Pathologiclesi
onsresul
ti
ngf r
om passi
ve
congestionincludeall
ofthefoll
owi except
ng, :
Nut megliver
Browni ndurat
ionofthelungs
Cyanoticindurati
onofthespleen
Strawberrygall
bladder
Stasisdermatiti
softhelegs

52.Ischemi cinjuryi
nthecent
ral
ner
voussy
stem
result
si n:
Liquefacli
venecr osi
s
Coagul ali
venecr osi
s
Caseousnecr osis
Gangrenousnecr osi
s
Fatnecr osis

EDEMA.HYPEREMI
A.HEMORRHAGE(
Dav
ydov–
40q)

1.
Accumul
ati
onoff
lui
dint
hepl
eur
alcav
iti
esi
s
cal
led:
Hydropericar
dium.
Hydrothorax.
Ascit
es.
Anasarca.
Hydrocele.

2.Accumul at
ionoff
lui
dint
heper
icar
dial
cav
ityi
s
called:
Hy dropericar
dium.
Hy drothorax.
Asci t
es.
Anasar ca.
Hy drocele.

3.Accumul at
ionoff
lui
dint
heper
it
oneal
cav
ityi
s
called:
Hy dropericar
dium.
Hy drothorax.
Asci t
es.
Anasar ca.
Hy drocele.

4.General
izededemawi t
hprof
ound
subcutaneoust i
ssueswel
li
ngi
scal
led:
Hy dr
opericar
dium.
Hy dr
othorax.
Ascites.
Anasarca.
Hydrocele.

5.
Pitt
ingedemaoccursi
n:
Hydropericar
dium.
Hydrothorax.
Asci
tes.
Anasarca.
Hydrocele.

6.
Mechani smsofedemaar ealloft
hef
oll
owi
ng,
EXCEPT:
I
ncreasedhy drost
ati
cpr
essur
e.
Hypoprot
ei nemia.
Sodium r
etention.
Cal
cium retenti
on.
I
nfl
ammat i
on.

7.
Mechani smsofedemaar ealloft
hef
oll
owi
ng,
EXCEPT:
I
ncreasedhy dr
ostat
icpr
essur
e.
Hypoproteinemia.
Hyperproteinemia.
Sodium retenti
on.
I
nfl
ammat ion.

8.
Mechani
smsofedemaar
eal
loft
hef
oll
owi
ng,
EXCEPT:
I
ncreasedhydrost
ati
cpr
essur
e.
Hypoprot
einemia.
Sodium r
etent
ion.
e
I
nfl
ammat ion.
Neoplasi
a.

9.
Organinar ter
ialhyper
emi
aischar
act
eri
zedby
:
Enl
arged,red,hot.
Enl
arged,blue,cold.
Enl
arged,brown, hot
.
Small
,red,cold.
Small
,brown, hot.

10.Organinacut ev enoushy
per
emi
ais
characteri
zedby :
Enlarged,red,hot.
Enlarged,blue-r
ed,cold.
Enlarged,brown, hot.
Smal l
,red,cold.
Smal l
,brown, hot.

11.Si
gnsofchr oni
cvenoushy per
emi
aar
eal
lof
thefol
lowing,EXCEPT:
Brownindurati
onoflungs.
Nutmegl i
ver.
Cyanotici
ndurati
onofspleen.
Cyanotici
ndurati
onofkidneys.
Fat
tyl
i
ver
.

12.Si
gnsofchr oni
cvenoushyperemiaareal
lof
thefol
lowing,EXCEPT:
Cyanosisofskin.
Paleskin.
Brownindurati
onoflungs.
Nutmegl i
ver.
Cyanotici
ndurati
onofspleenandkidney
s.

13.Si
gnsofchr oni
cvenoushy per
emi
aar
eal
lof
thefol
lowing, EXCEPT:
Brownindur ati
onoflungs.
Nutmegl iver.
Cyanoticindurati
onofspleen.
Cyanoticindurati
onofkidneys.
Waxyki dneys.

14.
Nutmegliverischaracter
izedby
:
Enl
arged,
hard,yell
ow-greywi t
hdar
k-r
eddot
s.
Enl
arged,
hard,dark-r
ed.
Enl
arged,
soft,yel
low-grey
.
Small
,har
d,reddish-y
ell
ow.
Small
.soft
,yell
ow-red.

15.
Cyanot
ici
ndur
ati
onofki
dneyi
schar
act
eri
zed
by:
Enl
arged,
hard,brown.
Small
.soft
,red.
Enl
arged,
hard,blue-r
ed.
Small
,har
d,red-brown.
Enl
arged,
soft,brown.

16.Whichpi gmentisaccumul
atedi
nthel
ungsi
n
browni ndurat
ion:
Hemi n
Hemosi deri
n.
Hemomel anin
Melanin.
Bil
ir
ubin.

17.
Hemor r
hagefrom nasal
cav
ityi
scal
l
ed:
Epi
staxi
s.
Haemopt oe.
Haemat emesis.
Maelena.
Metr
orrhagia.

18.
Hemor r
hagefrom r
espi
rat
orysy
stem i
scal
l
ed:
Epi
staxi
s.
Haemoptoe.
Haematemesis.
Mael
ena.
Metr
orr
hagi
a.

19.
Hemor r
hagefrom st
omachi
scal
l
ed:
Epi
staxi
s.
Haemopt oe.
Haemat emesis.
Maelena.
Metr
orrhagia.

20.
Hemor r
hagefrom i
ntest
inei
scal
l
ed:
Epi
staxi
s.
Haemopt oe.
Haemat emesis.
Maelena.
Metr
orrhagia.

21.
Hemor rhagefrom ur
inar
ysy
stem i
scal
l
ed:
Epi
staxi
s.
Haemoptoe.
Haematemesi s.
Haematuri
a.
Metr
orrhagia.

22.
Hemor r
hagef
rom eari
scal
l
ed:
Epi
staxi
s.
Haemoptoe.
Haematemesis.
Haematuri
a.
Otor
rhagi
a.

23.
Accumul ati
onofbl
oodi
ntheper
icar
dial
cav
ity
i
scall
ed:
Hemoperi
car di
um.
Hemothorax.
Hemoperi
toneum.
Hemarthr
osis.
Hematoma.

24.
Accumul ati
onofbl
oodi
nthepl
eur
alcav
iti
esi
s
cal
l
ed:
Hemoperi
car di
um.
Hemothorax.
Hemoperi
toneum.
Hemarthr
osis.
Hematoma.

25.
Accumul ati
onofbl
oodi
ntheper
it
onal
cav
ityi
s
cal
l
ed:
Hemoperi
car di
um.
Hemothorax.
Hemoperi
toneum.
Hemarthr
osis.
Hematoma.
26.
Accumul ati
onofbl
oodi
nthej
ointcav
ityi
s
cal
l
ed:
Hemoperi
car di
um.
Hemothorax.
Hemoperi
toneum.
Hemarthr
osis.
Hematoma.

27.Hemor rhagewithr
egul
arborder
sand
destr
uct i
onoft i
ssuei
scall
ed:
Hemat oma.
Hemor rhagicinf
il
tr
ati
on.
Petechiae.
Purpura.
Ecchymoses.

28.Hemor rhagewithi
rregul
arborder
sand
accumul at
ionofbloodintheint
ersti
ti
alt
issuei
s
cal
led:
Hemat oma.
Hemor rhagicinf
il
tr
ati
on.
Petechiae.
Purpura.
Ecchymoses.

29.
Minut
e hemor
rhages (
1-2 mm) into ski
n,
mucous membr
anes orserosalsur
faces ar
e
cal
led:
Hemat oma.
Hemor rhagi
cinf
il
tr
ati
on.
Petechiae
Purpura.
Ecchymoses.

30.
Small(3-5 mm) hemor rhages i
nto ski
n,
mucous membr anes orserosalsurf
aces ar
e
cal
l
ed:
A.Hemat oma.
B.Hemor rhagi
cinf
il
trat
ion.
C.Petechiae.
D.Purpura.
E.Ecchymoses.

31.Lar
ge (1-
2 cm)subcut aneous hemor
rhages
arecall
ed:
A.Hemat oma.
B.Hemor rhagi
cinf
il
tr
ati
on.
C.Pet echiae.
D.Pur pura.
E.Ecchy moses.

32.
Lar
ge(morethan2cm)hemor
rhagei
ntoski
n
i
scall
ed:
A.Hematoma.
B.Hemorrhagi
cinf
il
tr
ati
on.
C.Brui
se.
D.Purpur
a.
E.Ecchymoses.

33.
Rupt
ureofthev essel
wal
lmayber
esul
tof
:
A.Atheroscl
er osis.
B.Inf
lammat ion.
C.Invasi
onofcancer .
D.Hypoxia.
E.Int
oxicati
on.

34.
Rupt
ureofthehear twal
lmayber
esul
tof
:
A.Inf
lammat ion.
B.My ocardi
al i
nfar
cti
on.
C.Invasi
onofcancer s.
D.Hypoxia.
E.Int
oxicati
on.

35.
Corr
osionoft hev esselwal
lmayber
esul
tof
:
A.Ather oscl
er osi
s.
B.Inflammat ion.
C.Vascul arabnormal i
ti
es.
D.Hy poxia.
E.Intoxicati
on.

36.
Cor
rosi
onoft hevesselwal
lmayber
esul
tof
:
A.Ather
osclerosi
s.
B.Vascularabnor
mal i
ti
es.
C.I
nvasionofcancer
.
D.Hypoxia.
E.I
ntoxi
cati
on.

37.I
ncreaseofv ascularpermeabi
l
itymaybe
resul
tof :
A.At heroscl
erosis.
B.Vascul arhyal
inosis.
C.Inv asi
onofcancer .
D.Hy poxia.
E.Inflammat i
on.

38.I
ncreaseofv ascul arpermeabi
l
itymaybe
resul
tof :
A.At heroscl
er osis.
B.Vascul arhy al
inosis.
C.Inv asi
onofcancer .
D.Inflammat ion.
E.Intoxicati
on.

39.I
ncreaseofv ascularpermeabi
l
itymaybe
resul
tof :
A.At heroscl
er osi
s.
B.Vascul arabnormalit
ies.
C.Inv asi
onofcancer .
D.Inflammat ion.
E.Intoxicati
on.
40.
Cli
nicalsi
gnif
icanceofhemorrhagedepends
onall
thef ol
l
owing,EXCEPT:
A.Bl oodgroupofi ndi
vi
dual
.
B.Vol umeofbl oodloss.
C.Rat eofbloodloss.
D.Si teofhemor r
hage.
E.Di ameterofdamagedv essel
.

Hemodynamicdi
sorder
s-I
I:t
hrombosi
s,
embol
ism,
infar
cti
on,
shock,DI
C.(Kozmina-60)

1.Insept i
cshock, pathogenesisofendothel
i
al
celli
njuryinvol
vest hefoll
owingmechani sms
except :
Lipopolysacchari
def r
om lysedbacteriai
njur
es
theendot heli
um
Inter
leukin-
1causesendot heli
alcel
li
njury
TNF-αcausesdi r
ectcy tot
oxici
ty
Adher enceofPMNst oendot heli
um causes
endot hel
ialcel
li
njury

2.Anintactendotheli
um elabor
atest
hef
oll
owi
ng
anti
-t
hromboticfactorsexcept:
Thr
ombomodul in
ADPase
Ti
ssueplasminogenact
ivat
or
Thr
ombopl ast
in

3.Themostcommoncauseofar t
eri
al
thr
omboembol i
is:
Cardi
acthrombi
Aorti
caneury sm
Pulmonaryv eins
Aorti
catherosclerot
icpl
aques

4.Venousembol
iar
emostof
tenl
odgedi
n:
Intest
ines
Kidney
Lungs
Hear t
Brain

5.Pathologicchangesbet weensudden
decompr essionfrom hi
ghpr essuret
onor mal
l
evelsanddecompr essi
onf r
om lowpressur
eto
normal l
ev el
sare:
Moremar kedintheformer
Moremar kedinthelatt
er
Nodifferencebetweent het wo
Acuteformi smor emar kedinthelat
ter
Chronicfor mismoremar kedinthel
atter
6.Theinfarctoff
oll
owi
ngor
gani
sinv
ari
abl
y
hemor r
hagic:
I
nfarctkidney
I
nfarctspleen
I
nfarctlung
I
nfarctheart
I
nfarctli
ver

7.Milr
oy ’
sdiseasei
s:
Cerebraloedema
Pul
monar yoedema
Heredi
t ar
ylymphoedema
Posturaloedema
Ascit
es

8.Pickthecor rectsequence:
Renin-Angi otensinII-Angi otensi
nI–
Angiotensinogen-Al dost erone
Angiotensinogen–Reni n-Angi ot
ensi
nII-
AngiotensinI-Al dost erone
Renin–Angi otensinogen-Angi ot
ensi
nI-
AngiotensinII–Al dost er
one
Aldosterone–Reni n–Angi otensi
nogen-
AngiotensinII-Angi ot ensinI
AngiotensinI-Al dost erone–Reni n–
Angiotensinogen-Angi otensinII
9.Whichofthef
oll
owingistr
ue?
Art
eri
althr
ombiarewhit
eandocclusi
ve
Venousthr
ombiarewhiteandoccl
usiv
e
Art
eri
althr
ombiarewhit
eandmur al
Venousthr
ombiareredandmural
Venousthr
ombiarewhiteandmural

10.Disseminatedintravascul
arcoagulati
on(DIC)
i
schar act
eri
zedbyt hef oll
owingexcept:
Thrombocy t
openia
Microangi
opathichemol y
ticanemia
PresenceofFDPs( f
ibri
ndegr adat
ionproduct
s)in
theblood
Normal pr
othrombint ime
Thrombintimei sprolonged

11.Whi choft hefol


lowingisnotincl
udedinTTP
(t
hrombot icthrombocy t
openicpurpur
a)tr
iad?
DIC(dissemi natedi
ntravascul
arcoagul
ati
on)
Thrombocy t
openia
Microangiopat hi
chemol yt
icanemia
Fi
brinmi crothrombi
Leucocy t
osis

12.Mostcommonmani festat
ionofDI
C
(disseminat
edi
ntr
avascularcoagul
ati
on)i
s:
Bleeding
Thr ombosis
Mi
croangi
opat
hichemol
yti
canemi
a
Or
gandamage
Hi
ghtemperat
ure

13.A25- year-
oldfemal epresentswithahi stor
yof
l
osingf ourpregnanciesi nthepast5y ears.She
al
sohasshi storyofrecur rentpai
nsinherl egs
secondar ytorecurr
entt hrombosis.Her
sympt omsar emostl ikelyduet oadefici
encyof :
PAi nhibi
ti
ors
ProteinC
Plasmi n
Thrombi n
C’1inactivat
or

14.Procoagulantfact
orspr
oducedbyendot
hel
i
al
cell
sinclude
Thrombomodul in
Prostacycli
n
wonWi l
l
ebrandfactor
ThromboxaneA2
Fibr
inogen

17.Whati
sthemostcommonsi
teofor
igi
nof
thr
omboti
cpulmonar
yembol
i
?
Deepl
egveins
Lumenoflef
tvent
ri
cle
Lumenofr i
ghtventr
icl
e
Mesenteri
cveins
Super
fi
ciall
egv ei
ns

18.A9- year-ol
dboysuddenl ydevelopssev er
e
testi
cularpain.Hei stakentotheemer gency
room, wherehei sevaluat
edandimmedi ately
takentosur gery.Therehislef
ttestisisfoundt o
bemar kedlyhemor r
hagicduetotest i
culartorsi
on.
Thisabnor malitycausesahemor rhagici nf
arcti
on
becauseof
Arter
ialocclusion
Septicinfar
ct i
on
Thecol l
ateralbloodsupplyofthet esti
s
Thedual bloodsuppl yofthetesti
s
Venousoccl usion

19.Redi nfarct
ioni
scausedbyalloft
hefol
lowi
ng
events,except:
Coronaryar ter
ythrombosis
Pulmonar yembol i
sm
Torsionoft hetest
is
Super i
ormesent er
icart
erythr
omboemboli
sm
Portalveinthrombosis

20.Shockiscommonl yassoci
atedwi
thal
loft
he
fol
lowi
ngcondit
ions,except
:
Escher
ichi
acolisepsi
s
Myocardi
alinfar
cti
on
Choler
a
Acutepancreati
ti
s
Cerebr
alinf
arcti
on

21.Thef at
eoft hethr
ombusmaybeal
loft
he
foll
owing, except:
Dissolution
Recanal i
zat i
on
Organizat i
on
Embol izati
on
Malignization

22.Thecausesofi nfar
cti
onincl
udeall
oft
he
fol
lowingpat hologi
ccondit
i except
ons, :
Trombot i
cev ents
Embol i
cev ents
Arter
ialocclusion
Localvasospasm
Hemophi l
ia

23.Redi nfarctoccur
sinal lofthefol
lowi
ng
pathologicconditi
ons,except :
Venousoccl usi
on
Coronaryoccl usi
on
Looset i
ssue
Tissueswi thdualcir
culati
on
Previousl
ycongest edwi t
hbl oodti
ssues
24.Whi teinfarctresul
tsfr
om whi
chofthe
foll
owing:
Venousoccl usion
Arteri
alocclusion
Looset i
ssue
Tissueswi thdual cir
cul
ati
on
Previouslycongest edwit
hbloodt
issues

25.Redinf
arctoccur
sinwhi
choft
hef
oll
owi
ng
organs:
Lung
Heart
Spleen
Kidney
Lowerextr
emity

26.Whitei
nfar
ctcommonl
yoccur
sinal
lor
gans,
except:
Lung
Spleen
Kidney
Heart
Brain

27.Theformati
onofathrombusi
sfav
oredbyal
l
ofthefol
lowi
ng,except
:
Anemia
Endothel
ial
inj
ury
Pregnancy
Stases
Thrombocytosi
s

28.Thet ypeoft i
ssuenecr osiscommonly
associatedwi thmy ocar
dialinfar
cti
oni
swhichof
thefoll
owi ng:
Caseousnecr osi
s
Coagulationnecrosis
Enzymat icfatnecrosis
Gangrenousnecr osi s
Fibr
inoidnecr osi
s

29.Pul monar yembolimayor


igi
nat
efr
om al
lof
thefollowingsit except
es, :
Deepl egv eins
I
nf eri
orv enacava
Pelvicv ei
ns
Por t
alv ei
n
Rightat ri
alappendage

30.Thedev elopmentofendothel
i
al-
li
nedblood
channelst hatreestabl
i
shbloodfl
owt hr
ougha
vascularthrombusi sknownas:
Coll
ateralcirculat
ion
Recanalizati
on
Organi
zation
Hyali
nizat
ion
I
ncapsulation

31.Thei nit
ialstepofthethr
ombusf
ormat
ioni
s
whichoft hef oll
owing:
Acti
vationof t
hr ombin
Developmentoff ibr
inpl
ugs
Endotheliali
njury
Marginationofleukocyt
es
Trappingofr edcel l
s

32.Thet y
peoft issuenecr
osi
sassoci
atedwi
th
renali
nfarcti
onis:
Caseousnecr osi
s
Coagulati
onnecr osis
Enzymaticfatnecr osi
s
Gangrenousnecr osis
Liquef
activenecrosis

33.Dissemi natedi
ntrav
ascularcoagul
ati
onis
character
izedbyeachoft hef ol
l
owing,except
:
Decreasedf ibr
inol
ysi
s
Decreasedpl asmaf i
bri
nogen
Hemol ysi
s
Prolongedpar t
ialt
hromboplasti
ntime
Thrombocy topenia
34.Eachoft hefoll
owingcondit
ionsfav
orst
he
developmentoft hrombosi
s,except
:
Endothelial
inj
ury
Polycythemia
Stases
Thrombocy topenia
Congest i
on

35.Thetypeofnecr osi
smostoftencausedby
suddenischemiafrom vascul
aroccl
usi
onis:
Apoptosis
Caseousnecrosis
Coagulati
onnecrosis
Fatnecrosi
s
Fi
brinoi
dnecrosi
s

36.Al
lthef
oll
owingdisor
dersareassoci
atedwi
th
di
sseminat
edint
ravascul
arcoagul
ati except
on, :

I
nfecti
ons
Neoplasms
Massivetissuei
njur
y
Malnutri
ti
ons
Obstetr
iccompli
cati
ons

37.Theusual
sourceofpul
monaryembol
iis-
Thrombii
nthedeepvei
nsofthel
owerext
remi
ti
es
Thr
ombi
int
heveinsofupperextr
emit
ies
Thr
ombi
int
hevenaporta
Thr
ombi
int
hel
eftatri
alappendage
Thr
ombi
int
heuterusart
eri
es

38.Embol i
int hemai npul
monar yar
ter
yof
ten
cause:
Infar
cti
onofthel ung
Suddendeat h
Infar
cti
onoft hehear t
Disseminatedi ntr
avascul
arcoagul
ati
on
Acutepneumoni a

39.Theor i
ginofembol
ii
ncl
udeal
loft
he
fol
lowing,except
:
Thrombi
Fatdroplets
Airbubbles
Amy l
oidmasses
Microorganisms

40.Thef ormsofembolism i
ncl
udeal
loft
he
fol
lowing,except:
Pulmonar ythromboembol
ism
Fatembol ism
Viral
embol i
sm
Bacteri
alembol i
sm
Airembol i
sm
41.Theselecti
vestai
n,usedt
oident
if
yfati
nthe
fatembolisyndr
omei s:
SudanI I
I
Tolui
dinblue
Congor ed
PASr eacti
on
Hemat oxyl
inandeosi
n

42.Acutet ubularnecrosi
sdev
elopsi
nwhi
chof
thefoll
owingcondi ti
ons:
Amy loi
dosi s
Acut egl
omeni lonephr
iti
s
Kidneyinfarct
Systemict hromboembol i
sm
Hy povol
emi cshock

43.Thecommonestsi
teoft
hrombosi
sis
Vein
Art
ery
Heart
Capil
lar
y
Lymphatics

44.Thepulmonaryt
hrombusi sdif
fer
entfr
om
pulmonarythr
omboembol usal
lexcept
, :
Local
lyformed
Insmallart
eri
esandbranches
Fi
rmlyadherentt
ovesselwal
l
Headpale,t
ailr
ed
Lyi
ngfreetovessel
wall

45.Whi choft
hef ol
lowi
ngi
str
ueoff
atembol
i
sm,
except:
Traumaofbones
Traumaofsof tti
ssue
Diabetesmell
itus
Extensivebur
ns
Operationonhead

46.Airemboli
sm occur
sduet
otr
aumaof-
Largeart
eri
es
Neckv ei
ns
Softti
ssue
Lungs
Heart

47.Retrogradeembolism occur
sin-
Aort
a
Femor alar
tery
Port
al vei
n
Microcir
culator
yvessels
Heart

48.Paleinf
arct
ioni
snotseeni
n-
Intest
ines
Liv
er
Spl
een
Heart
Bri
ne

49.Thet hrombusi ncompar


ewi
thpost
mor
tem
bl
oodcl otis?
Layingfreeinthevessel
Wi t
hsmoot hsur
face
Attachedtot hevessel
’swal
l
Elasti
c
Gelati
nousandsof t

50.Thehistologicalf
eaturesofshocki
ncl
udes
fol
lowing,except:
ATN( acut
et ubularnecrosis)
Pulmonarycongest i
on
Depleti
onofl i
pidsinadrenalcor
tex
Hepaticnecrosis
Depleti
onofl ymphocytes

51.Fatemboli
sm mayber
ecogni
zedwi
th-?
SudanIII
Hematoxii
in-
eosi
n
Congo-red
ByVan-Guison
Methil
enblue
52.Whichoft hefollowi
ngi
smostf
requentsi
teof
venoust hrombosis?
Veinsofl owerext
r emit
y
Pelvi
cv eins
Portalvein
Hepaticv ei
n
Pulmonar yvei
ns

53.Stasisofbloodis–
Accumul at
ionofbloodint
hemi croci
rcul
atory
vessel
Defici
encyofbloodi nt
hemicr
ocircul
atoryvessel
Stoppingofbloodflowint
hemicr oci
rcul
atory
vessel
Stoppingofbloodflowint
hevenoussy stem
Accumul at
ionofbloodint
heaor t
a

54.Inast at
eofshockt her
eis:
Adecr easedhydrostat
icpr
essur
eandincr
eased
osmot i
c
Cardiovascul
arcoll
apse.
Acti
vepr ocessl
eadingtoincr
easedv
olumeof
bl
ood.
Decreasedpulserate.
Fever

55.Unfav
orabl
eout
comeoft
hrombosi
sis–
Asepti
cautol
ysi
s
Canali
zati
on
Vascular
isat
ion
Thromboembol i
sm
Petri
fi
cati
on

56.Thecauseofi schemi
cinf
arcti
s–
Stasi
sofbl ood
Lymphost asi
s
Venushy peremia
Arter
ial
thrombosis
Arter
ial
hy peremi
a

57.Inwhichdi seaseoccurtheparadoxal
embol i
sms?
Leftventr
icleaneurism
Aorti
ccoar ctati
on
Pulmonaryar terystenosi
s
Defectofthei ntr
aventri
cul
arsept
a
Tri
cuspidst enosis

58.Inshockcharacteri
sti
cfeaturei
s–
Cardiacf
ailure
Poorperfusionoft
issues
Cyanosis
Oedema
Acuteadrenalhemor r
hagicnecrosi
s

59.
All
oft
hef
oll
owi
ngcondi
ti
onsmaypr
edi
spose
topulmonar yemboli except
sm,
Protei
nSdef ici
ency
Malignancy
Obesity
Progesteronether
apy
Bed-ri
ddenpat i
ents

60.Thrombusi
ntheaneur
ismsi
s–
Obturat
ive
Congesti
ve
Dil
ati
ve
Mural
Ball

THROMBOSI
S.EMBOLI
SM.I
NFARCTI
ON.SHOCK
(
Davy
dov-
40)

1.
Favourablesi gni
fi
canceoft hrombosi smaybe
i
n:
A.Cor onar yarter
yinatherosclerosis.
B.Pari
et althrombusofl ef
tv entr
icleinmyocardi
al
i
nfar
cti
on.
C.Thrombosi sofv enafemor ali
sinphlebi
tis.
D.Cer ebral ar
teri
esinatherosclerosis.
E.Art
erialthrombosisinadgesoft raumati
cwound.

2.
Redt
hrombususual
l
yoccur
sin:
A.Art
er i
es.
B.Veins.
C.Aorta.
D.Capill
ari
es.
E.Lymphaticvessel
s.

3.
Whi
tet hr
ombususuallyoccur
sin:
A.Art
er i
es.
B.Veins.
C.Venules.
D.Capill
ari
es.
E.Lymphaticvessel
s.

4.
Mixedt
hrombusconsistsof:
A.Head,neck,body.
B.Head,body,ext
remit
ies.
C.Head,body,tai
l.
D.Head,neck,tai
l
.
E.Neck,body,
tail
.

5.
Signofthr
ombusi s:
A.Smoot hsurf
ace.
B.Soft.
C.Homogenouscutsur face.
D.At t
achedtovesselwall
.
E.Notattachedtovesselwal
l.

6.
Signoft
hrombusi
s:
A.Roughsurface.
B.Soft
.
C.Gomogenouscutsur face.
D.Smoothsur f
ace.
E.Notatt
achedt ovesselwal
l.

7.
Signofthrombusis:
A.Smoot hsurface.
B.Soft,wet.
C.Homogenouscutsur face.
D.Hard,dry.
E.Notat t
achedtovesselwal
l.

8.
Signofthr
ombusis:
A.Smoot hsurf
ace.
B.Laminatedcutsurf
ace.
C.Homogenouscutsur face.
D.Soft,wet.
E.Notattachedtovesselwal
l.

9.
Signofpostmor tem cl
otis:
A.Ruphsur face.
B.Lami nat
edcutsur face.
C.Homogenouscutsur face.
D.Har d,dr
y.
E.Attachedtov esselwall.
10.
Signofpost mortem cl
otis:
A.Ruphsur face.
B.Smoot hsurf
ace.
C.Homogenouscutsur f
ace.
D.Har d,dry.
E.At t
achedt ovesselwall
.

11.
Signofpost mortem clotis:
A.Ruphsur f
ace.
B.Har d,dry.
C.Lami natedcutsur f
ace.
D.Notat tachedtov esselwall.
E.At t
achedt ovesselwal l
.

12.
Signofpostmor tem cl
otis:
A.Ruphsur face.
B.Har d,dr
y.
C.Lami nat
edcutsur face.
D.Sof t,
wet.
E.At t
achedtov esselwall
.

13.
Ball
thrombusi slocal
i
zedi
n:
A.Leftventri
cle.
B.Rightventri
cle.
C.Leftatri
um.
D.Rightatri
um.
E.Aorta.
14.
Locali
zationofthr
ombot
icv
eget
ati
onsi
s:
A.Aor t
a.
B.Arteri
es
C.Veins.
D.Hear tvalves.
E.Capill
aries.

15.
Grouthofconnect i
vet
issuei
nthr
ombot
ic
massesiscal led:
A.Or ganizati
on.
B.Cal ci
fi
cat i
on.
C.Recanal izati
on.
D.Propagat i
on.
E.Embol ization.

16.
Depositi
onofcalci
um sal
tst
othr
ombot
ic
massesiscalled:
A.Organizati
on.
B.Calci
ficati
on.
C.Recanalizati
on.
D.Propagat i
on.
E.Embol i
zation.

17.
Det
achtmentoft
hrombusf
rom v
essel
wal
li
s
cal
l
ed:
A.Organi
zat
ion.
B.Calci
fi
cat
ion.
C.Recanalizati
on.
D.Propagat i
on.
E.Embolization.

18.
Paradoxicalembol ism mayoccur
sin:
A.Aneur i
sm ofl eftventr
icl
e.
B.Coar ctati
onofaor ta.
C.Stenosi sofpul monaryarter
y.
D.Vent ri
cularseptaldefect.
E.Atresiaofaor ta.

19.
Paradoxicalembol ism mayoccur
sin:
A.Aneur ism ofleftventr
icl
e.
B.Coar ctati
onofaor ta.
C.Stenosi sofpulmonar yarter
y.
D.Atresiaofaor ta.
E.Atri
al septaldefect.

20.
Exogenicty
peofemboli
sm i
s:
A.Thromboembol i
sm.
B.Cellembolism.
C.Amni oti
cfl
uidembol
ism.
D.Gasembol ism.
E.Foreignbodyemboli
sm.

21.
General
i
zati
onofinf
ect
ioni
sar
esul
tof
:
A.Thromboemboli
sm.
B.Microbi
alembol
ism.
C.Amnioti
cfl
uidemboli
sm.
D.Gasembolism.
E.For
eignbodyemboli
sm.

22.
Metastasi
sofmal i
gnantt
umor
sisar
esul
tof
:
A.Thromboembol i
sm.
B.Microbial
embolism.
C.Cellembolism.
D.Airembolism.
E.Forei
gnbodyembol ism.

23.
Fracturesofbonesmayleadt
o:
A.Thr omboembol i
sm.
B.Mi crobial
embolism.
C.Fatembol i
sm.
D.Gasembol ism.
E.Amni ot
icfl
uidembol
ism.

24.
Postpart
um peri
odmaybecompl
i
cat
edwi
th:
A.Thromboembol i
sm.
B.Microbialembol
ism.
C.Amni oti
cflui
demboli
sm.
D.Gasembol i
sm.
E.Foreignbodyemboli
sm.

25.
Commonlocal
i
zat
ionofi
schemi
cinf
arct
ioni
s:
A.Brai
n.
B.Lungs.
C.Smalli
ntest
ine.
D.Lar
geintest
ine.
E.Li
ver
.

26.
Commonl ocal
izat
ionofhemor
rhagi
c
i
nfar
ctioni s:
A.Brai n.
B.Lungs.
C.Smal li
ntesti
ne.
D.Largei ntesti
ne.
E.Liver .

27.
Causeofhemorrhagicinf
arct
ioni
s:
A.Thrombosisofartery
.
B.Emboli
sm ofarter
y .
C.Venoushyperemia.
D.Vasospasm.
E.Compressi
onofar tery
.

28.
Causesofischemicinfar
cti
onar
eal
loft
he
fol
l
owing,
EXCEPT:
A.Thrombosisofartery.
B.Embolism ofart
ery.
C.Vasospasm.
D.Compr essi
onofartery.
E.Venoushy per
emia.

29.
Mor
phol
ogi
cal
l
yinf
arct
ionofbr
ainoccur
sas:
A.Coagulati
venecr
osis.
B.Li
quefacti
venecrosi
s.
C.Gangrene.
D.Fatnecrosi
s.
E.Fi
bri
noidnecrosi
s.

30.
Morphol ogi
call
ymy ocardi
ali
nfar
cti
onoccur
s
as:
A.Coagul ati
venecrosis.
B.Liquefactiv
enecr osi
s.
C.Gangr ene.
D.Fatnecr osis.
E.Fibri
noidnecrosis.

31.
Unfourableout comeofi
nfar
cti
oni
s:
A.Organization.
B.Incapsulation.
C.Cy stf
or mat i
on.
D.Suppur ati
on.
E.Hemosi derosis.

32.
Typicalout comeofhemor
rhagi
cinf
arct
ioni
s:
A.Organi zation.
B.Incapsul ation.
C.Cy stformat i
on.
D.Suppur at i
on.
E.Hemosi derosis.
33.
Typicalout comeofcer
ebr
ali
nfar
cti
oni
s:
A.Organi zation.
B.Incapsul ation.
C.Cy stformat i
on.
D.Suppur at i
on.
E.Hemosi derosis.

34.Ty
peofshockwhi chassoci
atedwi
thact
ionof
str
ongpainfulir
rit
antis:
A.Car di
ogenic.
B.Tr aumatic.
C.Hy povolemic.
D.Neur ogenic.
E.Anaphy l
act i
c.

35.Typeofshockwhi chassoci
atedwi
thact
ionof
decreaseofcar diacoutputi
s:
A.Car di
ogenic.
B.Tr aumat ic.
C.Hy povolemic.
D.Neur ogenic.
E.Anaphy l
act i
c.

36.
Typeofshockwhi
chassoci
atedwi
thl
ossof
bl
ood:
A.Car di
ogenic.
B.Traumat ic.
C.Hy povolemic.
D.Neur ogenic.
E.Anaphy l
act i
c.

37.
Typeofshockwhichassoci
atedwi
thsev
ere
bur
nsis:
A.Car di
ogenic.
B.Traumat i
c.
C.Endotoxic.
D.Hy povol
emi c.
E.Anaphy l
actic.

38.
Typeofshockwhi chassoci
atedwi
th
i
nfect
iousdiseasesi
s:
A.Car di
ogenic.
B.Traumat ic.
C.Hy pov ol
emic.
D.Neur ogenic.
E.Endot oxi
c.

39.
Typeofshockwhichassoci
atedwi
thi
njur
yof
spi
nalcordis:
A.Car di
ogenic.
B.Traumat ic.
C.Hy povolemic.
D.Neur ogenic.
E.Anaphy
lact
ic.

40.Ty
peofshockwhi chassociat
edwith
generalizedr eacti
onsofhypersensi
ti
vi
tyis:
A.Car diogenic.
B.Tr aumat ic.
C.Hy pov olemic.
D.Neur ogenic.
E.Anaphy l
act i
c.
CellI
njur
y(Davydov -
14q)

1.
Var
iantofreversi
blecel
linj
uryi
s:
A.Coagulativ
enecrosis.
B.Gangrene.
C.Cell
ularswell
ing.
D.Fatnecrosis.
E.Apoptosis.

2.
Var
iantofreversi
blecel
linj
uryi
s:
A.Coagulativ
enecrosis.
B.Fatt
ychange.
C.Caseousnecr osi
s.
D.Fatnecrosis.
E.Apoptosis.

3.
Progr
ammedcelldeat
his:
A.Coagul
ati
venecrosi
s.
B.Gangrene.
C.Cell
ularswell
ing.
D.Fatnecrosis.
E.Apoptosis.

4.Typeofnecr osiswhichassoci
atedwi
th
formati
onofy ell
ow-whitehar
dareasi
s:
A.Coagul ati
venecrosis.
B.Li quefacti
venecr osi
s.
C.Gangr ene.
D.Fatnecr osis.
E.Fi bri
noidnecr osi
s.

5.Ty
peofnecr osiswhichusual
l
yoccur
sinbr
ain
andspi
nal cordis:
A.Coagul ati
venecrosis.
B.Liquefacti
venecr osi
s.
C.Gangr ene.
D.Fatnecr osi
s.
E.Fibri
noidnecr osi
s.

6.Typeofnecr osi
swhi chusual
l
yoccur
sin
tubercul
ousi nfct
ionis:
A.Coagul at
ivenecrosis.
B.Li quefactiv
enecr osi
s.
C.Gangr ene.
D.Caseousnecr osi
s.
E.Fi bri
noidnecrosis.
7.
Typeofnecr osiswhichusual
lyoccur
sinacut
e
pancr
eati
tisandappear sli
kechal
kywhite
pl
aquesinadi poseti
ssueis:
A.Coagul ati
venecrosis.
B.Liquefacti
v enecrosi
s.
C.Fatnecr osis.
D.Caseousnecr osi
s.
E.Fibri
noidnecr osi
s.

8.Ty
peofnecr osiswhichusuall
yoccur
sin
connect
ivetissueandbl oodv
essel
swalli
s:
A.Coagul ati
venecrosis.
B.Liquefactivenecrosi
s.
C.Fatnecr osis.
D.Caseousnecr osi
s.
E.Fibri
noidnecr osi
s.

9.
Typeofnecr osi
swhi choccursi
next
remi
ti
es
duetoart
er i
alobst
ructionis:
A.Coagul ati
venecrosis.
B.Liquefacti
venecrosis.
C.Fatnecr osi
s.
D.Gangr ene.
E.Fibri
noidnecrosi
s.

10.Ty
peofnecrosiswhichiscausedby
Clost
ri
dium per
fri
ngensis:
A.Coagulati
venecrosi
s.
B.Li
quefacti
v enecr
osi
s.
C.Fatnecrosis.
D.Drygangrene.
E.Gasgangr ene.

11.
Out
comeofnecr osi
swi
thf
ormat
ionofscari
s
cal
l
ed:
A.Organizat
ion.
B.Calci
fi
cati
on.
C.Ossif
icati
on.
D.Cystformation.
E.Suppurati
on.

12.Out
comeofnecr osi
swi
thf
ormat
ionofbone
ti
ssueiscalled:
A.Or ganization.
B.Cal ci
fi
cat i
on.
C.Ossi f
ication.
D.Cy stformat ion.
E.Suppur ation.

13.Out
comeofnecr osi
swit
hf or
mat
ionofcav
ity
fi
l
ledwi t
hserousf l
uidi
scal
led:
A.Or ganizat
ion.
B.Cal ci
ficat
ion.
C.Ossi fi
cati
on.
D.Cy stformat i
on.
E.Suppur
ati
on.

14.Out
comeofnecr osi
swhichassoci
atedwi
th
bacter
iali
nfecti
oniscall
ed:
A.Or ganizat
ion.
B.Cal ci
fi
cation.
C.Ossi f
icati
on.
D.Cy stformat i
on.
E.Suppur ati
on.

Cel
lInj
ury(
Kozmi
na-32q)

16.A49- year-
oldmandev elopsanacut e
my ocardial i
nfar
ctionbecauseoft hesudden
occlusionoft heleftant
eriordescendingcor
onary
arter
y .Thear easofmy ocardialnecr
osiswit
hin
thev entr
iclecanbestbedescr i
beas
Coagul ati
v enecrosis
Liquefactivenecrosis
Fatnecr osis
Caseousnecr osis
Fibri
noidnecr osis

17.Thedegr
adat
ionofi
ntr
acel
l
ularor
ganel
l
es
thr
ought heprocessinwhichautosomescombi
ne
wit
hpr i
mar ylysosomestoform
autophagolysosomesarecall
ed
Autophagy
Heterophagy
Heteropl
asmy
Homophagy
Endocytosi
s

18.Enzymat i
cdigesti
oni
sthepredomi
nantev
ent
i
nthef ol
lowingtypeofnecr
osi
s:
Coagulati
venecrosis
Li
quefactiv
enecr osi
s
Caseousnecr osi
s
Fatnecrosis
Gangrene

19.Apoptosi
shast hefoll
owi ngfeaturesexcept:
Thereiscel
lshri
nkageinapopt osi
s
Therearenoacuteinfl
ammat orycells
surr
oundingapoptosi
s
Theremaybesi ngl
ecell l
ossoraf fectcluster
sof
cel
ls
Apoptosi
sisseeninpat hologicprocessesonl y
Apoptosi
sisaform of« programmedcel ldeath»

20.Di
abeti
cfooti
sanexampl
eof
:
Drygangr
ene
Wetgangrene
Gasgangrene
Necrot
izi
nginf
lammati
on
Coagul
ati
venecrosi
s

21.Thecausat i
veor
gani
sm ofgasgangr
enei
s
Staphylococcus
Strept
ococcus
Leptospira
My cobacteri
um
Noneoft hese

22.Ar edinf
arcti
sleastl
i
kel
ytooccuri
n
Kidney
Lung
Heart
Brain
Alloftheseto

23.Theprocessofprogr
ammedgenedi rected
cel
ldeathcharact
eri
zedbycel
lshr
inkage,nucl
ear
condensati
onandfragment
ati
onisknownas–
Apoptosi
s
Chromatoly
sis
Pyknosi
s
Necrosi
s
Karyor
rhexi
s
24.Local i
zedareaofi
schemi
cnecr
osi
sismost
ly
associatedwith
Ascit
ese.
Petechiae.
Inf
arcti
on.
Embol iformat
ion.
Hemat oma.

25.Oneofmani fest
ati
onsofmet abol
i
c
derangement sincell
sis:
Apopt osis
The i ntracel
lul
ar accumulati
on of abnor
mal
amount sofvari
oussubstances
Hyper t
rophy
Met aplasi
a
Atr
ophy

26.Ear lypot enti


all
yreversibl
echangesi n
my ocar di
al cell
sinducedbyanoxi ai ncl
udeallof
thef ol
lowi ngcondi t
ions,except:
Fail
ur eofoxi dativ
ephosphor ylati
on
Depl et
ionofATP
I
nhibi t
ionofanaer obicglycol
ysisand
gly
cogenol ysis
Decr easei ncellul
arpH
I
ncr easei nintracell
ularNaandwat erandlossof
i
ntracel l
ularK
27.Whi chgroupoff act
or sismosti mpor tanti
n
thecell
ularpathogenesi sofi rreversiblecelli
njur
y?
Lipi
ddeposition, r
educedpr oteinsy nthesis,
nucleardamage
Mi t
ochondri
al condensat ion,glycolysis,sodium
celll
oss
Mi t
ochondri
al hyperplasia,lysozy mer elease,
membr aneinjur y
ReducedATP, increasedcal cium influx,
membr aneinjur y
Ribosomedet achment ,glycolysis,nuclear
damage

28.Whi choft hefoll


owingeventsismost
i
mpor tantint hedevelopmentofir
rev
ersi
blecel
l
i
njury?
Cellmembr anedamage
Increasedcel l water
My eli
nf igureaccumul ati
on
Lossofr ibosomes
Swel l
ingofmi t
ochondri
a

29Ther eversi
bleprocesscausedby
accumulationofgly
cosami nogl
ycansi
n
extr
acell
ularmatri
xduet otheincr
easeof
vascul
arper mi
abil
it
yi s:
Amy l
oidosis
Gl
ycogenoses
Hyal
inosi
s
Mucoidchanges
Fi
bri
noidchanges

30.All t
hepathol
ogi
cal
processesar
eir
rev
ersi
ble,
except :
Mucoi dchanges
Fibr
inoidchanges
Amy loidosi
s
Apopt osis
Necrosi s

31.Thef ol
lowingfeat
urescharact
eri
zer
ever
sibl
e
celli
njur
y,except:
Blebs
Endoplasmicreti
culum swel
li
ng
Dispersi
onofr i
bosomes
My el
infi
gures
Autophagy

32.Thef ol
lowingf eat
urescharact
eri
ze
i
rrever
siblecelli
njury,except
:
Nucleouspy knosis
Karyol
ysis
Karyorr
haxis
Endoplasmicr et
iculum swell
ing
Mitochondrialswelli
ng
33.Coagul ati
venecrosisischaracteri
zedbyal
lof
thefoll
owingpat hologicfeatur
es,except
:
Denaturati
onofcy toplasmi cpr
oteins
Karyorr
haxi s
Karyopiknosis
Breakdownofcel lor ganel
les
Lipi
ddeposi ti
on

34.Di gestionoft hecel


lbyl
ysosomeenzy
mesof
i
mmi gr antleukocytesi
ster
med:
Autoly sis
Apopt osi s
Heterol ysi
s
I
nflammat i
on
Met apl asi
a

35.Necr oti
ccel l
sar echaract
eri
zedbyal
loft
he
foll
owi ngfeatures,except:
Increasedeosi nophil
ia
Hy ali
ne-l
ikedropletsinthecytopl
asm
Glassyappear ance
Lysisofcy toplasm
Fragment ationofcy topl
asm

36.Oneoft hef
oll
owi
ngchangesi
ncel
l
sis
apoptot
ic:
Karyol
ysis
Nuclearpyknosi
s
Pl
asmol ysi
s
Breakdownof mil
ochondr
ias
Cell
ul.
irswell
ing

37.Oneoft hefoll
owingvar
iant
sofnecr
osi
scan
befoundint uber
culosi
s:
Caseousnecr osi
s
Gangrenousnecrosis
Li
quefacli
venecrosis
Fatnecrosi
s
Fi
bri
noidnecr osi
s

38.Oneoft hefol
lowingv
ari
ant
sofnecr
osi
sis
associ
atedwi t
hacutepancr
eat
it
is:
Coagulati
venecrosi
s
Li
quefacti
venecrosis
Caseousnecrosis
Gangrenousnecrosis
Fatnecrosi
s

39.Ischemi ci
njuryi
nthecent
ral
ner
voussy
stem
result
si n:
Liquefacti
venecrosi
s
Coagul ati
venecrosi
s
Caseousnecr osis
Gangrenousnecr osi
s
Fatnecr osi
s
40.Oneoft hefol
lowingvar
iant
sofnecr
osi
scan
befoundinwetgangr ene:
Coagulati
venecrosi
s
Li
quefacti
venecrosis
Caseousnecrosis
Gangrenousnecrosis
Fatnecrosi
s

41.Oneoft hefol
lowingv
ari
antsofnecrosi
scan
befoundingangr eneofl
owerext
remiti
es:
Coagulati
venecrosis
Li
quefacti
venecrosis
Caseousnecrosis
Gangrenousnecrosis
Fatnecrosi
s

42.Apoptosisischar act
eri
zedbyal l
ofthe
fol
lowingpathologicfeatur
es,except:
Nuclearchromat i
ncondensat i
on
Nuclearfr
agment ation
Cytopl
asmicbuddi ng
Organell
arswelli
ng
Phagocytosisoftheapopt oti
cbodies

43.Apopt
osi
soccur
sinal
loft
hef
oll
owi
ngev
ent
s,
except
:
Aging
Cel
l deathintumor
s
Deathofimmunecel l
s
Pathologi
catrophy
Cel
l autol
ysis

44.Apoptoti
cbodyi schar act
eri
zedbyal
loft
he
foll
owi except
ng, :
Eosinophi
li
ccytoplasm
Tightl
ypackedorganel l
es
Presenceofnuclearfragment s
Absenceofnuclearf r
agment s
Protei
ndroplet
sincy toplasm

45.Apoptot i
ccell i
schar acteri
zedbyalloft
he
fol
lowinghislologicalfeatures,except
:
Roundf orm
Oval f
orm
Eosinophil
iccy t
opl asm
Basophili
ccy toplasm
Densenucl earchr omat i
nf r
agment s

I
ntr
acel
l
ularaccumul
ati
ons(
Kozmi
na-
34q)

1.Increasedlipol
ysi
soff atst
ores,
whichcan
result
sf rom star
vat
ions,di
abetesmell
it
us,or
corti
costeroiduse,i
tmostl i
kel
ytocause
steatosis(fat
tyli
ver
)throughwhichoneofthe
l
i
stedmechani
sms?

Decreasedfr
eefat
tyaci
dexcr
etionfr
om t
hel
i
ver
l
eadsoff r
eefat
tyaci
daccumulati
onin
hepatocyt
es

ExcessNADH( highNADH/ NNADrat


io)causes
excessproductionofl
actatefr
ompyruvat
e,which
accumulatesinhepatocy
t es

I
ncr
easedfreef
attyaci
ddeli
veryt
othel
iverl
eads
t
otr
igl
ycer
ideaccumulati
oninhepat
ocy
tes

Inhi
biti
onofapoprotei
nsy nt
hesi
sbytheliver
l
eadst ophospholi
pidsaccumulat
ionin
hepatocytes
Inhi
biti
onofHMG- GoAr educt
asaacti
vit
yleadst
o
cholester
olaccumulat
ioninhepat
ocytes

2.A48- year-
oldmal ewhohasal onghi st
oryof
excessivedrinkingpr esentswithsi ngsof
alcoholi
chepat it
is.Mi croscopi
cexami nati
onofa
biopsyoft hi
spat ient’
sl i
verrevealsirr
egular
eosinophil
ichy ali
neincl usi
onswi thinthe
cytopl
asm oft hehepat ocytes.
Theseeosi nophi
l
ic
i
nclusionsarecomposedof
Immunoglobuli
n
Excesspl
asmapr ot
eins
Preker
ati
ninter
mediatef
il
aments
Basementmembr anemateri
al
Lipof
usci
n

3.I
nf at
tyli
vertochronicalcohol i
sm,t
hef
oll
owi
ng
mechanismsar einv ol
vedexcept :
I
ncreasedfreefattyacidsy nthesis
Decreasedtri
glyceri
deut i
l
ization
I
ncreasedα-glycerophosphat e
Bl
ocki nli
poprotei
nexcr et
ion
Decreasedfreefattyacidsy nthesi
s

4.Oneoft hepossi
blecausesofi
ntracel
lul
ar
accumul at
ionofmetabol
icsubst
ancesis:
Geneticdefect
s
Inf
lammat i
on
Embol i
sm
Necrosis
Activ
ationofoncogenes

5.Whatsubstancesar
eaccumulat
ingwi
thi
n
parenchymalcel
lsi
nsteat
osi
s?
Cholester
ol
Apoprolei
ns
Tri
glyceri
des
Vit
ami ns
Ket
onebodi
es

6.Fattychangeisoftensei
ningi
nal
loft
he
foll
owingorgans,except
:
Liver
Heart
Kidney
Muscles
Lung

7.Thecausesofst eat
osisi
ncludeall
oft
he
fol
lowingpathol
ogicstat
es,except
:
Obesity
Anoxia
I
nflammat i
on
Protei
nmal nut
ri
ti
on
I
ntoxicat
ion

8.Thestai
nusedtoident
if
yfati
s:
Hematoxyl
inandeosinst
ain
SudanII
Istai
n
Congoredstai
n
PASreacti
on
Metachr
omat i
cst
ain

9.Thestai
nusedtoident
if
ygly
cogeni
s:
Hematoxyl
inandeosinst
ain
SudanII
Istai
n
Congoredstai
n
PASreacti
on
Metachr
omat i
cst
ain

10.Thefat
tyl
i
verhasal
lpat
hol
ogi
cfeat
ures,
except:
Enlar
ged
2Yellow
Red
Soft
Greasy

11.Fattychangeisseenbyli
ghtmi
croscopyas:
I
ntracell
ulargr
anules
Basophili
cgranules
Extr
acellul
argranul
es
Vacuolesinthecy t
opl
asm ar
oundt
henucleus
Eosinophil
icgr
anules

12.Themostcommoncauseoff
att
ychangei
n
theheartis:
I
nflammat ion
Neoplasia
Hypoxia
Amy l
oidosis
Autoi
mmunedi seases

13.Fat
tychangei
nthehear
tchar
act
eri
zedby
:
Redheart
Smallsi
zehear
t
Ti
gerheart
Sol
idheart
Gooseheart

14.Cel l
s,whi chcanaccumul atecholest
erol
and
cholesterol est
ersinat heroscl
eroti
cplaque,
are:
Macr ophagesandsmoot hmuscl ecell
s
Leucocy tesandf ibroblast
s
Lymphocy tesander ythrocytes
Fi
brobl astsandl eucocy t
es
Eryt
hr ocytesandl ymphocy t
es

15.Thecel lsaccumul at
ingfatwithi
nthei
nti
mae
l
ay eroftheaor t
aandl argearteri
esin
atheroscl
eroticplaquesarecal l
ed:
Fi
br obl
asts
Epithel
ialcell
s
Foam cells
Lymphocy t
es
Leukocy t
es

16.Alloft
hefollowingmechanismscause
i
ntracel
lul
araccumul at
i except
on, :
Abnormal metaboli
sm
Protei
nmut at
ion
I
nflammat i
on
Enzy
medeficiency
I
ngesti
onofindigest
ibl
emat
eri
als

17.Ball
ooni
ngdegenerati
onofhepatocyt
esis
causedby:
Accumulati
onofwaterandcell
ularswel
li
ng
Retai
ningofbi
l
iar
ymat eri
al
Accumulati
onofi
ronorcoppersubstances
Accumulati
onoffatdropl
ets
Accumulati
onofl
ipofusci
n

18.Ballooningdegenerati
onofhepat
ocy
tes
resul
tsf rom:
Viral
hepat it
is
Alcoholicli
verdisease
Bil
iar
ymat eri
alaccumulat
ion
Obesity
Diabetesmel li
tus

19.Allofthefoll
owi ngfeat
ureschar
acteri
zethe
ball
ooningdegener ati
onofhepatocy
tes,except
:
Swollencell
s
Edemat ousappearance
Clumpedcy topl
asm
Largedropletsoffat
Largeclearspaces

20.Di
abet
esmel
l
itusi
schar
act
eri
zedbyt
he
accumul ati
onofgl y
cogeninall
cell
s,except
:
Epithel
ialcel
lsoftheproxi
mal t
ubul
es
Li
v ercell
s
b-cell
soft heisl
etsofLanger
hans
Smoot hmuscl ecells
Hear tmusclecell
s

21.
One of mani fest
ati
ons of met abol
ic
der
angement sincellsis:
Apoptosis.
B.I ntr
acell
ularaccumulati
onofabnor
malamount
s
ofvar
ioussubst ances.
Hypertr
ophy .
Metaplasia.
Atr
ophy .

22.
Mal l
ory’sbodi
esmaybef oundi
n:
Neurons.
Cardi
omy ocytes.
Hepatocytes.
Epi
theli
alcell
sofstomach.
Epi
theli
alcell
sofrenal
tubul
es.

23.Hyalinedropl
etsi
nrenal
tubul
arepi
thel
i
alcel
l
s
areseeni n:
Protei
nur i
a.
Li
piduria.
Hemat ur i
a.
Cy
li
ndrur
ia.
Di
spr
otei
nemia.

24.Russel ’
sbodiesmaybef
oundi
n:
Epitheli
oidcell
s.
Plasmacel l
s.
Lymphocy tes.
Histiocyt
es.
Mastcel ls.

25.Alzhei
mer di sease i s associat
ed wi
th
i
ntracell
ularaccumulat
ionsofpr
otei
nsin:
Li
v er.
Hear t
.
Spinalcord.
Brain.
Kidneys.

26.
Thest ainusedtoidenti
fyf
ati
s:
Hematoxy l
inandeosinstai
n.
RedoilO.
Congoredst ai
n.
PASreaction.
Metachromat i
cstai
n.

27.
Accumulat
ionofchol
est
erolandchol
est
erol
est
her
swi t
hf or
mati
onoftumorousmassesi s
cal
l
ed:
Atheroma.
Xantoma.
Adenoma.
Papil
loma.
Teratoma.

28.Xantoma i s a pat
hol
ogi
cal pr
ocess wi
th
accumul ati
onof:
Neutralfat.
Li
poids.
Cholesterol
.
Phosphol i
pids.
Li
popr otei
ns.

29.
Foam cel l
sarechar
act
eri
zedbyaccumul
ati
on
of:
Neutralfat.
Tri
glycerides.
Cholesterol.
Phosphol ipi
ds.
Li
popr otein.

30.Derangementofglycogenmet
abol
i
sm i
sseen
i
n:
Atheroscl
erosis.
Diabetesmelli
tus.
Viral
hepatiti
s.
Arter
ialhypert
ension.
Goi
ter
.

31.
Glycogenst or
agedi
seasei
scausedby
:
Geneti
cdi sorders.
Hypoxia.
I
nfect
ion.
I
mmunedi sorders.
I
ntoxi
cation.

32.
Muci
nousdegenerati
onisoccur
sin:
I
nfl
ammati
onofhear t
.
I
nfl
ammati
onofbrain.
I
nfl
ammati
onofbronchi.
I
nfl
ammati
onofspinalcord.
I
nfl
ammati
onofli
ver.

33.Tumor arising f
rom epithel
i
al cel
l
s wi
th
accumulat
ionofmuci ni
scal
led:
Mucinouster
at oma.
Mucinoussarcoma.
Mucinousmelanoma.
Mucinousneuroblast
oma.
Mucinouscarcinoma
Mi
scel
l
aneous3
8.Ty
phoi
df ev
eri
sanexampl
eof
:
Acut
einf
lammati
on
Chr
oni
cnonspeci
fi
cinf
lammat
ion
Chr
oni
cgr
anul
omat
ousi
nfl
ammat
ion
Chr
oni
csuppur
ati
vei
nfl
ammat
ion

9.Whi choneoftheli
stedstat
ementsisthebest
hi
stologicdefi
nit
ionofanabscess?
Acircumscribedcoll
ecti
onofneutr
ophil
swith
necroticcel
l
ulardebri
s
Alocal
izeddefectthatr
esult
sfr
om thesloughi
ng
ofnecr
oticofnecroti
cinfl
ammator
yt i
ssuefrom
thesur
faceofanor gan
Alocal
i
zedprol
i
fer
ati
onoff
ibr
obl
ast
sandsmal
l
bl
oodvessel
s
Anaggregateoft
woormor
eact
ivat
ed
macrophages
Theexcessi
vesecr
eti
onofmucusf
rom a
mucosalsur
face

10.
Thecar
dinal
signofi
nfl
ammat
ioncal
l
edr
ubor
i
smainlyt
heresultof:
Decr
easedint
erst
iti
alhydr
ost
ati
cpr
essur
e
Decr
easedv
ascul
arper
meabi
l
ityofcapi
l
lar
ies
I
ncr
easedv
ascul
arper
meabi
l
ityofv
enul
es
Vasoconst
ri
cti
onofmuscul
arar
ter
ies
Vasodi
l
atat
ionofar
ter
iol
es

11.Duringtheear
lyst
agesoft heinfl
ammator
y
response,hist
ami
ne-i
nducedincreasedv
ascul
ar
permeabili
tyismostl
ikel
ytooccurin

Ar
ter
ies
Pr
ecapi
l
lar
yar
ter
iol
es
Capi
l
lar
ies
Post
capi
l
lar
yvenul
es
1.Vei
ns

12.Whichoneoft hel
ist
edstatementsbest
descr
ibest
hepr ocesscal
ledchemotaxis?
Abnormalfusi
onofphagosomest oprimary
l
ysosomes
Att
achmentofchemical
stoext
racel
l
ularmat
eri
al
toi
ncreasephagocy
tosi
s
Di
lat
ionofbl
oodv
essel
sbychemot
her
apeut
ic
dr
ugs
Mov
ementofcel
l
stowar
dacer
tai
nsi
teorsour
ce
Transmi
grat
ionofcel
l
sfr
om bl
oodv
essel
int
o
ti
ssue

13.Duri
ngacut einfl
ammat ion,hi
stamine-i
nduced
i
ncreasedv ascul
arper meabil
it
ycausest he
for
mat i
onofexudat es( i
nfl
ammat oryedema).
Whichoneoft helistedcell
typesisthemostlikel
y
sourceofthehistami net
hatcausest hei
ncreased
vascul
arper meabili
ty?
Endothel
ialcell
s
Fi
brobl
ast
s
Ly
mphocy
tes
Mastcel
l
s
Neut
rophi
l
s

14.Whatt ypeofleukocyteactivel
yparti
cipat
esin
acuteinfl
ammat oryprocessesandcont ains
my el
operoxidasewithi
ni t
spr i
mary(azurophi
li
c)
granulesandalkali
nephosphat aseinit
s
secondary(specif
ic)granules?
Neutrophil
s
Eosi
nophi
l
s
Monocy
tes
Ly
mphocy
tes
Pl
asmacel
l
s

15.Duringtheinflammat oryr
esponse,
thepr
oper
orderofwhitecell ev
entsis:
Endotheli
aladherence, margi
nat
ion,
phagocy t
osis.chemot axi
s
Marginat
ion,
leukodi
apedesi
s,chemot
axi
s,
phagocyt
osis
Mar
ginal
i
on,
migr
ati
on,
chemot
axi
s,st
asi
s
Stasi
s,l
eukodi
apedesi
s,mar
ginat
ion
phagocyt
osis
Leukodi
apedesi
s,st
asi
s,degr
anul
ati
on

16.Themostrel
i
abl
eev i
denceofpurul
ent
i
nflammat
ionist
hepresencei
ntissueofwhi
chof
thefol
l
owing:
Lymphocy
tes
Cel
l
ularnecr
osi
s
I
ntr
acel
l
ularpi
gmentaccumul
ati
ons
Neut
rophi
l
s
Pl
asmacel
l
s
17.Fl
uidthatcol
lect
sduringacuteinfl
ammati
on
andthathasaproteincontentexceedi
ng3g/dl
andaspecifi
cgravit
yexceeding1.015isr
efer
red
toas:
Oedema
Ef
fusi
on
Tr
ansudat
e
Ser
um
Exudat
e

18.Theadher enceofneutr
ophi
lsandmonocyt
es
tothevascularendothel
i
um pri
ortomovement
i
ntotheextravascul
arspaceiscal
led:
Marginat
ion
Di
apedesi
s
Pav
ement
ing
Emi
grat
ion
Cl
ott
ing

19.Cell
st hatar
ecapableofphagocyt
osisof
part
iculat
emat t
erincl
udewhichofthefoll
owi
ng:
Neutrophil
s,macrophages
Ly
mphocy
tes,
mastcel
l
s
T-
cel
l
s,NK-
cel
l
s
Basophi
l
s,st
em cel
l
s
Endot
hel
i
alcel
l
s,pl
asmacel
l
s

20.Theunidir
ect
ionalmigr
ati
onofl
eukocy
tes
towardatargeti
srefer
redtoas:
Diapedesi
s
Chemot
axi
s
Opsoni
zat
ion
Endocy
tosi
s
Mar
ginat
ion

21.Allofthefoll
owi ngst atementsdescri
bing
l
eukocy t
eemi grat
ionf rom v essel
sinareasof
i
nflammat ionaretrue, except:
Leukocytespasst hroughgapsbet weenthe
vascul
arendot hel
ial cell
s
Neut
rophi
l
sar
ethef
ir
stcel
l
stoemi
grat
e
Lcukocyl
esdev
eloppseudopodst
oai
d
i
emi gr
ati
on
Li
pofuscinaccumul
ati
onaccompani
esl
eukocy
te
emigrat
ion
Accompany
ingl
ossofr
edcel
l
sispassi
ve
22.Inaninfl
ammatoryr
esponse,neutr
ophi
ls
rel
easemolecul
esthati
nduceall
oft hefol
l
owing
eff
ect except
s, :
Chemotaxisofmonocyt
es
Chemot
axi
sofl
ymphocy
tes
Degr
anul
ati
onofmastcel
l
s
I
ncreasedv
ascularper
meabi
l
ityi
ndependentof
hi
staminer
elease
Connect
ivet
issuedi
gest
ion

23.Mediat
orsofincr
easedvascul
arper
meabil
i
ty
i
nacuteinfl
ammat or
yresponsesi
ncl
udeal
lofthe
fol
lowi
ng,except
:
Leukot
ri
eneE4
Compl
ementcompl
exC5b-
9
Leukot
ri
eneC4
Br
ady
kini
n
Pl
atel
et-
act
ivat
ingf
act
or

24.Thefir
stcel
lstoarr
iveatt
heinj
uredar
eaint
he
i
nfl
ammat oryresponsearewhichofthef
oll
owi
ng:
Neutrophi
ls
Fi
brobl
ast
s
Ly
mphocy
tes
Macr
ophages
Fr
ythr
ocy
tes

25.Lysosomeswithi
nneutr
ophil
scont
ainal
lof
thefol
lowingenzy
mes,except
:
My el
operoxi
dase
Aci
dhy
drol
ases
Pr
oteases
I
nter
leuki
n-2
Cat
ioni
cpr
otei
ns

26.AcuteInfl
ammat i
onischaract
eri
zedbyal
lof
thefol
lowingfeat
ures,except
:
Relat
ivel
yshortdurat
ion
I
mmi
grat
ionofl
ymphocy
tesi
ntot
hei
njur
edar
ea
I
mmi
grat
ionofl
eukocy
tesi
ntot
hei
njur
edar
ea
Exudat
ionoff
lui
d
Exudat
ionofpl
asmapr
otei
ns

27.Majorfuncti
onsofmacr
ophagesar
eal
loft
he
fol
lowi
ng,except:
Phagocytefuncti
on
I
L-1-
synt
hesi
zi
ngf
unct
ion
Secr
etor
yfunct
ion
Ant
ibody
-sy
nthesi
zi
ngf
unct
ion
Heal
i
ngandr
epai
rfunct
ion

28.Theinf
lammat or
yr esponsel
eadst
oal
loft
he
fol
lowi
ng,except:
I
solati
onofinfect
edtissues
I
nact
ivat
ionofcausat
iveagent
s
Neut
ral
i
zat
ionoft
oxi
ns
Remov
alofdev
ital
i
zedt
issuedebr
is
Obesi
ty

29.Acut
einfl
ammat i
onincl
udesal
loft
he
fol
lowi
ngty except
pes, :
Purul
ent
Fi
bri
nous
Gr
anul
omat
ous
Ser
ous
Cat
arr
hal

30.Cardi
nal
signsofacut
einflammat
ion
i
ncludeal
lofthefol
l
owing,except
:
Local
heat
Redness
Pal
l
or
Swel
l
ing
Pai
n

31."Hair
yhear
t"i
sanexampleofwhi
choft
he
fol
lowingty
pesofinf
lammat
ion:
Purulent
Fi
bri
nous
Ser
ous
Cat
arr
hal
Hemor
rhagi
c

32.Purul
enti
nfl
ammat ioni
scharact
eri
zedby
whichofthefol
lowing:
Neutr
ophili
nfi
lt
rat
ionwitht
issuel
ysi
s
Fi
bri
ndeposi
ti
on
Mucuspr
oduct
ion
Ly
mphocy
tei
nfi
l
trat
ion
Neut
rophi
li
nfi
l
trat
ion

33.Themostcommonf
ormat
ioni
nthebody
cavit
iesandthespi
nalf
lui
diswhi
choft
he
foll
owing:
Fibri
nousinf
lammati
on
Ser
ousi
nfl
ammat
ion
Cat
arr
hal
inf
lammat
ion
Hemor
rhagi
cinf
lammat
ion
Gr
anul
omat
ousi
nfl
ammat
ion

34.Fibr
inousper
icar
dit
isoccur
sinal
ldi
seases,
except:
Acuterheumati
cfever
Sy
stemi
clupuser
ythemat
osus
Ur
emi
a
At
her
oscl
erosi
s
Acut
emy
ocar
dial
inf
arct
ion

35.Lobarpneumoni
aisanexampl
eofwhi
chof
thefoll
owingt
ypesofi
nfl
ammati
on:
Purulent
Fi
bri
nous
Ser
ous
Cat
arr
hal
Gr
anul
omat
ous

36.Abscessofthel
ungisanexampl
eofwhi
chof
thefoll
owingt
ypesofi
nfl
ammat i
on:
Purulent
Fi
bri
nous
Ser
ous
Cat
arr
hal
Gr
anul
omat
ous

37.Acutegastr
it
iswi
thabundantmucus
producti
onisanexampleofwhichoft
he
fol
lowingty
pesofinf
lammation:
Purulent
Fi
bri
nous
Ser
ous
Cat
arr
hal
Gr
anul
omat
ous

СHRONI
CINFLAMMATI
ON(
Dav
ydov–32Qs)
1.
Cel
lsofchr oni
cinf
lammat
ionar
e:
A.Neutrophil
s.
B.Ery
throcytes.
C.Monocy tes.
D.Myelocytes.
E.Adi
pocy t
es.

2.
Cel
lsofchr onici
nfl
ammat
ionar
e:
A.Lymphocy tes.
B.Ery
throcytes.
C.Neutrophil
s.
D.Myelocytes.
E.Adi
pocy t
es.

3.
Cel
lsofchr oni
cinf
lammat
ionar
e:
A.Neutrophil
s.
B.Pl
asmacel l
s.
C.Ery
throcytes.
D.Myelocytes.
E.Adi
pocy t
es.

4.
Cell
sofchr oni
cinf
lammat
ionar
e:
A.Neut rophil
s.
B.Erythrocytes.
C.My elocytes.
D.Adipocy tes.
E.Eosinophi l
s.
5.
Cel
lsofchr oni
cinf
lammat
ionar
e:
A.Neutrophil
s.
B.Eryt
hrocytes.
C.Astrocyt
es.
D.Giantcell
s.
E.Osteocytes.

6.
Cell
sofchr onici
nfl
ammat
ionar
e:
A.Neutrophils.
B.Epit
elioi
dcells.
C.Epit
hel i
alcel
ls.
D.Astrocytes.
E.Osteocy t
es.

7.
Cel
lsofchr oni
cinfl
ammat
ionar
e:
A.Neutr
ophi l
s.
B.Ery
throcytes.
C.Mastcells.
D.Epi
theli
alcells.
E.Chondrocytes.

8.
Cel
lsofchr onicinfl
ammat
ionar
e:
A.Fi
brolasts.
B.Osteoblasts.
C.Chondrobl asts.
D.Li
poblasts.
E.Lymphobl asts.
9.
Functi
onsofmacr ophagesar
eal lt
hef
oll
owi
ng,
EXCEPT:
A.Phagocytosi
s.
B.Degradat
ionofengul f
edpar
ti
cles.
C.Tissuedestr
ucti
on.
D.Formingofgiantcel
ls.
E.Produci
ngofmedi ators.

10.
Functi
onsofmacr ophagesar
eallt
hef
oll
owi
ng,
EXCEPT:
A.Phagocytosi
s.
B.Degradati
onofengulfedpart
icl
es.
C.Tissuedestr
ucti
on.
D.Syntesi
sogcol l
agen.
E.Formingofgiantcel
ls.

11.
Functi
onsofmacr ophagesar
eallt
hef
oll
owi
ng,
EXCEPT:
A.Phagocytosi
s.
B.Degradat
ionofengulfedpart
icl
es.
C.Produci
ngofant i
bodies.
D.Tissuedestr
ucti
on.
E.Formingofgiantcel
ls.

12.
Funct
ionsofmacr
ophagesar
eal
lthef
oll
owi
ng,
EXCEPT:
A.Phagocytosi
s.
B.Regenerati
onofepit
heli
alcel
ls.
C.Degradati
onofengulfedpart
icl
es.
D.Tissuedestr
ucti
on.
E.Formingofgiantcel
l
s.

13.
Whi chcellsareassoci
atedwi
thpr
oduci
ngof
mediators?
A.Ly mphocy tes.
B.Plasmacel l
s.
C.Eosi nophil
s.
D.Mustcel l
s.
E.Fibroblasts.

14.
Whi chcellsar echar
acter
ist
ical
l
yfoundin
i
nfl
ammat orysitesaroundanimalpar
asit
es?
A.Ly mphocy tes.
B.Plasmacel l
s.
C.Eosi nophil
s.
D.Mustcel l
s.
E.Fibroblasts.

15.Whichcel
l
sar ecent
ral
play
ersi
nanaphy
lact
ic
shock?
A.Ly mphocytes.
B.Plasmacells.
C.Eosinophi
ls.
D.Mustcell
s.
E.Fi
brobl
asts.

16.
Whi chcellsarecol
l
agenpr
oduci
ng?
A.Ly mphocy tes.
B.Plasmacel l
s.
C.Eosi nophil
s.
D.Mustcel l
s.
E.Fibroblasts.

17.
Inf
ectivegranulomasoccuri
n:
A.Tuberculosis.
B.Rheumat icfever.
C.Rheumat oidarthri
ti
s.
D.Sarcoidosis
E.Regionalil
eiti
s.

18.
Inf
ectivegranulomasoccuri
n:
A.Rheumat icfever.
B.Syphil
is.
C.Rheumat oidarthr
iti
s.
D.Sarcoidosis
E.Regionalil
eit
is.

19.
Inf
ect
ivegr
anul
omasoccuri
n:
A.Rheumat icfever.
B.Rheumat oidarthr
iti
s.
C.Leprosy.
D.Sarcoidosis
E.Regionalil
eit
is.

20.
Inf
ectivegranulomasoccuri
n:
A.Rheumat icfever.
B.Rheumat oidarthr
iti
s.
C.Sarcoidosis.
D.Typhoidfev er
.
E.Regionalil
eiti
s.

21.
All
ergicgranul
omasoccuri
n:
A.Tuberculosi
s.
B.Syphil
is.
C.Rheumat icfev
er.
D.Typhoidfever.
E.Leprosy.

22.Al
lergicgranulomasoccuri
n:
A.Tuber culosis.
B.Syphilis.
C.Rheumat oidarthr
it
is.
D.Typhoi dfever.
E.Leprosy .

23.
Granul
omaswi
thunknounet
iol
ogyoccuri
n:
A.Tuberculosis.
B.Syphil
is.
C.Rheumat icfever
.
D.Sarcoidosis.
E.Leprosy.

24.
Granulomaswi thunknounet
iol
ogyoccuri
n:
A.Tuberculosi
s.
B.Syphil
is.
C.Rheumat icfever
.
D.Regionalil
eiti
s.
E.Leprosy.

25.I
ntersti
ti
alinfl
ammat i
onischar
acter
izedby:
A.For mationofgr anulomas.
B.Infl
ammat oryinfi
lt
rati
onofthestr
omaofi nner
organs.
C.For mationofpseudopol yps.
D.Cel ldegenerati
onofi nneror
gans.
E.Format ionofcondy lomas.

26.
Typicall
ocal
i
zati
onofi
nter
sti
ti
ali
nfl
ammat
ion
i
sallthefol
lowi
ng,
EXCEPT:
A.Liver.
B.Kidneys.
C.Hear t
.
D.Lungs.
E.Spleen.
27.
Typicall
ocal
i
zati
onofi
nter
sti
ti
ali
nfl
ammat
ion
i
sallthefol
lowi
ng,
EXCEPT:
A.Liver.
B.Kidneys.
C.Hear t
.
D.Lungs.
E.Lymphnodes.

28.
Typicall
ocal
i
zati
onofi
nter
sti
ti
ali
nfl
ammat
ion
i
sallthefol
lowi
ng,
EXCEPT:
A.Liver.
B.Kidneys.
C.Hear t
.
D.Lungs.
E.Stomach.

29.
Typicall
ocali
zati
onofi
nfl
ammat
orypol
ypsi
s:
A.Smal li
ntest
ine.
B.Largeintest
ine.
C.Rectum.
D.Duodenum.
E.Appendix.

30.
Typicall
ocalizat
ionofi
nfl
ammat
orypol
ypsi
s:
A.Stomach.
B.Smal li
ntest
ine.
C.Largeintest
ine.
D.Duodenum.
E.Appendi
x.

31.
Typicallocali
zati
onofi
nfl
ammat
orypol
ypsi
s:
A.Oralcav i
ty.
B.Nasal cavit
y.
C.Pleuralcavit
y.
D.Perit
oneal cavit
y.
E.Jointcav i
ty.

32.
Inf
lammatorywart-
li
kelesi
oni
scal
l
ed:
A.Papil
loma.
B.Adenoma.
C.Condy l
oma.
D.Pseudopol
yp.
E.Carci
noma.
(Kozmina-48)

1.Morphol
ogicchangesseeni
nchroni
c
non-
specif
ici
nfl
ammat ioni
ncl
udeanincreasei
n:
Neutr
ophil
s,l
ymphocytesandl
iquef
acti
on
necr
osis.
Neut
rophi
l
s,macr
ophagesandf
ibr
osi
s.
Ly
mphocy
tes,
plasmacel
l
sandf
ibr
osi
s.
Giantcel
ls,
macr
ophagesandcoagul
ati
ve
necrosi
s.
2.Inchr
onicinf
lammat i
on,macr ophages
accumulat
einlargenumberinthef ol
lowingways:
Conti
nuousinfl
owofmonocy t
esf rom ci
rcul
ati
on
maintai
nedbychemot acti
cfactors.
Localpr
oli
fer
ati
onofmacr
ophagesbymi
tot
ic
di
visi
on
Prol
ongedi
mmobil
i
zationofmacr
ophageswi
thi
n
thesi
teofi
nfl
ammati
on
AandB
Al
lthet
hree

3.For
mationofgr
anulomais:
TypeIhy
persensi
ti
vi
tyreact
ion
Ty
peI
Ihy
per
sensi
ti
vi
tyr
eact
ion
Ty
peI
IIhy
per
sensi
ti
vi
tyr
eact
ion
Ty
peI
Vhy
per
sensi
ti
vi
tyr
eact
ion
Al
loft
hese

4.Granulomafor
mationi
smostf
requent
ly
associat
edwit
h:
Thehealingpr
ocess.
Acut
einf
lammat
ion.
Woundcont
ract
ion.
Fi
brobl
ast
sandneov
ascul
ari
zat
ion.
Aper
sist
enti
rr
it
ant
.

5.Anexampleofchr
oni
cfi
bri
noi
dinf
lammat
ioni
s:
Chroni
ccer
vici
ti
s
Chr
oni
cgast
ri
ti
s
Chr
oni
cchol
ecy
sti
ti
s
Noneoft
hese
Al
loft
hese

6.Epithel
i
oidcel
l
sint
uber
cul
argr
anul
omaar
e
deri
vedf r
om:
Neutrophil
s
Eosi
nophi
l
s
Ly
mphocy
tes
Monocy
tes
Macr
ophages

7.Epi
thel
i
oidcel
li
smodi
fi
ed–
Lymphocytes
Macr
ophages
Mastcel
l
Eosi
nophi
l
s
Neut
rophi
l
s

8.Gr
anul
omai
sseeni
nal
l
,except
:
TB
Yer
sini
a
My
copl
asma
Lepr
osy
Sy
phi
l
is

9.Whichofthefol
l
owingi
sat
ypi
cal
mycobacter
ium?
Mycobacter
ium mi
crot
i
My
cobact
eri
um cannet
i
My
cobact
eri
um af
ri
canum
My
cobact
eri
um ul
cer
ans

10.I
gM anti
bodyagai
nstPGL-
1ant
igeni
sused
fort
hediagnosi
sof:
Lepr
osy
Tuber
cul
osi
s
Sy
phi
l
is
Br
ucel
l
osi
s
My
copl
asmosi
s

11.Whichcat
egoryofleprosyi
snoti
ncl
udedi
n
Ridl
ey-
Jopl
ingclassi
fi
cati
on?
Midborder
li
neleprosy
Bor
der
li
net
uber
cul
oidl
epr
osy
I
nder
ter
minat
elepr
osy
Tuber
cul
oidpol
arl
epr
osy
Bor
der
li
nel
epr
omat
ous

12.Heparl
obatum i
sseeni
n:
Pri
marysyphi
li
s
Secondar
ysy
phi
l
is
Ter
ti
arysy
phi
l
is
Congeni
tal
syphi
l
is

13.Kil
l
ingofM.t
ubercul
osi
sthatgrowswit
hinthe
macrophagei
sbroughtaboutbythefol
l
owing
mechanisms:
Byreacti
veox
ygenspecies
Byoxy
gen-
independentbact
eri
cidal
mechani
sm
Byni
tr
icoxi
demechani
sm
Byhy
drol
yti
cenzy
mes

14.Tubercl
ebaci
ll
icausel
esi
onsbyt
hef
oll
owi
ng
mechanisms:
El
aborati
onofendotoxi
n
El
abor
ati
onofexot
oxi
n
Ty
peI
Vhy
per
sensi
ti
vi
ty
Di
rectcy
tot
oxi
cit
y

15.Thefoll
owingst at
ement
sarecor
rectf
or
tuber
clebaci
ll
i
, except:
Tubercl
ebacil
licanbecult
ured
Tuber
clebaci
l
liar
eanaer
obe
Tuber
clebaci
l
lit
hri
vebesti
ntheapexofl
ung
M.
smegmat
isi
snotpat
hogeni
ctoman

16.Tuber
clebaci
ll
ii
ncaseousl
esi
onsar
ebest
demonstr
atedin:
Caseouscentr
e
Mar
ginofnecr
osi
swi
thv
iabl
eti
ssue
Epi
thel
i
oidcel
l
s
Langhans′
giantcel
l
s
17.Leprosybaci
l
liar
e:
Notacidfast
Asaci
dfastast
uber
clebaci
l
li
Lessaci
dfastcompar
edt
otuber
clebaci
l
li
Mor
eaci
dfastcompar
edt
otuber
clebaci
l
li

18.Lepr
omi nt
estisal
way
sposi
ti
vei
n:
Lepromatousl
eprosy
Bor
der
li
nel
epr
omat
ousl
epr
osy
Tuber
cul
oidl
epr
osy
I
nder
ter
minat
elepr
osy

19.Spi
rochet
esar
emostdi
ff
icul
ttodemonst
rat
e
i
n:
Pri
marysyphil
i
s
Secondar
ysy
phi
l
is
Ter
ti
arysy
phi
l
is
Congeni
tal
syphi
l
is

20.Act
inomy
cosi
siscausedby
:
Fungus
Gr
am-
negat
ivebact
eri
a
Anaer
obi
cbact
eri
a
Aci
dfastbact
eri
a

21.Typical
ly,
sarcoidgranul
omahast
hef
oll
owi
ng
feat
ures,except
:
Noncaseat i
nggr anul
oma
Gi
antcel
l
shav
ecy
topl
asmi
cincl
usi
ons
Per
ipher
almant
leofl
ymphocy
tes
Fi
brobl
ast
icpr
oli
fer
ati
onatt
heper
ipher
yofa
gr
anuloma

22.Bydefini
ti
on,gr
anul
omasar
ecomposedof
Cholest
erolcl
eft
s
Col
l
agen
Endot
hel
i
alcel
l
sandf
ibr
obl
ast
s
Epi
thel
i
oidcel
l
s
Hemosi
der
in-
ladenmacr
ophages

23.Findingacid-
fastbaci
ll
iwi
thi
nper
ipher
al
nervesismostsuggest i
veof
Relapsingfever
Sy
phi
l
is
Lepr
osy
Tuber
cul
osi
s
Wei
l
′sdi
sease

24.A21- year
-ol
dcollegeat hletepresent switha
naggingcoughanda20- l
bwei ghtloss.Inadditi
on
tothechroniccoughandwei ghtloss,hismai n
sympt omsconsistoff ever,nightsweat s,and
chestpains.Examinationofhi ssput um reveals
thepresenceofrareaci d-f
astor ganisms.Hi s
sympt omsar emostlikelyduet oani nfecti
onwi t
h
K.pneumoni ae
L.pneumophi
l
a
My
cobact
eri
um av
ium-
int
racel
l
ular
e
My
cobact
eri
um t
uber
cul
osi
s
My
copl
asmapneumoni
a

25.Themostr el
iabl
ehist
opathol
ogicevi
denceof
chroni
cit
yinaninfl
ammat or
yprocessinorgansi
s
whichofthefoll
owing:
Hemor r
hages
Leucocy
tici
nfi
l
trat
es
Bl
oodv
essel
sdest
ruct
ion
I
nter
sti
ti
alf
ibr
osi
s
Counci
l
man'
sbodi
es
26.Alargeaggregat
eofepit
hel
ioi
dcell
sisseeni
n
ami cr
oscopicsecti
onofanovaryremovedat
surger
y.Yourdiagnosi
sis:
Granul
ationti
ssue
Py
ogeni
cgr
anul
oma
Gr
anul
osacel
ltumor
Gr
anul
ocy
tosi
s
Gr
anul
oma

27.Thegranulomai
npri
mar yt
ubercul
osi
sis
composedpr edomi
nant
lyofwhichoft
he
foll
owing:
Fibrobl
ast
s
Epi
thel
i
oidcel
l
s
Eosi
nophi
l
s
Pl
asmacel
l
s
Neut
rophi
l
s

28.Thegr anul
omatouscell
inf
il
trat
einpr
imary
syphil
isiscomposedpredominantl
yofwhichof
thefoll
owing:
Neutrophil
s
Monocy
tes/
macr
ophages
Pl
asmacel
l
s
Eosi
nophi
l
s
Ly
mphocy
les

29.Themil
iar
ylungtuber
culosi
sischar
act
eri
zed
bywhicht
ypeofinfl
ammat i
on:
Granul
omatous
Ser
ous
Fi
bri
nous
Suppur
ati
ve
Hemor
rhagi
c

30.Mostcompr ehensiv
elyt hechronic
i
nflammat ionischaracteriz
edbywhi choft
he
fol
lowing:
I
nfilt
rati
onwi t
hmononucl earcell
includi
ng
macr ophages,ly
mphocy t
es, andplasmacell
s
Ti
ssuedest
ruct
ion
Heal
i
ngbyconnect
ivet
issuewi
thangi
ogenesi
s
andf
ibr
osi
s
Al
loft
hese
Noneoft
hese
31.Thepossiblecausesofchroni
cit
yof
i
nflammat i
onarealloft
hefoll
owing,except
:
Persi
stenti
nfecti
onsbycertai
nmicroorgani
sms
Pr
olongedexposuretopotent
ial
l
ytoxi
cagent
s,
ei
therexogenousorendogenous
Aut
oimuni
ty(
aut
oimmunedi
seases)
Compl
etephagocy
tosi
s
Resi
stanceoft
heet
iol
ogi
cagent

32.Whatcel l
spl
aythemosti
mportantr
olei
n
chronictuber
cul
osi
sinf
lammati
on?
Macr ophages
Leucocy
tes
Eosi
nophi
l
s
Er
ythr
ocy
tcs
Pl
asmacel
l
s

33.All
oft
hesecel
lscanbef oundi
nchr
oni
c
i
nflammat
ioni
nfi
lt
rat
e,except:
Lymphocy
tes
Pl
atel
ets
Macr
ophages
Pl
asmacel
l
s
Eosi
nophi
l
s

34.Granulomatousinfl
ammationdev
elopsi
nal
l
ofthefol
lowingdi except
seases, :
Tubercul
osis
Lepr
osy
Sy
phi
l
is
Cat
-scr
atchdi
sease
Budd-
Chi
arysy
ndr
ome

35.Whatt ypeofnecr osi


scanbef
oundi
n
tuber
culosi
sgr anuloma?
Coagulati
onnecr osis
Li
quef
act
ivenecr
osi
s
Caseousnecr
osi
s
Enzy
mat
icf
atnecr
osi
s
Fi
bri
noi
dnecr
osi
s

36.Typi
calty
bercul
ousgranul
omais
charact
eri
zedbyall
ofthefoll
owi except
ng, :
Plasmacell
s
Ar
eaofcent
ral
necr
osi
s
Epi
thel
i
oidcel
l
s
Langhans-
typegi
antcel
l
s
Ly
mphocy
tes

37.Sy
phi
li
sgr
anul
omai
sal
socal
l
edas:
Fi
broma
Gumma
Tuber
cul
oma
Lepr
oma
Hepat
oma

38. Ty
picalsyphil
i
sgr anul
omai
schar
act
eri
zedby
al
l ofthefol
lowing,except
:
Areaofcent r
alnecrosis
Pl
asmacel
li
nfi
l
trat
e
Ly
mphocy
tei
nfi
l
trat
e
Pr
oduct
ivev
ascul
i
tis
Pl
atel
eti
nfi
l
trat
e

39.Macrophagesingr
anul
omatousinf
lammati
on
cantr
ansformintowhi
chofthefol
l
owingcel
ls:
Monocytes
Epi
thel
i
alcel
l
s
Epi
thel
i
oidcel
l
s
Pl
asmacel
l
s
Ly
mphocy
tes

40.Gummatousi
nfi
lt
rateinter
ti
arysyphil
i
scan
befoundi
nwhichofthefoll
owingorgans:
Aort
a
Test
es
Li
ver
Bonesandj
oint
s
Ski
nandsubcut
aneoust
issue

41.Ongr ossinspecti
on,sy
phil
it
icgummai s
characterizedbyallofthefol
l
owingfeat
ures,
except:
White-gray
Rubber
y
Sol
i
tar
y
Red-
brown
Tumor
-l
ike

42.Pr
oduct
srel
easedbyt
heact
ivat
ed
macrophagesthatleadt
oti
ssuei
njur
y,i
ncl
udeal
l
oft
hef ol
lowi
ng,except:
Fi
brogeni
ccytokines
Tox
icoxy
genmet
abol
i
tes
Col
l
agenases
Neut
rophi
l
echemot
act
ivef
act
ors
El
ast
ase

43.Productsrel
easedbytheacti
vat
ed
macrophages, t
hatl
eadtofi
brosi
sincl
udeal
lof
thefol
lowi except
ng, :
Growthfactor
s
Fi
brogeni
ccy
toki
nes
Angi
ogenesi
sfact
ors
Fi
bronect
in
Pr
oteases

44.Thelungswit
hmulti
plet
uber
cul
ous
granul
omasarecall
edas:
Tubercul
ouspneumoni
a
Br
owni
ndur
ati
onofl
ungs
Mi
l
iar
ytuber
cul
osi
s
Cav
itar
yfi
brocaseoust
uber
cul
osi
s
Tuber
cul
oma

45.Thepathologicchangesofvasav
asorum of
aort
ainsyphil
iti
cmesaor t
it
isar
echaract
eri
zedby
whichofthefoll
owing:
Migrat
orythr
ombophl ebit
is
Pr
oduct
ivev
ascul
i
tis(
obl
i
ter
ati
veendoar
ter
it
is)
Thr
omboangi
ti
sobl
i
ter
ans
Necr
oti
zi
ngar
ter
iol
i
tis
Thr
ombot
icmi
croangi
opat
hy

46.Themedialdestr
uct
ionofaortai
ntert
iar
y
syphi
li
smayleadt owhi
choft hefol
l
owing:
Aneuri
smaldil
atat
ionoftheaort
a
Mar
ian'
ssy
ndr
ome
At
her
oscl
erosi
s
Takay
asu'
sar
ter
it
is
Gi
antcel
lar
ter
it
is

47.Sar
coi
disi
sdevelopmenti
sassoci
atedwi
th
whichoft
hefoll
owing:
Mycobacl
eri
um l
eprae
My
cobact
eri
um t
uber
cul
osi
s
Tr
eponemapal
l
idum
Gr
am-
negat
ivebaci
l
li
s
Unknown

48.Theforei
gn-bodygranul
omasar
ecausedby
al
lthefoll
owing,except
:
Panicul
atematter
Sy
nthet
icmat
eri
al
Gr
am-
negat
ivebaci
l
lus
Veget
abl
emat
ter
Ber
yll
i
um par
ti
cles

ADAPTATI
ON(
Dav
ydov-24Qs)

1.
Typeofcel lularadapt
ati
onwhichoccur
sin
decr
easeofcel lsi
zeiscal
led:
A.Atrophy.
B.Hy pertr
ophy .
C.Hy perplasia.
D.Met aplasia.
E.Dy spl
asia.
2.
Typeofcel lularadaptat
ionwhi
choccur
sin
i
ncreaseofcel lsizeiscal
led:
A.At rophy .
B.Hy per tr
ophy .
C.Hy per plasia.
D.Met aplasia.
E.Dy splasia.

3.
Typeofcel lularadapt
ati
onwhi
choccur
sin
i
ncreasei ncel lnumberiscal
l
ed:
A.At rophy .
B.Hy per tr
ophy .
C.Hy per plasia.
D.Met aplasia.
E.Dy splasia.

4.Typeofcel lularadapt
ationwhi
choccur
sin
changeofcel ltypeiscal
led:
A.At rophy .
B.Hy per tr
ophy .
C.Hy perplasia.
D.Met aplasia.
E.Dy splasia.

5.Mechanismsofadapt i
veresponseinv
olveallof
thefol
lowing,EXCEPT:
A.Up- regul
ationofspeci
fi
ccellul
arr
eceptors.
B.Down- r
egulati
onofspecif
iccel
lul
arreceptor
s.
C.Al t
erat
ioni
nsignal
forprot
einsynthesis.
D.Inducti
onofnewprotei
nsynthesi
sbyt hetar
get
cel
l
.
E.Reducti
onofnewprotei
nsynthesi
sbyt hetar
get
cel
l
.

6.Mechanismsofadapt iveresponsei nvolveallof


thefol
lowing, EXCEPT:
A.Up- regulationofspecifi
ccellularreceptors.
B.Down- regulati
onofspeci f
iccellularreceptors.
C.Al t
erationi nsignalf
orproteinsy nthesis.
D.Al t
er at
ioni nsignalf
orli
pidsy nthesis.
E.Induct i
onofnewpr otei
nsy nt
hesi sbyt hetarget
cell
.

7.
Oneofv ar
iant
sofphy siol
ogicat rophyis:
A.Atrophyofbraininatherosclerosis.
B.Atrophyofthymusi nadults.
C.At
rophyofskelet
almusclebyi mmobi li
zedbroken
l
imb.
D.Atrophyofkidneyfrom pressur ebystones.
E.At
rophyofthyroi
dglandindi sfunctionofpi
tuit
ary
gl
and.

8.Mechanismsofadapt i
veresponseinv
olveallof
thefol
lowing,EXCEPT:
A.Up- regul
ationofspeci
fi
ccellul
arr
eceptors.
B.Down- r
egulati
onofspecif
iccel
lul
arreceptor
s.
C.Al t
erat
ioni
nsignalf
orprotei
nsynthesis.
D.Alterat
ioni
nsignal
forcarbohydr
atesynthesis.
E.Inducti
onofnewprotei
nsy nt
hesi
sbyt hetarget
cel
l
.

9.
Atr
ophyofor ganduet oprol
ongeddi
mini
shed
off
uncti
onal activi
tyiscal
led:
A.Neur opathicatrophy.
B.Endocr i
neat rophy.
C.Ischemicat rophy.
D.Disuseat rophy.
E.Pressureat rophy.

10.
Atr
ophyofor ganduet ogr
adual
dimi
nishedof
bl
oodsupplyi scalled:
A.Neur opathicatrophy.
B.Endocr i
neat rophy.
C.Ischemicat rophy .
D.Disuseat rophy.
E.Pressureat rophy.

11.
Atr
ophyofor ganwithnoobv
iouscausei
s
cal
l
ed:
A.Idiopat
hicatrophy
.
B.Endocrineatrophy
.
C.Ischemicatrophy.
D.Disuseatrophy.
E.Pr
essur
eat
rophy
.

12.Whichpi gmentcanbef oundincy


topl
asm of
heartandl iveringener
alat
rophy
?
A.Mel anin.
B.Hemosi derin.
C.Bi li
rubin.
D.Fer rit
in.
E.Li pofusci n.

13.Whichpi gmentcanbeaccumul at
edi
n
subcutaneousf atingener
alat
rophy?
A.Li pochr ome.
B.Lipof uscin.
C.Bi li
rubin.
D.Fer rit
in.
E.Hemosi der
in.

14.
Generalatrophy(cachexi
a)duet
obr
ain
pat
hol
ogyi scalled:
A.Ali
ment ary.
B.Endocr i
ne.
C.Cerebral.
D.Cancer ous.
E.Inf
ectious.

15.
Gener
alat
rophy(
cachex
ia)duet
ost
arv
ati
oni
s
cal
l
ed:
A.Ali
mentary.
B.Endocri
ne.
C.Cerebr
al.
D.Cancerous.
E.Inf
ecti
ous.

16.
Morphologi
calsi
gnsofat rophyar
e,EXCEPT:
A.Shri
nkageincellsize.
B.Reductionofmyof i
laments.
C.Reductionofendopl asmi
creti
cul
um.
D.Celldead.
E.Reducti
onofmi tochondri
a.

17.
Oneofvar
iant
sofphysi
ologichypert
r ophyis:
A.Enlar
gementofut
erusi
nl ei
omy oma.
B.Enl
argementofut
erusi
nl ei
omy osarcoma.
C.Enlar
gementofut
erusi
nov ari
ant umor s.
D.Enlar
gementofut
erusi
npr egnancy.
E.Enl
argementofut
erusi
nendomy omet ri
ti
s.

18.Concentr
iccardiachy pertr
ophyis
char
acteri
zedbyal lofthefollowing,
EXCEPT:
A.Hy pert
rophyofleftv entr
icle.
B.Increasedwal l
thickness.
C.Nor mal l
eftcavi
tydiamet er .
D.Reducedl eftcavit
ydi amet er.
E.Di
l
atedl
eftcav
itydi
amet
er.

19.Excentri
ccardiachy pertrophyischar
act
eri
zed
byall
oft hefol
lowing, EXCEPT:
A.Hy pertr
ophyofl eftv entr
icl
e.
B.Increasedwal lthickness.
C.Nor mal l
eftcavitydiamet er.
D.Di l
atedbordersofhear t.
E.Reducedl eftcav i
tydi ameter.

20.Hy
pertr
ophyofoneofpai r
edorganaf
ter
removalofanotherorgani
scall
ed:
A.Concent ri
c.
B.Excentric.
C.Controlater
al.
D.Endocr i
ne.
E.Ov er
wor kl
oading.

21.
Gener al
signsofhy pert
rophyare,EXCEPT:
A.Enlargementofcel ls.
B.IncreasedsynthesisofDNA.
C.Decr easedsynthesisofRNA.
D.Increasedproteinsy nt
hesis.
E.Incr
easednumberofor ganel
les.

22.
Endometr
ialhy
per
plasi
aisar
esul
tof
:
A.Chroni
cinfl
ammation.
B.Hormonal sti
mulationbyestrogens.
C.Hormonal sti
mulationbythyroi
dstimulat
ing
i
mmunoglobulins.
D.Acutei
nflammat ion.
E.Hormonal sti
mulati
onbypr ogesteron.

23.
Hy perplasi
aofepidermi
sandformati
onofskin
wart
sisar esultof:
A.Hor monal sti
mulati
onbyestr
ogens.
B.Hor monal sti
mulati
onbyprogest
eron.
C.Hor monal sti
mulati
onbythyr
oidsti
mulat
ing
i
mmunogl obulins.
D.Chr onicinf
lammat i
on.
E.Acut einfl
ammat i
on.

24.Hyperplasiaofhepat iccel l
sthatoccur
saf
ter
par
ti
alhepat oect
omyi sanexampl eof:
A.Compensat oryhy perplasia.
B.Hormonal hyperplasia.
C.Infl
ammat oryhy perplasia.
D.Phy si
ologichy perplasia.
E.Cont r
olateralhyperplasia.

(
Kozmi
na–24qs)

1.Thecell
ularadapt
ati
onwit
houtcel
l
prol
if
erat
ionincl
udesal
loft
hefol
lowi
ng,
EXCEPT:
Apoptosi
s
At
rophy
Hyperplasi
a
Hypertr
ophy
Metaplasia

2.Hyperpl
asiaischar
acteri
zedbywhichoft
he
fol
l
owing:
I
ncreaseinthesizeofcell
s
I
ncreaseinthenumberofcel l
s
I
ncreaseinthenumberofnucl ei
incel
ls
Shri
nkageinthesizeofcell
s
Atypi
aofcells

3.Theprolif
erati
onoft heglandul
arepi
thel
i
um of
afemaledur i
ngpr egnancyisanexampleof:
Compensat oryhy perplasi
a
Pathol
ogichy perplasia
Hormonal hyperplasia
Compensat oryhy pertr
ophy
Hormonal hypertrophy

4.Thehyperplasiaofhepati
ccel
l
st hatoccurs
aft
erpart
ialhepat oectomyi
sanexampleof :
Pathol
ogi
chy perplasi
a
Hormonal hyperplasia
Hor
monal hypert
rophy
Compensatoryhyperpl
asia
Compensatoryhypertr
ophy

5.Conditi
onsleadingt oendomet ri
alhyper
plasi
a
i
ncludeallofthefoll
owi ng,EXCEPT:
Polycyst
icovari
andi sease
Functi
oninggranulosacel l
tumor softheovary
Prol
ongedadmi nistr
at i
onofestrogenic
substances
Excessiveovari
ancor ti
calfunct
ion
Prol
ongedadmi nistr
at i
onofanalgesic
substances

6.Themostcommoncl i
nical
mani
fest
ati
onof
endometri
alhyperpl
asiais:
Abnormaluter
inebleeding
Mensesstopping
Pains
Purul
entdi
scharges
Mucousdischarges

7.Simpleendometr
ialhyperpl
asi
ais
charact
eri
zedbyallofthefoll
owi
ng,EXCEPT:
Incr
easeinthenumberandsi zeofendomet
ri
al
gl
ands
Atypi
aofglandcel ls
Complexendomet r i
alglands
I
ncreaseingland-to-st
romar at
io
Di
latedendomet r
ialglands

8.Hypoplasiaischaract
eri
zedbyall
oft
he
fol
l
owi ng,EXCEPT:
I
ncompl etedev el
opmentofanorgan
Decreasednumberofcel ls
I
ncreasednumberofcel ls
Underdevelopmentofanor gan
Decreasedf uncti
onofanorgan

9.Dy spl
asiai schar acter
izedbyal loft
he
fol
lowing,EXCEPT:
Abnor malor ganizationofcel l
s
Lossi ntheuni f
or mityofindividualcel
l
s
Lossi ncellarchitecturalorganizati
on
Replacementofoneadul tcelltypebyanot
her
adultcellt
ype
Variati
onofcel lsinsi zeandshape

10.Whichpat
hol
ogi
cpr
ocessr
esul
tsf
rom
dyspl
asia?
Apl
asia
Hypoplasia
Hyperpigmentat
ion
Cal
cif
ication
Neoplasia

11.Hyper
trophyasapr ocessischaract
eri
zedby
whichofthefol
lowing:
I
ncreaseinthesizeofcell
sandoft heorgan
Shri
nkageinthesizeofcell
sandoft heorgan
I
ncreaseinthenumberofcel l
s
Abnormalorganizat
ionofcel
ls
Vari
ati
onofcellsinsizeandshape

12.Themassi vegrowthofthegravi
dut er
uswi
th
l
argepl umpcell
sisanexampl eof:
Pathologi
chypertr
ophy
Hormonei nducedphysiol
ogichyper
trophy
Pathologi
chyperpl
asia
Dysplasia
Metaplasia

13.Hy per
trophyasanadaptiv
eresponseis
character
izedbywhi choft
hefol
lowing:
Pathologi
chy per
trophyoft
hebreastduri
ng
l
act ation
Hy pertrophyoftheskelet
almusclecell
sina
body -buil
der
Pat hologichyper
trophyoftheuterusdur
ing
pregnancy
Hy pertrophyoftheskelet
almusclecell
sin1
patientwi thi
mmobi li
zedbrokenli
mb
Hy pertrophyoftheendomet r
ium duetoovari
an
tumor

14.Cardiachyper t
rophyischar acter
izedbyal
l
pathol
ogicchanges, EXCEPT:
I
ncreasednumberofmy ocardi
al cel
l
s
I
ncreasedmassandsi zeoft heheart
I
ncreasedpr oteinsynthesis
I
nterst
iti
alcardiacfibrosi
s
I
nadequat evasculature

15.At r
ophyischar act
eri
zedbywhichofthe
foll
owing:
Variati
onofcellsinsizeandshape
Increaseinthesizeofcell
s
Shrinkageinthesizeofcell
sbylossofcell
substance
Increaseinthenumberofcel l
s
Abnor malorganizati
onofcel
ls
16.Oneoft hevari
antsofphy siologicatr
ophyis:
Atrophyofskel
etalmuscl ebythei mmobi li
zed
brokenli
mb
Atrophyofuter
usaf t
erpar t
urit
ion
Kidneyatr
ophyfrom pressure( hydronephrosi
s)
Atrophyoftheendomet r
ium byov ari
antumor
Atrophyofthebrai
ninat herosclerosis

17.Thecausesofpat hol
ogi
catr
ophyar
eal
lof
foll
owi ng,EXCEPT:
Lossofendocr inesti
mulati
on
Lossofi nner
vation
Dimi ni
shedbl oodsupply
Decr easedwor kload
Increasedwor kload

18.Thecausesofpat hol
ogicat
rophyar
eal
loft
he
fol
lowing,EXCEPT:
Aging
I
ntracell
ularf
ataccumulati
on
Pressure
I
nadequat enutr
it
ion
Denervati
on
19.Whatpigmentcanbefoundinthecyt
oplasm
ofheartandmuscl
ecell
sinagingatr
ophy?
Li
pofuscin
Melani
n
Hemosi der
in
Bil
i
rubin
Ferr
it
in

20.Met aplasiaischar acter


izedbywhi chofthe
foll
owi ng:
Rev ersibleincreaseint hesizeofcell
s
I
r r
ev ersibl
echangei nwhi choneadul tcel
ltypei
s
replacedbyanot heradultcellt
y pe
Rev ersiblechangei nwhi choneadul tcellt
ypeis
replacedbyanot heradultcellt
y pe
Rev ersibleabnor mal organi
zationofcells
Rev ersibleincreaseint henumberofcel ls

21.Whatt ypeofmet aplasiamayoccurinthe


respiratorytr
acti
nhabi tualcigar
ett
esmoker?
Epitheli
al metapl
asi
a; squamoust ocolumnar
Met aplasiatoundif
ferentiatedmesenchymal
cell
s
Connect i
veti
ssuemet aplasia
Epitheli
al metapl
asi
a: columnartosquamosus
Noneoft
hese

22.Ther eplacementoft henor mal secr


etor
y
col
umnarepi thelium bythe
nonfunctioning-strati
fi
edsquamousepi thel
i
um
mayoccuri nall organsoft hefollowi
ng,EXCEPT:
Bil
eductsoft hel i
v er
Excretor
yduct soft hesali
v aryglands
Excretor
yduct soft hepancr eas
Respirat
or yepithelium ofthebr onchi
Tubularepi t
helium oft hekidney

23.Hy dronephr
osisischaracteri
zedbyal
loft
he
fol
lowing,EXCEPT:
Thickeningoftherenalparenchyma
Thinningoftherenalparenchyma
Dil
atationoftherenalpelvi
s
Dil
atationoftherenalcaly
ces
Progressiveatr
ophyoft hekidney

24.Inurinarytr
actobstr
ucti
onall
pat
hol
ogi
c
processescanbef ound,EXCEPT:
Dil
atati
onoft hepelvi
sandcaly
ces
Acutetubularnecrosi
s
I
ntersti
ti
al i
nfl
ammat i
on
I
nter
sti
ti
alfi
brosi
s
Gl
omerularandtubul
arat
rophy

Ti
ssueRepai
r(Tugol
bai-18Qs)

1A74- year -
oldwomanpr esentswi thacutechest
painandshor tnessofbr eath.Cardiac
catheterizationdemonst ratesocclusionofthelef
t
anteriordescendi ngcor onar yart
ery.Laboratory
studiesandECGar econsi stentwithacute
my ocardial i
nfar cti
on.Whi chofthef oll
owingis
themostl ikelypat hologicf i
ndingint heaf
fected
heartmuscl e4weeksl ater ?

Capil
lary
-ri
chgranulati
onti
ssue
Coll
agen-ri
chscartissue
Granulomatousinfl
ammat i
on
Neutrophil
sandnecr oti
cdebr
is
Vascularcongesti
onandedema

2A4-year-oldboyfal
lsonarustynailand
punct
ureshi sski
n.Thewoundi scleanedand
cover
edwi thster
il
egauze.Whichoft hefol
l
owing
i
stheinit
ialeventi
ntheheali
ngpr ocess?
Accumulati
onofacuteinfl
ammat or
ycells
Deposit
ionofprot
eoglycansandcoll
agen
Dif
fer
entiat
ionandmigrati
onofmy ofi
brobl
asts
Formati
onofaf i
bri
nclot
Macrophage-medi
atedphagocytosi
sofcellul
ar
debri
s

3An82- year-
oldmandi es4yearsafter
developingcongest i
veheartf
ail
ure.Hehada
hist
oryofmul t
ipl
emy ocardi
ali
nfarct
sov erthe
past10y ear
s.At r
ichromestai
nofhear tmuscle
atautopsyi sshowni ntheimage.Whati sthe
predomi nanttypeofcoll
agenfoundinthismature
scartissue?

Ty
peI
Ty
peI
I
Ty
peI
V
Ty
peV
Ty
peVI

4A25- year-
oldwomansust ainsadeep,open
l
acerati
onov erherr
ightf
orear minamot orcy
cle
acci
dent.Thewoundi scl
eanedandsut ured.
Whichoft hefol
lowi
ngcellt
y pesmediat
es
contr
actionofthewoundtof acil
i
tat
eheal i
ng?

Endotheli
alcel
ls
Fi
broblasts
Macrophages
Myofi
br obl
asts
Smoot hmusclecel
l
s

5A70- year-
oldwomanwi thdiabetesdev elopsan
ulceronherrightleg(showni ntheimage) .The
ulcerbediscov eredwithgranul
at i
ontissue.
Whi chofthefollowingaretheprincipl
ecellular
component sfoundi nthebedoft hiswoundof
thiswound?
Fi
brobl
astsandendotheli
alcell
s
Myofi
brobl
astsandeosinophil
s
Neutr
ophil
sandlymphocy t
es
Pl
asmacel l
sandmacr ophages
Smoothmusclecell
sandMer kelcel
l
s

6A68- year -
oldmanpr esentsforr epairofan
abdomi nalaor t
icaneur ysm.Sev er ecompl i
cated
ather
oscler osisisnot edatsur gery,prompt i
ng
concernf orembol i
sm ofat heromat ousmat erial
tothekidney sandot herorgans.I fthepat i
ent
weretodev elopar enal cort
icalinfarctasar esul
t
ofsurgery ,
whi choft hef ol
lowingwoul dbet he
mostlikelyout come?

Chronicinf
lammat ion
Granulomatousi nf
lammati
on
Hemangi omaf ormation
Repairandr egener
ation
Scarformation
7A40- year-oldwomanpr esentswit
hapainl
ess
l
esiononherr ightearl
obe( showninthei
mage).
Sherepor t
st hatherearswer epier
ced4months
ago.Whi choft hefoll
owingbestexplai
nsthe
pathogenesisoft hisl
esion?

Clonal expansionofsmoot hmusclecel l


s
Exuber antformat i
onofgranul
ati
ontissue
I
ncreasedgr owt hofcapil
l
aryendot
helialcel
l
s
I
ncreasedt urnoverofextr
acel
lul
armat ri
x
proteoglycans
Mat urati
onar restofcol
lagenassembly

8A58- year-
oldwomanunder goeslumpect omy
forbreastcancer.Onemonthfoll
owingsur ger
y,
shenot i
cesaf i
rm
0.3-
cm nodulealongoneedgeoft hesurgical
i
ncisi
on.Biopsyoft hi
snodul
er ev
ealschronic
i
nflammatorycells,mul
ti
nucleatedgiantcel
ls,and
extensi
vef
ibrosis.Themult
inucleatedcell
sint hi
s
nodulemostlikel
yformedinr esponsetowhi chof
thefoll
owi
ngpat hogeni
cstimul i
?

Bacteri
alinfect
ion
Forei
gnmat eri
al
Lymphat i
cobst ruct
ion
Neoplasticcell
s
Vir
alinfecti
on

9A57- year-
oldmanwi thahi st
oryofalcohol
ism
presentswithyell
owdi scolorati
onofhisskinand
scler
ae.Laboratorystudiesshowel evatedserum
l
ev el
sofliv
erenzy mes( ASTandALT) .A
tr
ichromestainofal i
verbiopsyi sshowni nthe
i
mage.Asi mi l
arpatternofr egener
ati
onand
fi
brosiswouldbeexpect edi ntheli
verofapat i
ent
withwhichofthef ol
lowingcondi ti
ons?
Acutetoxicl
iverinj
ury
Chronicvi
ralhepatit
is
Ful
mi nanthepaticnecrosi
s
Hepatocell
ularcarci
noma
Thrombosisoft heportalvei
n

10A10- year
-ol
dboyt ri
psatschoolandscr apes
thepalmsofhishands.Thewoundsar ecleaned
andcov er
edwithsteri
legauze.Whichofthe
foll
owi
ngt er
msbestchar acter
izestheheali
ngof
thesesuperf
ici
alabrasi
ons?

Fi
brosi
s
Granul
ationti
ssue
Pri
maryi nt
enti
on
Regenerati
on
Secondaryint
enti
on
11A34- y
ear-
oldwomanhasabeni gnnev us
remov edfrom herbackunderl ocalanest
hesia.
Whichoft hefoll
owingf amili
esofcelladhesi
on
moleculesistheprincipalcomponentoft he
“pr
ovisionalmatri
x”thatformsdur i
ngearl
ywound
heal
ing?

Cadher i
ns
Fibronecti
ns
Integri
ns
Lami ni
ns
Selecti
ns

12A29- year-
oldcarpenterr
eceivesatraumati
c
l
acerati
ont oherlef
tarm. Whichofthef
ollowi
ngis
themosti mportantfact
orthatdetermi
nes
whetherthiswoundwi ll
healbypr i
maryor
secondaryint
ention?

Apposi
tionofedges
Depthofwound
Metaboli
cstat
us
Ski
nsiteaff
ected
Vascul
arsupply

13A9-year
-ol
dboyrecei
vesadeeplacer
ati
on
overhi
sri
ghteyebr
owplayi
ngicehockey
.The
woundiscl
eanedandsutur
ed.Whichofthe
fol
l
owingdescribesthepri
ncipalf
unct
ionof
macrophagesthatarepresentint
hewound24t
o
48hoursafteri
njur
y?

Anti
bodyproducti
on
Deposi
ti
onofcollagen
Hist
aminerel
ease
Phagocyt
osis
Woundcontracti
on

14A16- year-oldboysuffersaconcussi ondur i


ng
anicehockeygameandi sr ushedt othe
emer gencyr oom. ACTscanoft hebrainreveal
sa
cerebralcont usionoftheleftfront
al l
obe.Theboy
l
iescomat osef or3day sbutev entuall
yregains
consciousness. Whichofthef oll
owingcellsisthe
pri
ncipalmedi atorofscarformat i
oninthecentr al
nervoussy stem oft hi
spat i
ent?

Fi
broblast
s
Gli
alcell
s
Neurons
Oli
godendrocyt
es
Schwanncells

15A30- y
ear-
oldfir
efight
ersuffer
sextensi
ve
thi
rd-
degreeburnsoverhisarmsandhands.This
pati
enti
sathighriskfordevel
opingwhichofthe
fol
lowingcompl i
cat
ionsofwoundheal
i
ng?
Contractur
e
Dehiscence
I
ncisionalher
nia
Keloi
d
Traumat i
cneuroma

16A23- y ear-
oldmansuf fersacrushi nj
uryofhi
s
foot,
whi chbecomessecondar i
lyinfected.He
undergoesabel ow-the-kneeamput ati
on.Six
mont hslater,t
hepat ientcomplainsofchr oni
c
painatthesi t
eofamput ati
on.Afirm nodulei
s
i
dentifi
edatt hescarsi te.Abiopsyoft henodule
demonst rateshaphazar dgr owt
hofner ves
(shownint heimage) .Whichofthef oll
owingis
themostl i
kel
ydi agnosis?

Gangl
i
oma
Gangl
i
oneur
oma
Hamart
oma
Neur
alnev
us
Neur
oma

17A34- year -
oldmanpr esentswitha5-day
historyofapai nfulsoreonhishand.Phy sical
exami nationr evealsa0.5-cm abscessont he
extensorsur f
aceoft helefthandthatdrainsa
thick,purulentmat er
ial
.Diapedesi
sofleukocy t
es
i
nt oandar oundt hispati
ent’
sinfectedwound
occur sprimar il
yatwhi chofthefollowi
ng
anat omicl ocations?

Lymphaticcapil
l
ar i
es
Postcapi
ll
aryvenules
Precapi
ll
aryart
eriol
es
Smalldermalarteri
es
Smalldermalveins

18A35- year -
oldpr egnantwomanwi thahi stor
y
ofchronicgast rit
ispresentstotheemer gency
room compl ainingofacut eabdomi nalpain.
Physicalexami nat i
onrevealshepatomegal y,
ascit
es,andmi l
dj aundice.Thepatient
subsequent l
ydev el
opsacut ehepaticfail
ureand
expir
es.Aut opsyr evealsthrombosisoft he
hepaticveins(Budd- Chiarisyndr
ome) .Dur i
ngthe
autopsy,alesioni sidenti
fiedinthedistal
stomachandexaminedbyli
ghtmicr
oscopy
(shownint
heimage).Whi
chofthefoll
owingbest
descri
best
hisi
nci
dentalf
indi
ngatautopsy?

Carci
noma
Contracture
Di
v er
ticulum
Granuloma
Ul
cer

Neopl
asi
a

1.Thet erm ofabeni


gntumorisconst
ruct
edby
combiningthewor ddesi
gnati
ngthetumorcell
ori
gin plus whi
ch ofthe f
oll
owi
ng endi
ngs or
words:
A-genic
B-emi a
C-oma
D-it
is
E-osis(
-asi
s)

2.Lackofdi f
ferent
iat
ioni
schar
act
eri
sti
cofwhi
ch
ofthefoll
owi ng:
AHy perplasi
a
BHy poplasia
CAnaplasi a
DHy pertr
ophy
EDy spl
asia

3.Growt hoft umorwi


thoutr
egul
arbor
der
li
nei
s
call
ed:
A.Exophyti
c.
B.Unicentr
ic.
C.Endophytic.
D.Expansive.
E.I
nvasive.

4.Gr owth oftumorwi


thr
egul
arbor
der
li
ne i
s
called:
A.Exophy t
ic.
B.Unicentri
c.
C.Expansive.
D.Invasiv
e.
E.Endophy ti
c.
5.Gr
owt hoftumori
ntot
hel
umenofhol
l
owor
gan
i
scall
ed:
A.
Exophy t
ic.
B.
Unicentr
ic.
C.
Expansive.
D.
Inv
asive.
E.
Endophy ti
c.

6.Gr owthoftumori
ntot
hewal
lofhol
l
owor
gani
s
called:
A.Exophy t
ic.
B.Unicentr
ic.
C.Expansive.
D.Invasi
ve.
E.Endophy ti
c.

7.Spr eding ofmal i


gnantt umorst
hrough t
he
bl
oodst ream iscall
ed:
A.
Seedi ng.
B.
Hemat ogenousmet astasi
s.
C.
Lymphogeni cmet astasis.
D.
Perineur almetastasis.
E.
Throught henaturalpassages.

8.Spr eding ofmalignanttumor


sthr
ough t
he
serosalsurfacesi
scall
ed:
A.Seeding.
B.Hemat ogenousmetastasi
s.
C.
Lymphogeni
cmetastasi
s.
D.
Peri
neur
almetast
asis.
E.
Thr
oughthenatur
alpassages.

9.
Cancerist he:
A.
Fir
stleadingcauseofdeath.
B.
Secondl eadingcauseofdeath.
C.
Thirdleadingcauseofdeath.
D.
Fourthleadingcauseofdeath.
E.
Fif
thleadingcauseofdeath.

10.Nomencl atureoft
umor si
sbasedonwhichof
thefoll
owing:
A.Parenchymal componenthi
stogenesi
s.
B.Str
omal component.
C.Locali
zati
on.
D.Vascularcomponent.
E.I
nfl
ammat or
ychanges.

11.Neoplasti
ccell
sarecharacteri
zedbyalloft he
foll
owing,EXCEPT:
A.Loss of responsi
veness to nor malgr owt h
control
.
B.Behaveasparasit
es.
C.Theyalwaysneedinendocrinesupport.
D.Theyincreaseinsizeregardl
essoft hei
rl ocal
i
nv i
ronment.
E.Theyaredependonhostf ort hei
rnutr
iti
onand
bl
oodsuppl
y.

12.Neopl asti
ccellsarecharacter i
zedbyal
lofthe
foll
owi ng,EXCEPT:
A.Do notl oss of r esponsi
v eness to nor
mal
growt hcontrol.
B.Behav easpar asi
tes.
C.Theycont i
nuet ogr ow regar dl
essofnormal
regulatorycontrol
.
D.Theyi ncreaseinsi zeregardlessoftheirl
ocal
i
nv i
ronment .
E.Theyar edependonhostf ort heirnut
ri
ti
onand
bloodsuppl y.

13.Mor phol
ogi catypi
aischaract
eri
zedbyal
lof
thef
ollowing,EXCEPT:
ACellpleomor phism
BMet aplasi
a
CAlt
er ati
onofpar enchyma/str
omar at
io
DAtypicalmitoses
EEnlargedhyper chr
omaticnucl
ei

14.Dy spl
asti
ccellsarecharact
eri
zedbyal
loft
he
foll
owing,EXCEPT:
A.Lossofcellunifor
mi t
y.
B.Formationoftumorgi antcel
l
s.
C.Hyperchromaticenlar
gednuclei
.
D.Archit
ectur
alanarchy.
E.
Incr
easedmi
tot
icf
igur
es.

15.Dy spl
asti
ccell
sar echaracteri
zedbyal
loft
he
foll
owi ng,EXCEPT:
A.Lossofcel lunif
ormi t
y.
B.Hy perchromati
cenl ar
gednucl ei
.
C.Increasednormal mi t
oti
cfigures.
D.Mi tosesarenumer ousandat ypical
.
E.Archi t
ectur
alanarchy.

16.Dy splasticcell
sar echar
acteri
zedbyal
loft
he
foll
owi ng,EXCEPT:
A.Hy perchromat i
cenlargednuclei
.
B.Lossofcel luni
formity.
C.Increasednor mal mitot
icfi
gures.
D.Architecturalanarchy.
E.Mar kedpl eomorphism.

17.Dy splasti
ccell
sar echaracteri
zedbyal
lofthe
foll
owi ng,EXCEPT:
A.Lossofcel lunif
ormi t
y.
B.Hy perchromati
cenl ar
gednucl ei
.
C.Nucl eiar evari
able and bizarrein si
ze and
shape.
D.Increasednormal mi t
oti
cfigures.
E.Architectur
alanarchy.

18.Lack ofcel
ldi
ff
erent
iat
ion (
anapl
asi
a)i
n
mali
gnantt umorischar acteri
zedbyalloft
he
f
oll
owi ng,EXCEPT:
ACellularandnuclearpleomor phi
sm
BHy perchromati
cnucl ei
CIncreasednuclear-t
o-cytopl
asmicrat
io
DCoar selyclumpedchr omatin
EMet astases

19.Anaplast i
ccellsarecharacteri
zedbyalloft
he
fol
lowing,EXCEPT:
A.Markedpl eomor phism.
B.Mit
osesar enumer ousandat ypi
cal.
C.Nucleiarev ar
iabl
eandbi zar
reinsizeandshape.
D.Cell
s r esembl e v er
y closelyt heir nor
mal
counterparts.
E.Nucleoliareofastoundingsize.

20.Anapl asticcellsarecharacteri
zedbyall
ofthe
foll
owi ng,EXCEPT:
A.Mar kedpl eomor phism.
B.Mitosesar enumer ousandat ypi
cal.
C.Nucl eiarev ar
iabl
eandbi zarr
einsizeandshape.
D.Format ionoft umorgi antcel
ls.
E.Format ionofLanghansgi antcel
ls.

21.Anaplasi
ai s char
act
eri
zed by al
loft
he
fol
l
owing,EXCEPT:
ACell
ularandnucl
earpl
eomorphi
sm
BFormat
ionofminorgiantcell
s
CFormati
onofatypi
calmitoti
cfigures
DFormati
onofLanghans'giantcell
s
EHyper
chromati
cnuclei

22.Anapl
ast
iccel
lsar
echaracteri
zedby
:
A.Nucl
ear
-cyt
opl
asmicrat
iois1:1.
B.Nucl
ear
-cyt
opl
asmicrat
ioi
s1: 3.
C.Nucl
ear
-cyt
opl
asmicrat
iois1:5.
D.Nucl
ear
-cyt
opl
asmicrat
iois1:7.
E.
Nuclear
-cy
topl
asmicrat
ioi
s1: 9.

23.Anapl asticcellsarechar acteri


zedbyall
ofthe
foll
owi ng,EXCEPT:
A.Mar kedpl eomor phism.
B.Mitosesar enumer ousandat ypi
cal.
C.Nucl eiar
ev ar
iabl
eandbi zar r
einsizeandshape.
D.Lossoff unctionsisnotcor r
elatedwit
hdegree
ofanapl asia.
E.Lossoff unctionsiscorr elatedwi t
hdegreeof
anaplasia.

24.Tumorcel linvasion int


ot he extr
acell
ular
matr
ixcanbecharacterizedbyal
lofthefoll
owing,
EXCEPT:
ADetachmentofthetumorcel l
sfrom eachother
BInt
ravasat
ion
CAttachmenttomatrixcomponent s
DDegradati
onofext
racel
lul
armat
ri
x
EMigr
ationoft
umorcells

25.Al loft he f ol
lowing st atement s corr
ect l
y
descri
becelloncogenes, EXCEPT:
AOncogenesar eder i
vedf rom v i
ralDNAt hathas
beenincorporatedintothegenome
BOncogenesencodepr oteinsthatr esemblet he
productsofnormal genes
CSomeoncogenepr oductsar etheanal oguesof
growthfactors
DSomeoncogenepr oductsar etheanal oguesof
growthfactorreceptors
E Some oncogene pr oduct s act i
vate nuclear
tr
anscri
ption

26. Si
gnsofbeni gnt umor
sareal
loft
hef
oll
owi
ng,
EXCEPT:
A.Wel ldi
fferenti
ated.
B.Fewmi t
oses.
C.Slowlygr owing.
D.Invasiv
et ypeofgr owt
h.
E.
Expansi vet ypeofgrowth.

27.Si
gnsofbenigntumor
sar
eal
loft
hef
oll
owi
ng,
EXCEPT:
A.Well
dif
ferent
iat
ed.
B.Fewmitoses.
C.
Presenceofmet ast
asi
s.
D.
Slowlygrowing.
E.
Expansiv
et y
peofgrowth.

28.Signsofbeni gntumor
sar
eal
loft
hef
oll
owi
ng,
EXCEPT:
A.Rapidlygrowing.
B.Fewmi t
oses.
C.Slowlygrowing.
D.Welldiff
erenti
ated.
E.
Expansi vetypeofgrowt
h.

29.Signsofbenignt umor
sar
eal
loft
hef
oll
owi
ng,
EXCEPT:
A.Welldif
ferenti
ated.
B.Pleomorphiccells.
C.Fewmi toses.
D.Slowlygrowing.
E.
Expansi vetypeofgrowt
h.

30.Si gnofbeni gntumorsi


s:
A.Pleomor phi
ccel l
s.
B.Rapi dlygrowi ng.
C.Atypi calmitoses.
D.Sl
owl ygrowi ng.
E.
Inv asi v
et y
peofgr owt
h.

31.Si
gnofbeni
gnt
umor
sis:
A.
Pleomor phi
ccel l
s.
B.
Rapidlygrowi ng.
C.
Atypicalmitoses.
D.
Expansivety peofgrowt
h.
E.
Invasiv
et y
peofgr owth.

32. The gr owth of neoplasms is cr


it
ical
l
y
dependentonwhi chofthef
oll
owing:
ALocalizat
ion
BTumorst roma
CNeut r
ophili
mmi grat
ion
DLy mphati
cdr ai
nage
EInfl
ammat oryreacti
on

33.Themostchar acteri
sti
cf eatur
eofamalignant
neoplasm i n cont rast t o a r eact
ive or
i
nflammat ory ov ergrowth i s which of t he
fol
lowi ng:
AGr owt hfactorproduct i
on
BNecr osi
s
CLocal i
zati
on
DAut onomousgr owt hfollowingtheremov alof
all
prov okingfactors
EAdj acenttissuecompr ession

34.Thesequenceofev ent
sint
het umorcel
l
i
nvasi
oni
ntothebasementmembranesi
ncl
udes
al
loft
hefol
lowi
ng,EXCEPT:
ALooseningofi nt
ercel
lul
arjunct
ions
BAtt
achmentt ot hebasementmembr ane
CAccumul ati
onoft heneutr
ophil
s
DDegradationoft hebasementmembr ane
ETumorcel lmigrati
on

35.Si gns ofmal ignanttumor


s ar
e al
loft
he
fol
lowi ng,EXCEPT:
A.Pleomor phiccell
s.
B.Rapidlygr owing.
C.Atypicalmi t
oses.
D.Slowl ygrowing.
E.I
nv asivetypeofgr owth.

36.Si gns ofmal i


gnanttumor
s ar
e al
loft
he
foll
owing, EXCEPT:
A.Pleomor phiccell
s.
B.Rapidlygrowing.
C.Atypicalmitoses.
D.Invasivetypeofgrowth.
E.Expansivet y
peofgrowth.

37.Signofmal i
gnanttumorsi
s:
A.Wel ldi
fferenti
ated.
B.Fewmi t
oses.
C.Slowlygr owing.
D.Invasiv
et ypeofgr owth.
E.
Expansi vet ypeofgrowth.
38.Signofmal i
gnanttumor
sis:
A.Pleomorphiccells.
B.Fewmi toses.
C.Slowl
ygr owing.
D.Regularcell
s.
E.
Expansi vetypeofgrowth.

39.Amal i
gnanttumorischaracter
izedbyal
lof
thefoll
owing,EXCEPT:
AIncreasedabnormalti
ssuemass
BUncoor dinat
edinv
asi
vegr owth
CRelativel
yautonomousgrowth
DDecr easedabnormalt
issuemass
EMet astases

40.The mostchar acter


ist
ic manifest
ati
on of
mali
gnanttumoriswhichofthefol
lowing:
ACell
ularaty
piaandpl
eomor phi
sm
BCompr essi
onofsurr
oundingti
ssue
CLargesize
D.Necrosi
s
EMetastases

41.Cr it
eri
a,by which beni
gn tumors can be
dif
ferent i
atedf
rom mali
gnantones,ar
eallofthe
fol
lowi ng,EXCEPT:
AMat urit
y
BRateandcharact
erofgr
owt
h
CLocali
nvasi
on
DEdema
EMetast
ases

42.Themosti mpor t
antfeat
uretodi
sti
ngui
shthe
mali
gnantt umorfrom abeni gnoneiswhichof
thefol
lowing:
ALackofencapsul ati
on
BHighmi toti
crat
e
CNecr osis
DMet astases
ENuclearpleomorphism

43.Thecr iter
iabywhi chbeni
gntumorscanbe
dif
ferent i
atedfrom themali
gnantonesar
eallof
thefollowing,EXCEPT:
AMat urit
y
BRat eandchar act
erofgrowth
CLocal isation
DAnapl asia
EMet ast ases

44.Signsoftumor swi thint


ermedi
atemal
i
gnancy
areallofthefoll
owi ng,EXCEPT:
A.Pl
eomor phiccells.
B.Sl
owl ygrowing.
C.Absenceofmet astasis.
D.
Invasi
vetypeofgr
owth.
E.
Expansiv
et y
peofgrowt
h.

45.Signsoft umor swithint


ermedi
atemal
i
gnancy
areallofthef ollowing,EXCEPT:
A.Pl
eomor phiccel ls.
B.Sl
owl ygrowi ng.
C.Rapidlygrowi ng.
D.Absenceofmet astasis.
E.
Invasivetypeofgr owth.

46.Histol
ogicorigi
noft
umor
siscal
l
ed:
A.Morphogenesis.
B.Hist
ogenesis.
C.Thanatogenesis.
D.Pathogenesis.
E.
Cancer ogenesi
s.

47.Oncogeni cv i
rusis:
A.I
nfluenzav i
rus.
B.Parainf
luenzav ir
us.
C.Chikenpoxv i
rus.
D.AIDSv i
rus.
E.
Humanpapi l
lomav ir
us.

48.Selectf
rom t
hepr
oposedopt
ionsoncogeni
c
vi
rus:
A.I
nfl
uenzavir
us.
B.
Parainfl
uenzav i
rus.
C.
Chikenpoxv i
rus.
D.
AIDSv ir
us.
E.
Hepat i
ti
sBv i
rus.

49.Alloft he fol
lowing v
iruses pr
oved t
o be
capabl
e ofpr oducing mali
gnancies i
n human
bei
ngs,EXCEPT
AHumanpapi l
lomav ir
us
BCy t
omegalovir
us
CEpstei
n-Barrv i
rus
DHepatit
isBvirus
EHepatit
isCvirus

50.Secondarylesionsi
nmal
i
gnantt
umor
sar
eal
l
ofthefoll
owing,EXCEPT:
A.Hemorrhage.
B.Necr
osis.
C.Mucinaccumul ati
on.
D.Cal
cifi
cati
on.
E.
Amy loi
dosis.

51.Classif
icat
ionofneopl
asmsbyWHOi
sbased
on:
A.Eti
ologyoftumors.
B.Morphogenesisoftumors.
C.Locali
zati
onoftumors.
D.Hist
ogenesisoftumors.
E.
Dif
fer
ent
iat
ionoft
umor
s.

52.The f actor assessed int he hist


ologi
cal
gradingofamal ignanttumori swhichoft he
fol
lowing:
AThenumberofl ymphnodemet ast
ases
BThesi ze(diameter)ofthepr i
marytumor
CTheext entofi nv
asionoft hepri
marytumorinto
surroundi
ngst ruct
ures
DThedegr eeofcy tol
ogicaldif
fer
enti
ati
onoft he
pri
mar ytumor
EThepr esenceorabsenceofl i
vermetastases

53.Thei mportantfact
orsassoci
atedwit
hthe
i
ncreasing inci
dence oftumors ar
e al
lofthe
fol
l
owing, EXCEPT:
AAge
BDiet
CEnv i
ronment
DAcutei nfl
ammat i
on
EGenet i
cmakeup

54.Mostcommonl ocal
i
zat
ionofcanceri
nmeni
n
West er
ncount ri
esis:
A.Stomach.
B.Prostategland.
C.Thyroidgland.
D.Lungs.
E.
Lar
gei
ntest
ine.

55.Mostcommon l ocali
zati
on ofcanceri
n
womeni nWest er
ncountr
iesis:
A.Uterus.
B.Ovaries.
C.Thyroidgland.
D.Lungs.
E.
Br easts.

56.Mostcommonl ocal
i
zat
ionofcanceri
nmeni
n
Kyrgyzstanis:
A.Esophagus.
B.Stomach.
C.Thy r
oidgl
and.
D.Lungs.
E.
Lar geint
esti
ne.

57.Mostcommon l ocal
i
zat
ion ofcanceri
n
womeni nKy r
gyzst
ani
s:
A.Uterus.
B.Ovaries.
C.Breasts.
D.Thyroidgland.
E.
Lungs.

58.Tumorwit
hint
ermedi
atemal
i
gnancyi
s:
A.Amel
oblast
oma.
B.
Neuroblastoma.
C.
Gli
oblastoma.
D.
Osteoblastoma.
E.
Chondroblastoma

59.Negati
veef f
ect
sr esult
ingf r
om neopl
asi
aar
e
al
lofthefoll
owing,EXCEPT:
ACompr essionofadjacenttissues
BCachexia
CDestruct
ionofadjacenttissues
DParaneoplasti
csyndrome
ECancerobesity

60.Al
lofthefol
lowingneopl
asmsar
emal
i
gnant
,
EXCEPT:
A.Adenocar
cinoma
B.Melanoma
C.Seminoma
D.Chori
onepi
theli
oma

E.Papi
l
lar
ycy
stadenoma

EPI
THELIALTUMORS
1.The term ofa benign epi
thel
ialtumori s
const
ruct
edbycombiningthewor ddesignat
ing
thetumorcellori
ginpluswhichofthefoll
owing
endi
ngsorwor ds:
Asarcoma
Bcarcinoma
Coma
Diti
s
Eosis(-
asis)
2.Thet erm ofamal ignantepi
thel
ialt
umori s
constr
uctedbycombi ningtheworddesignat
ing
thetumorcellori
ginpluswhichofthefoll
owing
endi
ngsorwor ds:
Asarcoma
Bcarcinoma
Coma
Diti
s
Eosis(-
asis)

3. Beni gn tumor ar
isi
ng f
rom squamous
epit
helium i
scal
led:
A.Papil
loma.
B.Adenoma.
C.Li
poma.
D.Fi
broma.
E.Osteoma.

4. Benign epi thel


i
al neoplasm producing
fi
nger
-li
kepr oj
ect
ionfrom epi
thel
i
alsur
facesis
ref
err
edt oas:
APapi l
l
oma
BAdenoma
CFibroma
DSarcoma
ETeratoma

5.Selectthesignsinher
enti
nthepapi
l
loma:
Atissueat y
pism
Bcellul
aratypism
Cmet astasis
Dinvasivegrowt h
Ecancerpear ls

6.Papilloma can become compl


icat
ed byall
:
EXCEPT:
Ableeding
Binfl
ammat i
on
Cresorption
Dmal i
gnancywi t
hthedevel
opmentofsquamous
cel
lcarcinoma
Erel
apseaf terr
emoval

7.
Squamouscellpapi
ll
omai
slocal
i
zedi
n:
A.
Pelv
isofki
dney.
B.
Caly
cesofkidney.
C.
Vagina.
D.
Uret
er.
C.
Uret
hra.
8.Squamouscel l
papil
l
omai
slocal
i
zedi
n:
A.Pelv
isofki
dney.
B.Caly
cesofkidney.
C.Uret
er.
D.Mouth.
E.Uret
hra.

9.Transiti
onal
cel
lpapi
l
lomai
slocal
i
zedi
n:
A.Mout h.
B.Esophagus.
C.Skin.
D.Ureter.
E.Vagina.

10.Transiti
onalcel
lpapi
l
lomai
slocal
i
zedi
n:
A.Mout h.
B.Esophagus.
C.Vagina.
D.Gallbladder
.
E.
Ur i
narybladder.

11. Benign tumor ar


isi
ng f
rom col
umnar
epit
heli
um i
scal
led:
A.Papi
ll
oma.
B.Adenoma.
C.Li
poma.
D.Fi
broma.
E.
Ost
eoma.

12.Benignt umorsari
singf
rom epi
thel
i
alt
issue
arer
eferredtoas:
ASarcomas
BAdenocar cinomas
CPapill
omas
DAdenoma
EPolyps

13.Adenomai sl
ocal
i
zedi
n:
A.Mout h.
B.Esophagus.
C.Vagina.
D.Gallbladder
.
E.
Ur i
narybladder
.

14.Adenomai sl
ocal
i
zedi
n:
A.Mouth.
B.Esophagus.
C.Stomach.
D.Uret
er.
E.
Vagina.

15.Whi chhistologi
cty
peofadenomaar
isef
rom
gl
andul arparenchyma?
A.Acinar.
B.Tubular.
C.
Trabecular
.
D.
Papill
ary.
E.
Cystadenoma.

16.Whi chhistol
ogi
cty
peofadenomaar
isef
rom
ducts?
A.Acinar.
B.Tubular.
C.Trabecular
.
D.Papill
ary.
E.
Cy stadenoma.

17. Whi ch hist


ologi
c ty
pe of adenoma i
s
character
izedwit
hpredomi
nanceofst
roma?
A.Acinar.
B.Tubular.
C.Trabecular
.
D.Papil
lary.
E.Fi
broadenoma.

18.Whi ch type ofadenoma pr


oject
ed under
surfaceofmucousmembr ane?
A.Acinaradenoma.
B.Tubularadenoma.
C.Trabecularadenoma.
D.Adenomat ouspolyp.
E.Fi
broadenoma.
19.I n whator gans and t
issues can dev
elop
adenoma?
Askeletalmuscles
Bpituit
arygland
Csubst anceofthebrai
n
Dspleen
Ebones

20.Precancer
ousdiseaseoft
het
hyr
oidgl
andi
s:
Apapill
oma
Bpolyp
Cserouscy st
adenoma
Dfoll
i
cle
Eadenoma

21.Mal ignantt
umorar
isi
ngf
rom epi
thel
i
alcel
l
sis
call
ed:
A.Papil
loma.
B.Adenoma.
C.Carcinoma.
D.Sarcoma.
E.Terat
oma.

22.Theearli
estwayofmetast
asi
zi
ngamal
i
gnant
tumorfrom t
heepit
heli
um i
s:
Ahemat ogenous
Bperineur
al
Clymphogenic
Di
mplantat
ion
EI
ntr
acanacul
ar

23. Whi ch hi st
ologi
cty
pe of car
cinoma i
s
characteri
zedbyabsenceofinv
asi
vegrowth?
A.Adenocar cinoma.
B.Carci
nomai ncit
u.
C.Mucouscar cinoma.
D.Soli
dcar cinoma.
E.Medullarycar ci
noma.

24. Whi ch hi stologi


ct ype of carcinoma i
s
characterizedbyf ormationofcancerouspear
ls?
A.Adenocar ci
noma.
B.Nonker atini
zingsquamouscel lcarci
noma.
C.Kerati
nizi ngsquamouscel lcar
cinoma.
D.Transit
ional cellcar
cinoma.
E.Medullar ycarci
noma.

25. Whi ch hi stol


ogi
ct ype of car
cinoma i s
characteri
zed by f or
mati
on of t rabecul
ar
str
uctures?
A.Adenocar cinoma.
B.Mucouscar cinoma.
C.Soli
dcar cinoma.
D.Scir
rhouscar ci
noma.
E.Medullarycar ci
noma.
26. Whi ch hi st
ologi
ctype of carci
noma i
s
undif
ferentiated?
A.Carci
nomai ncit
u.
B.Adenocar cinoma.
C.Sol
idcar cinoma.
D.Nonkeratinizingsquamouscel
lcarci
noma.
E.Ker
ati
ni zi
ngsquamouscel lcar
cinoma.

27. Car cinoma hi st


ological
l
y - t r
abecul
ar
str
uctur
eswi thstr
oma.Whi chtypeofcar
cinomas
i
sit?
Ascir
rhouscar ci
noma
Bsoli
dcar cinoma
Cmucousorcol l
oidcar
cinoma
Dmedul l
arycar ci
noma
Esmal l
cell carci
noma

28. Whi ch hi stol


ogict ype of carci
noma is
characteri
zedbypr oduct
ionofexcessi
veamount
ofmucus?
A.Adenocar cinoma.
B.Coll
oidcar cinoma.
C.Soli
dcar cinoma.
D.Scir
rhouscar ci
noma.
E.Medullarycar ci
noma.

29.Carci
nomahist
ologi
cal
l
y-appearsi
gnetr
ing
cel
ls(excessi
vemucinaccumul
atesinthecel
l
andthenucleusispushedtowardst
heper
ipher
y).
Whichtypeofcar ci
nomasisit
?
Ascirr
houscarcinoma
Bsoli
dcarcinoma
Cmucousorcol l
oidcar
cinoma
Dmedul l
arycarci
noma
Esmallcellcar
cinoma

30.Inwhi chhi stol


ogi
ctypeofcar
cinomast
roma
predominat esanaplast
icpar
enchy
ma?
A.Adenocar cinoma.
B.Coll
oidcar cinoma.
C.Soli
dcar cinoma.
D.Scir
rhouscar ci
noma.
E.
Medul larycar ci
noma.

31. Whi ch hi st
ologi
ctype of carci
noma i
s
undif
ferenti
ated?
A.Carcinomai ncit
u.
B.Sci
rrhouscar ci
noma.
C.Adenocar ci
noma.
D.Nonker at
ini
zingsquamouscel
lcarci
noma.
E.Ker
at i
nizi
ngsquamouscellcar
cinoma.

32. Car ci
noma hi stol
ogi
cal
ly - stroma
predominates anaplasti
c par
enchy
ma. Whi
ch
ty
peofcar cinomasisit?
Ascirr
houscar ci
noma
Bsol
i
dcarcinoma
Cmucousorcol l
oidcar
cinoma
Dmedull
arycarci
noma
Esmall
cellcar
cinoma

33. In whi ch hist


ologictype of car
cinoma
parenchymapr edominatesst
roma?
A.Adenocarcinoma.
B.Coll
oidcarcinoma.
C.Soli
dcarcinoma.
D.Scir
rhouscar ci
noma.
E.Medullar
ycar ci
noma.

34. Car ci
noma hi stol
ogi cal
l
y - par
enchyma
predominatesstroma;gr ossl
y-consi
stencei
s
soft.Whichtypeofcarcinomasi si
t?
Asci r
rhouscarci
noma
Bsol i
dcarcinoma
Cmucousorcol l
oidcarcinoma
Dmedul lar
ycarci
noma
Esmal lcel
lcarci
noma

35.Mixthistol
ogictypeofcar
cinomai
s:
A.Adenosquamouscar cinoma.
B.Col
loidcarcinoma.
C.Sol
idcarcinoma.
D.Sci
rrhouscar ci
noma.
E.
Medul lar
ycar ci
noma.
36.Car ci
nomaswithepider
miscel
ldi
ff
erent
iat
ion
arecalled:
ASar comas
BAdenocar ci
nomas
CSquamouscel lcarci
nomas
DPapi ll
omas
ECy stadenomas

37. Frequent l
ocal
i
zat
ion of squamous cel
l
car
cinoma:
Askin
Bli
ver
Cbrain
Dintesti
ne
Estomach

38.Malignanttumor, ar i
singf
rom squamouscel
l
,
i
snamed:
Asquamouscel l
car cinoma
Bacinaradenocarcinoma
Cpapill
aryadenocarci noma
Dtransi
tionalcel
lcar cinoma
Epapill
oma

39. Which hi
stol
ogi
ct ype of carci
noma i s
charact
eri
zedbyfor
mati
onofirr
egularglandul
ar
str
uctur
es?
A.
Adenocar ci
noma.
B.
Nonker at
ini
zingsquamouscel
lcarci
noma.
C.
Kerati
nizi
ngsquamouscel l
car
cinoma.
D.
Transi
tionalcell
carci
noma.
E.
Medullarycarci
noma.

40.Whatdi ff
erenti
ated mal
ignantt
umorcan
devel
opf rom adenoma:
Asoli
dcar ci
noma
Bscirr
houscar ci
noma
Cadenocar cinoma
Dsmal lcellcarci
noma
Emucousorcol l
oidcarci
noma

41.Mal i
gnanttumors ar
isi
ng f
rom epi
thel
i
al
ti
ssuear eref
err
edtoas:
ASar comas
BAdenocar ci
nomas
CPapi ll
omas
DCy stadenomas
EPoly ps

42. The f ir
st site of met ast
asi
s f or
adenocar
cinoni
aofthecol
onwoul
dmostli
kel
ybe
towhichofthefol
l
owing:
ABrain
BLiver
CLung
DLymphnodes
ESpl
een

43.Car ci
nomas f
ormi
ng gl
andul
ar st
ruct
ures
cal
l
ed:
ASarcomas
BPapill
omas
CAdenocarci
nomas
DCystadenomas
EPolyps

44.Adenocarcinomaspr opert
y:
Adevelopsfrom connectiveti
ssue
Bdevelopsfrom theglandularepi
thel
i
um
Corgan-specif
ic
Dthereisnoat ypi
a
Edoesnotgi vemet ast
asis

45.Alloft he f ol
l
owing mor
phol
ogicfeat
ures
char
act eri
zetheadenocarci
nomacel
ls,
EXCEPT:
AVariationinsizeandshape
BHyper chromat i
cnucl
ei
CEnlargednucleol i
DHypochr omat icnucl
ei
EAtypicalmitoses

46. Which hi
stol
ogi
cty
pe of car
cinoma i
s
undi
ff
erent
iat
ed?
A.
Adenocarci
noma.
B.
Carci
nomai ncit
u.
C.
Nonkerat
ini
zingsquamouscel
lcarci
noma.
D.
Kerat
ini
zi
ngsquamouscellcar
cinoma.
E.
Smallcel
lcarci
noma.

47.Carcinomahi stologi
cal
lyconsi
stsofsmal
l
l
ymphocy te-
li
kecells.Whichty
peofcarci
nomas
i
sit?
Ascir
rhouscar ci
noma
Bsoli
dcar ci
noma
Cmucousorcol l
oidcarci
noma
Dmedullarycarci
noma
Esmallcellcarci
noma

48.Whichofthef ol
l
owingisabenigntumor?
AChondr obl
asti
costeosarcomaofthebone
BGranulomaoft hesoftt
issue
CTuberculoma
DPapil
laryserouscyst
adenomaott heovar
y
EPapil
larycarci
nomaoft hethy
roi
d

49.Al
lofthefol
l
owingneopl
asmsar
emal
i
gnant
,
EXCEPT:
AAdenocarci
noma
BMelanoma
CSeminoma
DChor
ionepi
thel
ioma
EPapi
l
larycy
stadenoma

50.Whi ch of the foll


owing defeat
s of the
repr
oduct i
vesystem ofwomeni saprecancer
ous?
ACy st
icatrophyoftheendomet hum
BEndocer vi
calpoly
ps
CCondy lomaacumi natum oft
hev ul
va
DEndomet r
ialpol
yps
EAtypicalhyperpl
asiaoftheendometri
um

51.Thei mpor t
antpr
ognost
icfeat
uresininvasi
ve
breastcancerareall
ofthefol
l
owing,EXCEPT:
AHi stol
ogictypeofthet
umor
BGr adeoft hetumor
CSi zeofthetumor
DSecondar ynecrosi
s
EPr esenceorabsenceofestrogenreceptorson
tumorcells

52.Themostcommonbeni gnt
umoroft
heov
ary
i
swhi choft hefol
l
owi
ng:
APapi l
loma
BFibroma
CCy stadenoma
DAdenocar cinoma
EMel anoma
53.The mostcommon beni gn t
umoroft
he
femalebreastiswhi
choft
hef
oll
owi
ng:
ACy stadenoma
BFibroadenoma
CSar coma
DFibroma
EAdenocar ci
noma

54.How call
edbenigntumorfr
om theglandul
ar
epi
thel
ium wi
thasigni
fi
cant
lydevel
opedstroma:
Afibr
oma
Badenoma
Cfibr
osarcoma
Dpapill
oma
Efi
broadenoma

55.Car cinoma hi stol


ogi
call
y consi
sts oftwo
patter
ns: adenosquamous carci
noma
(adenocarcinoma+squamouscel lcarci
noma)
.
Whicht ypeofcar ci
nomasi si
t?
Asci r
rhouscarcinoma
Bsol i
dcarcinoma
Cmucousorcol l
oidcarci
noma
Ddi morphouscar cinoma
Esmal lcellcar
cinoma

56. Car ci
noma hi stol
ogi
cal
ly - stroma
predomi
nates anapl
ast
ic par
enchy
ma. Whi
ch
t
ypeofcarcinomasisit
?
Afi
brouscancer
Bsol
idcarci
noma
Cmucousorcol l
oidcar
cinoma
Dmedullar
ycarci
noma
Esmallcel
lcarci
noma

57.Malignanttumor ,arisi
ngf
rom t
ransi
ti
onal
cel
l
,
i
snamed:
Asquamouscel lcarcinoma
Bacinaradenocar cinoma
Cpapill
aryadenocar cinoma
Dtransi
tionalcel
l carcinoma
Epapill
oma

58.Ori
ginofchor i
onepi
thel
i
oma:
Aendomet r
ium
Bplacenta
Cmy omet ri
um
Dumbi l
icalcord
Eteka–t issue

59. What i s t he charact


eri
sti
c of
chori
onepitheli
oma:
Adev elopsfrom ut
eri
net
issue
BExpansi vegrowth
Cf eatureofst romastr
uctur
e-manycol l
agen
fi
bers
D t he pr imar
y way of met
ast
asi
s i
s
hematogenous
Edonotmet ast
asize

60.Sel
ectt heor gan-speci
fi
ctumoroft
hepi
tui
tar
y
gl
and:
AScirr
houscar cinoma
Beosinophi l
icadenoma
Cpheochr omocy toma
DSmal lcellcarcinoma
Epapil
loma

61.In whi ch or
gan t
he pheochr
omocy
toma
dev
elops:
Apit
uit
arygland
Bovary
Cadrenalgl
and
DThyroidgl
and
Epancreas

34.Coll
oiddegener
ati
oni
soccur
sin:
Sali
varygland.
Prostategland.
Thyroidgland.
Pi
tuit
arygl
and.
Parat
hyroi
dgland

Mi
scel
l
aneous2
Ext
racel
l
ularaccumul
ati
ons

1.An87- year-oldmal edev el


opswor seningheart
fail
ur e.Wor kupr evealsdecreasedlef tventri
cul
ar
fi
ll
ingduet odecr easedcompl ianceoft heleft
vent ri
cle.Twomont hslaterthepatientdies,and
post mor tem sect i
onsr evealdepositsof
eosi nophilic,Congor edpositivemat eri
al i
nthe
i
nt erstiti
al ofhishear t.Whenv iewedunder
polar i
zedl ight.Thi smat eri
aldispl
ay san
appl e-greenbi refringence.Whati sthecor rect
diagnosi s?
Amy loidosis
Gly cogenosi s
Hemochr omat osis
Sar coidosi s
Seni leatrophy

2.I
ncasesofrenalf
ail
ureonlong-
ter
m
hemodi
aly
sis,
therei
sdev el
opmentoffol
l
owi
ng
ty
peofamy l
oid:
Amyl
oidl
ightchain(AL)
Amyl
oid-
associatedprot
ein(AA)
Amyl
oidβ2microglobuli
n(Aβ2m)
βamyl
oidprotein(Aβ)
I
mmunoglobulinheavychainamyloi
d(AH)

3.Themostcommonf or
m ofamy
loi
dint
hir
d
worldcountr
iesis:
Pri
mar y
Secondary
Heredit
ary
Locali
zed
Endocri
neamy loidosi
s

4.Cardiacamy loidosi softenpr


oduces:
Dil
atedcar di
omy opat hy
Constri
ctivecardiomy opathy
Restri
ctiv
ecar diomy opathy
I
schemi ccardiomy opathy
I
diopathiccardiomy opathy

5.I
nsenil
ecar
diacamy
loi
dosi
s,t
hebi
ochemi
cal
for
m ofamyl
oidis:
AL
AA
ATTR
Aβ2m
AL+AA

6.Heal
thri
skinobesit
yisduetowei
ghti
nexcess
ofthef
oll
owingforageandsex:
10%
20%
30%
40%
50%

7.Obesityi
sdueto:
Hyperpl
asiaofadipocy
tesonl
y
Hypertr
ophyofadipocyt
esonly
Hyperpl
asiaaswel l
ashyper
trophyofadi
pocy
tes
Fatt
ychangeinliv
eronly
Fatt
ychangeinliv
erandhartonly

8.Mostof t
ensecondar yamyloi
dosi
soccur
s
whent hefol
lowingpathologi
es:
Chronicsuppurati
on
Acuteinfl
ammat ion
Cel
lularnecr
osis
Hyali
nedegener ati
on
c.I
schemi a

9.Thesubst
ancewit
hf i
bri
l
larstructur
e,whi
ch
for
msundert
hepat
hologi
calcondi
tions,
is:
Reabsorpti
ondropl
ets
Russell
bodies
Li
pids
Amy l
oid
Calci
fi
cates

10. Thesubstancegi
vi
ngr
edcol
orwi
tht
heCongo
redst ai
nis:
Lipid
Hy ali
ne
Wat er
Amy l
oid
Glycogen

11.I nl ong-st
andinghy
pertensionanddi abet
es
mel li
tus,thewallsofar
ter
ioles,especi
all
yinthe
ki
dney ,become:
Ser ous
Thined
Hy ali
nized
Ulcered
Pigment ed

12.The pat hologic pr


otei
naceous substance,
accumul at
ing onl y bet
ween cellsinv ari
ous
ti
ssuesandor gansofthebodyis:
Glycogen
Hyaline
Water
Amyloi
d
Li
pid

13.The pat hol


ogi
c protei
naceous substance,
accumul at
ing both wit
hin cells and in the
extracel
lul
armatri
xinvari
oustissuesandorgans
oft hebodyis:
Glycogen
Hy ali
ne
Wat er
Amy l
oid
Lipid

14.Thecharacteri
sticsofamy loi
dfibr
il
sincl
ude
al
lofthefol
lowing,except:
Fi
bri
lcomposedofpai redfi
laments
Nonbranchi
ngfibri
ls
Fi
bri
lswithanindefinitedi
ameter
Fi
bri
lswithi
ndefinit
el engt
h
Fi
bri
lswithdefi
nitelength

15.The organ aff


ected in bot
h pr
imar
y and
secondar
yamyloi
dosisis:
Kidney
Stomach
Uterus
Brain
Lung

16.Thedepositsofamy l
oidseesi nallt
issues,
except:
Mesangium andcapil
l
aryloops
Basementmembr anesofbloodv essel
s
Tubularbasementmembr anes
Epit
heli
um inpr
oximalrenaltubul
es
Int
ersti
ti
um oft
hekidneys

17.Al l
ofthef ol
lowi
ngpathol
ogi
calpr
ocessesar
e
reversi
ble,except:
Mucoi dchanges
Cellul
arswel l
ing
Amy l
oidosi
s
Edema
Lipi
daccumul at
ioninhepat
ocyt
es

18.The organ mostcommonl


yand ser
iousl
y
damagedinamyloi
dosi
sis:
Stomach
Kidney
Lung
Li
v er
Uterus

19.
React
ivesy
stemicamyl
oidosi
sknowst
obeal
l
oft
hefol
l
owing,except
:
Secondaryamyloi
dosis
Complicati
onoftubercul
osi
s
Heredi
taryamyl
oidosis
Complicati
onofosteomyeli
ti
s
Complicati
onofbronchoect
ati
cdi
sease

20.Amy l
oidin pr
imaryamy
loi
dosi
sis usuall
y
syst
emic and bel ongs t
o t he fol
lowing
bi
ochemicalt
ype:
ALtype
AAtype
Aβtype
ATTRtype
Aβ2m t
ype

21.React ive systemic amy


loi
dosi
s occursin
associati
on wi t
h allofthefol
lowi
ng di
seases,
except:
Tuberculosis
Bronchiectasis
Chronicost eomyeli
ti
s
Rheumat oidarthri
ti
s
Hepatiti
sB

22.Reactive systemic amy


loi
dosi
s occursin
associ
ati
on with allofthefol
lowi
ng di
seases,
except
:
Rheumatoidart
hrit
is
Ankylosi
ngspondyli
ti
s
Chronicappendi
cit
is
Myeloma
Tubercul
osis

23.Secondar yamyloi
dosisdamagesal
loft
he
foll
owi ngor except
gans, :
Kidney s
Liver
Spleen
Adr enals
Brain

24.Macr oscopi
call
yt he organs af
fect
ed by
amy l
oidosis are char
acter
ized by al
lof the
fol
lowing,except
:
Enlar
ged
Fir
m
Waxyinappear ance
Smoot h
Soft

25.Thecommoncauseofdeathinpatient
swi
th
secondaryamy
loi
dosi
sisi
nsuf
fi
ciencyof:
Kidneys
Heart
Li
v er
Lung
Adrenal
s

26.The or gans that shoul


d be hist
ologi
cal
examinedinpatient
swithamy l
oidosi
sareallof
thefoll
owi
ng,except:
Kidney
Eye
Rectum
Gingiv
a
Skin

27. The r ever


sibl
e pr ocess caused by
accumul ation of gl ycosaminogl
ycans i n
extracell
ular matri
x due t o the incr
ease of
vascularper meabil
it
yis:
Amy l
oidosis
Glycogenoses
Hy al
inosis
Mucoi dchanges
Fibri
noidchanges

28. The i rrever


sibl
e pr ocess caused by
accumulati
on ofpr ot
eins wit
h hi
gh mol ecul
ar
weight associ at
ed wi th the dest
ruct
ion of
connecti
vetissueis:
Amy loi
dosi
s
Glycogenoses
Hyal
inosi
s
Mucoidchanges
Fi
bri
noidchanges

29.All thepathol
ogi
cal
processesar
eir
rev
ersi
ble,
except :
Mucoi dchanges
Fibr
inoidchanges
Amy loidosis
Apopt osis
Necrosi s

30.Inamyloidosismacroscopi
call
yki
dney
changedinthef oll
owingway:
Cutsurf
acei spaleandt r
ansl
ucent
Denseandshar plyreduced
Enlar
gedanddense
Softandenlarged
Truea+c

31.Al l
exampl esofext r
acell
ularhy
ali
near
e
change, except:
Oldscar
Hyalinearteri
oloscler
osis
Chronicglomer ulonephri
ti
s
Mallory’shyal
ine
Cornor aamy l
aceai nthebrai
ni nol
dage
32.Inamyloidosismacroscopi
cal
l
yli
verchanged
i
nt hefol
l
owi ngway :
Enlar
ged,pale,waxyandf i
rm
Denseandshar pl
yreduced
Enlar
gedandy ell
owcolor
Softandenlarged
Trueb+c

33.Inamyloidosismacroscopical
l
yspleen
changedinthef oll
owingway :
Cutsurf
acei stransl
ucentpaleandwaxy
Denseandshar plyreduced
Cutsurf
ace–map- l
ikeareasofamyl
oid
Softandenlarged
Truea+c

34.
Causesofhy al
inosi
sar
eal
loft
hef
oll
owi
ng,
EXCEPT:
Fi
bri
noidchanges.
I
nfl
ammat i
on.
Apoptosis.
Scl
erosis.
Necrosi
s.

35.Mostspeci
fi
chi
stol
ogi
cal
signi
nmucoi
d
changesis:
Metachromasi
a.
Metaplasi
a.
Met
ast
asis.
Met
aki
nesis.
Met
amorphosis.

36.
Mostcommonl ocal
i
zationofmucoi
dchanges
i
s:
A.Ner voussy stem.
B.Cardiovascul arsystem.
C.Urinarysyst em.
D.Bloodsy stem.
E.Endocr i
nesy stem.

37.Simplehy al
ineoccur
sin:
Diabetesmel li
tus.
Arteri
alhypertensi
on.
Rheumat i
cf ever.
Rheumat oidarthri
ti
s.
Systemiclupuser yt
hematosus.

38.Simplehyal
ineoccur
sin:
Diabetesmelli
tus.
Systemiclupuseryt
hematosus.
Atheroscl
erosi
s.
Rheumat i
cfever.
Rheumat oi
darthri
ti
s.

39.Li
pohyal
inoccur
sin:
A. Diabetesmel
li
tus.
Arter
ialhypert
ension.
Atheroscl
erosis.
Rheumat i
cf ev
er .
Rheumat oidarthr
iti
s.

40.Compoundhy al
inoccur
sin:
Diabetesmelli
tus.
Arter
ialhy
pertension.
Atheroscl
erosi
s.
D. Rheumat i
cfever
.
E. Vasculit
is.

41.Compoundhy alinoccursi
n:
Diabetesmel lit
us.
Art
eri
al hypertension.
Atherosclerosis.
Vasculit
is.
E. Rheumat oi
dar thri
ti
s.

42.
Thestainusedt oidenti
fyamy
loi
dis:
Hematoxyl
inandeosi nstai
n.
Metachr
omat i
cst ai
n.
Congo-r
edstain.
SudanII
Istai
n.
Tol
uidi
nbluest ai
n.

43.The pathol
ogi
c protei
naceous substance,
accumulat
ing onl
y between cell
sinv ari
ous
ti
ssuesand
organsofthebodyi
s:
A. Gl y
cogen.
B. Hy ali
ne.
C. Wat er.
D. Amy loi
d.
E. Lipi
d.

44.Specifi
cmet hodfordi
agnosi
sofamy
loi
din
fr
esht issueis:
Vir
chowt est
.
Rokitanskytest.
Ewingt est.
Massont est
.
Papanicolaoutest.

45.
Variantofsystemi
camy
loi
dosi
sis:
Seni
lecardiac.
Secondary.
Seni
lecerebral.
Endocri
ne.
Tumor-for
mi ng.

46.
Variantofsystemi
camy
loi
dosi
sis:
Senil
ecardiac.
Senil
ecerebral.
Heredofamili
al.
Endocri
ne.
Tumor
-f
ormi
ng.

47.
Vari
antofsy stemi
camy loi
dosi
sis:
Seni
lecardiac.
Seni
lecerebral.
Endocr
ine.
Hemodialysis-
associ
ated.
Tumor-f
ormi ng.

48.
Variantoflocal
izedamyloi
dosi
sis:
Senil
ecardiac.
Pri
mar y.
Secondary.
Heredofamili
al.
Hemodi al
ysis-
associat
ed.

49.
Variantoflocal
izedamyloi
dosi
sis:
Pri
mar y.
Secondary.
Senil
ecerebral.
Heredofamil
ial.
Hemodi al
ysi
s-associat
ed.

50.
Variantoflocal
i
zedamy
loi
dosi
sis:
Pri
mar y.
Secondary.
Heredofamil
ial.
Endocri
ne.
Hemodi
aly
sis-
associ
ated.

51.
Variantoflocal
izedamyloi
dosi
sis:
Pri
mar y.
Secondary.
Heredofamil
ial.
Hemodi al
ysi
s-associat
ed.
Tumor-formi
ng.

52.Locali
zati
onofper
icol
l
agenousamy
loi
dosi
sis:
Li
v er
.
Spleen.
Heart.
Kidneys.
Suprarenalgl
ands.

53.Locali
zati
onofper
icol
l
agenousamy
loi
dosi
sis:
Li
v er
.
Spleen.
Bowel.
Kidneys.
Suprarenalgl
ands.

54. Locali
zat
ionofper
icol
l
agenousamy
loi
dosi
sis:
Li
v er.
Spleen.
Ner ves.
Kidney s.
Supr
arenal
glands.

55.Local
izat
ionofper
ir
eti
cul
i
namy
loi
dosi
sis:
Heart.
Tongue.
Nerves.
Kidneys.
Bowel.

56.
Locali
zat
ionofper
ir
eti
cul
i
namy
loi
dosi
sis:
Heart.
Tongue.
Nerves.
Li
ver.
Bowel.

57.
Locali
zat
ionofper
ir
eti
cul
i
namy
loi
dosi
sis:
Heart.
Tongue.
Nerves.
Spl
een.
Bowel.

58.Causeofobesitywhichassoci
atedwi
th
excessivenutr
it
ioniscall
ed:
Pri
mar y.
Ali
ment ary.
Cerebral.
Endocr
ine.
Heredi
tar
y.
B

59.
Ty peofobesi
tywi
thunknowncausei
scal
l
ed:
Pri
mar y.
Ali
ment ary.
Cerebral.
Endocrine.
Hereditar
y.

1stdegr
60. eeofobesi
tyisassociat
edwit
h
i
ncreasi
ngofbodyweightoverthannor
malby
:
20-
29%.
10-
20%.
15-
25%.
20-
35%.
15-
29%.

2nddegr
61. eeofobesi
tyi
sassoci
atedwit
h
i
ncreasi
ngofbodywei
ghtovert
hannormalby
:
30-
40%.
30-
49%.
25-
45%.
30-
55%.
35-
55%.

3rddegr
62. eeofobesi
tyi
sassoci
atedwi
th
i
ncreasi
ngofbodywei
ghtov
ert
hannor
mal
by:
50-
99%.
45-
85%.
50-
75%.
60-
90%.
50-
90%.

4thdegr
63. eeofobesi
tyisassociat
edwith
i
ncreasi
ngofbodyweightoverthannor
mal by
:
90%andmor e.
85%andmor e.
120%andmor e.
95%andmor e.
100%andmor e.

64.Deposit
ionoffati
nabdomenar
eai
nobesi
tyi
s
call
ed:
Uppertype.
Middletype.
Lowertype.
Simmet r
ictype.
Dif
fusetype.

65.
Depositi
onoffati
nar
eaoff
aceandnecki
n
obesi
tyi
scalled:
Upperty
pe.
Middl
etype.
Lowertype.
Si
mmet r
icty
pe.
Di
ffuset
ype.

Pi
gment
ati
ons.Cal
cif
icat
ions

1.A38- year-
oldf emal epresentswi t
hi ntermit
tent
pelv
icpai n.Physi calexaminati
onr evealsa3- cm
massi nthear eaofherr i
ghtov ar
y .Histologi
cal
secti
onsf rom thisov ari
anmassr eveal apapill
ary
tumorwi thmul ti
pl e,
scatteredsmal l
,round, and
l
ami nat edcalcificati
ons.Thesest ructuresar e
mostl ikelyther esultof
Apopt osis
Dystrophi ccalcificat
ion
Enzymat i
cnecr osi s
Hyper parathyroidism
Metast aticcalcificat
ion

2.Thef ol
lowingpigmentsarestai
nabl
eby
Prussi
anbl uereactonexcept
i :
Hemosi deri
n
Ferr
iti
n
Hemat i
n
Hemochr omat osi
s
Bil
ir
ubin

3.I
diopat
hiccal
cinosi
scut
isi
sanexampl
eof
:
Necr oti
zinginfl
ammat ion
Dystrophiccalcif
icat
ion
Met astati
ccalcif
icat
ion
Calcifi
edt hrombiinveins
Calcifi
edbl oodvessels,especi
all
yont
hei
nter
nal
el
ast iclamina

4.Riskf actorsimpl i
cat
edi
ntheet
iol
ogyof
cholesterol gal
lstonesi
ncl
udet
hefol
lowi
ng
except :
Fami l
yhi story
Obesity
Hemol yticanemi a
Oralcont raceptiv
es
Geogr aphy

5.Thef ol
lowingt
ypeofgall
stonesi
sgeneral
ly
unassociatedwit
hchangesinthegal
lbl
adderwal
l
:
Cholester
ol
Mixed
Combi ned
Pigment
Uricacid

6.Thefol
lowi
ngtypeofr
enal
cal
cul
ii
sradi
olucent
:
Calci
um oxal
ate
Str
uvit
e
Uri
cacid
Cal
ci
um phosphat
e
Pi
gment

7.Thef ol
lowingty
peofr
enal
cal
cul
ii
s
i
nfecti
on-induced:
Calci
um oxalate
Str
uv i
te
Uri
caci d
Cysti
ne
Cholester
ol

8.Thecolorofor
ganshemosi
der
ini
s:
Black
Sky-bl
ue
Yell
ow
Broun
Goldenyell
ow

9.Thepigmenthemat
oidi
nismosti
dent
ical
wit
h:
Porphy
rin
Bil
i
rubi
n
Hemosideri
n
Hemozoin
Li
pochrome

10.Lipofusci
n,t
hegol
deny
ell
owpi
gmentseesi
n
heartmuscl e
Hypertrophy
At
rophy
Hyperplasi
a
Metaplasia
Dy
splasi a

11.Whatst ai
nisspecifi
cfori
ron:
Hemat oxyl
inandeosin
SudanI I
I
Prussianbluereact
ion
Congor ed
PAS-reacti
on

12.Hemosi derinhasal lofthefol


l
owi
ngf
eat
ures,
except:
Hemogl obin-der i
ved
Goldeny ell
ow-t o-br
own
Granularorcr ystall
i
ne
Synthesizedbyenzy metyrosi
nase
i
saggr egateoff err
it
in

13.Hemosi deri
nint
hel
ungaccumul
atesi
n:
Leukocytes
Lymphocy tes
Macrophages
Fi
broblasts
Eryt
hrocytes

14.I
diopat
hicpul
monar
yhemosi
der
osi
s
characteri
zesbyal l
oft
hef
oll
owi
ngpat
hol
ogi
c
sympt oms, except
:
Productivecough
Hemopht ysis
Anemi a
Heav yproteinur
ia
Weightloss

15.Thel ungsinpulmonaryhemosi derosi


sare:
Enlargedwithturbi
dflui
dexudingfrom cut
surface
Increasedinweightwithareasofred-brown
consolidati
on
Dimi ni
shedwithareasofred-br
ownconsol idat
ion
Dimi ni
shedwithturbi
dflui
dexudingf r
om cut
surface
Unchanged

16.Thecol orofhemosi
deri
ngranul
esst
ained
withPrussianbluer
eact
ionis:
Yell
ow
Brown
Orange-red
Pink
Blue-
black

17.Hemosiderosi
sseesi
nal
lpat
hol
ogi
c
pr except
ocesses, :
I
nfl
ammation
Her
edi
tar
yincreasedabsor
pti
onofdi
etar
yir
on
I
mpair
eduseofi r
on
Hemol
yti
canemi a
Mal
ari
a

18.Mor phologicchangesi ngeneti


c
hemochr omat osi
schar acter
izesbyal
loft
he
fol
lowi ng,except:
Met astaticcalci
fi
cati
onofmanyor gans
Deposi ti
onofhemosi derininmanyorgans
Deposi ti
onofhemosi derinintheski
n
Liv
erci rrhosis
Pancr eaticfi
brosis

19.Melaninhasallofthef
oll
owi
ngf
eat
ures,
except:
Locali
zedendogenous
Formedoff er
rit
in
Non-hemoglobin-
deri
ved
Black-
brown
Formedi nmelanocyt
es

20.Anincr
easedamountofmelani
nin
melanocyt
esandwit
hinbasal
kerat
inocy
tesi
s
al
soknownas:
Vacuoli
zat
ion
Vit
il
igo
Hyper
pigment
ati
on(
mel
anosi
s)
Al
bini
sm
Hyper
kerat
osi
s

21.Lipofuscinhasallofthefol
l
owingfeatur
es,
except:
Agingpi gment
Mostof tenseeni nki
dney
Yell
ow- brown
Noninjurioustothecell
sorthei
rfunct
ion
Endogenouspi gment

22.Lipofuscinpigmentgranulesincel
l
sresul
t
fr
om:
Hemosi derosis
Accumul ati
onofpr otei
nincytopl
asm
Accumul ati
onofl i
pidsincyt
oplasm
Cell
ularswelli
ng
I
ntracell
ularli
pidperoxi
dati
on

23.Lipof
uscingranulesincell
sseein:
Necrosis
Denervati
onatr
ophy
Brownat r
ophy
Atrophyfr
om pressure
Atrophyfr
om diminishedbloodsupply
24.Heartandl
i
verofapat i
entwi
thcancer
cachexi
amacroscopical
l
yseesas:
Dimini
shedandbrown
Dimini
shedandyell
ow
Enlar
gedandbrown
Enlar
gedandyell
ow
Unchanged

25.Lipof
uscinintheliv
ermayfoundsi
n:
Unchangedcel l
s
Cell
swi t
hballooni
ngdegener
ation
Cell
swi t
hhyalinedropl
ets
Cell
swi t
hregressi
vechanges
Necroti
ccell
s

26.Bili
rubinhasal loft
hefoll
owingfeat
ures,
except:
Theendpr oductofhem degradati
on
Derivedfrom breakdownerythrocy
tes
Bril
li
ant-y
ellow
Stainedinbl ue-
blackcol
orwithPrussi
an-bl
ue
reacti
on
Existsi
nt wof orms-conjugatedand
unconjugated

27.Jaundiceoccursinalloft
hefol
lowi
ng
pathol
ogicprocesses,except
:
I
ncreasedhepatocell
ularexcr
eti
on
Excessi
veproduct
ionofbil
ir
ubi
n
Reducedhepatocyt
euptake
I
mpai r
edconjugat
ionofbil
i
rubi
n
I
mpai r
edbil
ef l
ow

28.Bil
iaryductsobstruct
ionbygal
lst
onesmay
l
eadt o:
Li
verhemosi derosis
Li
versteatosis
Ball
ooningdegener at
ionofhepat
ocytes
Cholestasi
sandj aundice
Hemochr omatois

29.Compl icati
onsorwel l
-est
abl
ished
associ at
ionsofgal lst
onesincl
udealloft
he
fol
lowi ng,except :
Bil
iaryobst ructi
on
Brownat r
ophyoft heli
ver
Pancr eati
tis
Int
estinal obstructi
on
Mal i
gnancy

30.Whi chofthefoll
owi ngsi
tesi
sanexampl
eof
met astati
ccalci
fi
cati
on?
Theki dneyinnephrocalci
nosi
s
Themi t
ralval
veinmitralst
enosi
sofr
heumati
c
organ
Theleftant
eri
orascendingcoronar
yarter
y
aff
ectedbyatheromatousplaques
Thelunginvol
vedbymet astati
ccar
cinoma
Thelunginareasofoldtubercul
osi
sfoci

31.Pointoutthepost
tuber
cul
osi
slungl
esi
on:
Granul
oma
Cavit
y
Fi
brocalcif
icscar
Caseationinlymphnode
Caseationinlung

32.Dystrophiccalci
ficat
ionencount
ersi
nal
lof
thefoll
owi ngareas,except:
Coagulati
v enecrosi
s
Intr
acell
ularfataccumul at
ion
Caseousnecr osis
Liquef
activenecrosis
Enzymat i
cnecr osi
soff at

33.Ini
tiat
ionofintracel
lul
arcal
cif
icat
ionoccur
sin:
Cytoplasm
Lysosome
Nucleus
Endoplasmicr et
iculum
Mitochondria

34.Thecausesofmet
ast
ati
ccal
cif
icat
ionar
eal
l
oft hefol
lowing,except:
Diabetesmel li
tus
I
ncr easedsecretionofparathy
roi
dhor
mone
Dest r
ucti
onofbonet issue
VitaminD- r
elateddisorder
s
Renal fai
l
ure

35. Metast
ati
ccalcif
icat
ionmayoccurinal
loft
he
foll
owingor except
gans, :
Stomach( gast
ri
cmucosa)
Kidneys
Lungs
Liver
Hear t
,syst
emicarteri
esandpulmonaryvei
ns

36.Deficiencyofvi
tami
nDt
endst
ocause:
Hypercalcemia
Hypocalcemi a
Hyperpigment at
ion
Hypopigment ati
on
Calci
fi
cat i
on

37.Defi
ciencyofvi
tami
nDi
ntheadul
tsl
eadst
o:
Osteoscl
er osis
Osteomy elit
is
Osteomal atia
Osteonecr osis
Osteoporosi s
38.
Hemogl obi
nderi
vat
ivepi
gmentwhi
choccur
s
i
nnormal condi
ti
ons:
Hemosideri
n.
Hematoidi
n.
Hemin.
Hemomel ani
n.
Por
phyri
n.

39.
Hemoglobinderi
vat
ivepi
gmentwhi
choccur
s
i
nnormalcondit
ions:
Fer
ri
ti
n.
Hematoi
din.
Hemin.
Hemomelanin.
Por
phyri
n.

40.Hemoglobinderi
vat
ivepi
gmentwhi
choccur
s
i
nnor malcondit
ions:
Bil
i
rubin.
Hemat oi
din.
Hemi n.
Hemomel anin.
Porphyri
n.

41.
Hemogl obinder
ivat
ivepi
gmentwhi
choccur
s
onl
yinpathologi
cconditi
ons:
Hemosideri
n.
Hemat oi
din.
Ferr
it
in.
Bil
i
rubin.
Melanin.

42.Hemogl obinder
ivat
ivepi
gmentwhi
choccur
s
onl
yi npathologi
cconditi
ons:
Hemosi deri
n.
Porphyri
n.
Ferri
ti
n.
Bil
i
rubin.
Melanin.

43.Hemogl obinder
ivat
ivepi
gmentwhi
choccur
s
onl
yi npathologi
cconditi
ons:
Hemosi deri
n.
Hemat i
n.
Ferri
ti
n.
Bil
i
rubin.
Melanin.

44.I
ron-contai
ningpi
gmenti
s:
Bil
i
rubin.
Hemat oidi
n.
Hemosi deri
n.
Porphyrin.
Melanin.
45.I
ron-contai
ningpi
gmenti
s:
Bil
i
rubin.
Hemat oidi
n.
Ferr
iti
n.
Porphyrin.
Melanin.

46.I
ron-contai
ningpi
gmenti
s:
Bil
i
rubin.
Hemat oidi
n.
Hemat i
n.
Porphyrin.
Melanin.

47.The st ain used toident


if
yir
on-
cont
aini
ng
pi
gment sis:
Prussian-bluereacti
on.
PASr eaction.
Congo- r
edst ain.
SudanI I
Ist ai
n.
Tolui
dinbl uest ai
n.

48.
Inhemosi derosisorgansbecome:
Enl
arged,brown, hard.
Small
,red,soft.
Enl
arged,yell
ow, hard.
Enl
arged,brown, soft.
Small
,brown, hard.
49.Causeofgener alizedhemosider
osi
sis:
I
ntracerebralhemor rhage.
I
ntoxicati
ons.
Browni ndurati
onofl ungs.
Pulmonar yhemor rhagicinf
arct
.
Petechialhemor r
hagesi nskin.

50.Causeofgener al
izedhemosider
osi
sis:
I
ntracerebralhemor rhage.
Heterohemot ransfusions.
Browni ndurati
onofl ungs.
Pulmonar yhemor r
hagicinf
arct
.
Petechialhemor rhagesinskin.

51.Causeofgener
ali
zedhemosi
der
osi
sis:
Typhoidfever
.
Yell
owf ever
.
Malaria.
Diphteri
a.
Scarl
etfever.

52.
Causeoflocali
zedhemosi
der
osi
sis:
I
ntoxi
cati
ons.
Heter
ohemotransf
usions.
Malar
ia.
Bonemarrowdiseases.
Br
owni
ndur
ati
onofl
ungs.

53.
Causeofl ocali
zedhemosi
der
osi
sis:
I
ntoxicati
ons.
Heterohemotransf
usions.
Malaria.
Bonemar r
owdiseases.
I
ntracerebral
hemor r
hage.

54.
Causeoflocali
zedhemosiderosi
sis:
I
ntoxi
cati
ons.
Heter
ohemotransf
usions.
Malar
ia.
Bonemarrowdiseases.
Pul
monaryhemor rhagi
cinf
arct
.

55.Accumul ati
onofwhichpi
gmentmayl
eadst
o
decreaseofbl oodpr
essure?
Ferri
ti
n.
Hemosi deri
n.
Bil
i
rubin.
Hemomel anin.
Porphyri
n.

56.
Accumulat
ionofwhi
chpi
gmentmayl
eadst
o
j
aundi
ce?
Fer
ri
ti
n.
Hemosider
in.
Bi
li
rubi
n.
Hemomelani
n.
Por
phyri
n.

57.
Causeofpr ehepati
cjaundiceis:
Hepati
ti
s.
Li
vercir
rhosi
s.
Obstr
ucti
onofbi l
eductsbyst ones.
Compressionofbileductsbyt umors.
Hemolyti
cdiseaseofnewbor ns.

58.
Causeofpr ehepaticj
aundi
ceis:
Hepati
ti
s.
Li
vercir
rhosi
s.
Obstr
ucti
onofbi l
educt sbyst
ones.
Compressionofbileductsbytumors.
Heter
ohemot r
ansfusions.

59.
Causeofhepat ocell
ularj
aundiceis:
Obstr
ucti
onofbi l
educt sbystones.
Compressionofbileductsbytumor s.
Heter
ohemot r
ansfusions.
Hepati
ti
s.
Hemolyti
cdiseaseofnewbor ns.

60.
Causeofhepatocel
lul
arj
aundiceis:
Obst
ructi
onofbi
leductsbystones.
Compressi
onofbileduct
sbytumor s.
Heter
ohemot r
ansfusi
ons.
Li
vercir
rhosi
s.
Hemolyti
cdiseaseofnewbor
ns.

61.
Causeofpost hepati
cjaundiceis:
Obstr
ucti
onofbi l
educt sbystones.
Hepati
ti
s.
Heter
ohemot r
ansfusions.
Li
vercir
rhosi
s.
Hemolyti
cdiseaseofnewbor ns.

62.
Causeofpost hepati
cjaundicei
s:
Compressionofbileductsbytumors.
Hepati
ti
s.
Heter
ohemot r
ansfusions.
Li
vercir
rhosi
s.
Hemolyti
cdiseaseofnewbor ns.

63.Whi
chpi gmenti
s“t
earandwear
”:
Bil
i
rubi
n.
Hematoidin.
Hematin.
Li
pofuscin.
Melani
n.

64.Whi
chpigmenti
sty
rosi
n-der
ived:
Bil
i
rubi
n.
Hematoidi
n.
Hematin.
Li
pofusci
n.
Melani
n.

65.General
izedhy per
pigment
ati
onofmel
ani
n
occursin:
Addison’sdisease.
Melanosiscoli.
Lenti
go.
Nevus.
Melanoma.

66.
Focal hyperpigment
ati
onofmel
ani
noccur
sin:
Addison’sdisease.
Cachexia.
Avi
taminosis.
Nevus.
Xerodermapi gmentosum.

67.General
hypopi
gment
ati
onofmel
ani
noccur
s
i
n:
Leukoderma.
Albi
nism.
Viti
l
igo.
Cachexia.
Lenti
go.

68.
Focal
hypopi
gment
ati
onofmel
ani
noccur
sin:
Albi
ni sm.
Nev us.
Viti
l
igo.
Cachexi a.
Lentigo.

69.
Distrophiccalcif
icat
ionoccur
sin:
Hyperparathyroi
dism.
Hypervit
ami nosi
sD.
Hyperthyr
oidism.
Necrosis.
I
ncreasedbonecat aboli
sm.

70.Metastati
ccalcif
icat
ionoccur
sin:
Hyperparathyroi
dism.
Damagedhear tvalv
es.
Atheromas.
Necrosis.
Oldthrombi .

71.
Grossappear anceofcal
cif
icat
eis:
Har
d, greywish-
white.
Sof
t,yellow.
Har
d, greywish-
red.
Sof
t,grey wi
sh-whit
e.
Har
d, bl
ack.

72.
Areaofcal
cif
icat
ioni
nhi
stol
ogi
cal
examinat
ionwi
thhemat
oxy
li
nandeosi
nst
aini
ng
i
s:
Red.
Black.
Brown.
Pink.
Blue.

I
mmunopat
hol
ogy

1.
Сongeni
tal
absenceoft
hymusi
scal
l
ed:

А.
Aplasia.
B.
Hypopl asi
a.
C.
Dysplasia.
D.
Atr
ophy .
E.
Thymomegal y.

4.
Abnor
mal
dev
elopmentoft
hymusi
scal
l
ed:

А.
Aplasia.
B.
Hypopl asi
a.
C.
Dysplasia.
D.
Atr
ophy.
E.
Thymomegal
y.

24.Morphol
ogical
lyi
mmediat
ety
peof
hyper
sensit
ivi
tyoccur
sas:

А.
Fibri
noidnecrosis.
B.
Lympho- hysti
ocyti
cinfi
lt
rat
ion.
C.
Macr ophagealinfi
lt
rat
ion.
D.
Granulomat osi
s.
E.
Cytoplasmicbridgesbetweenlymphocy
tesand
macrophages.

25.Morphol
ogical
lyi
mmediat
ety
peof
hyper
sensit
ivi
tyoccur
sas:

A.
Lympho- hysti
ocyti
cinfi
lt
rat
ion.
B.
Macr ophagealinf
il
trat
ion.
C.
Granulomat osi
s.
D.
Cy t
oplasmicbridgesbetweenlymphocy
tesand
macrophages.
E.
Plasmaticsaturati
on.
26.Morphol
ogical
lyi
mmediat
ety
peof
hyper
sensit
ivi
tyoccur
sas:

A.Lympho- hystiocyti
cinfi
lt
rat
ion.
B.Mucoi dchanges.
C.Macr ophageal infi
lt
rat
ion.
D.Granulomat osis.
E.
Cy toplasmicbr idgesbetweenlymphocy
tesand
macr ophages.
27.Mor phologicall
yimmedi at
et y
peof
hypersensitiv
ityoccur sas:

A.
Lympho- hysti
ocyti
cinfi
lt
rat
ion.
B.
Fibri
noidchanges.
C.
Macr ophagealinfi
lt
rat
ion.
D.
Granulomat osi
s.
E.
Cytoplasmicbridgesbetweenlymphocy
tesand
macrophages.

28.Morphol
ogical
lyi
mmediat
ety
peof
hyper
sensit
ivi
tyoccur
sas:

A.
Lympho- hysti
ocyti
cinfi
lt
rat
ion.
B.
Fibr
inous-hemor r
hagicexudate.
C.
Macr ophagealinfi
l
trat
ion.
D.
Granulomat osi
s.
E.
Cyt
opl
asmicbri
dgesbet
weenl
ymphocy
tesand
macr
ophages.

29.Morphol
ogical
signsofi
mmediat
etypeof
hyper
sensit
ivi
tyareal
lthef
oll
owi
ng,EXCEPT:

A.
Ly mpho-hyst
iocyti
cinfi
lt
rat
ion.
B.
Mucoi dandFibrinoi
dchanges.
C.
Plasmaticsaturati
on.
D.
Fibri
noidnecrosis.
E.
Fibri
nous-hemorrhagicexudate.

30.Morphol
ogical
signsofi
mmediat
etypeof
hyper
sensit
ivi
tyareal
lthef
oll
owi
ng,EXCEPT:

A.
Mucoi dandFi brinoi
dchanges.
B.
Plasmat icsatur ati
on.
C.
Gr anulomat osis.
D.
Fibrinoidnecrosi s.
E.
Fibrinous-hemor rhagi
cexudate.

31.Morphol
ogical
signsofi
mmediat
etypeof
hyper
sensit
ivi
tyareal
lthef
oll
owi
ng,EXCEPT:

A.
Mucoi
dandFi
bri
noi
dchanges.
B.
Plasmat i
csaturati
on.
C.
Fibri
noidnecrosis.
D.
Fibri
nous-hemor r
hagicexudat
e.
E.
Macr ophageali
nf i
l
trat
ion.

46.Morphol
ogical
lydelay
edty
peof
hyper
sensit
ivi
tyoccursas:

A.
Mucoi dandFi brinoi
dchanges.
B.
Plasmat icsatur ati
on.
C.
Gr anulomat osis.
D.
Fibrinoidnecrosi s.
E.
Fibrinous-hemor rhagi
cexudate.

47.Morphol
ogical
lydelay
edty
peof
hyper
sensit
ivi
tyoccursas:

A.
Mucoi dandFibrinoidchanges.
B.
Plasmat i
csaturati
on.
C.
Fibri
noidnecrosis.
D.
Fibri
nous-hemor r
hagicexudate.
E.
Macr ophageali
nf i
l
trat
ion.

48.Morphol
ogical
lydelay
edty
peof
hyper
sensit
ivi
tyoccursas:
A.
Mucoi dandFibrinoi
dchanges.
B.
Plasmat i
csaturati
on.
C.
Fibri
noidnecrosis.
D.
Fibri
nous-hemor r
hagicexudat
e.
E.
Cytoplasmicbri
dgesbet weenlymphocy
tesand
macrophages.

49.Morphol
ogical
signsofdel
ayedty
peof
hyper
sensit
ivi
tyareal
lthef
oll
owing,
EXCEPT:

А.
Fibri
noidnecrosis.
B.
Lympho- hysti
ocyti
cinfi
lt
rat
ion.
C.
Macr ophagealinfi
lt
rat
ion.
D.
Granulomat osi
s.
E.
Cytoplasmicbridgesbetweenlymphocy
tesand
macrophages.

50.Morphol
ogical
signsofdel
ayedty
peof
hyper
sensit
ivi
tyareal
lthef
oll
owing,
EXCEPT:

A.
Lympho- hysti
ocyti
cinfi
lt
rat
ion.
B.
Macr ophagealinf
il
trat
ion.
C.
Granulomat osi
s.
D.
Cytoplasmicbridgesbetweenlymphocy
tesand
macrophages.
E.
Plasmaticsatur
ati
on.
51.Morphol
ogicalsi
gnsofdel
ayedty
peof
hyper
sensiti
vi
tyareall
thef
oll
owing,
EXCEPT:

A.
Lympho- hysti
ocyti
cinfi
lt
rat
ion.
B.
Mucoi dchanges.
C.
Macr ophagealinfi
lt
rat
ion.
D.
Granulomat osi
s.
E.
Cytoplasmicbridgesbetweenlymphocy
tesand
macrophages.

52.Morphol
ogical
signsofdel
ayedty
peof
hyper
sensit
ivi
tyareal
lthef
oll
owing,
EXCEPT:

A.Lympho- hystiocyti
cinfi
lt
rati
on.
B.Fi
br i
nous-hemor r
hagicexudate.
C.Macr ophageal infi
lt
rat
ion.
D.Granulomat osis.
E.
Cy toplasmicbr idgesbetweenl y
mphocy t
esand
macr ophages.
61.Mor phologicalsignsoftranspl
antr
eject
ionar
e
al
l t
hef oll
owing, EXCEPT:

A.
Lympho-hystiocyti
cinfi
lt
rat
ion.
B.
Edemaoft ransplant.
C.
Macrophageal inf
il
trat
ion.
D.
Granul
omat osis.
E.
Leukocy
tici
nfi
l
trat
ion.

62.
Morphol
ogicalsi
gnsoft
ranspl
antr
eject
ionar
e
al
lthef
oll
owing,EXCEPT:

A.
Lympho- hystiocy ti
cinfi
lt
rat
ion.
B.
Edemaoft ranspl ant.
C.
Macr ophageal infil
trat
ion.
D.
Cytoplasmicbr idgesbet weenlymphocy
tesand
macrophages.
E.
Leukocyticinfil
tration.

74.
Organspeci
fi
cimmunedi
seasei
s:

А.Hashi motothyroi
diti
s.
B.Rheumat oidarthr
it
is.
C.SystemicLupusEr ythematosus.
D.Scleroder
ma.
E.
Secondar ythrombocy topeni
a.
75.Organspeci f
icimmunedi seasei
s:

А.
Rheumat oi
darthr
iti
s.
B.
SystemicLupusErythematosus.
C.
Scleroder
ma.
D.
Secondarythr
ombocy topeni
a.
E.
Encephalomyel
iti
s.
79.
Organnon-
speci
fi
cimmunedi
seasei
s:

А.
Hashimot othyr
oiditi
s.
B.
Rheumat oidart
hrit
is.
C.
Encephalomy el
it
is.
D.
Polyneuri
ti
s.
E.
Disseminatedscler
osisofCNS.

80.
Organnon-
speci
fi
cimmunedi
seasei
s:

А.
Hashimot othyr
oidi
ti
s.
B.
Encephalomy el
i
tis.
C.
SystemicLupusEr yt
hematosus.
D.
Polyneuri
ti
s.
E.
Disseminatedscler
osisofCNS.

90.Vari
antofpr
imaryi
mmunodef
ici
ency
syndromeoccurs:

А.
Inl
eukemi a.
B.
Underr adial
therapy.
C.
Insarcoidosis.
D.
Inhypopl asi
aoft hy
mus.
E.
Inmalignantlymphoma.
95.Vari
antofsecondar
yimmunodef
ici
ency
syndromeis:

А.
Lui-
Barsy ndrome.
B.
Neseloffsyndrome.
C.
DiGeorgesy ndrome.
D.
Brutonsyndr ome.
E.
AIDS.

98.
Compl
i
cat
ionofi
mmunodef
ici
encysy
ndr
omes
i
s:

А.
Art
erialhypertensi
on.
B.
Myocar di
alinfarct
ion.
C.
Typhoidfever.
D.
Sepsis.
E.
Chronicpepticgast r
icul
cer
.

99.
Compl
i
cat
ionofi
mmunodef
ici
encysy
ndr
omes
i
s:

А.
Recur
renceofbronchialasthma.
B.
Recur
renceofchronichepatiti
s.
C.
Recur
renceofchroniccholecysti
ti
s.
D.
Recur
renceofchronicgastricpepti
cul
cer
.
E.
Recur
renceoftubercul
osis.
100.Compli
cati
onofi
mmunodef
ici
ency
syndromesis:

А.Arterialhypertensi on.
B.My ocar di
alinfarction.
C.Typhoi dfever.
D.Purulentpneumoni a.
E.Chroni cpepticgast riculcer.
A22- year -
oldwomannur singhernewbor n
developsat enderer ythemat ousar eaar oundthe
ni
ppleofherl eftbr east .At hick,yell
owf lui
dis
obser vedt odrainf rom anopenf i
ssure.
Exami nat i
onoft hisbr eastf l
uidundert helight
microscopewi llmostl i
kelyrev ealanabundance
ofwhi choft hef oll
owi ngi nfl
ammat orycell
s?
Blymphocy t
es
Eosinophi ls
Mastcel l
s
Neutrophi ls

A63- y
ear -
oldmanbecomesf ebri
l
eandbegi ns
expectorati
nglar
geamount sofmucopur ul
ent
sputum.Sput um cult
uresareposit
ivef
or
Gram-positiv
ediplococci
.Whichofthef oll
owing
mediatorsofinfl
ammat i
onprovi
despotent
chemot acti
cfact
or sforthedi
rect
edmi grati
onof
i
nflammat or
ycell
sint
otheal
veol
arai
rspacesof
thi
spat ient ?
Brady ki
nin
Histamine
My eloper oxi
dase
N-formy latedpepti
des

A59- year-oldmansuf fersamassiv ehear


tat
tack
andexpi res24hour slaterduetov ent
ri
cul
ar
arr
hy thmia.Hi st
ologicexami nat
ionofthe
aff
ect edhear tmuscl eatautopsywoul dshowan
abundanceofwhi choft hefoll
owing
i
nflammat orycell
s?
Fi
br oblasts
Lymphocy tes
Macr ophages
Neut r
ophi l
s

A5- year-oldboypunct ureshi


sthumbwi tharust
y
nail
.Fourhour slater,thethumbappear sredand
swol len.Ini
tialswel l
ingoft heboy’st humbi s
primar il
yduet owhi choft hefoll
owi ng
mechani sms?
Decr easedi ntravascul arhydr
ostaticpr essure
Decr easedi ntravascul aroncoti
cpr essur e
Increasedcapi ll
aryper meabili
ty
Increasedi ntravascul aroncoti
cpr essur e
An80- y
ear -
ol dwomanpr esentswi tha4-hour
historyoff ev er
, shaki
ngchi lls,
anddi sori
entati
on.
Herbl oodpr essur ei
s80/ 40mm Hg.Phy sical
exami nati
onshowsdi ff
usepur puraonherupper
armsandchest .Bloodcul t
ur esar eposi
ti
vef or
Gram negat iveor gani
sms.Whi choft hefollowi
ng
cytokinesispr i
mar il
yinvolvedint hepathogenesis
ofdi r
ectv ascul arinj
uryint hispat i
entwit
hsept i
c
shock?
Interf
eron-g
Interl
eukin-1
Platelet-
der i
v edgr owthfact or
Tumornecr osisf act
or-a

A24- year-oldint r
avenousdr ugabuserdev elopsa
2-dayhi storyofsev er eheadacheandf ever .His
temper atureis38. 7° C( 103°F)
.Bloodculturesare
positi
vef orGr am- posi t
ivecocciThepat i
enti s
givenint r
avenousant i
bioti
cs,buthedeter i
orates
rapidl
yanddi es.Acr osssect i
onofthebr ainat
autopsy( showni nt hei mage)reveal
stwo
encapsul atedcav i
ti
es.Whi chofthefol
lowi ng
termsbestchar acterizesthispathol
ogicf i
nding?
Chronici nfl
ammat i
on
Fibri
noidnecr osis
Granulomat ousi nfl
ammat ion
Suppur ativeinflammat ion
A36- year-
oldwomanwi thpneumococcal
pneumoni adev elopsar ightpleur
aleffusion.The
pleuralfl
uiddisplaysahi ghspecifi
cgr avit
yand
containslargenumber sofpol ymorphonuclear
(PMN)l eukocy tes.Whichoft hefoll
owi ngbest
characteri
zest hispleuraleff
usion?
Fibri
nousexudat e
Lymphedema
Purulentexudat e
Serosanguineousexudat e

A33- year-
oldmanpr esentswi t
ha5-weekhi stor
y
ofcalfpainandswel l
ingandl ow-gr
adef ever.
Serum levelsofcreatinekinaseareelevated.A
musclebi opsyrevealsnumer ouseosinophi l
s.
Whati sthemostl ikelyeti
ologyofthispatient
’s
myalgia?
Autoimmunedi sease
Bacteriali
nfect
ion
Muscul ardystr
ophy
Parasiti
cinfect
ion

A10- year-ol
dboywi t
hahi storyofr ecurr
ent
bacteriali
nfect
ionspresentswi thfeveranda
product i
vecough.Biochemi calanalysisofhis
neutrophilsdemonstrat
est hathehasani mpaired
abil
it
yt ogeneratereacti
veoxy genspeci es.Thi
s
pati
entmostl i
kel
yhasi nherit
edmut ati
onsinthe
genet hatencodeswhi
choft
hef
oll
owi
ngpr
otei
ns?
Catalase
CytochromeP450
Myeloper oxi
dase
NADPHoxi dase

A25- year-ol
dwomanpr esent swithahi storyof
recurrentshor t
nessofbr eathandsev ere
wheezi ng.Labor atoryst udiesdemonst ratethat
shehasadef i
ciencyofC1i nhibi
tor,anest er
ase
i
nhi bi
torthatregul atest heactivati
onoft he
classicalcompl ementpat hway.Whati sthe
diagnosis?
Chroni cgranulomat ousdi sease
Her edit
aryangioedema
My eloperoxidasedef i
ciency
SelectiveIgAdef iciency

A40- year-ol
dmancompl ainsofa2- weekhi stor y
ofincreasingabdomi nalpainandy ell
ow
discolor
at i
onofhissclera.Physicalexami nation
revealsrightupperquadrantpain.Labor at
or y
studiesshowel evatedserum levelsofalkaline
phosphat ase(520U/ dL)andbi l
i
rubin(3.0mg/ dL).
Al i
verbiopsyshowspor talfi
brosis,wit
hscat tered
foreignbodiesconsistentwithschistosomeeggs.
Whi choft hefol
lowinginfl
ammat or
ycel l
sismost
l
ikel
yt opredominatei
nthepor
tal
tract
sint
he
l
iverofthispat
ient?
Basophils
Eosinophil
s
Macr ophages
Monocy tes

A41- y
ear-oldwomancompl ainsofexcessive
menst r
ual bleedi ngandpel vicpainof4mont hs.
Sheusesani ntrauterinedev iceforcont
raception.
Endomet rialbiopsy( showni nt heimage)reveals
anexcessofpl asmacel ls(arrows)and
macrophageswi thi
nt hestroma.Thepr esenceof
thesecellsandscat teredlymphoi dfol
li
cleswithi
n
theendomet rialst r
omai sev i
denceofwhi chof
thefoll
owi ngcondi ti
ons?
Acuteinflammat i
on
Chronicinflammat i
on
Granulat
iont issue
Granulomat ousi nfl
ammat i
on

A62- year-
oldwomanunder goingchemot herapy
forbreastcancerpr esentswi t
ha3- dayhistoryof
feverandchestpai n.Cardiaccat heter
izat
ion
reveal
samar kedlyreducedej ecti
onf r
acti
onwi th
normal coronarybloodf l
ow.Amy ocar
dialbiopsy
i
sobt ained,andaPCRt estf orcoxsackievi
rusis
positi
ve.Histol
ogicexami nat i
onoft hi
spat i
ent’
s
myocardi
um wil
lmostli
kelyrev
eal
anabundance
ofwhichofthef
oll
owinginfl
ammator
ycel
ls?
Eosi
nophil
s
Lymphocytes
Macrophages
Mastcell
s

A58- year
-oldwomanwi thlong-standi
ngdi abet
es
andhy pert
ensiondev el
opsend- stagerenal
diseaseanddi esi
nur emia.Ashaggyf ibri
n-ri
ch
exudat eisnotedont hev iscer
al peri
cardi
um at
aut opsy(showni ntheimage) .Whi chofthe
followingbestexplainsthepat hogenesisoft hi
s
fibrinousexudate?
Ant ibodybindi
ngandcompl ementact i
vati
on
Chr onicpassivecongestion
Injuryandi ncr
easedv ascularper meabil
it
y
Mar gi
nati
onofsegment edneut rophil
s

A68- y
ear-
oldmanpr esentswi t
hfev er,shaking
chil
ls,
andshor tnessofbr eath.Phy sical
examinati
onshowsr alesanddecr easedbr eath
soundsov erbothlungf i
elds.Thepat ientexhibit
s
grunti
ngrespir
at i
ons,30to35br eathspermi nute,
withfl
ari
ngoft henar es.Thesput um i srust y
yell
owanddi splaysnumer ous
polymorphonuclearleukocy tes.Whi choft he
fol
lowingmediat orsofinfl
ammat i
oni schiefly
responsibleforthedev
elopmentoff
everi
nthi
s
pat i
ent?
Arachidonicacid
Interl
eukin-
1
Leukot r
ieneB4
Prostacy cl
i
n( PGI2)

A35- year -
oldwomanpr esentswi t
ha5- day
historyofapai nfulsoreonherback.Phy si
cal
exami nationr eveal
sa1-cm abscessov erherlef
t
shoul der.Bi opsyoft helesionshowsv asodi
lat
ion
andl eukocy temar ginati
on( showni ntheimage).
Whatgl ycopr oteinmedi atesinit
ialt
etheri
ngof
segment edneut rophil
st oendotheli
alcell
sinthis
skinl esion?
Cadher i
n
Ent actin
Integr i
n
Select in

A14- year-ol
dboyreceivesal acerationonhi s
foreheaddur i
nganicehockeygame.Whenhei s
fi
rstattendedtobyt hemedi c,therei sbl
anching
oftheski naroundthewound.Whi chofthe
foll
owingmechani smsaccount sf orthi
stransient
reacti
ont oneurogenicandchemi calsti
mul i
att he
sit
eofi njury
?
Const r
icti
onofpostcapill
aryv enules
Constr
ict
ionofprecapill
aryarter
iol
es
Di
lat
ionofpostcapill
aryvenules
Di
lat
ionofprecapil
laryarter
iol
es

An8- year -ol


dgi rlwi t
hast hmapr esent swit
h
respiratorydi stress.Shehasahi storyofal
lergies
andupperr espiratorytractinfections.Shealso
hashi stor yofwheezesassoci at
edwi t
hexercise.
Whi choft hef ol
lowi ngmedi atorsofi nfl
ammat ion
i
st hemostpower fulst
imul atorof
bronchoconst ri
ctionandv asoconst r
icti
oni
nt his
pat i
ent?
Brady kinin
Compl ementpr oteins
Interl
euki n-1
Leukot rienes

A75- year-
oldwomancompl ai
nsofr ecentonset
ofchestpai n,f
ev er,
andpr oductivecoughwi t
h
rust-coloredsput um.AchestX- rayreveal
san
i
nfilt
r ateintherightmi ddl
el obe.Sputum cult
ures
areposi t
iveforStreptococcuspneumoni ae.
Phagocy ti
ccellsinthispat i
ent’
saf f
ectedlung
ti
ssuegener atebact eri
ocidalhypochlorousacid
using
whi chofthef oll
owingenzy mes?
Catal ase
Cyclooxy genase
Myel
oper
oxidase
NADPHoxidase

A28- year-ol
dwomancut sherhandwhi ledici
ng
vegetablesint hekitchen.Thewoundi scleaned
andsut ured.Fiveday slater,thesiteofinjur
y
containsanabundanceofchr onicinflammat or
y
cell
st hatactivel
ysecr et
ei nterl
eukin-1,tumor
necrosisfact or-
,i
nterfer
on- ,
numer ous
arachidonicacidder i
vativ
es, andv ari
ousenzy mes.
Namet hesecel l
s
Blymphocy tes
Macr ophages
Plasmacel l
s
Smoot hmuscl ecell
s

A68-year-oldmanwi t
hpr ostatecancerandbone
metastasespresentswithshaki ngchillsandfev
er.
TheperipheralWBCcounti s1, 000/L( normal=
4,
000t o11,000/L).Whichoft hefollowi ngt
erms
bestdescribesthi
shemat ologicfinding?

Leukocytosi
s
Leukopenia
Neutrophil
ia
Pancytopenia

A25-
year
-ol
dmachi
nisti
sinj
uredbyamet
alsl
i
ver
i
nhislefthand.Ov erthenextfewday s,
the
woundedar eabecomesr eddened,t
ender,
swoll
en,andf eel
swar mt othetouch.Rednessat
thesi
teofinjuryi
nt hispati
entiscausedpri
maril
y
bywhichoft hefol
lowi ngmechanisms?
Hemorrhage
Hemostasis
Neutr
ophilmar gi
nation
Vasodi
lati
on

A37- year-ol
dmanwi thAI DSisadmi t
tedt othe
hospitalwi t
ha3- weekhi stor
yofchestpai nand
shortnessofbr eath.AnX- rayfi
l
m oft hechest
showsbi l
ateralnodul
ar i
ti
esofthelungs.A
CT-guidedl ungbiopsyi sshowni ntheimage.The
mul t
inucleatedcelli
nt hecenterofthisfiel
di s
mostl i
kelyder i
vedfrom whi chofthef ol
l
owi ng
i
nflammat orycell
s?
Basophi l
s
Capill
aryendot heli
alcells
Macr ophages
Myof ibroblasts

A10-year-
oldgirlpr
esentswitha2- weekhist
ory
ofpuff
inessaroundherey esandswel l
ingofthe
l
egsandankl es.Laborat
orystudiesshow
hypoal
buminemi aandproteinur
ia.Theurinar
y
sedi
mentcont ai
nsnoi nf
lammat orycell
sorred
bl
oodcel ls.Whi
chofthefol
lowi
ngterms
descr i
besthi
spati
ent
’speri
pher
aledema?
Effusion
Exudat e
Hydr opi
cchange
Transudate

A25- year -
oldwomandev elopsasor e,r
ed,hot
,
swol l
enl eftknee.Shehasnohi storyoftrauma
andnof ami li
alhist
oryofj ointdisease.Flui
d
aspi ratedf rom thejoi
ntspaceshowsan
abundanceofsegment edneut rophils.
Transendot hel
ialmigr
at i
onofacut einfl
ammatory
cellsi ntothispat i
ent’
sjointspacewasmedi at
ed
primar il
ybywhi choft hefollowingf amili
esof
prot eins?
Ent act i
ns
Fibrill
ins
Fibronect i
ns
Integr ins

A50- year-
oldwomani sdi scoveredtohave
met astat
icbreastcancer .Oneweekaf t
er
receivi
ngherf ir
stdoseofchemot her
apy,she
developsbact erial
pneumoni a.Whichofthe
foll
owingbestexpl ai
nsthispat i
ent’
s
suscept i
bil
i
tytobact eri
al i
nfecti
on?
Depletionofser um compl ement
I
mpairedneut
rophi
lrespi
rat
oryburst
I
nhibi
ti
onofcl
otti
ngfactoract
ivat
ion
Neutr
openi
a

A53- year-oldmandev elopsweakness, malaise,


coughwi thbl oodysput um, andni ghtsweat s.A
chestX- rayr evealsnumer ousapi cal densiti
es
bil
ateral
ly .Ex posuretoMy cobact erium
tuberculosiswasdocument ed20y earsago, and
M.t uberculosi sIidentif
iedi nthesput um.The
pati
entsubsequent lydiesofr espirat ory
i
nsuf fi
ci
ency .Thel ungsar eexami nedataut opsy
(showni nt hei mage) .Whi choft hef ollowingbest
character i
zest hehi st
opat hologi
cf eaturesoft hi
s
pulmonar ylesion?
Acut esuppur ati
veinflammat i
on
Chronici nfl
ammat i
on
Fatnecr osis
Granulomat ousi nfl
ammat ion

A59- year -
oldmanexper i
encesacut echestpain
andisr ushedt otheemer gencyr oom.Labor ator
y
studiesandECGdemonst rateanacut e
my ocardialinf
arcti
on;howev er,coronaryart
ery
angiogr aphyperfor
med2hour sl aterdoesnot
showev i
denceofthrombosis.Int r
avascular
thrombol ysist
hatoccurredint hispatientwas
medi atedbypl asminogenact i
vatorsthatwer e
rel
easedbywhichoft
hef
oll
owi
ngcel
l
s?
Cardi
acmy ocy
tes
Endothel
i
alcel
ls
Macrophages
Segmentedneutr
ophi
l
s

A68- year-oldcoal minerwit


hahi storyofsmoki ng
andemphy semadev el
opssev er
eai r
-flow
obst r
uctionandexpi r
es.Autopsyr evealsa“ bl
ack
l
ung, ”withcoal -dustnodul
esscat ter
ed
throughoutt hepar enchymaandacent raareaof
densef i
brosi s.Thecoal dustentrappedwi thin
thismi ner’
sl ungwassequest eredpr imari
lyby
whi choft hef ol
lowingcell
s?
Endot heli
al cel l
s
Fibroblasts
Lymphocy tes
Macr ophages

A40- year-oldmanpr esentswi t


h5day sof
productivecoughandf ever.Pseudomonas
aeruginosai sisolatedfrom apulmonar yabscess.
TheCBCshowsanacut eeffectcharacteri
zedby
mar kedleukocy tosis(50,000WBC/ L),andt he
dif
ferentialcountr evealsnumer ousimmat ure
cell
s( bandf orms) .Whichoft hefoll
owi ngterms
bestdescr ibesthesehemat ologicfi
ndings?
Leukemoi dreaction
Leukopeni
a
Myeloi
dmet apl
asia
Myeloprol
i
ferat
ivedi
sease

A19- year-oldwomanpr esentswith5day sof


fever(38°C/ 101°F)andsor ethroat
.Sher eports
thatshehasf eltfati
guedf ort
hepastweekand
hasdi f
ficultyswallowing.Aphy si
calexami nation
revealsgener al
izedl ymphadenopathy.I
ft his
patienthasav i
ral i
nfect
ion,aCBCwi llmostl i
kely
showwhi choft hef ol
lowinghematologicf i
ndings?
Eosinophilia
Leukopeni a
Lymphocy tosis
Neut r
ophilia

A40- year -
oldwomanpr esent swi t
han8- mont h
historyofpr ogr essivegener al
izedi tching,weight
l
oss, fat i
gue, andy ellowscl erae.Phy sical
exami nat i
onr eveal smi l
dj aundi ce.The
antimitochondr i
al antibodyt esti sposi ti
ve.Aliver
biopsydi sclosesper iductalinflammat ionandbi le
ductinj ury(showni nt heimage) .Whi choft he
foll
owi ngi nfl
ammat orycellsist hepr incipal
medi atorofdest ructivecholangi tisinthispatient?
Eosinophi ls
Bl y
mphocy tes
Tl y
mphocy t
es
Mastcel
l
s

A25- year-oldwomanpr esent swi tha2- week


hi
st oryoff ebri
leill
nessandchestpai n.Shehas
aner y
t hemat ous, macularf acial r
ashandt ender
j
oint s,par ti
cularl
yinherl eftwr istandel bow.A
CBCshowsmi ldanemi aandt hr ombocy topenia.
Corticost eroidsar eprescribedf orthepatient.
Thismedi cati
oni nducest hesy nthesi
sofan
i
nhibi torofwhi chofthef ollowingenzy mesi n
i
nflammat orycells?
Li
poxy genase
My eloper oxidase
Phosphol ipaseA2
Phosphol ipaseC

A22- year -
oldmandev elopsmar kedr ightlower
quadrantabdomi nal painovert hepastday .On
physical examinat i
onthereisr eboundt enderness
onpal pat i
onov ertherightl
owerquadr ant.
Laparoscopi csur geryisperformed, andt he
appendi xisswol len,eryt
hemat ous, andpar t
ly
coveredbyay ell
owishexudat e.Itisr emov ed,
andami croscopicsect i
onshowsi nfi
ltrati
onwith
numer ousneut r
ophils.Thepai nexper i
encedby
thi
spat ientispredomi nantl
yt her esultofwhi chof
thefollowingtwochemi cal
medi ators?
Compl ementC3bandI gG
Int
erleukin-1andt umornecr osi
sfactor
Histami neandser otonin
Prostaglandinandbr adykinin
A40- year-ol
dwomanhadl aparoscopicsur
gery3
mont hsago.Nowshehasasmal l0.
5cm nodule
beneat htheskinatt heincisi
onsitethatwas
sutured.Whi choft hefoll
owingcellt
y pesi
smost
l
ikelytobemostchar acter
isti
cofthe
i
nflammat oryresponseint hi
ssit
uation?
Mastcel l
Eosinophi l
Giantcel l
Neut r
ophi l

A39- year -oldmani ncursabur ninjurytohis


handsandar mswhi l
ewor kingonapr opane
fur
nace.Ov ert
henext3weeks, thebur nedskin
healswi thouttheneedf orskingr aft
ing.Whichof
thefol l
owi ngisthemostcr it
ical
f actorin
determi ni
ngwhet hertheski ninther egionofthe
burnwi llregenerate?
Goodcar diacoutputwi thtissueper fusi
on
Persist enceofski nappendages
Maint enanceofunder ly
ingconnect i
vetissue
Dimini shededemaander ythema

A58-
year-
oldwomanhashadacoughwit
hfever
f
or3days.Achestr
adi
ogr
aphr
eveal
sinf
il
tr
atesi
n
ther ightlowerl obe.Asput um culturegrows
Strept ococcuspneumoni ae.Thecl earanceof
theseor ganismsf rom thelungpar enchyma
woul dbemostef fecti
velyaccompl ishedthrough
gener at i
onofwhi choft hef ol
l
owi ngsubst ances
byt hemaj orinfl
ammat or
ycelltyper espondingto
thi
si nf ecti
on?
Plateletact i
vatingf actor
Prost aglandinE2
Kallikrein
Hy drogenper oxide

Acl i
nicalst udyi sperformedofpat ientswi t
h
phar yngeal infecti
ons.Themostt y picalcli
nical
cour seav erages3day sf r
om t hetimeofonset
untilthepat ientseest hephy sici
an. Mostoft hese
patientsexper iencefeverandchi l
ls.Onphy sical
exami nation, themostcommonf i
ndi ngsi nclude
swel li
ng,ery thema, andphar yngeal purulent
exudat e.Whi choft hefol
lowingt ypesof
i
nflammat iondi dthesepatientsmostl i
kelyhav e?
Granul omat ous
Acut e
Gangr enous
Resol ving

A56-y
ear-
oldmanhashadi
ncreasi
ngdyspneafor
6year
s.Hehasnocoughorf
ev er
.Hehadchronic
exposuretoi nhal ationofsi li
cadustf ormany
yearsinhisjob.Achestx- raynowshows
i
ncreasedi nterstitialmar kingsanpar enchymal1
to3cm sol idnodul es.Hi spulmonar yproblems
aremostl i
kel ytobemedi atedthr
oughwhi chof
thefoll
owingi nf l
ammat oryprocesses?
Neutrophil
icinf i
ltratespr oducingleukotri
enes
Forei
gnbodygi antcel lformat i
on
Plasmacel lsy nthesi sofimmunogl obuli
ns
Macr ophageel abor ati
onofcy t
oki
nes

A22- year -ol


dwomanhaspr emat urelaborwi th
premat ur erupt ureoff etalmembr anesat20
weeksgest ation.Priort othatt
ime, thepr egnancy
hadbeenpr oceedi ngnor mall
y.Ast i
llbirt
hoccurs
twoday sl ater.Microscopi cexami nationoft he
normal -sizedpl acent arevealsnumer ous
neutrophi lsint heamni onandchor ion, butno
vil
li
ti
s.Thepr emat urelaborwasmostl i
kely
medi atedbyt heef fectsf rom r
eleaseofwhi chof
thefollowi ngsubst ances?
I
mmunogl obul i
n
Prostagl andi n
Compl ement
Fibr
inogen

Af
tert
woweeksint
hehospi
tal
fol
lowi
ngafal
li
n
whi
chshei
ncur
redafr
act
ureofherl
eftf
emor
al
trochanter,a76-y ear-ol
dwomannowhasal eft
l
egt hatisswol l
en, parti
cularl
yherl owerlegbel
ow
theknee.Sheexper i
encespai nonmov ementof
thisleg,andt herei stender nesstopal pati
on.
Whi choft hef oll
owi ngcompl icat
ionsismost
l
ikelytooccurnextaf tertheseev ents?
Gangr enousnecr osisoft hef oot
Hemat omaoft het high
Dissemi natedi nt
rav ascularcoagul at
ion
Pulmonar yt hromboembol i
sm

A43- year -
oldwomanhashadachr oniccough
withf ev erandwei ghtlossforthepastmont h.A
chestr adiographrevealsmul t
iplenodulesfrom 1
to4cm i nsize,someofwhi chdemonst r
ate
cavitationi ntheupperl obes.Asput um sample
reveal st hepresenceofaci dfastbacill
i.Whichof
thef ollowingcel l
sisthemosti mportantinthe
devel opmentherl unglesions?
Macr ophage
Fibrobl ast
Neut rophi l
Mastcel l

A20- year-
oldmanhasexper i
encedpainf
ul
uri
nationfor4days.Aurethri
ti
sissuspected,
and
Neisseriagonor
rheaeiscult
ured.Numerous
neutrophil
sarepresenti
nasmearoft heexudate
fr
om t hepeni leurethra.Theseneutrophi
ls
under godi apedesistor eachtheorgani
sms.
Releaseofwhi choft hef oll
owingchemical
medi atorsi smostlikelytodr i
veneutr
ophil
exudat ion?
Histami ne
Prostagl andin
Hagemanf actor
Compl ement

Anepi sodeofmar kedchestpai nlasti


ng4hour s
bringsa51- year -
oldmant otheemer gencyr oom.
Hei sf oundt ohav eanel ev atedserum creati
ne
kinase.Anangi ogram rev ealsacompl ete
blockageoft hel eftcir
cumf lexarter
y2cm f r
om
i
tsor igin.Whi choft hef ollowi ngsubstances
woul dy oumostexpectt obeel abor
atedar ound
ther egionoft issuedamagei nthenext3day sas
ani nit
ialresponset opromot eheali
ng?
Histami ne
Immunogl oblulinG
Compl ementcomponentC3b
Vascul arendot helialgrowt hf actor

A94- y
ear-ol
dwomanhasdev elopedafeverand
coughov ert
hepast2day s.Staphyl
ococcus
aureusiscult
uredfrom hersput um.Shereceiv
es
acourseofant i
biot
icther
apy .Twoweeksl ater
shenol ongerhasapr oductivecough, butshesti
l
l
hasaf ev er.Achestr adiographr evealsa3cm
roundeddensi t
yinther i
ghtlowerl obewhose
l
iquefi
edcont entsform acent r
al ai
r-
fluidl
evel
.
Therear enosur roundi nginfi
ltr
ates.Whi chofthe
fol
lowingi st hebestdescr i
ptionf orthisout
come
ofherpneumoni a?
Hypertrophi cscar
Abscessf ormat ion
Regener ation
Bronchogeni ccar ci
noma

A36- year -oldwomanhasbeent aking


acet ylsal i
cy l
icacid( aspiri
n)forarthrit
isforthe
past4y ear s.Herj oi
ntpai nistempor aril
yreduced
viat hist her apy.Howev er,shenowhasoccul t
bloodi dent ifi
edinherst ool.Whichoft he
foll
owi ngsubst ancesi smostl ikelyinhibi
tedby
aspi ri
nt ocauset hiscompl i
cat
ion?
Leukot rieneB4
Interleuki n-1
Thr omboxane
Brady kinin

Asmal lsl
iv
erofwoodbecomesembeddedi nthe
fi
ngerofa25-year
-ol
dman. Hedoesnotremoveit,
andov ert
hennext3daystheareaaroundthe
sl
iverbecomesred,swol
l
en, andt
ender.
Neutrophil
smi gratei
ntotheinjuredtissue.
Expressionofwhi chofthefol
lowi ngsubstances
onendot helialcell
sismostinst r
ument alin
promotingt hisinfl
ammat or
yr eaction?
Inter
fer
ongamma
Hagemanf actor
Lysozyme
E-sel
ecti
n

Ani nfl
ammat orypr ocesst hathascontinuedf or3
mont hsincl udest het ransformationoft i
ssue
macr ophagest oepi thelioi
dcel l
s.Therear ealso
l
ymphocy t
espr esent .Ov ertime,fibr
oblastslay
downcol l
agenast hef ocusofi nf
lammat ionheals.
Theseev entsar emostl ikelytooccurasan
i
nflammat oryr esponset owhi chofthef oll
owing
i
nfectiousagent s?
My cobact erium t uber culosis
Pseudomonasaer uginosa
Cytomegal ov i
rus
Giardialambl i
a

A37-year-
oldmanhashadnauseaandv omi t
ing
for5weeks.Heexperiencedanepi sodeof
hematemesisyester
day.Onphy si
cal exami nation
hehasnoabnor malfi
ndings.UpperGIendoscopy
i
sper f
ormed,andthereisa1.5cdi amet erlesion
i
nt hegast
ricant
rum withlossoftheepi theli
al
surface.Thesefi
ndi
ngsar
emosttypi
calf
orwhi
ch
ofthef ol
lowingpat
hol
ogi
cpr
ocesses?
Abscess
Serositi
s
Granuloma
Ulcer

A17- year-oldtruckdr i
veri
sinvolvedinacolli
sion.
Hei ncursbl untforceabdomi nal t
rauma.I
n
responset othisinj
ury,cel
lsint i
ssuesoft
he
abdomenar estimulatedtoent ertheG1phaseof
thecel lcyclefrom theG0phase.Whi chofthe
foll
owi ngcel lt
ypesi smostlikelytoremai
ni nG0
foll
owi ngthisinjury?
Smoot hmuscl e
Endot heli
um
Skeletal muscle
Fibroblast

A19- year -oldwomanwhowor ksindoorsspends


adayout si degardening.Shedoesnotwearahat
orsunscr een.Thatev eningherpart
nerremarks
thatherf aceappear sr ed.Whichofthefoll
owing
dermal changesmostl ikel
yaccountsforherred
appear ance?
Neutrophi l aggr
egation
Hemor r hage
Edema
Vasodi
l
ati
on

A45- y ear -
oldwomanhashadachr onic,
non-pr oduct i
v ecoughf or3mont hs, al
ongwi th
i
ntermi ttentfever.Shehasachestr adiographthat
reveal smul ti
plesmal lparenchy mal nodulesalong
withhi larandcer vi
cal lymphadenopat hy.A
cervical lymphnodebi opsyisper f
ormed.
Microscopi cexami nati
onoft hebiopsyshows
noncaseat i
nggr anulomat ousi nfl
ammat ion.
Cultur esf orbacterial,fungal
, andmy cobacteri
al
organi smsar enegat ive.Whi choft hef oll
owing
chemi cal mediatorsi smosti mpor t
anti nthe
developmentofheri nflammat oryresponse?
Int
erf erongamma
Brady kini n
Compl ementC5a
Histami ne

A55- year-
oldmanhasahi stor
yof
hypercholester
olemiawi t
hcoronaryar t
ery
diseaseandsuf fer
edamy ocardiali
nfarct
ion2
yearsago.Henowpr esentswi
thcr ushi
ng
substernalchestpain.Whichoft hefoll
owing
l
abor ator
ytestsismostusef ulindiagnosingt
he
causeofhi schestpai n?
Increasedwhi t
ebloodcel l
count
Elevatedsedimentationrate
Decreasedserum complement
I
ncreasedserum troponi
n

A15- year -
oldgi r
lhashadepi sodesofsneezi ng
withwat er
yey esandr unnynosef orthepast2
weeks.Onphy sicalexami nationshehasr ed,
swol l
ennasal mucosal surfaces.Shehashad
similarepisodeseachSpr i
ngandSummerwhen
theamountofpol l
enint heai rishigh.Her
sympt omsar emostl i
kelytobemedi at
edbyt he
rel
easeofwhi choft hef ol
lowi ngchemi cal
medi ators?
Compl ementC3b
Plateletactivati
ngf actor(PAF)
Tumornecr osi sfactor(TNF)
Histami ne

A45- year-
ol dmanhasbeenwor ki
nghar dallday
l
ongcar r
yingloadsofbrickstobuil
dawal l
.He
takesanonst er
oidalant
i-
inf
lammat orydrug
(i
buprofen) .Whichofthefoll
owingprocessesis
thisdrugmostl ikel
ytodiminishinhisarms?
Thrombosi s
Pain
Necrosis
Fibri
nolysi
s

Wi
thi
nmi
nut
esf
oll
owi
ngabeest
ing,
a37-
year
-ol
d
mandev elopsmarkedrespir
atoryst ri
dorwith
dyspneaandwheezi ng.Heal sodev elopsswell
ing
andery themaseeninhisar msandl egs.An
i
njecti
onofepi nephri
nehelpstor eversethese
eventsandher ecoverswithi
nmi nut es.Whichof
thefoll
owi ngchemicalmediator
si smost
i
mpor tantinthepathogenesisoft hi
sman' s
condit
ion?
Bradykinin
Compl ementC5a
Nitr
icoxi de
Histamine

A72- y
ear -
oldwomandi dnotgeta' flu'shotinthe
fallasr ecommendedf orolderpersons.I nthe
wintertime,shebecamei ll
,asmanypeopl einher
communi t
ydid,witharespi
rator
yi l
lnesst hat
l
ast edf or3weeks.Dur i
ngthisil
lness,shehada
feverwi thanon-producti
vecough, mildchestpain,
my algias,andheadache.Whatwasherchest
radiographmostl ikel
ytohav eshowndur ingthis
i
llness?
Hilarmass
Intersti
tiali
nfi
lt
rat
es
Hilarlymphadenopat hy
Lobarconsol i
dati
on

I
nanexper
iment
,Ent
erobact
ercl
oacae
organismsar eaddedt oasol ut
ioncontaini
ng
l
eukocy t
esandbl oodpl asma.Engulfmentand
phagocy t
osisoft hemi crobesisobservedt o
occur .Nextasubst anceisaddedwhi chenhances
engulfment ,andmor ebacter
iaaredestroyed.
Whichoft hefollowi ngsubstancesintheplasma
i
smostl i
kel
yt opr oducet hi
seffect
?
Compl ementC3b
Glutathioneper oxidase
Immunogl obuli
nM
P-selectin

A43- year-
oldwomanhashadnauseawi t
h
vomi tingpersistingf orthepast5weeks.On
phy sicalexami nationt herearenoabnormal
findi ngs.Sheunder goesanupperGIendoscopy
andgast ri
cbiopsi esar eobtained.The
mi cr oscopicappear anceoft hesebiopsi
esshows
mucosal i
nfi
lt
rationbyl ymphocy t
es,
macr ophages, andpl asmacel ls.Whichofthe
followi ngmostl ikelycausedherf indi
ngs?
St aphy l
ococcusaur eussept i
cemi a
Ingest i
onofchi l
ipepper s
Di abet esmel l
itus
Inf ectionwi t
hHel i
cobact erpylori

I
nanexperi
ment,
lymphat
icchannel
sare
obser
vedi
nnormalsof
tti
ssueprepar
ati
ons.
Staphyl
ococcusaur eusor ganismsar e
i
nnoculatedintothet i
ssuesandt heimmunol ogi
c
responseobser v
edov erthenext24hour s.Which
ofthefoll
owingf uncti
onsi smostl i
kelytobe
servedbythesel ymphaticstopr oduceaspeci fi
c
i
mmuner esponset otheseor gani sms?
Carryly
mphocy testoper ipheralt i
ssuesites
Remov eextravascularti
ssuef luid
Transportanti
genpr esentingcel ls
Serveasar outefordissemi nationofi nf
ecti
on

Inanexper i
ment ,sur gi
cal woundsitesare
observedfollowingsut uring.Aningrowthofnew
capil
l
ariesisobser v edtooccurwi thi
nthef i
rst
week. Asubst anceel aboratedbymacr ophagesis
foundatthewoundsi t
etost i
mulatethiscapil
lar
y
prol
if
erati
on.Whi choft hef ol
lowi
ngsubst ances
i
smostl i
kelytohav ethisfuncti
on?
Plat
elet-
derivedgr owt hfactor
PhospholipaseC- gamma
Fibr
onectin
Fibr
oblastgrowt hf actor

A55- year
-oldmanwi t
hahistoryofi schemicheart
diseasehaswor seni
ngcongest iveheartfai
lure.
Hehasnot edincreasi
ngdy spneaandor t
hopnea
forthepast2mont hs.Onphy sical exami
nation
thereisdullnesstopercussionatl ungbases.A
chestx-rayshowsbi l
ateralpl
euralef
fusi
ons.A
l
eftthoracent esisisper formed,and500mLof
fl
uidisobt ained.Whi choft hefoll
owing
charact
er isti
csoft hisfluidwouldmostlikel
y
i
ndicatet hatitisat ransudate?
Cloudyappear ance
Highprot eincont ent
<3lymphocy tes/mi crol
iter
Presenceoff i
brin

Inacl i
nicalstudy,patientsunder going
l
apar oscopi ccholecy stectomyar ef oll
owedt o
documentt hepost -
sur gi
cal woundheal ing
process.Thesmal lincisi
onsar ecl osedwi th
sutur es.Ov erthe4weeksf oll
owingsur gery,the
woundsar eobserv edt oregaintensi l
estrengt h
andt her eisre-epi
theliali
zation.Oft hefoll
owi ng
subst ances, whichismostl ikel
yfoundt of unction
i
nt racell
ularlyincellsinvolvedint hi
swound
heal i
ngpr ocess?
Fibronect i
n
Lami nin
Ty r
osi nekinase
Hy aluronicacid

A31- year
-ol
dwomanhasal apar
otomy
perfor
medf orremoval
ofanovari
ancy st
.She
recoversunevent
ful
l
y,wit
hnocomplicati
ons.At
thet i
meofsur gery,
a12cm l ongmi dl
ine
abdomi nal i
nci
sionwasmade.Thet ensi
le
strengthi nthesurgicalscarwi
ll
increasesoher
nor mal activi
ti
escanber esumed.Mostoft he
tensilestrengthwill
likel
ybeachievedinwhichof
thef oll
owi ngtimeper i
ods?
Oneweek
Onemont h
Thr eemont hs
Sixmont hs

A9- year -
oldgi rlsustainsasmal l 0.5cm long
l
acer ationt oherr ightindexf ingerwhi leplayi
ng
'
QueenofSwor ds'withal etteropener .Whi chof
thef oll
owi ngsubst ances,oncont actwit
hi nj
ured
vascul arbasementmembr ane, act i
vat
esbot hthe
coagul ati
onsequenceandt heki ninsystem asan
i
niti
al responset ot hi
si nj
ury?
Thromboxane
Plasmi n
Plateletact ivati
ngf actor
Hagemanf actor

A65-year
-ol
dwomanhashadaf ev erf
orthepast
day.Onphysi
calexaminationhert emperatur
eis
39°Candbloodpressure90/ 50mm Hgwi thhear
t
rat
eof106/minute.Laboratorystudiesshowa
WBCcountof12,510/ microli
terandWBC
diff
erent i
al countof78segs, 8bands,11l y
mphs,
and3monos.Abl oodcultureispositi
vef or
Escher ichiacoli.
Hercentral venouspressur efal
l
s
mar kedl y.Shegoesi ntohy povol
emi cshockasa
resultoft hewi despreadinappropriat
er eleaseof
achemi cal mediatorderi
vedf rom macr ophages.
Shedev elopsmul tipl
eorganf ail
ure.Whi chofthe
foll
owi ngmedi at
or si
smostl i
kelytoproduce
thesef indings?
Nitr
icoxi de
Brady kinin
Histami ne
Prostacy clin

A20- year -
ol dwomansust ainsaninjuryt
oher
ri
ghtcal finamount ainbikingaccident.On
physical exami nationshehasa5cm l ong
l
acerat i
onont her i
ghtlat
er alaspectofherlower
l
eg.Thi swoundi sclosedwi thsutur
es.Wound
heali
ngpr oceedsov erthenextweek. Whichoft
he
fol
lowingf actorswi l
lbemostl ikel
ytoaidandnot
i
nhibitwounheal inginthispat i
ent?
Commensal bacteria
Decreasedt i
ssueper fusi
on
Presenceofsut ures
Corti
cost er oidtherapy

A24-
year
-ol
dpr
imi
grav
idai
slat
eint
hesecond
tr
imest erofpr egnancy .Sheexper i
encesthe
suddenonsetofsomecr ampi nglowerabdomi nal
pain.Thi sisi mmedi atelyfollowedbypassageof
somef luidperv aginaal ongwi thafoul-
smel li
ng
discharge.Thef etusisst ill
bor ntwoday slater.
Exami nat i
onoft hepl acent ademonst r
ates
extensiv eneut rophi l
i
ci nfil
tratesinthechor i
on
andamni on.Whi choft hef ollowingorganismsi s
mostl ikelytober esponsi bl
ef ort
hesefindings?
My cobact erium t uberculosis
Her pessi mpl exv irus
Escher i
chi acol i
Treponemapal l
idum

A19- year-
oldmani ncursast abwoundt ot he
chest.Thewoundi st r
eatedi ntheemer gency
room.Twomont hsl at
ert hereisaf i
rm,3x2cm
nodularmasswi t
hi ntactov erl
yingepit
helium in
theregionofthewound.Onexami nati
onthescar
i
sf i
rm, butnottender ,withnoer ythema.Thi s
massi sexcisedandmi croscopicall
yshows
fi
broblastswithabundantcol lagen.Whichoft he
fol
lowingmechani smshasmostl ikel
yproduced
thi
sser i
esofev ent s?
Keloi
df ormati
on
Developmentofaf i
brosar coma
Poorwoundheal ingf r
om di abetesmel l
it
us
Forei
gnbodyr esponsef r
om sut uri
ng
A45- year -
oldmanhashadaf everanddr ycough
for3day s,andnowhasdi ff i
cult
ybr eathi
nganda
coughpr oduct iveofsput um.Onphy si
cal
exami nationhi st emper atureis38. 5°C.Diffuse
ralesar eauscul tatedov erlowerl ungf iel
ds.A
chestr adiogr aphshowsar i
ghtpleur al
effusion.A
ri
ghtt horacent esisi sperfor med.Thef lui
d
obt ainedhasacl oudappear ancewi thacel lcount
showi ng15, 500l eukocy tespermi croli
ter,98%of
whi char eneut rophils.Whi choft hef ol
lowing
termsbestdescr ibeshi spl euralprocess?
Ser ousi nf l
ammat i
on
Pur ulenti nflammat ion
Fibrinousi nflammat ion
Chr oni cinflammat ion

Q..A52- year-oldwomanwi thnomaj ormedi cal


problemst akesal ongairpl
anef l
ightacr ossthe
Pacifi
cOcean.Uponar ri
val
atSy dney '
sKi ngsfor
d
Smithai r
por tfoll
owingthefl
ightfrom Los
Angeles,shecannotputhershoesbackonTher e
i
snopai nort enderness.Whichoft hefollowingis
themostl ikelyexplanati
onforthisphenomenon?
Activ
ationofHagemanf act
orhasl edt o
bradyki
ninpr oduct i
on.
Alotofdr inkswer eservedinthef i
rstclass
secti
on.
Femoralv
einthr
ombosisdevel
oped
Venoushydr
ostat
icpr
essurebecamei
ncr
eased.

Q.I nanexper i
ment ,alungt issuepr eparat i
oni s
exposedt oMy cobact er
ium tuber culosi
s
organisms. Overthenextweek, itisobser v
edt hat
granulomasf ormi nthelung.Wi t
hint he
granulomaar efoundi nfl
ammat or ycell
s
expressingclassIIMHCant i
gens.Thesecel l
s
el
abor atecytoki
nest hatpr omot efibroblastic
productionofcoll
agenwi thinthegr anulomas.
From whi chofthef ol
lowingper ipher al
blood
l
eukocy tesarethesecel l
sbear i
ngcl assI Ianti
gen
mostl i
kelytobeder i
ved?
Neutrophils
Bcel l
s
Monocy tes
NKcel ls

Q.A56- year-oldmanhashadi ncreasingdiff


icul
ty
breat hingf orthepastweek.Onphy si
cal
exami nationhei safebri
le.Auscultati
onofhis
chestr evealsdimi ni
shedbreathsoundsand
dullnesst oper cussionbilat
erall
y.Thereis2+
pit
tingedemapr esenttothelevelofhisthighs.A
chestr adiographr eveal
sbilat
eralpleural
effusions.Whi choft hefoll
owinglabor at
orytest
fi
ndi ngsi shemostl ikel
ytohav e?
Hypoal
buminemi
a
Gl
ucosuri
a
Neutr
ophi
li
a
Anemia

Q.A72- year -
oldmanpr esent switha3- dayhistory
ofpr ogr essivel
ywor seningpr oducti
vecough,
fever ,chil
ls,andsignsoft oxicit
y.Promi nent
phy sical f
indingsincl udesi gnsofconsol i
dat
ion
andr alesov ertherightlungbase. Sputum culture
i
sposi t
iveforSt r
eptococcuspneumoni ae.An
i
ntra- alveolarexudat ef i
l
lingt healveoliofthe
i
nv olv edpor ti
onoft hel ungi spr esent.Whichof
thef ollowingt ypesofi nflammat orycellsismost
l
ikelyapr omi nentfeat ureoft hisexudat e?
Basophi ls
Eosi nophi l
s
Lymphocy tes
Neut rophi l
s

Q.Ar outi
necompl etebloodcountper formedon
a22-year-oldmedi calstudentrevealsan
abnormal i
tyinthedi ff
erenti
alleukocytecount.
Shehasbeencompl ai
ningoff r
equentsneezing
and“wat ery”eyesdur i
ngt hepastsev eralweeks
andrepor t
st hatshef r
equentlyhadsuchepi sodes
i
nthespr ingandsummer .Whi chofthef ol
lowing
cel
ltypesismostl
i
kel
ytobei
ncr
eased?
Basophil
s
Eosi
nophils
Lymphocy t
es
Monocy t
es

A16- y
ear -ol dboypr esent swi tha24- hourhi story
ofsev er eabdomi nal pain, nausea, vomi ti
ng,and
l
ow- gradef ever .Thepai ni si niti
allyper iumbili
cal
i
nl ocat i
onbuthasmi gratedt other ightl ower
quadr antoft heabdomen, wi thmaxi mal
tender nessel icitedatasi teone- thir
doft heway
betweent hecr estoft hei leum andt heumbi li
cus
(McBur neypoi nt ).Thel eukocy tecounti s
14,000/ mm3, wi t
h74%segment edneut rophil
s
and12%bands.Sur ger yisper formed.Whi chof
thef oll
owi ngdescr i
best heexpect edf indingsat
theaf fect edsi te?
Fistula( abnor mal ductorpassage)connect i
ngt o
theabdomi nal wal l
Granul ationt i
ssue( newv essel sandy oung
fi
brobl ast s)wi thapr omi nenti nf i
ltr
ateof
eosinophi ls
Granul omat ousi nflammat ionwi t
hpr omi nent
aggr egat esofepi thelioi
dcel lsandmul t
inucleated
giantcel ls
Promi nentar easofedema, congest ion, anda
purulentr eact i
onwi t
hl ocal izedar easofabscess
f
ormat
ion

A2- year-oldboypr esent swi thr ecurrent


i
nf ect i
onsi nvol v
ingmul tipleor gansy st ems.
Extensi v
ei nvestigat i
onr esul tsinadi agnosi sof
chr onicgr anulomat ousdi seaseofchi ldhood.
Whi choft hef oll
owi ngmostcl oselychar acter
izes
theabnor malityint hispat ient’sphagocy ti
ccells?
Decr easedkillingofmi croor gani smsbecauseof
enhancedpr oduct ionofhy drogenper oxide
Def iciencyofNADPHoxi daseact iv
ity
Impai redchemot axi sandmi grationcausedby
abnor mal micr otubul eformat i
on
Inabi li
tytokillstrept ococci

Alabor atoryexperi
menti sperf
ormedtoevaluate
thechemot act
icpotenti
al ofagroupofpot
ential
medi ators.Whichofthef oll
owi
ngsubstances
mostl ikelyhasthegreatestaff
init
yfor
neutrophils?
C5a
Fucosy ltransf
erase
β2-Integri
n
P-selectin

A26-
year-
oldAfri
can-
Americanwomanhas
bil
ater
alhi
laradenopathy
,andradiogr
aphy
reveal
smulti
plereti
cul
ardensi
tiesinbot
hlung
fi
elds.Abr onchoscopicbiopsyreveals
granulomat ousinfl
ammat ionwithmultipl
egi
ant
cell
soft heLanghanst ypeandnoev i
denceof
caseousnecr osis.Whichoft hefol
lowingist
he
mostl i
kelydiagnosis?
Aspergill
osis
Coccidioidomycosis
Histoplasmosis
Sarcoidosis

Inal aboratoryexer ci
seformedi calstudents,an
unknowncompoundi sst
udi ed.Thest udentsar e
i
nf ormedt hatt hecompoundhasbeeni solated
from endot hel
ial cell
sandthati tssynthesiscan
bei nhibi
tedbyaspi ri
n.I
nthel abor at
ory,the
student sdemonst rat
ethatthecompoundi sa
pot entvasodilatorandplateletant i
aggregant.
Givent hesef i
ndi ngs,t
hesubst ancei smostl ikel
y
whi choft hefollowingmedi ators?
5-HPETE
LTC4
LXA4
PGI 2

A70-y ear-
oldmanpresentswiththesuddenonset
oflef
t-sidedweakness,spasti
cit
y,and
hyperactiveandpat
hologicref
lexes.Themost
seri
ousconsequencesoft hisdisorderar
ethe
resultofdamaget owhichofthefol
l
owi
ngcel
l
types?
Labil
ecel l
s
Mul t
ipotentadul
tprogeni
torcel
l
s
Permanentcel l
s
Stablecells

I
nflammat ionofst
omachi
scal
l
ed:
Gastrit
is.
Enterit
is.
Colit
is.
Proctit
is.

I
nflammat i
onofl
i
veri
scal
l
ed:
Gastrit
is.
Hepat i
tis
Colit
is.
Proctit
is.

I
nflammat i
onofki
dneyi
scal
l
ed:
Gastri
ti
s.
Hepatit
is.
Nephrit
is.
Proct
iti
s.

I
nfl
ammationofsmal
li
ntest
inei
scal
l
ed:
Gast
ri
ti
s.
Ent
eri
ti
s.
Coli
ti
s.
Proct
it
is.

I
nflammat ionofl
argei
ntest
inei
scal
l
ed:
Gastrit
is.
Enterit
is.
Colit
is.
Proctit
is.

I
nflammat ionofr
ect
um i
scal
l
ed:
Gastrit
is.
Enterit
is.
Colit
is.
Proctit
is.

I
nfl
ammat i
onofski
niscal
l
ed:
Gastr
it
is.
Hepati
ti
s.
Nephri
tis.
Dermatiti
s.

I
nfl
ammat i
onofbl
oodv
essel
siscal
l
ed:
Gastr
it
is.
Hepati
ti
s.
Nephri
tis.
Vascul
itis.

I
nfl
ammat
ionofl
ungt
issuei
scal
l
ed:
Pneumonia.
Angina.
Fur
uncle.
Carbuncl
e.

I
nfl
ammat ionoft
hroati
scal
l
ed:
Pneumonia.
Angina.
Fur
uncle.
Carbuncl
e.

I
nfl
ammat ionofhai
rfol
l
icl
eiscal
l
ed:
Pneumonia.
Angina.
Fur
uncle.
Carbuncl
e.

I
nflammat i
onofbr
aini
scal
l
ed:
Gastri
ti
s.
Hepatit
is.
Enter
it
is.
Encephali
ti
s.

Localsignofi
nfl
ammat
ion“
rubor
”means:
Heat.
Pain.
Redness.
Swell
ing.
Localsignofi
nfl
ammat
ion“
cal
or”means:
Heat
Pain.
Redness.
Swell
ing.

Localsignofi
nfl
ammat
ion“
dol
or”means:
Heat.
Pain.
Redness.
Swell
ing.

Localsignofi
nfl
ammat
ion“
tumor
”means:
Heat.
Pain.
Redness.
Swell
ing.

Localsignofi
nfl
ammati
onwhichassoci
atedwi
th
pressureonnerv
eendi
ngsbyexudat
eis:
Rubor.
Calor.
Dolor.
Tumor .

Localsi
gnofinf
lammationwhi
chassoci
atedwi
th
accumulat
ionofexudat
eis:
Rubor.
Cal
or.
Dolor
.
Tumor.

Calorasal ocalsignofi
nfl
ammationi
s
associatedwith:
Arter
ialhyperemia.
Pressureofner v
eendings.
Exudation
Int
erferencewithenzymefunct
ion

Tumorasal ocalsi
gnofinfl
ammationi
s
associatedwith:
Arter
ialhyperemia.
Pressureofner v
eendings.
Exudation.
Int
erferencewithenzy
mef unct
ion.

Dolorasal ocalsignofi
nfl
ammat
ioni
s
associatedwith:
Arter
ialhyperemia.
Pressureofner v
eendings.
Exudation.
Localpain.

Ruborasalocalsi
gnofi
nfl
ammat
ioni
s
associ
atedwit
h:
Arter
ialhy
peremia.
Pressureofner
veendings.
Exudati
on.
I
nterfer
encewithenzy
mef unct
ion.

Componentofal ter
ati
ostageofinf
lammat
ioni
s:
Necrosi
s.
Art
erial
hy peremi
a.
Phagocytosis.
Regenerationofepi
thel
ial
cel
ls.

Componentofalter
ati
ostageofi
nfl
ammat
ioni
s:
Degener
ationofcel
ls.
Art
eri
alhyperemi
a.
Phagocyt
osis.
Regener
ationofepi
thel
ial
cel
ls

Componentofexudationstageofi
nfl
ammat
ioni
s:
Degenerat
ionofcel
l
s.
Art
erial
hyperemi
a.
Necrosi
s.
Regenerat
ionofepi
theli
alcell
s.

Componentofexudationstageofi
nfl
ammat
ioni
s:
Degenerat
ionofcel
ls
Necrosi
s.
Formati
onofcelli
nfi
lt
rate
Regener
ationofepi
theli
alcell
s.
Componentofexudationstageofi
nfl
ammat
ioni
s:
Degenerati
onofcel
l
s.
Necrosi
s.
Regener
at i
onofepi
theli
alcell
s.
Phagocytosi
s.

Componentofexudationstageofi
nfl
ammat
ioni
s:
Degenerat
ionofcel
l
s.
Necrosi
s.
Pi
nocy t
osi
s
Regenerat
ionofepi
theli
alcell
s.

Componentofexudationstageofinf
lammat
ioni
s:
Degenerat
ionofcel
l
s.
Necrosi
s.
Regenerat
ionofepi
theli
alcell
s
Leakageofbloodpl
asmaf rom vessel
s.

Componentofexudationstageofinfl
ammat
ioni
s:
Degenerat
ionofcell
s.
Necrosi
s.
Regenerat
ionofepit
heli
alcell
s.
Emigrat
ionofbloodcell
sfrom vessel
s.

Componentofpr
oli
fer
ati
onst
ageofi
nfl
ammat
ion
i
s:
Degener
ati
onofcel
ls.
Necrosi
s.
Regenerat
ionofepit
hel
i
alcel
ls.
Emigrat
ionofbloodcel
l
sfrom vessel
s

Componentofpr oli
ferati
onstageofinf
lammat
ion
i
s:
Degenerat
ionofcells.
Necrosi
s.
Regenerat
ionoffibroblast
s.
Emigrat
ionofbloodcel lsfr
om vessel
s.

Typeofacut ei
nfl
ammat i
onwhi
chassoci
ated
wit
hf ormati
onofprot
ein-
poorf
lui
discal
l
ed:
Serous.
Fi
brinous.
Purulent
Hemor rhagi
c.

Typeofacut ei
nfl
ammati
onwhichassoci
ated
wit
hf ormati
onofhai
r-
li
kedeposi
tsi
scall
ed:
Serous.
Fi
brinous.
Purulent.
Hemor rhagi
c.

Typeofacutei
nfl
ammationwhi
chassoci
ated
wit
hf or
mati
onofpseudomembraneonmucosal
surf
acesiscal
l
ed:
Ser
ous.
Fi
bri
nous.
Pur
ulent.
Hemorrhagi
c.

Typeofacuteinf
lammati
onwhichassoci
ated
wit
hf ormati
onofthi
ckt
urbi
dyell
ow-gr
eenflui
dis
cal
led:
Serous.
Fi
brinous.
Purulent.
Catarrhal
.

Whichtypeofpneumoni aacutei
sassoci
atedwi
th
formati
onoff i
brinousexudat
e:
Lobarpneumoni a.
Bronchpneumoni a.
Int
erst
iti
alpneumoni a.
Septi
cpneumoni a.

Locali
zedty
peofpur
ulenti
nfl
ammat
ioni
scal
l
ed:
Phlegmon.
Empy ema.
Abscess.
Furuncl
e.

Di
ff
uset
ypeofpur
ulenti
nfl
ammat
ioni
scal
l
ed:
Phl
egmon.
Empyema.
Abscess.
Fur
uncle.

Typeofpurulenti
nfl
ammationwi
thaccumulati
on
ofpusinserosalcavi
ti
esandhol
l
owor gansis
cal
led:
Phlegmon.
Empy ema.
Abscess.
Furuncl
e.

Typeofpurulenti
nfl
ammationwi
thaccumulati
on
ofpusinserosalcavi
ti
esandhol
l
owor gansis
cal
led:
Phlegmon.
Empy ema.
Abscess.
Furuncl
e.

Purul
enti
nfl
ammat
ionofhai
rfol
l
icl
eiscal
l
ed:
Phl
egmon.
Empyema.
Abscess.
Fur
uncle.

Whi
choft
hef
oll
owi
ngar
ethoughtt
omedi
ate,
manyoft hesy st
emi cef
fectsofinfl
ammat i
onare
chemotact i
candst i
mulateadhesionmolecules:
Int
erl
eukin-1(IL-
1)andtumornecr osi
sfactor
C5aandl eukotr
ieneB-4
C3b.
Leukotr
ieneC4, D4andE4.

Aft
erini
ti
ati
onofanacuteinfl
ammatoryprocess
thi
rdinasequenceofchangesinv
ascularf
lowis:
Vasoconstr
ict
ion.
Redness.
Leukocyt
icmigrat
ion.
Vasodil
ati
on

Thet erm“croupous”inf
lammat
ioni
s
synony mouswi th
Catarrhali
nfl
ammat i
on
Fi
brinousinfl
ammat i
on
Sero-fi
bri
nousinflammation
Suppur at
iveinf
lammat i
on

Vascularphenomenoninacutei
nfl
ammation
hel
psi n:
Bri
ngingthedefensi
vecel
lsli
keneutr
ophi
l
sfrom

i
nteri
orofthevessel
stot
hesiteofi
rri
tat
ion
Mini
mi zi
ngtheeff
ectofi
rr
it
antbydi
luti
ngit
consi
derably
Li
mi t
ingtheinf
lammat
oryr
eact
iont
othesmal
l
est
possiblear
ea
All
oft hese

Whichoft
hecompl
ementcomponent
sactas
chemoki
nes?
C3b
C4b
C5a
C4a

Al
laretypesofti
ssuemacr
ophages,
except
:
Li
tt
oralcell
s
Hoff
bauercell
s
Ost
eoclasts
Ost
eoblasts

Maincytokinesactingasmedi at
orsof
i
nfl
ammat i
onar easunder,except:
I
nterl
eukin-
1( I
L-1)
Tumornecr osi
sf actorα(
TNF- α)
Nit
ri
coxide( NO)
I
nterf
eron–y( IF-
y )
Mecel
l
aneous4
MESENCHYMALTUMORS
1.Malignanttumorar
isi
ngf
rom mesenchy
mal
cell
siscalled:
A.Leukemi a.
B.Adenoma.
C.Carcinoma.
D.Sarcoma.
E.Teratoma.
2.Malignanttumor sar
isi
ngf
rom mesenchy
mal
ti
ssuear ereferredt
oas:
A.Sarcomas
B.Adenocar cinomas
C.Papill
omas
D.Cystadenomas
E.Pol
y ps
3.Beni gntumorar
isi
ngf
rom connect
ivet
issuei
s
call
ed:
A.Hy bernoma.
B.Leiomy oma.
C.Lipoma.
D.Fibroma.
E.Ost eoma.
4.Benignt
umorar
isi
ngf
rom f
ibr
oust
issuecal
l
ed:
A.Leiomyoma
B.Myoma
C.Chondroma
D.Rhabdomyoma
E.Fi
broma
5.
Beni gntumorar
isi
ngf
rom f
att
yti
ssuei
scal
l
ed:
A.
Hy bernoma.
B.
Leiomy oma.
C.
Lipoma.
D.
Fibroma.
E.
Ost eoma.
6.Sel
ectthenameofabeni
gnt
umoror
igi
nat
ing
fr
om adiposeti
ssue:
A.Adenoma
B.Li
poma
C.Chondroma
D.Rhabdomy oma
E.Li
posarcoma
7.Tumorhi st
ologicalconsist
ing oflobul
esof
mature adipose cel l
s separated by delicat
e
fi
broussepta,cal
led:
A.Chondr oma
B.Rhabdomy oma
C.Leiomy oma
D.Lipoma
E.Fibroma
8.
Beni gntumorarisi
ngfrom brownf ati
scal
led:
A.Hybernoma.
B.
Leiomyoma.
C.
Lipoma.
D.
Fibroma.
E.
Osteoma.
9.Sel
ectthenameofabeni
gnt
umoror
igi
nat
ing
fr
om brownf at
:
A.Hybernoma
B.Li
posarcoma
C.Chondroma
D.Rhabdomy oma
E.Fi
brosarcoma
10.Beni gnt
umorar
isi
ngf
rom smoot
hmuscl
esi
s
call
ed:
A.Hy bernoma.
B.Leiomy oma.
C.Rhabdomy oma.
D.Fibroma.
E.Ost eoma.
11.Leiomy omasoft heuteri
near echar
act
eri
zed
byallofthefoll
owing,EXCEPT:
A.Arisefrom ut
erinemuscl e
B.Mayunder godegener ati
vechanges
C.Arehor mone-dependentt umors
D.Usuall
yar emultipl
etumor s
E.Alwaysunder gomal i
gnization
12.
Beni
gnt
umorar
isi
ngf
rom st
ri
atedmuscl
esi
s
call
ed:
A.Hy bernoma.
B.Leiomy oma.
C.Rhabdomy oma.
D.Fibroma.
E.Ost eoma.
13.Beni
gntumorar
isi
ngf
rom bl
oodv
essel
sis
call
ed:
A.Hybernoma.
B.Lei
omy oma.
C.Rhabdomyoma.
D.Hemangioma.
E.Lymphangi
oma.
14.Themi croscopicalfeaturesofthecaver
nous
hemangiomaar eallofthef oll
owing,
EXCEPT:
A.Sharplydefined
B.Part
lyorcompl et
elyfil
ledwi t
hblood
C.Cavernousv ascularspaces, someti
meswi t
h
thr
ombosi s
D.Micromet astases
E.Separatedbyascantconnect i
veti
ssuestr
oma
15.Beni
gntumorar
isi
ngf
rom l
ymphv
essel
sis
call
ed:
A.Hybernoma.
B.Lei
omy oma.
C.Rhabdomyoma.
D.Hemangioma.
E.
Lymphangi
oma.
16.Benigntumorari
singf
rom bonei
scal
l
ed:
A.Osteoma.
B.Chondr oma.
C.Synovioma.
D.Mesot hel
ioma.
E.
Ost eosarcoma.
17.Benigntumorar
isi
ngf
rom ost
eobl
ast
sis
cal
led:
A.Chondrosarcoma
B.Adenoma
C.Osteoma
D.Fibr
oma
E.Papil
loma
18.Benignt umorari
singf
rom car
ti
lagei
scal
l
ed:
A.Osteoma.
B.Chondr oma.
C.Synovioma.
D.Mesot helioma.
E.
Chondr osarcoma.
19.Selectthenameofabenignt
umoror
igi
nat
ing
fr
om car t
il
aginoust
issue:
A.Chondr oma
B.Adenoma
C.Osteoma
D.Fibr
oma
E.Papi
l
loma
20.Benigntumorari
singf
rom i
nnerl
ayerofj
oint
s
i
scal l
ed:
A.Osteoma.
B.Chondr oma.
C.Synovioma.
D.Mesot hel
ioma.
E.
Hy bernoma.
21.Malignantt umorar
isi
ngf
rom car
ti
lagi
nous
ti
ssueiscal led:
A.Chondr osar coma
B.Adenocar cinoma
C.Osteosar coma
D.Fibr
osar coma
E.Papil
loma
22.Mal ignantt umorar
isi
ngf
rom connect
ive
ti
ssuei scal l
ed:
A.Angi osar coma.
B.Leiomy osarcoma.
C.Liposar coma.
D.Fibrosar coma.
E.Osteosar coma.
23.Malignanttumorar
isi
ngf
rom f
ibr
oust
issue
cal
led:
A.Leiomy osar
coma
B.Papil
loma
C.Chondrsarcoma
D.Rhabdomy oma
E.Fi
brosarcoma
24.Mal ignantt umorar
isi
ngf
rom f
att
yti
ssuei
s
call
ed:
A.Angi osar coma.
B.Leiomy osarcoma.
C.Liposar coma.
D.Fibrosar coma.
E.Osteosar coma.
25.Selectthenameofamal i
gnantt
umor
ori
ginati
ngf r
om adiposet
issue:
A.Lipoma
B.Liposarcoma
C.Chondr oma
D.Rhabdomy osar
coma
E.Fi
br oma
26.Mal ignantt umorar
isi
ngf
rom bonesi
scal
l
ed:
A.Angi osar coma.
B.Leiomy osarcoma.
C.Liposar coma.
D.Fibrosar coma.
E.
Ost eosar coma.
27.Mali
gnanttumorar
isi
ngf
rom ost
eobl
ast
sis
cal
led:
A.Chondrosar
coma
B.Adenocarcinoma
C.Osteosarcoma
D.Fi
brosarcoma
E.Papil
loma
28.Mal ignantt umorar
isi
ngf
rom bl
oodv
essel
sis
call
ed:
A.Angi osar coma.
B.Leiomy osarcoma.
C.Liposar coma.
D.Fibrosar coma.
E.Osteosar coma.
29.Mal ignantt umorar
isi
ngf
rom smoot
hmuscl
es
i
scal led:
A.Angi osar coma.
B.Leiomy osarcoma.
C.Liposar coma.
D.Fibrosar coma.
E.
Ost eosar coma.
30.Selectthenameofamalignantt
umor
ori
ginatingfrom smoot
hmusclecel
l
s:
A.Leiomy osarcoma
B.Papi l
l
oma
C.Chondr sarcoma
D.Rhabdomy oma
E.Fi
br osarcoma
31.
Mal
i
gnantt
umorar
isi
ngf
rom st
ri
atedmuscl
es
i
scalled:
A.
Angi osarcoma.
B.
Leiomy osarcoma.
C.
Liposarcoma.
D.
Fibrosarcoma.
E.
Rhabdomy osar
coma.
32.Selectthenameofamal ignantt
umor
ori
ginati
ngf rom stri
atedmuscl
ecell
s:
A.Mesot helioma
B.Chondr sarcoma
C.Fibrosarcoma
D.Osteosar coma
E.Rhabdomy osar
coma
33.Tumorhi stologicalconsi
sti
ngofl ar
ge, r
ound
orovalcel
lswi thgranulated,aneosinophil
ic
cyt
oplasm; and, asar ul
e,tr
ansversalst
ripesare
i
nsomecel l
s, call
ed:
A.Lei
omy osar coma
B.Papil
loma
C.Chondrsarcoma
D.Rhabdomy oma
E.Fi
brosarcoma
34.Benignt
umorar
isi
ngf
rom mel
anocy
tescel
l
s
i
scalled:
A.Chondroma
B.Adenoma
C.Nev us
D.Fi
broma
E.Papi
ll
oma
35.Commonestt ypeofpi
gment
ednev
usi
s:
A.Juncti
onalnevus.
B.Compoundnev us.
C.I
ntradermalnevus.
D.Juvenil
enevus.
E.
Bluenev us.
36.Whichtypeofnevusof
tenqui
ter
ichi
nmel
ani
n
pi
gment ?
A.Juncti
onalnevus.
B.Compoundnev us.
C.I
ntradermalnevus.
D.Juvenil
enevus.
E.
Bluenev us.
37.Whi chtypeofnevusconsi
stsofepi
thel
i
oid-
li
ke
cells?
A.Junct i
onalnevus.
B.Compoundnev us.
C.Intr
adermal nevus.
D.Juvenilenevus.
E.Bluenev us.
38.Whichtypeofnevushasincreasedr
iskof
progressi
ontomalignantmelanoma?
A.Juncti
onalnev
us.
B.Compoundnev us.
C.
Int
radermalnevus.
D.
Juvenil
enevus.
E.
Dysplast
icnevus.
39.Malignantt umorar
isi
ngf
rom mel
anocy
tes
cel
lsiscalled:
A.Chondr osarcoma
B.Adenocar cinoma
C.Osteosar coma
D.Fibr
osar coma
E.Melanoma
40.Whichty peofmelanomai sessent
ial
l
ya
malignantmel anomai nsit
u?
A.Lenti
gomal i
gnamel anoma.
B.Superfi
cialspreadi
ngmel anoma.
C.Acrall
ent i
genousmel anoma.
D.Pagetoidmel anoma.
E.
Nodul armel anoma.
41.Whicht y peofmelanomaoccur smor
e
commonl yont hesol
es, pal
msandmucosal
surfaces?
A.Lentigomal i
gnamel anoma.
B.Super fi
cialspreadi
ngmel anoma.
C.Acral l
ent i
genousmel anoma.
D.Paget oidmel anoma.
E.Nodul armel anoma.
42.Themi
croscopi
cal
feat
uresofmal
i
gnantski
n
melanomaar ealloft hefol
lowing,EXCEPT:
A.Brown-bl
ackcy toplasmicgranules
B.Largenucl
eiwi t
hi rregul
arcontours
C.Clumpedchromat inundernucl eol
armembrane
D.Glandul
arformat i
on
E.Formati
onoft het umornests
43.Themacr oscopicalf
eaturesofsecondaryl
i
ver
i
nvolvementi nmel anomaar eallofthefol
lowi
ng,
EXCEPT:
A.Enlargedl i
ver
B.Yell
ow- greencolorofimplants
C.Mul t
iplenodularimplant
s
D.Black-browncol orofimplants
E.Tuberoussur face
44.Development al
tumori
s:
A.Teratoma.
B.Nephr oblastoma.
C.Reti
nobl astoma.
D.Medul loblastoma.
E.
Neur obl astoma.
45.Embr yonictumorar
isi
ngf
rom embr
yoni
c
ti
ssuesi s:
A.Chordoma.
B.Amel oblast
oma.
C.Craniopharingeoma.
D.Medul l
oblastoma.
E.Branchialcysts.
46.Embr yonictumorar
isi
ngf
rom embr
yoni
c
ti
ssuesi s:
A.Chordoma.
B.Amel oblastoma.
C.Craniopharingeoma.
D.Nephr oblastoma.
E.Branchialcysts.
47.Embr yonictumorar
isi
ngf
rom embr
yoni
c
ti
ssuesi s:
A.Chordoma.
B.Amel oblast
oma.
C.Craniopharingeoma.
D.Neuroblastoma.
E.Branchialcysts.
48.Embr yonict umorar
isi
ngf
rom embr
yoni
c
ti
ssuesi s:
A.Chordoma.
B.Amel oblastoma.
C.Craniophar i
ngeoma.
D.Hepat oblastoma.
E.Branchialcyst s.
49.Embryonict umorar
isi
ngf
rom embr
yoni
c
vesti
gesis:
A.Nephroblastoma.
B.Ameloblastoma.
C.Reti
noblastoma.
D.
Medull
oblast
oma.
E.
Neur
oblastoma.
50.Embry oni ctumorar
isi
ngf
rom embr
yoni
c
vesti
gesi s:
A.Nephrobl astoma.
B.Chordoma.
C.Reti
nobl astoma.
D.Medullobl astoma.
E.Neuroblast oma.
51.Embry oni ct umorar
isi
ngf
rom embr
yoni
c
vesti
gesi s:
A.Nephrobl ast oma.
B.Crani
ophar ingeoma.
C.Reti
nobl astoma.
D.Medullobl ast oma.
E.Neuroblast oma.
52.Tumorar isingfrom hi
ndbr
aini
scal
l
ed:
A.Nephroblast oma.
B.Crani
ophar ingeoma.
C.Reti
noblastoma.
D.Medulloblast oma.
E.
Neur oblastoma.
53.Tumorari
singfr
om par
api
tui
tar
yresi
duesi
s
call
ed:
A.Nephrobl
astoma.
B.Crani
ophari
ngeoma.
C.
Reti
noblastoma.
D.
Medulloblast
oma.
E.
Neuroblastoma.
54.Tumorar isingfr
om enamel
organi
scal
l
ed:
A.Ameloblast oma.
B.Crani
ophar ingeoma.
C.Reti
noblastoma.
D.Medulloblastoma.
E.
Neur oblastoma.
55.Tumorari
singfr
om not
ochor
dor
gani
scal
l
ed:
A.Amelobl
astoma.
B.Crani
ophar
ingeoma.
C.Reti
nobl
astoma.
D.Chordoma.
E.
Neur obl
ast
oma.
56.Origi
nofWi
l
mst
umor
sis:
A.Hear t
.
B.Lung.
C.Li
v er.
D.Kidney.
E.
Spl een.
57.Whichtumori sari
sedf
rom l
i
ver
?
A.Nephroblastoma.
B.Hepatoblastoma.
C.Ret
inoblastoma.
D.Medulloblastoma.
E.
Neur
obl
ast
oma.
58.Whichembr yoni
ctumori
sbeni
ng?
A.Nephroblastoma.
B.Reti
nobl astoma.
C.Medulloblastoma.
D.Ameloblastoma.
E.
Neurobl astoma.
59.Whichmal i
gnantembr
yoni
ctumormay
tr
ansformt obeni gn?
A.Nephroblastoma.
B.Hepatoblastoma.
C.Melanoma.
D.Angiosarcoma.
E.Neuroblastoma.
60.Local
izat
ionofEwi
ngsar
comai
s:
A.Bone.
B.Cart
il
age.
C.Fat.
D.Synovi
um.
E.
Mesot heli
um.
61.Malignanttumorwhi
chassoci
atedwi
thAI
DS
i
scalled:
A.Angiosarcoma.
B.Lei
omy osar
coma.
C.Chondr osar
coma.
D.Kaposi’ssarcoma.
E.
Ost
eosar
coma.
62.Atumort hattendstospreadoverthesur
faces
of or gans or body cav it
ies rather than
metastasinngv i
abloodv essel
sorlymphaticsis
whichoft hef ol
lowing:
A.Coloncar cinoma
B.Thyroidcar ci
noma
C.Mesot heli
oma
D.Renal cel
l carci
noma
E.Hepatocellularcarci
noma
63.Firsthematogenousmetast
asesofthef
emur
softti
ssuef i
brosarcomacanbefoundi
nwhichof
thefollowingorgans:
A.Liver
B.Brain
C.Bones
D.Lungs
E.Regi onall
ymphnode
64.Listedtumorsoff
ibr
oust
issuear
ebeni
gn,
except:
A.Fibromadur um
B.Softf i
broma
C.Mesot heli
oma
D.Fibroli
poma
E.Elastofi
broma
65.Li
stededucat
ionsoff
ibr
oust
issuear
e
tumor
-li
kelesions,except
:
A.Fi
broushamar t
omaofi nfancy
B.Sof
tfibroma
C.Nodularfascii
ti
s
D.Ret
roperitonealfi
bromatosis
E.Kel
oid
66.Benigntumor scomposedofei thercav
ernous
spacesorser penti
necapil
lar
y-l
i
kechannels
contai
ningbloodorl ymphareallofthefol
lowi
ng,
EXCEPT:
A.Capil
laryl
y mphangiomas
B.Cavernoushemangi omas
C.Capil
larytelangi
ectases
D.Cavernousl ymphangiomas
E.Capil
lar
yhemangi omas
67.Themostf r
equentl
ocal
i
zat
ionoft
erat
omasi
n
chi
ldren:
Aov ari
es
Btesticles
Csacr ococcygealar
ea
Dmedi ast
inum
Ecav i
tyoft heskul
l
68.
Themostcharact
eri
sti
ctumorf
orchi
l
dhoodi
s:
Ahemangioma
Bmelanoma
Cgli
obl
astoma
Dchori
oncarci
noma
Eost
eosar
coma

1.Durati
onofblast
ogenesi
sis:
А.1st-
15thday
sofprenat
aldevel
opment
.
st t h
В.1 -20 daysofprenat
aldevel
opment
.
С.5th-
20thday
sofprenat
aldevel
opment
.
D.5th-
25thday
sofprenat
aldevel
opment
.
st t h
Е.1 -25 daysofprenat
aldevel
opment
.
2.Dur ati
onofembr yogenesi
sis:
th th
А.16 - 66 day sofprenat
aldevel
opment
.
В.15th-75thdaysofprenat
aldevel
opment
.
С.20th-80thdaysofprenat
aldevel
opment
.
th th
D.16 - 75 day sofprenat
aldevel
opment
.
Е.16th-80thday
sofpr enat
aldevel
opment
.
3.Dur ati
onofear l
yfet
ogenesisi
s:
А.80th-180thdaysofprenat
aldevel
opment
.
th th
В.76 - 180 day sofpr
enataldevel
opment
.
th th
С.70 - 170 day sofprenat
aldevel
opment
.
D.76th-175thdaysofprenat
aldevel
opment
.
Е.70th-180thday
sofprenataldev
elopment
.
4.Durationofl at
efetogenesisis:
th th
А.180 - 250 day sofprenataldeveel
opment.
th th
В.170 -280 day sofprenataldevel
opment.
С.185th-285thdaysofprenataldevel
opment.
th th
D.180 - 280 day sofprenataldevel
opment.
st th
Е.181 -280 day sofprenataldevel
opment.
5.Endogeniceti
ologi
cfact
orsofky
mat
opat
hies
are:
А.Genicmut at
ions.
В.Radiati
on.
С.Alcohol.
D.Met al
s.
Е.Inf
ecti
ons.
6.Exogeni
cet iologicfactorsofkymat
opat
hies
are:
А.Genicmut ations.
В.Endocri
nedi seasesofmot her.
С.Drugsusingdur ingpregnancy.
D.Gonadsov er ri
ping.
Е.Cri
ti
calperiodsofpar ent
sage.
7.Sy ndromescausedbyanomal i
esi nthesy st
em
ofaut osomesinclude:a)Sher
eshev sky-Ter
-Cher
Syndr ome;b)Kli
nefelt
er'
ssyndrome; c)Down’ s
syndr ome;d)t
hePat ausyndrome; e)Edwar d’s
syndr ome
A.a,b,c,
d
B.a,c,e
C.b,c
D.c,d,e
E.a,b,c,
d,e
8.Charact
eri
sti
cofPat
ausy
ndr
omei
s:
Apolydact
yly
Banencephaly
Cumbili
calhernia
DKidneydoubling
Eagyri
a
9. Sher eshevsky -
Turner’
s sy ndr
ome i s
characteri
zedbyev eryt
hingEXCEPT:
Alowgr owth
Bshor tfoldedneck
Ccoar ctati
onoft heaor t
a
Dhy poplasiaofthepeni s
Edisturbancesint heformationoft
heov ar
ies
10.Down’ sdisease-i
s:
А.Gamet opathy .
В.Blastopathy.
С.Embr yopathy.
D.Earlyfetopathy.
Е.Latefetopathy.
11.Ectopicpegnancy-i
s:
А.Gamet opathy .
В.Blastopathy.
С.Embr yopathy.
D.Earlyfetopathy.
Е.Latefetopathy.
12.Siamesetwins-i
s:
А.Gamet opat
hy.
В.Blast
opathy.
С.Embryopathy.
D.Ear
lyfetopathy
.
Е.Lat
efetopathy.
13.Congeni talmalfor
mat
ion-compl
eteabsence
ofanor gani scall
ed:
А.Agenesi s.
В.Aplasia.
С.Hypopl asia.
D.Hyper plasia.
Е.Dysplasia.
14.Congeni talmal f
ormati
on-absenceof
devel
opmentofanor ganwit
hpresenceofanl
age
(r
udiment )iscalled;
А.Agenesi s.
В.Aplasia.
С.Hypopl asia.
D.Hyperpl asia.
Е.Dysplasia.
15.Congeni talmal
for
mati
on-i
ncompl
ete
devel
opmentofanor gani
scal
l
ed:
А.Agenesi s.
В.Aplasia.
С.Hypopl asia.
D.Hyperpl asia.
Е.Dysplasia.
16.Congeni
tal
mal
for
mat
ion-ov
erdev
elopment
ofanor gani scall
ed:
А.Agenesi s.
В.Aplasia.
С.Hypopl asia.
D.Hyper plasia.
Е.Dysplasia.
17.Congeni t
almal
for
mation-absenceofl
umen
i
nf ol
lowor gansi
scal
led:
А.Atresia.
В.Stenosis.
С.Dysraphia.
D.Extrophia.
Е.Ectopia.
18.Congeni t
almal
for
mati
on-def
ectr
esul
ti
ng
fr
om fail
ureoffusi
on(i
namiddl
eli
neofor
gan)i
s
cal
led:
А.Atresia.
В.Stenosis.
С.Dysraphia.
D.Extrophia.
Е.Ectopia.
19.Congeni t
almalf
ormat
ion-abnor
mal
locat
ion
ofanor ganiscal
led:
А.Atresia.
В.Stenosis.
С.Dysraphia.
D.Extrophia.
Е.Ect
opi
a.
20.Congenital
malf
ormat
ion-absenceofbr
aini
s
cal
led:
А.Anencephaly.
В.Exencephaly.
С.Ini
oncephaly.
D.Microcephaly
.
Е.Hydrocephaly
.
21.Congeni talmalf
ormati
on-absenceof
occipit
al boneandlocat
ionofbr
aininpost
eri
or
cranialfossaiscall
ed:
А.Anencephal y.
В.Exencephal y.
С.Inioncephaly.
D.Mi crocephaly.
Е.Hy drocephaly.
22.Congenital
mal f
ormat
ion-
absenceofgy
riand
sul
ciofbrainiscall
ed:
А.Anencephaly.
В.Agyri
a.
С.Microgyri
a.
D.Microcephaly.
Е.Porencephaly
.
23.Congeni
talmalfor
mation-accumul
ati
onof
excessi
vecerebr
ospinal
flui
dwithi
nthevent
ri
cul
ar
system oft
hebrain:
А.Anencephaly.
В.Exencephaly
.
С.Ini
oncephaly.
D.Microcephal
y.
Е.Hydrocephal
y.
24.Congenitalheartdiseasewhi chassoci
ated
wit
habnor mal di
visi
onofcar di
accavit
iesi
s:
А.Commont runcusar teri
osus.
В.Coarctat
ionofaor ta.
С.Ventri
cularseptaldef ect
.
D.Patentductusar t
eriosus.
Е.Transposit
ionofgr eatarteri
es.
25.Congeni talheartdiseasewhi chassoci
ated
wit
habnor mal di
visi
onofcar di
accavit
iesi
s:
А.Commont runcusar ter
iosus.
В.Coar ctat
ionofaor ta.
С.Tricamer alheart.
D.Pat entductusar t
eriosus.
Е.Transposi t
ionofgr eatarteri
es.
26.Congeni talheartdiseasewhi chassoci
ated
wit
hanomal i
esofgr eatarteri
esis:
А.Atrialseptaldefect.
В.Vent r
icul
arsept aldefect.
С.Tricamer alheart.
D.Bicamer alheart.
Е.Transposi t
ionofgr eatarteri
es.
27.Congenitalheartdiseasewhichassoci
ated
wit
hanomal i
esofgr eatarter
iesi
s:
А.Atr
ialseptaldefect.
В.Ventri
cularseptaldefect.
C.Patentductusar t
eriosus.
D.Tri
camer alheart.
E.Bi
camer alheart.
28.TetralogyofFall
otischar act
eri
zedbyal
lthe
fol
lowing, EXCEPT:
А.Atri
al septaldef
ect.
В.Ventricularsept
aldefect.
С.Stenosi sofpulmonaryartery.
D.Rightv entri
cul
arhypertr
ophy .
Е.Dextrapositi
onofaorta.
29.Noninfect i
ousfet
opat
hyi
s:
А.Syphil
is.
В.Li
steri
osi s.
С.Toxoplasmosi s.
D.Mucov iscidosi
s.
Е.Cytomegal y.
30.Themal f
ormati
onoft
hecent
ral
ner
vous
syst
em incl
udes:
A.apus
B.acrani
a
C.syndacty
ly
D.hydrocel
e
E.pi
ocephaly
31.Themal for
mat
ionoft
hef
aci
alskul
li
s:
A.Syndactyl
y
B.simpodium
C.synoty
D.Monarchism
E.focomelia
32.Themal f
ormat
ionoft
hef
aci
alskul
li
s:
A.cycl
opia
B.Syndacty
ly
C.hypospadia
D.amelia
E.epi
spadia
33.Themal for
mat
ionoft
hegeni
to-
uri
nar
yor
gans
i
s:
A.Monar chi
sm
B.acrania
C.syndactyl
y
D.amel i
a
E.focomelia
34.Themalformat
ionoft
hegeni
to-
uri
nar
yor
gans
i
s:
A.anarchi
sm
B.apus
C.syndact
yly
D.cycl
opia
E.pi
ocephaly
35.Themal f
ormat
ionoft
hegeni
to-
uri
nar
yor
gans
i
s:
A.hypospadia
B.acrania
C.amel i
a
D.hydrocephal
y
E.cyclopi
a
36.Themal f
ormat
ionoft
hegeni
to-
uri
nar
yor
gans
i
s:
A.epispadi
a
B.Simpus
C.simpodium
D.apus
E.focomeli
a
37.Themal f
ormati
onoft
heost
eoar
ti
cul
ar
system i ncl
udes:
A.focomel i
a
B.acr ania
C.hy drocephaly
D.hy pospadi a
E.cycl opia
38.Themal formati
onoft
heost
eoar
ti
cul
ar
syst
em includes:
A.epispadia
B.amel i
a
C.hypospadia
D.anarchism
E.Monar
chi
sm
39.Amel i
aisanabsenceof
:
A.fingers
B.toes
C.fingersandtoes
D.upperandl owerextr
emit
ies
E.Br ushesandfeet
40.Simpusis:
A.fusi
onoffinger
s
B.fusi
onoftoes
C.fusi
onofthelowerextr
emities
D.fusi
onofupperextremit
ies
E.absenceofupperandlowerextremi
ti
es
41.From whichdayant
enat
alphaseofper
inat
al
peri
odi sst
arted?
t
h
А.150 day .
В.154thday.
С.156thday.
t
h
D.160 day .
th
Е.180 day.
42.Bodyweightofpr
emat
urenewbor
nisl
ess
than:
А.2000gr.
В.2300gr.
С.2500gr.
D.2600gr.
Е.2650gr
.
43.Bodyl
engt
hofpr
emat
urenewbor
nisl
esst
han:
А.35sm.
В.40sm.
С.45sm.
D.48sm.
Е.50sm.
44.Thesi gnsofpremat ur
it
yincludeevery
thi
ng
except:
Afluffybodyhairs
Blowl yi
ngear s
Cthet est
iclesarenotloweredintothescrot
um
D t he nucleus of ossifi
cati
on of t he l
ower
epiphysi
soft hefemuris0.2cm.
Enai l
platesful
lycoverthenailbed
45.Mor phological signsofasphyxi
aar eallt
he
foll
owi ng,EXCEPT:
А.Cy anosi sofski n.
В. Petechialhemor rhagesinmucosal membr anes
andser osalsurfaces.
С.Venoushy peremi aofinnerorgans.
D.For mat i
onoft hrombi i
nbloodv essels.
Е.Dar k-redfluidbloodi nheartchamber sand
bloodv essel lumens.
46.Whichv
ari
antofneonat
alrespi
rat
orydistr
ess
syndr
omeisassoci
atedwithsever
ebl ockadeof
gasexchange?
А.Massiveaspi rati
onofamniot
icf
luid.
В.Pri
mar yatelectasi
s.
С.Secondaryat elect
asi
s.
D.Int
ra-
alveolaredemaandhemor rhages.
Е.Hyali
nemembr anedi
sease.
47.ForHy alinemembr anedi seaseisNOTt ypi
cal:
Alungsar edar kr edcolor,"r
ubber "consi
stency
Bat el
ect asisoft helungs
Cinf l
ammat oryreacti
oni nthest r
oma
Dhy alinemembr anebase-f i
brin
Ehemor r
hagei nt helumenoft healveoli
48.Whi chv ari
antofneonat alrespir
atorydistr
ess
syndrome i s associ ated wi th сongeni tal
surfactantdef iciency?
А.Massi veaspi rati
onofamni oti
cf l
uid.
В.Pr i
mar yat electasi
s.
С.Secondar yat elect
asis.
D.Intra-alveolaredemaandhemor rhages.
Е.Hy alinemembr anedisease.
49.Thepr esenceofst
il
lbornlungswit
hagr eeni
sh
ti
nge,as wel las the appear ance of dense
greenishmassesont hei ncisi
onunderpressure,
pointsto:
Abr onchopulmonarydysplasia
Bdi seasesofhyali
nemembr anes
C sy ndromeofmassi veaspi rat
ionofamni oti
c
f
lui
dandmeconi um
Datel
ectasi
softhelungs
Econgenit
albr
onchiect
asi
s
50.Af ter t
he compl
eti
on of what week of
pregnancydoesthecal
culat
ionoftheperi
nat
al
peri
odbegin?
A.20weeks
B.22weeks
C.24weeks
D.25weeks
E.28weeks
51.Classif
icat
ionoft heper
inat
alperi
od.Speci
fy
the correct sequence:а) early neonat
al;b)
i
ntranatal
;c) ant enatal
;d) late neonat
al;e)
prenatal
A.c,b,
a
B.e,a,
d
C.b,c,
a
D.a,b,
c,d
E.a,d
52.Att hebaseoft hefor
mati
onofspinalher
nias
thereisav iolat
ionof:
At hemul ti
pli
cationofcell
s
Bcel l
mi grati
on
Ccel ldi
fferenti
ation
D apopt osis i n t he process of nor mal
embr yogenesi s
EAdhesi
onoft
issues
53.Att hebaseoft hef
ormationofsy
ndacty
ly
thereisav iolat
ionof:
At hemul ti
pli
cationofcel
ls
Bcel l
mi grati
on
Ccel ldi
fferenti
ation
D apopt osis i n t he process of normal
embr yogenesi s
EAdhesi onoft issues
54. What char acter
izes the sy ndr
ome of
congeni t
alr
ubell
a:
Acatar act
B defectoft heint
ervent
ri
cularseptum oft
he
heart
Cdeaf ness
Dhepat osplenomegaly
Ealll
isted
55.Wi thicter
icform ofhemoly
ticdi
seasef
rom
thesideoft hebr
aintakespl
ace:
Aicteri
cencephalit
is
Bicteri
cencephalopathy
Cbili
rubinosis
Dmeni ngiti
s
Eever y
thingispossi
ble
56.Theascendi
ngr
out
eofi
nfect
ionoft
hef
etus
i
smor e:
AStaphyl
ococcus
Bvir
uses
C)Li
steri
a
Danaerobici
nfect
ion
EAlll
ist
ed
57.Infectionwithcyt
omegal
ovir
usoft
hef
etus
mostof tenoccurs:
Afecal-oral
Bhemat ogenous
Clymphogeni cal
ly
Dascendi ngway
Ealloptionsareequal
l
ypossi
ble
58.Forbirt
hinj
uryismor etypical:
Aepiduralhemorrhage
Bsubarachnoidhemor r
hage
Chemor rhageint
ot hebrai
nv entri
cles
Dhemor rhageinthesubcorti
cal nuclei
Eruptur
eoft hecerebel
lum
59.Topneumopathyofnewbor ndoesNOTappl
y:
Ahyali
nemembr anedi sease
Bedematoushemor r
hagicsyndrome
Cmassiveaspir
ationsy ndrome
Dbronchopul
monar ydy spl
asia
Esecondaryat
electasis
60.Themostcommoncompl
i
cat
ionofneonat
al
pneumopat
hyi
s:
Abronchiect
asis
Bpneumoni a
Cemphy sema
Dpneumoscl er
osis
Et
uberculosi
s
61.Deficiencyofthesur
fact
antl
eadst
o:
Aat el
ectasis
Bhemor r
hages
Camy loidosis
DPneumoscl erosi
s
Efibri
noidnecrosis
62.Airl
essdar kr edlungs,"rubber
"consi
stency
char
act eri
ze:
Adiseaseofhy ali
nemembr anes
Baspirationpneumoni a
Ccongeni talbronchiectasi
s
Dprimar yatelectasi
s
EHy poplasiaofthel ungs
MAJIDAABDUL
GROUP:O5

You might also like